You are on page 1of 446

Review of

POST GRADUATE MEDICAL ENTRANCE EXAMINATION


22,000 MCQ'S
FULLY REFERENCED AND TOPIC WISE ARRANGED

VOL. III

OBS&GYNAE
SURGERY
FORENSIC
ANAESTHESIA
RADIOLOGY
SKIN
PSYCHIATRY

By

ArvindArora
M.S. Orthopaedics
Assistant Professor
Seth GS. Medical College &
KEM Hospital, Mumbai

Amit Tripathi
M.B.B.S, B.R.D.M. C.
M.S. Opthalmics (Stu.), AG-ftistribo,tor:
...f~ f't£. ?'f.

' Ashish Gupta


1
.'\~)
-!U'l.'- . ~~,,~~·:',
-·· •. , ,

MBBS, MD. (Radiodiagnosis) KGM'lfip ;ic:)-li :!fl, Gronr"' 1'1'"''',


Ex SR, GB Pant Hospital, MAMC, New0/Je?iz'f~~;~J~~~'i:~~;~~i1~~~~i~
Assistant Prof SRMS IMS, Bareilly · ·

PULSE PUBLICATIONS
First Floor Ayodhya Das Trade Compound, Vijay Chowk, Gorakhpur-273001
Review of ,_

Post Graduate Medical Entrance Examination

9th Edition :Nov 2013

Price Rs. : 600/-

Published by : Dr. Rakesh Dubey & Rahul Singh


PULSE PUBLICATIONS
First floor Ayodhaya Das Trade Compound, Vijay Chowk, Gorakhpur- (0551) 2203494, 2341804
~ob. 09984628111, 09235392282, 09312784703
E-mail : pulserahul@yahoo.com, rakeshdubey 11 @yahoo.co.in

Review ofP.GM.E. Exam. by Amit Tripathi, Arvind Arora & Ashish Gupta
© 2013 Author & Publisher Both

No part of this publication should be reproduced, stored in a retrieval system, or transmitted in any form
or by any means: electronic, mechanical, photocopying, recording, or otherwise without the prior written
permission from the publisher.

This books has been published on good faith that the material provided by author(s) is original every
effort is made to ensure accuracy of material, but the publisher,printer and author will not be held responsibile
for any inadvertent error(s). In case of any dispute, all legal matters to be settled under Gorakhpur Jurisdiction
only.

Laser Typesetting at:


Pulse Computer & Printing Techonology

Printed at:
~agic International Pvt. Ltd.
TABLE OF CONTENTS
PAGE No.
QUESTION ANSWER
GYNAE & OBS
GYNAECOLOGY
1. GYNAECOLOGICAL ANATOMY 3 115
2. PHYSIOLOGY & HISTOLOGY 3 115
3. GYNAECOLOGICAL DIAGNOSIS . 5 116
4. MALFORMATION OF THE FEMALE 6 117
GENERATIVE ORGANS
5. SEX INTERSEXUALITY 6 117
6. VULVA 6 116
7. VAGINA 6 116
6. SEXUALLY TRANSMITTED DISEASE IN 9 119
THE FEMALE
9. TUBERCULOSIS OF THE GENITAL TRACT 10 120
10. URINARY SYSTEM 11 120
11. URINARY FISTULAS . ' 11 120
12. THE PATHOLOGY OF CONCEPTION 12 121
13. CONTRACEPTIVES 15 123
14. DISORDERS IN MENSTRUATION 24 126
15. CHORIOCARCINOMA 29 131
16. PROLAPSE 29 131
17. FIBROMYOMA OF THE UTERUS 31 132
16. CARCINOMA CERVIX 32 133
19. CARCINOMA ENDOMETRIUM (Uterus) 36 135
20. OVARY 37 136
21. CARCINOMAOVARY 39 137
22. RADIOTHERAPY & CHEMOTHERAPY 42 139
AND PHARMACOLOGY IN GYNAECOLOGY
23. PELVIC INFLAMMATORY DISEASE 43 139
24. ENDOMETRIOSIS 44 140
25. OPERATIVE GYNAECOLOGY 45 140
26. MISCELLANEOUS (Gynae) 46 141

OBSTETRICS
27. OBSTETRICAL ANATOMY 46 143
26. PLACENTA & FETAL MEMBRANE 49 143
29. PHYSIOLOGICAL CHANGES DURING 50 144
PREGNANCY
30. ENDOCRINOLOGY IN RELATION TO 51 144
REPRODUCTION
31. DIAGNOSIS OF PREGNANCY 52 145
32. FETAL SKULL AND MATERNAL PELVIS· 53 146
33. ANTENATAL CARE &ASSESSMENT OF FETAL 54 146
WELLBEING
34. NORMAL LABOUR 57 146
35. NORMAL PUERPERIUM 59 149
36. ABORTION 60 150
37. ECOTOPIC PREGNANCY 63. 151
38. TROPHOBLASTIC DISEASE 65 153
39. MULTIPLE PREGNANCY 66 153
40. HYDRAMNIOSAND OLIGOHYDRAMNIOS 68 154
41. HYPERTENSIVE DISORDERS IN PREGNANCY 68 155
42. ANTEPARTUM HAEMORRHAGE 71 156
43. MEDICAL & SURGICAL ILLNESS COMPLICATING 72 157
PREGNANCY
44. GYNAECOLOGICAL DISORDERS IN OB$. 80 161
45. PRETERM LABOUR & INTRAUTERINE FETAL DEATH 81 162
46. PREGNANCY IN Rh-NEGATIVE WOMEN 81 162
47. CONTRACTED PELVIS 82 162
48. TRIAL OF LABOUR 82 163
49. ABNORMAL UTERINE ACTION 83 163
50. MALPRESENTATION 83 163
51. OBSTRUCTED LABOUR 85 164
52. COMPLICATION OF 3rd STAGE OF LABOUR 86 165
53. INJURY TO THE BIRTH CANAL 88 165
54. ABNORMALITY OF PUERPERIUM 88 166
55. FETUS & NEW BORN AND THEIR DISEASES 89 166
56. INTRA UTERINE GROWTH RETARDATION 90 167
57. PHARMACOTHERAPEUTIC IN OBSTETRICS 91 167
58. INDUCTION OF LABOUR 92 168
59. OPERATIVE OBS 92 168
60. DIAGNOSIS IN OBSTETRICS 94 169
61. MISCELLANEOUS (Obs) 97 171

SURGERY
1. WOUNDS, TISSUE REPAIR & SCARS. 183 327
2. FLUID ELECTROLYTE 185 328
3. BLOOD TRANSFUSION 187 330
4. SHOCK 188 330
5. INFECTION 188 331
6. TRANSPLANTATION 191 332
7. PLASTIC & RECONSTRUCTIVE SURGERY 191 333
SKIN LESION
8. BURNS 195 335
9. ARTERIAL DISORDERS 197 336
10. AMPUTATION 201 339
11. VENOUS DISORDERS 201 339
12. LYMPHATIC SYSTEM 204 340
13. TRAUMA 205 341
14. ORTHOPEDICS 209 343.
15. NERVOUS SYSTEM 210 344
16. CLEFT LIP & PALATE 214 . 346
17. MAXILLOFACIAL INJURIES 214 347
18. ORAL CAVITY 215 347
19. SALIVARY GLAND 216 348
20. E.N.T 218 349
21. NECK 219 349
22. THYROID GLAND 221 351
23. PARATHYROID & ADRENAL GLANDS 226 354
24. BREAST 227 355
25. THORAX 235 359
26. LUNG 238 361
27. HEART & PERICARDIUM 240 362
28. THE OESOPHAGUS 240 362
29. STOMACH & DUODENUM 246 365
30. THE LIVER 253 370
31. THE SPLEEN 258 372
32. THE GALL BLADDER & BILE DUCTS 259 373
33. THE PANCREAS 266 377
34. THE PERITONIUM 271 380
35. SMALL & LARGE INTESTINE 274 381
36. INTESTINAL OBSTRUCTION 284 387
37. THE VERMIFORM APENDIX 287 389
38. THE RECTUM 288 390
39. THE ANAL CANAL 289 391
40. HERNIA 291 392
41. URINARY TRACT (GENERAL) 294 394
42. THE KIDNEY & URETURS 295 394
43. THE URINARY BLADDER 301 398
44. THE PROSTATE & SEMINAL VESICALES 302 399
45. URETHRA & PENIS 305 400
46. THE TESTIS & SCROTUM 308 402
47. NEOPLASIA 311 404
48. MISCELLANEOUS 315 406

FORENSIC MEDICINE
1. SEC - I & II : Law & Medicine, Personal 415 463
lndentification, Medicolegal Autopsy, Exhumation
2. SEC- Ill : Medicolegal aspects of Death, 422 468
Investigations, Hanging, Strangulation,
Starvation, Cold & Heat
3. SEC- IV : Mechanical Injuries, Firearms and 429 472
Firearm Injuries, Regional Injuries, Thermal Injuries
4. SEC - V : Impotence, Sterility, Sterilisation, 436 476
Virginity, Pregnancy, Delivery, Natural Sexual
Offenses, Unnatural Sexual Offenses,
5. SEC -VI : Forensic Psychiatry 440 478
6. SEC- VII : Forensic Examination of Biological Fluid, 442 479
Stains & Other Materials, Collection and
Preservation of Biological Materials
7. POISONS 443 480
8. MISCELLANEOUS 454 487

ANAESTHESIA
1. ANAESTHETIC EQUIPMENTS 495 525
2. PREOPERATIVE ASSESSMENT & 498 527
MONITORING IN ANAESTHESIA
3. FUNDAMENTAL CONCEPTS 500 527
4. MUSCLE RELAXANTS 500 528
5. GENERAL ANAESTHESIA 504 529
6. REGIONAL ANAESTHESIA 511 534
7. COMPLICATIONS OF ANAESTHESIA 516 536
8. ANAESTHESIA FOR SPECIAL SITUATIONS 517 537
9. CARDIOPULMONARY RESUSCITATION AND 520 538
MECHANICAL VENTILATION
10. MISCELLANEOUS 521 538

RADIOLOGY
1. FUNDAMENTALS IN RADIOLOGY 543 575
2. RESPIRATORY SYSTEM 545 576
3. CARDIOVASCULAR SYSTEM 550 579
4. GIT AND HEPATOBILIRAY SYSTEM 553 580
5. UROGENITAL SYSTEM 557 582
6.. SKELETAL SYSTEM 559 583
7. NERVOUS SYSTEM 562 585
8. OBSTETRICS AND GYNAECOLOGY 564 586
9. NUCLEAR MEDICINE 566 587
10. RADIOTHERAPY 567 588
11. MISCELLANEOUS 571 591

SKIN

1. ANATOMY OF SKIN 595 627


2. SKIN LESIONS AND DISORDERS OF PIGMENTATION 597 628
3. DERMATITIS 599 629
4. PAPULOSQUAMOUS DISORDERS 601 630
5. VESICULOBULLOUS (BLISTERING) DISORDERS 604 632
6. PHOTOSENSITIVE DISORDERS 608 634
7. BACTERIAL INFECTIONS 609 634
8. FUNGAL INFECTION, SCABIES, PEDICULOSIS 613 636
9. SEXUALLY TRANSMITTED DISEASES 616 638
10. AUTOIMMUNE SKIN DISORDERS 618 639
11. · SKIN TUMORS 621 640
12. MISCELLANEOUS 621 641

PSYCHIATRY

1. SYMPTOMS &SIGNS IN PSYCHIATRY AND 645 685


CLASSIFICATION
2. ORGANIC MENTAL DISORDERS 647 686
3. SUBSTANCE ABUSE 651 688
4. SCHIZOPHRENIAAND OTHER PSYCHOTIC 654 690
DISORDERS
5. MOOD DISORDERS 659 692
6. ANXIETY DISORDERS, STRESS DISORDERS 664 695
AND GRIEF
7. SOMATOFORM DISORDERS 668 696
8. PERSONALITY DISORDERS 670 698
9. SLEEP DISORDERS AND EATING DISORDERS 672 698
10. CHILD PSYCHIATRY 674 699
11. TREATMENT IN PSYCHIATRY 676 700
12. COGNITIVE DEVELOPMENT AND 678 701
DEFENSE MECHANISM
13. PHARMACOTHERAPY IN PSYCHIATRY 679 702

***
GY GY

OBS ICS
·.:.;
GYNAECOLOGY & OBSTETRICS

GYNAECOLOGICAL ANATOMY 15. Which of the following conditions is commonly


encountered in the female newborn and usually
1. All are related to the lateral vaginal fornix requires no therapy ? (Karnataka 03)
except- (JIPMER 90) a) Enlarged clitoris b) Labial fusion
a) Ureters b) Mackenrodt's c) Mucoid vaginal discharge d) Prolapsed urethra
c) Inferior vesical artery d) Uterine artery 16. Size of graffin follicle is- (Mahara 02)
2. Para -ovarian cyst are derived from- (JIPMER 92) a) 2mn b)3mm
a) Wolfum duct b) Mullerian duct c) 4nm d)6mm
c) Gartners duct d) Pronephros 17. The shape of aNulliparous cervix is- (AI 07, 98)
3. The epithelial lining of cervical canal is -(TN 90) a) Circular b) Transverse
a) Low columnar b) High columnar c) Lonitudinal d) Fimbriated
c) Stratified squamous d) Ciliated columnar 18. Bartholin's glands corresponds to- (UP 07)
4. Bartholinsductopeninto- (JIPMER 92) a) Bulbourethral glands b) Urethra glands
a) Labiamojora c) Paraurethral glands d) Cowper's glands
b) Labia minora 19. The following artery is not a branch ofthe anterior
c) Lowervagina division of internal iliac artery-(DELHIPG Mar. 09)
d) Groved between labia minora and hymen a) Uterine b) Internal pudendal
5. Falseaboutvaginais- (TN99) c) Iliolumbar d) Vaginal
a) Partly or wholly derived from urogenital sinus 20. Before puberty, what is the ratio between the cervical
b) Rich in glands length and uterine body? (UP SC II 10)
c) Lined by stratified squamous epithelium a) 1:2 b)2: 1
d) Posterior fornix lengthier than anterior fornix c) 1 : 3 d) 1 : 41
6. Epithelial lining ofBartholins duct is-(JIPMER 93) 21. During the development of female genital organs,
a) Columnar b) Transitional Bartholin'sglanddevelopsfrom- (Karn 11)
c) Cuboidal d) Squamous a) Genital fold b) Genital tubercie
7. Venous drainage of upper part of uterus and c) Urogenital fold d) Urogenital sinus
placentaisthrough- (PG/95)
22. Main support of the uterus is formed by-
a) Ovarian vein b) Uterine vein
a) Mackendrot's ligament (Jipmer ll)
c) Internal iliac vein d) None
b) Broad ligament
8. Clitorisdevelopsfrom- (PG/93)
a) Urogenital Sinus b) Labio Scrotal Swelling c) Fallopian tubes
c) Genital folds d) Genital tubercle d) Round ligament
9. Scrotoum is analogus to- (Kerala 95) 23. Cervix to body ratio in a prepubertal girl-
a) Labiaminora b)Labiamajora a) 1:1 b)2: 1 (Jipmer 11)
c) Uterus d) Vagina c) 1:2 d)3: 1
10. Which of the following is Schiller positive-(AIJMS 79, 24. Cortex of ovary contains all except- (Jipmer 11)
a) Erosion b) Ectropion DNB 91) a) Hilus cells b) Corpus luteum
c) Carcinomacervix d) All of the above c) Primordial germ cells d) Graafian follicle
11. Lymphatics from clitoris drain into- (PGI 2000)
a) Superficial inguinal LNs b) Deep inguinal LNs PHYSIOLOGY AND HISTOLOGY
c) Lymphnode of cloquet d) Obturator LN
12. Ovarydevelopfrom- (PGI02,JIPMER91) 25. The ovulatory period corresponds to- (TN 90)
a) Mullerian duct b) Genital ridge a) 14 days before menstruation
c) Genital tubercle d) Mesonephric duct b) The day of menstruation
e) Sinovaginal bulbs c) Increase in basal temperature
13. Transverse vaginal septum corresponds to -(PGI04) d) 14 days after menstruation
a) External os b) Vesical neck 26. Gonadorophinisa- (JIPMER 92)
c) Bladder base d) Hymen a) Steroid b) Lipoprotien
e) Above the external urethral meatus c) Polypeptide d) Glycoprotien
14. True about isthmus are all EXCEPT- (TN 03) 27. Maximum function of Corpus luteum occurs-
a) Measures 20 mm in length a) At ovulation (JIPMER 93)
b) Forms lower uterine segment b) Before ovulation
c) Mucosa similar to endometrium c) 3 days after ovulation
d) Present between cervical mucosa and endometrial d) 9 days after ovulation
mucosa

l)c 2)a 3)b 4)d 5)b 6)a 7)a 8)d 9)b 10) None 11) b,c 12) b 13)a 14)a
15)c 16)a 17)a 18)c 19)c 20)b 2l)d 22)a 23)b 24)a 25)a 26)d 27)d
GYNAE&OBS[4]

28. The probable source of relaxin is-(Jipmer 9I,DNB 89) 42. Primary Oocyte remains in ovary in which stage-
a) Ovary b) Adrenal cortex a) Metaphase b) Anaphase (SGPGI 05)
c) Liver d) Bartolins gland c) Interphase d) Prophase
d) Anterior pituitary gland 43. In a young female of reproductive age with regular
29. Gonadotropins are- (JIPMER 78, Delhi 90) menstrual cycles of28 days ovulation occurs around
a) Proteins b) Glycoproteins 14th day of periods. When is the f'Irst polar body
c) Phospholipids d) Polysaccharides extruded- (AJIMS May 05)
30. Ovulation occurs- (Kerala 94) a) 24 hrs prior to ovulation
a) 14 days before next mensturation b) Accompanied by ovulation
b) 14 days after mensturation
c) 48 hrs after the ovulation
c) Within 16 hrs. ofLH surge
d) At the time offertilization
d) 16 days before next mensturation
44. After how many days of ovulation embryo
31. In tbe absence of fertilization and implantation
implantation occurs ? (AIIMS 06)
the corpus luteum persists for- (AP 9 5)
a) 2-3 days b) 6-8 days a) 3-5 days b)7-9 days
c) 12-14days d)28-30days c) 10-12 days d) 13-15 days
32. False statement regarding hormone status in 45. The sensitivity ofthe uterine musculature is -
menopausalwomenis- (MARE 98) a) Enhanced by progesterone (AIIMS 06)
a) Decresed FSH b) Enhanced by estrogen
b) Increased LH c) Inhibited by estrogen
c) Decreased estrogen d) Enhanced by estrogen and inhibited by progesterone
d) Decreased progesterone 46. In those mammals which are seasonal breeder,
33. FSH secretion is stimulaed by- (MAHE 98) the females are receptive only once a year; the
a) LHRH b)TRH cycle is termed as- (AIIMS 06)
c) Testosterone d) Estradiol a) Follicular b) Estrous
34. LH surge precedes ovulation by- (MAHE 98) c) Menstrual d) Luteal
a) 12hrs. b)24hrs. 47. Corkscrew shaped endomentrial glands is seen in-
c) 36 hrs. d) 48 hrs. a) Early proliferative phase (APPG 06)
35. Corpus luteum functions maximally without an b) Late proliferative phase
implantation for................days- (PGI 2000) c) Early secretory phase
a) 9 b) 12 d) Late secretory phase
c) 6 d) 15 48. Following are the features ofinhibin except-
36. Hot flushes in postmenopausal women are best a) Nonsteroidal water soluble protein
treated with which ofthe following- (Al2002)
b) Secreted by Graf:fian follicle (Karn pgmee 06)
a) Estrogens b) Danazol
c) Stimulates FSH secretion
c) OCpills d)Tamoxifen
d) Increased decretion ofinhibin occurs in polycystic
37. Average age range of attaining menopause is-
overian disease
a) 40-50 b)45-55 (TN03)
c) 55-60 d) 58-68 49. Source of progesterone during normal menstrual
38. Cornification Index is maximum in- (TN 03) cycle- (MAHE 07)
a) Early secretory phase a) Corpus luteum
b) Late proliferative phase b) Stroma
c) Late secretory phase c) Surface epithelium of ovary
d) Early proliferative phase d) None of above
39. Endometrial biopsy is done in which phase of 50. The best predictor of ovulation is - (Comed 08)
menstrual cycle- a) Estrogen peak
a) 5-7 days b) 12-14days b) Follicle stimulating hormone (FSH) surge
c) 18-20 days d) 22-24 days c) Onset of the LH surge
40. Sertoli cells have receptors for- (AI 04) d) Preovulatory rise in progesterone
a) lnhibin 51. Which of the following is the earliest change in the
b) Luteinising hormone endometrium after ovulation ? (UPSC-II 08)
c) Follicle Stimulating Hormone a) Secretion in the lumina of the endometrial glands
d) Melatonin b) Subnuclear vacuolation
41. Inhibin inhibits the secretion of which hormone- c) Cockscrew shaped glands
~FSH ~LH (TN04) d) Stromal cells become oedematous, enlarged and
c) Estrogen d) Prolactin polyhedral in shape

28)a 29)b 30)a 3l)c 32)a 33)a 34)b 35)a 36)a 37)b 38)b 39)d 40)c 4l)a
42)d 43)b 44)b 45)d 46)b 47)d 48)c 49)a 50)c 5l)b
GYNAE & OBS [ 5]

52. Ovary secretes- (UP 08) 62. TrueabouthCG- (PG/Nov.10)


a) Oestrogen & Progesterone a) a. subunit identical to LH, FSH & TSH
b) Oestogen progesteroe, Testosterone and androgens b) Causes involution of corpus luteum
c) FSH&LH c) Doubles in 7-10 days
d) ACTH, Inhibin, TSH d) Max. level seen at 60-70 days of gestation
53. Ovlation occurs- (UP 08) e) Detected in serum and urine 8-9 days after
a) After 14 days of menstruation ovulation
b) Before 14 days of menstruation 63. Which of the foUowing is seen in the ovulatory phase-
c) After 16 days of menstruation a) Stimulation of continuation of reduction division
d) Before 16 days ofmenstration of oocytes (AIIMSMay 11)
54. Endometrial biopsy is done at- (UP 08) b) Inhibin A is increased
a) Premenstrualy phase b) Mid-cycle phase c) FSH increases steroid synthesis in granulosa cells
c) LH Surge d) After menstruation d) Activin causes FSH to act on granulosa cells
55. Naturally occurring estrogens are- (PG1 Dec 08) 64. The peak level of serum LH (nglml) required for
a) Estrone b) Estriol ovulation- (Karn 11)
c) Ethinylestradiol d) Estradiol a) 25 b)50
e) Diethylstilbestrol c) 75 d) 100
56. Characteristics of the fertile phase of the menstrual 65. During a normal pregnancy, the changes occurring
cycle include aU of the foUowing except- in the urinary tract include the foUowing except-
a) Peak levels of oestrogen (COMED 09) a) Increase in the Glomerular filtration rate
b) Clear, abundant cervical mucus b) Dilatation of the ureters (UPSC 1111)
c) Ovulation c) Elevation and thickening of the trigone
d) Production of progesterone d) Hypertonicity of the Ureteric smooth muscle
57. The "fern pattern" of cervical mucus seen in the 66. Fertilization occurs in- (Jipmer 11)
first half of menstrual cycle is because of: a) Uterine cavity b) Infundibulum
a) Highmucoproteinlevel (UPSC-11 09) c) Isthmus d)Ampulla
b) Low mucoprotein level
c) High sodium chloride level GYNAECOLOGICAL DIAGNOSIS
d) Low sodium chloride level
58. After how many hours ofLH surge does the ovulation 67. Pap smear is taken from the- (TN 90)
occur ? (UPSC-11 09) a) Lateral formix b) anterior formix
a) 12-24hours b)24-36hours c) Posteriorformix d)Anyformix
c) 12-36 hours d) 24-48 hours 68. Ultrasound is diagnostic in - (TN 91)
59. TrueabouthCG- (PG/June09) a) Endometrial carcinoma b) Cancer cervix
a) Produced by cytotrophoblast c) Leiomyosarcoma d) Fibroid uterus
b) Prevent involution of corpus luteum 69. All of following are indications of doing a PAP
c) Value reach maximum level at 20 wk of pregnancy smearexcept- (AI96)
d) Its secretion starts after 2 day of ovulation a) > 40 yrs b) Pregnantfemale
60. True statement regarding oogenesis is/are- c) Sexually active female d) Menorrhagia
a) Primary oocyte arrests in prophase oflst meiotic 70. What percentage of post menopausal ovaries are
division (PGJ May 10) identified on transvaginal ultrasonography-
b) Primary oocyte arrests in prophase of II nd a) 60% b)70% (PG/96)
meiotic division c) 80% d) 100%
c) Secondary oocyte arrests in Metaphase of I '1 71. Hysteroscopy can diagnose aU, except- (PG1 98)
meiotic division a) Asherman's syndrome b) Septate uterus
d) Secondary oocyte arrests in Metaphase of lind c) Adenomyosis d) TB endometritis
meiotic division 72. The Colposcopic features suggestive of
e) 1st polar body is extruded during 1st meiotic malignancy are except- (PG1 99)
division of primary oocytes a) Condyloma b) Vascular atypia
61. True statement regarding Implantation- c) Punctation d) White epithelium
a) Takes place in Blastocyst stage 73. Evaluation ofa pt. with post menopausal bleeding
b) Occur after 7 day of fertilization doneby- (PGIJune05)
c) Occur on 5th day after ovulation a) Pap smear b)USG
d) Takes place in morula stage c) Endometrial biopsy d) Dilatation & curettage

52)b 53)b 54)a 55)a,b,d 56)d 57)d 58)b 59)b 60)a,d,e 61)a,c 62)a,d,e 63)a 64)c 65)d
66)d 67)None 68)d 69)d 70)d 71)None 72)a 73)a,c,d
GYNAE & OBS [ 6]

74. Hysteroscopy means visualization of- (PGI 05) 86. In complete mullerian duct aplasia all of tbe
a) Genital tract b) Fallopian tube following are likely to be absent Except- (AI 08)
c) Uterine cavity d) Cervix a) Ovaries b) Fallopian tubes
e) Abdominal cavity c) Uterus d) Vagina
75. Tbe investigation of choice in a 55 year old 87. Wbicb of tbe following condition present with
postmenopausal woman wbo bas presented with absence of both Mullerian· and Wolffian duct
postmenopausal bleeding is- (AI 06) structures- (AI 10)
a) Pap smear b) Fractional curettage a) Antimullerian Hormone deficiency
c) Transvaginal ultrasound d) Ca-125 estimation
b) Androgen Insensitivity syndrome
76. Wbicb oftbe following are distension media used
c) FSH receptor defect
in Hysteroscopy- (PGI Dec 08)
a) Carbon dioxide b) Glycine d) Ovotesticular syndrome
c) Distill water d) Air
e) Dextrose
SEX & INTERSEXUALITY
77. Following are tbe indications for 'byster-
osalpinograpby' expect- (DELHI PG Mar. 09)
88. Turner's syndrome is associated with- (DNB 90)
a) Infertility a) 45 Chromosomes b) Presence ofBarr bodies
b) Pelvic infections c) Low FSH levels d) All of the above
c) Recurrent abortions 89. The chromosomal complement in persons with
d) Abnormal uterine bleeding Klinefelter's syndrome is- (Karn 94)
a) 45,XO b)47XXX
MALFORMATION OF THE FEMALE c) 46XY d)47XXY
90. Barr body is seen in- (Kerala 94)
GENERATIVE ORGANS a) Turners syndrome b) Klinefelters syndrome
c) Testicular feminisation d) 46 XY
78. To diagnose Uterus didelpbys, procedure of choice 91. All are seen in testicular feminization syndrome
is- (AIIMS 92) except- (PGI 99)
a) Laproscopy b)IVP a) 46 XY b) Primary amnenorrhoea
c) Hysterosalpingography d) USG c) Short stature d) Vagina may present
79. Ideal age for repair ofVaginal agenesis is- 92. During sexual differentiation in males- (Karnat 05)
a) 6 months b) 3 years (AIIMS 92) a) Leydig cells produce mullerian inhibiting substance
c) At puberty d) Before marriage b) Primitive gonads differentiate into testis due to
80. Bicornuate uterus is due to- (PGI 98) the presence of SRY gene
a) Incomplete fusion of uterine cavity c) Androgen binding protein is responsible for the
b) Incomplete fusion of paramesonephric duct development of male external genitalia
c) Incomplete fusion of mesonephric duct d) Wolifian duct regresses
d) Incomplete formation of vagina 93. A 15 year old female presents with primary
81. Cocbleate uterus is- (ORRISA 98) amenorrhea. Her breasts are Tanner 4 butsbe bas no
a) Acute antiflexion axillary or pubic hair. Tbe most likely diagnosis is-
b) Acute retroflexion a) Turner's syndrome (AI 06)
c) Acute retroversion of uterus b) Mullerian agenesis
d) None of the above c) Testicular feminization syndrome
82. Raloxifene can cause- (APPG 03) d) Premature ovarian failure
a) Endometrial carcinoma b) Ovarian carcinoma 94. Tbe treatment for a case of virilizing adrenal
c) Breast carcinoma d) Cervical carcinoma hyperplasia is - (AI 06)
83. Complete failure of mullerian duct lusion will a) Estrogens b) Antiandrogens
resultin- (UPPGMEE04) c) ACTH d) Cortisone
a) Uterus didephys b) Arcuate uterus 95. Features ofTurner's dyudrome is/are-(PGIJune 06}
c) Subseptate uterus d) Unicomuate uterus a) Secondary amerorrhea b) Genotype XO
84. Tbe most important indication for surgical repair c) Infertility d) Streak ovary
of a Bicornuate Uterus is- (AIIMS NOV 05)
e) Coarctation of aorta
a) Infertility b) Dysmenorrhoea
96. Gonadal sex oftbe fetus is determined by-
c) Menorrhagia d) Habitual abortion
a) Secretion of testosterone (COMED 06)
85. Dietbylstiboestrol causes tbe following defects
b) Secretion of antimullerian hormones
except- (APPG06)
c) Sex determining region on the 'Y' chromosome
a) Renal anomalies b) Perifimbiral cyst
d) Secretion of estrogen
c) T-shaped uterus d) Vaginal adenosis

74)c 75)b 76) a,b,e 77) b 78) c 79)d 80)c 8l)a 82)c 83)a 84)d 85)a,b 86)a 87)b
88)a 89)d 90)b 91)c 92)b 93)c 94)d 95) b,c,d,e 96) c
GYNAE & OBS [ 7]

97. Which of the following is used for hirsutism in 106. Match list I (Syndrome) with list II (Conditions) and
female expect- (APPG 06) select the correct answer using the codes given below
a) Spironolaotone b) Oxynandrolone the lists - (UPSC-II 09)
c) Finasteride d) Flutamide List I (Syndrome) List II (Conditions)
98. In an adult phenotypic female, absence of uterus A. Androgen 1. .Mullerian agenesis
suggests the diagnosis of which one ofthe following? insensitivity with 46 XX
a) Mayer Rokitansky Kuster Hauser syndrome syndrome
B. Mayer Rockitansky- 2. Female phenotype, 46
b) Congenital adrenal hyperplasia (UPSC-11 08)
Kuster-Hause XY with absent
c) True hermaphrodite
syndrome Mullerian system
d) Sa-reductase deficiency C. Swyer's syndrome 3. Hypo gonadotrophic
99. The first sign of puberty in girls is- (AI 08) hypogonadism with
a) Breast budding anosmia
b) Growth spurt (Peak height velocity) D. Kaliman syndrome 4. Female phenotype with
c) Menarche 46 XY gonadal
d) Pubic and axillary hair growth , dysgenesis with patient
100. All of the following statements about Androgen Mullerian system
insensitivity syndrome are true Except- (AI 08) Codes:
a) Patients have an XY genotype A B C D
b) Pubic hair are abundant ~ 3 1 4 2
c) Short vagina may be present b) 2 4 1 3
d) Ovaries are absent c) 3 4 1 2
101. Androgen insensitivity syndrome true is- (Aiims d) 2 1 4 3
a) Phenotype may be completely female May 08) 107. All ofthe following are true about MRKH (Mayer-
Rokitansky - Kuster - Hauser) syndrome
b) Predominantly ovarian component in gonads
except- (AIIMS Nov 09)
c) Always in female
a) Absent uterus b) Absent cervix
d) Testes formed abnormally and receptors are normal c) Absent ovary d) Absent vagina
102. 10 year old girl with primary amenorrhoea, absent 108. A 20 years female present primary amenorrhea,
breasts, malformed uterus. The most likely absent breast, hypoplastic uterus. Most prabable
diagnosis- (Aiims May 08) diagnosis is? (AIIMS Nov 09)
a) MRKH syndrome a) Turner's syndrome
b) Turner's syndrome b) Mixed gonadal dysgenesis
c) Swyersyndrome c) Meyer Rokitansky kuster Hauser syndrome
d) Mixed gonadal dysgenesis d) Androgen insensitivity syndrome
103. All of the following are true about imperforate hymen 109. Causes of primary amenorrhea are all except-
except- (DPGEE 08) a) Rokitansky syndrome (AIIMS May I 0)
a) It may cause hematocolpos b) Sheehan syndrome
b) It is treated with a cruciate incision c) Kollman syndrome
c) It is diagnosed in the neonate d) Turner syndrome
d) It may cause urinary retention 110. Primary Amenorrhea with absent uterus, normal
breasts and scant pubic hair is seen in- (AI I 0)
104. Mayer-Rokitansky-Kuster-Hauser syndrome
a) Mayer Rokitanski Kuster Hauser syndrome
consists of-
b) Turner's syndrome
a) Ovaries, uterus fallopian tubes present
c) Androgen insensitivity syndrome
b) Uterus absent, fallopian tube ovaries present d) Noonan syndrome
c) All absent 111. Primary Amenorrhea with normal ovaries normal
d) Uterus present tubes and ovaries absent external genitalia and normal breasts is seen in -
105. Clinical feature (s) of Turner syndrome is/are all a) Mayer-Rokitanski-Kuster-Hauser syndrome
except- (PGI June 09) b) Turner's syndrome (AI IO)
a) Short stature c) Androgen Insensitivity syndrome
b) Cryptorchidism d) Noonan syndrome
c) Gonadal meladevelopment 112. A nineteen-year old female has short stature,
d) Biscuspid aortic valve widespread nipples and primary amenorrhoeA. She
e) Lymphoedema most likely has karyotype of- (DPG I 0)
a) 47,XX+l8 b)46.XXXY
c) 47XXY d)45XO

97)b 98)a 99)a lOO)b lOl)a 102)b 103)c 104)b 105)b 106)d 107)c lOS) a 109)b llO)c
lll)a 112)d
GYNAE&OBS[8)

113. Commonest genetic defect causing gonadal failure 123. Mycotic vulvovaginitis is due to- (Karn 90)
· withprimaryamenorrhea- (Maharashtra 10) a) Candida b) Aspergillus
a) Kallman syndrome c) Cryptococcus d) Pseudomonas
b) Abnormal X chromosome 124. Vulval candidiasis is associated with-(Delhi 96,PGI 89)
c) 21-hydroxylase deficiency a) Addison's disease b) Cushing's isease
d) Turner syndrome c) Diabetesmellitus d)Acromegaly
114. Which one of the following is not a feature of 125. Commonest genital infection in females is-(Kerala 96)
Thrner's syndrome? (UPSC-I 10) a) Candidiasis b) Trichomoniasis
a) Webbed neck b) Horse-shoe kidney c) Syphilis d) Gonorrhea
c) Coarctation of aorta d) Mental retardation e) Chanchroid
115. True about MRKH (Mayer-Rokitansky-Kuster- 126. Involvement of pelvis in a case of vaginal carcinoma
Hauser)syndrome- (PG!May 10) of stage _ _ - (AI 97)
a) Absent uterus b) Absent ovary a) I b)II
c) Absent vagina d) XX phenotype c) III d)IV
e) XY phenotype 127. The maturation index (MI) on vaginal cytology
116. True about Klinefelter syndrome- (PGI May 10) is a diagnostic meth()d for evaluating the-
a) XXY b)XO a) Adequacy of cytotoxic drug therapy (UPSC 2K)
c) Male hypogonadism d) Female hypogonadism b) Gender of an anatomically abnormal child
e) iFSH c) Malignant change at squamocolumar junction of
117. A 13-year-old, obese, unmarried girl presents with ceiVix
the history of amenorrhea and cyclical abdominal d) Endocrine status of the patient
pain. On examination, the secondary sex characters 128. Protective bacterium in normal vagina is-(J&K01)
are found to be well developed. What should be the a) Peptostreptococcus b) Lactobacillus
next step- (UPSC II 11) c) Gardenella vaginalis d) E. coli
a) Carry out the progesterone challenge test 129. True about Bacterial vaginosis- (PGI June 05)
b) Assess the TSH and Prolactin levels a) Itching
c) Carry out a per-rectal examination b) Grey discharge
d) Keep her under observation for the next three c) Clue cells found
months d) Fishy odour discharge
e) Caused by Gardnerella vaginalis
VULVA 130. True about trichomonus vaginalis- (PGI June 05)
a) Flagellated parasite b) Fungal infection
118. Vulval carcinoma metastazes to which lymph c) Curdy white discharge d) Pruritus
group- (JIPMER93) e) Sexually transmitted
a) Para aortic nodes 131. Gardenerella vaginalis infection is diagnosed by
b) Superficial inguinal nodes all except- (APPGE 05)
c) Internal iliac nodes a) PH <4.5 b) Fishy odour
d) External iliac nodes c) Clue cells d) Positive whiff test
119. True about carcinoma valva- (PGI 04) 132. A 40 years old woman presented to the gynecologist
a) Spreads to superficial ingunial nodes with complaints of profuse vaginal discharge. There
b) Spreads to illiac nodes was no discharge from the cervix on the speculum
c) Seen after menopause examination. The diagnosis of bacterial vaginosis
d) Viral predisposition was made based upon all of the following findings on
e) Radiotherapy given microscopy except- (AI 06)
a) Abundance of gram variable coccobacilli
VAGINA b) Absence of Lactobacilli
c) Abundance of polymorphs
120. Nabothian follicle occurs in - (TN 91) d) Present of clue cells
a) Erosion of cervix b) Carcinoma body of uterus 133. Consider the following characteristics of vaginal
c) Carcinoma cervix d) Carcinoma vagina discharge- (UPSC 07)
121. pHofthevaginaislowestduring- (UPSC96) 1. Green and thick 2. pH< 4.5
a) Ovulation b) Menstruation 3. Clue cells 4. Whiff test positive
c) Pregnancy d) Puerperium Which of the above characteristics are present in
122. Most common form of vaginitis- (Kerala 96) bacterial vaginosis -
a) Senile vaginitis b) Trichomonas vaginitis a) 1 and 3 only b) 2 and 3 only
c) Gonococal vaginitis d) Monilia vaginitis c) 1 and 4 only d) 3 and 4 only

113)d 114)d 115)a,c,d 116)a,c,e 117)c 118)b 119)All 120)a 12l)c 122)b 123)a 124)c 125)b 126)c
127)d 128)b 129) b,c,d,e 130)a,d,e 131) a 132) c 133)d
GYNAE & OBS [ 9]

134. With reference to vagina, which one of the 146. True about bacterial Vaginosis- (PGI2000)
following statements is not correct- (UPSC 07) a) Typical whitish grey discharge
a) It has mucus secreting glands b) Metronidazole is DOC
b) It is supplied by uterine artery c) "Cluecells"seen
c) It is lined by stratified squamous epithelium d) Vaginal pH usually between 4 to 6
d) Its posterior wall is covered by peritonium e) Highly motile organism seen on wet mount
135. Senile vaginitis is due to- (Corned 07) 147. True about vaginal candidiasis - (PGI 01)
a) Gonococcal infection b) Cancer cervix a) Risk is not increased in pregnancy
c) Diabetes d) Oestrogen deficiency b) Associated with intense pruritus
136. A woman presents with a fluctuant non tender c) Typical "Cottage-cheese" discharge occurs
swelling at the introitus. The best treatment is-(AI 08) d) Most commonly by candida glaberata
a) Marsupialisation b) Incision and drainage 148. Bacterial vaginosis is characterised by- (PGI 02)
c) Surgical resection d) Aspiration a) Alkaline pH b) Strawberry clot in vagina
137. True statement regarding sarcoma botryoides- c) Creamy discharge d) Clue cells in wet mount
a) Involvement of vagina (PGI May 10) 149. Cervicitisiscausedby- (PGI03)
b) Grape like growth seen a) Pseudomonus b) Staphylococcus
c) Common in old age c) Chlamydia d) Trichomonas
d) Malignant e) N. Gonorrhoea
138. All are risk factor for vaginal candidiasis except- 150. Acute PID is treated by- (PGJ 03)
a) Diabetes mellitus b) HIV (AJJMSNov 10) a) IV antibiotics (broad spectrum)
c) Hypertension d) Pregnancy b) Drainage ofTOM
139. Vaginal adenosis preceed- (Jipmer 11) c) Abdominal hysterectomy
a) Clear cell carcinoma d) Laparoscopic exploration
b) Endometrial carcinoma 151. Bacterial vaginosis is characterised by- (PGJ 03)
a) PH<4.5
c) Cervical carcinoma
b) Cluecell
d) Squamous cell carcinoma of vagina
c) Amide test is positive
d) Lactobacillus is found abundantly
SEXUALLY TRANSMITTED
e) Sexuallytransmitted
DISEASES IN THE FEMALE 152. Non gonococcal urethritis is caused by- (PGI99)
a) Chlamydia b)LGV
140. Herpes genetalis causes- (Kerala 94) c) Syphilis d) Gardnella vaginalis
a) Koilocytosis in smear b) Atypical cells 153. Treatment of choice in Gardenella vaginitis in
c) Carcinoma cervix d) Sterility pregnancy- (APPG03)
141. In chlamydial cervicitis drug of choice is- (Kerala a) Clindamycin b) Metronidazole
a) Tetracycline b) Septran 96) c) Erthromycin d) Tetracycline
c) Chlomphenicol d) Erythromycin 154. GardenereUa infection, not seen is- (APPG 03)
e) Metronidazole a) pH< 4.5 b) Amine test positive
142. Asymptomatic gonorrhoea in females is dne to c) Discharge per vaginum d) Pruritus
infections of- (Al97) 155. A 30 year old woman with a bad obstetric history
a) Vagina b) Urethra presents with fever. The blood culture from the
c) Endocervix d) Fundus patient grows gram - positive small to medium
143. One ofthe following in adult female genital tract coccobacilli that are pleomorphic, occurring in
is not affected by gonococcal infections - short chaius. Direct wet mount- (AI 04)
a) Vulva b) Vagina (Orissa 99) a) Listeria monocytogenes
c) Cervix d) Fallopian tube b) Corynebacterium sp
144. An infertile woman underwent c) Enterococcus sp
hysterosalphingogram. It showed bilateral beaded d) Erysipelothrix rhusiopathiae
tubes with enlargement at ampulla, The organism 156. The most sensitive method for detecting cervical
responsible is- (Kerala OJ) Chlamydia trachomatis infection is- (All India 04)
a) Chlamydia b) Gonococci a) Direct fluorescent antibody test
c) Tuberculosis d) None b) Enzyme immunoassay
145. Gonococci has atrmity for- (TN 02) c) Polymerase chain reaction
a) Columnar epithelium d) Culture on irradiated McConkey cells
b) Glandular epithelium 157. Whiff test is used to detect- (APPGE 04)
c) Stratified sqamous epithelium a) Trichomoniasis b) Candidial cervicitis
d) Squamous epithelium c) Bacterial vaginosis d) Gonococcal cervictis

134)a 135)d 136)a 137)a,b,d 138)c 139)d 140)a,c 141)a 142)c 143)b 144)c 145)a 146) a,b,c,d
147)b,c 148)d 149)b,c,d,e 150)a,b,d 15l)b,c,e 152)a 153)b 154)a 155)a I 56) a 157)c
GYNAE & OBS [ 10]

158. Which one ofthe following is not a characteristic 171. Clue cell is seen in- (AIIMS Nov 10)
feature of bacterial vaginosis? (UPSC-II 08) a) Bacterial vaginosis b) Candidial vaginosis
a) Presence of clue cells b) Thick white discharge c) Chlamydiasis d) Trichomonas
c) VaginalpH>4.5 d) Fishy odour 172. 'Clue cells' are seen in- (AI 11)
159. Salpingitis I Endosalpingitis is best confirmed by- a) Bacterial Vaginosis b) Vaginal Candidiasis
a) Hysteroscopy+ Laparoscopy (AI 08) c) Chlamydia! vaginosis d) Trichomoniasis
b) X-ray 173. Which of the following statements about Chlamydia
c) Hysterosalpingography trachomatis is true - (AI 11)
d) Sonosalpingography a) May be isolated an arrtificial media by culture of
160. Which is most commonly implicated in genital purulent endocervical discharge
(vulval) warts? (AiimsMay08) b) Most genital chlamydial infections are
a) HPV16 b)HPV 18 symptomatic
c) HPV31 d)HPV6 c) Use of OCP is associated with increased ris of
161. Clue cells are seen in- (Aiims May 08) asymptomatic chlamydia! infection
a) Bacterial vaginosis b) Candidiasis d) Penicillin is the drug of choice for treatment
c) Trichomoniasis d) Gonorrhoea 174. The diagnostic criteria for bacterial vaginosis
162. Trichomonas-true is- (PGI June 08) include the following except- (UPSC II 11)
a) Foul smelling vaginal discharge a) Vaginal pH <4.5
b) Vaginal pH is 4 b) Homogenous vaginal discharge
c) Strawberycervix c) Presence of clue cells
d) Infertility d) Positive whiff test
e) Abortion 175. A woman, who is in the reproductively active age
163. Which of the following can be diagnosed on vaginal group, presents with a history of greenish and frothy
wet mount smear- (PGI Dec 08) vaginal discharge. On per speculum examination,
a) Trichomonas vaginalis b) Candida albicans she has multiple punctuate strawberry-like spots.
c) Herpes simplex d) N.gonorrhea What is the likely diagnosis- (UPSC II 11)
e) Atrophic vaginitis a) Candidiasis b) Trichomoniasis
164. Clue cells are found in ? (AIIMS Nov 08) c) Gonococcal vaginitis d) Chlamydia infection
a) Candidial vaginosis b) Bacterial vaginosis 176. Clue cells are seen in- (Jipmer 11)
c) Trichomoniasis d) Chlamydia! infection a) Bacterial vaginosis b) Trichomoniasis
165. Creamy fishy odor is characteristically seen in - c) Candidiasis d) Herpes
a) Trichomonas b)Gardnella (AI09)
c) Candida d) Chlamydia TUBERCULOSIS OF THE GENITAL TRACT
166. A lady presented with creamy white vaginal
discharge with fishy odour, drug of choice is - 177. The commonest complication of pregnancy
a) Doxycycline b)Ofloxacin (AIIMSMay09) after complete treatment of genital tuberculosis
c) Metronidazole d) Clindamycin is- (AI91)
167. Genitalwartsinpregnancytreatmentis-(AllMSNov09) a) Abortion b) Ectopic pregnancy
a) Podophylin b) Salicylic acid & lactic acid c) Malpresentation d) Intra uterine death
c) Cryotherapy d) Imiquinod 178. TB endometritis is caused by- (AIIMS 98)
168. HPVvaccineis? (AIIMS Nov 09) a) Hematogenous spread b) Lymphatic spread
a) Monovalent b) Bivalent c) Retrograde spread d) Direct spread
179. The most common cause of tubal block in India
c) Quadrivalent d) Both bivalent and Quadrivalent
is- (AI06)
169. Clue cell seen in ? (AIIMS Nov 09)
a) Gonorrhea infection b) Chlamydia infection
a) Bacterial vaginosis
c) Tuberculosis d) Bacterial vaginosis
b) Candida 180. Which is the most common site ofinvolvement in
c) Trichomonas vaginalis genital tuberculosis -(UPSC 06, JIPMER 86, TN 95)
170. With regard to trichomonal vaginitis the following a) Uterine endometrium b) Fallopian tubes
statements are true EXCEPT- (Comed 10) c) Ovaries d)Cervix
a) Discharge is greenish and frothy 181. Most common site ofThberculosis of genital tract?
b) It is described as strawberry vagina a) Endometrium b) Ovary (APPG 08)
c) Culture ofdischarge is not necessary for diagnosis c) Fallopian tube d) Cervix
d) It is treated with doxycycline

158)b 159)a 160)d 161)a 162) a,c 163) a,b,d 164) b 165) b 166) c 167) c 168)d 169) a 170)d 171)a
172)a 173)c 174)a 175)b 176)a 177)b 178)c 179)c 180)b 181)c
GYNAE & OBS [ 11]

URINARY SYSTEM 193. Sonia who underwent vaginal hysterectomy and


lymphnode dissection, presents with incontinence
182. Stress in continence is a common symptom in - of urine and passing some amount of urine which
a) Prolapse uterus b)Fibroid (Kerala95) is under her control. Most likely diagnosis-
c) Adenomyosis d)VVF a) Vesicovaginal fistula (AIIMS OJ)
183. Marshall-Marchetti-Krant surgery is done for- b) Uretero vaginal fistula
a) Stress in continence (Karn 96) c) Stress incontinence
·b) Urge incontinence d) Urge incontinence
c) Vesicovaginal fistula 194. Vesico vaginal fistula in India is most commonly
d) Bladder obstruction due to- (UPSC 85, PGI 88)
184. Bonney's test is used to demonstrate- (DNB 91) a) Carcinomacervix
a) Stress incontinence b) Urge incontinence b) Gynaecological surgery
c) Fibroids d) True incontinence
c) Prolonged labour
185. Incontinence in elderly female is due to most
d) Carcinoma bladder
commonly- (CUPGEE 99)
195. A primipara who had a prolonged labour and difficult
a) Detrusor instability
b) True stress incontinence vaginal delivery three months ago presents with
c) Vesicovaginal fistule complaints of incontinence ofloose stools and Datos
d) Outlet obstruction from the day of delivery. The most likely
186. Urethral caruncle is best treated by- (UPSC 05) diagnosis - (UPSC 99)
a) Administration of antibiotics a) Chronic diarrhoea b) Recto-vaginal fistula
b) Excision followed by repeated dilatation c) Haemorrhoids d) Complete perineal tear
c) Resection and end to end anastomosis 196. 'BOARI'SOPERATION'is- (AP 97)
d) Chemical cauterization a) Renal pelvic flap b) Urinary diversion
187. Kelly's plication operation done in- (PGI June 05) c) Bladder flap d) Uretero-rectal anastanosis
a) Stress incontinence b) Vault prolapse 197. Chassar Moir technique is used in- (AMU 05)
c) Rectal prolapse d) Uterine prolapse a) Vesicovaginal fistula b) Stress incontinence
e) Cervical incontinence c) Urethrocele d) Enterocele repair
198. In women with ureterovaginal fistula the following
URINARY FISTULAS statements are false except- (J & K 05)
a) Produces free fluid in the abdominal cavity
188. Commonest cause of recto vaginal fistula is- b) 40 % heals spontaneously
a) Following Wertheim's operation (TN90) c) It is associated with hydronephrosis on the
b) Pressure necrosis during labour affected side
c) Improperrepairofperineal tear d) Should be repaired as soon as diagnosed on IVP
d) Abnormal presentation 199. Patients ofRectovaginal fistula should be initially
189. Ureterovaginal fistula formation is common at- treated with- (AI 05)
a) Infundibulopelvic ligament (PGI 96) a) Colostomy b)Primaryrepair
b) Vaginal vault c) Colporrhaphy d) Anterior resection
c) Distal to uterine artery near vaginal vault
200. All are treatment for SUI except- (MAHE 07)
d) Distal uterine artery in cardinal ligament
a) Kelly's plication
190. Manifestation ofuretero vaginal fistula is -(PGI 96)
b) Marshall- Marchett- Krants operation
a) Overflow incontinence b) Hydronephrosis
c) Continous incontinence d) Stress incontinence c) Bursch colposuspenion
191. One week after an extended hysterectomy, the d) All of the above
patient leaks urine per vaginum. In spite of the 201. ChassarMoiroperationisdonein- (UP07, 05)
leakage, she has to pass urine from time to time. a) Vesicovaginal fistula b)Vesicouterine fistula
The most likely cause is- (UPSC 97) c) Urethrovaginal fistula d) Rectovesical fistula
a) Vesico-vaginal fistula b) Ureterovaginal fistula 202. In VVF surgery, bladder drainage should be done
c) Stress incontinence d)Overflowincontinence for- (DPGEE 08)
192. In a case of incontenence of urine, dye filled into a) 10 days b) 12 days
the urinary bladder does not stain the pad in the c) 6 days d) 14 days
vagina, yet the pad is soaked with clear urine. 203. Which is not true about VVF? (APPG 08)
The most likely diagnosis is- (UPSC 2K) a) Amenorrhea b) Hydronephrosis
a) Vesicovaginal fistula ~U~a d)All
b) Uretero-vaginal fistula 204. Which of the following leads to cyclical haematnria?
c) Urinary stress incontenence a) Vesicouterinefistula b)VVF (APPG08)
d) Urethrovaginal fistula c) Rectovaginal fistula d) all

182)a l83)a 184)a 185)b 186)d 187)a 188)c 189)a 190)c 191)b 192)b 193)b 194)c 195)d
196)c 197)a 198)d l99)a 200)d 20l)a 202)a 203)a 204)a
GYNAE & OBS [ 12]

205. A 28 year female, after 6 month oflower segment 214. Which is not assisted reproduction technique-
caesarean section complains of cycHc hematuria with a) GIFT (AI 95)
normal menstrual cycle. What is probable b) ZIFT
diagnosis - (PGI Dec 08) c) IVF and embryonal transfer
a) Vesicouterine fistula b) Vesical endometriosis d) Artificial insemination into uterus
c) Choriocarcinoma d) Vesicovaginal fistula 215. Commonest indication of IVF is an abnormality
e) Urethrovaginal fistula in- (AI95)
206. The recommended non surgical treatment of stress a) Uterus b) Fallopian tube
incontinence is- (AI 09) c) Anovulation d)Azoospemia
a) Pelvic floor muscle exercises
216. Best investigation to assess tubal potency is-
a) Rubin's test (TN95, AI 89)
b) Bladder training
b) Hysterosalphingogram
c) Electrical stimulation
c) Laprotomy
d) Vaginal cone I weights
d) Laproscopic chromotubation
207. A 52 year old lady presents with constant leakage of 217. The best time of the menstrual cycle for
urine and dysuria two weeks after a compHcated total endometrial biopsy in infertility investigation is -
abdominal hysterectomy. A diagnosis ofVesicovaginal a)
First day of menstruation (UPSC 96)
fistula is suspected. The most important test for the b)
One week after the onset of menstruation
diagnosis is- (AI 10) c)
Just near due date of the next menstruation
a) Triple swab test b) Urine culture d)
On the 15th day of the onset on menstruation
c) Cystoscopy d)IVP 218. Tube testing is done under- (Karn. 9 5)
208. A 52 year old lady with a vesicovaginal fistula after a) Ketamine b) General anesthesia
abdominal hysterectomy is not responding to c) No anesthesia d) Local anaesthesia
conservative management. Most useful/important 219. Post-coital test showing non-motile sperms in the
next investigation is- (AI I 0) cervial smear and motile sperms from the posterior
a) Triple swab test b) Urine culture fornix suggests- (UPSC 97)
c) Cystoscopy d)IVP a) Faulty coital practice b) Immunological defect
209. A women presents to the Gynecology outpatient c) Hypospadiasis d)Azoospermia
departemnt with history of stress incontinence. 220. The commonest indication for in vitro fertilisation
Which of the foDowing is the treatment of choice for a) Tubal block (complete) (TN 98, PGI 89, 90)
genuine stress incontinence- (AI II) b) Oligospermia
a) Burch Colposuspension c) Uterine anomalies
b) Kelly's Procedure d) Cervical incomplence
c) Sling Suspension Procedure
221. Most reversible from ofinfertility- (PGI 97)
a) Anovulation b)Endometrosis
d) Tension Free Vaginal Taping
c) Oligospermia d) Tubal factor
222. Asthenosperima means- (UPSC 96)
THE PATHOLOGY OF CONCEPTION
a) Failure of the formation of sperms
b) No spermatozoa in the semen
210. In fertile period- (Kerala 94)
c) Reduction in the motility of sperms
a) Oestrogen decreases
d) Sperm count less than 20 millionlml of semen
b) Oestrogen increases
223. The test for detecting anti sperm antibodies -
c) Progesterone increases
a) Post-coital test b) Palm-leaf test (TN 99)
d) Progesterone decreases
c) Fern test d) SpinnBarkierttest
211. Endometrial biopsy for infertility is taken on .•• 224. Increased FSH level in azoospermic male
day- (JIPMER 90) indicates- (UPSC 2002)
a) 5th to 7th b) 12th to 4th a) Testicular atrophy b) Hypothalamic failure
c) 23rd to 26th d) 9th toll th c) Cryptorchidism d) Hypospadias
212. The effective sperm count normaly is -(PGI 80, DNB 225. Antisperm antibodies are usuaRy present in the-
a) 20million/ml b)30million/ml 90) a) Cervix b) Vagina (UPSC OJ)
c) 40 millionlml d) 50 millionlml c) Uterus d) Fallopian tube
213. Obselete method for testing tubal patency is- 226. Azoospermia with raised FSH level is seen in-
a) Hystero salpingogram (JIPMER 95) a) Obstruction of vas (AIIMS 92)
b) Chromo salpingogram b) Cryptorchidism
c) Rubin's test c) Testicular failure
d) None of the above d) Hypothalamic failure

205)a 206)a 207)a 208)c 209)d 210)c 21l)c 212)d 213)c 214)d 215)b 216)d 217)c 218)c
219)b 220)a 22l)a 222)c 223)a 224)a 225)a 226)c
GYNAE & OBS [ 13]

227. Male infertility occurs if the sperm count is less 240. In the postovulatory pulse, the appearance of the
than- (UPSC 95) endometriumofUSGis- (Bihar 03)
a) IOOmill/ml b) 75 milllml a) Single line
c) 50mill/ml d)20mill/ml b) Broken line
228. In semen banks , semen is preserved at low c) Single line surrounded by halo
temperature using- (JIPMER 81, DNB 90) d) Double line with halo
a) Dry ice b) Deep freeze 241. Fern testis due to- (SGPGI 05)
c) Liquid nitrogen d) Liquid air a) Presence of sodium chloride under progestron effect
229. Drugs used for ovulation induction- (POI 02) b) Presence of sodium chloride under estrogen effect
a) Clomiphene citrate b) Danazol c) LH/FSH
c) Cyproterone acetate d) Tamoxifen d) Mucus secretion by glands
230. Cervical hostility is tested by following 242. Post coital test (PCT) is done for- (PGI June 05)
except- (PG/97) a) Cervical receptivity b) Sperm motility
a) Spinbarkeit b) Post coital test c) Absolute sperm count d) Viable sperm count
c) Miller kuzrole test d) Keller test e) Endometrial function
231. Common causes ofinfertility are- (PG/2000) 243. Intrauterine insemination means implantation
a) Chlamydia b) Gonorrohea of- (PGI June 05)
c) Mycoplasma d) Pneumococcus a) Semen b) Washed semen
232. Persistent anovulation not treated leads to AlE- c) Million of sperm d) Fertilized ova
a) Hirsutism (PG/99) 244. PESA I MESA is helpful in- (Orissa 05)
b) Ovarian carcinoma a) Pre testicular azoospermia
c) Endomatrial carcinoma b) Testicular azoospermia
d) Increased risk of CVS disease c) Post testicular azoospermia
233. Infertility is seen in- (PGI 02) d) Asthenospermia
a) Fibroid uterus b) Endometriosis 245. Aspermia is the term used to describe- (AI 05)
c) Adenomyosis d) PID a) Absence of semen
234. Fallopian tube patency is checked by- (POI 02) b) Absence of sperm in ejaculate
a) Hystrosalpingography b) Laparoscopy c) Absence of sperm motility
c) Hysteroscopy d) USG d) Occurence of abnormal sperm
e) CTscan 246. In the perspective of the busy life schedule in the
235. Time of ovulation is detected by- (PGI 03) modern society, the accepted minimum period of
a) UrineLH b)UrineFSH sexual cohabitation resulting in no offspring for
c) UrineHCG d) Senum Estradiol a couple to be declared infertile is- (AIIMS May 05)
e)BBT a) Oneyear b)Oneandahalf-year
236. Best test for measuring Graman follicle maturation c) Two years d) Three year
is- (Jipmer 03) 247. An Antihormonal substance used to induce
a) Fern test ovulation is- (AI 07)
b) Endometrial biopsy a) Mefiprestone b) Clomiphene citrate
c) Transvaginal ultrasonography c) Tamoxifen d) Raloxifen
d) Abdominal ultrasonography 248. Aspiration of sperms from testes is done in-
237. Sonosalphingography is done for- · (IN 03) a) TESA b) MESA (AI 07)
a) Measuring basal body temperature c) ZIFT d) GIFT
b) To detect pregnancy 249. True about clomiphene citrate- (PGI June 07)
c) Testing tubal patency a) Commonly causes hyperstimulation syndrome
d) Determining anovulatory cycle b) Used for ovulation induction
238. According to WHO criteria, the minimum normal c) Multiple pregnancies seen in 3- 8% cases
sperm count is- (AIIMS 03) 250. Asherman's syndrome can be diagnosed by all
a) 10 million/ml b) 20 millionlml except- (AIIMSNov07)
c) 40millionlml d)60millionlml a) Hysterosalpingography b) Saline sonography
239. In azoospermia, the diagnostic test which can c) Endometrial culture d) Hysteroscopy
distinguish between testicular failure and 251. Which is a not an essential criteria according to
obstruction in the vas deferens is- (UPSC 04) WHO for normal semen analysis- (AIIMS Nov 07)
a) Estimation ofFSH level a) Sperm count> 20 millionlml
b) Estimation of testosterone level b) Volume> 1 ml
c) Karyotyping c) Spermwithnormalmorphology(strictcriteria)> 15%
d) FNAC of testes d) Motility> 25% with rapidly progressive motility

227)d 228)c 229)a,d 230)a 23l)a,b,c 232)b 233)a,b,d 234)a,c 235)a,d,e 236)c 237)c 238)b 239)a
I
240)a 24l)b 242)a 243)a,b,c 244)c 245)a 246)a 247)b 248)a 249)b,c 250)c 251)b
GYNAE & OBS [ 14]

252. Which one of the following is the WHO criterion 263. The most common cause of secondary amenorrhoea
for oligospermia in seminal fluid analysis ? is - (DELHI PG Mar. 09)
a) Spenn count less than 50 million!HPF(UPSC-II 08) a) Polycystic ovarian disease
b) Spenn count less than 40 million!HPF b) Premature ovarian failure
c) Spenn count less than 30 million!HPF c) Pregnancy
d) Spenn count less than 20 million!HPF d) Prolactinoma
253. TrueaboutPCOD- (PGI Dec 08) 264. The features of Sheehan's syndromes may include
the following except- (DELHI PG Mar. 09)
a) iLH&!FSH b)iFSH&!LH
a) Amenorrhoea
c) i LH & i FSH d) Hyperinsulnaemia
b) Galactorrhoea
e) iTSH c) Loss of pubic and axillary hair
254. Fallopian tube dysmotility is seen in? d) Hypothyroidism
a) Noonan syndrome (AIIMS Nov 08) 265. Polycystic ovarian syndrome is associated with the
b) Turner syndrome followingexcept- (UPSC-II09)
c) Kartagener syndrome a) Ovarian carcinoma
d) Marfan syndrome b) Endometrial hyperplasia
255. Best indicator of ovarian reserve is ? c) Impaired glucose tolerance
a) FSH b)Estradiol (AIIMSNov08) d) Dyslipidemia
c) IH d) FSH/LH ratio 266. True about WHO normal sperm analysis-
256. All are true about polycystic ovarian disease except? a) Sperrncount>20million/hpf (PGI Nov 09)
a) PersistentlyelevatedLH (AIIMS Nov 08) b) Progressive motility> 50%
b) Increased LHIFSH ratio c) Norrnalmorphology>30%
c) Increased dheas d) Spenn count> 20 million per ml
d) Markedly increased prolactin e) Rapidly progressive motility more than 30%
267. Impaired ciliary motility in - (AIIMS Nov 09)
257. Which of the following is not a cause of primary
a) Nuvian syndrome
amenorrhea? (AI 09)
b) Kartagener's syndrome
a) Sheehan's syndrome b) Kallman's syndrome 268. A 35 yr old woman presents with primary infertility
c) Turner's syndrome d) Rokitansky syndrome & palpable pelvic mass. Her CA-1251evel is 0 U/ml.
258. Which of the following is true about obstructive Diagnosis is - (AIIMS May I 0)
azoospermia- (AI 09) a) Ovarian Ca b) Endometrioma
a) FSHandLH b)NonnalFSHandNonnalLH c) Tuberculosis d) Borderline ovarian tumor
c) LH,NormalFSH d)FSH,NonnalLH 269. Testfor ovarian reserve- (AIIMS May I 0)
259. True about clomiphene citrate all except- (AIIMS a) IH b)LHIFSHratio
a) Enclomiphene has antiestrogenic affect May 09) c) FSH d) Estradiol
b) Chance of pregnancy is three fold as compared to 270. Which of tbe following methods for assessment of
placebo female fertility during a menstrual cycle can best
c) Risk of multiple pregnancy is 6-10% predict the timing of ovulation- (AI I 0)
d) It can also be used for male infertility with a) Basal body temperature (BBT)
oligozoospermia b) Fern test
260. lntraterine Insemination is indicated in all c) Spinnbarkeit phenomenon
d) Hormonal study
except- (PGIJune09)
271. All of the following are associated with polycystic
a) Unexplained infertility b) Endometrosis
ovariansyndromeexcept- (AI IO)
c) Male infertility d)PCOD a) Ovarian carcinoma b) Endometrial carcinoma
e) Tubal factor infertility c) Insulin resistance d) Osteoporosis
261. TrueaboutPCOD- (PGIJune09) 272. Best investigation to assess tubal patency is-
a) i LH&!FSH b)iFSH&!LH a) Rubin's test (DPG IO)
c) i LH & ! FSH d) Hyperinsulinaernia b) Hysterosalpingogram ·
e) iTSH c) Laparotomy
262. A patient with Stein Ieventhai syndrome is treated by d) Laparoscopic chromotubation
which oftbe following methods-(DELHIPG Feb. 09) 273. A 28-year old lady, Rani is suspected to have
a) FSH polycystic ovarian disease. Sample for testing LH
b) Clomiphene citrate and FSH are best on the following days of menstrual
c) Bilateral wedge resection of ovaries cycle- (DPG I 0)
d) Progesterones a) 14 b)8-10
c) 13-15 d)24-26

252) d 253) a,d 254) c 255) a 256)d 257)a 258)b 259)b 260) a,d,e 261) a,d 262) b 263) c 264) b 265) a
266) b,d 267) b 268) b 269) c 270)d 271)d 272)d 273)a
GYNAE & OBS [ 15]

274. Ifa patient of polycystic ovary syndrome on metformin 285. The following hormonal changes mark the
conceives, how soon should the metformin be Polycystic Ovarian Disease except- (UPSC II II)
stopped? (UP SC II I 0) a) Hyperinsulinaemia
a) Immediately following the diagnosis of pregnancy b) Hyperandrogenism
b) Afterthe 1st trimester c) Raised LH, Raised FSH
c) After the 2nd trimester d) Raised LH, Low-to-normal FSH
d) Before the onset of labonr 286. The absence of fructose in the seminal fluid indicates
275. True about PCOS- (PGI May I 0)
adefectof- (UPSC II 11)
a) HighFSHILHratio
a) Testicular tubular epithelium
b) U nilaterallarge ovarian cyst
c) Hirsutism b) Seminal vesicles
d) i ed risk ofDM c) Leydig cells
e) OCP is given for treatment d) Hypothalamic-pituitary axis
276. Durgs used for ovulation induction are-(PGINov. I 0)
a) GnRH b) Clomiphene Citrate CONTRACEPTIVES
c) Gonadotropins d) Letrozole
e) Danazole 287. The success rate following tubalrecannalisation
277. Thbal patency is checked by all except-(PGINov. I 0) is low after- (AI 9I)
a) Hysterosalpingography b) Laparoscopy a) lrwins method b) Pomeroys' method
c) Folloscopy d) Colposcopy c) Fimbriectomy d) Madlener'smethod
e) Sonosalpingography 288. Contraception of choice for a working women
278. Intrauterine insemination is indicated in all with irregular and profuse bleeding- (KERALA 9I)
except- (PGINov. 10) a) IUSD b)Barrier
a) Unexplained infertility b) Endometriosis c) Spermicide d) Oral pills
c) Maleinfertility d)PCOD 289. CopperTactshy- (JIPMER 9I)
e) Tubal factor infertility a) Causing aseptic endometritis
279. In a study it is observed tbatthe right ovary ovulates b) Increasing cervical mucus
more thant the left, all are possible explanations for c) Inhibiting tubal motility
the cause except- (AIIMS Nov 10) d) Inhibiting ovulation
a) Anatomical asymmetry 290. Method of sterilisation least suited for
b) Difference in blood supply to both sides Recanalisation is- (AI 92)
c) Right handedness is more common in population a) Clips b) Bipolar cauterisation
d) Some embryological basis c) Fallopian rings d) Pomeroys technique
280. Test for ovarian reserve- (AIIMS May 11) 291. Incidence of ectopic pregnancy is highest with-
a) IH b) LH/FSH ratio a)O.Cpills b)CopperT (AI93)
c) FSH d) Estradiol c) Multiload IUCD d) Progestasert
281. All of the following hormonal observations in PCOD 292. The drug in Norplant sub-dermal implant hormonal
are true, except- (AI 11) contraceptive is- (KARN 94)
a) HighLH/FSH b) High Androgens a) Norethisterone b) Norethynodrel
c) HighProlactin d)HighLH c) Medroxy- progesterone d) Levenogesterol
282. Decreased motility offallopian tube is seen in- 293. Post- coital contraceptive methods include all of
a) Chnrg- strauss syndrome (AI 11) the following except- (KARN 94)
b) Kartagener's syndrome a) RU-486 b)Danzol
c) Noonan syndrome c) Copper-T insertion d) Novoxynol
d) Turner syndrome 294. Failure rate of tubectomy is minimum
283. Which of the following is true about obstructive in- (JIPMER 95)
azoospermia- (AI 11) a) Madlener procednre b) Parkland procedure
a) iFSHandiLH c) Pomeroy procednre d) Irving procednre
b) NormalFSHandNormalLH 295. Which one of the following drugs can reduce the
c) iLH,NormalFSH efficacy oflow dose oral contraceptive pill-
d) iFSH,NormalLH a) Penicillin b) Tetracycline
284. As herman's syndrome typically results from- c) Ampicillin d) Rifampicin
a) Excessive curettage during dilatation and
296. Which one of the following is a non - steroidal
curettage (UPSC II 11)
contraceptive- (UPSC 96)
b) Post-partiim haemorrhage
a) Ru-486 b) Minipill
c) Prolonged usage of oral contraceptives
c) NET-EN d) Centchroman
d) Use of intrauterine contraceptive device

274) None 275) c,d,e 276) b,c 277)d 278) a,d,e 279) c 280) c 281) c 282) b 283) b 284)a 285)c 286)b
287) c 288) d 289) a,d 290) b 29l)d 292)d 293)d 294)d 295)d 296)d
GYNAE&OBS[16]

297. All offollowing are useful post- coital contraceptives 311. Nonoxynol- 9 (marketed as 'Today') is alan-
except- (AI 96) a) Hormonal contraceptive (UPSC 96)
a) IDCD b) High dose combination pill b) Intrauterine contraceptive
c) RU 486 d) Progesterone only pill c) Barrier contraceptive
298. Best contraceptive for a young unmarried female d) Post - coital contraceptive
is - (DELHI 96) 312. All of the following are absolute contraindincations
a) Oral pills b) IDCD for IUCD insertion, except- (UPSC 96)
c) Diaphragm d) Vaginal pessary a) Pelvic infection
299. Mestranol acts as a contraceptive by- (DELHI 96) b) Pregnancy
a) Inhibiting FSH secretion c) Undiagnosed abnormal vaginal bleeding
b) Inhibiting LH secretion d) Valvular heart disease
c) Inhibiting tubal motility 313. All ofthe following statements regarding lost IUCD
d) Inhibiting nidation are true except-
300. AmountofoestrogeninMALA-Dis- (AMU95) a) Pregnancy should be excluded before other
a) 30mg b)50mg evaluadtion is done
c) lOmg d)80mg b) Gentle probing should be done with a uterine sound
301. Following is commonest constituent of c) Utrasonography is the best method of diagnosis
contraceptive foams- (AIIMS 95) d) Laparotomy is required in all cases
a) Progestrone b) Nonoxylol- 9 314. All ofthe following are the advantageofDepot-provea
c) Estradiol d) Phensolone sulfate except- (UPSC 96)
302. Where is a missed IUCD most commonly found- a) It can be given to lacting mothers
a) Inside the uterus (CUPGEE 95) b) A 150mg dose is effective for a periodofl2 weeks
b) In the adominal cavity outside the uterus c) It gives good cycle control
c) Both are equally commonly encountered d) It prevents ectopic pregnancy
d) None is a common site 315. After vasectomy aspermia will become evident in-
303. All are true about centchroman except- (AP 96) a) 2 to 3 weeks b) 4 to 6 weeks (UPSC 96)
a) Anti oestrogenic c) 8to10weeks d)l0tol2weeks
b) Not a teratogen 316. Oral contraceptive pill containing progesterone
c) Long acting pill which is given in small quantities for 30 days a
d) Failure is 10 per 100 women years monthisknownas- (MAHE 98)
304. Best recanalization after tubectomy is possible with- a) Micro pill b) Sequential pill
a) lsthmo - Isthemic anastomosis (AI 97) c) Combined pill d) None of the above
b) Isthmo- ampullary 317. A case of vasectomy is said to have failed as the
c) Ampullo- amullary vasetomised person's wife gives birth to a child
d) Comou -isthmic ten months after the operation. Which one of the
305. Commonest side effect of Cu-T- (AIIMS 96) foDowing is the most probable cause- (ICS 98)
a) Bleeding b) Pain a) Failure of the husband to use a condom after
c) Perforation d) Pelvic inflammatory disease vasectomy
306. Ectopic dpregnancy is seen maximum with- b) Surgical failure
a) IDCD b)OCP (JIPMER98) c) Recanalisation
c) Barrier method d) Tubal sterilization d) Wife had extra- marital contact
307. OCP's increases the risk of which Carcinoma- 318. Billing's method of contraception refers to-
a) Caendometrium b)CaCx (JIPMER 98) a) Monitoring basi body temperature (UPSC 99)
c) Breast d) Ovarian Ca b) Cervical mucus method
308. Absolute contra indication for copper Tis-
c) Rhythm method
a) Endometriosis b)PelvisTB (AIIMS97)
d) Coituds interruptus method
c) Anaemia d) Heart disease
319. OCPprotectsagainstaDexcept- (AI 99)
309. Oral pill maypreventthedevelopmentof- (AP 97)
a) Hepatic adenoma b) Fibroadenoma breast
a) Carcinoma breast b) Ovarian malignancy
c) Ca ovary d) Uterine malignancy
c) Sub - mucous fibroid d) All of the above
320. IUCD actsbymechexcept- (AIIMS98)
310. Peritoneum is opened in all of the following
a) Ovulation inhibition
sterilization procedures except- (AP 97)
b) Chronic endometritis
a) Mini lap b) Laparoscopy
c) Uterine endometrial atrophy
c) Vasectomy d) Transvaginal tubectomy
d) Tubal dysmotility

297)d 298)a 299)b 300)a 301)b 302)a 303)d 304)a 305)a 306)a 307)b 308)b 309)b 310)c
3ll)c 312)d 313)d 314)c 315)d 316)a 317)a 318)b 319)a 320)a
GYNAE & OBS [ 17]

321. Most efficient tnboplasty is-(AIIMS 98, UPSC 84, 86) 332. Machanism of action of spermicidal jelly- (UP 2K)
a) Isthmo- isthmic b) Isthmo ampullary a) Acrosomal cap degeneration
c) Fimbria! reconstitution d) Cornuoplasty b) Altered vaginal enzyme
322. Estrogen in the OCP causes all the following except- c) Alterd cervival mucus
a) Carcinoma in situ cervix (TN 99) d) Absence of glucose transport
b) Breast carcinoma 333. Maximum chances offailure are seen in- (UP 2K,
c) Endometrial carcinoma a) Laproscopic yoon 's ring AIIMS 98)
d) Tbromoembolism b) Vaginal fimbriectomy
323. Best contraceptive pill during lactation- (AP 98) c) Hysteroscopic tubal block
a) Fixed dose pills b) Progesterone pills d) Pomerey's method
c) Mini pills d) Long acting pills 334. When a Cu -Tis implanted in myometrium, then
324. The low Dose progesterone - only type of oral the treatment is- (CUPGEE 2K)
contraceptive acts by- (KARNAT 99) a) Hysteroscopic removal
a) Inhibition of the midcycle surge of leteinizing b) Removal by IUCD hook
hormone c) D&C
b) Inhibition of follicle stimulating hormone d) Hysterectomy
secretion 335. Post coital contraception are all except- (DNB OJ)
c) Preventing ovulation a) High dose estrogen b)OCP
d) Rendering cervical mucus less penetrable by c) IUCD d)DMPA
sperm 336. Safe period in rhythms method is : (In a female
325. Which contraceptive helps in preventing witb 28 days menstnal cycle)- (MAHE 2001)
carcinomacervix- (ORRJSA 99) a) Intial 14 days
a) Barrier contraceptive b) Later 14 days
b) Intra cervical device c) First and last 7 days
c) Intra uterine contraceptive device d) 10 to 21 days of the cycle
d) Oral pill
337. Which one of the following is not used for
contraception- (UPSC 2002)
326. Which one of the following is the contraceptive
a) Oestrogen - progesterone combined pills
method of choice for a commercial sex worker-
b) Progestin pills
a) OC pills b) IUCD (UPSC 2K)
c) Danazol
c) Tubectomy d) Barrier method
d) Intrauterine contraceptive device
327. Yuzpe method is a type of- (UPSC 2K)
338. RU486 is- (TN 2002)
a) Post- coital hormonal contraception
a) Anti progestogen b}Anti oestrogen
b) Male contraceptive method
c) Anti androgen d) Anti gonadotropin
c) Post- coital IUCD contraception
339. Following vasectomy for family planning, a
d) Minilap sterilization patient should be advised to use some other
328. Oral combined contraceptive pills contain which method of contraception, till- (UPSC 97)
one ofthe following sets of hormone- (UPSC 2K) a) Removal of all sutures
a) Ethinyl oestradiol, progestrogen and oestrone b) Pain completely subsides
b) Progestrogen and Ethinyl oestradiol c) Twoweeks
c) Progetrogen andoestrone d) Eight weeks
d) Oestrone and ethinyl oestradiol 340. Contraception of choice for a postpartum village
329. What should be done if a woman misses taking woman with one child- (KERALA 91)
Saheli for 4 consecutive days- (AIIMS 99) a) Barrier b) Copper T
a) Take 4 tablets when she remembers c) Oral pills d) Spermicide
b) 2 tablets taken twice daily 341. Do pot medroxyprogesterone acetate is sparingly
c) Start as new cycle used as a contraceptive because it causes-
d) Husband should use condoms .a) Cardiovascular complications (AIJMS 92)
330. Cu-T embedded in myometrium- it is to be removed b) Lactational failure
by- (CULCUITA 2K) c) Breast cancer
a) Hysteroscope b) Currete d) Irregular menstrual bleeding
c) Hook d) Laparoscopy 342. Diabetic woman witb malignant retinopathy
331. True about mifepristone is- (UP 2K) requires-
a) Can be used for producing early abortions a) Permanent sterilization
b) Mid Luteal defect b) Conventional contraceptives
c) Acts on intra cytoplamic receptors c) IU devices
d) Prevents fertilzation by aseptic inflammation d) Oral pills

32l)a 322)c 323)b,c 324)c,d 325)a 326)d 327)a 328)b 329)a 330)a,b,c 33l)a 332)None 333)c
334)a 335)d 336)c 337)None 338)a 339)None 340)b 34l)d 342)b
GYNAE & OBS [ 18]

343. A lady with 3 living children comes with 8 wks. 354. OCP's are contraindicated in AlE- (PGI 99)
Pregnancy with CuT Inutero, treatment suitable is- a) Smoking 35 years b) Coronary occlusion
a) Continue pregnancy &Remove CuT during delevecy c) Polyscystic ovarian ds d) Cerebrovascular ds
b) Remove CuT and permanent sterilisation 355. Contraindications to IUCD are- (PGI 2000)
c) Remove Cu-T (UP 96) a) Pelvic infl. disease (PID)
d) None b) Congenital uterine abnormality
344. All of the following statements concerning methods c) Liver disease
of limiting family size, are true, except- (UPSC 97)
d)DM
a) Women who do not breastfeed are much more
e) Heart disease
likely to become pregnant than those who do
b) Globally the number of tubectomies far exceeds 356. Which of the following statements is true about
the number of vasectomies mifepristone- (PGI 01)
c) The barrier methods (condoms and diaphragms) a) Also calledRU-486
are perhaps the most effective at limiting family size b) It is a 19-norsteroid
d) All anovulatory drugs are particularly likely to c) Acts on receptors
cause side effects in women over 35 years of age d) Given only intravenously
and women Who smoke e) Used for menstrual regulation
345. IUCDsinnotcontaindicatedin- (JIPMER 98) 357. Oral contraceptive causes- (PGI 01)
a) Previous pelvic inflammatory disease a) Carcinoma endometrium b) Carcinoma cervix
b) Irregular periods c) Bile duct proliferation d) Hepatic adenoma
c) Previous LSCS e) Breast cancer
d) Cacervix 358. Absolute contraindication of OC pill is- (PGI 02)
346. Long acting IUCD's are all except- (AIIMS 97) a) Breast ca b) Mentally ill
a) Progestasert b) CuT c) Migraine d) Fibroid
c) Lippes loop d) Multiload
e) Hyperlipidemia
347. Absolute contra-indication of Norplant-
359. Wbichoneofthefollowingisthemostcommon problem
a) Acute thrombophlebitis (Kerala 97)
b) Hypertension associated with the use of condom- (UPSC 02)
c) Diabetes mellitus a) Increased monilial infection of vagina
d) Hypercholesterolemia b) Premature ejaculation
348. In Cu-T 200-200 means- (PGI 97) c) Contact dermatitis
a) 200mgofCu b)200mm2 ofCu d) Retention of urine
c) 200cu.mmofCu d) None of the above 360. Which one of the following intrauterine
349. Progestasert is effective for- (AP 97) contraceptive devices has the lowest pregnancy rate-
a) 1 year b) 2 years a) Lippes' loop b) Cu- 7 (UPSC 02)
c) 4 years d) 5 years c) Cu T- 200 d) Levonorgestrel IUD
350. Copper- T with threads is visible in a case of early 361. A primipara with a cardiac lesion (MI) has come
pregnancy. Treatment of choice- (PGI 98) on the 40th day of delivery asking for contraception.
a) Remove CU-T only The contraceptive of choice is- (UPSC 02)
b) Suction evacuation with copper - T removal a) Condom with spermicidal jelly
c) Reassurance and continue pregnancy b) Oral contraceptive pill
d) Lapratomy
c) Intrauterine contraceptive device
351. A lady with IUCD becomes pregnant with tail of
d) Laproscopic sterilization
IUCD being seen next course of action is-
a) MTP (PG/98) 362. Which one of the following hormonal contracep
b) Remove the IUCD tive CANNOT be used during lactation -(UPSC 02)
c) Continue the pregnancy a) Mini pill
d) Remove IUCD and terminate pregnancy b) Norplant
352. OC pills intake cause psychiatric symptoms, and c) DMPA
abdominal pain. Diagnosis is- (PGI 98) d) Combined oral contraceptivetives
a) Acute intermettent porphyria 363. Third generation oral contraceptive pills contain
b) Systemic lupus ing norgestrel and gestodene along with estrogens-
c) Thrombosis a) Are more lipid friendly (UPSC 02)
d) Anemia b) Decrease the risk of venous thromboembolism
353. OC pills efficiency is reduced by simultaneous use of- c) Increase the risk of break through bleeding
a) Rifampicin b) Carbamazepine (PGI 98) d) Are not used for emergency contraception
c) Propranolol d) Tricyclic antidepressants

343)b 344)c 345)a,c 346)a 347)a 348)b 349)a 350)a 35l)b 352)a 353)a,b 354)c 355)a,b,d,e
356)a,b,c,e 357)b,d 358)a,e 359)c 360)d 36l)a 362)d 363)a
GYNAE & OBS [ 19]

364. Medical Termination of Pregnancy Act was passed 377. OCP's are contraindicated in all except-
by the parliament oflndia in- (UPSC 02) a) Intermittent vaginal bleeding (SGPGI 05)
a) 1950 b) 1965 b) Uterine fibroids
c) 1971 d) 1981 c) H/0 thromboembolism
365. Ideal time for insertion of Iippes loop is- d) Cardiac abnormalities
a) Before menstruation (Jipmer 03) 378. Which one ofthe following is longest acting IUCD
b) 2 - 3 days after period is over a) Progestasert b)Cu-T-380 (AMU05)
c) On the J41h day of menstrual cycle c) Mirena d) Norgesterol
d) Any time during the cycle 379. The most appropriate drug for emergency
366. Pearl Index is a measure of- (TN 03)
contraception is- (AMU 05)
a) Potency of contraceptives
a) Misoprostol b) Ethnyl estradiol
b) Fertilitymte
c) Levenorgesterol d) Mifepristone
c) Potency of disinfectant
d) Couple protection rate 380. To avoid conception D.M.P.A. given- (HPU 05)
367. Low dose progestational contraceptives primarily a) Monthly internals b) 3 - monthly
acton- (AIIMS03) c) 6- monthly d) Yearly
a) Oviductal motility b) Uterine endometrium 381. Copper-Tis preferably inserted postnatal after-
c) CeiVix d) Pituitary a) 2weeks b)4weeks- (Karnat 05)
368. Life for as long as 10 Years is of which IUCD- c) 5 weeks d) 8 weeks
a) CuT200 b) Cu250 (Mahara 02) 382. Pearl's index indicates- (PGI June 05)
c) Cu T380A d) Profestasert a) Malnutrition b) Population
369. The use of combined oral contraceptive pill is c) Contraceptive failure d) LBW
associated with an increased incidence of- e) IUGR
a) Bacterial vaginosis (AJIMS 03) 383. Reversible methods of contraception are -(PGJJune
b) Chlamydial endocervictis a) Female sterilization b) OCP 05)
c) Vaginal warts c) IUCD d) Barrier
d) Genital herpes e) Depot injection
370. Contraception contraindicated in AIDS -(Orissa 04) 384. Composition ofNova- T- (PGI June 05)
a) OCpill b) Sterilization a) Copper & silver b) Copper & aluminium
c) IUCD d) None c) Copper only d) Copper & Selenium
371. Which one is not true regarding Centchroman? e) Copper&molybdenum
a) It is antiestrogenic (UPSC 04) 385. Contreceptive advice in lactating mother-
b) It acts on the endometrium a) Combined pill b) Mini pill (PGI 05)
c) It is a synthetic hormone c) IUCD d)Barriermethod
d) It is developed in India
e) Depot injection
372. In which of the following situations it is preferable
386. Absolute contraindications to the use of combined
notto insert IUCD- (UPSC 04)
oral contraceptives include the following, except-
a) Post - menstrual
b) Emergency postcoital a) Porphyria (J & K 05)
c) Six weeks after delivery b) Cerebral haemorrhage
d) Immediately following mid- trimester abortion c) Previous herpes genitalis
373. Which one of the following is not used as d) Past history of trophoblastic disease
emergency contraception- (UPSC 04) 387. Which one of the following is the ideal
a) Levonogestrel b) Monophasic pill contraceptive for a patient with heart disease-
c) DMPA d) Progestasert a) IUCD b)Depoprovem (AJ05)
374. Antiprogesterone compound RU- 486 is effective c) Diaphragm d) Oral contraceptive pills
for inducing abortion if the duration of pregnancy 388. The most common side effect of IUD insertion is-
is less than- (All India 04) a) Bleeding b)Pain (AJ05)
a) 63 days b) 72 days c) Pelvic infection d) Ectopic pregnancy
c) 88 days d) 120 days 389. Characteristics of an ideal candidate for Copper-
375. NovaThasthefollowing- (Bihar03) T insertion include all of the following except-
a) Silver core b) Copper core a) Has borne at least one child (AIIMS May 05)
c) Platinum core d) Iron core b) Is willing to check IUD tail
376. Contraceptive which is not an interceptive is- c) Has a history of ectopic pregnancy
a) Cu-T b) Norplant (Bihar 03) d) Has normal menstrual periods
c) RU- 486 d) OC pills

364)c 365)b 366)a 367)c 368)c 369)b 370)c 37l)b,c,d 372)d 373)c,d 374)a 375)a 376)b 377)b
378)b 379)c 380)b,c 38l)d 382)c 383)b,c,d,e 384)a 385)b,d,e 386)c 387)c 388)a 389)c
GYNAE & OBS [ 20]

390. In a young female of reproductive age an absolute 400. Eligibility criteria for sterilization include the
contraindication for prescribing oral contraceptive following except- ( UPSC 06)
pills is - (AIIMS May 05) a) Age of the female should be above 22 years
a) Diabetes b) Hypertension b) Age of the male should be below 60 years
c) Obesity d) Impaired liver function c) Ageofthefemale shouldnotbemorethan45 years
391. Use of oral contraceptives decreases the incidence d) Couple should have at least two children
of all of the following except- {AIIMS May 05) 401. Methods used for Laparoscopic sterilization
a) Ectopic pregnancy include the following except- (UPSC 06)
b) Epithelial ovarian malignancy a) Electrocoagulation b) Falope ring
c) Hepatic adenoma c) Irving d) Filchie clip
d) Pelvic inflammatory disease
402. Which of the following IUD is used for patients
392. All of the following mechanisms of action of oral
with menorrhagia- (APPG06)
contraceptive pill are true, except- {AI 06)
a) C4250 b) Multiload
a) Inhibition of ovulation
b) Prevention of fertilization c) Nova T c) Progestasert(Levonorgestrol)
c) Interference with implantation of fertilized ovum 403. "Today" sponge contains which of the following-
d) Interference with placental functioning a) Nonoxynol d) Delfen (APPG 06)
393. Which of the following statements is correct c) Femshield d) Desogestrel
regarding levonorgestrel releasing intrauterine 404. IUD CU380A life span is- (Manipal 06)
system? {AI 06) a) !Year b)4Year
a) There is increased incidence of menorrhagia c) 5Year d) lOYear
b) This system can be used as hormone replacement 405. Yuzpe method is used in- (Manipal 06)
therapy a) Post coital hormonal regime
c) This method is useful for the treatment of b) Intrauterine contraceptive method
endometerial hyperplasia c) Post coital intrauterine contraceptive device
d) Irregular uterine bleeding can be problem initially d) Tukalligation
394. All of the following mechanisms might account 406. Non contraceptive benefits of OC pills may be seen
for a reduced risk of upper genital tract infection in all ofthe following conditions except- {AI 07)
in users of progestin- releasing IUDs, except- a) Caendometrium
a) Reduced retrograde menstruction {AI 06) b) Ca breasts
b) Decreased ovulation c) Rheumatoid arthritis
c) Thickened cervical mucus d) Endometriosis
d) Decidual changes in the endometrium 407. The following statements are true about
395. Emergency contraception prevents pregnancy
Intrauterine devices (IUD) except- (AI 07)
by all of the following mechanisms, except-
a) Multiload Cu-375 is a third generation IDD
a) Delaying/inhibiting ovulation {AI 06, AI 03)
b) The pregnancy rate of Lippes loop and Cu - T
b) Inhibiting fertilization
200 are similar
c) Preventing implantation of the fertilized egg
d) Interrupting an early pregnancy c) IDD can be used for Emergency Contraception
396. Which is the safest contraceptive method for a within 5 days
woam with sickle-cell anaemia- (UPSC 06) d) Levonorgestrel releasing IDD has an effective life
a) Intrauterine device of5 years
b) Low dose progesterone pill 408. Which of the following IUCD's has a life span of
c) Condom or diaphragm 10 years- (AI07)
d) Low dose oestrogen-progesterone pill a) Cu380A b)Cu200
397. OCP given protection against following cancers- c) NOVA-T d)Multiload
a) Endometrial b) Ovary {PGI June 06) 409. Most common site for female tubal sterilization
c) Carvix d) Breast is- (AI07)
e) Liver a) Isthmus b)Ampulla
398. Spermicidal agent are- (PGI June 06) c) Cornua d) Infundibulum
a) Nonoxynol b) Menfegol 410. The most common site of ligation in female
c) Progestasert sterilisation is - (Aiims May 07)
399. Emergency contraception include the following a) Isthmus b)Ampullary
except- ( UPSC 06) c) Cornual d) Fimbria}
a) Oral contraceptive b) Copper - T
c) Laevonorgestrol tablets d) DMPA

390)d 39l)c 392)d 393)a 394)b 395)d 396)c 397) a,b 398) a,b 399) d 400) c 40l)c 402)d 403)a
404)d 405)a 406)b 407)a 408)a 409)a 410)a
GYNAE & OBS [ 21 ]

411. Math List I with List II and select the correct answer 2. When sperms are absent from two consecutive
using the code given below the lists M samples of semen, the vasectomy is considered
Listl List II complete for contraception
(Drug for indiction of (Daily dose) Which of the statements given above is/are correct-
ovulation a) 1 only b) 2 only
A. Clomiphene 1. 2.5 mg BDS c) Both 1 and 2 d) Neither 1 nor 2
B. Metformin 2. 50 mg OD 420. A 20 year old girl was raped around 13th day of
C. Bromocriptine 3. 75 IU OD her regular cycle. The best possible emergency
D. Gonadotrophin 4. 500mg1DS contraception to be offered would be- (UPSC 07)
Code:
a) Centchroan tablets
A B C D
b) Cu-T insertion
a) 3 1 4 2
c) Levonorgestrel tablets
b) 3 4 I 2
d) Ethinylestradiol tablets
c) 2 1 4 3
d)2 4 1 3 421. Laparoscopic approaches are usually performed-
412. Which one of the following is not a correct statement a) Immediately post-partum (UPSC 07)
regarding the use of condom M (UPSC 07) b) > 4 weeks post-partum
a) Air shoud be squeezed out of the tip c) > 6 weeks post-partum
b) It should be tested by inflating d) Anytime
c) It should be unrolled on erect penis 422. MIRENAis- (AIIMSNov07)
d) K~Y jelly may beused for lubrication a) Progesterone IUCD b) Hormonal implant
413. The levonorgestrel IUD prevents the pregnancy c) Anti progesterone d) Used for MTP
primarily by M (UPSC 07) 423. Chronic treatment with tamoxifen can cause
a) Suppressing ovulation carcinoma of- (Corned 07)
b) Making endometrium unreceptive a) Ovary b) Endometrium
c) Making cervical mucus thick c) Cervix d) Vulva
d) Making inflammatory changes within endometrium 424. Oral contraceptive pill of choice in a lactating
414. The highest incidence of ectopic pregnancy woman is- (Corned 08)
amongst contraceptive users is observed with M
a) Monophasic pill b) Biphasic pill
a) Combined contraceptive pills (UPSC 07) c) Triphasic pill d) Mini pill
b) Cu~T intrauterine contraceptive device 425. CoT 380A IUCD should be replaced once in-
c) Progestasert intrauterine device a) 4 years b) 6 years (Corned 08)
d) Lippes loop intrauterine device c) 8 years d) 10 years
415. Which one of the following is the most suitable 426. Least pregnancy failure rate in- (UP 08)
situation for prescribing the progestin only pili- a) IUCD
a) Perimenopausal patients (UPSC 07) b) Oestrogen + Progesterone
b) Emergency contraception c) Oestrogen alone
c) Lactational mother d) Progesterone alone
d) Diabetic mother 427. Which ofthe following statements about calendar
416. Return to fertility is delayed after cessation of oral method (Rhythm method) is false- (AI 08)
contraception by- (UPSC 07) a) Abstinence is needed for only a few days in a month
a) 2 months b) 3 months b) It is associated with no costs
c) 4months d) 6 months c) Safe period can also be observed using
417. The combined oral contraceptive pills decrease temperature rhythm or mucous method
the risk of the following except- (UPSC 07) d) Ectopic pregnancy is a reported complication of
a) Endometrial cancer b) Ovarian cancer calendar method
c) Breast cancer d) Ectopic pregnancy 428. Least failure in sterilization occurs with- (AI 08)
418. During pomeroy's method offemale sterilization, a) Falope ring b) Bipolar cautery
which portion of tube is ligated- (UPSC 07) c) Unipolar cautery d) Hulka clip
a) Isthmus b)Ampullary 429. Which of the following procedures is associated with
c) Isthmo-ampullary d) Cornual maximum chance ofr~nalization during surgery
419. With references to vasectomy, consider the for reversal of tubal ligation- (AI 08)
following statements- (UPSC 07) a) Isthumo - isthmic anastomosis
1. Non-scalpel vasectomy and conventional b) Isthumo - ampullary anastomosis
vasectomy have complication rates of 0.4% and c) Ampullo- ampullary anastomosis
3% respectively. d) Cornual obstruction

411)d 412)b 413)b,c 414)c 415)c 416)d 417)c 418)c 419)b 420)c 421)c 422)a 423)b 424)d
425)d 426)b 427)a 428)c 429)a
GYNAE & OBS [ 22]

430. The progesterone of choice for emergency 442. Which of the following is a contraindication to
contraception is- (AIIMS May 08) laparoscopic sterelization- (AI 09)
a) Norethisterone b) Medroxyprogesteronacetate a) Obesity b) Hiatus hernia
c) Levonorgestrel d) Desogestrel c) Cardiacfailure d)Anemia
431. Decreased ovulation/impaired ovulation caused by- 443. Contraindication for IUCD insertion includes-
a) Minipill (Progesterone only pill) (PGI June 08) a) Pelvic infammatory disease (PGI June 09)
b) IUCD b) Previous ectopic pregnancy
c) Mifepristone
c) IDV
d) OCP
d) Undiagnosed vaginal bleeding
e) Norplant
432. Emergency contraception- (PGI June 08) e) Liver disease
a) Combination of estrogen & progesterone 444. Method of sterilization least suited for
b) Mifepristone Recanalisation is- (DELHI PG Feb. 09)
c) Levonorgestrol a) Clips b) Bipolar cauterization
d) Misoprostal c) Fallopian rings d) Pomeroy's techniques
e) IUCD 445. Contraindications for the use of intrauterine
433. Contreceptive methods with failure rate< 5- contraceptive device include all except-
a) Coppt- T b) Vginal sponge (PGI June 08) a) Pelvic inflammatory disease (DELHI PG Mar. 09)
c) Condom d) OCP b) Thromboembolic disease
e) Tubectomy c) Pelvic tuberculosis
434. Contraindications ofiUCD- (PGI June 08) d) Ovarian cancer
a) Undiagnosed vaginal bleeding 446. The failure rate of combined oral contraceptive
b)PID (Estrogen+Progesterone) - (DELHI PG Mar. 09)
c) Smoking a) 1-2% b) 5-6%
d) Obesity
c) 10-12% d) 16-18%
e) Diabetes
44 7. Post-coital contraception is achieved by all EXCEPT:
435. Which intrauterine device has the longest life span?
a) Progestasert b) Mirena (DPGEE 08) a) High degree of progesterone (UPSC-II 09)
c) CuT200B d) Norplant b) By administration ofRU 486
436. Most common employed sterisation technique? c) GnRH agonist
a) Pomeroys b) Madlener (APPG 08) d) Administration of prostaglandins
c) Irving d) Uchida 448. Which ofthe following is NOT related to the use of
437. Pill given in lactation? (APPG 08) levonorgestrel releasing intrauterine contraceptive
a) Minipill b) Combined pill device? (UPSC-II 09)
c) Both d) None a) Reduction of blood loss
438. Mechanism of action of mini pill? (APPG 08) b) Reduction of pain and dysmenorrhoea in
a) Rendering cervical mucus thick endometriosis and adenomyosis
b) Inhibiting ovulation c) Inhibition of ovulation
c) Both d) Amenorrhoea in 50% of cases
d) None 449. What is the drug of chocie for emergency
439. Most reversible method of sterilisation ? contraception? (UPSC-II 09)
a) Mini lap (APPG 08)
a) Yuzpe regimen (combined oral pill)
b) Pomeroys method
b) High dose oestrogen alone
c) Laparoscopic sterilisation
d) Hysteroscopic sterilisation c) Levonorgestrel only pill d) Danazol
440. All of the following statements aboutDMPAare true, 450. Which of the following is NOT an absolute
Except- (AI 09) contraindication for insertion of intrauterine device
a) Failure rate is 0.3 per HWY (IUD)? (UPSC-II 09)
b) Reduces Menorrhagia a) Endometrial cancer b) Severe dysmenorrhea
c) Can be used in seizures c) Pelvic tuberculosis d) Puerperal sepsis
d) No protective effect against CA endometrium 451. TrueaboutuseofLNGIUCD- (PGINov09)
441. The most effective contraceptive method a) Endometriosis
recommended in lactating mothers is- (AI 09) b) Endometrial hyperplasia
a) Barriermethod c) Used in contraception
b) Progesterone only pill d) Endometrial carcinoma
c) Oral contraceptive pills e) Uterine fibroid
d) Lactational amenorrhea

430)c 43l)a,b,c 432)a,b,c,e 433)a,d,e 434)a,b,e 435)b 436)a 437)a 438)a 439)c 440)d 44l)b 442)c
443)a,b,c,d 444)b 445)b 446)a 447)a 448)c 449)c 450)b 45l)All
GYNAE & OBS [ 23]

452. Side effect of combined oral contraceptive includes- 463. The composition of Mala-N is- (Corned 1 0)
a) Liver disorder b) Acne (PGI Nov 09) a) Norgestrel 0.30 mg and Ethinyl Oestradio130 g
c) Cause Anaemia d) Weight gain b) Norgestrel3.0 mg and Ethinyl Oestradiol3 .0 mg
e) Increase risk ofPID c) Norgestrel 0.30 mg and Ethinyl Oestradiol30 g
453. Emergency contraceptive(s) is/are- (PGI Nov 09) d) Norgestrel30 mg and Ethinyl Oestradiol30 mg
a) Mifepristone 464. Not a method of emergency contraception-
b) PGF2a a) Mifepristone b) Danazole (PG!Nov. 10)
c) Copper containing IUD c) RJCD d) Levonorgestrel
d) Combined estrogen & progesterone e) DMPA
e) Progesterone only pills
465. OCPs should be not given in- (PGI Nov. 1 0)
454. Progesterone of choice in emergency contraception
a) Liver disease
is- (AIIMSNov09)
b) Thrombo-embolic disease
a) Norethisterone b} Medroxy progesterone
c) Dysmenorrhoea
c) Desogestrel d) Levonorgestrel
455. Drug which interferes or hampers the effectiveness d) Cyclic menorrhagia
ofOCP are all except- (AIIMS Nov 09) e) Breastcarcinoma
a) Aspirin b) Tetracycline 466. Not used in emergency contraceptions-
c) Rifampicin d) Phenytoin a) LNG intra uterine device (AIIMS Nov 1 0)
456. Ideal contraceptive for lactating mothers is- b) Oral mifepristone
a) Barrier method (AIIMS May 10) c) Orallevonorgesterol
b) Combined OCP d) CuT intrauterine device
c) Lactational amenorrhoea 467. A 30 yrs woman 2+0, hypertension have
d) Progesterone only pill menorrhagia. Which is best treatment for her-
457. A primipara with a cardiac lesion (Ml) bas come on a) Combined pills (AIIMS May 11)
the 40th day of delivery asking offor contraception. b)MIRENA
The contraceptive choice is- (DPG 10) c) Hysterectomy
a) Condom with spermicidal jelly d) Transcervical resection of endometrium
b) Oral contraceptive pill 468. Ideal contraceptive for lactating mothers is-
c) Intrauterine contraceptive device a) Barrier method (AIIMS May 11)
d) Laparoscopic sterilization b) Combined OCP
458. Yuzpe method is a type of- (Maharashtra 1 0) c) Lactational amenorrhoea
a)
Emergency post-coital hormonal contraception d) Progesterone only pill
Emergency post-coital TIJCD contraception
b) 469. Ideal contraceptive for newly married couple is -
c)
Male contraception a) Barrier method (AIIMS May 11)
d)
Reversibleminilap
b) Combined OCP
459. Worldwide, which is the most commonly used copper-
c) RJCD
bearing intrauterine contraceptive device?
d) Progesterone only pill
a) CopperT-200 b)CopperT-3 80 (UPSC/110)
c) GyneFix d) Copper-7 470. Ideal contraceptive for a couple living in different
460. The following are the contraindications to the use of cities meeting only occasionally- (AIIMS May 11)
combined oral contraceptive pills, except- a) Barriermethod b)IUCD
a) Breastfeeding (UP SCI! 10) c) OCP d)DMPA
b) History of deep venous thrombosis 471. Mifepristone may be used for all oftbe following
c) Active viral hepatitis Except- (AJJJ)
d) Bronchial asthma a) Threatened Abortion b) Ectopic pregnancy
461. Mini pill should be started on the- (UP SC Ill 0) c) Fibroids d) Molar pregnancy
a) First day of the cycle 4 72. Which of the following progesterones is preferred
b) Second day of the cycle in combination with estrogen in Low dose Oral
c) Third day of the cycle contraceptive pills- (Alll)
d) Fifth day of the cycle a) Desogesterol b) Norethisterone
462. What is the sequance of events in termination of c) Norgesterel d) Levonorgestrel
pregnancybymedicalmetbod? (UPSCII 10) 473. Which of the following methods of contraception
a) Mifepristone- Bleeding- Misoprostol- USG should be avaoided in women with epilespy- (AI 11)
b) Misoprostol- Mifepristol- USG- Bleeding a) Oral contraceptive pills b) TIJCD
c) Mifepristone- Misoprostol- Bleeding- USG c) Condoms d) Diaphragm
d) Mifepristone- Misoprostol- USG- Bleeding

452) a,b,d 453) a,c,d,e 454) d 455) a 456)c 457)a 458)a 459)b 460)d 46l)a 462)c 463)a 464)e
465) a,b,e 466) a 467) b 468) c 469) b 470)a 47l)a 472)a 473)a
GYNAE & OBS [ 24]

474. A 37 year old multiparous woman seeks advice 486. In precoccios females, mensturation occurs -
regarding contraception. She is suffering with a) After 13 years of age (TN91}
menorrhagia for last one year. Hemoglobin is 8.5 b) At 13 years of age
gm/dl. The contraception of choice for her will be- c) Less than 10 years of age
a) CopperT380A b)Mirena (Karn 11) d) Below 12 years of age
c) Tubectomy d) None of the above 487. Which is not a feature of Dysfunctional uterine
475. The containdications for Progestasert include the bleeding- (JIPMER 92)
following except- (UPSC II 11) a) Tender uterus b)Uterinesizeof8-10weeks
a) Pelvic Inflammatory Disease c) Menorrhagia d) Proliferative endometrium
b) Uterine fibroids 488. Spasmodic dysmenorrhoea is seen in -(AIIMS 92)
c) Previous history of ectopic pregnancy a) DUB b) Ovarian Cyst
d) Previous history of abortion c) Submucous fibroid d) Endometriosis
476. Which of the following is a third generation 489. Precocious puberty is those who menstruate-
intrauterine device- (UPSC II 11) a) 14-16years b) 10-12years (Karn96)
a) Cu-7 b) TCu-200 c) 12 years d) Before 10 years
c) TCu-380A d) Progestasert 490. Treatment ofDUB in a young female is- (PGI 95)
477. Hormonal contraceptives are contraindicated in ~Hormones b)RT
women- (UPSC llll) c) D&C d) Hysterectomy
a) Less than 25 years of age 491. Treatment of dysfunctional uterine bleeding is
b) Who are normotensive except- (Kerala 96)
c) Who have thromboembolic disorders a) Oralcontraceptivepills b) Oestrogen
d) Who have anaemia c) Progesterone d) Ovariotomy
478. Complications of vasectomy includes all except- e) Danazol
a) Sperm granuloma (Jipmer 11)
492. Progressive secondary Ammenorrhoea is
associated with- (CUPGEE 96)
b) Infection
a) Endometriosis b) Psychosomatic cause
c) Decreased libido
c) Uterine anomally d) None
d) Anti sperm antibody
493. First investigation to be done for post menopausal
479. Best contraception for lactating mother is-
bleeding- (AP 96)
a) Monophasic pill b) Biphasicpill (Jipmer 11)
a) Endometrial biopsy b) Dilatation & Currettage
c) Mini pill d) Combined pill
c) Fractional currettage d) Hysteroslpingography
480. Best IDCD for a woman with menorrhagia-
494. Metropathic hemorrhagica is best treated
a) Lippe's loop b) Cu-T 375 (Jipmer 11)
by- (TN97)
c) Cu-T 200 d) Levonorgestrel IUCD a) Curettage of uterus b) Progestogen
481. OCP's are usefulin the prevention of- (Jipmer 11) c) Ostrogen d) Clomiphene
a) Ca. cervix b) Ca. endometrium 495. In a woman presenting with amenorrhoea headeche
c) Ca. Colon d) Ca. Breast blured vision & galactorrhoea appropriate
482. Anti-progestogen used as emergency contraceptive investigation - (AI 97)
is- (Jipmer 11) a) Prolactin levels b) LH
a) Mifepristone b) Desogestrel c) FSH d)HCG
c) Nor ethisterone d) Cerozette 496. Commonest type of Dysfunctional uterine
bleeding- (PGI 97)
DISORDERS IN MENSTRUATION a) Secretory b) Proliferative
c) Hyperplastic d) Mixed
483. Physiological amenorrhoea is present in all 497. Bony deformities, hyperpigmentation of the skin,
except- (KARNATAKA 90) precocious puberty is seen in- (AP 97)
a) Pregnancy b) Lactation a) Alport's syndrome
c) Pre- puberty d) Post- puberty b) McCune Albright's syndrome
484. Asherman's syndrome is characterised by-(TN90) c) Laurence-Moon-Biedl syndrome
a) Amenorrhoea b) Dysmenorrhoea d) Frohlich's syndrome
c) Leukorrhoea d) Metrorragia 498. A 45-years old woman with diagnosis of atypical
485. Primary amenorrhoea is when- (TN 91) adenomatous hyperplasia of the endometrium
a) Mensturation does not occur even after 18 year should be treated by- (UPSC)
b) Mensturation does not occur even after 15 year a) Progesterone
of age b) Danazol
c) Imperforate hymen exists c) Hysteroscopic resection of endometrium
d) None of the above d) Hysterectomy

474)b 475)d 476)d 477)c 478)c 479)c 480)d 481)b 482)a 483)d 484)a 485)b 486)c 487)a
488)c 489)d 490)a 491)d 492)b 493)c 494)b 495)a 496)c 497)b 498)d
GYNAE & OBS [ 25]

499. DUB isdueto- (MAHE98) 509. A 42year old female presents presents with history
a) Endometriosis of polymenorrhea for the last 6 month. She is
b) Fibroid nulliparous.What is the next step in management-
c) Endometrial carcinoma a) Hysterectomy (AJJMS 2K)
d) Irregular shedding of endometrium b) Dilatation and curettage
500. Which one of the following is the primary c) Oral contraceptive pills
treatment for puberty menorrhagia in a 16 years d) Progesterone only pills
old girl with 3 gm% Hb- (UPSC 99) 510. Kamala, 42 year old multiparous lady complains
a) Dilatation and curettage with blood transfusion of polymenorrhea since a few months. Which of
b) Danazol with blood transfusion the following will be the first line of management
c) Progestogen with blood transfusion forher- (Al2002)
d) A combination of oestrogen and progestogen a) Hysterctomy b) Dilatation & Curettage
with blood transfusion c) TAHwithBSO d) Progesterones
501. Which of the following is an example of 511. The progestogen with the greatest haemostatic effect
dysfunnctional uterine bleeding- (TN 99) used in dysfunctional uterine bleeding (DUB) is-
a) Hypothyroidism b)PID a) Medroxy progesterone (UPSC 2001)
c) Cervical erosion d) Carcinoma cervix b) Hydroxy progesterone
502. A 36 old woman has noticed the absence of menses c) Norethisterone
for the last 4 months. A pregnancy test is negative. d) Dydrogesterone
Serum levels of luteinizing hormone and follicle 512. Treatment of puberty menorrhagia- (PGJ 2002)
stimulating hormone are elevated and the serum a) Estrogen b) Estrogen+ progesterone
estradiol levels is low. These findings suggest- c) GNRH analogues d) Dilation & curettage
a) Bilateral tubal obstruction (AJIMS 99) e) Antiprostaglandins
b) Polycystic ovarian disease 513. Primary amenorhoea ia a feature of all except -
c) Premature menopause a) Turner's syndrome (A/97)
d) Exogenous estrogen administration b) Stein-leventhal syndrome
503. Cause of unilateral dysmenorrhea- (ORRISA 99) c) Rokitansk:y-Kutsner-Hauser syndrome
a) One hom of malformed uterus d) Kallman syndrome
b) Endometrisis with unilateral distribution 514. A 50 year old woman nulliparous, diabetic and obese
c) Small fibroid at the utero tubal junction presenting with post-menopausal bleeding can be
d) All of the above any ofthefollowingexcept- (PGJ 99)
504. True about Rokitansky Kuster Hauser syndrome a) Carcinoma in situ of cervix
are all except - (AI 99) b) Carcinoma endometrium
a) No Uterus b) No ovulation c) DUB
c) No vagina d)46XX d) None of the above
505. All are causes of posmenopausal bleeding except- 515. The most common histological finding of
a) Carcinoma in situ of cervix _(PGJ 2000) endometriuminDUBis- (PGI 1999)
b) Ca. endometrium a) Hypertrophic b) Hyperplastic
c) Ca. ovary c) Cystic glandular hyperplasia d) Dysplastic
d) Ca. Fallopian tube 516. "Swiss-cheesett endometrium is seen in-
506. Which of the following is not a feature of a) Simple endometrial hyperplasia (PGI 2000)
Mullerian dysgenesis - (AJJMS 2K) b) Cystic glandular hyperplasia
a) 46 XX karyotype b) Absent vagina c) Proliferative endometrium
c) Absent uterus d) Ovarian agenesis d) Endometriosis
507. Post menopausal woman who is overweight, 517. Treatment for 32 yr. old multipara with dysfunction
hypertensive and diabetic presents with bleeding uterine bleeding (DUB) is- (PGI 2000)
PV. The most likely clinical diagnosis is - a) Progestrogens b) Danazol
a) Endocervical carcinoma (UPSC 2K) c) Prostaglandins d) Endometrial ablation
b) Dysfunctional uterine bleeding e) Hysterectomy
c) Carcinoma endometrium 518. LH secretion is decreased by- (PGI 03)
d) Senile endometritis a) Bromocriptine b) Hyperthyroidism
508. The most useful investigation in this above patient .., ) Prolactinoma d) Phenothiazine therapy
would be- (UPSC 2K) e) HCXi
a) Transvaginal ultrasonography 519. Causes of secondary amenorrhea are- (PGJ OJ)
b) Endometrial sampling a) Turner's synd b) Endometriosis
c) Doppler ultrasound of pelvis c) Asherman's synd d) Thyroiditis
d) CT scan of pelvis e) Epilepsy

499)d 500)d 50l)None 502)c 503)d 504)b 505)a 506)d 507)c 508)b 509)b 510)b 511) a,c 512) a,b
513)b 514)a 515)b 516)b 517)a 518)e 519)c,d,e
GYNAE & OBS [ 26]

520. Puberty menorrhagia is treated by- (PGI 02) 530. In a case of secondary amenorrhoea who fails to
a) Progesterone get withdrawal bleeding after taking estrogen and
b) Progesterone and estrogen progesterone, the fault lies at the level of-
c) GnRH analogues a) Pituitary b) Hypothalamus (Karnat 05)
d) Danazol c) Ovary d) Endometrium
e) Surgery 531. Average blood loss in normal menstruation-
521. Positive progesterone challenge test in a patient a) 50ml b)80ml (PGIJune05,
of secondary amenorrhoea, seen in- (PGI 04)
c) lOOml d) 120ml JIPMER86)
a) Asherman Syndrome
e) lOml
b) Endometrial TB
c) Hypopituitarism 532. Initial evaluation in adolescent abnormal uterine
d) Premature ovarian failure bleeding- (PGI June 05)
e) PCOD a) Hemogram b) Platelet Count
522. Withdrawal bleeding with progesterone seen in c) USG d)D&C
otherwise amenorrhoeic woman due to- (PGI 97) e) Examination under anesthesia
a) Hypogonadotrophic hypogonadism 533. Causes of secondary dysmenorrhoea in young
b) Anovulation female- (PGI June 05)
c) Ovarian failure a) T.B. b) Adenomyosis
d) TB. endometritis c) CIN d) Endometriosis
523. Cause of dysfunctional uterine bleeding can be- e) Subserous fibroid
a) Uterinepolyp (PGIOJ) 534. The most common cause of secondary
b) Fibroid amenorrhoeainlndiais- (AI 05)
c) Granulosa cell tumour a) Endometrial tuberculosis
d) Irregular ripening or endometium b) Premature ovarian failure
e) Irregular shedding of endometrium c) Polycystic ovarian syndrome
524. A patient with amenorrhea had bleeding after d) Sheehan's syndrome
giving a trial of progesterone. This implies- 535. The risk of Asherman syndrome is the highest if
a) Sufficient estrogen (PGIOJ) dilatation and curettage (D&C) is done for the
b) Sufficient progesterone following condition- (AIIMS 06)
c) Normal ovarian function a) Medical termination of pregnancy
d) Intact endometrium b) Missed abortion ,
e) Intact pituitary axis c) Dysfunctional uterine bleeding
525. Increased LH: FSH ratio is found in- (UPSC 02) d) Postpartum hemorrhage
a) Premature menopause 536. A 13-year-old girl presents in the casualty with
b) Sheehan syndrome acute pain in the lower abdomen. She has history
c) Polycystic ovary syndrome of cyclical pain for last 6 months and she has not
d) Turner's syndrome attained her menarche yet. On local genital
526. In Testicular Feminisation Syndrome (Anrogen
examination, a tense bulge in the region of hymen
Insensitivity Syndrome) Gonadectomy is indicated-
was seen. The most probable diagnosis is-
a) As soon as it is diagnosed (UPSC 04)
a) Rockytansky Kuster syndrome (AIIMS 06)
b) At puberty
b) Testicular feminization syndrome
c) Only when malignancy develops in it
c) Imperforate hymen
d) When hirsutism is evident
d) Ashermna's syndrome
527. Failure of bleeding after withdrawal of oestrogen
537. A 30-year old housewife reports with 6 months
indicates- (SGPGI 04)
a) Uterine factor b) Ovarian factors amenorrhoea. Her serum LH and FSH are high
c) Pituitary factor d) Hypothalamic factor with low estradiol levels. What is the most likely
528. Halban's disease is due to- (JIPMER 05)
cause of amenorrhoea- (UPSC 06)
a) Persistent corpus luteum a) Pituitary tumor
b) Deficient corpus luteum b) Polycystic ovarian disease
c) Persistent trophoblast c) Exercise induced
d) Deficient trophoblast d) Premature menopause
529. A female presents with primary amenorrhea and 538. In cystic glandular hyperplasia what is seen-
absent vagina, the next investigation to be done a) Normal menstrual bleeding (Manipal 06)
is- (SGPGI05) b) Amenorrhoea and bleeding
a) LHIFSH·assay b) Chromosomal analysis c) Hypomenorrhoea
c) Urianalysis d) Laproscopy d) None of the above

520)a,b 52l)e 522)b 523)d,e 524)a,c,d,e 525)c 526)b 527)a 528)a 529)b 530)d 53l)a 532)a,b,c
533)b,d,e 534)a 535)d 536)c 537)d 538)b
GYNAE & OBS [ 27]

539. Most common cause for postmenopausal bleeding 548. Which ofthe following treatments for menorrhagia
in Indian woman is- (AI 07) is not supported by evidence - (AI 09)
a) Ca cervix b) Ca endometrium a) Combined oral contraceptive pills
c) DUB d) Ca ovary b) Progesterone
540. HRT is useful in all except- (AIIMS May 07) c) Tranexamic acid
a) Flushing b) Osteoporosis d) Ethamsylate
c) Vaginal atrophy d) Coronary heart disease 549. A 45 year old post menopausal woman presents with
541. Most common cause of postmenopausal bleeding vaginal bleeding. Transvaginal ultrasound reveals
in women in.India is- (AIIMS May 07) an endometrial thickness of8.0 mm. The next step
a) Cancer cervix b) Cancer endometrium in the management should be- (AI 09)
a) Hysterectomy
c) HRT d) Senile endometritis
b) Start progesterone therapy
542. Premature menopause occurs before the age of-
c) Histopathological examination of curettage
a) 40 years b) 35 years (MAHE 07)
d) Follow up sonography
c) 45 years d) 50 years 550. Which ofthe following statement is/are true-
543. Child with primary amenorrhoea with negative a) Adult female serum FSH level is 5-2.0 mU/ml
progesterone challenge test but positive combined b) Normal plasma testosterone level in adult female is
progesterone & estrogen test. Diagnosis may be - 20-80nglml (PGI June 09)
a) Mullerian agenesis b) PCOD (PGI June 07) c) Oligomenorrhoea means episodes of menstrauation
c) Asherman syndrome · d) Prolactinoma at intervals of more than 35 days
544. WhiCh of the following drugs are effective in the d) Menorrhagia means cyclic bleeding >50ml
treatment ofdysfunctional uterine bleeding when e) PCOD clinical features includes anovulation,
the endometrial biopsy shows a secretory pattern ? obesity & infertility
1. Medroxyprogesterone acetate 551. A 55 year old woman has single episode of
2. Combined preparations of progestogen and postmenopausal bleeding. Management of this patient
oestrogen includes- (PGI June 09)
3. Prostaglandin synthetase inhibitors a) Papsmear
Select the correct answer using the code given below- b) Endometrial biopsy
a) 1, 2 find 3 b) 1 and 2 only c) Wait & watch
c) 2 and 3 only d) 1 and 3 only d) Enquire about recent hormonal intake
545. Premature menopause occurs before the age of- 552. A 35 years old woman with dysmenorrheal and
a) 40 years b) 35 years (Manipal 08) menorrhagia of 6 months duration showed an
c) 45 years d) 50 years enlarged uterus of 20 weeks which was tender. The
546. Which of the following statement (s) is/are true- possible diagnosis is- (DELHI PG Feb. 09)
a) Adenomyosis b) Firboid
a) Primary amenorrhoea is failure of onset of
c) Carcinoma endometrium d) PID
menstruation beyond age of 14 (PGI Dec 08)
553. Match listl (Condition/endometrial pathology) with
b) Secondary amenorrhoea refer to failure of
list II (Choice of treatment) and select the correct
occurence of menstruation for 12 months who answer using the codes given below the
have previously menstruated lists - (UPSC-II 09)
c) Polymenorrhoea means episodes of menstruation List I (Condition/ List II (Choice of
at interval ofless than 28 days endometrial pathology treatment
d) Oligomenorrhoea means episodes of menstruation A. Simple hyperplasia 1. Reassurance
at intervals of more than 35 days B. Ovulatory menorrhagia 2. Progestins for 6 months
e) Primary amenorrhoea is failure of onset of C. Puberty menorrhagia 3. Oral contraceptive pills
menstruation beyond age of 15 years in presence D. Irregular shedding 4. Hematinics
of secondary sexual development Codes:
547. A 45 yr old lady presented with recurrent uterine A B C D
bleeding. On transvaginal USG thickness of a) 2 3 4 1
endometrium was found to be 8 mm. What should be b) 2 4 3 1
the next step in the management ofthis patient ? c) 1 3 4 2
a) Histopathology (AIIMS Nov 08) d) 1 4 3 2
b) Hysterectomy 554. Which of the following treatments for menorrhagia
c) Progesterone is not supported by evidence? (AIIMS May I 0)
d) OCP a) Tranexamic acid b) Ethamsylate
c) CombinedOCP d) Progesterone

539)a 540)d 54l)a 542)a 543)d 544) c · 545) a 546) d,e 547) a 548)d 549)c 550)a,c,e 55l)All
552)a 553)c 554)b
GYNAE & OBS [ 28]

555. A lady presented with secondary amenorrhea 6 564. All of the following agents are used in the treatment
months after having an abortion. Her FSH levels ofhotflashes,except- (AI 11)
were mesured as 6 miU/ml what is the most probable a) Tamoxifen b) Venlafaxine
diagnosis~ (AI IO) c) Gabapentin d) paroxetine
a) Pituitary failure b) Ovarian failure 565. Precocious puberty is diagnosed when- (Karn I I)
c) Fresh pregnancy d) Uterine synechiae a) Secondary sexual characteristics appear before 8
556. Metropathia hemorrhagica is best treated years, menstruation occurs before 10 years
by- (DPG IO) b) Secondary sexual characteristics appear before
a) Curettage of uterus b) Progestogen 1Oyears,menstruation occurs before 8 years
c) Ostrogen d) Clomiphene c) Secondary sexual characteristics appear before 6
557. Hot flushes correspond to surge of-(Maharashtra I 0) years, menstruation occurs before Syears
a) FSH b)LH d) Secondary sexual characteristics appear before 8
c) GnRH d) Estrogen years, menstruation before 6 years
558. A 19 year old patient came to the out patient 566. A 40-year-old woman presents with excessive
deparmentwith complaints ofPrimary Amenorrhea. menstrual bleeding. The most appropriate f'wst
She had well developed breast and pubic hair. surgical treatment will be- (UPSC II 11)
However there was absence of vagina and uterus. a) Hysteroscopy b) Myomectomy
Likely diagnosis is- (AIIMS Nov I 0) c) Dilatation and curettage d) Hysterectomy
a)XYY 567. A 15-year-old unmarried girl presents with
b) Mullerian agenesis complaints of dysmenorrhoea for the last about one
c) Gonadal dysgenesis year. She achieved menarche at 12 years of age. On
d) Kleinfelter's syndrome abdominal and rectal examination, she has no
559. A 20 year average weight old female complains of abnormality. What will be the most appropriate
ligomenorrhea along with facial hair. Preliminary management- (UPSC II 11)
investigation reveal raised free testosterone levels. a) Prescribe antibiotics
b) Prescribe clotrimazole vaginal ovules
Which of the following could be likely etiology-
c) Reassure her and prescribe analgesics
a) Idiopathic hursutism (AIIMS Nov IO)
d) Perform dilatation and curettage
b) PCOD
568. A 30-year-old woman with three children has
c) Adrenal hyperplasia
dysfunctional uterine bleeding. What will be the most
d) Testosterone secreting tumour
appropriate management- · (UPSC II II)
560. Which of the following is not a cause of primary
a) Abdominal hysterectomy
amenorrhea? (AI I I)
b) Medical management with danazol
a) Sheehan's syndrome c) Transcervical endometrial resection
b) Kallman's syndrome d} Levonorgestrel-releasing intrauterine device
c) Turner's syndrome 569. A woman who has secondary amenorrhea
d) Rokitansky syndrome experiences withdrawal bleeding following
561. A female with 45XO genotype has primary progesterone administration. What is the likely
amenorrhoea. Most likely diagnosis is- (AI 11) diagnosis- (UPSC II 11)
a) Gonadal Dysgenesis a) Anovulation
b) Androgen Insensitivity syndrome b) Premature ovarian failure '
c) Meyer Rokitansky Kuster Houser Syndrome c) Hypothalamic amenorrhea
d) Congenital adrenal hyperplasia d) Asherman's syndrome
562. Which ofthe following treatments for menorrhagia 570. A 60-year-old woman presents with postmenopausal
is not supported by evidence- (AI 11) bleeding per speculum examination reveal no
a) Combined oral contraceptive pills abnormality, and the Pap smear is normal. What
b) Progesterone will be the most appropriate management-
c) Tranexamicacid a) Keep her under observation (UPSC II 11)
d) Ethamsylate b) Administer haemostatics
563. A 45 year old post menopausal woman presents with c) Measure endometrial thickness with ultrasound
vaginal bleedng Transvaginal ultrasound reveals an d) Hysterectomy
endometrial thickness of8.0 mm. The next step in 571. Pain during menstruatio.n starting few days before
the management should be- (AI 11) menstruation and ends with menstruation is-
a) Hysterectomy a) Spasmodic dysmenorrhoea (Jipmer 11)
b) Start progesterone therapy b) Congestive dysmenorrhoea
c) Histopathological examination of curettage c) Membranous dysmenorrhoea
d) Follow up sonogaphy d) Menorrhagia

555)d 556)b 557)a 558)b 559)c 560)a 56l)a 562)d 563)c 564)a 565)a 566)c 567)c 568)d
569)a 570)c 571)b
GYNAE & OBS [ 29]

CHORIOCARCINOMA 584. Bad prognostic markers of Chorio-carcinoma


treatment are- (PGI 04)
572. Gestational Trophoblastic tumours occur most a) Liver metastasis b) Lung metastasis
commonly after- (AIIMS 91) c) Previous H. mole d) High H.C.G titre
a) Cesearean section b) Spontaneousabortion e) Chemotherapy started 12 months after pregnancy
c) Pre term delivery d) Full term delivery 585. Villous pattern is lostin- (TN 03)
573. Chorion epithelioma mostly follow- (JIPMER 80,
a) Invasive mole b) Tubal mole
a) Abortion b)Nonnallabour UPSC90)
c) Hydatidiform mole d) Choriocarcinoma
c) EctopiC pregnancy d) Hydatidiform mole
e) None of the above 586. Which of the following endocrinological
574. Choriocarcinoma commonly metastasises to- condition may be associated with hydatiform
a) Liver b) Brain (AIIMS 84, 94) mole? (DELHI PG Feb. 09)
c) Lungs d) Placents a) Hypothyroidism b) Hyperthyroidism
575. A young patient is diagnosed to have choriocarcinoma c) Diabetes d) Hyperprolactinernia
the treatment of choice is- (MP 98) 587. The treatment of choice for hydatiform mole with a
a) Hysterectomy uterine size of 28 weeks is - (DELHI PG Mar. 09)
b) Chemotherapy a) Suction evacuation
c) Chemotherapy followed by hysterectomy b) Intra-amniotic saline followed by oxytocin
d) Hysterectomy followed by chemotherapy c) Evacuation by rnisoprostol and Mifepristone
576. Thmour marker for chorio carcinoma-(TN87,AI93 d) Methotrexate administration
a) HCG b)AFP MAHE98) 588. The most common karyotype of a partial mole is-
c) CEA d) Desmin a) Diploid46XX b)Diploid46XY (UPSC-II09)
577. A 35 year old lady P2+0+0+0 is diagnosed to have c) Triploid d) Haploid
choriocarcinoma. The treatment. of choice 589. Trueaboutcompletemole- (PGINov.JO)
is - . . (AIIMS 2K) a) Presence of foetal parts and cardiac activity
a) Hysterectomy b) Dilation and curettage b) Normal unterine size
c) Chemotherapy d) Radiation c) Beta-hCG doubling time is 7-10 days
578. A patient with choriocarcinoma,,is not reponding d) Pre-eclampsia at< 24 weeks
to the usual doses of methotrexate. She is having e) PV bleeding is commonest presentation
jaundice. Which drug will you prefer for further 590. Snowstorm appearance on USGseenin-(PGINov.JO)
treatment- (AIIMS 2K) a) Hydatiform mole b) Invasive mole
a) Actinomycin D b) Cyclophosphamide c) Twin pregnancy d) Ectopic pregnancy
c) Cisplatin d) Chlorambucil e) Choriocarcinoma
579. The ideal treatment for metastatic choriocarcinoma
in the lungs in a young women is- (PGI 99) PROLAPSE
a) Chemotherapy b) Surgery with radiation
c) Surgery d) Wait and watch 591. Urinary incontinence in utero vaginal prolapse is
580. True about conplete hydatidiform mole is -(PGI OJ) mostlyisdueto- (AIIMS91)
a) Chromosome pattern is XX a) Detrusor instability b) Stress incontience
b) It is.ofmatemal origin c) Urge incontinence d) True incontinence
c) Enlarged ovarian cyst occurs 592. Not a complication of prolapsed uterus:- · (PGI 82
d) It is common in developed countries a) Carcinoma cervix UPSC 87)
e) Associated with pre-eclampsia b) Elongation of cervix
581. Snow-storm pattern seen in- (PGI 03)
c) Cystocele
a) H. Mole b) Ectopic pregnancy d) Decubitus ulcer
c) Fibroid d) Intrauterine death
593. A 25-years old nulliparous woman with third degree
582. TrueaboutH.Mole- (PG/03)
uterine descent but no cystocele or rectocele or
a) Complete mole seen in human only
rectocele or enetrocele is best treated with -
b) Trophoblastic proliferation
a) Fothergill'srepair (UPSC)
c) Hydropic degeneration
b) Abdominal sling operation
d) Villus pattern absent
c) Amputation of the cervix and reconstruction
583. CompleteH.Moleare- (PG/03)
a) Triploid d) Lefort's operation
b) Diploid 594. The best way to treat decubitus ulcer in a case of
c) Increased ~-HCG genitalprolapseisby- (KARNAT99)
d) 2% cases may convert to carcinoma a) Bed rest b) Antibiotics
e) Chance ofmalignant conversion less than partial mole c) Antiseptic dressing d) Reduction with tampoon

572)b 573)d 574)c 575)b 576)a 577)c 578)a 579)a 580) a,c,e 581) a 582) b,c 583) b,c,d 584) a,b,d,e
585)d 586)b 587)a 588)c 589) d,e 590) a 591)b 592)a 593)b 594)d
GYNAE & OBS [ 30]

595. A28 year old women, para3, with 2nd degee prolapse, Select the correct answer using the code given below-
wbich treatment should be done- (AJIMS 97) a) 1, 2 and 3 b) 1 and 2 only
a) Fothergill's operation c) 2 and 3 only d) 1 and 3 only
b) Wertheims 606. Which one of the following is not an operation for
c) Pelvic floor exercise uterine inversion? (UPSC-11 08)
d) Vaginal hystrectomy with pelvic repair a) '0' Sullivan's b) Haultain's
596. A young nulliparous woman has 3rd degree of c) Spinelli's d) Fentoni's
uterovaginal prolapse without any cystocele or
607. Among the four steps in the Manchester operation
rectocele. There is no stress incontinence. The
done for prolapse, which one of the following is the
uterus is retroverted. Uterocervical lenth is 3
inches. All other symptoms are normal. The best correct order in whicb they are done? (UPSC-11 08)
treatment plan for her will be- (UPSC 2K) a) Amputation of cervix, D & C, Fothergill suture,
a) Observation and reassurance till child bearing is over haemostatis
b) Shirodkar's vaginal repair b) D & C, Amputation of cervix, Fothergill suture,
c) Shirodkar's abdominal sling haemostatis
d) Fothergill's opration c) D & C, Fothergill suture, Amputation of cervix,
597. Presence of decubitus ulcer in proplapse haemostatis
indicates - (UPSC 2K) d) D & C, Amputation of cervix, haemostatis,
a) Infection b) Malignancy Fothergill suture
c) Circulatory disturbances d)Machanical trauma 608. Pouch of Douglas with vaginal wall prolapse of
598. The appropriate chocie for treatment of uterus is known as - (UP 08)
Nulliparous prolapse is- (Karn 01) a) Cystocele b) Urethrocele
a) Sling operation b) Manchestar repair c) Enterocele d) Rectocele
c) Ward Mayo,s operation d) Pessry treatment 609. Birth trauma is a risk factor for- (Manipal 08)
599. Indication ofManchester operation in prolapse-
a) Prolapse uterus b) Endometriosis
a) Nulliparous (PG/03)
c) PID d) Abortions
b) Women of< 35 yrs age
c) Patient who wants child bearing function 610. Fothergill's repair is also known as- (DPGEE 08)
d) Congenital elongation of cervix a) LeFort's repair
600. Cause of decubitus ulcer in uterine prolase is- b) Shirodkar's abdominal sling operation
a) Friction b) Venous congestion (PGI 99) c) Manchester operation
c) Intercourse d) Trauma d) Khan's sling operation
601. In Fothergill's operation the following are 611. ligament used to maintain anteversion? (APPG 08)
undertaken except- (TN 03) a) Triangular ligament b) Uterosacral ligament
a) Amputation of cervix c) Broad ligament d) Round ligament
b) Anterior colporrhaphy 612. Which of the follwoing condition is treated by Sacral
c) Colpoperineorrhaphy colpoplexy? (APPG 08)
d) Plication of round ligament a) Stress incontinence b) Rectovaginal fistula
602. In which condition, a rubber-ring passary is an c) Retroverted uterus d) Vault prolapse
appropriate management for a woman who is 613. A young nulliparous woman had 3rd degree of
having utero-vaginal prolapse- (UPSC 06) uterovaginal prolapse without any cystocele or
a) Oldage
rectocele. There is no stress incontinence. The
b) Active reproductive age
uterus is retroverted. Uterocervical length is 3
c) Early pregnancy
d) Prolapse associated with carcinoma of cervix inches. All other symptoms are normal. The best
603. Birth trauma is a risk factor for- (MAHE 07) treatment plan for her will be -(DELHIPG Feb. 09)
a) Prolapse uterus b) Endometriosis a) Observation and reassurance till child bearing is over
c) PID d) Abortions b) Shirodkar's vaginal repair
604. In which one of the following conditions is sling c) Shirodkar's abdominal sling
operationdone- (UPSC07) d) Fothergill's operation
a) Multiple prolapse b) Nulliparous prolapse 614. What is the most common cause of vault prolapse
c) Cystocoele d) Rectocoele following hysterectomy? (UPSC-II 09)
605. Which of the following are the causes of retroversion a) Chronic cough
ofthe uterus? (UPSC-II 08) b) Obesity
1) Pelvic inflammatory diseases c) Diabetes mellitus
2. Prolapse uterus d) Failure to identifY and repair enterocele
3. Endometriosis

595)a 596)c 597)c 598)a 599) b,d 600) b 601)d 602)c 603)a 604)b 605)d 606)d 607)b 608)c
609)a 610)c 6ll)b 612)d 613)c 614)d
GYNAE & OBS [ 31 ]

615. A 25-year old nulliparous woman with third degree 626. Which 9fthe following is the false about fibroid,.
uterine descent but no cystocele or recto-cele or a) Usually malignant (TN 99)
enetrocele is best treated with- (DPG 10) b) Rare before 20 years
a) Fothergill's repair c) Usually asymptomatic
b) Abdominal sling operation d) More common in nulliparous
c) Amputation ofthe cervix and reconstruction 627. Submucosalfibroiddetectedby- (PGI02)
d) Lefort's operation a) Hysteroscopy
616. A woman of cb,ild-bearing age develops a second-
b) Hysterosalpingography
degree uterine prolapse with supravaginal
c) USG, (transabdominal)
elongation of the cervix. What will be the most
appropriate management- (UPSC II 11) d) Laparscopy
a) Amputation of the cervix 628. Utrine fibromyoma is associated with- (PGI 02)
b) Fothergill's operation a) Endometriosis b) Pelvic inflammatory disease
c) Vaginal hysterectomy and pelvic floor repair c) Ovary Ca d) Amenorrhea
d) Sling operation e) Tamoxifen
626. Drugs that reduce the size of fibroid are- (PGI 03)
FIBROMYOMA OF THE UTERUS a) Danazol b)GnRh
c) Ru-486 d) Estrogen
617. Fundulmyomascommonlypresentas- (TN90) e) Progesterone
a) Inversion of uterus b) Dysmenorrhoea 630. Tostartwith,allfibriodsare- (PGI98)
c) Urinaryretension d) Menorrhagia a) Interstital b) Submucus
618. Commonest site for fibroid is- (AIIMS 91) c) Subserous d) Ovarian
a) Submucous b) Intramural
631. Uterine Leimyoma is LEAST likely to
c) Subserous d) Cervical
undergo- (SGPGI05,Al88)
619. The precentage of myomas undergoing malignant
a) Malignant change b) Hyaline change
transformation- (AJIMS 91)
~10% ~5%
c) Calcification d) Red degeneration
c) l% d)0.5% 632. Red degeneration in a case of fibroid with pregnancy
620. Treatment of choice in a perimenopausal woman pccurs most often during- (SGPGI 05)
with bleeding PV due to multiple fibroids is- a) First trimester b) Second trimester
a) TAHwithBSO (AIIMS 92) c) Third trimester d) Puperium
b)TAH 633. Not true about Red degeneration of myomas is-
c) Vaginal hystrectomy a) It occur commonly during pregnancy (MAHA 05)
d) Enucleation of fibfoids b) Immediate surgical intervention is needed
621. Treatment of choice in a 45 year old multipara c) Due to interference with blood supply
(para 3) with a single fibroid in the uterus 12 weeks d) Treated with analgesics
size and asymptomatic- (AIIMS 94) 634. A 30 year old, para two, with two live children has
a) Hysterectomy b)Myomectomy menorrhagia for 2 years. She was ligated 4 years
c) Hysteroscopic removal d) No treatment needed back. On investivation she is found to have a 2 em x
622. Calcareous degeneration occurs most commonly 2 em submucous myoma. What will be the best
in which type offibroids- (PGI 97)
management option for her- (UPSC 06)
a) Submucous b) Subserous
a) Total abdominal hysterectomy
c) Interstitial d) Cervical
623. Retention ofUrineis most likely to be caused by- b) Danazol400 mg twice daily for 3 months
a) Subserous fibromyoma (MP 98) c) Gn RH analogues
b) Interstitial fibromyoma d) Hysteroscopic myoma resection
c) Submucous fibromyoma 635. Fibroid uterus may present with all of the following
d) Posterior cervical fibromyoma except- (AJ07)
624. Red degeneration of uterine fibroid- (MP 98) a) Amenorrhoea b) Pelvic mass
a) Is aseptic infarction c) Infertility d) Polymenorrhea
b) Only occurs in pregnancy 636. All are used to shrink fihroids except -(UP 07, 06)
c) Causes leucopenia with lymphocytosis a) Estrogen b) Danazol
d) Is due to emboli occluding the major blood vessels c) Mifepristone d) GnRH analogue
supplying the myoma 637. All are used to shrink fibroids except -(DPGEE 08)
625. Malignant potential is most in- (AIIMS 98) a) Mifepristone b) Estrogen
a) Subserous fibroid b) Submucous c) Danazol d) GnRH analogues
c) Intramural d) Cervical fibroid

615)b 616)b 617)d 618)b 619)d 620)a,b 62l)a 622)b 623)d 624)a 625)a,c 626)a 627)a,b,c
628)a,b 629)a,b,c,e 630)a 631)a 632)b 633)b 634)c 635)a 636)a 637)b
GYNAE & OBS [ 32]

638. Operative haemorrhage during myomectomy can be 650. The incidence of lymph node metastasis in
minimised by prior administration of -(PGIDec 08) carcinoma cervix stage n is- (KARN 94)
a) GnRHanalogue b)OCP a) 5-100/o b) 15-20%
c) Danazol d)LH c) 25-40% d)50-60%
e) Mifepristone 651. Carcinoma cervix with bilateral hydro ureter due
639. A 26 yr women has 7 x 8 em leiomyoma, management to ureteric obstruction is stage- (KERALA 95)
option include- (PGI June 09) a)l b)II
a) Followup b)OCP c) m d) IV
c) Myomectomy d) Hysterectomy 652. The most common cause of death in carcinoma
640. The risk of sarcoma developing in a fibroid nterns is cervix is- (UPSC 96)
approximately- (DELHI PG Mar. 09)
a) Haemorrhage b) Uraemia
a) <1% b) 10%
c) Infection d) Metastases
c) 300/o d) 50%
653. All offollowing are true about carcinoma of cervix
641. Which of these can be nsed for uterine artery
embolization for fibroid uterus? (UPSC-11 09) except- (DELHI 96)
a) Polyglactin b) Ethacridine lactate a) Commonest genital cancer in females in India
c) Polyvinyl alcohol d) Methylene blue b) Blood spread is not very common
642. Sarcomatous change should be suspected in case of c) No cytological test is available for screening in
uterine fibroid when there occurs-(Maharashtra 10) early phase of disease
a) Increase in the size b)Pain d) Disturbance of normal menstrual pattern may be
c) Bleeding d) All of the above the first manifestation
643. The approximate incidence of sarcomatous changes 654. Carvical intraepithelial neoplasia III with no
in cases offihroid uterus is- (Karn 11) colposcopy activity treatment of choice- {AP 96)
a) 0.5% b)5% a) Hysterecetomy b) Radiotherapy
c) 25% d)50% c) Conisation d) Follow-up after 1 year
655. According to FIGO classification the investigation
CARCINOMA CERVIX in Carcinoma cervix are all except- (AIIMS 96)
. . . a) X-RayChest b)IVP
644. A35 years old ladywith3 children and bad undergone c} Pelvic ultrasound d) Cystoscopy
tubal ligation shows a positive pap smear, next line 656. Treatment ofmicroinvasive Ca cervix in a G3P0L0~
ofmanagementis- (JIPMER 90) 42 year old lady - (AI 97)
a) Irradiation
a) Laser evacuation b) Hysterectomy
b) Hysterectomy
c) Conisation d) Wertheim's hysterectomy
c) Hysterosalphingo oophorectomy
657. A 30 years old lady was found to have PAP smear
d) Colposcopy and cervical biopsy
645. The commonest cause of death in carcinoma positive, cervical,biopsy negative, and endometrical
cervixis- (Al91) currettage positive, next thing to do is- (AIIMS 97)
a) Septicemia b)Haemorrhage a) Conisation b) Hysterectomy
c) Cachexia d) Renal failure c) Werthiems Hysterectomy d) Repeat PAP smear
646. Best method for cancer cervix screening is..: (AI 92) 658. · Investigations for carcinoma cervix in the early
a) Pap smear b) Colposcopy stage are all except- (AP 97)
c) Biopsy d) Colpomicroscopy a) Surface biopsy
647. Carcinoma cervix with involvement of upper 2/3 b) Ultrasound
ofvaginaisstage---- (JIPMER 92) c) Colposcopic guided biopsy
a) ITA b)IIB d) Schiller's test
c) IDA d)IDB 659. A patient with carcinoma cervix developed abdominal
648. In young women the method of diathermy · distension, abnormal mentation and intractable
conization should be- (JIPMER 80, DNB 90) hiccoughs. The likely cause is- (AIIMS 99)
a) Superfical conization b) Deep conization a) Uremia b) Brain metastasis
c) Any of the above d) None ofthe above c) Abdominal metastasis d) Liver metastasis
649. The cone of tissue removed in the operation of 660. A patient is diagnosed to have CIN IL She approaches
conization consists of- (AIIMS 78, DNB 90) .you for advice. You can definitely tell her the risk of
a) All the racemose glands lesion progressing to malignancy as- (AIIMS 2K)
b) Eroded area of the cervix a) 5% b)l5%
c) Some of the racemose glands c) 600/o d) 30%
d)A+B

638)a,c,e 639)a,b,c 640)a 641)c 642)d 643)a 644)d 645)d 646)a 647)a 648)a 649)d 650)b 651)c
652)b 653)c 654)c 655)c 656)b 657)a 658)b 659)a 660)a
GYNAE & OBS [ 33]

661. Cone biopsy is indicated in all the following 673. InCacervixlymphaticsdrainto- (PGI 02)
conditions except- (AIIMS 2K) a) Obturator
a) Micro invasive carcinoma b) External iliac
b) Indefinite diagnosis on colposcopy c) lnguinallympbnode
c) CINill d) Femorallympbnode
d) Cervical metaplasia e) Hyopogastric lympbnode
662. Time taken for conversion of CIN cervix to 674. A 26 year old female with 3 living issue having
invasivecarcinomais- years-(J&K OJ) cervical erosion which bleeds to touch, diagnosis
a) 5 b)lO canbedoneby- (PGI 03)
c) 15 d)20 a) Pap smear b) Excision biopsy
663. A 35 years old lady presents with post coital bleeding
c) Hysteroscopy d) Colposcopy
and foul smelling discharge is most important
675. 45 yrs old lady on punch biopsy shows carcinoma
investigation to arive at a diagnosis is- (MP 98)
is situ of cervix. The treatment is- (PGI 03)
a) Endometrial aspiration
b) Dilation and currettage a) Conisation b) Vaginal hysterectomy
c) Endometrial biopsy c) Diathermy d) Cryosurgeny
d) Cervical biopsy e) LASER
664. Treatment of choice for Ca in sitn cervix is - 676. Most common type of human papilloma virus
a) Follow-up with serial pap smear (PGI 97) causingCaCervixare- (PGI03)
b) Conization a) 16&18 b)1&33
c) Hysterectomy c) 6& 11 d)4& 14
d) Laser vaporization e) 2&5
665. A young lady comes with mild erosion of cervix and 677. HPV causes- (PGI 04)
pap smear shows dysplasia next step is- (PGI 98) a) Vulval Ca b) Cervical Ca
a) Antibiotics b) Coloposcopy c) Endometrial Ca d) Endometritis
c) Cryosurgery d) Conisation e) DUB
666. Rx of carcinoma in sitn of cervix- (PGI 98) 678. Cone biopsy of cervix is indicated in- (PGI 04)
a) Simple Hysterectomy b) Conisation a) Parametrial invasion
c) Laser d) Cryosurgery b) Abnormal pap smear
e) All c) Endometrial Ca
667. Cervix carcinoma arises from- (PGI 99) d) Endocervical curettage positive
a) Squamocolumnar junction b) Isthmus e) Clear cell Ca
c) Cervical lip d) Internal os 679. In carcinoma cervix, the causative etiological
668. Earliest symptom of carcinoma cervix is- (PGI 99) factoris- (UPSC02)
a) Irregular vaginal bleed b) Post coital bleed a) Condylomaacuminata b)HPV 16,18
c) Foul smelling discharge d) Pain c) HPV 6,11 d) HSV 1 and2
669. Patient with CIN-11 positive on pap's smear, next 680. Fmding on colposcopy suggestive of malignancy-
step will be- (PGI 2000) a) Candidiasis b) Papilloma (Orissa 04)
a) Repeat pap's smear c) Punctuation d) Condyloma
· b) Cone biopsy 681. Unsatisfactory colposcopy means-
c) Colposcopic directed biopsy a) Failure to visualize cervix (Karnataka 02)
d) Punch biopsy b) Failure to visualize transformation zone
670. Carcinoma cervix is more common in- (PGI OJ) c) Failure to visualize squamous epithelium
a) HlV b) Multiparity d) Failure to visualize columnar epithelium
c} Smoking d) Nulliparity 682. . HPYwhich is strongly associated with carcinoma
e) Familyhistory cervix is - (SGPGI 05, Kam 02)
671. Predisposing factors for carcinoma cervix- a) Type 6 b) Type 11
a) HlV b)HPV (PGI OJ) c) Type 16 d) Type 32
c) HSV d)EBV 683. Type of papilloma virus which is strongly
e) CMV ,.. associated with carcinoma cervix is- (AMU 05)
672. During cervical CA surgery to prevent the a) 33 b)34
complication the vault prolapse, should treated with- c) 56 d)55
a) Total Hysterectomy (PG/ 02) 684. Human papilloma virus (HPV) serotype most often
b) Vaginal hysterectomy associated with invasive carcinoma cervix is -
c) Subtotal hysterectomy a) HPV -11 b) HPV- 16 (Karnat05)
d) Abdominal Hysteroctomy c) HPV-18 d)HPV-31

661)d 662)b 663)d 664)a,b,c 665)b 666)e 667)a 668)b 669)c 670) a,b,c 671) a,b,c 672) a,d 673) a,b,e
674)b,d 675)b 676)a 677)a,b 678)b,d 679)b 680)c 681)b 682)c 683) a 684) b,c
GYNAE & OBS [ 34]

685. False statement about treatment of Ca cervix- 695. A34 year old lady who bas completed her family,
a) Radiotherpay is helpful in all stages(PGJJune 05) shows a positive Pap smear (C.I.N.DI). What will be
b) Prognosis of surgery good if done in early cases the further arrangement? (UPSC-11 08)
c) When radiotherapy given paraaortic nodes should a) Wertheim's hysterectomy
be included b) Ward Mayo's hysterectomy
d) Chemotherapy reserved for late cases c) Simple abdominal hysterectomy
e) From Ib onwards same prognosis with both d) Cervical conisation
surgery & radiotherapy 696. A 45 year old woman comes with Pap smear report
686. In which of the following genital tract matignancy, of squamous metaplasia. What advice should be given
to her? (UPSC-11 08)
the risk of metastasis to ovary is the least - {AI 06)
a) Colposcopy b) Cervical biopsy
a) Carcinomacervix
c) Conization d) No treatment
b) Carcinoma endometrium
697. A 45 years multiparous women, on punch biopsy
c) Carcinoma fallopian tube
shows carcinoma in site, Best treatment modatities
d) Uterine sarcoma is- {UP08)
687. Radical hysterectomy is done in which one of the a) Conisation
following- {APPG 06) b) Wertheem's hysterectomy
a) Carcinoma in situ c) Radiotherapy
b) Endo ca cercix stage IA d) Abdominal· hysterectomy with basal saphingo
c) Micro invasive carcinoma of cervix ophrectomy
d) Carcinoma cervix stage IB 698. Virus associated with cancer cervix is- {AI 08)
688. Presence of hydronephrosis in stging of carcinoma a) HPV b)HlV
cervix uteril denotes FIGO .......... disease- (Kam c) EBV · d)HTLV
a) Stagell-B b) Stage III-A pgmee 06) 699. Which of the following statements about squamous
c) Stage ID-B d) Stage IV-A cell carcinoma of cervix is false- {AI 08)
689. Most common agent responsible for Ca cervix is- a) Common at squamocolumnar junction
a) HPV16 b)HPV 18 · (AI07) b) CT scan is mandatory for staging
c) HPV31 d)HPV33 c) Post coital bleeding is a common symptom
690. Human papilloma virus is associated with this d) HPV 16 and 18 are associated with high risk of
carcinoma- {MAHE 07) carcinogenesis
a) Carcinoma cervix b) Carcinoma uterus 700. A 35 year old female, with post coital bleeding, next
c) Fibroids d) None of the above step- (Aiims May 08)
691. Poor prognostic factors in the management of a) Clinical examination and pap smear
b) Visual inspection with acetowhite
cancer cervix are the following except- (UPSC 07)
c) Visual inspection with lugol's iodine
a) Young age
d) Colposcopy directed biopsy
b) Well-differentiated squamous cell carcinoma
701. True about carcinoma in women- (PG!June 08)
c) Hydroureter
a) Pap smear reduces incidences ofCa cervix
d) Adenocarcinoma b) Ca cervix is M.C. malignancy in Indian women
692. Commonest tumor of the cervix is- (0Jmed08, TN89) c) Breast Ca is the M.C. malignancy in Indian women
a) Sqamous cell carcinoma b) Adenocarcinoma d) Next to Cervical Ca, Lung carcinoma is next most
c) Sarcoma d) Adenoacanthoma common malignancy
693. Patient diagnosed as squamous cell intraepithelial 702. Most cost effective and least invasive mode of
lesion which of the following has the highest risk treating squamous intraepitheliallesion of cervix
for progression to carcinoma- (AIIMS Nov 07) is - {DPGEE 08)
a) Low grade squamous intraepithelial neoplasia a) Electrocauterization b) Cryotherapy
b) High grade squamous intraepithelial neoplasia c) Laser therapy d) Any of the above
c) Squamous intraepithelial neoplasia associated with 703. DiffenceofCISfrominvasivecarcinoma?
HPV16 a) Age> 40 years {APPG 08)
d) Squamous intraepithelial neoplasia associated with b) Incidental detected
HN c) Bilateral
694. Cervical smear fixation is done by- (AIIMS Nov 07) d) Histology similar to invasive
a) Ethyl alcohol b) Acetone 704. Risk factors for Ca cervix- {PGI Dec 08)
c) Xyline d) Formalin a) Vtrginity b) Multiple sex partners
c) Genital warts d) Circumcised partner
e) Late menarche

685)e 686)a 687)d 688)c 689)a 690)a 691)b 692)a 693)b 694)a 695)d 696) d 697) d 698) a
699)b 700)a 701)a,b,c,d 702)a 703)d 704)b,c
GYNAE & OBS [ 35]

705. Common site of origin of cervical carcinoma is/are- 714. A 35 yr old P 3+0 is observed to have CIN grade m on
a) Endocervi:x (PGI Dec 08) colposcopic biopsy. Best treatment will be-
b) Ectocervix a) Cryosurgery b) Conization (AIIMSMay 10)
c) Squamocolumnar junction c) LEEP d) Hysterectomy
706. All of the following investigations are used in FIGO 715. A lady undergoes total radical hysterectomy for
staging of carcinoma cervix except? stage lb Ca cervix. It was found that Ca extends to
a) CECf (AIIMS Nov 08) lower part of body ofutems and upper part of cervix.
b) Intravenous pyelography Next step of management will be- (AIIMSMay 10)
c) Cystoscopy a) Chemotherapy b) Radiotherapy
d) Proctoscopy c) Chemoradiation d) Follow-up
707. A 35 year old female P3 + 0 is observed to have CIN 716. Which ofthe following agents is most commonly
grade III on colposcopic biopsy. Which of the associated with carcinoma cervix- (AI 1 0)
following would be the best initial management- a) HPV16 b)HPV18
a) Cryosurgery b) Conization (AI 09) c) HPV33 d)HPV35
c) LEEP d) Hysterectomy 717. A 40yearold woman presents with abnormal cervical
708. True about staging of cervical cancer- cytology in PAP smear suggestive oc CIN ID (HSIL).
a) Staged clinically (PGI June 09) The next, beststepinmanagementis- (AI 10)
b) When there is doubt regarding staging, earlier stage a) Hysterectomy
is selected b) Colposcopy and LEEP
c) Carcinoma extending beyond cervix to pelvic wall c) Colposcopy and Cryotherapy
is stage II d) Conization
d) Stage II includes canncer extending to uterus 718. The colposcopic features suggestive of malignancy
e) Identification ofparaaortic L.N in clinical stage I are all except- (DPG 10)
makes it stagelV a) Condyloma b) Vascular atypia
709. A young lady comes with mild erosion of cervix and c) Punctuation c) White epithelium
pap smear shows dysplasia. Next step is- 719. A patient with carcinoma cervix who has completed
a)Antibiotics b) Colposcopy radiotherapy comes with uraemi A. The most
c} Cryosurgery .d) Conisation commoncaliseis- {DPG 10)
710. Carcinoma cervix originates in the- a) Bilateral ureter invasion
a) Colunmarepithelium (DELHI PG Mar. 09) b) Radiation nephritis
b) Squamous epithelium c) Ureteric stenosis due to radiation
c) Transformationzone d) Unconnected causes
d) Uterine isthmus 720. In colposcopy, the following are visualized except-
711. The most appropriate management of a 32 weeks a) Upper273rdendocervix (DPG 10)
pregnant lady with carcinoma cervix stage 2b is- b) Lower 173rd endocervix
a) Chemoradiation followed by labour induction and c) Vault of vagina
vaginal delivery (UPSC-II 09) d) Lateral fornix
b) Labour induction, vaginal delivery followed by 721. True statement (s) regarding ca cervix involving
radiotherapy parametrium but not pelvic involvement-
c) Lower segment caesarean section followed by a) StageiiA (PGIMay10)
radiotherapy b) Stage liB
d) Classical caesarean section followed by c) Radiotherapy should be given
radiotherapy d) Hysterectomy can be useful
712. Which one of the following histological types is the e) Staging should be done only after cystoscopy
most virulent carcinoma of cervix? (UPSC-II 09) 722. In Ca cervix low grade squamous intraepithelial
a) Small cell keratinizing lesion (LSIL) ill Bethesday system involves-
b) Large cell keratinizing a) CINI b)CINII (PGIMay10)
c) Small cell non-keratinizing c) CINUI d) Squamous metaplasia
d) Large cell non-keratinizing 723. A 35 yr old P 3+o is observed to have CIN gradeiD on
713. 35 year female with post coital bleeding. Next step colposcopic biopsy. T/t includes- (PGI May 10)
is? (AIIMS Nov 09) a) Cold knife conization
a) Pap smear with clinical examination b) Hysterectomy
b) Visual examination with lugo L iodine c) Radical hysterectomy
c) Visual examination with acetic acid d) Lap assisted hysterectomy
d) Colposcopy directed biopsy e) LEEP

705)a,b,c 706)a 707)c 708)a,b,c 709)a 710)c 7ll)d 712)c 713)a 714)c · 715)d 716)a 717)b 718)a
719)a 720)a 721)b,c,e 722)a 723)e
GYNAE & OBS ( 36]

724. Treatment of CA cervix stage Ib includes- 734. Carcinoma endometrium involving uterine cavity
a) Surgery b) Chemotherapy (PGI Nov. 10) more than 10 ems long with early myometrial
c) Radiotherapy d) Cryotherapy involvement is- (Al 2K)
e) lEEP a) Stage Ia b) Stage Ib
725. Adenocarcinomacervixisalw- (PG!Nov.JO) c) Stage lla d) Stage lib
a) HPV6 b)HPVll 735. True about endometrial carcinoma- (PGI OJ)
c) HPV16 d)HPV 18 a) Predisposed by DM, hypertension & obesity
e) HPV51 b) Adenosquamous type is commonest
726. Treatment ofCa Cervix lliB include-(AllMSNov 10) c) Commonly associated with Ca. Cervix
a) Wertheims hysterectomy d) Common age group affected is between 20-40 yrs
b) Schauta's hysterectomy 736. The most malignant endometrial carcinoma is-
c) Chemotherapy a) Adenocarcinoma (Jipmer 03)
d) Intracavity brachytherapy followed by external b) Adeno acanthoma
beam radiotherapy c) Mixed adeno squamous
727. A35-year- old woman who bas completed her family d) Clear cell carcinoma
shows a positive Pap smear (Cervical intraepitbefial 737. Endometrial Ca risk factors is/are- (PGI 04)
neoplasia ill (CIN ill)). What is to be done next- a) Hypertension b) Diavetes
a) Simple hysterectomy (UPSCII 11) c) Nulliparity d) Tamoxifen therapy
b) Wertheim's hysterectomy e) Multiparity
c) Conisation 738. In a case of endometrial cancer, if metastasis is seen
d) Cryotherapy in the vagina, wbatFIGO stage it would be-(UPSC 04)
a) Stage III a b) Stage III b
CARCINOMA ENDOMETRIUM (UTERUS) c) Stage III c d) Stage IV a
739. An 80 year old female who bas never taken estrogen
728. Second stage of carcinoma endomtrium is treated
develops pink vaginal discharge. An endometrial
by- (AI 91)
biopsy shows adenocarcinoma of the endometrium.
a) Surgery
fapanicolaousmear is negative. Ofthe following what
b) Radiotherapy followed by surgery
is the most important indicator of prognosis is ?
c) Chemotherapy
a) Body habitus b)LevelofCA-125
d) Progesterone followed by surgery
c) Nutritional status d) Histologic type of tumor
729. Commonest histology of carcinoma of endometrium
740. Percentage change in cystic glandular hyperplasia
is - (Kerala 94)
turning to mafignancy- (PGIJune 05)
a) Squamous cell b) Clear cell
a) 0.1 b)0.2
c) Adeno carcinoma d)Anaplastic carcinoma
c) 1 d) 10
730. The treatment of choice in Endometrialcarcinoma
e) 15
Stage lis - (Karn. 94)
741. The following are indications for post operative
a) External radiotherapy
rediotherapy in a case of carcinoma endometrium
b) Intracavitary radiation
except- (Al!MS May 05)
c) Hysterectomy
a) Myometrial invasion of more than half thickness
d) Chemotherapy
b) Positive lymph nodes
731. All are risk factor for endometrial carcinoma
c) Endocervical involvement
except- (TN 98)
d) Tumor positive for estrogen receptors
a) Diabetes b) Hypertension
7 42. The risk of complex hyperplasia of endometrium
c) Obesity d) Multiparity
with atypia progressing to malignancy in a
732. In Endometrial carcinoma lymph node spread is
postmenopausal woman is about- (Al!MS May 05)
notto- (Al!MS 97) a) 3% b)8%
a) Paraortic b) Inguinal c) ISO/o d)28%
c) Inferiormesentric d) Pre sacral 743. The risk of endometrial carcinoma is the highest
733. Carcinoma of the endometrium with metastasis with the following histological pattern of
in the deep inguinallymphnodes is of stage- endometrial hyperplasia- (Al!MS 06)
a) I b)II (AIIMS99) a) Simple hyperplasia without atypia
c) Ill d)IV b) Simple hyperplasia with atypia
c) Complex hyperplasia without atypia
d) Complex hyperplasia with atypia

724)a,b,c 725)c,d,e 726)d 727)c 728)b 729)c 730)c 731)d 732)c 733)d 734)b 735)a 736)d
737)a,b,c,d 738)b 739)d 740)c 74l)d 742)d 743)d
GYNAE & OBS [ 37]

744. Indication for radiotherapy in carcinoma 753. The risk of progression to endometrial cancer from
endometrium include all except- (AIJMS Nov 07) simple hyperplasia without atypia is;. (UP SC II 10)
a) Pelvic node involvement a) 1% b)3-5%
b) Deep myometrial involvement c) 8-10% d)25-30%
c) Enlarged uterine cavity 754. Which of the following is not seen in Corpus Cancer
d) Poor differentiation syndrome- (AIIMS Nov 10)
745. Which one of the following conditions of the a) Diabetes mellitus b) Hypertension
endometrium is associated with a significantly
c) Obesity d) Multiparity
increased risk of development of cancer?
755. Endometrial cancer involving 50% ofendometrium,
a) Adenomatous hyperplasia (UPSC-II 08)
b) Hyperplasia with atypia extending to vagina, lymph nodes negative with
c) Cystic glandular hyperplasia positive peritoneal cytology is staged as-
d) Proliferative phase a) 3a b) 3b (AIIMS Nov 10)
746. At laparotomy for endometrial cancer the growth c) 3cl d)3c2
was found invading inner half of myometrium, right 756. The following at risk factors for carcinoma
ovary and upper part of vagina. What is the staging? endometrium except- (Karn 11)
a) II b b) ill a (UPSC-II 08) a) Early menopause
c) illb d)IVa b) Endometrial hyperplasia
747. A case of choriocarcinoma 30 year old having one c) Nulliparity
child, presenting with growth in the uterus and d) Obesity.
single metastasis in lung (stage m score 4). What 757. Endometrial carcinoma with worst prognosis-
would be the appropriate treatment for her? a) Clear cell carcinoma (Jipmer 11)
a) Single agent chemotherapy (UPSC-II 08) b) Adenocarcinoma grade-II
b) Multiple agent chemotherapy c) Adenocarcinoma
c) Hysterectomy with single agent chemotherapy
d) Adeno acanthoma
d) Hysterectomy with multiple agent chemotherapy
748. What is the ideal treatment for a 55 yr female with
OVARY
simple hyperplasia of endometrium with atypia?
a) Simple hysterectomy (AI 08)
758. Which is not true of dermoid cyst of ovary-
b) Medroxy progesterone acetate (MPA)
a) Commonly more than 10 em (AI 93)
c) Levonorgesterol (LNG)
d) IUCD b) Has sebaceous material
749. TrueaboutFIGOendometrialcarcinomalllb- c) Bilateral in 30%
a) Vaginal metastasis seen (PGI June 08) d) Lined by epithelial cells
b) Serosa involved 759. Polcystic ovaries are associated with- (Delhi 96)
c) Rectum a) Hypopituitarism
e) Pelvis b) Ovarian malignancy
750. Which of the following associated with endometrial c) Endometrial carcinoma
cancer? (APPG 08) d) Adrenal cortical hyperplasia
a) Metropathica haemorrhagica 760. The condition where glistening spots are found
b) Dysgermniona on tubal secosa- (AIIMS95)
c) Fibroid a) Walthead cell nests b) Metastases from ovary
d) All c) Parovarian cysts d) Tubal fibrosis
751. All of the following is/are risk factor (s) for 761. A 25-year old married infertile woman having
Endormentrial carcinoma except- (PGI June 09)
regular menstruation, fever, lower abdominal pain
a) Multiparity
and dysmenorrhoea presents herself at the OPD.
b) Obesity
On examination, there are bilateral soft tender
c) Early menopause
d) Unopposed Estrogen therapy masses of"3" diameter in both fornices and uterus
e) Hypertension is of normal size. The most likely diagnosis
752. A 80-year old female who has never taken estrogen is - (UPSC 97)
develops pink vaginal discharge. An endometrial a) Cystic ovaries b) Tubo-ovarian masses
biopsy shows adenocarcinoma of the endometrium. c) Ectopic pregnancy d) Tuberculous salpingitis
Papanicolaou's smear is negative. Ofthe following, 762. Precocious puberty is seen with which ovarian
what is the most important indicator of prognosis- tumour- (PGI 98)
a) Body habitus b)LevelofCA-125 (DPG 10) a) Dermoid b) Gynandroblastoma
c) Nutritional status d) Histologic type of tumour c) Granulosa cell tumour d) Arrhenoblastoma

744)c 745)b 746)c 747)a 748)a 749)a,b,c 750)a 751)a,c 752)d 753)a 754)d 755)b 756)a 757)a
758)a,c 759)c 760)c 761)b 762)c
GYNAE & OBS [ 38]

763. The first step in the management of hirsutism due 775. Which of the following statements in incorrect
to stein Ieventhal syndrome is~ (PGI 99) regarding polycystic ovarian disease? (AI 06)
a) OCP b)HMG a) Elevated LH hormone
c) Spironolactone d) Bromocriptine b) Can cause infertility
764. In Polycystic ovarian diseases, all ofthe following c) May be associated with abnormal glucose
are seen except- (PGI 0 I)
tolerance test
a) Endometrial carcinoma b) Increased FSH
c) Streak ovaries d) Insulin resistance d) Results in postdated pregnancy
e) Hirsutism 776. Which of the following ovarian tumor is most
765. Schiller-Duval body is seen in- (PGI OJ) prone to undergo torsion during pregnancy?
a) Germ cell tumor b) Yolk-sac tumor a) Serous cystadenoma (AI 06)
c) Choriocarcinoma d) Granulosa cell tumor b) Mucinous cystadenoma
e) Dysgerminoma c) Dermoid cyst
766. PCOD which of the following is seen- (PGI 02) d) Theca lutein cyst
a) Hirsutism b) 2° almenorrhoea 777. Most common ovarian cyst to undergo torsion is-
c) Streak ovaries d) i FSH/LH a) Benign cystic teratoma (AI 07)
e) i Oestrogen (E2) b) Dysgerminoma
767. Ovarianreservetestsare- (PGI03) c) Ovarian fibroma
a) lH b)FSH d) Brenners tumour
c) Plasma progesterone d) Endometrial biopsy
768. Theca lutein cysts seen in- (PGI 03)
778. Endodermal sinus tumour is assocaited with-
a) H. Mole b) Choriocarcinoma a) Schiller duval body (AIIMS May 07)
c) Thecacellhyperplasia d)PCOD b) Multinucleate giant cells
e) Ectopic pregnancy c) R-Scells
769. Trueaboutdermoidcystofovary- (PGI 03) d) Plasma cells
a) It is a teratoma 779. Ovarian tumour which is bihiteral- (Aiims May 07)
b) Frequently undergo torsion a) Dysgerminoma b) Endodermal sinus tumour
c) X-ray is diagnostic c) Immature teratoma d) Embryonal cell carcinoma
d) Invariably turns to malignancy 780. True about sertoli cell tumours of ovary are all
e) Contains sebaceous materials & hair except- (UP 07)
770. Which causes torsion of ovarian tumor -(Kerala 03) a) Secrete male hormone
a) Trauma b) Uterine contraction b) Secrete female hormones
c) Physical movements d) All
c) Androblastomas
771. Tumor marker most helpful in follow up of a case of
Epithelial carcinoma of ovary is- (Kam 02) d) Gynandroblastoma
a) CA-125 781. Most common benign ovarian tumours is- (UP 07)
b) Serum Alpha Fetoprotein a) Serous cyst adenoma
c) Serum human chorionic gonadotropin b) Mucinous cyst adenoma
d) Human Placental Lactogen c) Dermoid cyst
772. A 45 year old female is having bilateral ovarian d) Brenners tumour
mass, ascites and omental caking on CT scan. 782. Poly cystic ovarian syndrome is characterized by
There is high possibility that patient is having- all except- (UP 07)
a) Benign ovarian tumor (AI 03) a) Hyper androgenaemia
b) Malignant epithelial ovarian tumor b) Normal or raised oestrogen
c) Dysgerminomaofovary c) RaisedLH
d) Lymphoma of ovary d) Normal FSHILH ratio
773. A 28 yr old lady, Rani, is suspected to have 783. The following are long term complications ofPCOS
polycystic ovarian disease•. Sample for testing LH (Polycystic Ovarian syndrome) except -(UPSC-II 08)
& FSH are best taken on the following days of
a) Type II Diabetes mellitus
Menstrual cycle- (MAHA 05)
a)1-4 b)8-10 · b) Cardiovascular disease
c) 13-15 d)24-26 c) Endometrial cancer
774. The following hormone is raised in polycystic d) Ovarian cancer
ovarian syndrome- · (AI 06) 784. HAIR-AN syndrome is found in- (UPSC-II 08)
a) 17-0H progesterone a) Congenital adrenal hyperplasia
b) Follicular stimulating hormone b) Luteal phase defects
c) Luteinizing hormone c) Polycystic ovarian syndrome
d) Thyroid stimulating hormone d) Pheochromocytoma

763)a 764) b,c 765) b 766) a,b,e 767) b 768) a,b 769) a,b,c,e 770) a 77l)a 772)b 773)b 774)c 775)d
776)c 777)a 778)a 779) a 780) b 781) c 782) d 783) d 784) c
GYNAE & OBS [ 39]

785. A lady with CA ovary in follow up with raised CA 125 794. Which is not a germ cell tumour- (JIPMER 91)
levei,NEXTSTEP- (AiimsMay08) a) Dysgerminoma
a)Cf b) Teratoma
b) PET c) Granulosa theca cell tumour
c) MRI d) Embryonal cell carcinoma
d) Clinical exam and serial follow up ofCA125 795. Post menopausal women has a 4x4 em ovarian mass
786. Feminizing ovarian tumour is- (DPGEE 08) the correct line management is- (AI 92)
a) Wait and watch b) Surgical exploration
a) Granulosa cell tumour b)Arrhenoblastoma
c) Progesterone pills d) Clomiphene therapy
c) Hilus cell tumour d) Gynandroblastoma
796. Which ovarian cyst does not undergo malignancy-
787. A 35 year asymptomatic women on USG shows cyst a) Mucinous b)Papillary (AIJMS92)
in ovary of7 em with negative serology. Treatment c) Dermoid d) Granulosa Theca
include- (PGI Dec 08) 797. o/o of ovarian tumours of non epithelial origin in
a) Laproscopic cystectom childhood is- (JJPMER 95)
b) Laprotomy a) 20% b)30%
c) FollowupwithCA 125 c) 40% d) 50%
d) Follow up after l-2 cycle ofCOP 798. % of malignancy of ovaries in childhood is-
e) Ovariotomy a) 10% b)20% (JIPMER 95)
788. A 25 year old married nullipara undergoes c) 30% d)40%
laproscopic cystectomy for ovarian cyst which on 799. Commonest complication of mature treatoma is-
histopath reveals ovarian serous cisadenocarcinoma. a) Torsion b) Rupture (AI 95, AP 88)
What should be the next management? c) haemorrhage d) Malignancy
a) Serial Ca 125 measurement and follow up 800. One offollowing are true about krukenberg tumour
b) Hysterectomy and bilaterals alpingoophorectomy ofovaryexcept- (A/96)
c) Unilateral salpingoophorectomy (AIIMS Nov 08) a) Usually bilateral
d) Radiotherapy b) Enlarged ovaries
c) Cystic areas in ovary
789. A patient is diagnosed with ovarian cancer and
d) Signet ring cells on histology
planned for staging lapratomy. Surgical staging
801. Reinke crystals are seen in- (AIIMS 95)
would involve all of the following except- (AI 09)
a) Papillary cystadenoma b) Dermoid
a) Peritoneal washings b) Peritoneal biopsy
c) Dysgerminoma d) Hilus cell tumour
c) Omental resection d) Palpation of organs
802. Commonest type of ovarian tumour in 20-years
790. A 15 15 em ovarian cyst has been diagnosed in an
old female is- (AJJMS 97)
8 weeks pregnant lady. Further management
a) Germ cell b) Epitheloid
includes- (PGI Nov. 10)
c) Metastatic d) Sex cord
a) Only follow up without surgical intervention 803. Granulosa cell tumour of ovary is associated with-
b) Laparotomy at 14-16 weeks a) Carcinoma endometrium of uterus (AIIMS 96)
c) Caesarean section and ovariotomy at term b) Carcinoma ovary
d) Surgery after delivery c) Carcinoma cervix
e) Immediate operation d) Carcinoma Vagina
791. A 35-yr old woman presents with primary infertility 804. A 30-year old multiparous woman is found to have
& palpable pelvic mass. Her Ca-125 level is 90 a 5 to 6 em cyst in one ovary the most appropriate
U/ml. diagnosis is- (AIIMS May 11) line of management would be- (UPSC 97)
a) Ovarian Ca b) Endometrioma a) Laparotomy and cystectomy
c) Tuberculosis d) Borderline ovarian tumor b) Laparotomy and unilateral slpingo oophorectomy
792. Cali-Exner bodies are seen in the microscopic c) Laparoscopic aspiration of the cyst
examination of the following Ovarian tumors- d) Observation to see if it disappears by itselfin three
a) Clear cell tumor (Kam 11) months
b) Granulosa cell tumor 805. Which one of the following is an important tumour
c) Hilus cell tumor marker in the diagnosis ofendodermalsinus tumour-
d) Theca cell tumor a) Hm b) Ca-125 (UPSC 99, PGI 96)
c) LDH . d) Alpha foe to protein
CARCINOMA OVARY 806. Carcinoma involving both the ovaries with
rupture of the capsule and ascitic fluid cytology
793. Teratoma arises form- (TN91) positive for malignant cells is of stage -(AIIMS 99)
a) Totipotent cells b) Mesodermal cells a) I b)ll
c) Ectodermal cells d) Endodermal cells c) III d)IV

785)b 786)a 787)d 788)c 789)d 790)b 79l)b 792)b 793)a 794)c 795)b 796) None 797) d
798)a 799)a 800)b 801)d 802)a 803)a 804)d 805)d 806)a
GYNAE & OBS [ 40]

807. Size of ovary above which considered to bemalignant- 820. All are true about serous cystadenoma of the ovary
a) 2cm b)5cm (CMCOJ) EXCEPT- (UPPGMEEO~
c) Scm d) 10cm a) Bilateral
808. Marker for ovarian cancer is- (TN 02) b) Unilateral
a) Ca 125 b) Ca 19-19 c) Concentric calcifications
c) ca 15-5 d) Hypercalcemia d) Multiloculated, sticky, gelatinous fluid
809. Featue is USG suggestive of ovarian malignancy 821. How much is tbe risk of ovarian cancer increased
is- (PGI99) above normal in a woman with non antuosomal
a) Papillary pattern b) Septations dominant genotype witb one first degree relative
c) Bilaterality d) Clear fluid
with ovarian cancer- (Karnataka 04)
810. Ca 152 is the tumor marker of- (PGI 99)
a) 2-3 times b) 5 times
a) Serous cystadenoma
c) 10 times d) 20 times
b) Mucinous cystadenoma
822. Increase Alfa-fetoprotein is seen in all except-
c) Dysgerminoma
d) Arrhenoblastoma a) Open spina bifida (SGPGI 05)
811. True about yolksac tumour- (PGI 02) b) Dysgerminoma
a) Also called endodermal sinus tumour c) Endodermal sinus tumor
b) Almost have elevatedAFP level d) Bronchial aplasia
c) Schiller Duval bodies seen 823. Treatment modalities for ovarean cancers is-
d) Highly malignant a) Staging laparotomy . (SGPGI 05)
812. TrueaboutBrennertumour- (PGI 03) b) Surgery
a) BIL c) Chemotherapy
b) Resembles fibroma d) Combined approach
c) Accounts for 20% of all ovarian tumors 824. Which of the following statement is true regarding
d) Common in postmenopausal age group benign cystic teratoma- (SGPGI 05)
813. Call-Emer bodies are found in -(PGI 03, 88, UPSC a) Rarely Undergo torsion
a) Granulosa cell tumour 85, 88) b) Metastasis is common
b) Mucinous cell tumour c) 10% are BIL & malignant
c) Serous cell tumour d) Contains Call-exner bodies·
d) Dysgerminoma 825. Which of tbe following combination is not true-
e) Leydig cell tumour a) CallHexnerbodies-GCtumor (AMU05)
814. Meig's syndrome is associated with- (PGI 99) b) Rokitansky bodies - teratoma
a) Teratoma b) Brenner tumour c) Schillar dual bodies- serous tumor
c) Theca cell tumour d) Fibroma
d) Reinke crystal - Hilus cell tumor
815. Pseudo myxoma peritonei is seen in- (PGI 98)
826. Ovarian tumor limited to true pelvis with
a) Serous cystadenoma
Negative nodes histological confirmed seeding of
b) Pseudomucinous cystadenoma
abdominal portioned surface. Tbe exact grading-
c) Mucinous cystadenoma
d) Teratoma a) IDA b)IDB (HPU05)
816. Common ma6lgnant germs cell tumours of ovary- c) me d)V
a) Choriocarcinoma (PGI 04) 827. Complications ofbenign ovarian tumors areAJE..
b) Arrhenoblastoma a) Torsion b)Bleeding(MAHE05)
c) Brennertumor c) Pseudomyxoma d) Metastasis
d) Serous cystadenoma 828. MC ovariantumorinyoungeragegroup-(PGJ.June05)
e) Teratoma a) Dysgerminoma b) Dermoid
817. Chemotherapeutic drug effective in the treatment c) Mucinous cystadenoma d) Fibroma
of epithelial ovarian cancer is- (Karnataka 02) e) Granulosa cell tumor
a) Carboplatin b)Paclitaxel 829. The following tumours commonly metastasis to
c) Cyclophosphamide d) Methotrexate the ovary, except- (J & K 05)
818. Commonest tumor of ovary occurring in a young a) Malignant melanoma b) Stomach
womanis- (TN03) c) Oesophagus d) Lymphoma
a) Immature teratoma b) Yolk sac tumor 830. Chemotherapyfordysgerminomais- (APPGE05)
c) Endometroid tumor d) Dysgerminoma a) Cisplatin, etopisides, belmoycin
819. Hob nail cells are seen in- (TN 03) b) Cyclophosphamide, vincristine, prednisolone
a) Hilus cell tumor b) Clear cell carcinoma c) Adriamycin, cyclophosphamide, cisplatin
c) Dysgerminoma d) Arrhenoblastoma d) Methotrexate, oncovin, procarbacine

807)c 808)a 809) a,b,c 810) a,b 811) a,b,c,d 812) b,d 813)a 814)d 815)c 816)e 817)b 818)d 819)b
820)d 821)a 822) b 823) d 824) c 825) c 826) a 827) d 828) a 829)c 830)a
GYNAE & OBS [ 41 ]

831. Presence of more than two germ cell layer in a 842. Ovarian epithelial carcinoma constitutes what
tumor is called- (Orissa 05) percent of ovarian malignancies- (MARE 07)
a) Dysgerminoma b) Teratoma a) 80% b)95%
c) Theca cell tumor d) Seminoma c) 60% d)70%
832. All of the following are the markers for malignant 843. What is the most common ovarian tumour to 1,111dergo
germ cell tumors of ovary except- (AI1MS Nov 05) torsion- (UPSC 07)
a) CA- 125 b) Alphafetoprotein a) Mucinous cystadenoma
c) jl-HCG d)LDH b) Serous cystadenoma
833. The most common pure germ cell tumor of the c) Dermoid
ovary is- (AI 05)
d) Brenner tumour
a) Choriocarcinoma b) Dysgerminoma
844. Which one of the following is not an epithelial
c) Embryonal cell tumor d) Malignant Teratoma
tumouroftheovery- (UPSC 07)
834. In a case of Dysgerminoma of ovary one of the
a) Serous cystadenoma
following tumor markers is likely to be raised-
a) SerumHCG (AI 05) b) Brennertumour
b) Serum alphafetoprotein c) Clear cell tumour
c) Serum lactic dehydrogenase d) Endodermal sinus tumour
d) Serum inhibin 845. Which one ofthe following groups has only epithelial
835. Which of the following is the most common pure tumorsofovary? (UPSC-1108)
ma6gnant germ cell tumor of the ovary- a) Serous cystadenoma, mucinous cystadenoma,
a) Choriocarcinoma (AIIMS May 05) endometrioid carcinoma
b) Gonadoblastoma b) Mucinous cystadenoma, endometrioid carcinoma,
c) Dysgerminoma stromal cell carcinoma
d) Malignant teratoma c) Endometrioid carcinoma, Brenner's tumor, sex cord
836. Which of the following strategy has been tumor
recommended to reduce the hereditar risk for d) Dermoid, endometrioid carcinoma, mucinous
ovarian cancer in woman with BRCA I& BRCA cystadenoma
Ilmutation- (AIIMSNov05) 846. Sex cord stromal cell tumours do not include which
a) Use of Oral Contraceptive Pills one of the following? (UPSC-11 08)
b) Screening with Transvaginal Ultrasound a) Granulosa cell tumour b) Theca cell tumour
c) Screening with CA-125 c) Arrhenoblastoma d) Brenner's tumour
d) Prophylactic oophorectomy 847. An unmarried 20 year old girl has uuilateral ovarian
837. In a suspected of ovarian cancer, imaging work- cyst on x 8 em. with CA125 at 500 miU/ml. There
up is required for all of the following information,
are thick septa and papillation in cyst. On
except-
laparotomy, capsule is intact. Other ovary and viscera
a) Defectionofadenexallesion (AI 06)
· are normal. What is the appropriate surgery for
b) Characterization of the lesion
her (along with staging procedure)? (UPSC-11 08)
c) Staging
d) Assess respectability a) Ovarian cystectomy
838. CA-125 is a marker antigen for the diagnosis of- b) Salpingo-oophorectomy
a) Colon cancer b) Breast cancer (AIIMS 06) c) Hysterectomy with unilateral salpingo-
c) Brain cancer d) Ovarian cancer oophorectomy
839. The pseudomyxoma peritonei occurs as a d) Panhysterectomy
compHcation of the follQwing ovarian tumour- 848. Cali-Exner bodies are seen in- (Aiims May 08)
a) Serous cystadenoma (AIIMS 06) a) Granulosa cell tumor b)YolkSactumor
b) Mucinous cystadenoma c) Choriocarcinoma d) Dysgerminoma
c) Dysgerminoma 849. Marker for granulosa cell tumor- (Aiims May 08)
d) Gonadoblastoma a) CA19-9 b)CA50
840. Features of dysgerminoma are- (PG1 June 06) c) Inhibin d) Teratoma
a) Unilateral b) Postmenopausal 850. Ovarian epithelial carcinoma constitutes what
c) Vrrilising d) Cut section gritty percent or ovarian maHgnancies- (Manipal 08)
e) i AFP a) 800/o b) 95%
841. Cystoglandular hyperplasia is associated c) 60% d)70%
with- (COMED 06) 851. Whiff test is used for? (APPG 08)
a) Theca cell tumour b) Hilus cell tumour a) Candida b) Gardenella
c) Gynandroblastoma d)Dysgerminoma c) Trichomonas d)LGV

83l)b 832)a 833)b 834)c 835) c . 836) d 837)None 838)d 839)b 840)a 841)a 842)b 843)c 844)d
845)a 846)d 847)a 848)a 849)c 850)b 85l)b
GYNAE & OBS [ 42]

852. Which is a germ cell tumour? (APPG 08) 863. Which tumour marker is most often elevated in
a) Dysgerminoma b) Brenners tumour ovarian granulosa cell tumour? (UP SC II 10)
c) Clear cell tumour d) Mucinous tumours a) lnhibin b) Alpha fetoprotein
853. What is the stage of ovarian cancer with h/1 c) Beta-HCG d) CA 125
involvement with capsule intact, no ascites ? 864. The following features are true about Dysgerminoma
~~ ~m ~"G~ EXCEPT- (Karn 11)
c) IC d)II a) It is the commonest benign germ cell tumour
854. True about dysgerminoma- (PGI June 09)
b) It can be associated with dysgenetic gonads
a) Radiosensitive
c) It can be bilateral in 10% ofcases
b) Most common maegnant germ cell tumour
c) B/L in 80% cases d) It is sensitive to both chemotherapy and
d) i AFP radiotherapy
e) common in postmenopausal 865. Marker of endodermal sinus tumor is -(Jipmer 11)
855. The most common ovarian neoplasm to undergo a) a. fetoprotein b)HCG
torsion is - (DELHI PG Mar. 09) c) IDH d)CA-125
a) Mucinous cyst adenocarcinoma
b) Benign cystic teratoma RADIOTHERAPY & CHEMOTHERAPY AND
c) Immature teratoma
d) Dysgerminoma
PHARMACOLOGYINGYNAECOLOGY
856. The..most characteristic presentation of fallopian
tube carcinoma is- (DELHI PG Mar. 09)
866. Danazolis used in all except- (JIPMER 91)
a) Mass abdomen a) Hirsutism
b) Bleedingpervaginum b) Endometriosis
c) Excessive watery discharge per vaginum c) Dysfunctional uterine bleeding
d) Pain abdomen d) Fibroid
857. An ovarian neoplasm in a 14-year old girl is most 867. Danazolis not used in:.. (AIIMS 92)
likely to be- (DELHI PG Mar. 09) a) Endometriosis
a) Germ cell tumor b)DUB .
b) Epithelial tumor c) Precocious puberty
c) Sertolic Leydig cell tumor d) Endometrial Carcinoma
d) Granulosa cell tumor 868. GnRH anarilogues are useful in all except-
858. The incidence of bilaterality in a dermoid cyst is a) Endometriosis (AP 97)
approximately- (DELHI PG Mar. 09) b) Hyperprolactinemia
a) 100/o b) 30% c) Precocious puberty
C) 500/o d) 70%
d) Menstrual disturbances
859. Which of the following tumor markers is most likely
869. Clomiphene citrate is- (AP 97)
to be raised in a case of dysgerminoma of the ovary-
a) SerumHCG (AI 09)
a) Anti-androgen b) Synthetic steroid
b) Serum Alphafetoprotein (AFP) c) Anti estrogen d)GnRHanalogue
c) Serum Lactate dehydrogenase (LDH) 870. Danazol is used in all of the following except-
d) SerumCA 19-9 a) Fibroadenosis ofbreast (UPSC99)
860. Call exner bodies seen in- (AIIMS Nov 09) b) Dysfunctional uterine bleeding
a) Granulosa cell tumour c) Pituitary adenoma
b) Endodermal sinus cell tumour d) Endometriosis
c) Ovarian fibroma 871. Chronic treatment with tamoxifen can cause-
d) Teratoma a) Ca. ovary b) Ca. endometrium (AI 99)
861. Pain of ovarian carcinoma is referred to- c) Ca. cervix d) Germ cell tumor
a) Back of thigh (AIIMSMay 10) 872. GnRH analogue can be used for all except-
b) Gluteal region a) Fibroadenoma uterus (AI 99)
c) Anterior surface of thigh b) Cancer prostate
d) Medial surface of thigh c) Cancer endometrium·
862. The psendomyxoma peritonei occnrs as a d) Precocious puberty
complication of the following ovarian tumour-
873. Danazolis a- (CUP.GEE 01)
a) Serous cystadenoma (DPG 10}
a) Androgen derivative b) Oestrogen
b) Mucinous cystadenoma
c) Dysgerminoma c) Progesterone d) FSH derivative
d) Gonadoblastoma

852)a 853)b 854) a,b 855) b 856) c 857)a 858)a 859)c 860)a 861)d 862)b 863)a 864)a 865)a
866)a 867)d 868)b 869)c 870)c 87l)b 872)c 873)a
GYNAE & OBS [ 43]

874. Chronic treatment with tamoxifen can cause -(UP 2K) 887. In a patient with pelvic inflammatory disease due to
a) Cancer endometrium b) Cancer crevix tuberculosis, which of the following statements is true-
c) Cancer breast d) Cancer vagina a) Mycobacterium can be grown from menstrual blood
875. Commonest complication of cryotherapy is- (AI 95) b) Associated with infertility (PGI OJ)
a) Bleeding b) Persistent watery discharge c) Ectopic pregnancy is common
c) Pain d) Ulceration d) Dysmenorrhagia
876. Brachytherapy is used in- (PGI 2000) e) Associated with PID
a) Stage I b Ca Cx 888. TB endometritis causes infertility by- (PGI 98)
b) Ovarian Ca a) Causing anovulation
c) Stage IV Ca vagina
b) Destroying endometrium
d) Stage 2 fallopian tube Ca
c) Tubal blockage
877. All of the following are advantage of using Raloxifene
d) Ciliary dysmotility
over estrogen in post menopausal women.except-
889. Commonest cause of Pelvic Inflammatory Disease
a) Reduces fracture rates (AI 04)
b) Avoids endometrial hyperplasia is - (Kerala 03)
c) Reduces incidence of venous thrombosis a) 1B b)UTI
d) No increase in incidence ofbreast carcinoma c) Fungus d) Cu-T
878. Different routes ofHRT treatment include AlE- 890. Pelvic inflammatory disease is caused by all
a) Transdermal patch b) Vaginal pellets except- (MAHA 05)
c) Injectables d) Oral (MAHE 05) a) Schistosomiasis b) Crohn's disease
879. Estrogen administration in a menopausal woman c) Syphilis d) Leprosy
increases the- (AIIMS 06) 891. During laparoscopy the preferred site for
a) Gonadotrophin secretion b) LDL-cholesterol obtaining cultures in a patient with acute pelvic
c) Bone mass d) Muscle mass inflammatory disease is- (AIIMS May 05)
880. Drugs used for post menopausal symptoms include a) Endocervix b) Pouch of Douglas
the following except- (Karn pgmee 06) c) Endometrium d) Fallopian tubes
a) Conjugated estrogens b) Tibilone 892. Most common cause of pelvic inflammatory
c) Alendronate d) Danzol disease in virgin girls is - (UP 07)
881. Anti progesterone drug is- (Karn pgmee 06) a) Gonorrhoea b) Chlamydia
a) Cyproterone b) Mifepristone c) Treponema pallidum d) Tubercular
c) Spirinolactone d) Tomoxifen 893. In trichomonas- vaginalis- (UP 07, 05)
882. Oestrogen causes AlE- (MAHE 07) a) Greenish discharge
a) Weight gain b) White curdy discharge
b) Fluid and water retention c) Blood stained discharge
c) Disappearance of comedones
d) Milky discharge
d) Venous thrombosis
894. The commonest type of genital tuberculosis is-
883. Tamoxifen is- (UP 07)
a) Endosalpingitis b) Exosalpingitis (Corned 07)
a) Non steroidal antiprogesterone
b) Non steroidal antioestrogenic c) Endometritis d) Interstitial salpingitis
c) Synthetic progestogen norethindrone 895. True about genital infection in Bacterial vaginosis-
d) Competitive inhibitor of the Sa.- reductase a) Clue cells present (PGI Nov 09)
884. Side effect of clomephine citrate includes all except- b) Gardnerella is causative organism
a) Multiple pregnancy (AIIMS Nov 07) c) Pruritus seen
b) Increase risk of ovarian ca d) Grey & foul smelling discharge
c) Multiple polycystic ovary e) Erythema
d) Teratogenic effect on off springs 896. A homogenous white non-inflammatory discharge
that smoothly coats the vaginal walls and vestibules
PELVIC INFLAMMATORY DISEASES with positiveAmsel's criterion is characteristic of-
a) Candidaalbicans (UPSC-1 10)
885. Commonest organism causing Acute Salphingitis- b) Trichomonas vaginalis
a) Mycoplasma b) Chalmydia (AIIMS 92) c) Gardnerella vaginalis
c) Gonococal d) Treponema d) Niesseria gonarrhoea
886. The following is a characteristic symptom to 897. The commonest site of affection of genital
chronic pelvic inflammatry disease- (MP 98) tuberculosis is- (Karn 11)
a) Dysuria b) Constipation a) Cervix b) Endometrium
c) Backache d) Fever
c) Fallopian tubes d) Vagina

874)a 875)b 876)a 877)c 878)b 879)c 880)d 88l)b 882)None 883)b 884)d 885)c 886)c 887)a,b,c
888)b 889)d 890)d 89l)d 892)d 893)a 894)a 895) a,b,d 896) c 897) c
GYNAE & OBS [ 44]

ENDOMETRIOSIS 909. Post menopausal estrogen production is due to-


a) Peripheral aromatization of androstenedione
898. Which is True of endometriosis- (Jlpmer 91) b) Adrenal-Directproduction (PGI 2K)
a) Always associated with tubal blood c) Ovarian tumour
b) Painful d) Ovary testosterone secretion
c) Amenorrhoea 910. Drugs used in endometriosis include- (PGI 01)
d) Surgery is curative a) Testosterone b) Danazol
899. Which is not used in the treatment ofendometriosis- c) GnRH d) Progestogen
a) Danazol b)Tamoxifen (AI92)
e) Estrogen
c) Medroxprogesterone d) GNRH analogues
911. Endometriosis commonly associated with-
900. In endometriosis, cause ofinfertility is-
a) BIL chocolate cyst of ovary (PGI 02)
a) Immobilityoftubes (Jipmer95)
b) Adenomysis
b) Anovulation
c) Tubal block c) Fibroid uterus
d) Destruction of the ovary d) Luteal cyst
901. Symptoms of adenomyosis is- (Kerala 9 5) e) Endometritis
a) Menorragia b) Dysmenorrhia 912. Treatment of endometriosis included- (PGI 02)
c) Abdominal lump d) Infertility a) Estrogen b) Progesterone
902. Regarding extrauterine endometriosis false is - c) OCP d) Danazol
a) It is hormone dependent (AIIMS 94) e) GnRH
b) May involve pleura and lung 913. Which one of the following is the most common
c) Commonest site is ovary extrauterine site to be affected by endometriosis?
d) Ovary contains cyst with clear fluid a) Vagina (UPSC 02)
903. A 40 year old primiparous woman suspected to be b) Rectovaginal septum
suffering from endometriosis is subjected to c) Sigmoidcolon
diagnostic laparoscopic findings indicate. uterus d) Broad ligament (except tubes and ovaries)
normal both the ovaries show presence of chocolate 914. In the etiology of endometriosis Sampson's theory
cysts; endometriotic deposits are seen on the round is- (Jipmer 03)
ligament right side, both the faDopian tubes and the a) Implantation theory
pouch of Douglas; moderately dens adhesions are b) Coelomic metaplasia theory
present between the faDopian tubes and the pouch of c) Metastatic theory
Douglas. The treatment of choice in this case is - d) Histogenesis by induction
a) Total hysterectomy with bilateral salphingo 915. Medical treatment for endometriosis includes the
Oophorectomy (UPSC 97) following, except- (UPSC 04)
b) Danazol therapy a) Gn RH analogues b) Progesterones
c) Progesterone therapy c) Coticosteroids d) Danazol
d) Fulguration of endometriotic deposits 916. The severity of pelvic pain in endometriosis
904. Treatment of Endometriosis is all except- correlates best with- (UPSC 04)
a) Danazol b) MPA (AIIMS 97) a) Numberofimplants b) Depth of invasion
c) Gestrinone d) Tibalone c) Stage of disease d) Ca 125 levels
905. Treatment of adenomyosis- (AP 97) 917. Endometriosis is explained by- (SGPGI 05)
a) Estrogens a) Sampson's Implantation theory
b) Estrogens & Prosgeterone b) Metastatic epithelium
c) Total hysterectomy c) Histogensis by induction
d) Laser d) Coelomic metaplasia theory
906. Which of the following in the drug of choice to 918. Treatment of endometriosis in an infertile female-
treat endometriosis- (UPSC 2K) a) Danazol b) Clome (MAHE 05)
a) Testosterone propionate b) Norethisterone c) GnRH analogue d) Progesterone
c) Medroxy progesterone d)Danazol 919. Which one ofthe following sites of endometriosis
907. The best method for the diagnosis of enqometriosis is not treated by medical therapy- (UPSC 06)
is- (UPSC OJ) a) Recto sigmoid d) Ovarian
a) Laparoscopy b) Ultrasound c) Episiotomy d) Pulmonary
c) MRI d)SerumCA-125 920. Which one of the following drugs not used in
908. Pain in endometriosis correlates best with - endometriosis - (UPSC 07)
a) Depth b) Multiple sites (PGI 2K) a) Danazol b) Pregestins
c) iCa 125 d) Stage of disease c) GnRH agonisits d) Misoprostol

898)b 899) b 900) a,b,c 901) a,b 902) d 903) a 904) d 905) c 906) d 907) a 908) None 909) a,b 91 0) b,c,d
9ll)a 912)b,c,d,e 913)d 914)a 915)c 916)None 917)All 918)b 919)b 920)d
GYNAE & OBS [ 45]

921. Which one of the following conditions of the 931. Ward Mayo operation is done for- (HPU 01)
endometrium is associated with a significantly a) Vagina hysterectomy
increased risk of development of cancer? b) Abdominal hysterectomy
a) Adenomatous hyperplasia (UPSC-11 08) c) Perineal hysterectomy
b) Hyperplasia with atypia d) Both a and b
c) Cystic glandular hyperplasia 932. Laparoscopic cholecystectomy is best avoided in
d) Proliferative phase patient with- (UPSC 97)
922. Which one of the following is the definitive treatment a) Hypertension
of Adenomyosis? (UPSC-II 08) b) Diabetes
a) LN 6-Intrauterine device b) GnRH alalogues c) Obesity
c) Danazol d) Hysterectomy d) COPD (chronic obstructive pulmonary disease)
923. True about Endometrial carcinoma clinical stage m 933. Ureter is identified at operation by- (AIIMS 96)
disease - (PGI June 09) a) Rich arterial plexus
a) Vaginal metastasis b) Pulsatile movement
b) ParaaorticL.N.involvement c) Relation to lumbar plexus
c) Pelvic LN. involvement d) Accompanied by renal vein
d) Peritoneal lavage is done 934. Mini laparotomy is done in all except-(CUPGEE OJ)
e) InguinalL.N. involvement a) Ectopic pregnancy b) Internal sterilisation
924. Which of the following is most common extra uterine c) Uterus elevations d) Tubectomy
site to be affected by endometriosis ? 935. In colposcopy, following are visualized except-
a) Vagina (DELHIPGFeb. 09) a) Upper 213rc1 endocervix (PGI 97)
b) Rectovaginal septum b) Lower 1/3'd endocervix
c) Sigmoidcolon c) Vault of vagina
d) Broad ligament (except tubes and ovaries) d) Lateral fornix
925. The sites of endometriosis in order of frequency 936. Gas most commonly used in laparoscopy is-
are- (Corned 10)
a) C02 b) S02 (PGI 97)
a) Umbilicus, vulva, rectovaginal septum
c) N2 d)02
b) Pelvic peritoneum, ovaries, laparotomy scar
937. In suction evacution, the pressure applied is-
c) Vagina, appendix, uterine ligaments
a) 0-11 mgHg b)200-400mmHg (PGI98)
d) Ovaries, uterine ligaments, rectovaginal septum
926. The most common site of endometriosis is- c) 400-600mgHg d)600-800mmHg
a) Ovary (Karn 11) 938. 'Kelly's stitch' operation is done for-
b) Pouch of Douglas a) Stress incontinence (CUPGEE 2K)
c) Recto vaginal septum b) Urge incontinence
d) Sigmoid colon c) Overflow incontinence
927. Progesterone causes - (Jipmer 11) d) Neurogenic bladder
a) Myometrial quiescence 939. The intra- abdominal pressure laparoscopy should
b) Myometrial activation be set between- (AIIMS 03)
c) Increase in oxytocin receptors a) 5-8mmofHg b)10-15mmofHg
d) Causes cervical ripening c) 20-25 mm ofHg d) 30- 35 mm ofHg
940. Which surgical procedure has the highest
OPERATIVE GYNAECOLOGY incidence of ureteric injury? (AI 06)
a) Vaginal hysterectomy
928. D & C is useful in the diagnosis of- (Kerala 94) b) Abdominal hysterectomy
a) Genital TB b) Endometrial carcinoma c) Wertheim's hyseterectomy
c) DUB d)All d) Anterior colporraphy
929. Kelly's suture is done in- (Calcutta 2K)
941. What is the most common complication after
a) Stress incontinence radical hysterectomy- (UPSC 06)
b) Cervical incontinence a) Ureteral injury b) Bladder atony
c) Genito-urinaryprolapse c) Prolapse of vaginal vault d) Bowel dysfunction
d) Vaginoplasty 942. An are indications for Hysterectomy except-
930. Trans-cervical endometrial resection (TER) is a) Couvelaire Uterus (MAHE 07)
usedinAJE- (PGI 99) b) Rupture uterus
a) Endometriosis b)DUB
c) PPH not contolled by ligation of internal iliac artery
c) Carcinoma endometrium d) Submucous fibroid d) Twin pregnancy

921)b 922)d 923)a,b,c,d 924)d 925)d 926)a 927)a 928)d 929)a 930)a,c,d 931)b 932)c 933)b 934)a
935)a 936)a 937)c 938)a 939)b 940)c 941)a 942)a,d
GYNAE & OBS [ 46]

943. An are indications for hysterectomy except- 955. The condition of genital organs which may lead
a) Couvelaire uterus (Manipal 08) to lesions in surgical scars, rectum, lymph nodes,
b) Rupture uterus lung is- (AIIMS 95)
c) PPH not controlled by ligation of internal iliac artery a) Teratoma b) Endometrosis
d) None of the above c) Adenomyosis d) Fibroid
944. The ideal distension medium for operative 956. Inheritence is a major factor in- (CUPGEE 96)
hysteroscopy using electrocautery is -(UP SC Ill 0) a) Endometrial Ca b) Cervical Ca
a) C02 b) N-saline c) Both d) Neither
c) 5% dextrose saline d) 1.5% glycine 957. Diabetes predisposes to all except- (AP 96)
a) Moniliasis b) Trichomoniasis
MISCELLANEOUS (GYNAE) c) Chlamydia d) None
958. Nabothian follicle is due to- (TN97)
945. Fixed pelvic tumour occurs in- (TN 91) a) Inflannnation of cervical glands
a) Malignancy b) Bartholin gland infection
b) Endometriosis c) Infection of sweat gland
c) Growth extension into ligamenta d) None of the above
d) All of the above 959. Ovarian function after hysterectomy- (UPSC 97)
946. Systemic metastasis is commonest in- (AIIMS 91) a) Increases b) Decreases within2-3 years
a) Ovarian Carcinoma c) Remains the same d) Decreases after 5 years
b) Endometrial Carcinoma 960. False statement regarding carcinoma of fallopian
c) Choriocarcinoma tubes- (TN 98)
d) Carcinoma cervix
a) Adenocarcinoma
947. A 65 year old with bleeding PN. On examination
b) Veryrare
senile vaginits. Patient requires immediate-
c) Occurs in multiparous women {50-60 years)
a) Cytology and colposcopy (UPSC 89,90)
d) Constitues about 10-20 % malignancies
b) Oestrogen therapy and colposcopy
c) Cytology and fractional curettage 961. One of the following is a congenital cyst- (AP 97)
d) Fractional curettage a) Inclusion. cyst b) Ovarian cyst
948. The incidence of stump carcinoma is- (AJIMS 84, c) Gartner's cyst d) None
a) 6% b) 10% DNB 90) 962. Estrogen dependent conditions are all except-(MAHE
c) 16% d)20% a) Endometriosis b) Endometrial carcinoma 98)
949. The colour of vulva in Paget's disease is- c) Fibroids d) Ca. cervix
a) Blue b)White (AIIMS79,DNB91) 963. Sub urethral metastasis is seen in - (MP 2K)
c) Red d)Yellow a) Carcinomacervix
e) Black b) Carcinoma endometrium
950. Lesion in female child born to a mother treated c) Choriocarcinoma
with stilbesterol is- (KERALA 94) d) Vaginal carcinoma
a) Vaginal adenosis 964. Gynaecological cancer unusual in young
b) Sarcoma Botyroides women- (ORR/SA 98)
c) Cerviacal erosion a) Cancer ovary b) Cancer cervix
d) Carcinoma endometrium c) Cancer vulva d) Choriocarcinoma
951. In perimenopausal women with menorrhagia we 965. FIGOstagingisusedin- (ORR/SA 98)
ruleoutcarcinomaof- (JIPMER 95) a) Carcinoma breast b) Bone tumour
a) Ovary b) Uterus c) Leukaemia d) Carcinoma cervix
c) Fallopian tube d) Endometrium 966. Ball's operation is done for- (ORR/SA R)
952. In differential diagnosis of lump in right fornix a) Ca Cx b) Ca. ovary
whichisnottrue- (JIPMER 95) c) Ca. Fallopian tube d) Pruritus vulvae
a) Ovarian cyst b) Hydro salpinx 967. Characteristic feature of Cancer fallopian
c) Appenicular mass d) Sub mucosal fibroid tube- (MAHE 2001)
953. Post menopausal bleeding is most commonly due a) Watery discharge PN b) Hemorrhage
to- (KERALA 95) c) Pain d) Sepsis
a) Carcinoma cervix b) Fibroid 968. Powder burn appearance on laparoscopy is
c) Endometriosis d) Genital TB characteristic of- (FN 2002)
954. All of the following tumour metastatize to ovary a) Endometriosis
except- (DELHI 96) b) Endometroid tumour
a) Gastric carcinoma b) Colonic carcinoma c) Epithelial ovarian tumour
c) Cervical carcinoma d) Breast carcinoma d) Endometrial cancer

943)a 944)d 945)d 946)c 947)a 948)b 949)b 950)a 95l)a 952)d 953)None 954)None 955)b
956)a 957) b,c 958) a 959)b 960) c,d 961) c 962)d 963)b 964)c 965) d 966) d 967) a 968) a
GYNAE & OBS [ 47]

969. Most common cause of acute cervicitis is- 982. A 47 year old woman complains of postcoital
a) Streptococcus b) Gonococcus (BHU 87) bleeding, nearly as heavy as menses. The most
c) E. coli d) Proteus likely origin of her bleeding would be cervical-
970. Mucin producing glands are seen in all except- a) Polyps b) Ectropion (Kerala 04)
a) Vagina b) Oesophagus (Al 98) c) Carcinoma d) Nabothian cysts
c) Cervix d) Duodenum 983. True about galactorrhoea- (PGI June 05)
971. Treatment of gynaecomastia is AlE- (PGI 98) a) Found in pregnancy & lactation
a) Hormonal therapy b) Always BIL
b) Simple mastectomy c) Surgery done
c) Subcutaneous liposuction mastectomy d) AIW adrenal gland tumor
d) None
984. All of the following appear to decrease hot :flushes
972. Gyanecomazia is caused by- (PGI 98)
in menopausal women except- · (AI 05)
a) Kallman's syndrome
a) Androgens b)Raloxifene
b) 5-a reductase deficiency
c) Androgen insensitivity c) Isoflavones d) Tibolone
d) All of above 985.. Me Donald stitch is applied in the following
e) None conditions except- (UPSC 07)
973. M.C. vaginal carcinoma is- (PG/99) a) Incompetent os b) Septate uterus ·
a) Squamous cell Ca b) Adeno carcinoma c) Placenta previa d) Bad obstetrical history
c) Botroid's tumor d) Columnar hyperplasia 986. Goniometer is used for- (AIIMS Nov 07)
974. A 35 year old woman with dysmenorrhea and a) Amount of vaginal secretions
menorrhagia of 6 months duration showed an b) To measure width of genital hiatus
enlarged uterus of 20 weeks which was tender, c) Gonococcal colony count
the possible diagnosis is- (PGI 99) d) Urethrovesical angle
a) Adenomyosis b) Fibroid 987. Surgical staging is done for the following genital
c) Carcinoma endometrium d) PID malignancies except- (UPSC-11 08)
975. 40 yrs female, mass in pelvis detected clinically, a) Ovarian malignancy
following investigations shonld be done except- b) Gestational trophoblastic neoplasm
a) Cf b) Laparoscopy (PGI 2K) c) Endometrial carcinoma
c) PAP smear d)USG d) Fallopian tube malignancy
976. All are true regarding Sarcoma Botryoides except- 988. Human Chorionic Gonadotropin (bCG) monitoring
a) Seen in vagina (PGI OJ) is helpful for- (PGI Dec 08)
b) Grapelike clusters are seen a) Molar pregnancy b) Multiple pregnancy
c) Seen in elderly women
c) Ectopic pregnancy d) Twin pregnancy
d) It is an adenocarcinoma
e) Choriocarcinoma
e) Farnillial incidence is common
989. o:. Fetoprotein concentration in serum is elevated in-
977. Gynaecomastia is seen in- (PGI 02)
a) Hepatoblastoma (PGI Dec 08)
a) Secondary syphillis
b) Lepromatous leprosy b) Hepatoma
c) HlV c) Endodermal sinus tumour
978. True about CA vulva assoiciatedJpredisposed by- d) Gastrointestinal cancer
a) Paget'disease (PG/02) e) Cirrhosis
b) Vulval intraepithelia Neoplasis 990. Which gynaecological malignancy is caused by mv
c) Bowen's disease infection- (PGI Dec 08)
979. True regarding postmenopausal osteoporosis is- a) CaCervix b)CaEndometrium
a) Decreased vitamin D (PGI 02) c) Ca Ovary d) Ca Vagina
b) Decreased serum calcium e) CaBreast
c) Normal serum chemistries 991. A 25-year-old patient who has had an abortion fourt
d) Decreased vitamin C months ago has come with the history of profuse
e) Amenorrhea vaginal bleeding. On examination. Uterus is bulky,
980. Weiner- Reid's test is used in- (Manipal 04) both the ovaries are enlarged, pregnancy test is
a) Abruptio placenta b) Placenta previa positive. What is the probable clinical diagnosis?
c) Gestational diabetes d) None a) Incomplete abortion (UPSC-I! 09)
981. Pseudomeig syndrome seen with- (Manipal 04) b) Choriocarcinoma
a) Fibroids b) Adenomyosis c) Malignant ovarian tumor
c) Ovarian tumour d) Endometriosis d) Ectopic pregnancy

969)b 970)a 971)d 972)c 973)a 974)a 975) c 976) c,d,e 977) b,c 978) a,b,c 979) c,e 980) a 98l)c
982)c 983)c 984)b 985)b 986)d 987)b 988)All 989)All 990) a 991) b
GYNAE & OBS [ 48]

992. True statement(s) regarding vulval carcinoma is/ OBSTETRICS


are- (PGI Nov 09)
OBSTETRICAL ANATOMY
a) Squamous cell hyperplasia predisposes
b) Paget's disease of vulva predisposes 1001. The Obliterated umbilical vein is called-
c) Lichen sclerosus is a risk factor a) Ligamentum teres (AIIMS 79, Bihar 90)
d) Condylomata accuminate predisposes b) Urachus
e) Vulval dystrophy cause invasive lesion c) Ligamentum arterisoum
993. A lady with placenta previa delivered a baby. She had d) Falciform ligament
excessive bleeding and shock. After resuscitation 1002. The chromosomal complement of a primary
most likely complication would be- (AIIMS May 10) oocyte is - (KARNATAK 94)
a) Galactorrhoea b) Diabetes insipidus a) 23X b)23Y
c) Loss ofmenstruation d) Cushing's syndrome c) 46XX d)46XY
994. Clomphene citrate- True statement is- 1003. The second maturation divison of the human
a) Enclomiphene is anti-estrogenic (AJIMS May 10) ovum occurs at the time of- (KARN 94)
b) Increases pregnancy rate 3 times as compared to a) Fertilisation b) Implantation
placebo c) Ovulation d) Puberty
c) Incidence of twin pregnancy is 5-6% 1004. Umbilical artery pressure is- (UPSC 96)
d) It has been shown to increase fertility in a) 30to40mmHg b)60mmHg
oligospennic males in randomized controlled trials c) lOOto 120mmHg d)Greaterthan 120mmHg
995. Sentinel lymph node biopsy is most useful for which 1005. Inplantation of fertilised ovum occurs- (TN 97)
of the following gynecological malignancies-(AI 10) a) 24 hrs after entry into uterine cavity
a) Carcinoma Endometrium b) Carcinoma cervix b) 3-4 days after into uterine cavity
c) Carcinoma vulva d) Carcinoma vagina c) 11 days after entry into uterine cavity
996. Inlaparoscopy,chancesofhypercarbiacan be reduced d) None of the above
by keeping intraperitoneal C02 pressure- 1006. Blighted ovum is characterised by- (AI 98)
a) 10 b) 12 (Maharashtra 10) a) Avasular villi b) Synctial knot
c) 13 d) 14 c) Hypertrophied villi d) Absent trophoblast
997. A 16-year-old girl presents with primary 1007. Oogonia are derived from- (JIPMER 92)
amenorrhoea and repeated periodic pain. On a) Amnion b)Yolksac
examination, a suprapubic mass is felt up to the c) Stroma of ovary d) Germinal epithlium
umbilicus. The most likely diagnosis is - 1008. Weight of normal uterus is- (BIHAR 90)
a) Large ovarian cyst (UP SCIIl 0) a) 30-45 gms b)45-60 gms
b) Cryptomenorrhoea c) 60-70gm d) 80-100 gm
c) Bladder-neckhypertrophy 1009. Chromosomal no. of primary spermatocyte is-
d) Uterine leiomyoma a) 46XY b)22XY (PGI99)
998. Clomiphene citrate- true statement is- c) 22XX d)46XX
a) Enclomiphene is anti-estrogenic (AJIMS May 11) 1010. Female pelvis as compared to the male pelvis bas
b) Increases pregnancy rate 3 times as compared to allexcept- (PGI01)
placebo a) Narrow sciatic notch
c) Incidence of twin pregnancy is 5-6% b) Shallow and wide symphysis pubis
d) It has been shown to increase fertility in c) Suprapubic angle is acute
oligospennic males in randomized controlled trials d) Light and graceful structure
999. Advantage of anastrozole over the other aromatase e) Pre auricular sulcus is larger
inhibitors is - (Kam 11) 1011. Primary oocyte- (PGI 02)
a) Causes less gastro-intestinal side effects a) Is formed after single meiotic division
b) Causes less osteoporosis b) Maximum in number at 5th month fetus
c) Has more anti-estrogenic activity c) Is in propb.ase arrest
d) Has more anti-progestonic activity d) Also k/a blastocyst
1000. Fibrocystic disease is due to high levels of- 1012. Zygote reaches the uterine cavity as- (Kerala 03)
a) 32 celled b) 16 celled
a) Estrogen b) Progesterone (Jipmer 11)
c) 8 celled d) 2. celled
c) IR d) Testosterone
1013. Non stress test is contraindicated in- (Orissa 04)
a) IUGR b) Post maturity
c) Rh incompatibility d) PIH .

992)All 993)c 994)a 995)c 996)b 997)b 998)a 999)b lOOO)a 100l)a 1002)c 1003)a 1004)b 1005)d
1006)a 1007)b 1008)b 1009)a lOIO)a,c lOll)b,c 1012)b 1013)c
GYNAE & OBS [ 49]

1014. The formation of primordial follicles in human 1027. All are fetal membranes except- (TN03)
fetus is completed by- (Karnataka 04) ~ Chorion b)Amnion
a) 4 weeks b) 8 weeks c) Decidua capsularis d) Yolk sac
c) 13 weeks d) 18 weeks 1028. Motile spermatozoa found on a wet mount of
1015. Ligamentum teres is formed after- (COMED 06) vaginal secretions are indicative of intercourse
a) Obliteration of the umbilical vein within the past- (Karnataka 03)
b) Obliteration of the ductus venous a) 6 hours b) 12 hours
c) Obliteration of the ductus arteriosus
c) 24 hours d) 48 hours
d) Obliteration of the hypogastric artery
1029. Amniotic fluid at 38 weeks in normal pregnancy
1016. Living ligature ofthe uterus is- (Maharashtra 10)
a) Endometrium is - (UPSC 04)
b) Middle layer of myometrium a) 800cc b) llOOcc
c) Inner layer of myometrium c) 1500cc d) 1800cc
d) Parametrium 1030. Thickened placenta is not seen in- (Jharkand 03)
1017. The umbilical cord normally contains - a) Triploidy b) Syphilis
a) Two arteries and two veins (UP SC II 10) c) Rh incompatibility d) Intrauterine infections
b) One artery and one vein 1031. What is the pH range of amniotic fluid-
c) Two arteries and one vein a) 5.5-6.0 b) 6.0-6.5 (UPSC 06)
d) One artery and two veins c) 6.5-7.0 d)7.0-7.5
1032. By which day after fertilization, is placental
PLACENTA & FETAL MEMBRANE circulation established- (UPSC 06)
a) 11th day b) 13th day
1018. Golden colour amniotic fluid is seen in- (AP 96) c) 15th day d) 17th day
a) Rh incompatability b) Foetal death
1033. The foetal blood is separated from the
c) IUGR d) Foetal distress
syncytiotrophoblast with all except- (UP 07)
1019. Weightofnormalplacentais- (A/96)
a) 250gm b) 500 gm a) Foetal blood capillary membrane
c) 125 gm d) 350 gm b) Mesenchyme inter villous blood space
1020. Weight of placents ......... % offoetus- (CMC 98) c) Cytotrophoblast
a) 10 b)20 d) Decidua- parietalis
c) 30 d)45 1034. The weight of placenta atterm is- (DPGEE 08)
e) 50 a) 500gm b)250gm
1021. During the uterine contraction of labour, the c) lOOOgm d)750gm
uterine blood flow- (AMU 87) 1035. Match list I (Abnormality in the fetomatemal unit in
a) Decreases b) Increases placenta) with list ll (Coqdition caused/etiological
c) Does not change d) Temporarily cases effect) and select the correct answer using the codes
1022. Saffron coloured meconium is seen in- (TN 98) given below the lists- (UPSC-l/ 09)
a) Post maturity b) TB List I List ll (Conditions
c) Breech d) Normal in appearance (Abnormality in the caused/etiological
1023. The amniotic fluid is in balance by- (PGI OJ) fetomatemal unit in effect)
a) Excretion by renal kidneys
placenta)
b) Maternal homeostasis
A. Absence of 1. Increased exchange of
c) Fetal intestinal absorption
d) Fetal membrane absorption Nitabuch's layer nutrients
e) Fetal sweating B. Absence of 2. Adherent placenta
1024. Trophoblast give rise to- (PGI03) secondary invastion
a) Placenta b) Chorion of trophoblast & PIH
c) Amnion d) Decidua C. Thinning of 3. Vasospasm
e) Fetal limb cytotrophoblasts
1025. Oxygenated blood from plecenta to heart in utero D. Future ofbody stalk 4. Umbilical cord
is by- (PGI 97) Codes:
a) Umbilical vein b)NC A B C D
c) Ductus arteriosus d) None of the above a) 2 3 4
1026. The thickness of endometrium at the time of b) 2 3 4
implantation is- (PGI 99) c) 4 3 2
a) 3-4mm b)20-30mm d) 4 3 2
c) 15-20mm d)30-40mm

1014)b 1015)a 1016)b 1017)c 1018)a 1019)b 1020)b 102l)a 1022)a 1023)a,b,c,d 1024)a,b,c 1025)a 1026)a
1027)d 1028)c 1029)a 1030)a 1031)d 1032)d 1033)d 1034)a 1035)b
GYNAE & OBS [50]

1036. Which of the following embryonic structnres is 1047. Which of the following is increased in pregnancy~
responsible for secretion of human chorionic a) Globulin b) Fibrinogen (PG1 OJ)
gonadotrophin which could be detected in the maternal c) Uric acid d) Leukocytes
urine during early pregnancy? (Karn JJ) e) Transferrin
a) Amnion b) Embryobjast 1048. Insulin resistance in pregnancy is because of-
c) Epiblast d) Syncytiotrophoblast a) Human placental lactogen (PG1 OJ)
103 7. Fetns is called an embryo until- (Jipmer 11) b) Thyroid hormone
a) 6 weeks b) 8 weeks c) Progesterone
c) 10 weeks d) 12 weeks d)hCG
e) Estrogen
PHYSIOLOGICAL CHANGES DURING 1049. Physiological changes of pregnancy ofinclude-
PREGNANCY a) Insulin leves increase (PG1 02)
b) Increased BMR
1038. How many days after ovulation is placental c) Hypothyroidism
circulation establishes - (DNB 9J) d) GH decreases
a) 18to21 b)9to 11 e) Blood volume decreases
c) 7to9 d)30to 13 1050. Regarding idiopathic cbolestasis of pregnancy-
e) 40to43 a) Deep jaundice is present (PG1 02)
1039. Which is not increased in pregnancy- (UP 96) b) Pruritus is the first symtom
a) Vital capacity b) Blood volume c) Maximum incidence during Ill trimester
c) Extracellularfluid · d) Weight d) Increased liver transaminase
1040. Increased calories required during pregnancy- e) Hepatic necrosis present
a) 300 b) 550 (A11MS 97) 1051. What are maternal physiological changes in
c) 800 d)400 pregenancy- (PGI 03)
a) ied cardiac output b) ied tidal volume
1041. Blood congulation prome in pregnacy is-
c) i ed vital capacity d) ted fibrinogen
a) Increase in fibrinogen level of 10~25%
e) ted plasma protein concentration
b) Decrease in factor X, XI, XII (AI1MS 96)
1052. Signs of heart disease in pregnancy- (PG1 03)
c) Decrease in plasminogen activity
a) Diastolic murmur
d) Increase in platelet count b) Systolic murmur
1042. Which of the following is truly physiological in c) Tachycardia
pregnancy- (PGI 93) d) Dyspnea on exertion
a) Albuminuria b) Increased BP e) Nervousness or syncope on exertion
c) Mild pedal edema d) Increased GFR 1053. In pregnancy which is abnormal fmding?
1043. In pregnancy- (Delhi 92) a) Venous hum (Jipmer 03)
a) Plasma fibrinogen levels are increased b) Third heat sound
b) Fibrinogen levels are decreased c) Diastolic murmur
c) Thyroglobulins are decreased d) Supra- clavicular murmur
d) IgD are markedly increased 1054. During pregnancy there is an increased
1044. Which of the following cardiovascular change is respiratory sensitivity to Carbon dioxide due to
abnormalinpregnancy- (PGI 2000) higher circulating levels of- (Karnataka 04)
a) Enlarged cardiac shilhoute a) Progesterone b) Estrogen
b) Increased Slsplit c) Estriol d) Prolactin
c) Right axis deviation ofECG 1055. Maximum cardiac output during pregnancy is
d) Early diastoic murmur seen in - (AI 07)
e) HR increased by 10 to 15 per minute a) 1st trimester b) lind trimester
1045. True about various changes in pregnancy is/true c) Labour d) Immediate postpartum period
a) Fibrinogen levels are increased (PG1 2000) 1056. A pregnant woman in ill (third) trimester has normal
b) Uric acid levels are increased blood pressure when standing and sitting. When
c) Sr. potassium is decreased supine, blood pressure drops to 90/50. What is the
d) Sodium retention diagnosis ? (UPSC-11 08)
1046. Most common cause of platelet .l-in pregnancy- a) Compression of uterine artery
a) Immune b) Incidental (PG1 2000) b) Compression of aorta
c) Idiopathic d) Infection c) Compression ofiVC (Inferior Vena Cava)
e) Benign Gestational d) Compression of internal iliac vessels

1036)d 1037)b 1038)a 1039)a 1040)a l04l)a 1042)c,d 1043)a 1044)c,d 1045)a,d 1046)e 1047)a,b,d,e
1048) a,b,c,e 1049) a,b 1050) b,c,d 1051) a,b,e 1052) a,d,e 1053) d 1054) a 1055) d 1056) c
GYNAE & OBS [51 ]

1057. Which clotting factors decreases during pregnancy- 1067. Fetal growth is maximally influenced by- (Al99)
a) 1 &3 b) 11 & 13 (COMED09) a) Insulin b)GH
c) 10 & 13 d) 7 & 8 c) Thyroxine d) Cortisol
1058. Haemodyuamic changes in pregnancy include the 1068. HCGdoublesevery days- (Kerala200J)
foUowing except- (UPSC-/1 09) a) 2 b) 6
a) Increase in cardiac output c) 8 d)4
b) Increase in heart rate 1069. The hormone responsible for the decidual and
c) Decrease in vascular resistance atrias stella reaction of ectopic pregnancy is -
d) Increase in mean arterial pressure a) HCG b) Progesterone (Kerala OJ)
1059. Uterinebloodflowattermis- (Al/MSNov09) c) Estrogen d) HPL
a) 50-70rnl/min b) 100-150rnl/min 1070. The role of human placentallactogen is- (Al92)
c) 175-200rnl/min d) 500-750rnl/min a) Stimulate milk production
1060. In pregnancy, there is a physiological increase of b) Fetal breast development
the- (UP SCI/ JO) c) Growth of featus
a) Blood pressure in the third trimester d) Endocrine regulation
b) Blood viscosity 1071. Hormone which does not cross placenta-(MA.HE 0J)
c) Cardiac output a) Thyroxine b) Oestrogen
d) Peripheral resistance of the blood vessels c) Insulin d) None
1061. True statement regarding cholestasis in pregnancy- 1072. Hormonal assay in human is commonly doneby
a) Recurrs in subsequent pregnancy (PG/ May J 0) using- (PGI 98)
b) Ursodeoxyholic acid relieves pruritus a) RIA b) Densiometry
c) Mild jaundice occurs in majority of patients c) Flurometry d)HPLC
d) Pruritus may precedes laboratory findings 1073. Raised beta-HCG levels are seen in- (PG/ OJ)
e) Serum alkaline phosphatase is most sensitive a) DM b) Preeclampsia
indicator c) Ectopic pregnancy d) Rh. incompatibility
1062. True regarding changes during pregnancy? e) Down synd.
a) Hyperplasia of parathyroid (PGI May J 0) 1074. Following hormones secreted by placenta
b) Hyperplasia of thyroid exclnssively- (PGI 02)
c) Increased Pigmentation a) HCG b) Estrogen
d) iBMR c) HPL d)PRL
e) i Insulin 1075. RegardinghCG- (PG/02)
1063. Which of the following is present normally in the a) A glycoprotein
urine of a pregnant women in the third trimester- b) Has 2 subunits
a) Glucose b) Lactose (Al/MS Nov J 0)
c) Increases to maximum in ill trimester
c) Galactose d) Fructose
d) Secreted by trophoblast
1064. Which of the following haemotological parameters
e) Alfa subunit is specific
does not tindergo a physiological increase during
1076. Which is not secreated by placenta? (Karala 03)
normal pregnancy- (UPSC II 11)
a) Blood volume b) Red celi'volume a) HPL b)HCG
c) Leukocyte count d) Platelet count c) Prolactin d) Progesterone
1065. Which of the following regarding CVS changes in 1077. a and psubunits are not seen in- (Kerala 03)
pregnancy is false? (Jipmer 11) a) FSH b)HCG
a) Blood volume increases c) Prolactin d) Insulin
b) Heart rate increases 1078. hCGissecreted by- (AIJMS06)
c) Cardiac output increases a) Trophoblast cells b)Amniotic membrane
d) Blood viscosity increases c) Fetal yolk sac d) Hypothalamus
1079. HCGsecretedfrom- (PGI June 08)
ENDOCRINOLOGY IN RELATION TO a) Placenta b) Pancreas
REPRODUCTION c) Lung d) Liver
e) Breast
1066. In early pregnancy, level of HCG in plasma 1080. In early pregnancy the doubling time of the
doublesevery- (DNB9J) concentration of human chorionic gonadotropin in
a) 2 days b) 4 days plasma is- (Corned J0)
c) Weekly d) Only gradually rises a) 1 day b) 2 days
c) 3 days d) 4 days

1057) b 1058) d 1059) d 1060) c 1061) a,b,c,d 1062) a,b,c,d 1063) a 1064) d 1065) d 1066) a 1067) a 1068) a
1069)b 1070)c 107l)c 1072)a 1073)e 1074)a,b,c 1075)a,b,d 1076)c 1077)c 1078)a 1079)a 1080)b
GYNAE & OBS [52]

1081. Progesterone is associated with the following except- 1093. Signs positive in early pregnancy are- (PGI 2000)
a) Breast swelling (UPSC I 11) a) Hegar's sign b) Palmer's sign
b) Increased myometrial contractility c) Goodell's sign d) Osiander's sign
c) Thermogenesis 1094. In transvaginal ultrasound, earliest detection of
d) Endometrial proliferation gestation sac is by- (PGI 2000)
1082. Which of the following hormones is the best a) 21 days after ovulation
indicator of maternal-foetal- placental unit- b) 21 days after implantation
a) Prolactin (UPSC II 11) c) 28 days post ovulation
b) Human placentallactogen d) 14 days after ovulation
c) Progesterone
1095. Earliest detection of pregnancy by ultrasound is
d) Estriol
by- (PGI2000)
DIAGNOSIS OF PREGNANCY a) Gestation sac b) Fetal node
c) FHS d) Fetal skeleton
1083. At what period, cystitis (Frequency) is present in 1096. Fetal heart acitivity can be detected by sonography
pregnancy- (DNB 90) atabout- (PGI02)
a) Before 16 weeks b) After 18 weeks a) 5 weeks b) 6 weeks
c) After 20 weeks d) After 24 weeks c) 7 weeks d) 8 weeks
1084. Urine is collected for examination in a pregnant e) 9weeks
femaleby- (PGI90) 1097. Changes that are found in 2nd trimester of
a) Mid stream collection b) Suprapubic puncture pregnancy- (PGI 03)
c) Catheterisation d) Early morning sample a) Braxton-Hicks contraction b) Show
1085. Definitive sign of pregnancy- (Al95) c) Lightening d) Quickening
a) Uterine touffle b) Ballottment e) Broad ligament pain
c) Fetal heart sound d) Amenorrhea 1098. Pregnancyisconfirmedby- (PGI04)
1086. Hegar sign can be elicited in- (Assam 95) a) Morning sickness
a) Early pregnancy b) Late pregnancy b) Amenorrhoea
c) During labour d) During peurperium c) Foeatal heart activity
1087. Palmer's sign is pregnancy refers to (8-12 weeks)- d) Foetal movement by examiner
a) Rhythmic contractions of uterus (AP 96) e) Foetal sac in USG
b) Pulsations is the lat. fornix 1099. Not a sign of early pregnancy- (Manipal 06)
c) Discoloration of vagina
a) Lemon's sign b) Osiander's sign
d) None
c) Piskacek's sign d) Palmer's sign
1088. All are positive signs of pregnancy except- (FN 98)
1100. The sorting of the uterus with lateral implantation
a) Vomiting b) Fetal movements
c) Fetal heart sounds d)Excessive salivation is known as- (UP 07)
1089. The earliest diagnostic test of pregnancy- (FN 98) a) Chadwick's sign b) Hegar's sign
a) Ultrasound b)BetaHCG c) Goodell's sign d) Piskacek's sign
c) Fetal movements d) Fetal heart sounds 1101. Which one of the following is not a sign of early
1090. Osiander's sign means- (JIPMER2K) pregnancy- (UPSC 07)
a) Pulsation in the lateral vaginal fornix a) Goodell's sign b) Hegar's sign
b) Bluish colour change in the vagina c) Cullen's sign d) Palmer sign
c) Softening of the cervix 1102. What is the gestationallage of which the maternal
d) On bimanual palpation the finger can be hCG levels are maximum in a normal pregnancy-
approximated as if nothing is in between a) 8 to 10weeks b) 12 to 14weeks (UPSC07)
1091. Palmer's sign elicits- (Karn 99) c) 16to18weeks d)After20weeks
a) Intermittent uterine contractions 1103. Expected date ofdelivery is calculated by all except-
b) Softening of cervix a) Nine calender months plus 7 days (UP 08)
c) Pulsations in fornix b) 280 days or 40 weeks
d) Compresibility of isthmus c) 266 days or 38 weeks
1092. In early pregnancy clinical signs of feeling the d) 10 lunar months
cervix and the body of bulky uterus separated 1104. Which ofthe following measured by clinically-
because of softened isthumus at 6-10 weeks of a) True conjugate (UP08)
gestation- (Orrisa R) b) Obstetric conjugate
a) Goodell's sign b) Chadwick's sign c) Diagonal conjugate
c) Piskacek's sign d) Hegar's sign
d) Transverse diameter

1081)b 1082)d 1083)a 1084)d 1085)c 1086)a 1087)a 1088)d 1089)b 1090)a 1091)a 1092)d 1093)a,b,c,d
1094)a 1095)a 1096)c 1097)a,d 1098)c,d,e 1099)a llOO)d llOl)c 1102)a 1103)c 1104)c
GYNAE & OBS [53]

1105. Which of the following statmentis untrue regarding c) Lower border of symphysis pubis and the third
fundal height- (UP 08) piece of sacrum
a) Fundal height between pubic symphysis and d) Lower border of symphysis pubis and the lip of
umbilicus at 16 weeks sacrum
b) Fundal height atthe level of umbilicus at 20 weeks 1113. Posterior fontanelles ossify at the age of-
c) Fundal height at the level of ensiform cartilage at a) 1 year b) 2 year (HPU OJ)
36weeks c) 3 year d) 4 year
d) Fundal height ofthe level ofxiphisternum at 40 weeks 1114. Commonest of non-engagement at term, in primi
1106. Quickening is felt at- (PGI Nov 09)
is- ~m~
a) 8-12wk b) 16-20wk
a) CPD b) Hydramnios
c) 20-24wk d)24-28wk
c) Brow presentation d) Breeach
e) 28-32wk
1115. Engaging diameter of face presentation- (PGI 2K)
1107. Chadwick sign means- (Maharashtra 10)
a) Bluish discoloration or dusky hue of the vestibule a) Submentovertical b) Submentobregmatic
and anterior vaginal wall c) Mentovertical d) Suboccipitofrontal
b) Pulsations felt through lateral fornices 1116. Diameter of engagement in face presentation-
c) Softening of cervix a) Mentovertical (PGI 02)
d) All of the above b) Submentovertical
1108. Match List-1 with List-11 and select the correct c) Suboccipitobregmatic
answer using the code given below the Lists- d) Submentrobregmatic
List-1 (Changes List-IT (Site of e) Suboccipitovertical
of pregnancy) change) 1117. Most important diameter of pelvis during labour
A. Jacquemier's sign 1. Breast is- (PGI02)
B. Chloasma gravidarum 2. Abdomen a) Interspinal diameter of outlet
C Striae gravidarum 3. Uterus b) Oblique diameter of inlet
D. Montgomery's tubercles 4. Vagina c) AP diameter of oulet
5.Face (UPSCII 10} d) Intertubercular diameter
Code: 1118. The smallest diameter ofthe true pelvis is -(AI 05)
A B C D a) Interspinous diameter
a) 5 3 4 1 b) Diagonal conjugate
b) 4 2 1 3 c) True conjugate
c) 4 5 2 1
d) Intertuberous diameter
d)3 5 2 4
1119. Engagement offoetal head is with reference to-
1109. Which of the following is the most sensitive/gold
standard test for assessing fi HCG in maternal a) Biparietal diameter (Karn pgmee 06)
serum- (AI II) b) Bitemporaldiameter
a) Radioimmune assay b)ELISA c) Occipitofrontal diameter
c) Latex test d) Bioassay d) Suboccipito frontal diameter
1120. The shortest diameter in the fetal head is-
FETAL SKULL AND MATERNAL PELVIS a) Biparietaldiameter (AJIMS06)
b) Suboccipito frontal diameter
1110. The shortest diameter of pelvic inletis- (AI 91) c) Occipitofrontal diameter
a) Obstetric conjugate b) True conjugate
d) Bitemporal diameter
c) Diagonal conjugate d)Transverse diameter
1121. When the vertex is well flexed presentation is-
1111. Engagement offoetal head is- (UPSC 99)
a) Cephalic b)Vertex (MAHE07)
a) When the occiput has gone below the pelvic inlet
c) Face d)Brow
b) When the widest diameter of the presenting part
1122. The shortest diameter of foetal skull is - (UP 0 7)
has passed through the pelvic inlet
c) When the presenting part can be push into the a) Sub occipito frontal b) Sub mentobregmatic
pelvic cavity c) Mento vertical d) Subementovertical
d) Synonymous with fixation ofhead 1123. Sub pubic angle is- (UP 07)
1112. Diagonal conjugate is defined as the distance a) <65° b) 65°-75°
between- (UPSC 2K) c) 8SO d) 110°- 120°
a) Upperborder of symphysis pubis and the sacral 1124. Face to pubes delivery is possible with which
Promontary cephalic presentation- (UPSC 07)
b) Lower border of symphysis pubis and the sacral a) Mento-anterior b) Mento-posterior
promontary c) Occipito sacral d) Brow presentation

1105) b 1106)b 1107)a 1108)c 1109)a lllO)a llll)b 1112)b 1113)None 1114)a 1115)a,b 1116)b,d 1117)a
1118)a ll19)a 1120)d 112l)b 1122)b 1123)c 1124)c
GYNAE & OBS [54]

1125. Whe the vertex is well flexed presentation is - 1138. AFPis raised in AlE- (PGI 99)
a) Cephalic b) Vertex (Manipal 08) a) Polycystic kidney b) Trisomy
c) Face d) Brow c) IUD d) Oesophageal atresia
1126. Breech presentation is mostly mistaken for- 1139. The most common anomaly seen with chorionic
a) Face presentation b) Brow (Manipal 08) villous biopsy done in early gestation is- (PGI 99)
c) Shoulder d) Vertex a) Neural tube defects b) Limb abnormalities
1127. Engaging diameter in browpresentation?(.4PPG 08) c) Kidney abnormalities d) CH
a) Mentovertical b) Occipiti.posterior 1140. In pregnancy which vaccine is not given- (UP 2K)
c) Mento-anterior d) Mento-posterior a) Influenza b) Rubella
1128. Most important diameter of pelvis during labour is- c) Yellow fever d) Polio
a) Interspinous diameter of outlet (DPG 10) 1141. About Amniocentesis true is following except-
b) Oblique diameter of inlet a) It carries risk of miscarriage (PGI 2000)
c) AP· diameter of outlet b) Always done as a blind procedure
d) Intertubercular diameter c) Done between 10-18 weeks
1129. The true conjugate ofthe pelvic brim measures- d) Chromosomal abnormality can be detected
a) 10.5cm b) 11.5 em (UP SCIIJO) 1142. Genetic abnormality in the fetus can be diagnosed
c) 12.5cm d) 13.5 em by- (PGI02)
1130. While making a pelvic assessment in a gravid woman a) Maternal serum b) Maternal urine
at term, the obstetrician can measure with the c) Amniotic fluid d) Chorionic villi
examining finger the following diameter - e) Fetal blood
a) Diagonal conjugate (UPSC II 11) 1143. Folic acid supplement reduces the risk of -
b) True conjugate a) Neural tube defect (PGI 03)
c) Diameter of pelvic inlet b) Toxaemia of pregnancy
d) Obstetric conjugate c) Down synd.
d) Placenta pravia
ANTENATAL CARE AND ASSESSMENT OF 1144. Antenatal screening is done using the following
FETAL WELL BEING except- (TN 03)
a) Cord blood b)Amniotic fluid
1131. Calcium requirement per day during the third c) Chorionic-Villi-d)J~eripher.aU:y.m.phocyte::>-----
trimesterofpregnancy- (JIPMER 92) 1145. Preconceptional use ofthe following agent leads
to reduced incidence of neural tube defects-
a) 20mg b) lOOmg
a) Folic acid b) Iron (AIIMS 03)
c) 750mg d) lOOOmg
c) Calcium d)VitaminA
1132. Most appropriate time for chorionic villi sampling 1146. All of the following are ultrasonographic fetal
in pregnancy is- (AI 96) growth parameters except- (All India 04)
a) 16-18weeks b) 16-20weeks a) Biparietal diameter
c) 12-14weeks d) 9-11 weeks b) Head circumference
e) 80-10 weeks c) Transcerebellar diameter
1133. Diagnositc serum markers in Down's syndrome d) Femur length
are all except- (MAHE 2001) 1147. Screening by using maternal serum alpha
a) Free oestriol b) Alpha feto protein fetoproteins helps to detect all of the following
c) Hffi d) Progesterone except- (All India 04)
1134. Dietary supplement alone can not compensate for a) Neural tube defects b) Duodenal atresia
the increased requirement of which nutrient in c) Talipes equinovarus d) Omphalocele
pregnant women- (AI 91) 1148. Which of the following tests on maternal serum
a) Iron b) Iodine is most useful in distinguishing between open
c) Magnesium d) Calcium neural tube defects and ventral wall defects in a
1135. Serum alpha feto protein is decreased iu-(AIIMS 92) fetus- (AI 04)
a) Carcinoembryo antigen
a) Intra Uterine death b) Downs syndrome
b) Sphingomyelin
c) Spine difida d) Exomphalos c) Alfa- feto protein
1136. Alfa feto proteins are not increased in- (AI 93) d) Pseudocholinesterase
a) Spina bifida b) Post maturity 1149. Open Neural tube defect is best diagnosed by-
c) Esophageal atresia d) Anencephaly a) USG (SGPGI 05)
1137. J.ed maternal serum a-FP is seen in- (PGI 03) b) Maternal serum alfa fetoprotein
a) Multiple pregnancy b) Trisomy 21 c) X-ray
c) Open neural tube defect d) IUD d) Amniotic fluid acetycholinesterase

1125)a 1126)a 1127)a 1128)a 1129)aorb 1130)a 113l)d 1132)d 1133)d 1134)a 1135)b 1136)b 1137)b,d
1138)b 1139)b 1140)a,b,c 114l)b 1142)a,c,d,e 1143)a 1144)d 1145)a 1146)c 1147)c 1148)d 1149)d
GYNAE & OBS [55]

1150. All of the following disorders can be diagnosed 1161. The best way of diagnosing Trisomy-21 during
antenately except- (SGPGI 05) second trimester of pregnancy is- (AI 06)
a) Haemophilia b) Thalassemia a) Triple marker estimation
c) Sickle cell anemia d) Pernicious anaemia b) Nuchal skin fold thickness measurement
1151. Chorionic villous sampling done before 10 weeks c) Chorionic villus sampling
may results in - (MAHA 05, AIIMS 97) d) Amniocentesis
a) Fotalloss 1162. In the Manning scoring system of biophysical profile
b) Fetomatemal haemorrhage for fetal monitoring, which parameter is not included-
c) Oromandibular limb defects a) Fetal tone (AIJMS 06)
d) Sufficient material not obtained b) Fetal gross body movements
1152. In which of the following conditions would c) Oxytocin challenge test
maternal serum a-fetoprotein values be the d) Non-stress test
highest- (AIIMSNOV05) 1163. Appropriate material for antenatal diagnosis of
a) Down's syndrome b) Omphalocele genetic disorders include all ofthe following except-
c) Gastroschisis d) Spina bidida occulta a) Fetal blood b)Amniotic fluid (AIJMS06)
1153. Which one of the following biochemical parameters c) Chorionic villi d) Maternal urine
is the most sensitive to detect open spina bifida - 1164. A women in second trimester was found to have over
a) Maternal serum alpha fetoprotein (AI 05) distended uterus. Common causes include-{PGJJune
b) Amniotic fluid alpha fetoprotein a) Wrong date b) Hydramnios 06)
c) Amniotic fluid acetyl cholinesterase c) Distended bladder d) Twins
d) Amniotic fluid glucohexaminase e) Fibromyoma
1154. Use of one of the following vaccination is absolutely 1165. Early date for detection offetal heart- (PGI 06)
contraindicated in pregnancy- (AI 05) a) 6.0- 6.5 week b) 6.5-7 week
a) Hepatitis-B b) Cholera c) 7.1-7.5 week d) 8 week
c) Rabies d) Yellow fever 1166. DopplerultrasonographyiniUGR& preecclampsia
1155. All of the following are biochemical markers shows notch in which artery- (Manipal 06)
included for triple test except- (AIIMS May 05) a) Umbilical artery b) Uterine artery
c) Internal iliac artery d) Vitiline artery
a) Alfa fetoprotein (AFP)
1167. Nuchal translucency at 14 wks is suggestive of-
b) Human chorionic gonadotropin (HCG)
a) Downs syndrome (Trisomy 21) (AI 07)
c) Human Placental Lactogen (HPL)
b) Oesophageal
d) Unconjugated oestriol
c) Edwards syndrome (trisomy 18)
1156. The best time to do chorionic villous sampling d) Foregut duplication cyst
is- (AIJMSMay05) 1168. Amniotic fluid contains acetyl cholinesterase
a) Between 6-8 weeks b) Between 7-9 weeks enzyme. What is the diagnosis ? (Aiims May 07)
c) Between 9-11 weeks d) Between 11-13 weeks a) Open spina bifida
1157. The one measurement offetal maturity that is not b) Gastrochisis
affected by a 'bloody tap' during amniocentesis is- c) Omphalocele
a) LIS ratio (AIJMS Nov 05) d) Osteogenesis imperfecta
b) Phosphatidyl glycerol 1169. Short structured primi gravida has height less
c) a fetoprotein than- (MAHE 07)
d) Bilirubin as measured by A.OD 450 a) 140cm b) 145 em
1158. The best period of gestation to carry out chorion c) 150cm d) 135 em
villous biopsy for prenatal diagnosis is- (AI 05) 1170. Ideal number of antenatal visits- (James Pg 72)
a) 8-lOweeks b) 10-12weeks a) 12-14 b)6-8
c) 12-14weeks d) 14-16weeks c)7-9 d)I0-11
1159. Prenatal diagnosis at 16 weeks of pregnancy can be 1171. Minimum number of antenatal visits- (MAHE 07)
performed using all ofthe follwing, except- (AI 06) a) 3 b) 1
a) Amniotic fluid b) Maternal blood c) 5 d) 6
c) Chrionic villi d) Fetal blood 1172. On Doppler, the most ominous high indicating
1160. Which one of the following congenital foetal compromise is- (UPSC 07)
malformation of the fetus can be diagnosed in first a) Increased pulsatility index in umbilical artery
trimester by ultrasound? (AI 06) b) Increased systolic - distolic blood flow ratio
a) Anencephaly b) Inencephaly c) Increased cerebral artery flow
c) Microcephaly d) Holoprosencephaly d) Absent diastolic flow

1150)d 115l)c 1152)c 1153)c 1154)d 1155)c 1156)d 1157)b ll58)b 1159)d ll60)a 116l)d ll62)c 1163)d
1164)All 1165)a 1166)b 1167)a 1168)a 1169)a l170)a 117l)a 1172)d
GYNAE & OBS [56]

1173. Consider the following- (UPSC 07) 1184. True about chorionic villous bippsy- (PGI June 08)
l. A reactive non-stress test a) Strongly a/w fascio-mandibular defects
2. Absence or declerations b) Done in 10-12 weeks
3. A sinusoidal pattern c) Rh immunoglobulin prophylaxis not necessary
Which of the above findings in an antepartum d) Done to diagnose genetic disorders
cardiotocography indicates foetal well-being- 1185. Ideal number of antenatal visits- (Manipal 08)
a) 1 and 2 only b) 2 and 3 only a) 12-14 b)6-8
c) 1 and3 only d) 1,2and3 c)7-9 d)l0-11
1174. IncreasedAFPievelseenin- (AIIMSNov07) 1186. Minimum number of antenatal visits -(Manipal 08)
a) Down's syndrome a) 3 b) 1
b) Molar pregnancy c) 5 d) 6
c) Overestimated gestational age 1187. Short structured primigravida has height less than-
d) Congenital nephrotic syndrome a) 140cm b) 145 em (Manipal 08)
1175. In which of the following, chorionic villus sampling c) 150cm d) 135 em
procedurecanbeadoptedfordiagnosis?(UPSC-JI08) 1188. Biophysical profde scoring (Manning) include-
a) Achondroplasia b) Neural tube defects a) Nonstress test . (PGI Dec 08)
c) Marfan syndrome d) Trisomy 18 b) Fetal body movements on USG
1176. LowAFPisfoundin- (UP 08) c) Fetal breathing movements on USG
a) Neutral tube defect b) Trisomy d) Contraction stress test
c) Multiple pregnancy d) Esophageal obstruction e) Fetal cardiotocography
1177. Neural tube defects is prevented by- (UP 08) 1189. High risk pregnancy are- (PGI Dec 08)
a) Folicacid b)VitaminB 12 a) Height<l50cm
c) Vitamin B6 d) Vitamin C b) Age>35year
1178. Which ofthe following conditions is not diagnosed c) Twin pregnancy
by Chorionic Villous bipsy? (AI 08) d) Previous lower segment caesarean section
a) Neural tube defects b) Down's syndrome e) Weight <45 kg
c) Phenylketonuria d) Sickle cell anemia 1190. Transabdominal CVS can be done in-
1179. Which of the following is not correct regarding a) 7-9weeks b) 11-13 weeks {AIIMSMay 09)
Doppler studies ofthe umbilical circulation? (AI 08) c) 9-11 weeks d) 13-15weeks
a) Changes in flow velocity waveforms of the 1191. All are true about aneuploidy except -(;fllMSMay09)
umbilical artery may be important in clinical a) 30% of trisomy 21 fetus die in utero
management of high - risk pregnancies b) 80% of trisomy 18 fetus die in utero
b) Normal pregnancy shows a characteristic decrease c) Occurrence of aneuploidy has no relation with
in the Systolic I Diastolic (SID) ratio with advancing the progression of mother's age
gestational age d) Not recalled
c) Maternal smoking leads to an increase in SID ratio 1192. The second trimester screening protocol for
d) Absence of end - diastolic flow is normal at term detection of fetal aneuploidy called the "triple
1180. Which of the following dietry supplements is screen" includes the assessment of serum levels of
recommended for a pregnant lady on heparin- all except- (DELHI PG Mar. 09)
a) Folic acid b)Calcium (AI 08) a) Alpha-fetoprotein
c) Zinc d) Copper b) Chorionic gonadotropin
1181. Pre-conceptional intake of which ofthe following c) Unconjugated estriol
results in decrease in incidence of neural tube d) Pregnancy associated plasma protein-A
defect? (AIIMS May 08) 1193. Consider the following markers- (UPSC-II 09)
a) Vit.A b) Folate I. Nuchal translucency 2. PAPP-A
c) VitE d)Vit. C 3. GIT 4. InhibinA
1182. Regarding Alpha Fetoprotein true statement is- Which of the above markers are included in the first
a) Major source of fetal life is yolk sac trimester screening of Down's syndrome?
b) Commonly elevated in Wilm's tumor a) 1 and 2 b) 2 and 3 only
c) Max level at 20th week c) 1 and4 d)2,3and4
d) Half-life 5-7 days 1194. "Variable decelerations" on an electronic fetal heart
1183. AboutAFPtrueA/E- (PGI June 08) rate monitor imply- (UPSC-II 09)
a) MSAFP detected 16-18 wks of gestation a) Umbilical cord compression
b) Diabetic patients have raised AFP level b) Fetal head compression
c) MSAFP is unrelated to the period of gestation c) Fetalanemia
d) Highest fetal level seen around 13 wks d) Uteroplacental insufficiency

1173)a 1174)d 1175)d 1176)b 1177)a 1178)a 1179)d 1180)b 1181)b 1182)a 1183)b,c,e 1184)a,b,d 1185)c
1186)a 1187)a 1188)a,b,c 1189)b,c,d 1190)b 1191)c 1192)d 1193)a 1194)a
GYNAE & OBS [57]

1195. Intra-uterine assessment offetal distress is indicated 1205. The maternal serum alpha-fetoprotein
by all, except- (DELIH PG Feb. 09) concentration is elevated in the foUowing conditions
a) Accelerationof15/min except- (UPSCII 11)
b) Deceleration of30/min a) Multiple gestation
c) Variable deceleration 5-25/min b) Foetal neural tube defect
d) Fetal HR < 80/min c) Foetal osteogenesis imperfecta
1196. Feto-maternal transfusion is detected by-(DELIHPG d) Gestational trophoblastic disease
1206. In the quadruple test conducted as a part of second
a) Kleihauertest b) Spectrophotometry Feb. 09)
trimester screening for Down's syndrome, the
c) Benzidine test d) Colorimetry
fourth marker which has been added to the triple
1197. Biophysical profile scoring includes -(PGI Nov 09) test is- (UPSCII 11)
a) FHR accerleration a) PAPP-A
b) Fetal breathing b) Inhibin-A
c) Foetal movement c) Human Placental Lactogen (HPL)
d) Placenta praevia grading d) Acetylcholinesterase
e) Uterine artery waveform 1207. As per the Government of India guidelines, the dally
1198. Elements ofthe Bishop score Used for Assessment dose of elemental iron recommended for prophylaxis
of inducibility include the foUowing except- duringpregnancyis- (UPSC II 11)
a) Station (Maharashtra 10) a) 50mg/dayfor 100 days
b) Fetal movements b) 100 mg/day for l 00 days
c) Cervical dilatation c) 150 mg/day for 100 days
d) Cervical position d) 200 mg/day for 100 days
1199. Normal Biophysical Profile (Manning Score) is- 1208. Weight gain in pregnancy depends on all of the
a) 8-10 b)6-8 (Maharashtra 10) following factors, Except- (AI 11)
a) Socioeconomic status
c) 4-6 d) 3--4
b) Pre- pregnancy weight
1200. Biophysical profile scoring includes -(PGI Nov. 10)
c) Smoking
a) Fetalmuscletone d) Ethinicity
b) Fetal breathing movements 1209. Which one of the following congenital
c) Contraction stress test malformations of the fetus can be diagnosed earliest
d) Placental structure in a first trimester ultrasound? (AI 11)
e) Amniotic fluid a) Anencephaly b) Prosencephaly
1201. A 32 yr old mother who has a Down's boy is now 9 c) Microcephaly d) Meningocele
wks pregnant, she doesn't want another Down's. As 1210. Modified biophysical profile consists of-
her physician you should tell her- (AJJMS Nov 10) a) AFI&Fetalmuscletone (Jipmer 11)
a) USG at this stage will confirm presence or absence b) AFI & Non stress test
b) Chorionic villous biopsy at this stage will confirm c) Non stress test & Fetal muscle tone
presence or absence d) Non stress test & Fetal breathing movements
c) Triple test/maternal AFP at this stage will confirm 1211. Neural tube defects can be prevented by the
presence or absence administration of- (Jipmer 11)
d) Nothing need be done as there is minimal risk since a) Vit-B12 b)Folicacid
c) Elementary iron d) Tetanus toxoid
her age is< 35 yr
1212. Best serological marker ofDown's syndrome in first
1202. Which of the following is the msot specific marker
trimester is - (Jipmer 11)
for Neural Tube Defects- (AI 11)
a) Nuchal translucency b) Skeletal abnormalities
a) Actylcholinesterase
c) ~-HCG d) Serum estriol
b) Pseudocholinesterase
c) Alpha feto protein (AFP) NORMAL LABOUR
d) Human chorionic Gonadotrophin (HCG)
1203. Levels of alfa-feto-protien (AFP) in maternal serum 1213. In a primigravida head engages at- (Kerala 94)
steadily increases till - (Karn 11) a) 36 weeks b) 1'1 stage
a) 6 weeks b) 13 weeks c) Tenn d) 2nd stage
c) 22 weeks d) 35 weeks 1214. First stage oflabour is upto - (Kerala 94)
1204. 'Triple Bubble' sign during fetal ultrasonography is a) Rupture of membranes
seen in - (Karn 11) b) 3/5 dilatation ofcervix
a) Diaphragmatic hernia b) Duodenal atresia c) Fulldilatationofcervix
c) Jejunal atresia d) Hydrops fetalis d) Crowing ofhead

1195)a 1196)a 1197)a,b,c 1198)b 1199)a 1200)a,b,e 120l)b 1202)a 1203)d 1204)c 1205)d 1206)b 1207)b
1208)c 1209)a 1210)b 1211)b 1212)c 1213)a 1214)c
GYNAE & OBS [58]

1215. Definite sign of placental separation in stage ill of 1225. What is the pressure inside uterus during second
partutitionis- (JIPMER 95) stage oflabour- (MAHE 07)
a) Gushing ofblood a) 100-120mrnofHg b)200-220mrnofHg
b) Lengthening of cord c) 300-400mrnofHg d)25mrnofHg
c) Filling ofplaceents in vagina 1226. A G2P lAO presents with full-term pregnancy
d) Increase in blood pressure with transverse lie in the first stage oflabour. On
1216. All of following are features of pseudocyesis examination, cervix is 5 em dilated, membranes are
intact and foetal heart sounds are regular. What
except- (AI 96)
would be the appropriate management in this case-
a) Enlargement of abdomen
a) Wait for spontaneous evolution and expulsion
b) Morning sickness b) External cephalic version (UPSC 07)
c) Amenorrhoea c) Cesarean section
d) Enlargement of uterus 1227. IVErgotamineshouldbegiven- (UP 08)
1217. Which is not a feature of pseudocyeis - a) After the delivery of shoulder
a) Amenorrhoea (CUPGEE 96) b) During breech extraction
b) Abdominal destension c) In twin pregnancy, first delivery of the child
c) Fetal heart sounds are audible d) During face presentation
d) None 1228. Which of the foUowing interventions is not included
1218. Linving ligature of the uterus is- (Orissa R) in active management oflllrd stage oflabour?
a) Endometrium a) Administration of a Uterotonic within 1 minute of
b) Middle layer of myometrium delivery (AI 08)
c) Inner layer of myometrium b) Immediate clamping cutting and ligation of cord
d) Perimetrium c) General massage of uterus
1219. During active labour cervical dilatation per hour d) Controlled cord traction
inprimiis- (CIPOJ) 1229. Which movements occurs during labor-
a) 1.2cm b) 1.5cm a) Flexion b) Extension (PGIJune 08)
c) 1.7cm d)2cm c) Internlaroration d) Descent
1220. Assessment of progress oflabour is best done by- e) Asynclitisms
a) Station of head (PGI 97) 1230. AU are signs of recent delivery except-(Manipal 08)
b) Rupture of membrane a) Colostrum b) Carbunchulae myrtiformis
c) Contraction of uterus c) Lochia d) All of the above
d) Partogram 1231. In an uncomplicated twin pregnancy normal delivery
1221. Duration oflatent phase oflabour is affected by should be attempted in the following situation -
a) Early use of conduction anaesthesia & sedation a) First baby-vertex and second baby transverse lie
b) Unripedcervix (PGI 2000) b) Both babies and breech presentation
c) Hypertonic uterine contraction c) First baby transverse lie and second longti tudinal
d) Preclampsia lie (Manipal 08)
1222. Cardinalmovementsoflabourare- (PGI2000) d) All of the above
a) Engagement ~ descent ~ flexion ~ internal 1232. What is the pressure inside uterus during second
rotation ~ exstention ~ restitution ~ external stageoflabor- (Manipal 08)
a) 100-120mrnofHg b)200-220mrnofHg
rotation ~ expulsion
c) 300-400 mrn ofHg d) 25 mrn ofHg
b) Engagement ~ flexion ~ descent ~ internal
1233. latent period in primigravida is? (APPG08)
rotation ~ extention ~ expulsion
a) 2 hours b) 6 to 8 hours
c) Engagement ~ flexion ~ descent ~ external
c) 10 to 12 hours d) 14 to 16 hours
rotation ~ expulsion '"'
1234. True about pain is/are characterzed by-
d) Engagement ~ extention ~ internal rotation
a) Painful Uterine contraction (PGI June 09)
~ external rotation~ expulsion
b) Short vagina
1223. The graph showing relationship between cervical c) Formation of the bag of waters
dilatation and duration oflabour is- (Kerala 03) d) Progressive descend of presenting part
a) Partogram b) Cervicograph e) Dilation of cervix
c) Growth curve d) Dilatation chart 1235. Indications(s) of outlet forcep is/are-
1224. Normal partogram include the following except- a) Membrane absent/rupture (PGI Nov 09)
a) Cervical dilatation in X-axis (Karn pgmee 06) b) Head 0 station
b) Descent of head in Y-axis c) Cephalopelvic disproportion
c) Sigmoid shaped curve d) Full cervical dilation
d) Alert line followed 4 hrs later by action line e) APposition of head

1215)a,b 1216)b,d 1217)c 1218)b 1219)a 1220)d l221)a,b 1222)a 1223)b 1224)a 1225)a 1226)c 1227)a
1228) c 1229)All 1230) b 1231) a 1232) a 1233)b 1234) a,c,e 1235)a,d,e
GYNAE & OBS [59]

1236. Prelabour pains are mediated through- NORMAL PUERPERIUM


a) Tll-Tl2 b)T2-L3 (AIIMSNov09)
c) Sl-S3 d)L3-IA 1245. The bladder shoulb be empty within- (DNB 90)
1237. Which of the following interventions is not a) 2-3 hrs of delivery b) 3-4 hrs of delivery
recommeded in active management of third stage of c) 4-5 hrs of delivery d) 6-8 hrs of delivery
labour- (AI 10) 1246. Strongest stimulus oflactation is by-(JIPMER 95)
a) Administration of Uterotonic within 1 minut of a) Metachlopramide
delivery b) Hot partum haemorrhage
b) Immidiate clamping, cuttingand ligation of cord c) Bromocriptine
c) Gentle massage of uterus d) Suckling
d) Controlled cord traction 1247. Peurparium is the period- (IN 97)
1238. Active managment of third stage labor includes all a) 6 weeks following delivery
ofthefollowingexcept- (AI 10) b) 3 weeks following delivery
a) Oxytocin injection c) One week following delivery
b) Ergometrine injection d) None of the above
c) Controoled cord traction 1248. Decidua is cast of in peuparium as- (AI 9 7)
d) Gentle massage of uterus a) Lochia b) Membranes
1239. After delivery of head, the delivery ofthe rest of the c) Placental bits d) Meconium
body can be hastened by- (Maharashtra 10) 1249. Weight of uterus after 8 weekS of delivery-
a) Moderate pressure on uterine fundus a) 800gms b) lOOOgms (A/97)
b) Traction on fetus in the direction of the long axis c) 500gms d) lOOgms
of its body · 1250. 100 %protection against conception in a lactating
c) Traction on the head mother is for about- (CMC 98)
d) All of the above a) 1 month b) 2 weeks
1240. ACOG defines arrest of first stage of labour as c) 2 months d) 3 months
completed latent phase with uterine contractions of e) 4months
strength > 200 m without cervical changes 1251. Lochia is seen for- (AP 98)
for (Maharashtra 10) a) 1-4 days b) 5-10 days
a) 1 hour b) 2 hours c) 10-14days d) 14-21 days
c) 3 hours d) 4 hours 1252. Which one of the following sets of conditions is
1241. Allowing the cord blood passage to fetus before · attributedto normal physiology of puerperium-
clamping the umbilical cord should be AVOIDED to a) Tachycardia and weight gain (USPC 2K)
prevent- (Maharashtra 10) b) Retention of urine, constipation and weight gain
a) Maternal alloirnmunization c) Constipation, tachycardia and retention of urine
b) Prematurity d) Retention of urine and constipation
c) Growth retardation of fetus 1253. Cause (s) of retention of urine in reproductive age
d) All of the above group- (PGI 2000)
1242. Regarding the use of a ventonse, which one of the a) Cervical fibroid
following statements is not correct? (UP SC II 10) b) Retroverted gravid uterus
a) In can be applied when the cervix is incompletely c) Unilateral hydronephrosis
dilated d) Severe UTI
b) The cup should be centrally placed on the vertex e) Posterior urethral valve
c) The largest size of the cup is preferred 1254. C.I. for breastfeeding are except- (PGI 2000)
d) The maximum pressure should not exceed 0.8 kg/ a) Hepatitis -B infection of mother
crn2 b) Lithium treatment of mother
1243. The correct method to conduct the third stage of c) Acute bacterial mastitis
normallabouris- (Corned 10) d) Teracycline treatment of mother
a) Fundal pressure 1255. Initiation oflactation is affected by- (PGI 01)
b) Bimanual massage a) Oxytocin b) Prolactin
c) Controlled cord traction c) ffi>L d) Thyroid hormone
d) Manual removal e) Progesterone
1244. True about vaccum extraction offetns-(PGI May 10) 1256. Contraindication to breast milk feeding- (PGI OJ)
a) Can be used in non dilated cervix a) Mother is sputum negative
b) Can be used in incompletely dilated cervix b) Bromocriptine therapy for mother
c) Used in Face presentation c) Heavy breast engorgement
d) Applied 3cm post. to Anterior fontanel d) Ca breast
e) Applied 3 em ant. to posterior fontanel e) Mother on domperidone

1236)a 1237)b 1238)None 1239)d 1240)b 124l)a 1242)b 1243)c 1244)b,e 1245)d 1246)d 1247)a 1248)a
1249)d 1250)c 1251)d 1252)d 1253)a,b 1254)a 1255)b 1256)None
GYNAE & OBS [ 60]

1257. Thue about fixed retroversion- (PG/02) 1269. All are signs of recent delivery except- (MAHE 07)
a) PID a) Colostrum b) Carunculae myrtiformis
b) Posteriorwallmyoma c) Lochia d) All of the above
c) Endometriosis 1270. Following delivery, uterus becomes a pelvic organ
d) Congenital retroversion a~r- (UPSCO~
e) Anterior cervical myoma a) 2weeks b)4weeks
1258. J.. Lactation seen in- (PG/03) c) 6weeks d) 8 weeks
a) Maternal anxiety b) Antibiotic therapy 1271. Immediately after third stage oflabour in a case
c) Crackednipple d) Breast abscess of fuU-term delivery, the fundus of the uterus is -
e) Bromocriptine therapy a) At the level ofXiphistemum (UPSC 07)
1259. Pulserateinpuerperium- (Kerala 03) b) At the level of umbilicus
a) Increases b) Decreases c) Just below the level of umbilicus
c) Nonnal d) Variable d) Just above the symphysis pubis
1272. Which of the following statements about breast
1260. About colostrum true statements are- (PGI 03)
infections during lactation is most correct- (AI 08)
a) Secreted after 10 days of childbirth
a) Are caused by bacteria from infant's
b) Rich in immunoglobulin gastrointestinal tract
c) Contains more protein b) The drug of choice for treatment is Amoxicillin
d) Contains less fat c) The most common etiological organism is E. coli
e) Daily sectretion is about 10 mVday d) Can lead to abscess formation for which and I & D
1261. Lochia rubra is seen upto- (Orissa 04) may be required
a) 5 days b) 10 days 1273. Breast milk is known to transmit- (Manipal 08)
c) 15 days d) 20 days a) Untreated tuberculosis b)CMV
1262. The following drugs can be used to suppress c) Varicella d) Rubella
lactation, except- (UPSC 04) 1274. The iron content in 100 gm of breast milk is-
a) Cabergoline b) Pyridoxine a) 2mg b)0.2mg (DPG08)
c) High dose estrogens d) Metoclopramide c) 5mg d) lOmg
1263. Galactokinesis means- (Jipmet 04) 1275. Lactation suppression in hypertensive post-partum
a) Sustaining lactation b) Secretion of milk patient is achieved by- (COMED 09)
c) Ejection ofmilk d) Synthesis of milk a) Bromocriptine
1264. Physiological chill in labour seen in-(Manipal 04) b) Oestrogen alone
a) pt stage · b) 2nd stage c) Oestrogen and testosterone
c) 3rd stage d) 4th stage d) Breast binder and analgesics
1265. Postpartum decidual secretions present are 1276. In the puerperium, which of the foUowing hormonal
referred to as- (MAHE 05) changes are responsible for lactogenesis?
a) Lochia b) Bleeding per vaginum 1. A sudden fall in the oestrogen levels after delivery
c) Vasa- previa d) oecidua- capsularis 2. Reduction of prolactin inhibiting factor from the
1266. In puerperium the lochia seen p/v is in which of hypothalamus · (UP SCI! 10)
the foUowing sequences- (UPSC 06) 3. Release of prolactin from the anterior pituitary
a) Lochia alba-Lochia serosa-Lochia rubra 4. Release of oxytocin from the posterior pituitary
b) Lochia serosa-Lochia rubra-Lochia alba Select the correct answer using the code given below:
a) 1 and2 only b) 1 and4
c) Lochia alba-Lochia rubra-Lochia serosa
c) 1,2and3 d)2,3and4
d) Lochia rubra-Lochia serosa-Lochia alba
1267. What is the maximum duration for which ABORTION
lactational amenorrhoea can prevent conception-
a) 1 month b) 2 months (UPSC 06) 1277. The following are modes of induction of
c) 4 months d) 6 months midtrimester abortions except- (TN 91)
1268. In normal puerperium, aU are true regarding a) Intra anmictic saline
involution of uterus except- (UP 07) b) Intra anmiotic prostaglandins
a) At the end of the 6 weeks, becomes non pregnant c) Menstrual regulation
state d) Hysterotomy
b) The number of muscle fibres is not decreased but 1278. The best method for inducing mid trimester
there is substantial reduction of the myometrial abortionis- (AIIMS91)
cell size a) Injection ofHypertonic Saline
c) At the end of the 4th week uterus becomes as b) Ethacrydine
pelvic organ c) Prostaglandins
d) Weight ofuretus about 60 gram at end of the weeks d) DandC

1257)a,c,e 1258)a,e 1259)a 1260)b,c,d 1261)a 1262)d 1263)c 1264)d 1265)a 1266)d 1267)b 1268)c 1269)d
1270) a 1271) c 1272) d 1273) a 1274) b 1275) d 1276) b 1277) c 1278) c
GYNAE & OBS [ 61 ]

1279. The commonest chromosomal abnormality in 1288. Prostaglandins can be used for medical
early spontaneous abortions is- (KARN 94) termination of pregnancy by all routes except-
a) Monosomy X b) Autosomal trisomy a) Intravenous b) Intramuscular (UPSC OJ)
c) Triploidy d) Tetraploidy c) Extra amniotic d) Intra amniotic
1280. All offollowing methods are used for MTP in 2nd 1289. Following is used in I trimester MTP- (TN 2002)
trimester except- (DELHI 96) a) Mifepristone
a) Intra- ammotic saline b) Prostaglandins b) Mifoprostal
c) Hysterotomy d) Menstrual regulation c) Mifepristone & misoprostol
1281. All are true regarding menstrual regulation d) Laminaria tent
except- 1290. The minimum age for giving consent for medical
a) It comes under MTP act (AP 96) termination of pregnancy is : (Jipmer 9 5)
b) Blood loss is less a) 16years b) 18years
c) Done within 2 weeks c) 20 years d) 24 years
d) General - anesthesia not needed 1291. Legally :Abortion" is termination of
1282. Induction of abortion is best by- (AIIMS 96) pregnancy- (Karnataka 01)
a) Oxytocin b)PGE2 gel a) Before 6 weeks b) Before 16 weeks
c) Strippingofmembrane d) Oestrogen c) Before 26 weeks d) Before full term
1283. A 25 years old female reports in the casualty with 1292. Recurrent abortion is caused by- (PGI 97)
history of amenorrgoea for two and half moots ~ ~ b)HSV
and abdominal pain and bleeding per vaginum c) Syphilis d) Toxoplasmosis
for one day. On examination, vital parameters 1293. Sponteneous abortion in 1st trimester is caused
and other systems are normal. On speculum by- (PGI 2000)
examination, bleeding is found to come from Os. a) Trisomy21 b) Monosomy
On bimanual examination, uterus is of 10 weeks c) Trauma d) Rh-incompatibility
size, soft and Os admits one finger. The most 1294. Cervical incompetence is characterised by-
a) l'ttrimester abortiom (PGI 03)
likely clinical diagnosis is- (UPSC 99)
b) 2nd trimester abortion
a) Threatened abortion b) Missed abortion
c) Premature rupture of membrane
c) Inevitable abortion d) Incomplete abortion
d) Circlage operation done
1284. Which one the following statements regarding
1295. Recurrent spontaneous abortion is seen in AlE-
MTP is incorrect- (UPSC 99)
a) TORCH agent (PGI 03)
a) It is governed by the Act of 1971
b) Uterine pathology
b) The MTP Act permits termination of pregnancy c) Herpes
for medical reasons only d) Balanced paternal translocation
c) It can be performed by trained personnel e) None
d) It should be done only in those places which 1296. According to MTP Act, 2 doctor's opinion is
fulfill the critc;:ria laid down in the Act required when pregnancy is- (PGI 03, KARN 98)
1285. Which one of the following methods of MTP is a) lOweeks b) 6weeks
associated with the complication of c)> 12weeks d)>20weeks
coagulopatby- (UPSC 99) e) 8weeks
a) Aspirotomy b) Hypertonic saline 1297. In extra-amniotic 2nd trimester medico legal
c) Ethacridine lactate d) Prostaglandins termination ofpregnanacy, which of the following
1286. Which of the following cannot be used for MTP used- (PGI 04}
in a patient with bronchial asthma- (ORR/SA 99) a) Ethacrydine Lactate b) Prostaglandin
a) Prostaglandin b) Oxytocin c) Hypertonic saline d) Glucose
c) N.T.T. d) Ethacrydine e) Urea
1287. Internationally accepted deimition of abortion is 1298. All of the following drugs have been used for
the expulsion of the products of conception- medical abortion, except- (AIIMS 03)
a) Before 28th week of gestation (Kerala 2K) a) Mifepristone b) Misoprostol
or lkg (weightoffoetus) c) Methotrexate d) Atosiban
b) Before 24th week of gestation or 750 gms 1299. Decidual casts bleeding per vaginum suggestive
(weight offoerus) of- (Manipal 04)
c) Before 20th week of gestation or 750 gms a) Inevitable abortion b) Threatened abortion
(weight offoetus) c) Tubal abortion d) None
d) Before 20th week of gestation or 500 gms 1300. MostcommoncauseofAbortion- (HPU05)
(weight offoetus) a) Ovofoetal factor b) Maternal hypoxias
e) None of the above statements is correct c) Uterine fibroid d) Cervical incompetence

1279)b 1280)d 1281)None 1282)b 1283)c 1284)b 1285)b 1286)a 1287)d 1288)a 1289)c 1290)b
129l)None 1292)c,d 1293)a,b,c 1294)b,c,d 1295)e 1296)c 1297)a,b 1298)d 1299)c 1300)a
GYNAE & OBS [ 62]

1301. The medical termination of pregnancy act does 1311. All ofthe following are known causes ofrecnrrent
not protect act of termination of pregnancies abortion Except- (AI 08)
after- (Kamat 05, AP 88) a) TORCH infections b)SLE
a) 20 weeks b) 24 weeks c) Rhincompatibility d) Syphilis
c) 28 weeks d) 30 weeks 1312. Treatment of cervical incompetence- (PGI June 08)
1302. Abortion is defined as "expulsion of fetus" less a) Shirodkar 's stitch b) McDonald's stitch
tban--gms- (MAHE05) c) B-lynch suture d) Fothergill's operation
a) 500 b)800 e) Sling operation
c) 900 d) 1000 1313. A patient with a H/o recurrent abortions and
1303. Procedures done for cervical incompetence hypercoagulable state is likely to be suffering from-
include- (PG/ 03, June 06) a) Anti-phospholipid antibody syndrome
a) Shirodkar's procedure b) Heparin induced thrombocytopenia (Manipal 08)
b) Me. Donald's procedure c) Leiden mutation
c) Porander operation d) Anti-thrombin III deficiency
d) Khann's sling operation 1314. lncorrectaboutincompetent cervix? (APPG 08)
e) Abdominal sling operation a) Hegar dilators used
1304. A female with recurrent abortion and isolated b) Most common cause of frrst trimester abortion
prolongedAPTT is most likely associated with- c) Both
a) Lupus anticoagulant (Aiirns May 07) d) None
b)DIC 1315. Mifepristone may be used for all of the following
Except- . . (AI 09)
c) Von wilebrand disease
d) Hemophilia a) Threatened abortion b) Ectopic pregnancy
1305. A patient with a history of recurrent abortions c) Fibroids d) Molar pregnancy
and hypercoagulable state is likely to be suffering 1316. Recurrent abortion is/are caused by- (PG/June 09)
form- (MAHE 07)
a) Rubella b) Thrombophilla
c) Diabetic mellitus d) Syphilis
a) Anti-phospholipid antibody syndrome
e) Chronic hypertension
b) Heparin induced thrombocytopenia
1317. A pregnant woman has previous history of recurrent
c) Leiden mutain
early pregnancy loss. Which of the following
d) Anti-thrombin 2 deficiency
investigations should be ordered- (PGI Nov 09)
1306•. G3 with previous second trimester abortion presents
a) Blood glucose
with 22 week of gestation, abdominal pain, USG
b) VDRL
shows funneling of internal os what will be ideal c) Rubella virus screening
management? (Al/MS Nov 07) d) Toxoplasma serology
a) Dinoprost and bed rest e) Hemogram & blood grouping
b) Misoprost with bed rest 1318. A lady presented with features ofthreatened abortion
c) Fothergills suture at 32 weeks of pregnancy. Which of the following
d) Me Donald stitch statements with regard to antibiotic usage is not
1307. All the following are used in firSt trimester except- correct- (A/10)
a) Dilatation and evacuation (Corned 08) a) Antibiotic prophylaxis even with unruptured
b) Ru486 membranes
c) Suction and evacuation b) Metronidazole if asymptomatic but significant
d) Ethacrydine extra amniotic bacterial vaginosis
1308. Dose of Anti-D gamma globullin following first c) Antibiotics if asymptomatic but significant
trimester abortion is- (Corned 08) bacteremia
a) 50 g b) 100 g d) Antibiotics for preterm premature Rupture of
c) 200 g d)300 g membranes
1309. Partogram is used to- (Corned 08) 1319. Which of the following is not an indication for
a) Assess the fetal well-being in labour antiphospholipidantibodytesting? (DPG 10)
b) Assess the conditioon of the baby at birth a) 3 ormoreconsecutivefirsttrimesterpregnancy losses
c) Record the events of pregnancy b) Unexplained cerebrovascular accidents
d) Assess the progress of labour c) Early onset sever preeclampsia
1310. Most common cause ofabnormality in first trimester d) Gestational diabetes
abortion- (UP 08) 1320. Abortion is/are caused by- (PG! May 10)
a) Cervical incompetance a) Borrelia recurrentis b)HIV
b) Chromosomal abnormality c) Listeria d) Syphilis
c) Placental & membrane abnormality e)HBV
d} Retroversion

1301)a 1302)a 1303)a,b l304)a 1305)a l306)d 1307)d 1308)a 1309)d 1310)b 13ll)a 1312)a,b 1313)a
1314)b 1315)a 1316)b,c 1317)a,b,d,e 1318)a 1319)d 1320)b,c,d
GYNAE & OBS [ 63]

1321. A 25-year-old woman with a history of three 1332. Best modality for diagnosing unruptured tubal
consecutive abortions has been investigated pregnancy- (PGI 97)
thoroughly to detemine the cause of recurrent a) Abdominal USG b) Transvaginal USG
pregnancy loss. In the absence of a demonstrable c) X-ray Abdomen d) CT scan
cause, what is the chance of a viable birth in 1333. Ectopic pregnancy differs from threatened abortion
subsequentpregnancy- (UPSCII JJ) by- (MAHE 98)
a) Less than 20% b)20-30% a) Minimal vaginal bleeding
c) 4()0/o d) 60% b) Pain follows bleeding
c) Uterine size corresponds to gestational age
ECTOPIC PREGNANCY d) Manifests at an early age
1334. Most constant symptom present in undisturbed
1322. Ectopicpregnancyismorecommonin- (PG/90) ectopic- (PGI 98, UP OJ)
a) Tuboplasty b) Endometriosis a) Pain in lower abdomen b) Bleeding PN
c) Copper T users d)All c) Ammenorrhoea d) Hypotension
e) Retroverted uterus
1323. The expelled products in ectopic pregnancy originate
1335. Diagnostic criteria for primary abdominal
from- (JIPMER 9J)
pregnancy- (ORR/SA 99)
a) Decidua basalis b) Decidua vera
a) Spigelberg criteria b) Rubin's criteria
c) Decidua Capsularis d) Chorionic villi
c) Studiford criteria d) Wrigly criteria
1324. Arias stella reacion is not seem in- (AIIMS 9J) 1336. Best endometrial reaction in ectopic pregnancy is-
a) Ovarian pregnancy a) Arias Stella reaction (MP 2K)
b) Molar pregnancy b) Secretory phase
c) Interstital pregnancy c) Decidual reaction without chorionic villi
d) Salphingitis isthmica nodosa d) Decidual reaction with chorionic villi
1325. Best investigtion to diagnose ectopic pregnancy 1337. A22 years old nnllipara presents with an unruptured
is - (KERALA 94) ectopic pregnancy. What is thetreatment of choice-
a) Urine pregnancy test b) Laprascopy a) Liner salpingostomy (AIIMS 2K)
c) USG d) Hysteroscopy b) Salpingectomy
1326. In a young patient with abdominal pain and c) Resection and end to end anastomosis
features most suggestive of ectopic pregnancy is- d) Milking of the tube
a) Amenorrhea (AI 95) 1338. Which of the following is the most reliable
b) Vomiting indicator of Ectopic gestation- (AIIMS 2K)
c) Palpation of tender adnexal mass a) No gestational sac in USG
d) Increase beta HCG in urine b) Arias Stella reaction
1327. Most common cause of ectopic pregnancy is- c) Culdo centsesis showing blood in the posterior
a) ruDC b) Previous tubal disease (AI 96) cui- de sac
c) Endometriosis d) Minipill d) Absence of the normal doubling ofHCG levels
1328. Commonest cause of ectopic gestation is- 1339. Ectopic pregnancy is most commonly associated
with- (PGI OJ)
a) Previous salphingitis (KERALA 96)
a) Endometriosis b) Congenital tubal anomalies
b) Dysfunction of cilia
c) Tuberculosis d) Tubal inflammatory diseases
c) Uterine abnormalities
e) Retroverted uterus
d) Delayed fertilisation of ovum
1340. About ectopic pregnancy true statements are-
e) Hydrosalphynx a) RisingtitreofHCG (PGI 03)
1329. Ovarian pregnancy is due to- (KERALA 96) b) Negative pregnancy test excludes the diagnosis
a) Previous sexually transmitted infections c) Common after tubal surgery
b) Fertilisation of ovum in ovary d) Seen in patients taking GnRH therapy
c) Bicornuate uterus e) Common in patients taking OCP
d) Pelvic tuberculosis 1341. Drugs treatment of ectopic pregnancy are-(PGI 03)
e) None of the above a) MTX b)Actinomycin-D
1330. Medical treatment ectopic pregnancy is-(AJJMS 96) c) HCG d)RU486
a) Methotrexate b) Progesterone e) KO
c) Oestrogen d)Adriamycin 1342. Which of the following drug is not used medical
1331. Most common cause of ectopic pregnancy is- management of ectopic pregnancy- (AIIMS 03)
a) Progestasert b) Cu-T (AIIMS 96, PGI 99) a) Potassium chloride b) Methotrexate
c) OCP d) Minipill c) Actinomycin D d) Misoprostol

1321)d l322)d 1323)b 1324)d 1325)b 1326)d 1327)b 1328)a 1329)b 1330)a 1331)a 1332)b 1333)None
1334) a 1335) c 1336) c 1337) a 1338) a 1339) a,b,c,d 1340) a,c,d,e 1341) a,b,d,e 1342) d
GYNAE & OBS [ 64]

1343. Spigelberg's criteria, not true· (APPG 03) 1352. Commonest site of tubal pregnancy is- (UP 08)
a) Tubes and uterus normal a) Ampulla b) Isthmus
b) Uterus attached to broad ligament c) Fimbria d) Interstitial
c) No ovarian tissue lining 1353. The cause of fetal death in ectopic pregnancy is
d) Uterus attached to pelvic wall postulatedas- (AiimsMay08)
1344. Ectopic pregnancy lasts longer in· (Jipmer 04) a) Vascular accident
a) Interstitial b) Isthumoampullary b) Nutritional adequancy
c) Endocrine insufficiency
c) Isthmic d)Ampullary
d) Immune response of mother
1345. Medical treatment of Ectopic gestation involves
1354. Most valuable diagnostic test in a case of suspected
use of all the following drugs except· (MAHE 05, ectopic pregnancy- (Aiims May 08)
a) Prostaglandins b) Methotrexate Karn 94) a) Serial f3-hCG levels
c) RU-486 d) Dexamethasone b) Transvaginal USG
1346. The following drug is not helpful in the treatment c) Progesterone measurement
of ecropic Pregnancy- (AI 05) d) Culdocentesis
a) Methotrexate b) Misoprostol 1355. True about ectopic pregnancy- (PGI June 08)
c) Actinomycin- D d) RU 486 a) Transvaginal USG first imaging of choice
1347. Modern diagnostic aids to diagnose ectopic b) AIW decidual reaction
pregnancy- (PGI June 06) c) Doppler is of no significance
a) IKXi b) Transvaginal USG d) In ectopics intestitial ring sign seen
c) AFP d) Gravindex e) HCG level is sufficient for diagnosis
1348. A 32-year old woman with two live children was 1356. Which method is most responsible for ectopic
brought in emergency with the history of missed gestation - (PGI June 08)
period for 15 days, spotting since 7 days ado pain a) Pomeroy b) Modified Pomeroy
abdomen since 6 hours. Her pulse was 120/min, c) Electrocoagulation d) Laporoscopic Hulka clips
Pallor ++, systolic BP 8 mm of Hg. There was e) Irving operation
fulness and tenderness on per abdomen 1357. Afemalehashistoryof6weeksamenorrhoea, USG
examination. Copper- T thread was seen through shows no sac, serum beta HCG-1000 lUlL. What
external os on per speculum examination. On p/v would be next management? (AIIMS Nov 08)
examination, cervical movements were tender, a) Medical management
uterus was anteverted, bulky and soft. There was b) Repeat HCG after 48 hours
fulness in pouch of Douglas. She is most likely c) RepeatHCGafter 1 weeks
suffering from- (UPSC 07) 1358. Indication(s) of methotrexate chemotherapy in
a) Pelvic inflammatory disease ectopic pregnancy is/are- (PGI June 09)
b) Missed abortion with umbilicus a) Fetal cardiac activity present
c) Ruptured ectopic pregnancy b)Size<4cm
d) Threatened abortion c) -hCG disease
1349. What is the treatment of choice of unruptured d) Liver disease
tubal pregnancy with serum beta hCG titre of e) Patient hemodynamically stable
200IU/ml.- (UPSC 07) 1359. The drug used for medical management of ectopic
a) Single dose of methotrexate pregnancy is- (DELHI PG Mar. 09)
b) Variable doses of methotrexate a) Methotrexate b) Misoprostol
c) Expectant management c) Mifepristone d) Methyl ergometrine
d) Laparoscopic salpingostomy 1360. Spiegelberg's criteria for diagnosis of ovarian
1350. Which one of the following causes the greatest pregnancy include the following except-(UPSC-IJ09)
risk of ectopic pregnancy- (UPSC 07)
a) Tube on the affected side must be intact
a) Pelvic inflammatory disease b) Gestational sac must occupy the position of ovary
c) Gestational sac is connected with
b) Intrauterine contraceptive devices use
infundibulopelvic ligament
c) Previous ectopc pregnancy
d) Ovarian tissue should be present in the wall of
d) Previous medical termination of pregnancy
gestational sac on histopathology
1351. Medical management of ectopic pregnancy can be
1361. Tubal pregnancy is/are increased by-
done in the following except- (UPSC-II 08)
a) Recent oral combined pills use (PGI Nov 09)
a) Period of gestation 5 weeks
b) IUCDuse
b) f3HCG level6000 IU c) Previous tubal surgery
c) Absent fetal cardiac activity d) Chlamydia infection
d) Gestational sac diameter 3 em e) Previous ectopic

1343)b,c,d 1344)a 1345)d 1346)b 1347)a,b 1348)c 1349)d 1350)a 135l)b 1352)a 1353)a 1354)b
1355) a,b,d 1356) d 1357) b 1358) b,e 1359) a 1360) c 1361) b,c,d,e
GYNAE & OBS [ 65]

1362. Most sensitive diagnosis test for ectopic pregnancy- TROPHOBLASTIC DISEASE
a) Transvaginal USG (AIIMSNov09)
b) Culdocentesis 1371. Treatement of choice in vesicular mole is-(Kerala 94)
c) MRI a) Insitu hysterectomy b) Suction evacuation
d) Serial monitoring ofb-HCG c) Methotrexate d)D&C
1363. The pathognomonic feature of abdominal pregnancy 1372. The immediate complication of vesicnlar mole
evacnvation is- (Kerala 91)
is- (DPG 10)
a) Bleeding b) Infection
a) Fetus appears lateral to lumbar spine on X-ray
c) Incomplete evacuvation d) Sepsis
b) Small uterus 1373. Confirmation of the diagnosis of vesicular mole
c) Fetus easily palpable is by- (Kerala 91)
d) Positive pregnancy test a) Vaginal examination b) X-rays
1364. Conservative treatments for tubal pregnancy include c) Abdominal examination d) Ultrasonogram
thefollowingEXCEPT- (Maharashtra 10) 1374. TheatmentofHydatidiform mole in a 40 yrold women
a) Salpingostomy b) Salpingotomy is- (A/92)
c) Salpingectomy d) Resection and anastomosis a) Hysterotomy b) Hystrectomy
1365. Which one of the following is diagnosed by c) Suction evacuvation d) D and C
Spiegelberg criteria? (UP SC II 10) 1375. In case of Hydatidiform mole with age 40 yrs. &
a) Molar pregnancy b) Ovarian pregnancy Completed family, the treatment of choice is-
c) Uterine pregnancy d) Twin pregnancy a) Suction evacuation b)D&C (A/96)
1366. Which of the following is the most appropriate c) Total hysterectomy d) Panhystectomy
management in a ruptured tubal pregnancy? 1376. The point of distinction between partial mole to
a) Quick resuscitation followed by laparotomy and complete mole is- (Delhi 96)
a) Partial mole is triploid or tetraploid
excision of the offending tube (UP SC II 10)
b) Partial mole is more prone to turn malignant
b) Blood transfusion immediately after the clamps
c) Typical of partial mole is cellular atypia
have been placed to control the bleeding d) Partial mole shows trophoblastic proliferation
c) Autotransfusion of the fresh blood harvested from with absent villi
the peritoneal cavity 1377. The essential investigation to be included in
d) Excision of the offending tube and the ipsilateral follow-up of hydatidiform mole is- (Delhi 96)
ovary (salpingo-oophorectomy) a) Ultrasound abdomen b) Chest Xray
1367. True statement regarding ectopic pregnancy- c) Serum levels ofHCG d) Serum levels ofTSH
a) Pregnancy test positive (PGI May 10) 1378. Hydatidiform mole is associated with- (AIIMS 95)
b) ~hCG levels should be > 1000 mlU/ml for a) Follicular ovarian cysts b) Theca lutein cysts
earliest detection by TVS c) Ovarian carcinoma d) Ovarian atrophy
c) ~hCG levels should be < 1000 mlU/ml for 1379. For a multiparous 40 year old woman having molar
earliest detection by TVS pregnancy, the treatment of choice wonld be-
d) Methotrexate is used a) D & C followed by regular follow up (UPSC 97)
1368. Not trne about ectopic pregnancy is- b) Hysterectomy
a) Previous ectopic is greatest risk (PG!Nov.10) c) Hysterectomy and tubectomy
d) VAT followed by radiation
b) POP doesn't increase risk
1380. Which ofthe following endocrinological condition
c) Incresed risk with pelvic infections
may be associated with hydatiform mole-(Orissa 99)
d) Increased risk with IVF
a) Hypothyroidism b) Hyperthyroidism
e) IUCD use increase risk c) Diabetes d) Hyperprolactinemia
1369. Management of unruptured tubal pregnancy 1381. Gestational trophoblastic disease with jaundice,
includes- (PGI Nov. 10) best drug is- (SGPGI 05)
a) Methotrexate a) Methotrexate b) Adriamycin
b) Prostaglandins c) Actinomycin-D d) Cyclophosphamide
c) Hysterectomy 1382. The highest incidence of Gestational Trophoblastic
d) Laparoscopic salpingostomy Disease is in - (AI 05)
e) Salpingectomy a) Australia b) Asia
1370. Most common site of ectopic pregnancy- c) NorthArnerica d) Western Europe
a) Arnpullaoffallopian tube (Jipmer 11) 1383. Choriocarcinoma commonly metastasize to-
b) Isthmus a) Brain b) Lung (PGI June 06)
c) Interstitial portion of tube c) Vagina d) Ovary
d) Abdomen e) Cervix

1362)a 1363)a 1364)c l365)b 1366)b 1367)a,b,d 1368)b 1369)a,b,d,e 1370)a 1371)b 1372)a 1373)a 1374)b
1375)c 1376)a 1377)c 1378)b 1379)b 1380)b 1381)c 1382)b 1383)a,b
GYNAE & OBS [ 66]

1384. A 25 years old female was diagnosed to have 1394. Conversion of a complete hydaliform mole into
choriocarcinoma management is- (PGI June 06) invasive mole is indicated by all of the following
a) Chemotherapy except- (Al10)
b) Radiotherapy a) PlateauHCG
c) Hysterectomy b) Enlarged Uterine size
d) Hysterectomy & then radiotherapy c) Persistance of Theca-lutein cysts
1385. Molar pregnancy is diagnosed in- (Manipal 06) b) Suburethral nodule
a) I trimester b) ll trimester 1395. Conversion of a complete hydatiform mole into
c) III trimester d) All of the above choriocarcinoma is indicated by all, except -
1386. Most common gestational trophoblastic disease a) PlateauHCG levels (AI 10)
foUowing hydatiform mole is- (AI 07) b) Enlarged uterine size
a) Invasive mole c) Persistance of theca lutein cysts
b) Choriocarcinoma d) Suburethral nodule
c) Placental site trophoblastic tumour 1396. According to the FIGO staging of gestational
d) Placental nodule trophoblastic tumors, a lady with choriocarcinoma
1387. Hydatiform mole- characterized histologicaUy by- having lung metastasis will belong to which stage -
a) Hyalinemembranedegeneration (UP07) a) I b) II (Karn 11)
b) Hydropic degeneration of the villous stroma c) III d)IV
c) Non-proliferation of cytotrophoblast 1397. Treatmentofmolarpregnancyis- (Karn 11)
d) Non proliferation of syncytiotrophoblasts a) Dilatation and curettage
1388. The following conditions are associated with molar b) Hysterotomy
pregnancy except- (UPSC 07) c) Laparotomy
a) Pregnancy induced hypertension d) Suction and evacuation
b) Thyrotoxicosis
1398. In a woman with molar pregnancy with a uterus size
c) Gestational diabetes
of28 weeks, the treatment of choice is-(UPSC U 11)
d) Hyperemesis gravidarum
a) Medical induction with prostaglandins
1389. The current imaging technique of choice for the
b) Hysterotomy
diagnosis of hydatidiform mole is- (UPSC 85 07,
c) Hysterectomy
a) Computed tomography Al/MS 84, PGI 88)
d) Suction evacuation
b) Ultrasonography
c) PlainX-rayabdomen
MULTIPLE PREGNANCY
d) Magnetic resonance imaging
1390. IDtrasound of a 36 year old gravida reveals small
1399. Incidence of multiple pregnancy in Inida is on
grape-like cystic structures without the evidence
the increase because - (Kerala 91)
of a developing embryo. A diagnosis of complete
a) Change in diet b) Clomiphene therapy
hydatidiform mole is made. Further analysis is
c) Repeated MTP d) Old age of marriage
mot likely to reveal thatthe- (UPSC 07)
a) Genotype of the mole is 46 XX and is completely 1400. Which type ofTwins have the highest mortality-
paternal in origin a) Monoamniotic monochrionic (JIPMER 93)
b) Genotype of the mole is triploid b) Diamniotic dichorionic
c) hCG levels are markedly decreased c) BinovularTwins
d) Serum levels of alpha-fetoprotein are elevated d) Siamese Twins
1391. Most common site of metastasis in choriocarcinoma- 1401. Easiest method of determining zygosity is-
a) Liver b)Lung (UP08) a) Blood grouping (J/PMER 92)
c) Kidney d) Spleen b) Sex offetus
1392. TrueaboutH.mole- (PG/June08) c) Placental examination
a) Always a/w raised uterine size for gestational age d) Amniotic fluid examination
b) RaisedHCG 1402. In twin pregnancy delivery by cesarean section
c) Hysterectomy in selected cases is advocated in all the above conditions except-
d) Chemotherapy is the treatment of choice a) Second twin in transverse lie (Karn. 94)
e) Thyrotoxicosis position
1393. Which of the foUowing statements about partial mole b) First twin in transverse lie position
is false- (Al10) c) Monoamniotic twins
a) Usually associated with triploidy d) Weight less than 1500 gm.
b) Rarely causes persistatn gestational trophoblastic 1403. Double Monster, fused in pelvic region is
Neoplasia called- (Karn. 96)
c) Usually present as Missed abortions a) Ischiopagus b) Thoracopagus
d) Can be reliably diagnosed by USG in early gestation c) Synencephalic d) None ofthese

1384)a 1385)a 1386)a 1387)b 1388)c 1389)b 1390)a 1391)b 1392)a,b,c,e 1393)d 1394)d 1395)None>d
1396)c 1397)d 1398)d 1399)b 1400)d 1401)c 1402)a 1403)a
GYNAE & OBS [ 67]

1404. Most common cause of perinatal mortality in 1414. In an uncomplicted twin pregnancy normal
Twins- (CMC 98) delivery should be attempted in the following
a) Dystonia b) Hemorrhage situation- (MAHE 07)
c) Anaemia d) Interlocking a) First baby - Vertex and second baby tmnsverse
e) Prematurity lie
1405. The sepration of normally situated placenta in a b) Both babies and breech presentation
case of multiple pregnancy may be due to the c) First baby transverse lie and second longitudinal
following except- (Kerala 2K) lie
a) Increased incidence oftoxaemia d) First baby transverse lie and second frank breech
b) Sudden escape of liquor following rupture of 1415. What is the most common type of conjoined twin-
membranes a) Cranippagus b) Thoracopagus (UPSC 07)
c) DeficiencyofvitaminB 12 c) Ischiopagus d) Pygopagus
d) Deficiency of folic acid 1416. Multiple pregnancy is not associated with an
e) Sudden shrinkage of the uterus following increased with an increased incidence of which one
delivery of the first baby ofthefollowing? (UPSC-1108)
1406. Monocbrorionic monoamniotic twin occurs if a) Hyperemesis gravidarum
division occurs- (JIPMER 2002) b) Congenital malformations
a) Before 24 hrs b) 1-4 days c} Pregnancy induced hypertension
c) 4-8 days d) >8 days d) Post date pregnancy
1407. According to Hellins law chances of twins in 1417. Discordant twin complication in multiple pregancy
pregnancy are- (PGI 2000) is due to ? (APPG 08)
a) lin60 b)lin70 a) Twin Twin transfusion syndrome
c) 1 in 80 d) 1 in 90 b) Dizygotic twins
e) 1 in 10 c) Fetus papyraceous
1408. Most common type of twin pregnancy is- (PGI 97) d) Monoamniotic twins
a) Vertex+ transverse b) Both vertex 1418. Vaginal delivery may be recommended in- (AI 09)
c) Vertex+ breech d) Both breech a) Monochorionic monoamniotic twins
1409. Doppler USG in twins is used for- (PGI 98) b) First twin vertex, second twin breech
a) Twin to win transfusion b) Conjoined twin c) First twin extended breech, second twin vertex
c) Diagnosis oftwins d) AU of the above d) First twin mentoanterior, and second twin breech
1410. Multiple pregnancies occur most commonly with 1419. Vaginal delivery is allowed in all of the following,
a) T. Clomiphene (IN 03) Except- (AI 09)
b) T. Clomiphene and dexamethasone a) Monochorionic monoamniotic twins
c) T. Dexamethasone b) First twin vertex and second twin breech
d) Pulsatile GnRH thempy c) First twin extended beech an second twin vertex
1411. In superfecundation which of the following is seen- d) First twin mentoanterior and second breech
a) Fetilization of 2 ova released at same time, by 1420. Vaginal delivery can be allowed in all except-
sperms released at intercourse on 2 different a) Monochorionic, monoamniotic twins
occasions (Jharkand 03) b) Mentoanterior (AIIMSMay 10)
b) Fertilization of 2 ova released at same time by c) Extended breech
sperms released at single intercourse d) Dichorionic twins with first vertex & second breech
c) Both 1421. Which of the following statements about 'Multiple
d) None Pregnancy' is true - (AI 10)
1412. A double beaded monster is known as a -(SGPGI 04) a) Fetuses of same gender excludes dichorionicity
a) Diplopagus b) Dicephalus b) Twin peak sign is seen in dichorionicity
c) Craniopagus d) Heteropagus c) Thick separating membrane is a feature of
1413. If division of fertilized egg occurs at 4~8th day what monochorionic twins
kind of monozygotic twin pregnancy will it give d) Chorionicity can be reliably detected only after 16
rise to - (SGPGI 04) weeks of gestation
a) Diamnionic dichroionic 1422. The commonest complication of twin pregnancy
b) Diamnionic monochorionic deliveryincludes- (DPG 10)
c) Monoamnionic monochrionic a) Interlocking b) Abruptio-placentae
d) Conjoined twins c) Post-partum haemorrhage d) Obstructed labour

1404)e 1405)c 1406)d 1407)c 1408)b 1409)d 1410)d 141l)a 1412)b 1413)b 1414)a 1415)b 1416)d 1417)a
1418)b 1419)a 1420)a 1421)b 1422)c
GYNAE & OBS [ 68]

14 23. Which ofthe following is not a criterion for antenatal 1433. All are associated with hydramnios except-(PGI 2K)
diagnosis of Twin-Twin transfusion syndrome? a) Premature labour b) Gestational diabetes
a) Oligohydramnios in donor fetus c) Renal agenesis d) Increased amniotic fluid
b) Dichorionicity (Maharashtra 10) 1434. Causes of polyhydramnios include- (PGI OJ)
c) Hemoglobin difference of> 5 g/dL between the a) Diabetes mellitus b) Preeclampsia
two foetuses c) Esophageal atresia d) Renal agenesis
d) Weight difference of > 20% between the two e) Anencephaly
1435. Causes ofHydramnios-
foetuses
a) Anencephaly b) Oesophageal atresia
1424. Monoamniotic monochorionic twins develop when
c) Renal agenesis d) Posterior urethral value
the division of cell mass occurs- (UP SC II I 0)
e) Twins
a) Within 72 hours after fertilization 1436. Polyhydramnios is associated with all except-
b) Between 4th and 8th day of fertilization a) Diabetes b) Open spina bifida
c) After 8th day of fertilization c) Multiple pregnancy d) Renal agenesis
d) After 2 weeks of development of embryonic disc 1437. Polyhydramnios not associated with -(AIIMS Nov 07)
1425. Blood chimerism is most likely to occur in -(Alii) a) BIL renal agenesis b) Anencephaly
a) Monochorionic Monozygotic twins c) Open spina bifida d) Tracheoesophageal fistula
b) Monochorionic Dizygotic twins 1438. Non-immune hydrops fetalis is due to the following
c) Dichorionic Dizygotic twins except- (UPSC-II 08)
d) Singleton pregnancy a) a thalassemia major
b) Functional abnormalities of heart
HYDRAMNIOS AND OLIGOHYDRAMNIOS c) ABO incompatibility
d) Toxoplasmosis
1426. Polyhydramnios is not seen in- (Jipmer 91) 1439. AU of the following are causes of polyhydramnios
a) Anencephaly except- (DPGEE 08)
b) Amnion nodosum a) Material diabetes
b) Hydrocephalus
c) Esophageal atresia
c) Anencephaly
d) Tracheo esophageal fistula
d) Chorioangioma of placenta
1427. Bleeding is offetaloriginin- (JIPMER 91) 1440. Chorangioma ofplacenta associated with? (APPG 08)
a) Placenta previa b) Vaso previa a) Oligohydramnios b) Polyhydramnios
c) Placenta accreta d) Abruptio placenta c) Premature labour d) IUGR
1428. Fetal blood loss occurs in- (AJIMS 92) 1441. Which ofthe following conditions is associated with
a) Uterine rupture b) Placenta previa polybydraminios- (AI 10)
c) Vasa previa d) Circumvallate placenta a) Posterior Urethral valves
1429. Placenta with umbilical cord attached to its b) Cleft palate
margin is called - (Karn 99) c) Congenital diaphragmatic hernia
a) Battledore placenta d) Bladder exstrophy
b) Circumvallate placenta 1442. Oligohydramnios is/are associated with -(PGIMay I 0)
c) Succenturiate placenta a) Neural tube defect b) Renal agenesis
d) Velamentous placenta c) Postmature birth d) Premature birth
1430. Clinical signs ofHydraminos can be demonstrated
when fluid collection is more than- (Orissa R) HYPERTENSIVE DISORDERS IN PREGNANCY
a) lltr. b) 2 ltr.
c) 3ltr. d) 4ltr. 1443. Drug of choice for pregnancy induced
1431. Primary antiphospholipid syndrome is hypertension is- (Al91)
characterised by all of the following clinical a) Hydrallazine b) Clonidine
features except- (UPSC 96) c) Labetelol d) Propranolol
a) Recurrent foetal loss 1444. Postpartum eclampsia starts after how many
b) Arterial and venous thrombosis hours of delivery- (DNB 91)
c) Foetal cardiac abnormalities a) 12-24 b)24-48
d) Intrauterine growth retardation c) 48-72 d) 72-96
1432. Oligohydramnios is associated with- (AIIMS 85) 1445. The single most effective drug in eclampsia
a) Anencephaly b) Esophageal atresia is- (AMU 85, PGI 78, Kerala 94)
a) Dilantin b) Phenobarbital
c) Renal agenesis d) All of the above
c) Magnesium sulphate d) Paradehyde

1423)b 1424)c 1425)b 1426)b 1427)b 1428)c 1429)a 1430)b 143l)None 1432)c 1433)c 1434)a,c,e
1435)a,b,e 1436)d 1437)a 1438)c 1439)b 1440)b 144l)b 1442)b,c 1443)a 1444)a,b 1445)c
GYNAE & OBS [ 69]

1446. Patient of eclmpsia on magsulph regime are 1457. Which of the following seen in pre eclampsia-
monitored by all ofthe follwing except- (UPSC 96) a) Hypertension b) Proteinuria (PGI 01)
a) Respiratory rate c) Convulsions d) Pedal edema
b) Urine output e) lUGR
c) Krieejerk 1458. Drug which can be used in pregnancy with
d) Serum Na and K levels eclampsia at 32 weeks of gestation is- (Jipmer 03)
144 7. One of following antihypertensives is not used in a) Enalapril b) Lorsatan potassium
c) Hydralazine d) Frusemide
pregnancy- (AI 96)
1459. InseverePffi,BPis- (MAHE 05, Karn 95)
a) Enalapril b) Methyldopa
a) 160/lOOmmHg b) 150/lOOmmHg
c) Hydralazine d) Nifedipine c) 160/lOOmmHg d) 120/80mmHg
1448. All of following are predictive tests for Pm except 1460. Notagravesignofpre-eclampsia- (APPGE 05)
a) Rolling over test (AI 96) a) Headache
b) Urinary protein b) Decreased reflexes
c) Gain in weight> 2kg in one in one month c) Epigastric discomfort
d) Ultrasound d) Urine output< 600 ml/day
1449. Cause of convulasion in ecclamopsia (KERAlA 96) 1461. Which of the following drug is contraidicated to
a) Cerebral anoxia due to aterial spasm treat hypertension with pregnancy- (AIIMS 05)
b) Hypovolemia a) Enalapril b) Methldopa
c) Hypocalcemia c) Nifedipine d) Labetolol
d) Shock 1462. Which of the following antihypertensives is not
1450. Proved to be effective in the management of safe in pregnancy- (AIIMS Nov 05)
preeclampsia- (PGI 96) a) Clonidine b) ACE inhibitors
a) Zinc b)Calcium c) a-methyldopa d)Amlodipine
c) Magnesium d) None 1463. A 27-year primigravida presents with pregnancy
1451. Earliest sign of preeclampsia is- (AIIMS96, induced hypertension with blood pressure of 150/
a) Hypertension Jipmer98) 100 mmHg at 32 weeks of gestation with no other
b) Albuminuria complications. Subsequently, her blood pressure
c) Paedal oedema is controlled on treatment. If there are no
d) Excessive weight gain complications, the pregnancy should be best
1452. Pregnancy induced hypertention, sudden vision terminated at- (AIIMS 06)
loss is due to- (JIPMER 99) a) 40 completed weeks b) 37 completed weeks
a) Retinal detachment b) CRAO c) 35 completed weeks d) 34 completed weeks
1464. Consider the following statements- (UPSC 06)
c) Vitreous hemorrhage d)CRVO
1453. A24 year old woman with 36 weeks of pregnancy, The-requisites for giving magnesium sulphate in the
treatment of eclampsia include -
suddenly complains of headache and blurring of
1. More than five fits
vision. Her B.P is 170/110 mm of Hg. Urinary
2. Patient not being in labour
albumin is +++and fundus examination shows
3. Availability of calcium gluconate by the side of
areas of retinal haemorrhage. The line offurther
the patient.
management would be- (UPSC 2K)
4. Presence of knee-jerk
a) Conservative treatment
a) 1 and 2 only b) 3 and4 only
b) Anticonvulsive therapy
c) 1,2and3 d)2,3and4
c) Induction of labour 1465. Pregnancy induced hypertension is hypertension
d) Caesarean delivery that develops after- (Karn pgmee 06)
1454. Eclampsia is mostly seen at- (CULCUITA 2K) a) 18 weeks of pregnancy b) 20 weeks ofpregnancy
a) Antepartum b) Intrapartum c) 24 weeks of pregnancy d) 28 weeks ofpregnancy
c) Postpartum d) During parturition 1466. The imminent treatment of eclampsia include all
1455. Giant's roll over test for PIH is except- (UP 07)
doneat- (ORRISAR) a) Tocolytics
a) 22-24 weeks b) 28-32 weeks b) Prevention of fits
c) 24-26 weeks d) 32-34 weeks c) Contemplate delivery
1456. In PIH an impending sign of eclampsia is- d) Decreases blood pressure
a) Visual disturbance (PGI 98) 1467. Most important factor in management of
b) Wt gain of 2 Ib per week preecclampsia- (PGIJune07)
c) Severe proteinuria of I Og a) Severity of hypertension ·
d) Pedal edema b) Proteinuria

1446)d 1447)a 1448)b,d 1449)a 1450)c 145l)d 1452)a 1453)b,c 1454)a 1455)b 1456)a 1457) a,b,d,e
1458)c,d 1459)a 1460)a 1461)a 1462)b 1463)b 1464)b 1465)b l466)a 1467)a
GYNAE & OBS [ 70]

1468. Which of the following medications is the drug of controlled on treatment. If there are no
choice for severe preeclampsia? (AI 08) complications, the pregnancy should be best
a) Labetalol b) Metoprolol terminated at- (DPG 10)
c) Alpha methyldopa d)Nifedipine a) 40 completed weeks b) 37 completed weeks
1469. Risk factor for pre-eclampsia- (PGI June 08) c) 35 completed weeks d) 34 completed weeks
a) Chronic hyertension b) Smoking 1479. Risk factors for pre-eclampsia includes-
c) Obesity d) Multiparity a) Age> 35 yr (PGI May 10)
e) Placenta praevia b) Obesity
1470. Drug of choice in eclampsia ? (APPG 08) c) Previous h/o preeclampsia
a) Magnesium sulphate b) ACE inhibitors d) Multigravida
c) Ringer lactate d) All e) Antiphospholipid syndrome
1471. All of the following may be seen in severe pre- 1480. TrueaboutMgS04 - (PGIMaylO)
eclampsia/eclampsia except? (AI 09)
a) Tocolytic
b) Used in Management of eclampsia
a) Cerebral hemorrhage b) Pulmonary edema
c) Cause neonatal respiratory depression
c) Severe bowel ischemia d) Acute renal failure
1481. Best regimen for Eclampsia is- (AIIMS Nov 1 0)
1472. A30 year priml has 36 week pregnancy B.P ofpatient
a) MgSO b) Lytic cocktail
is 160/11 0. Urine albumin is 3+ & platelet count is 4
c) Phenytoin d) Diazepam
80000/nm3• Management of patient includes- 1482. A 37-year old second gravid, previous LSCS at 37
a) Betamethasone (PGI June 09) wks of pregnancy presents with blood pressure of
b) MgS0 4 150/100, urine albumin++. On pelvic examination
c) Labetalol cervix is found to be soft with 50% effacement,
d) Immediate caesarean section station is - 3, pelvis adequate and cervical os is closed.
e) Induction oflabour Most appropriate step at the momentwouid be-
1473. lnPffi an impending sign of eclampsia is- a) Antihypertensive regime and then induce labour
a) Visual disturbance (DELHI PG Feb. 09) b) Wait and watch for 10 days (AIIMS Nov 1 0)
b) Wt. gain of2lb per week c) Induce labour spontaneously
c) Severe proteinuria of 10 kg d) Do caesarean section
d) Pedal edema 1483. Which can be used in pregnancy- (AIIMS May 11)
1474. The drug of choice for prevention of seizures in a a) ACE inhibitors b) Aldosterone
patient with severe preeclampsia is- (DELHI PG Mar. c) AT receptor antagonist d) Propylthiouracil
a) Phenytoin b) Magnesium sulphate 09) 1484. Which of the following is the earliest sign of
c) Diazepam d) Nifedipine magnesium toxicity in a patient with ecclampsia on
1475. What is the most commonly observed fetal effect in treatmentwithmagnesiumsulphate- (AI 11)
women receiving magnesium sulphate therapy for a) Loss of deep tendon reflexes
pre-eclampsia/eclampsia- (UPSC-II 09) b) Respiratory Depression
a) Respiratory depression c) Crdiac arrest
b) Variability in fetal heart rate pattern d) Decreased urine output
c) Cerebral palsy 1485. During a routine prenatal visit, a 22-week gravid
d) Intestinal obstruction woman is found to be affected with ankle oedema and
1476. The following changes may be seen in pre-eclampsia new onset hypertension. The urine analysis reveals
marked proteinuria. Which of the following, if it
except- (UPSC-II 09)
were to occur, would substantiate the diagnosis of
a) Decreased antithrombin III
eclampsia- (UPSCI/ 11)
b) Hemodilution
a) Molar pregnancy b) Hyperuricaemia
c) Thrombocytopenia c) Seizures d) Thrombocytopenia
d) Elevated uric acid 1486. A pregnant woman presented with pre eclampsia.
1477. Which of the following is seen in severe Intravenous magnesium sulphate is given. What is
preeclampsia except- (PGI Nov 09) the first sign of hypermagnesemia? (Jipmer 11)
a) Oliguria a) Loss oftendon reflex
b) Pulmonary oedema b) Respiratory depression
c) Foetal growth restriction (IUGR) c) Hypotonia
d) Diastolic BP > 110 mmHg d) Hypotension
e) SystolicBP:?: 160mmHg 1487. Proteinuria in pre-eclampsia is due to-
1478. A 27-year old primigravida presents with pregnancy a) Glomerulardamage (Jipmer 11)
induced hypertension with blood pressure of 150/ b) Degree of hypertension
100 mrn!Hg at 32 weeks of gestation with no other c) Release of vasoconstrictors
complications. Subsequently, her blood pressure is d) Liverfailure

1468)a 1469)a,c 1470)a 1471)c 1472)a,e 1473)a 1474)b 1475)a 1476)b 1477)None 1478)b 1479)All 1480)a,b,c
148l)a 1482)d 1483)d 1484)a 1485)c 1486)a 1487)a
GYNAE & OBS [ 71 ]

ANTEPARTUM HAEMORRHAGE 1498. In accidental haemorrhage, TOC- (PGI 98)


a) Induction of labour
1488. The following is false of placenta previa- (AI 92) b) Rx ofhypofibrinogenemia then blood transfusion
a) Associated with toxemia c) Stimultaneous empying of uterus and blood
b) Painless recurrent bleeding transfusion
c) Maternal blood loss d) Wait and watch
d) Severe bleeding may occur 1499. Couvelaire uterus is seen in- (PGI 98)
1489. The first consideration in the treatment of a) Placenta previa b) Accidental haemorrhage
placetal abruption is- (AIIMS 79, 84) c) PIH d)PPH
a) Immediate delivery 1500. Termination of pregnancy in placenta praevia is
b) Administration of fibrinogen indicated in- (PGI 03}
c) Amniocentesis to establish diagnosis a) Active bleeding
d) Promt restoration of an effective circulation with b) Active labour
l.V. fluid c) Gestational age> 34 wks with live fetus
e) Oxytocin to effect early delivery d) Fetal malformation
1490. Expectant line of management in placenta previa e) Unstable lie
is contra-inidcated in- (Karn 94) 1501. Most common cause of placenta previa-
a) Preterm fetus b) Live fetus a) Myomectomy b) Primigraida
c) Breech presentation d) Active labour c) Multigravida d) Previous casanian section
1491. One of following is not true about vasa previa- 1502. Causes of ante-partum hemorrhage are all except-
a) Bloodofmaternalorigin (AI96) a) Placenta previa (TN03)
b) May be a cause of antepartum hemorrhage b) Atonic uterus
c) Blood offetal origin c) Abruptive placenta
d) Singer's test positive d) Circumvalate placenta
1492. In a case of antepartum hemorrhage with BP 1503. Antepartum hemorrhage occurs after how many
80/60 mmHg, treatment of choice is - (AI 96) weeks- (TN 04)
a) Blood transfusion b) Morphine a) 12 weeks b) 18 weeks
c) Placentography d) Panhystectomy c) 20 weeks d) 28weeks
1493. All are true about abruptia placenta 1504. Classical presentation ofplecenta previa-
except- (AIIMS 98) a) Anterior b) Central (MAHE 05)
a) Absent foetal heart b) Uterine tenderness c) Lateral d) Posterior
c) Profuse bleeding d) Hypotension 1505. A primigravida presents to casually at 32 weeks
1494. Covellier's uterus seen in- (PGI 98) gestation with acute pain abdomen for 2 hours,
a) Accidental hemorrhage vaginal bleeding and decreased fetal movements.
b) Pregnancy with DIC She should be managed by- (AI 06)
c) Placenta previa a) Immediate cesarean section
1495. A pregnant woman presents with a placenta b) Immediate induction oflabor
pravia of a major degree. The fetus is malformed. c) Tocolytic therapy
Which of the following will be the best line on d) Magnesium sulphate therapy
management- (JIPMER 2K) 1506. In placenta previa conservative treatment not
a) Cesarian section b) Oxytocin drip done in case of- (PGI June 06)
c) Ruptureofmembranes d)InstillationofPGE2 a) Active labour b) Anencephaly
1496. A second gravida P+O+O presented 32 weeks of c) Dead baby d) Severe placenta previa
pregnancy with grade III placenta previa and e) Premature fetus
contractions. What is the treatment of choice- 1507. Couvelaire uterus present in- (UP 07)
a) Bed rest and sedation (AIIMS 2K) a) Placenta praevia b) Abruptio placentae
b) Bed rest and dexamethasone c) Vas praevia d) All of the above
c) Rest, nifidepine and dexamethasone 1508. Medial to lateral structure ofFallopina tube are-
d) Bed rest and nifidepine a) Ampulla, Isthmus, Infundibulum (UP 08)
1497. A positive "Stallworthy's sign" is suggestive of b) Infundibulam, isthmus, Ampulla
which of the following conditions- (Kerala 2K) c) Interstitial, isthmus, ampulla, infundibulum
a) Twin pregnancy d) Infundibulam, ampulla, isthmus, Interstitial
b) Breech presentation 1509. Which ofthe following drugs is not useful in the
c) Vesicularmole management of postpartum hemorrhage (PPH)-
d) Low lying placenta a) Mifepristone b) Misoprostol (AI 08)
e) Pregnancy induced hypertension c) Oxytocin d) Ergotamine

1488)a 1489)d 1490)d 1491)a 1492)a 1493)c 1494)a 1495)a 1496)None 1497)d 1498)c 1499)b 1500)a,b,d
150l)c 1502)b 1503)d 1504)d 1505)a 1506)a,b,c 1507)b 1508)c 1509)a
GYNAE & OBS [ 72]

1510. All are causes of antepartum hemorrhage except? MEDICAL & SURGICAL ILLNESS
a) Battledore placenta (DPGEE 08) COMPLICATING PREGNANCY
b) abruptio placenta
c) Circumvallate placenta 1520. Unequivocal evidence of heart disease in
d) Placenta praevia pregnancy is- (Kerala 91)
a) Systolic murmur
1511. Which of the following is true about vasa previa
b) Diastolicmurmur
except? (AIIMSMay09)
c) Diastolic thrill and murmur
a) Incidence is 1:1500 d) Systolic thrill and murmur
b) Mortality rate of20% with undiagnosed case 1521. Which is not transmitted to the baby at delivery-
c) Associated with low lying placenta a) Toxoplasmosis (JIPMER 91)
d) Caesarian section is indicated b) gonococcus
1512. The risk factors for placental abruption include the c) Herpes simplex type II
following except- (DELHI PG Mar. 09) d) Hepatitis-B
a) History ofplacental abruption in previous pregnancy 1522. Which of the following is correct about
b) Hypertension Asymptomatic bacteriuria during pregnancy-
c) Diabetes mellitus a) 5% progress to UTI (JIPMER 91)
d) Preterm rupture of membranes b) Seen in 20% of pregnant women
c) Not associated with Renal anomalies
1513. Abortions in the second trimester mainly occur due
d) Seen only in mulltigravida
to- (UP SC II 10)
1523. Which heart disease has the worst prognosis in
a) Retroverted gravid uterus pregnancy- (AI 92)
b) Congenital anomalies of the uterus a) Pulmonary stenosis b) MS
c) Congenital anomalies of the foetus c) VSD d)ASD
d) Hormonal deficiencies 1524. Pregnant lady on oral anticoagulants should
1514. The regimen used for expectant management of swithcb over to Heparin at- (AIIMS 92)
placentapreviais- (AIIMS Nov 10) a) 6weeks b) 12 weeks
a) Macafee & Johnson regimen c) 24weeks d)36weeks
b) BrandtAndrewmethod 1525. Surgery for mitral stenosis during pregnancy is
c) Credes method ideally done at- (AIIMS 92)
d) Mcdonald's regimen a) 14weeks b)20weeks
1515. Classifiction for abruptio placenta is-(AIIMSNov 10) c) 28 weeks d) 32 weeks
1526. A multipara with 34 weeks pregnancy tachycardia,
a) Page's classification b) Jhonson regimen
fever hepatosplenomegaly, pallor bas- (UPSC 98)
c) Apt staging d) Macafee regimen a) Malaria
1516. Which test differentiate maternal and fetal blood b) Iron deficiency anaemia
cell- (AIIMSNov10) c) Physiological anaemia
a) Apt test b) Kleihauer-Betk:e test d) Megaloblastic anaemia
c) Bubblin test d) Osmotic fragility test 1527. Which of the following is an absolute indication
1517. All oftbe following are included in the expectant for cesarian section in pregnancy associated with
management of placenta praevia, except- (AI 11} heart disease- (AIIMS 2K)
a) Cervical Encirclage a) Pulmonary stenosis b) Coarctation of aorta
b) Anti- D administration c) Eisenmenger syndrome d) Ebstein's anomaly
c) Corticosteroids 1528. All are correct of management of Heart disease
d) Blood transfusion during labour except- (AIIMS 92)
1518. A fourth-gravida with three living children presents a) Minirnaltrialoflabour
at 38 weeks of pregnancy with abdominal pain and b) Shortening of second stage of labour
c) LSCS only for obstetrical indications
vaginal bleeding. On examination, the uterus is tense
d) Induction at 38 weeks
and tender, and the foetal heart sounds are absent.
1529. DIC is seen in- (Kerala 94)
What is the probable diagnosis- (UPSC II 11)
a) Retained placenta b) Abruptio
a) Accidental haemorrhage b) Placenta praevia c) Intra uterine death of foetus d) All
c) Vasa praevia d) Ectopic pregnancy 1530. The commonest pathogen responsible for acute
1519. Antepartum hemorrhage offetal origin is seen in- pyelonephritis in pregnancy is- (Karn. 94)
a) Abruption placenta (Jipmer 11) a) Escherichia coli
b) Placenta previa b) Kiebsiella pneumoniae
c) Vasa previa c) Proteus rnirabilis
d) Circumvallateplacenta d) Pseudomonas aeruginosa

1510)a 1511)b 1512)c 1513)b 1514)a 1515)a 1516)a 1517)a 1518)a 1519)c 1520)b,c,d 1521)a 1522)None
1523)b 1524)d 1525)a 1526)a 1527)b 1528)d 1529)d 1530)a
GYNAE & OBS [ 73]

1531. Which of the following is not a complication of 1543. Pregnancy should be stongly discouraged in
fibroid in pregnancy- (JIPMER 95) womenwith- (UPSC)
a) Pre term labour b) Postpartum haemorrhage a) Mitral stenosis b)ASD
c) Abortion d)None c) VSD d) Eisenmenger's syndrome
1532. Causes ofDIC are all except- (AI 95) 1544. Maternofetal transmission in toxoplasmosis
a) Abruption placentae b) Fat emboli occurs maximally in- (AI 99)
c) Amnioticfluidembolism d)IUD a) 1"1 trimester b) 2nd trimester
1533. In coagulation failure serum level of fibrinogen
c) 3m trimester d)At delivery
is less than- (Karn. 95)
a) l50mg b) lOOmg 1545. In pregnancy sulfonylurea are stopped and
c) 200mg d)250mg insulin is started becuase of all except -(AIIMS 98)
1534. The most important cause of the coagulation a) Sulphonylureas cannot control sugar well due to
failure in obstetrics is - (Karn. 96) hyperglycemia
a) Intrauterine foetal death b) Sulphonylureas predispose to hypoglycemia in
b) Accidental haemorrhage infant
c) Placenta previa c) Insulin cannot cross placenta
d) Rupture of the uterus d) Sulphonylureas causes pregnancy induced
1535. Drug of choice for medical termination of hypertension
pregnancy in a second gravida with asthma at 16 1546. A woman, G2 P1, presents with 10 weeks gestation
weeks gestation is- (UP 97) and history ofDVT in the previous pregnancy in
a) Prostaglandin puerperium. The management includes all except-
b) Ethacridine lactate a) Heparin to be started immediately (AIIMS 99)
c) Hypertonic saline b) Warfarin to be continued 6 weeks after delivery
d) Intra-amniotic dexamethasone c) Risk of thrombosis recurring is 10-12%
1536. Maximum maternal mortality is with-(Al.lli1S 97, 06) d) No anticoagulant needed in antenatal period
a) Hepatitis B b) Hepatitis C 154 7. The transmission ofHSV to the baby is maximum
c) Hepatitis D d) Hepatitis E at- (AHMS 99)
1537. Most common heart disease in pregnancy is- a) l'1 trimester b) 2nd trimester
a) ASD b)Eisenmenger's (A/97) c) 3m trimester d) At parturition
c) MS d)AS 1548. During pregnancy corrective cardiac surgery is
1538. Indication ofAcylovir in pregnancy- (PGI 96) commonlyindicatedin- (UPSC 2K)
a) Disseminated herpes a) Mitral stenosis
b) Chickenpox in first trimester b) Aortic stenosis
c) Prophylaxis in recurrent herpes c) Atrial septal defect
d) All of the above d) Ventricular septal defect
1539. Mode of delivery in pregnant lady with herpes 1549. The best test in diabetic mother to diagnose
depends upon- (PGI 96)
baby'slungmaturity- (AIIMS 99)
a) Active lesion b) Previous history
a) Phosphatidyl inositol
c) Cell culture d) None of the above
b) Sphingomyelin/Lecithin ratio·
1540. Dessiminated intravascular coagulation is present
c) Ultrasonography
in all except- (AIIMS 96)
a) Prolonged pregnancy d) None of the above
b) Amniolotic fluid embolism 1550. Which of the following is the most dangerous
c) Septic shock cardiac lesion in pregnancy- (Al/MS 2K)
d) Abruptio placenae a) Aortic stenosis
1541. Glucosuria during routine investigation of b) Mitral regurgitation
antenatal visit indicates that there is need for- c) Pulmonary stenosis
a) Gestational diabetes treatment (UPSC) d) Mitral valve prolapse
b) Dietary control 1551. A G2Pl +0+0, Diabetic mother presents at 32
c) lnsuline treatment weeks of pregnancy. She gives a history of full
d) Glucose tollerance test screening term fetal demise during her last pregnancy. Her
1542. During delivery, the risk of transmission of sugar is now controlled and the child is stable.
maternal infection to the foetus is the highest in What is the best plan of action- (AIIMS 2K)
a) Rubella (UPSC) a) Await spontaneous delivery
b) Cytomegal virus b) Induce at 38weeks
c) Herpes simplex virus c) Induce at 40 weeks
d) Human papilloma virus d) Cesarian section 38 weeks

l53l)d 1532)b 1533)b 1534)b 1535)c 1536)a 1537)c 1538)d 1539)a 1540)a 154l)d 1542)c 1543)d 1544)c
1545)d 1546)d 1547)d 1548)a 1549)a 1550)a 1551)d
GYNAE & OBS [ 74]

1552. In valvular heart diseases complicating pregnancy 1564. In pregnancy, which type of anemia is not common
the following statements are true except- inlndia- (PG/97)
a) A closed mitral valvulotomy (Kerala 2K) a) VitaminB 12 anemia
can be carried out if symptoms of mitral stenosis b) Folicacidanemia
are severe c) Iron+ folic anemia
b) Open heart surgery is associated with a reduction d) Iron deficiency anemia
in foetal loss 1565. Baby born to DM have following except- (PGI 98)
c) Mitral regurgitation is usually well tolerated a) Hypercalcemia b) Hypokalemia
d) A maternal mortality of 15% has been reported c) Hypoglycemia d) Obesity
in women with critical aortic stenosis 1566. In heart disease, prophylactle forceps is applied
at head station of- (PGI 98)
e) It is mandatory to continue full anticoagulation
a) -1 b)+1
in patients with mechanical valvular prosthesis
c) 0 d)+2
1553. The following complications are likely to increase 1567. In diabetes, which can occur in the fetus- (PGI 0 J)
in a case severe anaemia during the pregnancy a) Pre-eclampsia b) Polyhydranmios
except- (Kerala 2K) c) Fetal anomalies d) Placenta previa
a) Pre-eclampsia b) Inter-current infection e) Abruptio placentae
c) Heart failure d) Preterm labour 1568. True about indications for prevention of Rh-
e) Sub involution isoimmunization- (PGI OJ)
1554. Large placenta is seen in which of the following- a) Given to the newborn within 72 hrs. ofbirth
a) IUGR b) Syphilis (Kerala OJ) b) Required when baby is Rh+ & mother Rb-
c) CMV d) Rubella c) Can be helpful in ABO incompatibility
1555. Respiratory distrees syndrome in the new born is d) Can be given upto one month of age of baby
commonly assocaited with the following conditions 1569. True about fatty liver of pregnancy- (PGI OJ)
except- . . (UPSC2002) a) Common in third trimester
a) Diabetic pregnancy b) Microvesicular fatty changes
b) Prematurity c) Lysosomal injury is the cause
c) Following an elective caesarean se.ction d) Alcohol is main cause
d) Intrauterine growth retardation e) Recurrence is very common
1556. In pregnancy amount of glucose used in GTT- 1570. A lady presents with 16 weeks pregnancy with
a) 50mg b) 75 mg (CMC 200J) acute appendicitis. Management includes-
c) 100mg d) 125 mg a) Conservative (PG/02)
1557. Follicular ovarian cysts are seen in- (Jipmer 92) b) Do early surgery
a) Molar pregnancy b) Tubal pregnancy c) Appendicectomy with MTP
c) Fibroid uterus d)Threatened abortion d) Appendicectomy after childbirth
1558. With iron treatment Hb increases by.,. (JIPMER 98) e) Medical therapy and appendicectomy onreccurence
a) 2 gntlweek b) 1 gntlweek 1571. Indications of oral glucose tolerance test (GTT)-
c) 2 gmtmonth d) 1 gntlmonth a) H/0 DM in maternal uncle (PGI 03)
1559. Not implicated in congenital transmission is- b) Toxaemia of pregnancy
a) Hepatitis A b) Toxoplasmosis (UP 96) c) Previous big baby
c) Herpes d) Syphilis d) Previous H/0 gestational diabetes
1560. Rh isoimmunisation is increased in all except- e) Hydropic degeneration
a) Increased maternal age (AIIMS 97) 1572. Indication of anti-D immunoglobulin is/are-
b) Caesarean section a) Vaginal bleeding (PGI 03)
c) APR b) Fn1
d) Post dated pregnancy c) Mid trimester abortion
1561. Treatment of toxoplasmosis in pregnant women d) After amniocentesis
is with - (Karn 94) 1573. For antenatal fetal monitoring in a diabetic
a) Spiramycin b) Suramin pregnancy all· of the following are useful
c) Pyrimethamine d)Nifurtimox except- (Karnataka 02)
1562. Commonest cardiac lesion seen pregnancy is- a) Non - stress test b) Biophysical profile
a) MR b)AS . (TN 95) c) Doppler flow study d) Fetal kick count
c) MS d) Carctation of aorta 1574. Hypothyroidism is associated with the following
1563. Hydrops foetalis is seen in following except- clinical problems, except- ( UPSC 04)
a) Rh incompatibility b) Syphilis (PGI 97) a) Menorrhagia b) Early abortions
c) ABO incompatibility d)CMV c) Galactorrhoea d) Thromboembolism

1552)b 1553)e 1554)b,d 1555)d l556)c 1557)a 1558)b 1559)a 1560)a 1561)a,c 1562)c 1563)None
1564)a 1565)a 1566)d 1567)b,c 1568)b 1569)a,b l570)b,c 157l)a,c,d,e 1572)a,b,c,d 1573)d 1574)d
GYNAE & OBS [ 75]

1575. Zidovudine given for HIV in pregnancy 1583. During first trimester of pregnancy risk of fetal
because- (UPPGMEE04) malformation in a pregnant woman with insulin
a) Decreases chance of vertical transmission dependent diabetes is best predicted by -
b) Decrease severity of infection transmission a) Blood sugar values (Karnat 05)
c) Decrease severity of infection in mother b) Glycosylated haemoglobin levels
d) Cause no benefit c) Serum alpha fetoprotein levels
d) Serum unconjugated estriol levels
1576. A fetomaternal transfusion of more than 30 ml
1584. With oral iron therapy, rise in Hb% can be seen
has been found in what percentge of women at
after- (MAHE 05)
delivery? a) 1 week b) 3 weeks
a) Less than 1% b) 5% c) 4 weeks d) 6 weeks
c) 100/o d) 15% 1585. Antitubercular drug contraindicated in
1577. You are called to the operating room. The general pregnancy- (PGI June 05, Orrisa 98, PGI 89, OJ)
surgeons have operated on a woman to rule out a) Streptomycin b) Rifampicin
appendicitis and the signs of an abominal pregnancy c) INH d) Ethambutol
with an 18 week fetus and placeta attached to the e) Pyrazinamide
omentum. The best course of action in the case is- 1586. The short retroviral regime administration in the
a) Removal of both fetus and placenta peripartum period decreases the risk of vertical
b) Laparoscopic ligation of umbilical cord transmission by- (MAHE 05)
c) Removal of the fetus only a) 30% b)50%
d) Closely follow until viability and then deliver by c) 65% d)75%
laparotomy 1587. In pregnancy, the most common cause of transient
1578. Causes of non - immune hydrops fetails are all diabetes insipidus is- (MAHE 05)
except- (Jipmer 03) a) Severe pre-eclampsia b) Hydramnios
c) Multiple pregnancy d)IUGR
a) Parvo virus infection b) Down's syndrome
1588. A mother is HbsAg positive and anti HbcAg
c) a - thalassemia d) Rh - incompatibility
positive. Risk of transmission of hepatitis B in
1579. Following renal disorder is associated with worst child is - (MAHE 05)
pregnancy outcome- (AIIMS 03) a) 20% b)50%
a) Systemic lupus erythematosus c) 0% d)90%
b) lgA neuropathy 1589. Transplacental transmission of toxoplasmosis
c) Autosomal dominant polycystic kidney disease occurs maximally when- (APPGE 05)
d) Scleroderma a) Infection to mother occurs within 6 months before
1580. A Para II poorly compensated cardiac patient has pregnancy
delivered 2 days back. You will advice her to- b) Infection to mother occurs beyond 6 months
a) Undergo sterilization (tubectomy) after 1 week before pregnancy
b) Undergo sterilization after 6 weeks (UPSC 04) c) In first trimester
c) Suggest her husband to undergo vasectomy d) In last trimester
d) Take oral contraceptive pills after 6 months 1590. Which of the following test is the most sensitive
1581. Which of the following is not an indication for for detection of iron depletion in pregnancy-(AIJMS
antiphospholipid antibody testing? (All India 04) a) Serum iron b) Serum transferrin Nov 05)
c) Serum ferritin d) Serum Erythropoitin
a) 3 or more consecutive first trimester pregnancy
1591. Infants of diabetic mother are likely to have the
losses
following cardiac anomaly- (AI 05)
b) Unexplained cerebrovascular accidents a) Coarctation of aorta
c) Early onset severe pre- eclampsia b) Fallot's tetralogy
d) Gestational diabetes c) Ebstein's anomaly
1582. A class C diabetic patient delivers at term, it is d) Transposition of great arteries
important to check her blood sugar levels immediately 1592. Which of the following statements is incorrect in
postpartum, since there may be a decrease in the relation to pregnant women with epilepsy- (AI 05)
insulin requirements of diabetic paitents. This can a) The rate of congenital malformation is increased
be partly explained by- in the offspring of women with epilepsy
a) Increased food intake b) Seizure frequency increases in approximately
b) Decreased activity 70%ofwomen
c) Decrease in plasma chorionic somatomammotropin c) Breast feeding is safe with most anticonvusants
d) Decrease in plasma progesterone d) Folic acid supplementation may reduce the risk
of neural tube defect

1575)a 1576)a 1577)c 1578)d 1579)a 1580)c 158l)d 1582)c,d 1583)b 1584)b 1585)a 1586)b 1587)a
1588)d 1589)d 1590)c 159l)d 1592)b
GYNAE & OBS [ 76]

1593. The drug of choice in treatment of typhoid fever 1604. Acute pyelonephritis in pregnancy all of the
in pregnancy is- (AIIMS NOV 05) following are true except- (Manipal 06)
a) Ampicillin b) Chloramphenicol a) Left kidney is inolved in 50% of patients
c) Ciprofloxacin d) Ceftriaxone b) Most common isolate is E. coli
1594. Not complication of gestational diabetes c) More common in later 1/2 of pregnancy
mellitus - (Manipal 06) d) Responds to amino glycosides
a) Hydrocephalus b) Macrosomia 1605. Caudal Regression Syndrome is seen in babies of
c) Monilial vagintis d) Nephropathy mothers having- (AI 07)
1595. At what period does tuberculosis flare up most a) Gestational diabetes b)PIH
commonly in a pregnant patient- (AI 06) c) Cardiac disease d) Anaemia
a) First trimester b) Second trimester 1606. A large Baby is born. The likely complication in
c) Third trimester d) Puerperium pregnancy is- (AI 07)
1596. Which of the following tumors is not commonly a) Gestational diabetes
known to increase in pregnancy? (AI 06) b) Gestational hypertension
a) Glioma b) Pituitary adenoma c) Cardiac disease
c) Meningioma d) Neurofibroma d) Anaemia
1597. Which vitamin deficient is most commonly seen 1607. Hypothyroidism is pregnancy is least likely
in a pregnant mother who is on phenytoin therapy associated with- (AI 07)
for epilepsy? (AI 06) a) Recurrent abortions
a) VitaminB6 b)VitaminB 12 b) Polyhydramnios
c) VitaminA d) Folic acid c) Pregnancy induced hypertension (PIH)
1598. With which ofthe following types ofviral hepatitis d) Prematurity •
infection in pregnancy, the maternal mortality is 1608. Antiepileptic which is not associated with
the highest ? (AIIMS 06) congenital malformation when used in pregnant
a) Hepatitis-A b) Hepatitis-B woman is-
c) Hepatitis-C d) Hepatitis-E a) Phenytoin b) Phenobarbitone (AI 07)
1599. Glucose tolerance test is indicated in a pregnant c) Carbamazepine d)Valproate
lady in case of- (PGI June 06) 1609. Highesttransmisstion of hepatitis B from mother
a) Big baby b) Repeated abortion to fetus occurs if the mother is infected during -
c) Previous GDM d) Eclampsia a) 1st trimester b) lind trimester (AI 07)
e) H/0 diabetes in maternal uncle c) IIIrd trimester d) At the time of implantation
1600. Feature of diabetes mellitus in pregnancy- 1610. A 9 month .old pregnant lady presents with
a) Postdatism (PGI June 06) Jaundice and distension, pedal edema after
b) Hydranmios delivering normal baby. Her clinical condition
c) Neonatal hyperglycemia deteriorates with increasing abdominal distension
d) idefect and severe ascites. Her bilirubin is 5 mg/dl, S.
e) PPH alkaline phosphatase was 450 niL and ALT (345
1601. True statements is/are- (PGj June 06) lu). There is tender hepatomegaly 6 em below
a) MS surgery better avoided in pregnancy costal margin and ascitic fluid shw protein less
b) MR with PHT - definite indication for termination than 2 mg%. Diagnosis is- (AIIMS May 07)
of pregnancy a) Acute fatty liver of pregnancy
c) Aortic stenosis in young age is due to Bicuspid b) HELLP syndrome
valve c) Acute fulminant, liver failure
d) Isolated TR always due to infective endocarditis d) Budd chiari syndrome
e) MS with pressure gradient 10 mmHg-indication 1611. Most common heart disease which is associated
for surgery with maximum mortality during pregnancy-
1602. Which one of the following is not associated with a) Eisenmenger syndrome b) MS (AIIMS May 07)
diabetes in pregnancy- (UPSC 06) c) AS d)VSD
a) Sacral agenisis of the foetus 1612. Premature baby of34 weeks was delivered. Baby
b) Advanced placental grading had bullous lesion on the body. X ray shows
c) Baby weight> 3.5 kg periostitis what is the next investigation -
d) Hydramnios a) VDRL for mother & baby (AJIMS May 07)
1603. Caesarean section is preferred in- (APPG 06) b) ELISA for HIV
a) Toxoplasmosis b) Herpes c) PCRforT.B.
c) CMV d) Varecella Zoster Virus d) Hepatitis surface antigen for mother

1593)d l594)a 1595)d 1596)a 1597)d 1598)d 1599)a,b,c,d 1600)b,d,e 160l)a,b,c,e 1602)b 1603)b 1604)a
1605)a 1606)a 1607)b 1608)b 1609)c 1610)d 1611)a 1612)a
GYNAE & OBS [ 77]

1613. Consider the following statements- (UPSC 07) 1623. Which ofthe following modalities have shown best
A woman suffering from gestational biabetes is likely result for Pre menstrual syndrome? (AI 08)
to develop- a) SSRI b) Progesterone
1. PHI c) Oestrogen d) Anxiolyties
2. Macrosomia(foetal) 1624. Which of the following congenital infections is
3. lUD associated with minimal teratogenic risk to the
Which of the statements given above is/are fetus- (AI08)
correct- a) lllV b) Rubella
a) 1 and 2 only b) 2 only c) Varicella d) CMV
c) 1 and 3 only d) I, 2 and 3 1625. A pregnant mother presents with history of delivery
1614. The screening test for gestational diabetes mellitus of a previous child with congenital adrenal
that has the highest sensitivity is- (Corned 08) hyperplasia (CAH). The best management protocol
a) Glycosylated Hb for the current pregnancy is- (AI 08)
b) Blood fructosamine a) To start prednisolone after establishing whether
c) 50 gram glucose challenge test fetus is affected by chorionic Villous sampling
d) Random blood sugar b) To start dexamethasone as soon as pregnancy is
1615. Drug contraindicated in patient with rheumatic confirmed
heart disease in PPH is - (AJIMS Nov 07) c) To start dexamethasone after determining sex of
a) Oxytocin infusion b) Methyl ergometrine the fetus by karyotyping
c) Misoprostol d) Carboprost d) To start prednisolone after determining sex of the
1616. Highest maternal mortality is seen in following fetus with USG
congenital heart disease- (AJIMS Nov 07) 1626. Diabetes in pregnancy all except-
a) Eisenmengers complex b) Pulmonary stenosis a) Glucose challenge test is done between 24-28 week
c) Coarctation of aorta d) VSD b) 50 gm of sugar given as screening test
1617. Causes of pruritus in 3rd trimester of pregnancy
c) Insulin resistance improves with pregnancy
include the following except- (UPSC-I 08)
d) Diabetes control before conception is important
a) Prurigo gestationis
to prevent malformation
b) Obstetric cholestasis
1627. Which is the characteristic lesion of pregnancy ?
c) Pemphigoid gestationis
a) Vitiligo b)Pemphigus (APPG08)
d) Pruritic folliculitis
c) Tinea versicolor d) Chloasma
1618. Which female genital malignancy is most common
1628. Specific congenital abnormality associated with DM?
in pregnancy ? (UPSC-II 08)
a) Caudal regression syndrome (APPG 08)
a) Ovarian cancer b) Vaginal vulvar cancer
b)VSD
c) Endometrial cancer d) Cervical cancer
c) ASD
1619. Cholestasis may lead to the following complications
except- (UPSC-II08) d)TOF
a) Intrauterine fetal death 1629. Not a leading cause ofDIC in pregnancy?
b) Meconium stained liquor a) Prolonged pregnancy (APPG 08)
c) Preterm labour b) A.brubtio placenta
d) Neonataljaundice c) Heart disease
1620. The dermoid cyst, diagnosed at6 weeks of pregnancy. d) IUP
Best treatment modalities- (UP 08) 1630. True regarding asymptomatic bacteriuria in
a) LSCS along with removal of cyst pregnancy are- (PGI Dec 08)
b) Only when it undergoes torsion a) The incidence of acute pyelonephritis is high
c) Immediately b) t Morbidity in pregnancy
d) At 14-16 weeks of pregnancy c) t Chance of premature infant
1621. Maximum incidence of congenital malformation are d) All antimicrobial agents cannot be given in
seen in first trimester infection crosses the pregnancy
placenta- (UP 08) e) t Risk of chronic renal lesion in later life
a) Toxoplasmosis b) Rubella 1631. Regarding transmission of HIV to infant from
c) CMV d) Cryptosporidium infected HIV mother, which statement is/are true-
1622. All are true about Anti phospholipid syndrome a) Give zidovudine to mother (PGI Dec 08)
except- (UP 08) b) 25% chance of vertical transmission
a) Associated with SLE c) Avoid breast feeding
b) Thrombosis of uteroplacental vessels d) Vaccinate infant with OPV & MMR
c) Recurrent placental haemorrhage e) If infant is healthy for 5 month, then infant is not
d) Give false positive results in syphilis infected with HIV

1613)d 1614)c 1615)b 1616)a 1617)d 1618)a 1619)d 1620)d 1621)b 1622)c 1623)a 1624)a 1625)b 1626)c
1627)d 1628)a 1629)a 1630)All 1631)a,b,c
GYNAE & OBS [ 78]

1632. Which of the following antibiotics are safe to treat 1641. A multipara with 34 weeks pregnancy tachycardia,
UTI in pregnancy- (PGI Dec 08) fever hepatosplenomegaly, pallor has-
a) Aminoglycosides b) Fluroquinolones a) Malaria (DELHI PG Feb. 09)
c) Cephalosporins d) Sulfonamides b) Iron deficiency anemia
e) Tetracyclins c) Physiological anemia
1633. Vasopressor of choice in pregnancy is? d) Megaloblastic anemia
a) Ephedrine (AIIMS Nov 08) 1642. Which of the following is not associated with diabetes
b) Phenylephrine in pregnancy? (DELHI PG Feb. 09)
c) Methoxamine a) Sacral agenesis of the foetus
d) Mephenterrnine b) Advanced placental grading
c) Baby weight> 3.5 kg
1634. A pregnant lady acquires chicken pox 3 days prior
d) Hydramnios
to delivery. She delivers by normal vaginal route.
1643. The maximum chances of HIV infection being
Which ofthe following statements is true ?
transmitted to the fetus/infant in an HIV infected
a) Both mother and baby are safe (AIIMS Nov 08) motherareduring- (UPSC-II09)
b) Give antiviral treatment to mother before delivery a) First trimester b) Third trimester
c) Give antiviral treatment to baby c) During labour d) During breastfeeding
d) Baby will develop neonatal varicella syndrome 1644. Which one of the following statements is NOT
1635. Transmission of HIV from mother to child is correct about obstetric cholestasis? (UPSC-II 09)
prevented by all the following except- a) In results in pruritis without rash
a) Oral zidovudine to mother at 3rd trimester along b) Associated with markedly high bilirubin and raised
with oral zidovudine to infant for 6weeks liver enzymes
b) VitAprophylaxistomother (AIIMSNov08) c) It is an indication of termination of pregnancy at
c) Vaginal delivery 37weeks
d) Stopping breast feeding d) Risk of recurrence is high in future pregnancies
1636. Investigation of choice in cholestasis of pregnancy? 1645. True about lllV +ve mother- (PGI Nov 09)
a) Serum bilirubin levels (AIIMS Nov 08) a) Caesarean section is safer than vaginal delivery
b) Serum bile acids levels b) Vaginal delivery is safer than C.S.
c) Serum alkaline phosphatase levels c) Mother not taking ART should advised for C.S
d) Serum glutathiones transferase levels d) Baby does not need drug therapy
1637. All of the following drugs can be given to a mother e) Start ART & continue throughout pregnancy
with lupus after 35th week of gestation, Except- 1646. A pregnant patient with prosthetic valve should be
a) Chlaroquine (AI 09) switched to heparin in- (AIIMS Nov 09)
a) 32 weeks b) 36 weeks
b) Methotrexate
c) 40weeks d) Onset oflabour
c) Sulphadiazine I Sulphasalazine
1647. A pregnant lady bad no complaints but mild cervical
d) Prednisolone
lymphadenopathy in first trimester. She was prescribed
1638. True about congenital diseases in diabetes mellitus
spiramycin but she was non-compliant. Baby was
is all except- (AIIMS May 09) born with hydrocephalous and intracerebral
a) Results due to free radical injury calcification. Which of these is likely cause?
b) 6-10% cases are associated with major congenital a) Toxoplasmosis b) CMV (AIIMSMay IO)
abnormality c) Cryptococcus d) Rubella
c) 1-2% ofnewboms are associated with single 1648. A pregnant female at 35 wks of pregnancy is
umbilical artery disgnosed with SLE. Drugs that can be used to treat
d) Insulin can be given are all except- (AIIMS May I 0)
1639. Normal pregnancy can be continued in- a) Corticosteroids b) Sulphasalazine
a) Primary pulmonary hypertension (AIIMS May 09) c) Methotrexate d) Hydroxychloroquine
b) Wolf-Parkinson-White syndrome 1649. At what gestational age should a pregnancy with
c) Eisenmenger syndrome cholestasis of pregnancy be terminated-
d) Marfan syndrome with dilated aortic root a) 34weeks b)36weeks (AIIMSMayiO)
1640. The effect (s) of diabetic mother on infant is/are- c) 38 weeks d) 40 weeks
a) Brain enlargement as a part ofmacrosomai 1650. All are seen in gestational diabetes except-
b) Hyperglycemia in infant (PGI June 09) a) Previous macrosomic baby (AIIMS May I 0)
c) First trimester abortiobn b) Obesity
d) Unexplained fetal death c) Malformations
e) caudal regression d) Polyhydramnios

1632)a,c,d 1633)a 1634)d 1635)c 1636)b 1637)b 1638)c 1639)b 1640)c,d,e 164l)a 1642)b 1643)c
1644)b 1645)a,c,e 1646)b 1647)a 1648)c 1649)c 1650)c
GYNAE & OBS [ 79]

1651. Treatment of choice for intrahepatic cholestasis in 1660. True statement regarding use of antiepileptic
Pregnancyis- (AI 10) drugs in pregnancy- (PGI May 10)
a) Cholestyramine a) Valproate is associated with NTD
b) Ursodiol (Ursodeoxycholic acid) b) Multiple drug should be given
c) Coricosteroids (Dexamethasone) c) Carbamazepine is safe
d) Antihistaminics d) Phenytoin can. produce foetal hydantoin
1652. All of the following are cardiac contraindications to syndrome
pregnancy,except- (AI 10)
1661. Regarding H1N1 influenza in pregnancy, true
a) Eisenmenger's syndrome
statment (s) is/are- (PGI May 1 0)
b) Pulmonary hypertension
c) Coarctation of aorta a) Oseltamavir is used
d) WPW syndrome b) H IN I more dangerous in pregnancy
1653. The drug of choice to treat Chlamydia infection in c) Should start tit only after confirmation
pregnancyis- (AI 10) 1662. Regarding transmission of HIV to infant from
a) Tetracycline b) Doxycycline infected HIV mother, which statement is/are
c) Erythromycin d) Azithromycin true- (PGIMay 10)
1654. The drng of choice to treat chlamydia infection in a) Start zidovudine during labour
pregnancyis- (AI 10) b) 25% chance of vertical transmission
a) Tetracycline b) Doxycycline c) A void breast feeding
c) Erythromycin d)Penicillin d) Vaccinate infant with OPV & MMR
1655. Awomanpresentswithleakageoffiuidpervaginum e) CS cause less transmission
and meconium stained liquor at 34 weeks of 1663. Most common causes of maternal anaemia in
gestation. The most likely organism causing pregnancy- (PGI Nov. 10)
infection would be- (AI 10) a) Acute blood loss b) Iron deficiency state
a) Listeria monocytogenes b) Toxoplasmosis
c) GI blood loss d) Hemolytic anaemia
c) CMV d) Herpes
e) Thalassemia
1656. All of the following interventions are recommended
to prevent mother to child transmission of Hiv, 1664. Prolactinoma in pregnancy, all are true except-
except- (AI 10) a) Most common pituitary tumor but rarely
a) Avoid Ergometrine in third stage of labour symptomatic (AIIMS May 11)
b) Highly active antiretroviral therapy (HAART) b) Increase in prolactin levels worse prognosis
c) Elective caesarian section c) Macroadenoma > 1 em is associated with bad
d) Intrapartum zidovudine prognosis
1657. With reference to the transmission ofHIV from d) Regular visual checkup
mother to child, which one of the following 1665. All are seen in gestational diabetes except-
statements is not correct? (UP SC II 10) a) Previous macrosomic baby (AIIMS May 11)
a) The rate of transmission of HIV from mother to b) Obesity
child is between 15-48%. c) Malformations
b) In majority of cases, transmission of virus occurs d) Polyhydrarnnious
during intrapartum period 1666. A pregnant lady acquires chicken pox 3 days prior
c) Single dose of 200 mg of nevirapine at the onset to delivery. She delivers by normal vaginal route.
oflabour eliminates the risk ofHIV transmission
Which of the following statements is true?
to the newborn
a) Both mother and baby are safe (AIIMS May 11)
d) HIV is transmitted through breast milk
1658. A pregnant woman presented at 28 weeks of b) Give antiviral treatment to mother intrapartum
gestation with haemoglobin level of7 gm%; and c) Give antiviral treatment to baby
peripheral smear reveals it to he of microcytic d) Baby will develop neonatal varicella syndrome
hypochromic type. What would he the correct option 1667. Antiphospholipid syndrome is associated with all
oftherapy? (UP SC II 10) except- (AIIMS May 11)
a) Blood transfusion a) Recurrent abortion b) Venous thrombosis
b) Oral iron therapy c) Pancytopenia d) Antibody to lupus
c) Injectable iron therapy 1668. 36 weeks pregnant diabetic female with NST non
d) Oral iron and folic acid therapy reactive. What should be done next-
1659. Which parenteral iron preparation does not cause a) Induction of labour
anaphylaxis on intraveneous administration? b)CS
a) Iron dextran b) Iron sorbitol (UP SC II 10) c) Do NST after lhr
c) Iron fumerate d) Iron sucrose d) Proceed to biophysical profile

1651)b 1652)d 1653)d 1654)c 1655)a 1656)a 1657)c 1658)b 1659)d 1660)a,d 166l)a,b 1662)a,b,c,e 1663)a,b
1664)c 1665)c 1666)d 1667)c 1668)d
GYNAE & OBS [ 80]

1669. Which of the following drugs should not be used in 4. Prophylactic intravenous methergine at the birth
the conduct of labour in a woman with rheumatic of anterior shoulder
heart disease~ (AI 11) Select the correct answer from the code given below:
a) Methylergometrine b) Carboprost a) 1 only b) 1 and 3 only
c) Synctocin d) Misoprostol c) 4 only d) 1,2, 3 and4
1670. Which of the following is the best marker for 1679. Anti epileptic indicated in pregnancy - (Jipmer 11)
intrahepatic cholestasis of pregnancy- (AI 11) a) Valproic acid b) Carbamazepine
a) AST & ALT (Transaminases) c) Trimetazadine d) Dilantin
b) Bile Acids
c) Bilirubin GYNAECOLOGICAL DISORDERS IN OBS
d) Alkaline Phosphatase
1671. All ofthe following drugs can be given to a mother 1680. Procedure of choice in a woman with 12 wks
with lupus after 35 week of gestation,Except-(AI 11) pregnancy and atypical pap smear is- (AJ93)
a) Chlaroquine a) Cone biopsy b) MTP with cone biopsy
b) Methotrexate c) Hystrectomy d) Colposcopy
c) Sulphadiazine/ Sulphasalazine 1681. In Retroverted gravid uterus, urinary retention
d) Prednisolone usually appears at- (PGI 93)
1672. A primigravida in first trimester was observed to be a) 6-8 weeks b) 8-10 weeks
sputum positive for acid fast bacilli. She had no c) 12-14weeks d) 16weeks
previous history of tuberculosis. Which of the 1682. Acute urinary retention in gravid retroverted
following should be the treatment strategy -(AI 11) uterus occurs in- (Kerala 95)
a) Category I DOTS b) Category II DOTS a) 12-14weeks b)20-24weeks
c) 16-20weeks d)24-28weeks
c) Category ill DOTS d) Defer treatment 3
1683. Following is the emergency management ofbleeding
1673. Which of the following viruses is least likely to cross
vulvar varices during pregnancy- (AIIMS 95)
placenta - (AI 11)
a) Pressure b) Cautery
a) Rubella b)HetpesSimplexVJruS
c) Simple vulvectomy d) Observation only
c) HIV d)HBV
1684. Commonesttumorinpregnancyis- (AIIMS96)
1674. All of the following interventions are recommended
a) Granulosa cell tumour
to prevent mother to child transmission of Hiv,
b) Mucinous cystadenoma
Except- (AI 11)
c) Mature dermoid
a) Avoid breast feeding d) Fibroma
b) Highly ActiveAntiretroviral Therapy (HAART) 1685. The best treatment of an asymptomatic ovarian
c) Vaginaldelivery cyst in the first trimester - (AIIMS 99)
d) Intrapartum Zidovudine a) hmnediate laporatomy
1675. Ideal time to perform screening test for gestational b) Laporatomy in second trimester
diabetes (in weeks) is- (Karn 11) c) Laporatomy after delivery
a) 6-8 b) 12-14 d) Leave it alone till it becomes symptomatic
c) 24-28 d)36-38 1686. Treatment of Red degeneration of fibroid in
1676. The perinatal complications of a diabetic pregnancy pregnancy- (PGI 03)
include- (UPSCII 11) a) Analgesics
1. Small for Gestational Age baby b) Laparotomy
2 Stillbirth c) Termination of pregnancy
3. Hypoglycaemia d) Removal at caesarean section
4. Respiratory distress syndrome 1687. Regarding fibroid uteri- (PGI02)
Select the correct answer from the code given below- a) Estogen-dependant
a) 1 and4 only b) 1 and 2 only b) Capsulated
c) 1, 2 and 3 only d) 2, 3 and4 only c) Can lead to red degeneration in preganancy for
1677. Antimicrobial prophylaxis is essential for a woman which urgent surgery is required
in labour who has- (UPSC II 11) d) Danazo1 used in treatment
a) Hypertension b) Renal disease 1688. Treatment of choice for ovarian cyst in a postpartum
c) Diabetes mellitus d) Heart disease patient is~ (AI 07)
1678. In a pregnancy complicated by heart disease, which a) hmnediateremoval
of the following is/are contraindicated- b) Removal aster 2 weeks
1. External cephalic version (UPSC II 11) c) Removal after 6 weeks
2. LSCS d) Removal after 3 months
3. Corrective surgery of the heart lesion

1669)a 1670)b 1671)b 1672)a 1673)b,d 1674)c 1675)c 1676)d 1677)d 1678)c 1679)b 1680)d 1681)c
1682)a 1683)d 1684)c 1685)b 1686)b,c,d 1687)a,d 1688)a
GYNAE & OBS [ 81 ]

1689. In a gravid woman with placenta praevia, the following 1698. Nile blue sulphatase test in amniotic fluid is
foetal complications are known to increase - for- (AllMSMay07)
1. Congenital malformations (UPSC /Ill) a) Lung maturity b) Kidney maturity
2. Intrauterine growth retardation c) Liver maturity d) Skin maturity
3. Prematurity 1699. IUFD causes all except- (MARE 07)
Select the correct answer from the code given below: a) PIH b)DIC
a) 1,2and3 b) 1 and2only c) Psychological upset d) Infection
c) 2 and 3 only d) 1 and 3 only
1700. All are risk factors for preterm delivery except-
1690. A pregnant woman with 16 weeks gestation is
a) Absence offetal fibronectin at< 37 weeks
diagnosed to have an ovarian cyst of11 em diameter.
The best timing for the removal ofthe ovarian cyst b) Previous history ofpreterm baby (MARE 07)
is- (UPSCIIJJ) c) Asymptomatic cervical dilatation
a) Immediately d) Chlamydia! infection of genital tract
b) At the time of caesarean section 1701. Drug given to reduce the uterine contraction during
c) Immediately after delivery preterm labour with least side effect- (AllMS Nov 07)
d) Three months after delivery a) Ritodrine b) Nifidipine
c) Magnesium sulphate d) Progesterone
PRETERM LABOUR AND INTRAUTERINE 1702. Which one of the following genital nfections is
FETAL DEATH associated with preterm labour? (UPSC-II 08)
a) HmanPapillomavirus
1691. One of the following indicates death of fetus in b) Trichmonas vaginitis
uterus- (Kerala 90) c) Menilial vaginitis
a) Spalding sign d) Bacterial vaginosis
b) Failure ofuterus to enlarge 1703. IUFDcausesallexcept- (Manipal 08)
c) Blood statined discharge a) PIH b)DIC
d) Absence of fetal movements c) Psychological upset d) Infection
1692. Intrauterine death at 36 weeks. Treatment is- 1704. All are risk factors for preterm delivery except-
a) Continue upto term (RORTAK 98) a) Absent foetal fibronectin level at< 37 weeks
b) Wait for spontanious expulsion
b) Previous H/o preterm baby (Manipal 08)
c) Syntocinon+ARM
c) Asymptomatic cervical dilatation
d) Hysterectomy
d) Chlamydia! infection of genital tract
e) LSCS
1693. In a preg women of 28 weeks gestation IUD is 1705. Adverse effect(s) oftocolytic agents in pregnancy
earliest demonstrated X-ray- (PGI 98) is/are- (PGI Nov 09)
a) Increased flexion a) Puhnonary edema b) Hypoglycemia
b) Overlapping of cranial bone c) Hypokalemia d) Arrhythmia
c) Spalding's sign e) Hypertension
d) Gas in vessels 1706. Tocolytic of choice in heart disease-(AIIMS Nov 09)
1694. Risk of preterm delivery is increased if cervical a) Nifedepine b) Atosiban
length is - (AI 05) c) MgSo4 d) Salbutamol
a) 2.5cm b)3.0cm 1707. All are true about Prelabour rupture of
c) 3.5cm d)4.0cm membrane (PROM) except- (PGI May 10)
1695. After 28 weeks of gestation true is/are- a) Amnioinfusion is done
a) Viable (PGI June 06) b) Amoxiclav antiobiotic should be given
b) >lOOOgm c) Asceptic cervical examination
c) Lecthin/Sphingomyelinratio > 2
d) Steroid is used
d) Type II pneumocytes present
e) Preterm labour
e) Phosphatidylglycerol present
1696. In a lady of 32 weeks pregnancy injection
dexamehtasone is given to prevent- (AI 07) PREGNANCY IN Rh-NEGATIVE WOMEN
a) RDS b) Neonatal convulsion
c) Neonataljaundice d) Cerebral palsy 1708. In an Rh negative mother who was delivered an
1697. On Transvaginal Sonography (TVS) which ofthe Rh positive baby, prophylactic antiD is indicated
following shapes of cervix indicate imminent a) If the indirect Coomb's Test (ICT) is negative
preterm labour- (AI 07) b) If the ICT is positive (UPSC 96)
~ T b)Y c) If the ICT is positive with rising titres
c) U d)O d) As a routine procedure

1689)a 1690)a 1691)a 1692)b 1693)d 1694)a 1695)a,b,d 1696)a 1697)c 1698)d 1699)a 1700)a 1701)d
1702) d 1703) a 1704) a 1705) a,c,d 1706) b 1707) a 1708) a
GYNAE & OBS [ 82]

1709. In hydrops foetalis, on ultrasonography, the 1720. The following is true ofNaegele's pelvis-
earliest sign is the development offoetal- a) Absence of one ala (AJIMS 91, AP 96)
a) Pericardia! effusion b) Ascites (UPSC 96) b) Both alae absent
c) Pleurlaeffusion d) Skin edema c) Kyphotic spine
1710. Which does not cause Hydrops fetalis- (PGI97) d) Triradiate pelvis
a) Syphilis b) Rh immunisation 1721. The following is true of Roberts pelvis- (AI92)
c) ABO incompatibility d) None of the above a) Triradiate pelvis b) Single ala absent
1711. The dose of anti D gamma globulin given after c) Both alae absent d) Wide peovic brim
term delivery for a Rh negative mothe and Rh 1722. True about anthrapoid pelvis, sacrosiatic notch
positive baby is- (Kerala 2K) is - (CUPGEE 99)
a) SOmicrogram b)200microgram a) Wide and shallow b) Wide and deep
c) 300microgram d) lOOmicrogram c) Naqgw and shallow d) Narrow and deep
e) All the above doses are incorrect 1723. If one ala of sacrum is absent, it is called-(Karn 99)
1712. Most severely affected in child in Rh-iso- a) Rickety flat pelvis b) Naegele's pelvis
immunization patients- (SGPGI 05) c) Robert's pelvis d) Kyphotic pelvis
a)· Rh negative mother with Rh positive in 2nd child 1724. Triradiate pelvis is seen in- (PGI 97)
b) Rh positive mother with Rh negative in 2nd child a) Rickets b) Chondrodystrophy
c) Rh positive mother with Rh negative in 1st child c) Osteoporosis d) Hyperparathyroidism
d) Rh positive mother with Rh negative in 2nd child 1725. Dystocia dystrophia syndrome- (APPG 06)
1713. At 28 weeks gestation, amniocentesis reveals aAOD a) Android pelvis b) Platypelloid pelvis
450 of 0.20 which is at the top of third zone of the c) Anthrapoid d) Gynaecoid pelvis
Iiley curve. The most appropriate management of 1726. Naegele's pelvis is- (Manipal 06)
such a case in- (AIIMS May 05) a) Single pelvis ala absent
a) Immediate delivery b) Both pelvis ala absent
b) Intrauterine transfusion c) Absence of 4th sacral vertebra
c) RepeatAmniocentesis after 1 week d) Absence of 1st sacral vertebra
d) Plasmapheresis 1727. In Muller's maneuver to predict cephalopelvic
1714. Causes of hydrops fatalis- (PGI June 08) disproportion (CPD)- (Maharashtra 10)
a) Parvo virus infection b) HZVinfection a) The vaginal finger is at ischial spine
c) Down's syndrome d) Toxoplasma
b) The vaginal finger is at sacral promontory and
1715. The least amountoffetal Rh+ve blood needed to cause
thumb at the pubic symphysis
isoimmunization in a Rh negative mother is-
c) The vaginal finger is at tip of sacrum
a) 0.1 ml b) 0.01 ml (DPGEE 08)
c) 0.001 ml d) lOml d) None of the above
1716. Mother's blood group is Rh-ive. Indirect Comb's is
+ve. The foHowing will be seen in baby- TRIAL OF LABOUR
a) Oligohydramnios (PGI Nov 09)
1728. Trial oflabour is contraindicated in- (PG190)
b) Abnormal umbilical artery waveform deceleration
c) Hydrops fetalis a) Elderlyprimi b) Previous cesarian
d) IUGR c) Malpresentation d) Occipito posterior
e) Anemia 1729. All the following are contra-indications for Trial
1717. Non immune hydrops fetalis is caused by- (All 0) labour EXCEPT- (Karnat. 99)
a) CMV b) Parvovirus a) Breech presentation
c) HSV d)HIV b) Outlet contraction
1718. Nonimmune hydrops fetalis is caused by all EXCEPT- c) Primi gravida
a) ParvoB19virus (Corned 10) d) Post caesarean pregnancy
b) Chromosomal abnormalities 1730. Trial labour is contraindicated in all of the
c) Alpha-thalassaemia following except- (APPG 06)
d) ABO incompatibility a) Primigravida b) Heart disease
c) Cesarean section d) PIH
CONTRACTED PELVIS 1731. Oxytocin sensitivity test is used to assess the-
a) Foetal well-being (UPSC 07)
1719. CPD is best assessed by - (TN 91) b) Period of gestation
a) CT scan b) Ultrasound c) Cervicalripening
c) Radiopelvimetry d) Bimanual examination d) Uterine response for induction

1709)d 1710)c 17ll)c 1712)a 1713)c 1714)a,c,d 1715)a 1716)c,e 1717)b 1718)d 1719)a 1720)a 1721)c
1722)a 1723)b 1724)a 1725)a 1726)a 1727)a 1728)a,b,c 1729)c 1730)a 1731)d
GYNAE & OBS [ 83]

ABNORMAL UTERINE ACTION 1739. Pathological contraction ring is due to -(Jipmer 11)
a) Obstructed labour b) Uterine inertia
1732. Bandl's ring is caused by- (AIIMS 94) c) Preterm labor d) Oligamnios
a) Uterine inertia 1740. Recurrent breech presentation is due to- (Jipmer 11)
b) Cephalopelvic disproportion a) Placenta previa b) Hydrocephalus
c) Malepresentation c) Septate uterus d) Short cord
d) None
1733. Bandl's ring is also called as- (PGI 98) MALPRESENTATION
a) Constriction ring b) Schroeder's ring
c) Retraction ring d) Cervical dystocia 1741. Earliest engagement is seen in ... breech - (TN90)
1734. Factor predisposing for face presentation- a) Frank b) Complete
a) Pelvic contraction (PGI Nov 09) c) Incomplete d) Flexed
b) Anencephaly 1742. The cause of of breech- (PGI 90)
c) Prematurity a) hydramnoid b) Septate uterus
d) Foetal goitre c) Hydrocephalus d)All
e) Bicarbonate uterus 1743. Assisted breech delivery is done for- (Al91)
1735. A 20-year-old full-term primigravida is brought to a) Extended breech
the casualty with labour pains for last 24 hours and b) Complete breech
a hand prolapse. On examination, she has pulse 96/ c) Weight of baby less than 2 kg.
min, BP 120/80 mmHg, and mild pallor. The d) History of previous two successful breech delivery
abdominal examination reveals the uterine height 1744. Delivery is not possible per vaginum in-
at 32 weeks, the foetus in transverse lie and absent a) Mento anterior (Jipmer 91)
foetal heart sounds. On vaginal examination, the left b) Sacro posterior
arm of the foetus is prolapsed and the foetal ribs are c) Persistent mento posterior
palpable. The pelvis is adequate. What would be the d) Extended breech
1745. In case of unstable lie offetus, the placenta is usually-
best management option? (UP SC II 10)
a) Cornual b)Lateralwall (AIIMS91)
a) External cephalic version
c) Fundus d) Lower segment
b) Decapitation and delivering the baby vaginally
1746. A method of delivering the breech is- (DNB 91)
c) Internal podalic version
a) Scanzoni manoeuvre
d) Lower segment caesarean section
b) Mauriceau 's manoeuvre
1736. A lady presents at 37 weeks of gestation with uterine c) Ritgenmanoeuvre
contraction and pain suggestive oflabour for 10 d) Piper manoeuvre
hours. On examination cervix is persistently 1 em 1747. Most unfavourable presentation for vaginal
dilated and uneffaced. What should be the next line delivery is- (ALL INDIA 95)
oftreatment? (AI 11) a) Mento posterior b) Mento anterior
a) Sedation and wait c) Occipito posterior d) Deep transverse arrest
b) Induction with rupture of membranes 1748. Cord prolapse is most commonly associated
c) Augmentation with Oxytocin &Amniotomy with- (CUPGEE 96)
d) Caesarian section a) Transverse lie b) Breech
1737. A pregnant mother is referred with a. prolonged c) Contracted pelvis d) Prematunity
second stage oflabour. On examination, the foetal 1749. In an after coming head the following bone is
heart sound is 120/min, and the head is at -1 station perforated during decapitation- (Karn. 96)
with severe moulding. What will be the most a) Occiput b) Parietal
appropriatemanagement- (UPSC II 11) c) Palate d) Frontal
a) Apply obstetric forceps and deliver 1750. Commonest cause of breech presentation- (Al97)
b) Apply ventouse and deliver a) Prematurity b) Postmaturity
c) PerformLSCS c) Oligohydramnios d) Polyhydramnios
d) Start pitocin drip 1751. Cord prolapse is seen leastin- (JIPMER 98)
1738. A multiparous patient presents with 36 weeks of a) Complete breech b) Frank breech
pregnancy in labour. On examination, the cervix is c) Knee breech d) Footing breech
fully dilated. The foetal heart rate is 170/min, and 1752. The treatment of choice in a primigravida at term
the head is at +2 station. What will be the most in labour with transverse lie and cervical dilatation
appropriate management- (UPSC II 11) of 6 em with infact membrane is- (MP 96)
a) Wait for the normal delivery a) Wait and watch till full dilationof Cervix
b) Apply ventouse and deliver b) Internal podalic version and breech extraction
c) Apply forceps and deliver c) Lower segment caesarean section
d) PerformLSCS d) Bioplarversion

1732)b,c 1733)c 1734)a,b,d 1735)d 1736)a 1737)c 1738)c 1739)a 1740)c 1741)a 1742)d 1743)All 1744)c
1745)d 1746)b 1747)a 1748)a 1749)a 1750)a 1751)b 1752)c
GYNAE & OBS [ 84]

1753. Which of the following methods is commonly 1762. Shoulder dystocia results in the following except-
employed in delivery of aftercoming head in breech a) Sternomastoid swelling (PGI 97)
presentation- (UPSC 97) b) Erb's palsy
a) Burn-Marshall method c) Klumpkes paralysis
b) Forcepts delivery d) None
c) Mauriceau Smellie-veitmethod 1763. Cause of death in breech delivery-(PG/97, UPSC97)
d) Pinard's manoeurve a) Intracranial haemorrhage b) Aspiration
1754. a multigravida at term with transverse lie and c) Atlantoaxial dislocation d) Asphyxia
hand prolapse with foetal heart sound of 140/min 1764. 38 weeks primi in early labour with transverse
is best managed by- (UPSC ) presentation TOC is- (PGI 98)
a) Internal podalic version a) Allow for cervical dilatation
b) Caesarean section b) Internal podalic version
c) External cepphalic version c) LSCS
d) Breech extraction d) Forceps
1755. Most common congenital anomaly associated with 1765. Best method to deliver arms in breech- (PGI 98)
face presentation is- (CMC 98) a) Lovset's method b) Smellie veit
a) Anencephaly b) Hydrammons c) Pinard's d) Any of the above
c) Microcephaly d) Hydrocephalus 1766. In deep transverse arrest the delivery of baby is
e) Macrosomia conductedbyAIE- (PGI99)
1756. Arrest of after coming head in breech with chin a) Caesarean section
to pubis, which maneuver is used- (PGI 98} b) Vaccum extraction
a) Maurice c) Keilland forcep
b) Burnsmarshal d) Manual rotation and forcep delivery
c) Pinards 1767. In intrauterine death with transverse lie, the
d) Posterior rotation of chin and piper's forceps following are treatment options except- (PGI 99)
1757. The complications of shoulder presentations are a) Decapitation b) Evisceration
all of the following except- (TN 99) c) Craniotomy d) Caesarean section
a) Fetal death b) Uterine rupture 1768. True about Frank Breech- (PGI 02)
c) Obstructed labour d) Shoulder dystocia a) Thigh extended, leg extended
1758. Following are contra indications to External b) Thigh flexed, knee extended
Cephalic Version except- (AIIMS 92) c) Both are flexed
a) Contracted pelvis d) Buddha's attitude
b) Antepartum hemorrhage e) Common in prinri
c) Multiple pregnancy 1769. Breech presentation with hydrocephalus is managed
d) Hydranmois by- (PGI 02)
1759. The cause of failed external cephalic version a) Cesarean section
(ECV) in a 34 weeks pregnancy is- (Kerala 97) b) Trans-abdominal decompression
a) Oligohydranmios b) Breech c) PV decompression
c) Hydramnios d) None of the above d) Craniotomy of aftercoming head
1760. Which one of the following pairs of terminologies 1770. Causes of Face presentation- (PGI 03)
is NOT correctly matched- (UPSC 2K) a) Anencephaly b) Prematurity
a) Breech delivery Bum Marshal tech. c) Hydramnios d) Contracted pelvis
b) Neglected shoulder Bandle's ring or e) Placenta praevia
presentation retraction ring 1771. Causes of breech presentation are- (PGI 03)
c) Retained placenta - True knot of cord a) Hydrocephalus b) Oligohydranmios
d) Reverse rotation to Face to pubic c) Pelvic contracture d) Placenta praevia
occipito sacral position delivery 1772. In a case of direct occipito- posterior position (Face
1761. A 30 yrs old multigravida presented with to pubis delivery) most commonly encountered
transverse lie with hand prolapse in lind stage of problem is- (Karnataka 02)
labour with dead fetus. The treatment is- (PGI 03) a) Intracranial injury b) Cephalohematoma
a) Chemical caesarean section c) Paraurethral tears d) Complete perineal tears
b)lSCS 1773. Face to pubis delivery is common in which type
c) Craniotomy of pelvis- (Jipmer 03, PGI 97, AI 97)
d) Decapitation a) Android b) Anthropoid
e) Cleidotomy c) Platypelloid d) Gynaecoid

1753)a 1754)b 1755)a 1756)d 1757)d 1758)d 1759)a 1760)c 1761)b,d 1762)d 1763)a,d 1764)c 1765)a
1766)None 1767)d 1768)b,e 1769)c 1770)a,b,d 1771)a,b,c,d 1772)d 1773)b
GYNAE & OBS [ 85]

1774. The commonest cause of occipito- posterior position 1785. In intrauterine death with transverse lie, the
of featl head during labour is- (AIIMS 03) following are treatment options, except -(DELHI PG
a) Maternal obesity b) Deflexion of fetal head a) Decapitation b) Evisceration Feb. 09)
c) Multiparity d) Android pelvis c) Craniotomy d) Caesarean section
1775. The commonest cause of breech presentation 1786. Patient with occipito posterior position. What is the
is- (AIIMS 03)
management- (AIIMS Nov 09)
a) Prematurity b) Hydrocephalus
a) Wait and watch for progress of labour
c) Placenta previa d) Polyhydramnios
1776. The most common form of fetal traumatic injury b) Cesarian section
incurred during breach extraction is- (SGPGI 05) c) Oxytocin influsion
a) Rupture of the liver d) Rupture of membranes
b) Rupture of the spleen 1787. AU of the following maneuvers are used in shoulder
c) Intraadrenal hemorrhage dystocia, except- (AI 1 0)
d) Intracranial hemorrhage a) Woods corkscrew maneuver
1777. Best treatment of Cord prolapse is- (HPU 05) b) Me Roberts maneuver
a) Replace the cord in vagina c) Suprapubic pressure
b) Caesarean section d) Mauriceau-smellie veit maneuver
c) Immediate vaginal delivery 1788. The most unfavourable prognosis is of-
d) None of above
a) Mentoposterior position (Maharashtra 1 OJ
1778. A 25-year old G2P1AO with history of previous
b) Occipitoanterior position
vaginal breech delivery of a term, live baby comes
c) Occipitoposterior position
with full term pregnancy with breech presentation.
What is the best option- ( UPSC 06) d) Mentoanterior position
a) Caesarean section 1789. Loveset maneuver is usefulformanagementofbreech
b) Assisted breech delivery with- (Maharashtra 10)
c) External cephalic version a) Extended legs b) Flexed legs
d) Watchful expectancy c) Extended arm d) Nuchal displacement of arm
1779. Deep transverse arrest is seen in all 1790. Match List-1 with List-11 and select the correct
except- (Manipal 06) answer using the code given below the Lists-
a) Android pelvis b) Epidural analgesia List-1 (Manoeuvre) List-11 (Indication)
c) Transverse lie d) Uterine inertia A. Pinard's manoeuvre 1. Delivery of the extended
1780. External Cephalic Version (ECV) is contraindicated arms in a breech
in- (A/07) B. Lovset's manoeuvre 2. Delivery of a foot into the
a) Primigravida b) Flexed breech vagina in a breech
c) Aaemia d) PlH C. Mauriceau-Smellie- 3. Breech presentation at
1781. Breech presentation is mostly mistaken for- Veit manoeuvre term
a) Face presentation b)Brow (MAHE 07) D. External cephalic 4. Delivery of the after-
c) Shoulder d) Vertex coming version head of
1782. Delivary of the after - coming head in breech a breech
presentation following forceps/methods are used A B c D (UPSCIIJO)
except- (UP 07) a) 2 4 1 3
a) Wrigely's forceps b)3 1 4 2
b) Kielland's forceps c) 2 1 4 3
c) Das's variety forceps d) 3 4 1 2
d) Mauriceau smellie, veit technique
1783. Techniques of delivery of after coming head in OBSTRUCTED LABOUR
breech presentation- (PGI June 07)
a) Bums - Marshal method 1791. Shoulder dystocia is seen in predominantly-
b) Forceps delivery a) Transverse lie b) Hand Prolapse (TN 90)
c) Modified Mauriceau Smellie - Veit technique
c) Anencephaly d) Cord around neck
d) Lovset's maneuver
1792. Anencephaly is associated with the following
1784. When in labor, a diagnosis of occipito posterior
factor- (Karn 96)
presentation is made. The most appropriate
a) Hydramnios b) Postmaturity
management in this case would be -(Aiims May 08)
a) Emergency CS c) Prematurity d) All of these
b) Wait and watch for progress of labour 1793. Anencephaly can best be diagnosed at (by USG)-
c) Early rupture of membranes a) 10-12weeks b) 12-14weeks (AIIMS96)
d) Start oxytocin drip c) 14-16weeks d) 16-20weeks

1774)d 1775)a 1776)d 1777)b 1778)b 1779)c 1780)d 1781)a 1782)a 1783)a,b,c 1784)b 1785)c 1786)a
1787)d 1788)a 1789)c 1790)c 1791)c 1792)d 1793)a
GYNAE & OBS [ 86]

1794. Anencephaly causes all except- (TN 98) 1804. The presence of a retraction ring at the junction of
a) Prematurity b) Postmaturity upper and lower uterine segment in labour indicates-
c) Hydrarnnios d) Face resentation a) Prolonged labour (UP SC II I 0)
1795. Anencephaly is earliest diagnosed at- (Rohtak 97) b) Cervical dystocia
a) 6 weeks b) 10 weeks c) Obstructed labour
c) 14 weeks d) 18 weeks d) Precipitate labour
e) 20weeks 1805. 30 yrs old female comes with obstructed labour and
1796. Anencephaly is associated with all the following is febrile & dehydrated with IUFD and cephalic
except- (AIIMS 2K) presentation. Which is the best way to manage-
a) Face presentation a) Craniotomy (AIIMS May 11)
b) Adrenal gland hypertrophy b) Decapitation
c) Hydrarnnios c) Cesarean section
d) Post maturity d) Forceps extraction
1797. All are the risk factors associated with macrosomia 1806. All of the following maneuvers are used in shoulder
except- (AI 05) dystocia, except- (Aill)
a) Maternal obesity b) Prolonged Pregnancy a) Woods Corkscrew Maneuver
c) Previous large infant d) Short Stature b) McRoberts Maneuver
1798. Sudden hyperflexion of thigh over abdomen c) Suprapubic Pressure
(Mcrobert's procedure), which of the following nerve d) Mauriceau- Smellie Veit Maneuver
iscommonlyinvolved- (AIIMS Nov 08)
a) Common peroneal nerve COMPLICATION OF 3rd STAGE OFLABOUR
b) Obturator nerve ·
c) Lumbosacral trunk 1807. The causes of post partum hemorrhage in a
d) Lateral cutaneous nerve of thigh retracted uterus - (PGI90)
1799. All are done in management of shoulder dystocia a) Vaginal tear b) Carvicallaceration
except- (AIIMS Nov 08) c) Retained placenta d) Atony of uterus
a) Fundal pressure b) Mcroberts manoeuver 1808. Which is not true of placenta accreta-
c) Suprapubic pressure d) Woods manoeuvre a) May penetrate serosa (JIPMER 93)
1800. A female at 37 wks of gestation has mild labour pain b) Invades myometrium
for 10 hours and cervix is persistently 1CM dilated c) Absence ofNitabuch membrane
but non effaced. What will be next appropriate d) More common in primigravida
management? (AIIMS Nov 08) 1809. Post partum haemorrage is present when blood
a) Sedation and wait loss exceeds - (UPSC 84, Karn 96)
b) Augmentation with syntocin a) 200cc b)400cc
c) Cesarean section c) 500 cc d) 700 cc
d) Amniotomy e) 1000cc
1801. A woman comes with obstructed labour and is grossly 1810. A 30-year old woman para 6 delivers vaginally
dehydrated. Investigations reveal fetal demise. What following normal labour with spontaneous
will be the management? (AIIMS Nov 08) delivery of an intact placenta. Excessive bleeding
a) Craniotomy b) Decapitation continues,despite mannal exploration, bimanual
c) Cesarean section d) Forceps extraction massage, intravenous oxytocin and administration
1802. A primi patient present's in obstructed labour with of0.2 mg methergin Iv, which one of the following
a neglected shoulder presentation. The mother would be the next step in the management ofthis
shows signs of exhaustion and the fetus is dead. The patient-
treatment of choice is- (AI 09) a) Packing the uterus
a) Craniotomy b) Dcapitation b) Immediate hysterectomy
c) Cesarian section d) Wait and watch c) Bilateral Internal Iliac ligation
1803. A pregnant women, previous LSCS with hematuria. d) 1M injection ofPGF 2 a
Diagnosis is? (AIIMS Nov 09) 1811. During PPH Intenal Iliac Ligation done at-
a) Impending scar rupture of uterus a) Origin of Internal iliac artery (KARN 95)
b) Prolonged labour b) Anterior division oflnternal iliac artery
c) Uretreral injury c) Posterior division oflntenal iliac artery
d) Cystitis d) Common iliac artery

1794)None 1795)b 1796)b 1797)d 1798)d 1799)a 1800)a 1801)c 1802)b 1803)b 1804)c 1805)c 1806)d
1807)a,b,c 1808)d 1809)c 1810)d 18ll)b
GYNAE & OBS [ 87]

1812. Maximum chances of postpartum Haemorrhage c) Oxytocin drip


areseenin- (AMU95) d) Reassess for occipitoposterior position and
a) Multiparity b) Primipara cephalopelvic disproportion
c) Abnormal lie d) All 1824. A primigravida with full-term pregnancy in labour
1813. All predispose to placenta accreta except- (AP 96) for one day is brought to casualty after dai handling.
a) Multiparity b) Previous ceserian On examination, she is dehydrated, slightly pale;
c) Placenta pracvia d) Previous D & C pulse is tOO/minute, BP 120/80 mm Hg.Abdominal
1814. Atonic post partum hemorrhage occurs in all examination reveals a fundal height of 36 weeks,
except- (TN 98) cephalic presentation, foetal heart absent, mild
a) Multiple pregnancy b) Hydramnois uterine contractions present. On p/v examination,
c) Erythroblastosis fetalis d) Prolonged labour
cervix is fully dilated, head is at puis one station,
1815. Post partum haeorrhage is blood loss of 500 cc
caput with moulding present, pelvis is adequate.
or more within- (ORRISA 98)
a) 6 hors of the beginnig of 3ro stage of labour Dirty, infected discharge present. What would be
b) 12 hors of the begining of3rd stage oflabour the best management option after initial work-up-
c) 18 hours of the begining of3rd stage oflabour a) Cesarean section (UPSC 07)
d) 24 hours ofthe begining of3rd stage oflabour b) Oxytocin drip
1816. Atonic uterus is more common in- (PGI 97) c) Ventouse delivery
a) Caesarean section b) Multi gravida d) Craniotomy and vaginal delivery
c) Primigravida d) Breech delivery 1825. An elederly multiparous woman with intrauterine
1817. Common cause of retained placenta is- (Jipmer 03) foetal death was admitted with strong labour pains.
a) Atonic uterus The patient suddenly goes in shock with cyanosis
b) Condtriction ring respiratory disturbances and pulmonary oedema.
c) Placenta accreta The most likely clinical diagnosis is- (UPSC 07)
d) Poor voluntary expulsive effort a) Rupture of uterus
1818. Common cause of death in inversion of b) Congestive heart failure
uterus- (Jipmer 03) c) Amniotic fluid embolism
a) Neurogenic shock b) Hemorrhage d) Concealed accdental haemorrhage
c) Pulmonary embolism d) Infection 1826. Risk factors for PlacentaAccreta incude all ofthe
1819. All of the following used for postpartum following except- (AI 08)
hemorrhageexcept- (APPGE05) a) Previous LSCS scar
a) Misoprostol b) Carboprost b) Previous curettage
c) Mifepristone d) Methergine c) Previous myomectomy
1820. The following complications during pregnancy d) Previous placenta praevia
increase the risk of postpartum hemorrhage
1827. Placenta a creta alw- (PGI June 08)
(PPH) except- (AI 06)
a) Placenta praevia b) Uterien scar
a) Hypertension b) Macrosomia
c) Multiple pregnancy d) Multipara
c) Twin pregnancy d) Hydramnios
1821. Whatis the used in treatmentfor PPH except- e) Uterine malformations
a) Oxytocin b) Carboprost (Manipal 06) 1828. Treatment ofPPH includes- (PGIDec08)
c) Ergometrine d) Ritodrine a) B-lynch suture
1822. All are causes of atonic postpartum haemorrhage b) Uterine tamponade
except- (UP 07) c) Ligation of internal iliac artery
a) Over distension of the uterus d) Ligation of external iliac artery
b) Cervical teat e) Ligation of common iliac artery
c) Prolonged labour 1829. Rupture uterus is most commonly encountered after
d) Constriction ring which one of the following surgeries ?(UPSC-II 09)
1823. A 20 year old primigravida is admitted with-term a) Classical cesarean b) Metroplasty
pregnancy and labour pains. At 4.00 AM she goes c) Myomectomy d) Hysterotomy
into active phase oflaboutwith 4 em cervical dilatation. 1830. Management ofPPH includes- (PGI Nov 09)
Membranes rupture duringp/v examination showing a) Uterine artery ligation
clear liquor. A repeat p/v examination after 4 hours b) Hysterectomy
ofgood uretine conditions reveals as cervical dilatation c) Sengstaken-Blakemore ballon temponade
of5 em. What should be the next step in management- d) Internal iliac artery ligation
a) Reassess after 4 hours (UPSC 07) e) Laser endometrial abalation
b) Immediate cesarean section

l812)a l813)a 1814)c l815)d 1816)b l817)a 1818)a,b,c 1819)c 1820)a 182l)d l822)b 1823)d 1824)c
1825)c 1826)d 1827)a,b,d,e 1828)a,b,c 1829)a 1830)a,b,c,d
GYNAE & OBS [ 88]

1831. B-Lynch stitch is applied on the uterus for the 1841. Indicators of impending uterine rupture during
treatment of- (UP SC Ill 0) labour include all of the following except- (AI 06)
a) Incompetent os a) Fetal distress
b) Atonic PPH b) Hematuria
c) Bleeding from placental bed of placenta praevia c) Fresh bleeding per vaginum
d) Ruptured uterus d) Passage of meconium
1832. Third degree perineal tear is said to have occurred 1842. ill perineal tear is involvement of -(Karn pgmee 06)
when it involves - (Karn 11) a) Vaginal mucosa b) Urethral mucosa
a) Anal sphincter b) Perineal muscles c) Levator ani muscle d) Anal Sphincter
c) Rectal mucosa d) Vaginal mucosa
ABNORMALITY OF PUERPERIUM
1833. The most common cause of secondary postpartum
haemorrhage is- (UPSC !Ill) 1843. Commonest cause for puerperal sepsis is-
a) Intrauterine infection a) Streptococci b) Anaerobes (Kerlala 94)
b) Subinvolution of uterus c) Gonococci d) Staphyococci
c) Oestrogen therapy administered to inhibit the 1844. Puerperal sepsis is due to spread of infection along
lactation which route- (AIIMS 95)
d) Retained fragments of placenta or membranes a) Direct extension b) Arterial
c) Venous d) Lymphatics
INJURY TO THE BIRTH CANAL 1845. Puerperal pyrexia is fever for 24 hours or more
after child birth if temperature is more than -
1834. In a patient with third degree perineal tear, a) 99°F b) 99.5° F (Orrisa 98)
presenting after 1 weeks, repair should be done- c) 100°F d) 100.4°F
a) Immediately b)2weeks (AJIMS92) 1846. The most common site of puerperal infection
c) After6 weeks d) After 12 weeks is- (JIPMER 02)
1835. Uterine rupture is least common with -(JIPMER 02) a) Episiotomy wound b) Placental site
a) LSCS c) Vaginal laceration d) Cervical laceration
b) Classical section 1847. All are complication of formula fed baby over
c) Inverted T shaped incision human milk fed baby except- (JIPMER 95)
d) T Shaped incision a) Necrotizing enterocolitis b) Otitis media
1836. Incidence of scar rupture in a pregnant lady with c) Hypocalcemia d) VitKdeficiency
previous LSCS is- (JIPMER 2K) 1848. The cause of 'postpartum blues' is- (UPSC 04)
a) 0.2% b)0.5% a) Decreased estrogen
c) 0.7% d)0.9% b) Decreased progesterone
1837. In classical caesarean section more chances of c) Increased prolactin
rupture of uterus is in- (JIPMER 95)
d) Decreased estrogen and progesterone
1849. A patient has just delivered a live baby and is referred
a) Upper uterine segment
to in state of shock. On abdominal examination,
b) Lower uterine segment
uterus is not felt. On pelvic examination, a mass is
c) Utero cervical junction seen protruding through the vagina. What is the
d) Posterior uterine segment most likely diagnosis- (UPSC 07)
1838. Cervical lacerations are most common in a) Cervical fibroid polyp
the- (UPSC 96) b) Third degree uterine prolapse
a) 3 o' clock position b) 6 o' clock position c) Acute inversion of uterus
c) 9 o' clock position d) 12 o' clock position d) Rupture of uterus
1839. Complete perineal teariscommonin- (UPSC02) 1850. Which complication you would expect during
a) Face to pubes delivery internal podalic version in case of transverse lie -
b) Breech delivery a) Uterinerupture (AIIMSNov07)
c) Internal podalic version b) Cervical laceration
d) Manual removal of placenta c) Uterine atony
1840. A woman delivers a 9 - lb infant with a midline d) Vaginal laceration
episiotomy and suffers a third degree tear. Inspection 1851. Leastmanagementin Mento-posterior presentation-
shows which of the following structures is a) Vaginal delivery (UP 08)
intract- (Karnataka 03) b) Forceps delivery
a) Anal sphincter b) Perineal body c) Manual rotation
c) Perineal muscles d) Rectal mucosa d) Caesarean section

1831)b 1832)a 1833)d 1834)d 1835)a 1836)b 1837)a 1838)a 1839)a 1840)d 1841)bord 1842)d 1843)a,b
1844)a,b,c 1845)d 1846)b 1847)d 1848)d 1849)c 1850)a 185l)d
GYNAE & OBS [ 89]

1852. After a full term normal delivery patient went into 1865. Meconium is excreted by a uew born till ... day-
shock.Mostprohablecauseis- (AIIMSNov 10) ~2 ~3 w~
a) Inversion of uterus b) PPH c) 6 d)4
c) Amniotic fluid embolism d) Eclapsia 1866. The number of fontanelles present in a new born
child is-
FETUS AND NEWBORN & THEIR DISEASES a) 1 b)2
c) 4 d)6
1853. Singers Alkali denaturation test is done with- 1867. Breathing movements and swallowing movement
a) MatemalHb b)FetalHb (TN91)
seen in the foetus by- (Orissa R)
c) Amniotic fluid d) Menstrual fluid
a) 13-14 weeks b) 16 weeks
1854. Foetal Hb is different from HBA in that it is-
c) 12weeks d)20weeks
a) More sensitive to alkali denaturation (DNB 91)
b) More resistant to alkali denaturation 1868. Foetal breathing movements is seen earliest at-
c) More sensitive of acid denaturation a) 8 weeks b) 11 weeks (Orissa 2K)
d) All of tbe above c) 16weeks d)24weeks
1855. Which is incorrect about cephalhematom- (A192) 1869. Hematoma ofthe sternomastoid muscle detected
a) Does not cross suture line in a 16 days old infant requires- (AIIMS 02)
b) Fluctuation is seen a) Immediate surgical evacuation
c) Present ofbirtb b) Surgical intervention within 2 weeks
d) Presents few hours after birth c) Prophylactic antibiotic therapy
1856. Internal organs in the fetus develop at -(Kerala 91) d) No immediate therapy
a) 24wks b) 12 wks 1870. Non immune hydrops is not a feature of which of
c) lOwks d) 6wks thefollowing- (AJIMS 2K)
1857. Fetal hemopoieisis first occurs in- (Karn 94, a) Cbromosomalabnormalities
a) Yolksac b)Liver MAHE04) b) Alpha thalassemia
c) Spleen d)Bonemarrow c) Renal malformations
1858. Breast feeding should be started in normal d) Foetal cardiac abnormalities
delivery- (Kerala 94) 1871. In non immune hydrops which ofthe following is
a) Soon after delivery b) After 2 hrs NOT seen- (AIIMS 2000)
c) After 6 hrs d) After 4 hrs a) Skin oedema b) Ascites
1859. Alkali resistant haemoglobin is- (Kerala 95) c) Large placenta d) Cardiomegaly
a) HbA b) HbA,C 1872. Fetal trisomy - 21 is associated with all of the
c) HbS d)HbF
following EXCEPT- (Karnataka 02)
1860. LifespanofneonataiRBCis- (Assam95)
a) Increased Nuchal skin fold thickness
a) 120days b) lOOdays
b) Decreased serum alpha fetoprotein level
c) 6 months d) one year
c) Decreased serum human chorionic gonadotropin
1861. Which is abnormal in classic haemorrhagic
disease of newborn- (UP 2K) level
a) Platelet count b) Thrombin time d) Decreased unconjugated estriol level
c) Fibrinogen level d)APTT 1873. Rule of Hasse is used to determine- (AIIMS 03)
1862. According to registrar society oflndia commonest a) The age of fetus b) Height of an adult
causeofiMRinindiais- (All India 97) c) Race of a person d) Identification
a) Prematurity 1874. Urine formation in intrauterine life starts at-
b) Diarrhoea a) 3 months b) 4 months (Jipmer 04)
c) Malnutrition c) 5 months d) 6 months
d) Acute Respiratory Infection 1875. Fetal stage starts at- (Jipmer 04)
1863. In well baby clinic a neonate is brought with liver a) 9 weeks b) 3 weeks
two fingers is enlarged. The cause is- (AIIHPH 98) c) 6weeks d) 12 weeks
a) Normal phenomena b) Hepatitis 1876. Intra-uterine assessment of foetal distress
c) Choledochal cyst d) Biliary atresia indicated by- (PGI 04)
1864. Commonest cause of neonatal death in India is- a) Acceleration ofl5/min
a) Prematurity (AIJMS 83) b) Deceleration of30/min
b) Congenital malformations c) Variable deceleration 5-25/min
c) Metabolic disease d) Fetal HR<80/min
d) Birth injury e) Fetal HR 160-180/min

1852)a,c 1853)b 1854)b 1855)c 1856)d 1857)a 1858)a 1859)d 1860)b 1861)b l862)a 1863)a 1864)a 1865)d
1866)d 1867)c 1868)b 1869)d 1870)b 187l)d 1872)c 1873)a 1874)a 1875)a 1876)b,c,d
GYNAE & OBS [ 90]

1877. A 25 year old woman had premature rupture of 1885. Which is not true about cephalohaematoma?
membranes and delivered a male child who a) Not limited by sutures (APPG 08)
became lethargic and apneic on the 1•t day of and b) Swelling subsides in 3 months
went into shock. The mother had a previous c) Caused by periosteal injury of skull
history of abortion 1 year back. On culture, her d) None
vaginal swab growth of a hemolytic colonies on 1886. Infants born to mothers who are heavy drinkers are
blood agar was found. On staining these were at increased risk of which of the following problems ?
found to be gram positive cocci. Which of the a) Mental retardation (COMED 09)
following is the most likely etiological agent- b) Chronic lever disease
a) Streptococcus pyogenes (AI 04)
c) Excessive birthweight
b) Streptococcus agalactiae
d) Chorioretinitis
c) Peptostreptococci
d) Enterococcus faecium
INTRAUTERINE GROWTH RETARDATION
1878. The umbilical cord stump of a newborn most
frequently sloughs off about the- (Kerala 04) 1887. IUGRis caused by all except- (MP 98)
a) Second day after delivery a) Rh incompatibility b) Smoking
b) Fifth day after delivery c) DM d)CRF
c) 1Qth day after delivery 1888. IUGRischaracterizedbya!e- (PGIOJ)
d) 15th day after delivery a) Polycythemia b) Meconium aspiration synd
1879. Glucocorticoids given to a case of Preterm c) HMD d) Hypocalcemia
Labour help in reducing all of the following 1889. IUGRseenin- (PG/02)
neonatal complications EXCEPT- (Karnataka 02) a) Rubella b) Syphillis
a) Respiratory distress syndrome c) CMV d) Chickenpox
b) Intraventricular hemorrhage e) HPV
c) Necrotising enterocolitis 1890. True about intrauterine fetal death (IUD)-
d) Bronchopulmonary dysplasia a) Gas bubbles in great vessels (PGI 03)
1880. Which of the following is cause of non immune b) halo's sign +ve
hydrops- (SGPG/05) c) Overlapping of skull bone
a) B/L renal agensis d) i Amniotic fluid volume
b) Keil blood group isoimmunization 1891. In a case of intrauterine growth retardation, the
c) Rh-isoimmunization most dependable method of determining fetal well
d) Hypoproteinemia being is- (SGPGI 05)
1881. Causes of physiological jaundice- (MAHE 05) a) Ultrasonic fetal cephalometry
a) i Hb-F destruction b) Contraction stress test (oxytocin challenge test)
b) J. Conversion of bilirubin to urobilinogen c) Amniotic fluid triglyceride
c) Inadequate conjugation ofbilirubin d) Uteroplacental blood flow
d) All the above 1892. All are the causes ofintrauterine growth retardation
1882. Lanugo hair appears at- (UP 07)
except- (AI 05)
a) 4 months b) 5 months
a) Anemia
c) 6 months d) 7 months
b) Pregnancy induced hypertension
1883. Non immune hydrops fetalis is caused by all
c) Maternal heart disease
except- ( Comed 07, UP 2k)
a) ParvoB19virus d) Gestational diabetes
b) Chromosomal abnormalities 1893. Which ofthe following Doppler imdigns in IUGR
c) Alpha thalassaemia is associated with worst prognosis- (AI 07)
d) ABO incompatibility a) Reversal of diastolic flow
1884. Consider the following statements regarding a b) Absence of diastolic flow
cephalhaematoma- (UPSC-II 08) c) Absence of systolic flow
I. Present at birth d) Dicrotic notch
2 It can occur after a normal delivery 1894. Birth weight of a baby can be increased by-
3. The commonest site is over the parietal bone a) Cessation of smoking (Aiims May 07)
4. The bleeding is sub-periosteal b) Aspirin
Which of the statements given above are correct in c) ca++ and vitaminD supplement
the context of cephalhaematoma? (UPSC-II 08) d) Bed rest
a) 1 and 4 b) 1 and 2 only
c) 1,2and3 d)2,3and4

1877)b 1878)c 1879)d 1880)a 1881)d 1882)a 1883)c 1884)d 1885)a 1886)a 1887)a,c 1888)c 1889)a,c 1890)a,c
1891)b 1892)d 1893)a 1894)a
. GYNAE & OBS [ 91 ]

1895. Intrauterine foetal growth retardation may be 1906. Paracervical block relieves pain from all but one
associated with the following except- (UPSC 07) of the following-
a) Maternal drug addiction a) Pain from dilatation of the cervix
b) Maternal smoking b) Uterine pain
c) Foetal viral infection c) Relieves pain from the lower third of vagina and
d) Regular maternal aspirin intake episiotomy can by performed
1896. Difference between Prematurity & IUGR that d) Relieve pain from the upper third of vagina
Prematurity bas- (PGI June 08) 1907. The nerve roots blocked in pudendal nerve block
a) Sole creases all over feet is- (AI 93)
a) L1,2,3 b)L2,3
b) Breast module 2 mm
c) S2,3,4 d)S4
c) Ear cartilage well formed-good elastic recoil
1908. Drug strictly contraindicated in lactation-(PGJ 98)
d) Skin glistening thin
a) Lithium b) Carbamazepine
e) Poor muscle tone
c) Penicillin d) Ovarian
1909. Concentration of MgS0 4 in the treatment of
PHARMACOTHERAPEUTIC IN OBSTETRICS
eclampsiainmEqiL. (PGI 99)
a) 7-10 b) 10-15
1897. Oxytocin cballenge test for assessing fetal well
being is contraindicated in all except -(jiPMER 91) c) 2-4 d)4-7
a) Placenta previa b) Previous 2 LSCS 1910. Antimalarial(s)to be avoided in pregnancy-
c) Breeeh d) Premature labour a) Chloroquine b) Quinine (PGI OJ)
1898. Contraction stress test is used to detect -(.llPMER 93) c) Primaquine d) Anti-folates
a) Fetal hypoxia b) Placenta previa e) Tetracyclines
c) Hydramnois . d) Cord prolapse 1911. Use of which of the following drug is contra -
1899. Metbylergometrine is contraindicated is-(AIIMS 97) indicated in pregnancy? (AIIMS 03)
a) Multiparty b) Bicornuate uterus a) Digoxin b) Nifedipine
c) Amoxycillin · d) Enalapril
c) Postpartum haemorrhage d) Eclampsia
1900. Oxytocin is preferred over ergometrine in- 1912. All of the following are known side effects with
a) Atonic uterine bleeds following delivery the use of tocolytic therapy EXCEPT- (AIIMS 03)
b) In induction of labour (PGI97) a) Tachycardia b) Hypotension
c) Uterine bleeding following abortion c) Hyperglycemia d)Fever
· d) Following expulsion ofhydatidiform mole 1913. Therapeutic level of magnesium in serum for
1901. In which of the following conditions would treatment of eclampsia is- (UPSC 04)
prophylatic metbergin be contraindicated- a) 2-7mEq/L b)9-12mEq/L
a) Susppected multiple pregnancy ( UPSC 97) c) 15-17mEq!L · d)30-35mEq!L
b) Anaemia 1914. Misoprostol has been found to be effective in all
c) Cardiac disease oftbefollowingexcept- (AI 05)
d) Rh-ve mother a) Missed abortion
1902. Half life (biological) of oxytocin- (J & K 0I) b) Induction oflabour
a) 2-3min b)3-4min c) Menorrhagia
c) 5-6 min d) 7-8 min d) Prevention of post-partum hemorrhage (PPH)
1903. Magnesium sulphate potentiates the hypotensive 1915. Which one oftbe following methods for induction
action of (UPSC 97)
of labour should not be used in patient with
a) Methyl dopa b)Nifedipine previous lower segment caesarean section ?
c) Enalapril d) Hydralazine a) Prostaglandin Gel (UPSC 06)
b) Prostaglandin tablet
1904. Which does not affect the foetus- (AI 98)
a) Lytic cocktail b) Magnesium sulphate c) Stripping of the membrane
c) Warfarin d) Phenytoin d) Oxytocin drip
1905. Which of the following congenital malformations 1916. All are true aboufoxytocin except- (MAHE 07)
is seen in a child of a mother who is on treatment a) Causes regular uterine
with oral anticoagulants- (AIIMS 2K) b) Octapeptide
a) Craniofacial malformations c) .Promotes development of lobules of breast
b) Renal agencies d) Has ADH-like action
c) Long bone defects 1917. Oxytocin-induced myometrial contractions are
inhibited by all ofthe following except- (Gamed OT
d) Chondrodysplasia punctata
a) Ergometrine b) Terbutaline
c) Magnesium d) Halothane

1895)d 1896)b,d,e 1897)c 1898)a 1899)d 1900)b 1901)a,c,d 1902)b 1903)b 1904)b 1905)a,d 1906)c 1907)c
1908)a 1909)d 1910)c,d,e 19ll)d 1912)d 1913)a 1914)c 1915)b 1916)c 1917)a
GYNAE & OBS [ 92]

1918. Which of the followng anteretroviral therapy is 1930. Bishop scoring of more tban .•••••indicates labour
contraindicated in pregnancy- (UP 08) bas begun- (Assam 95)
· a) Nevirapine b) Efavirenz a) 3 b) 5
c) Lamivndine d) Stavudine c) 6 d) 7
1919. All of the following agents are tocolytics except- . 1931. Contraindication of induction oflabour-(Calcutta 2K)
a) Ritodrine b) Salbutamol (AI 08) a) PIH b) Bad obstetrical history
c) Isoxsuprine d) Misoprostol c) Dibetes d) Heart disease
1920. All are true about oxytocin except- (Manipal 08) 1932. All ofthe following drugs are effective for cervical
a) Causes regular uterine ripening during pregnancy except- (All India 04)
b) Octapeptide a) Prostaglandins E2 b) Oxytocin
c) Promotes development oflobules of breast c) Progesterone d) Misoprostol
d) Has ADH like action 1933. All of the following are used for induction of
1921. Which of the following drug (s) is/are not tocolytic
labour, except- (AIIMS 04)
agent- (PGI Dec 08)
a) PG F2 a tablet b) PG E2 tablet
a) Magnesium sulphate b) Betamethasone
c) PGE2 gel d) Misoprostol
c) Ritodrine d) Nifedipine
e) Misoprostol 1934. Bishops score includes all of the following ~xcept-
1922. Which of the following medications CtJn be safely a) Dilatation (AI 07)
used in pregnancy- (AI 09) b) Effacement
a) Propythiouracil b) Methotrexate c) Station of presenting part
c) Warfarin d) Tetracycline d) Interspinous diameter
1923. True about medroxyprogesterone acetate- 1935. Induction atterm is indicated in all of the following
a) Glucose intolerance occur {PGI Nov 09) except- (AI 08)
b) Pregnancyrate0.3/100womenyear a) Hypertension · b) Diabetes Mellitus
c) Anemia improves c) Heart disease d) Renal disease
d) Wt. gain . 1936. Which one oftbe following methods for induction of
e) 150 mg 3 monthly labour should not be used in patients with previous
1924. True about misoprostol- (PGI Nov 09) lower segment caesarean section ?
a) PGE2 a) Prostaglandin (DELHI PG Feb. 09)
b) Administered rectally b) Prostaglandin tablet
c) Needrefrigeration c) Stripping ofthe membrane
d) Used in postpartum haemorrhage d) Oxytocin drip
e) Induction oflst TM abortion 1937. What is the maternal risk of using misoprostol for
ripening of cervix during induction oflabour-
INDUCTION OF LABOUR a) Tachycardia (UPSC-II 09)
b) Hypotension
1925. Partogram in pregnancy includes all of the c) Tachy-systole/hyperstimulation of uterus
followingexcept- (UPSC96) d) Bradycardia
a) Uterine contraction 1938. Half-lifeofiVoxytocin? (Maharashtra 10)
b) Cervical dilatation in time scale
a) 1 minute b) 3 minutes
c) Descent of foetal head in time scale
c) 7 minutes d) 9 minutes
d) Non-stress Test
1926. Bishop's score include all except- (PGI 98)
OPERATIVE OBS
a) Dilatation of cervix
b) Effacementofcervix
1939. Vacuumextractionisdonewben- (PG/90)
c) Contractions ofutems
a) Cervix dilated more than 8 em.
d) Station of the head
1927. Cervical ripening is mainly due to the action of- b) Persistent occipito posterior
a) PGE2 b) PGF2 (TN99) c) Deep transverse arrest
c) PGI2 d) PGG2 d) Acute destress in lind state
1928. Indication for induction oflabour is- (Kerala 94) 1940. Internal podalic version is done- (AI 91)
a) Placenta previa b) PlH at term a) At 32 weeks in transverse lie
c) Heart disease d) Breech b) Transverse lie of 2nd twin
1929. A.R.M. is contraindicated- (UPSC 96) c) When there is minimal amniotic fluid
a) In pregnancy beyond 40 weeks d) All of the above
b) When the mother is RH-ve 1941. Internal rotation occurs atthe- (TN 91)
c) When the mother is diabetic a) Brim b) Cavity
d) When the foetus is dead · c) Ischial spine d) Outlet

1918)b 1919)d 1920)c 192l)b,e 1922)a 1923)All 1924)b,d,e 1925)d 1926)c 1927)a 1928)a,b 1929)d
1930)c 193l)b,d 1932)c 1933)a 1934)d 1935)c 1936)b 1937)c 1938)b 1939)a,b,c 1940)b 194l)c,d
GYNAE & OBS [ 93]

1942. The effective pressure to be achieved in vacuum 1954. Vaccum extraction is C.I. in- (PGI 98)
extraction is- (AIIMS 92) a) Premature babies b) Heart disease
a) 0.1 kg/cm2 b) 0.4 kg/cm2 c) Microcephaly d) Polyhydramics
c) 0.6kg/cm2 d) 1.2kg/cm2 1955. The preferred method for management of neglected
1943. Classical caesarian section is indicated in all shoulder presentation in a rural hospitalin India is-
except- (AIIMS92) a) Wait for spontaneous delivery (Kerala 97)
a) Post Mortam delivery b) Embryotomy
c) Caesarian section
b) Lower segment fibroid
d) None of the above
c) Post RVF repair
1956. Outlet forceps is least likely to lead to- (AIIMS 98)
d) Post VVF repair a) Complete perineal tear b) Extension ofLMLE
1944. Which is an absolute indication for LSCS- (AI 93) c) Cervical tear d) Vuval hematoma
a) Type IV placental previa b) Fetal Distress 1957. Which of the following is the indication ofinternal
c) Previous LSCS d) Breech presentation podalic version- (TN 99)
1945. Indication for classical cesarian section is- a) 2•d of the twin b) Shoulder presentation •
a) Obstructed labour (Kerala 94) c) Breech d) Mentoposterior position
b) Carcinoma cervix 1958. While doing caesarean section, the lower segment
c) Placenta previa of the uterus is physically identified by - ( UPSC 2K)
d) Twin pregnancy a) Loose attachment of visceral peritonium
1946. Midline episiotomy has all the following advantage b) Dilated venous sinuses
over mediolateral episiotomy except- (Kam 94) c) Deflection of uterine artery towards upper segment
a) Better healing b) Extention less common d) Thinness of its wall as compared to the upper
c) Less blood loss d) Dyspareunia later is less segment
1947. Absolute indication for caesarean section is- 1959. Treatment for neglected shoulder presentation
a) Dystocia (JIPMER 95) is - . (TN 9 5, UPSC: 85, PGI 88)
b) Previous rupture of uterus a) Caesarean section b) Vaginal delivery
c) Breech presentation c) Decapitation d) Eviseration
d) Fetal distress 1960. Indication of classical caesarian section- (PGI 03)
1948. Caesarian section is must in which type of a) CaCervix b) Kyphoscoliosis
placenta previa-: (Assam 95) c) Previous 2 LSCS d) HSV infection
a) Type I b) Type II ant. · e) Contracted pelvis
c) Type III d) All of the above 1961; Contra-indication ofVacuum Extraction- (PG/04)
1949. Incorrect about external cephalic version is- a) Prematurity b) Brow presentation
a) Done at 34 weeks (AP 96) c) Foetal distress d) Floating head
b) Abruption of placenta is a complication e) Undilatedcervix
c) Done under general anesthesia 1962. Classical Caesarian Section is not indicated in-
d) None a) Lower segment (Kerala 03)
1950. Only indication for internal version nowadays b) Carcinoma Cx
is - (JJPMER98) c) Fibrid uterus
a) Brow presentation b) Face presentation d) Central placenta previa
c) Second fetus of twins d) Breech 1963. Which statement is true regarding VENTOUSE
(Vacu~m Extractor)- (AIIMS 03)
1951. On external cephalic version, fetal bradycardia
occurred. The next course of action is- (AP 97) a) Minor scalp abrasions and subgaleal hematomous
a) Reversion to the original position immediately to new born are more frequent than forceps
by external version b) Can be applied when fetal head is above the level
b) Internal podalic version of ischial spine
c) Caesarian section c) Maternal trauma is more frequent than forceps
d) Rupture of the membranes d) Cannot be used when fetal head is not fully rotated
1964. Ventouse application, the prerequisite is- (APPGE
1952. During suction evacuationin MTP the negative
a) Full dilatation of cervix b) Station +2 04)
pressure of suction should be- (UPSC)
c) Premature d) Head engaged
a) 100to200mmHg b)200to300mmHg
1965. A case of central placenta praevia with anencephaly
c) 400 to 600 mm Hg d) 700 to 900 mm Hg
foetus should be delivered by- (SGPGI 05)
1953. In MTP the suction pressure is -----------mm
a) Caesarean section
Hg- (PG/98)
b) Induction of labour
a) 100-200 b)200-300
c) Hall breech extraction
c) 600- 700 d) 700- 800 mm!Hg
d) Application ofWillet's forceps

1942)None 1943)c 1944)a 1945)b,c 1946)b 1947)None 1948)c 1949)c 1950)c 195l)a 1952)c 1953)c
1954)a 1955)c 1956)None 1957)a 1958)a 1959)c 1960)a,e 196l)All 1962)d 1963)a 1964)d 1965)a
GYNAE & OBS [ 94]

1966. What is the graphical record of cervical dilatation 1976. The indications of an elective caesarean section
in centimeters against duration oflabour in hours include all of the following, except- (UP SC II 10)
called- (UPSC 06) a) Placenta praevia
~ Partognun b)Pictognun b) Cephalopelvic disproportion
c) Hysterograph d)Amniograph c) Previous lower segment caesarean section
1967. Contraindications to ventouse delivery include all
d) Carcinoma cervix
of the following except- (COMED 06)
a) Fetal coagulopathies 1977. Risk of scar rupture in classical caesarean section
b) Extreme prematurity is- (Comed 10)
c) Mento transverse position a) 0.5 to 1.5% b) 1.5 to 4%
d) Occ;ipito transverse position c) 4to9% d) 10to 12%
1968. Indication for internal podalic version in modern 1978. Pre-requisites for forceps delivery are all EXCEPT-
obstetrics - a) Adequate analgesia (Karn 11)
a) 2nd twin with transverse lie b) Fully dilated cervix
b) Anencephaly c) Relaxed uterus
' c) 1st twin with transverse lie d) Ruptured membranes
d) Breech presentation 1979. The commonly used frequency range in obstetrical
1969. In modern obstetrics, the internal podalic version is ultrasonography is- (Karn 11)
indicated in the presence of- (UPSC-II 08) a) 3.5-5KHz b)20-25KHz
a) Transverse lie with obstructed labour c) 3.5-5MHz d)20-25MHz
b) 2nd transverse twin
c) Cord prolapse with live baby DIAGNOSIS IN OBSTETRICS
d) Extended breech presentation
1970. Which ofthe following is not a contraindication of 1980. Investigation contraindicated in antenatal
vaginal delivery after previous cesarean section expectant mother is- (TN 91)
(CS)? (AI 08)
a) X-rays b) Ultrasound
a) Previous classical CIS
c) Urine examination d) Blood examination
b) No vaginal delivery in the past
c) Breech presentation , 1981. Crown-Pump length of 120 mm means the
d) Puerperia! infection in previous pregnancy gestationalageis- (JIPMER91)
1971. Not a contraindication for external cephalic version? a) lOweeks b) 14weeks
a) Breech presentation (APPG 08) c) 20 weeks d) 24 weeks
b) Obstructed labour 1982. Which of the following is not an indication of
c) Oligohydramnios Amnniocentesis for Chromosomal detection-
d) First baby of twin pregnancy a) Diabetesmelitus (JIPMER91)
1972. Indications for caesarean section in pregnancy are b) Previous Down's child
allexcept- (AIIMSMay09) c) Maternal age more than 35
a) Eisenmenger syndrome d) Parents with Chromosomal anomaly
b) Aortic stenosis 1983. Patogram help in the detection of- (JIPMER 92)
c) M.R. a) PPH
d) Aortic regurgitaion b) Abruption placetae
1973. A woman delivers a 9 lb infant with a midline c) Obstracted labour
episiotomy and suffers a third degree tear. Inspection d) lncordinate uterine action
shows which of the following structures is 1984. Radiological investigations in a female of
intact- . (DELHIPGFeb. 09) reproductive age should be restricted to- (AJ93)
a) Anal sphincter b) Perineal body a) Peroid of menstruation
c) Perineal muscle d) Rectal mucosa b) First 10 days of menstrual cycle
1974. The estimated risk of uterine rupture is a woman c) Last 10 days of mentrual cycle
· with a previous lower segment uncomplicated d) 1Olh to 20th day of cycle
cesarean section is approximately- 1985. Early decceleration pattern on cardiotocography
a) 0.5-2% b)4-6% (DELHIPGMar. 09) indicates- . (JIPMER 93)
c) 8-1oPAD d)14-16% a) Head compression b) Fetal distress
1975. The following are contra-indications to External c) Cord prolapse d) Hypoxia
Cephalic Version except- (DPG 10)
1986. To diagnosis foetal well being of foetus, most
a) Contracted pelvis
reliable method- (UPSC 97)
b) Ante-partum haemorrhage
a) L:Sratio b)HPLlevels
c) Multiple pregnancy ·
d) Hydramnios c) Kick test d) Ultrasoupd

1966)a 1967)d 1968)a 1969)b 1970)c 197l)d 1972)a 1973)d 1974)a 1975)d 1976)c 1977)c 1978)c 1979)c
1980)a 1981)c 1982)a 1983)c 1984)b 1985)a 1986)d
GYNAE & OBS [ 95]

1987. Maternal serum Alpha Feto Protein (MSAFP) raised 1998. Increased acetyl cholinesterase in amniotic fluid
in all the following conditions except- (Karn. 94) is seen in- (AI 97)
a) Spina bifida b) Multiple pregnancy a) Neural tube defect
c) Omphalocele d) Down's syndrome b) Renal agenesis
1988. X-rayabdomeninpregnancyis_- (Kerala 95) c) Trachea oesophageal fistula
a) Harmful to the foetus d) llJGR .
b) to assess gestational age 1999. Detection of Fetal abnormalities is possible by all
c) To localise placenta except- (AI 9 7)
d) To rule out congenital anomalies a) Maternal blood b) Fetal blood
1989. Hydrocephalus in the foetus is diagnosed by- c) Amnion d) Decidua (maternal)
2000. Manning score for fetal well being includes all
a) Large frontanellae with widely separated sutures
except- (AI 9 7)
as felt onPV examination (Kerala 95)
a) Non stress test
b) High floating head
b) Oxytocin Challenge test
c) Serial ultrasound c) Breathing movements
d)CPD d) Body movements of fetus
1990. Best investigation to diagnose foetal age is- 2001. Determination of gestational age in 1st trimester
a) Serial Ultrasound (Kerala 95) byUSG- (PGI 96)
b) Aminocentesis a) Crown-Rump length
c) Fundal height measurement b) Biparietal diameter
d) X-ray c) Abdominal cocircumferance
1991. Screening for Down's syndrome should be done d) Femur lengths
in the age group_ _in pregnancy- (A/IMS 94) 2002. While assessing foetal wall being by
a) 30 cardiotocography significans is attached to -
b) 35 a) Late decelaration (AIIMS 96)
c) All in the reproductive age group b) Early decelaration
d) None c) Sinusoidal wave
1992. All of following are indications for X-ray d)Variable decelartion
pelvimetry in pregnancy except- (AI 96) 2003. Foetal heart can be detected earliest with
a) Severe CPD transvaginal sonography at (from the last menstrual
b)'Breech presentation in a primigravida period)- (Kerala 97)
c) Suspected outlet contraction a) 35 days b) 38 days
d) Previous caesarian section c) 53 days d) 46 days
1993. All of following are true about fetal distrees 2004. All are signs of fetal distress except- (PGI 97)
expect- (AI 96) a) Tachycardia
a) Mecorium stained liquour b) Bradycardia
b) FHS < lOOmint. c) Decreased scalp pH
c) Scalp blood pH 7.3 d) Increase fetal movements
d) Excessive fetal movements 2005. Chorionic villi biopsy is not helpful in -(AIIMS 97)
1994. The best investigation for dating pregnancy is- a) Pheny liketonuria b) Thalassemia
a) Crown rump length at 9 weeks (Delhi 96) c) Neural tube defects d) Haemophilia
b) BPD at 18 weeks 2006. All are used for Karyotyping in amniocentesis.
c) Abdominal circumference at 24 weeks except- (AI 99)
d) BPD at 24 weeks a) Monocyte b) Lymphocyte
1995. Earliest fetal anomaly to be detected by USG- c) Fibroblast d) Amnion cells
a) Hydrocephalous (Karn. 95) 2007. Acetyl cholinesterase assay can be used to
b) Anencephaly diagnose- (AI 99)
c) Achondroplasia a) Open neural tube defect b) Oesophageal atresia
d) Spina bifida c) Duodenal atresia d) Biliary atresia
1996. Antenatally the measurement most useful in 2008. Triple test for Down's syndrome is by using all
ultrasound- (AP 96) except- (AI 99)
a) Crown Rump Length b) Femur length a) Maternal HCG b) Maternal estriol
c) BPD d) Abd. circumferance c) Maternal a feto protein d) Maternal HPL
1997. Cord compression causes__deceleration - 2009. AFP levels are highest in- (AIIMS 98)
a) Early b) Late (AP 96) • a) Foetal serum b) Placenta
c) Variable d) None c) Amniotic fluid d) Maternal serum

1987)d 1988)All 1989)a 1990)a 1991)b 1992)a,d 1993)c 1994)a 1995)b 1996)a,d 1997)c 1998)a 1999)c,d
2000)b 2001)a 2002)a,c,d 2003)d 2004)None 2005)c 2006)a 2007)a 2008)d 2009)a
GYNAE & OBS [ 96]

2010. Cardiac activity of the fetus seen by ultrasound 2.023. A drop in fetal heart rate that typically lasts less
by- (TN99) than 2 minutes and is usually with umbilical cord
a) 4 weeks b) 5 weeks compression is called- (AIIMS 03)
c) 6weeks d)7weeks a) Early deceleration
2011. Normal crown- rump length at birth is- (AP 98) b) Late deceleration
a) 20-25 ems. b) 38-50 ems. c) Variable deceleration
c) 80-72 ems. d) 78-85 ems. d) Prolonged deceleration
2012. The postion used for bimanual examination in the
2024. Earliest ultrosound sign of pregnancy in a
out-patient department is- (Karn. 90)
transabdominal ultrasound scan is-
a) Lithotomy placenta
b) Supine position a) Visible gestational sac (Karnataka 03)
c) Sim's position b) Apparent embryonic structures
d) Dorsal position with thighs flexed c) Fundal endometrial thickening
2013. The most specific finding in amniotic fluid to d) Identifiable cardiac movements
diagnose neural tube defect is ....•. - (AIIMS 99) 2025. Which of the following explanations is NOT an
a) Acetyl choline esterase explanation for decreased variability of the fetal
b) Acetyl glucosamine heart rate tracing- (Karnataka 03)
c) Cysteinase a) Fetal "sleep state" b) Prematurity
d) Glucose oxidase c) Barbiturate ingestion d) Fetal stimulation
2014. Chronic villous sampling is done at- (OrissaR) 2026. Variable decelerations on electronic fetal
a) 4-6 weeks b) 7-8 weeks monitoring in a woman who is in labour indicates-
c) 9-11 weeks d) 12-13 weeks a) Congenital heart disease (COMED 06)
2015. Normal size of CBD on USG in pregnancy- b) Compression of head
a)24mm b)6-9mm (DNBOJ) c) Administration of sympathomimetic drugs to the
c) 10-15mm d) 15-20mm mother
2016. External genitalia earliest diagnosed by USA d) Umbilical cord compression
by- (J &K 2001) 2027. Amniocentesis is best diagnostic if done in _ __
a) 10 weeks b) 14 weeks weeks- (DELHI PG Feb. 09)
c) 16 weeks d) 18 weeks a) 8-10 b) 12-14
2017. FetalHypoxiaisassessedby- (A.l. 89) c) 16-18 d)20-22
a) Scalp blood pH 2028. Chorionic villi biopsy is not done before 9 weeks
b) Alpha feto protein levels because of- (DELHIPGFeb. 09)
c) FetaI heart monitoring a) Abortion
d) None b) Feto-maternal risk of hemorrhage
2018. Fetal Respiratory movements occur earliest at- c) Limb malformation
a) 12wks b)l6wks(JIPMER91) d) Too little material
c) 20wks d) 11 wks 2029. True about amniocentesis- (PGI Nov 09)
2019. Indication offetal distress is- (Kerala 94) a) Spinal needle is used b) Fetoscope is used
a) Decrease in fetal scalp blood pH c) Doneb/wl2-16wks. d)Doneb/w15-20wks.
b) Meconium staining e) Fetal loss in the procedure is more than 5%
c) Late deceleration of heart rate
2030. The presence of increased levels of which of the
d) All of the above
following in amniotic fluid is an indicator of open
2020. Fetal bradycardia is defined as- (TN 99)
neural tube defect in the fetus? (AIIMS May 1 0)
a) Less than 100 beat/min
a) Phosphatidylesterase
b) Between 100 and 200 beat/min
b) Acetyl cholinesterase
c) Less than 120 beat/min
c) Pseudocholineesterase
d) Less than 120 beat/min for a period of 15 min of
continuous monitoring d) Butyrylcholineesterase
2021. Fetal scalp blood pH less than-- is abnormal- 2031. Which of the following is the most specific marker
a) 725 b)7.3 (AMC2K,JIPMER91) for Neural tube defects- (AI 10)
c) 7.35 d) 7.4 a) Actylcholinesterase
2022. Mannigs score includes- (PGI OJ) b) Pseudocholinesterase
a) Fetal movements c) Alpha feto protein (AFP)
b) Respiratory movements d) Human chorionic Gonadotrophin (HCG)
c) Placental localization 2()32. AFP is raised in - (AI 10)
d) Fetal tone a) Teratoma b) Yolk sac tumor
e) Fetal heart rate c) Choriocarcinoma d)Dysgerrrrinoma

2010)c 201l)b 2012)d 2013)a 2014)c 2015)c 2016)a 2017)a,c 2018)d2019)d 2020)d 202l)a 2022)a,b,d
2023) c 2024) c 2025) d 2026) d 2027) c 2028) c 2029) a,c 2030) b 2031) a 2032) b
GYNAE & OBS [ 97]

2033. Increased Nuchal Translucency at 14 weeks 2046. There is steady fall in serum iron at- (PAl 80)
gestation is seen in- (AJ10} a) 12th week b) 20th week
a) Turner's syndrome b) Down's syndrome c) 24th week d) 28th week
c) Hydrocephalus d) Skeletal dysplasia e) 32rutweek
2034. Earliest fetal anomaly to be detected by 2.()47. Blood will interfere with the nitrazine test for
USG- (DPG 10) detecting ruptured membrances because-(DNB 90)
a) Hydrocephalous b) Anencephaly a) It is acid
c) Achondroplasia d) Spina bifida b) It is alkaline
2035. SerumAFP (alpha fetoprotein) levels are increased c) It contains increased amounts of sodium chloride
at 16 weeks of pregnancy in all the following d) In contains decreased amounts of sodium chloride
conditions, except- (UP SC II 10) e) It reacts with the normal flora of vaginal bacteria
. a) Gastroschisis b) Neural tube defects to give an acid reaction
c) Multiple pregnancies d) Down's syndrome 2048. Foetal requirement of iron is -(AIIMS 78, DNB 90)
2036. Abnormal foetal heart-rate patterns on electronic a) 200mg b)300
foetal monitoring include the following, except- c) 400 d)500
a) Bradycardia< 120/minute lasting for 15 minutes 2049. Which ofthe following is not used in DIC-
b) Tachycardia> 170/minute lasting for 15 minutes a) Heparin (AIIMS 90, Bihar 89)
c) Early decelerations (UP SC II 1 0) b) Epsilon amino caproic acid
d) Late decelerations c) Blood transfusion
d) Intravenous fluids
MISCELLANEOUS (OBS) 2050. The first investigation to be done if a woman reports
with post term pregancy is- (JIPMER 95)
2037. Study of fetal parts in first trimester with least a) Ultrasonography b) Vaginal examination
radiation hazard- (TN 90) c) Blood glucose profile d) X-ray abdomen
a) X-ray abdomen b) Pelvimetry 2051. In postmaturity which among the following does
c) CT scan. d) Ultrasound not occur- (AIIMS 94)
2038. Multiparity is associated with all except- (AI 91) . a) Fetal distress b)Hydraminios
a) Pregnancy induced hypertension c) Transverse lie d) None
b) Anaemia 2052. A patient had undergo suction evacuation for 10
c) Postpartum haemorrhage weeks pregnancy. Three days later she came back
d) Malpresentation with profuse vaginal bleeding and abdominal
2039. Which vaccine is not indicated during pregnancy- pain. The most likely diagnosis is- (UPSC 96) .
a) Polio b)Typhiod(JIPMER91) a) Uterine atony b) Cervical injury ·
c) Rubella d) TT c) Retained products d) Uterine perforation
2040. Large of date baby may be due to- (JIPMER 92) 2053. Increased gestational period is associated with
a) Beckwith syndrome b) Diabetic mother following congenital anomaly - (Delhi 96)
c) Genetic predisposition d) All a) Spina bifida b) Anencephaly
2041. Which is more common in primigravida than c) Meningomyelocele d) Omphalocele
multigravida- (AJIMS 92) 2054. Elderly primi is above- (Karn 96)
a) PPH b) Placenta previa a) 30 years b) 35 years
c) Malpresentation d) Hyperemesis c) 37 years d) 45 years
2042. Post term labor is seen in- (AIIMS 92) 2055. All of the following are high risk pregnancies·
a) Hydramnois b) PID except- (AIIMS 85)
c) Anencephaly d) Multiple pregnancy a) Twins
2043. Post term pregnancy is that which continues b) Previous history of postmaturity
beyond- (AIIMS 92) c) Threatened abortion
a) 300 days b) 294 days d) Previous history of manual removal of pacenta
c) 280 days d) 270 days 2056. The condition where a thin walled sac. develop
2044. Stain used for maturity assessment of amniotic in myometrium often containing fetal parts-
fluids cells- (JIPMER 93) a) Uterus didelphys (AIIMS 95)
a) Leishman's stain b) Sudan red b) Uterine incarceration
c) Nile blue sulfate d) Congo red c) Hernia of uterus
2045. What is the value of color index in microcytic d) Uterine sacculation
hypochronic anemias associated wi.th pregnancy - 2057. Routine pelvic examination is contrainidicated
a) One b) Less than one (AIIMS 91) in- (AIIMS 85)
c) More than one d) Three a) Carcinoma cervix b) Cord prolapse
e) Four c) Placenta previa d) PIH

2033)b 2034)b 2035)d 2036)c 2037)d 2038)a 2039)c 2040)d 2041)d 2042)c 2043)b 2044)c 2045)b 2046)b
2047) b 2048) b 2049) b 2050) a 2051) b,c 2052) c 2053) b 2054) a 2055) b 2056) d 2057) c
GYNAE & OBS [ 98]

2058. Premature rupture ofmembraue is rupture- 2069. Single umbilical artery is associated with all of
a) At less than 32 weeks (AI 97) the following except- (UPSC 2K)
b) Before onset of labour a) Polyhydramnios
c) Before 2nd stage oflabour b) Advanced maternal age
d) All of the above c) Foetal growth retardation
2059. lu a 24-28 weeks prenancy with premature d) Increased incidence o foetal malformation
rupture of membrane you will always give- 2070. A primipara presented at 10 weeks of gestation
· a) Steroids b) Antibiotics (PGI 96) with lower abdominal pain. On examination,
c) Tocolytics d) Analgesia there was pallor and lower abdominal tenderness.
2060. "Chemical pregnancy" means- (UPSC 97) What is the likely possibility- (AIIMS 99)
a) Negative beta HCG and absent gestational sac a) Ruptured ectopic b) Twisted ovarian cyst
b) Positive beta HCG and present gestational sac c) Red degeneration d) Acute appendicitis
c) Positive beta HCG and absent gestational sac 2071. 22 year old Devi, presents with shock and minimal
d) Negative beta HCG ~nd crenated sac margin spotting per vagina. The most likely diagnosis ·
2061. Indications for termination of pregnancy in is- (AI2K)
hyperemesis gravidarum include all except- a) Threatened abortion b) Ruptured ectopic
a) Jaundice (AP 97) c) Twisted ovarian cyst d) Ruptured lutein cyst
b) Oliguria 2072. All are seen in non immuno hydrops fetalis
c) N euro logoical complications except- (MP 2K)
d) Encephalopathy a) Pericardia! effusion b) Large placenta
2062. Opthalmoscopic examination is indicated in all c) Skin edema d) Ascites
ofthe following except- (AP 97) 2073. All ofthe following are part of manning's criteria
a) Pre-eclampsia b) Placenta praevia except- (AIIMS 2K, MP 2K)
c) Hyperemesis gravidarum d) Raisedi.C.T. a) Foetal resiratory movements
b) Non stress test
.2063. Early cord clamping is done is all except- (MP 98)
c) Foetal body movements
a) Rhisoimmunization b) Foetal asphyxia
d) Vibroacoustic stimulation
c) Postmaturity d) Prematurity
207 4. Leela 22 yrs old primipara presents with post dated
2064. A primigravida is fully dilated with vertex
pregnancy. First thing you would like to do
presentation. The head is well rotated and the
is- (AIIMS OJ)
station is +2. There is late deceleration on foetal
a) Ultrasonography b) Review mentrual history
heart monitoring. If the pelvis is adequate, the
c) Induce labour d)LSCS
most appropriate management in this case would 2075. Smoking in pregnancy causes- (CUP GEE 2K)
be- (UPSC99)
a) IDGR b)PIH
a) Waiting for spontaneous delivery c) APH d)PPH
b) Forceps delivery . 2076. pH of amniotic fluid at later weeks of
c) Vacuumextraction gestation- (TN 02)
d) Caesarean section a) 4.5 b)6.5
2065. Fundal height is more than period of gestation in c) 7 d)7.2
all except- (Rohtak 98) 2077. Supine hypotensive syndrome occurs in- (PGI 02)
a) Hydramnios b)IDD a) Obesity b) Ascites
c) Twin pregnancy d) Hydratidiformmole c) Early pregnancy d) Advanced pregnancy
e) Uterinemyoma e) Immediately following delivery
2066. Vitamin which is taken to prevent neural tube 2078. Pregnant women who have had five or more
defect in fetus- (AI 99) previous babies differ from those who have had
a) Thiamin b) Folic acid none in thatthey have a greater risk of- (AMC 85)
. c) Pyridoxine d) Vitamin C a) Developing anaemia
2067. Diagnostic test for predicting congenital anomaly b) Complications due to Rhisus incompatibility
in a diabetic pregnancy is- (AIIMS 98) c) Malpresentation
a) Hb Ac b) Frutasamine d) Involuntary passing of urine while coughing or
c) AmnioticAFP d) SerumAFP laughing
2068. If a patient comes with complaints of post dated e) All of the above
pregnancy what is the first thing that you will do 2079. Head's Reflex is due to stimulation of- (AIIMS 91)
a) USG (JIPMER 2K) a) J. Receptors
b)NST b) Irritant receptors of lungs
c) Review the menstrual history once more c) Pleural stretch receptors
d) X-ray abdomen d) Diaphragmatic nerves

2058) b 2059) b 2060) c 2061) b 2062) b 2063) c 2064) b 2065) b 2066) b 2067) a 2068) c 2069) b 2070) a 2071) b
2072) None 2073) d 2074) b 2075) a,c 2076) d 2077) d 2078) e 2079) b
GYNAE & OBS [ 99]

2080. Gas in great vessels offoetus indicate- (Kerala 94) 2091. Differential diagnosis ofHyper emesis gravidarum-
a) Fetal distress b) Post mature fetus a) Gastritis b) U.T.I. (PGI 03)
c) Fetal death d) Premature fetus c) Toxaemia of pregnancy d) Reflux oesophagitis
2081. Vaginal delivery in subsequent pregnancy can be 2092. ,J. HCG level seen in- (PGI 04)
permitted in all of the following patient situations a) Down syndrome b) DM
except~ · (MP 98) c) Multiple pregnancy d) Ectopic pregnancy
a) Previous LSCS for antepartum hemorrhage e) Oesophageal atresia
b) Pervious LSCS for transverse lie 2093. Vaginal delivery is contra- indicated in -
c) Myomectomy done 2 year back a) Central placenta praevia (Jipmer 03)
d) Repair ofvesico vaginal fistula b) Previous LSCS
2082. A 26 year old women in the first trimester of c) Eclampsia
pregnancy has been admitted with retching and d) Ante - partum hemorrhage
repeated vomiting with large haematemesis. Her 2094. Abortifacient causing ototoxicity is- (TN 03)
pulse rate is126/minute and blood pressure b 8® a) Lead b) Quinine
mm Hg systolic. The most likely diagnosis is- c) Mercury d) Ergot
a) Mallory-Weiss syndrome (UPSC95) 2095. B Lynch suture is applied on - · (AI 03)
b) Bleeding from oesophageal varices a) CeJvix b) Uterus
c) Peptic ulcer c) Fallopian tubes d) Ovaries
d) Hiatus hernia 2096. A woman in her first pregnancy reports that sl!ne
2083. ARM is contraindicated in- (PG/98) smokes one pack of cigarettes a day. An ultrasound
a) Pl. previa b) Hydramios is ordered in the thirty second week of the pregnan~y
c) Ace. Hem d) Twins to evaluate for which of the following- (Karn 03)
2084.· Spalding's sign is ®een in~ (PG/99) a) Amniotic fluid volume b) Fetal size
a) Still born b) Live born c) Fetal abnormalities d) Fetal.motion
c) Premature d) Dead born 2@97. Maximum permissible radiation do!le in pregun:illntey
2085. Adult hemoglobin appears in fetus at the is - (AIIMS 03)
gestational age of(in weeks)- (PGI 2000) a) 0.5rad b) l.Orad
a) 51h b) lOtl' c) 1.5rad d)3.0rad
c) 15111 d) 20th 2098. A lady comes with 8 weeks amenorrhea, shock &
e) 25th past history.ofTB diagnosis is- (Jharkand 03)
2086. Which of the following is associated with a) Ruptured ectopic b) Septic Shock
hydrocephalus- (PGI OJ) c) DIC d)None
a) Diabetes melitus b) Pre eclampsia 2099. Maturation index during pregnancy- (APPGE 04)
c) Abruptio placentae d) Breech presentation a) 0,40,50 b)50,40,0
e) Spina bifida c) 0, 0, 100 d) 0, 95, 5
2087. Delayed labour occurs in- (PGI OJ) 2100. ilnprovidentmotheris- (AMU05)
a) Early use of epidural anesthesia with analgesia a) Mother having three children & lost one
b) Early use of analgesia b) Mother having three children & lost two
c) Unripened cervix c) Mother with all children lost
d) Preeclampsia d) Mother giving birth to quadruples
e) Use of sedative early in course oflabor 2101. MMRisexpressedin- (PG/June 05)
2088. USGs/ofetaldeath- (PGI OJ) a) Per 1,000 live birth b) Per I 0,000 live birth
a) 'Halo' sign ofhead b) Heart beat absent c) Per Lac live birth d) Per I 0 Lac live birth
c) Spalding sign d) Hegar's sign 2102. Banana & Lemon sign seen in which fetal
· 2089. Pyometra is seen in- (PGI 02) anomalies- (PGI June 05)
a) Stenosing veriety of cervical carcinoma a) Neural tube defects b) Hydrops fetalis
b) Senile vaginitis c) Twins d) IUD
c) Senile endometritis e) Down syndrome
d) Endometriosis 2103. All the following can cause DIC during pregnancy
e) Abnormalitis of uterus except- (AIIMS May 05)
2090. A lady__.Of150 em height with Hb of11gm%; BP of a) Diabetes mellitus b) Amniotic fluid embolism
160/110 mm Hg & 12 kg gain during her pregnancy c) Intrauterine death d) Abruptio placentae
delivered an IUGR baby, the causes in this case are- 2104. A multipara with previous LSCS comes at38 weeks
a) Maternal infection b) Short stature (PGI 03) pregnancy in shock. DD includes- (PG/ June 06)
c) Hypertension d) wt. gain a) Placenta previa b) Abruptio placenta
e) Hb% c) Rupture uterus

2080)c 2081)d 2082)a 2083)b 2084)d 2085)e 2086) d,e 2087)a,b,c,e 2088)b 2089)a,b,c 2090)c 2091)a,b,d
2092)d 2093)a 2094)b 2095)b 2096)b 2097)a 2098)a 2099)d 2100)c 210l)a 2102)a 2103)a 2104)a,b,c
GYNAE & OBS [ 100]

2105. Obesity in pregnancy causes all of the following c) Excess dosage of oxytocin
complication except- · (Manipal 06) d) Complicated manual removal of placenta
a) Abnormal uterine action 2116. Common cause of"Sepsis syndrome" in obstetrics
b) Fetal neural tube defect includes- (Maharashtra I 0)
c) Precipitate labour a) Antepartum pyelonephritis
d) Venous thrombosis b) Puerperal infection
2106. With reference to syndromic approach in c) Chorioamnionitis
reproductive tract infections, consider the following d) All of the above
statements- (UPSC 07) 2117. Match List-1 with List-11 and select the correct
1. Single drug can be used for treatment answer using the code given below the Lists -
2. The diagnosis of exact disease is not relevant List-1 (Pregnancy List-11 (Therapy)
3. The management is disease specific complications)
4. This is an important part of family Health A. Anti-phospholipid 1. Intravenous
Awareness compaign syndrome immunoglobulins
Which ofthe statements given above are correct B. Acute toxoplasmosis 2. Ursodeoxycholic
a) 1 and 2 only b) 1 and 3 only acid
c) 2and4only d)l,2,3and4 C. Unexplained pregnancy 3. Low-dose aspirin
2107. The risk ofMonogolism in a mother at the age of losses and heparin
20 years is 1 : 3000. What would be this ration D. Cholestatic jaundice 4. Spiramycin
when she is 45 years old- (UPSC 07) Code: (UPSCIIIO)
a) 1:6000 b) 1 : 3000 A B C D
c) 1: 1040 d) 1 :50 a) 2 4 1 3
2108. The Kleihauer test for detecting foetal b) 2 1 4 3
erythrocytes is based on the fact that- (UPSC 07) c) 3 4 1 2
a) Adult erythrocytes are larger than those of foetus d) 3 1 . 4 2
b) HbA has higher oxygen affinity than HbF 2118. The following drugs can cause osteoporosis, except-
c) HbF is more resistant acid elution that HbA a) GnRH analogue b) Danazol (UP SC II I 0)
d) HbA takes up erythrosin stain less than HbF c) Corticosteroid d) Mefiprostone
2109. Age of gestation when ovaries and testis are first 2119. Prolong latent phase is/are seen in -(PGI May I 0)
distinguishable- (Corned 07) a) Placenta praevia b) Unripe cervix
a) 4 weeks b) 8 weeks c) Abruptio placentae d) Excessive sedation
c) 12weeks d)l6weeks e) Early epidural analgesia
2110. Blood in urine in a patient in labour, diagnosis is - 2120. DID of acute abdomen in pregnancy are all
a) Impending scar rupture b) Urethral injury except- (PGINov.10)
c) Obstructed labour d) Cystitis a) Cystitis b) Threatenedabortion
2111. Minimum duration between onset of symptoms and c) Cervical incompetence d) Appendictis
death is seen in- _..(AI 09) e) Ruptured ectopic
a) APH b)PPH 2121. Vaginal delivery can be allowed in all except-
c) Septicemia d) Obstructed labor a) Monochrionic, monoamniotic twins
2112. Macrosomia is/are associated with- (PGI Nov 09) b) Mentoanterior (AIIMS May 11)
a) Gestational diabetes mellitus c) Extended breech
b) Maternal obesity d) Dichorionic twins with first vertex & second breech
c) Hypothyroidism 2122. In a neonate the skin ofsoles and palms is showing
d) Hyperbilirubenemia yellow discoloration, approximate level of serum
e) Foetal goitre bilirubin (mgldl) will be- (Karn II)
2113. Weight gain in pregnancy depends on all of the a) 5-7 b) 10
following factors, except - (AI 10) c) 12 d)>l5
a) Socioeconomic status b) Prepregnancy weight 2123. A nulliparous woman presents with acute lower
c) Smoking d) Ethinicity
abdominal pain. She has a history of missed periods.
2114. A primipara presented at 10 weeks of gestation with The ultrasound examination shows an empty uterus.
lower abdominal pain. On examination there was The cervical movements are very tender. The vital
pallor and lower abdominal tenderness. What is the
signs are stable. How will you manage her-
likely possibility? (DPG 10)
a) Treat her as a case ofPelvic Inflammatory Disease
a) Ruptured ectopic b) Twisted ovarian cyst
b) Perform a laparotomy (UPSCII II)
c) Red degeneration d) Acute appendicitis
c) Admit her for observation
2115. Most common cause of rupture uterus?
d) Perform. a laparoscopy
a) Separation of previous caesarean scar
b) Internal version (Maharashtra 10)

2105)c 2106)c 2107)d 2108)c 2109)b 2110)c 2111)b 2112)a,b 2113) c2114)a 2115)a 2116)d 2117)c 2118)b,d
2119)c,d;e 2120)c 212l)a 2122)d 2123)"d
GYNAE & OBS [ 101]

Recent Years NEETIDNB Questions 2136. Not transmitted congenitally- (NEETIDNB Pattern)
a) HSV b)HIV
2124. MC ovarian tumors originate from- c) Candida d) Rubella
a) Mullerian epithelium (NEETIDNB Pattern) 2137. MC cause of neonatal congenital infection-
b) Stroma a) CMV (NEETIDNB Pattern)
c) Germ cells b) Rubella
d) Connective tissue c) Toxoplasmosis
2125. OC pills decrease the risk of-(NEETIDNB Pattern) d)HSV
a) Stroke b) CVD 2138. Caesarean section is recommended for-
c) Endometrial ca d) Hepatic adenoma a) Rubella infected mother (NEETIDNB Pattern)
b) HSV infected mother
2126. HRT improves- (NEETIDNB Pattern)
c) CMV infected mother
a) Bone density b) Coronary artery disease
d) Measles infected mother
c) Dementia d) Endometrial cancer
2139. MC congenital malformation ofDM in neonates-
2127. OC pills must be started on -(NEETIDNB Pattern) a) ,NTDs (NEETIDNB Pattern)
a) 5th day b) 3rd day b) Hydrocephalous
c) 1st day d) When menses cease c) Sacral agenesis
2128. HIV positive couple with pregnant female, d) Multicystic kidneys
contraceptive of choice is - (NEET/DNB Pattern) 2140. A women is diagnosed with cervical carcinoma
a) Abstinence b) Condoms stage 2b, treatment of choice is -
c) No restrictions d) OCPs a) Hysterectomy (NEETIDNB Pattern)
2129. Exclusive complication of monochorionic twins- b) Chemoradiation
a) Cord entanglement (NEETIDNB Pattern) c) Primary radiation
b) Twin to twin transfusion d) None of the above
c) Discordant growth 2141. Management of neglected shoulder presentation
d) Abortion is- (NEETIDNB Pattern)
2130. 1st sign of IUD - (NEETIDNB Pattern) a) Vaginal delivery b) Caesarean section
a) Spalding sign c) External version d) Internal version
b) Air in heart 2142. The diagnosis of uterine anomalies is-
c) Hyper flexion of spine a) Hysterosalpingography (NEETIDNB Pattern)
d) Egg shell cracking feel of the fetal head b)MRI
2131. Fetal cells can be detected in maternal blood using- c) CT Scan
a) Dct (NEETIDNB Pattern) d) Plain radiography
b) let 2143. Most common cause of pyometra-
a) Endometritis (NEETIDNB Pattern)
c) Kleihauer betke test
b) CA endometrium,
b) Bubble test
c) Cacervix
2132. Fetal exsanguination can occur if-
d) Radiation
a) Vasa previa (NEETIDNB Pattern) 2144. Fetal trunk movements in third trimester is-
b) Placenta praevia a) Less perceived by nullipara
c) Polyhydramnios b) Increased in IUGR (NEETIDNB Pattern)
d) Oligohydramnios c) Are more pronounced
2133. 34 wks pregnancy, polyhydramnios is present when d) None of the above
volume is greater when - (NEETIDNB Pattern) 2145. PPH treatment of choice is -(NEETIDNB Pattern)
a) 2000cc b) 1500cc a) PGE2 b) PGEl
c) lOOOcc d) 500cc c) PGF2 alpha d) Oxytocin
2134. Modified BPP includes- (NEETIDNB Pattern) 2146. Fetal wt at 20 wks- (NEETIDNB Pattern}
a) Non stress test (NST) a) 150 gms b) 200 gms
b) Amniotic fluid index (AFI) c) 300 gms d) 400 gms
c) Both 2147. Strawberry cervix.is found in-(NEETIDNB Pattern)
d) None a) Trichomoniasis b) Candidiasis
2135. Olighydramnios, causes include- c) Bacterial vaginosis d) Gonorrhoea
a) DM (NEETIDNB Pattern) 2148. Spiegelberg criteria is for- (NEETIDNB Pattern)
b) Esophagal atresia a) Cervix b) Fallopian Tubes
c) Rh isoimmunisation c) Ovaries d) Uterus
d) Renal agenesis

2124)a 2125)c 2126)a 2127)c 2128)b 2129)b 2130)b 213l)c 2132)a 2133)a 2134)c 2135)d 2136)c 2137)a
2138)b 2139)a 2140)b 214l)b 2142)a 2143)a 2144)c 2145)c 2146)c 2147)a 2148)c
GYNAE & OBS [ 102]

2149. Largest skull diameter is - 2160. HPV infects first which cells in cervix-
a) Mento vertical (NEETIDNB Pattern) a) Basal and parabasal cells(NEETIDNB Pattern)
b) Occipito - frontal b) Corneal cells
c) Suboccipito - frontal c) Granulosa cell
d) Suboccipito - bregmatic d) Spinus cells
2150. Ideal contraceptive in RHD -(NEETIDNB Pattern) 2161. Clue cells are - (NEETIDNB Pattern)
a) Barrier b) IUCD a) Epithelial cells b) Lymphocytes
c) OCPs d) DMPA c) Neutrophils d) Macrophages
2151. Recent biomarker for earliy detection of surface 2162. Which is distinguishing feature of complete mole
epithelial tumours of Q)Vary -(NEET/DNB Pattern) · from partial mole- (NEETIDNB Pattern)
a) Ca 125, b) Ca 19~9, · a) Hydropic changes in proliferating villi
c) Ca 15-5, d) Osteopontin b) Proliferation of blood vessels into villi
2152. Bartholin's gland.s nie iliW ll'el~tllililll:il to vaginal omice-- c) Mutation of p-57 gene in chorionic cells
a) Posterior (NEET/DNB Pattern) d) Presence of fetus
b) Anterior &162L Which is associated with llcea§t cballllce!il ®f ectopic
c) Anterolateral p!regnancy - (NEETIDNB Pattern)
d) Posterolateral a) Tubectomy b) IUCD
2:2.§3. 43 yr~ 7 day p®st !liy®tell"ectomy fever9 burning c) Oral contraceptive pills d) Tubal ligation
micfurition with dribbfifi.u!lg of urine but abie to pas!J U64l, Ni!ii a ft1:ata~~ oflHf£8:LILJ!l> syndrome-
urine voiulliltarUy- (NEET/DNB Pattern) a) Hemolysis (NEETIDNB Pattern)
a) VVF b)UVF . b) Elevated liver enzymes
c) Urge incontinence d) Stress incontinence c) Low platelet count
2!54. Most common symptom in ectopic pregnancy- d) Renal failure
a) Abdominal pain (NEETIDNB Pattern) 2ll.65. Most senuntiv® to dilated Jllll'elllahllliU:'JUlr!!l~ mbe defect
b) Bleeding per vagina a) AFP (NEETIDNB Pattern)
c) Amenorrhoea b) Sphingosine
d) Fainting attacks c) Human gonadotropin
2155. Cervical cytology smear reveal CIN2- next s:tep- d)USG
a) Colposcopy (NEETIDNB Pattern) 2166. In FIGO staging of cancer cervix investigation
b) Cryocautery includes all except - (NEETIDNB Pattern)
c) Hysterectomy a) CT scan b) PET scan
d) Laser ablation c) MRI d) Laparoscopy
2156. ln_wbat condition does vulvectomy bas to be performed 2167. What is the contraindication of laparoscopic
sometimes - (NEETIDNB Pattern) sterilization - (NEETIDNB Pattern)
a) Granuloma inguinale a) Post partiun state
b) Post MTP
b) Chlamydia trachomitis
c) Gynaecologic malignancies
c) Herpessimplex
d) 3 previous child birth
d) Candidial infection
l16ill. What distinguishes placental site trophoblastic
2157. First radiological-evidence Gllfiotrauterine death of
tumor from Choriocarcinoma -
fetus - (NEET/DNB Pattern) a) Low HCG (NEETIDNB Pattern)
a) Spalding sign b) Low HCG and low HPL
b) Air in heart c) High HCG and low HPL
·c) Hyper flexion of spine d) High HCG and high HPL
d) Crowding of the ribs shadow 2169. Palm leaf appearance of cervicaL mucns is caused
2158. Which test is used to determine presence offetal by- (NEETIDNB Pattern)
cells in maternal circulation - a) Oestrogen b) Progesterone
a) Apt test (NEETIDNB Pattern) c) FSH d) Testosterone
b) Kleihauer betke test 2170. Rate limiting dose of radiotherapy to rectum and
c) Indirect coombs test bladder in gynaecologic malignancies -
c) Bubble test a) 4000cGy (NEETIDNB Pattern)
2159. Vasa previa may lead to- (NEETIDNB Pattern) b) 5000cGy
a) Rupture of membranes b) Fetal exanguination c) 6000cGy
c) Fetal death. d) All of the above d) 7000cGy

2149)a 2150)a 215l)a 2152)d 2153)b 2154)a 215S)a 2156)a 2157)b 2158)b 2159)d 2160)a 216l)a 2162)a
2163)c 2164)d 2165)a 2166)d 2167)a 2168)a 2169)a 2170)b
GYNAE & OBS [ 103]

2171. Woman presenting with chlymadial vaginal 2184. Cause of fibroid torsion- (NEET/DNB Pattern)
discharge needs- (NEET/DNB Pattern) a) Infection b) Site of origin
a) Ciprofloxacin b) Doxycyclin c) Both d) None
c) Metronidazole d) Azithromycin 2185. Cause of infertility in hypothyroidism-
2172. Fetal blood loss is more in -(NEET/DNB Pattern) a) Decrease prolactin (NEETIDNB Pattern)
a) vasa previa b) Increased prolactin
b) placenta previa c) Both
c) placental abruption d) None
d) circumvallate placenta 2186. First sign ®fwou.nd dehiscence in uterine rupture
2173. Monochorionic monoammotic placenta deveiop!l if during pregnancy- (NEETIDNB Pattern)
division takes palce ~ (NEETIDNB Pattern) a) Tachycardia b) PV discharge
a) Before 72 hrs b) Between 4th & 8th day c) Bloody micturition d) Bradycardia
c) After 8th day d) After 2 weeks 2187. Gestational sac on usg is first seen at __ wks
2174. Oligohydraminos is associated with all except- from LMP- (NEETIDNB Pattern)
a) Sacral agenesis (NEETIDNB Pattern) a) 2 b) 4
b) Polycystic kidney c) 5 d) 6
c) Renal agenesis 2188. 51 yr f with abdominal mass & ascites. On HIP
d) PROM ovarian Ca is +ve for - (NEETIDNB Pattern)
2175. HCG is secreted by- (NEET/DNB Pattern) a) Ca 125 b) Ca 19-9
a) Trophoblast cells b) Amniotic membrane c) AFP d)hCG
c) Fetal yolk sac d) Hypothalamus 2189. Laproscopic tubal ligation contraindication-
2176. Size of uterus in inches .is -(NEETIDNB Pattern) a) Postpartum state (NEETIDNB Pattern)
a) 5X4X2 b) 4X3Xl b) Post MTP
c) 3X2Xl d)4X2Xl c) Gynaecologic mahgnancies
2177. The amount of blood loss in each menstrual cycle d) 3 previous child birth
is about - (NEETIDNB Pattern) 2190. Therapeutic level of serum magnesium needed to
a) lOcc b) 35cc treat pre-ecle.mpsia - (NEETIDNB Pattern)
c) 50cc d) 1OOcc a) 1-2 Meq I 1 b) 3-4 Meq I 1
2178. Beading is seen in case of genital TB in women c) 4-7 Meq I 1 d) .7-9 Meql 1
in- (NEET/DNB Pattern) 2191. Estrogen Level in follicular phase of Menstrual
a) Tubes b) Ovary Cycle- (NEETIDNB Pattern)
c) Cervix d) Vagina a) 100-200 pg/ml b) 200-300 pg/ml
2179. Commonest cause of postpartum hemorrhage in c) 300-400 pg/ml d) 400-500 pg/ml
multipara is - (NEETIDNB Pattern) 2192. An 18 yrs young female presents with primary
a) Fibroid b) Retained placenta amenorrhea, cyclic abdominal pain and voiding
c) Uterine atony d) Uterine perforation difficulties. On examination breast and pubic hair
2180. In puerperial period sepsis is M/C due to - are well developed but vagina can't be appreciated.
a) Uterine infection · (NEETIDNB Pattern) Diagnosis is - (NEETIDNB Pattern)
b) Ovarian infection a) Mayer-Rokitansky-Kauster Syndrome
c) Vaginal infection b) Turner syndrome
d) All of the above c) Noonan sundrome
2181. Pt witb low testosterone & low count will show can d) Kallmann syndrome
show all exept- (NEETIDNB Pattern) 2193. Methotrexate is used in Ectopic Pregnancy when-
a) Decreased FSH a) P~tient is Hemodynamically Stable (NEETIDNB
b) Decreased LH b) Serum hCG level >3000 IU/L Pattern)
c) Increased FSH c) Tubal diameter > 4 em without fetal cardiac activity
d) Obstruction of spermatic duct d) None of the above
2182. Beaded feel of fallopian tube on hsg is seen in - 2194. Call Exner bodies are found in-
a) TB (NEETIDNB Pattern) a) Granulosa Cell Tumour (NEET/DNB Pattern)
b) Chlamydia b) Embryonal cell carcinoma
c) Gonococcal infection c) Choriocarcinoma
d) Syphillis d) Androblastoma
2183. At 9 wk best measure to calculate the gestational 2195. Amount ofblood passed through placenta on delayed
age - (NEETIDNB Pattern) cord clamping- (NEET/DNB Pattern)
~BPD ~CRL a) 50-lOOml b) 100-200ml
c) Fetal femer length d) Embryonic movements c) 120-150ml d) 150-180ml

2171)d 2172)a 2173)c 2174)a 2175)a 2176)b 2177)b 2178)a 2179)c 2180)a 218l)d 2182)a 2183)b 2184)b
2185)b 2186)c 2187)c 2188)a 2189)a 2190)c 2191)a 2192)a 2193)a 2194)a 2195)a
GYNAE & OBS [ 104]

2196. Test to detect maternal sensitization- 2209. Pregnancy changes are all except-
a) Direct coombs test (NEETIDNB Pattern) a) Respiratory alkalosis (NEETIDNB Pattern)
b) Indirect coombs test b) Paco2 decreased
c) Both c) Pa o2 increased
d) None d) Systemic vascular resistance increases
2197. Fetal lung maturity is signified by- 2210. G2 female in first trimester With first child 10yrs
a) L:S > 2 (NEET/DNB Pattern) of age getting chicken pox. Next step is-
b) >37 weeks gestation a) Varicella vaccine (NEETIDNB Pattern)
c) level of phosphatidyl choline b) Oral acyclovir
d) non reactive NST c) Both
2198. Amniotic fluid quantity at birth (ml)- d) None
a) 500 b) 1000 (NEETIDNB Pattern) 2211. Non-contraceptive metabolic effect of estrogen are
all except - (NEETIDNB Pattern)
c) 1500 . d) 2000
a) Increased fatty ·acids
2199. 1996 · ASRM classification of endometritis
b) Increased plasma lipid and lipoproteins
· includes - (NEET/DNB Pattern)
c) Increased total cholesterol
a) Intensity of pain b) Location
d) Decreased HDL
c) Size d) Number
2212. Commonest cause of anovulatory infertility-
2200. Hemodynamic change in pregnancy are all except-
a) PCOS (NEETIDNB Pattern)
a) Increased systemic vascular resistance
b)TB
b) Increased blood volume (NEETIDNB Pattern)
c) Endometriosis
c) Plasma volume
d) Thyroid dysfunction
d) Cardiac output
2213. Sarcoma botyroides in vagina is seen in which
2201. Heart disease in pregnancy, most common -
age- (NEET/DNB Pattern)
a) ASD b) MS (NEETIDNB Pattern)
a) Less than 8yrs b) 8-16 yrs
c) VSD d) MR
c) 16-24 yrs d)> 24 yrs
2202. Highest mortality in pregnancy - ·
2214. Interspinous diameter- (NEETIDNB Pattern)
a) Eisenmenger syndrome (NEETIDNB Pattern)
a) 10.5 em b) 11 em
b) Fallot's tetrology
c) 11.5 em d) 12 em
c) Coarctation of aorta
2215. 35 weeks pregnancy, painless blood discharge, most
d) Marfan's syndrome
likely diagnosis is - (NEETIDNB Pattern)
2203. Uretheral discharge is seen in-(NEETIDNB Pattern)
a) Placenta previa b) Abruptio placenta
a) Gonorrhea b) Chylamydia
c) Ectopic pregnancy d) None
c) Herpes d) Candida
2216. Curdy vaginal discharge is seen in which infection-
2204. Jacquemier sign is - (NEETIDNB Pattern)
a) Bacterial vaginosis (NEETIDNB Pattern)
a) Softening of cervix
b) Candidiasis
b) Bluish discoloration of anterior vaginal wall
c) Trichomoniasis
c) Mucous discharge
d) Chlamydia
d) Increased pulsations
2217. In pregnancy which is the most common test for
2205. In A-P, oval pelvis is- (NEETIDNB Pattern)
HIV- (NEETIDNB Pattern)
a) Android b) Platypoid
a) PCR b) CD4 cell count
c) Anthropoid d) Gynecoid
c) FllSA d) Western blot
2206. HIV maximum transmission occurs during -
2218. VVF in obstructed labour is repaired -(NEETIDNB
a) Child birth (NEET/DNB Pattern)
a) Immediately b) 3 weeks Pattern)
b) Breast feeding
c) 3 months d) 6 months
c) labour
2219. In total abdominal hysterectomy following are
d) Antepartum
removed- (NEETIDNB Pattern)
2207. Not a laproscopy instrument-(NEET/DNB Pattern)
a) uterus
a) Trocar b) Pneumoperitoneum needle
b) uterus & cervix
c) Doyen's retractor d) Fiberoptic camera
c) uterus , cervix & fallopian tube
2208. Ovarian cancer drugs used are -
d) uterus, cervix, fallopian tube & ovary
a) Paclitaxel + doxorubicin (NEETIDNB Pattern)
2220. GnRH analogue are all except -(NEETIDNB Pattern)
b) Cisplatin + doxorubicin
a) Soserelin b) Gonadorlein
c) Docetaxel + doxorubicin
c) Goserelin d) Buserelin
. d) Paclitaxel + cisplatin

2196)b 2197)a 2198)b 2199)b 2200)a 2201)b 2202)a 2203)a 2204)b 2205)c 2206)a 2207)c 2208)d 2209)d
2210)b 22ll)d 2212)a 2213)a 2214)a 2215)a 2216)b 2217)b 2218)c 2219)b 2220)a
GYNAE & OBS [ 105]

2221. CervicalcarcinomaisduetoHPV-(NEET/DNBPattem) 2235. FIGO staging with cervical carcinoma involving


a) 6, 8 b) 16, 18 parametrium- (NEETIDNB Pattern)
c)30,32 d)'l5,17 a) ITa b) lib
2222. Anemia in pregnancy is most commonly due to - c) rna d) Illb
a) Nutritional (NEETIDNB Pattern) 2236. Role of lactobacilli in vaginal secretions -
b) Hemorrhage a) To maintain alkaline pH (NEETIDNB Pattern)
c) Hereditary b) To maintain acidic pH
d) Bone marrow insufficiency c) Nutrition
2223. Procedure done in vault prolaps - d) None
a) Manchester operation (NEETIDNB Pattern) 2237. Most common degeneration offibroids-
b) Fothergills operation a) Calcareous b) Hyaline (NEETIDNB Pattern)
c) Khanna's abdominal sling operation c) Red d) Cystic
d) Le forte operation 2238. Definitive management of adenomyosis is
2224. Biophysical protlle includes all except - a) GNRH analogue (NEETIDNB Pattern)
a) NST (NEETIDNB Pattern) b) Danazole
b) Muscle tone c) IH
c) Amniotic fluid d) Hysterectomy
d) Acetyl choline level 2239. Drug commonly used in t/t of endometriosis -
2225. PID is most commonly caused by-(NEETIDNB Pattenl) a) IH b) GnRH (NEETIDNB Pattern)
a) Chlamydia b) Mycoplasma c) MPA d)FSH
c) Tubercular bacillus d) E. Coli 2240. Duration of contraception after DMPA is-
2226. Investigation for postmenopausal bleed are all a) 21 months b) 1-3 months (NEETIDNB
except- (NEETIDNB Pattern) c) 3-6 months d) >6 moriths Pattern)
a) PAP smear b) Laproscopy 2241. Methergin is given for prophylaxis of-
c) Fractional curettage d) Ultrasound
a) Anemia (NEETIDNB Pattern)
2227. H. mole karyotype is- (NEETIDNB Pattern)
b) Cardiac disease
a) 46XY b) 46:XX
c) Renal disease
c) 45XX d)45XY
d) Lung disease
2228. Staging of ca cervix with endometrial involvement-
2242. Investigation of choice in post menopausal
a) Stage I b) Stage 2 (NEETIDNB
bleeding- (NEETIDNB Pattern)
c) Stage 3 d) Stage 4 Pattern)
a) PAP smear b) Laproscopy
2229. Staging of endometrial ca with involvement of
paraaortic lymph node involvement- c) Fractional curettage d) Ultrasound
a) IllC b) liB (NEETIDNB Pattern) 2243. Gold standard diagnostic technique for diagnosis
c) lliA d) IIIB of endometriosis - (NEETIDNB Pattern)
2230. Ovarian carcinoma chemotherapy for a) Laproscopy b) Ca 125 level
epithelial- (NEETIDNB Pattern) c) Ultrasound d) MRI
a) BFC b) CHOP 2244. Markedly deflexed head of baby causes which
c) MOPP d) None of the above diameter to engage- (NEETIDNB Pattern)
2231. Amiotic fluid at 36-38 weeks -(NEETIDNB Pattern) a) Occipitofrontal b) Suboccipitorontal
a) 500 b)1000 c) Mentovertical d) Submentovertical
c) 1500 d) 2000 2245. Hegar sign all are true except -
2232. Fallopian tube tuberculosis- a) Bimanual palpation method
a) Most common type of genital TB b) Difficult in obese (NEETIDNB Pattern)
b) Size of the tubes is unchanged c) Can be done at 14 weeks
c) Is asymptomatic (NEETIDNB Patteni) d) Present .in 2/3rd of cases
d) Primary focus of infection is always in fallopian 2246. All of the following occurs because of prostaglandin
tubes use except - (NEETIDNB Pattern)
2233. Least diameter of gynecoid pelvis is - a) Excess water retention
a) Transverse (NEETIDNB Pattern) b) Flushes
b) Oblique c) Increased motility of bowel
c) Diagonal conjugate d) Nausea
d) Obstetric conjugate 2247. HRT is given in - (NEETIDNB Pattern)
2234. FIGO staging in endometrial Ca with paraaortic a) Symptomatic postmenopausal women
lymph node - (NEETIDNB Pattern) b) Following hysterectomy
a)IIIb b)lllc · c) Because a women has asked for it
c) IVa d) IV b d) All of the above

222l)b 2222)a 2223)d 2224)d 2225)a 2226)c 2227)b 2228)b 2229)a 2230)a 223l)b 2232)a 2233)d 2234)b
2235)b 2236)b 2237)b 2238)d 2239)b 2240)c 224l)a 2242)c 2243)a 2244)a 2245)c 2246)a 2247)d
GYNAE & OBS [ 106]

2248. New born can be given breast milk after how much 2259. Deep transverse arrest is seen in- (NEETIDNB
time following normal delivery-(NEET/DNB Pattern) a) Android pelvis b) Gynaecoid Pattern)
a) Half hour b) I hours c) Anthropoid d) Platypoid
c) 2 hours d) 3 hours 2260. Ca ovary with left supraclavicular LN, stage is-
2249. Ectopic pregnancy highest likely cause- a) N . b) III (NEET/DNB Pattern)
a) IDCD (NEET/DNB Pattern) c) IT d) I
b)PID 2261. Asherman syndrome false is-(NEET/DNB Pattern)
c) Artificial fertility technique a) Associated with menstrual irregularities
d) Tubal damage b) Progesterone challenge test is negative
2250. Bartholin cyst treatment of choice - c) Synechiae formation in uterus .
a) Excision (NEET/DNB Pattern) d) May be secondary to TB
b) Antibiotics 2262. Rarest presentation is w (NEETIDNB Pattern)
c) Marsupialization a) Cephalic b) Breech
d) Drainage c) Shoulder d) Vertex
2251. Prolapsed of uterus in nulliparous women, treatment 2263. Drug of choice for pneumocystis carmn m
is - (NEETIDNB Pattern) pregnancy- (NEETIDNB Pattern)
a) Sling used involving rectus sheath a) SMZ!TMP b) Primaquine
b) Anterior colporrhaphy c) Dapsone d) Pentamidine
c) posterior colporrhaphy 2264. · Emergency contraceptive should must be started
d) Manchester operation with in how much time after unprotected intercourse
2252. A pregnant woman in first trimester has four fold a) 24 hrs b) 48 hrs (NEET/DNB Pattern)
rise in IgG against toxoplasmosis. it indicates - c) 72 hrs d) 96 hrs
a) Protective antibodies (NEETIDNB Pattern)
2265. contraction stress test false is -
b) Acute infection
a) Oxytocin not used (NEETIDNB Pattern)
c) Chronic infection
b) Invasive method
d) None of the above
2253. Women with menorrhagia IUCD of choice- · c) Detects fetal well being
a) NOVA T (NEET/DNB Pattern) d) Negative test is associated with good fetal outcome
b) LNG IUD 2266. Maximum chance of transmission during delivery-
c) Mirena a) HSV b) CMV (NEETIDNB Pattern)
d) Gyne fix c) VZN d) Rubella
2254. In pregnancy cardiac output increases by what %- 2267. External version is done after -(NEETIDNB Pattern)
a) 40-50% (NEETIDNB Pattern) a) 34 weeks b) 36 weeks
b) 30-40% c) 38 weeks d) 40 weeks
c) 20-30% 2268. 35 year old with history of repeated D&C. She now
d) 10-20% has secondary ammenorhea. What is your diagnosisw
2255. In pregnancy plasma volume increased maximum a) Hypothyroidism (NEET/DNB Pattern)
at what gestational age? (NEET/DNB Pattern) b) Kallman syndrome
a) 10 wks b) 20 wks c) Sheehan's syndrome
c) 25 wks d) 30 wks d) Ashennan's syndrome
2256. Spinnaker is maximum shown at which phase - 2269. Drug causing abruption placenta-(NEETIDNB Pattern)
a) Menstrual phase (NEET/DNB Pattern) a) Methadone b) Cocaine
b) Owlatory c) Amphetamine d) Fluoxetine
c) Post ovulatory 2270. Drug of choice in premenstrual syndrome -
d) Pre follicular a) Antipsychotics b) SSRI (NEET/DNB Pattern)
2257. Most common puerperal infection is because of- c) OCP d) Depo progesterone
a) Anaerobic streptococcus (NEETIDNB Pattern)
2271. Fertile period of female is measured by -
b) Staphylococcus ·
a) lH b) FSH (NEET/DNB Pattern)
c) E. Coli
c) Estrogen d) Oxytocin
d) Klebsiella
2258. Most of ectopic pregnancies are at ampulla as - 2272. Antihypertensive of choice in pregnancy is -
a) It is the narrowest part (NEET/DNB Pattern) a) Methydopa (NEETIDNB Pattern)
b) Tubal movements are least here b) Labetolol
c) Salpingitis produces least crypts here c) Hydralizine
. d) Plicae are most numerous here d) ern

2248)a 2249)b 2250)c 2251)a 2252)b 2253)c 2254)a 2255)d 2256)b 2257)a 2258)d 2259)a 2260)a 226l)b
2262)c 2263)a 2264)c 2265)a 2266)a 2267)b 2268)d 2269)b 2270)b 227l)a 2272)a
GYNAE & OBS [ 107]

2273. I pill is used when - (NEETIDNB Pattern) 2286. Presenting part in transverse lie- (NEETIDNB
a) Accidental sexual exposure a) Shoulder b) Face Pattern)
b) OCP forgotten c) Vertex d) Brow
c) Of choice in young 2287~ Definitive indication ofLSCS -(NEETIDNB Pattern)

d) All of the above a) Mento ant b) Persistent menta post


2274. 35 yr old with 4 months amenorrhea with increased c) Occipita posterior d) Vertex
2288. Normal weight of term placenta in gms -
FSH, decreased estrogen. What is the diagnosis-
a) 300 b) 500 (NEET!DNB Pattern)
a) Premature ovarian failure
c) 700 d) 1000
b) PCOD (NEETIDNB Pattern)
2289. Severely anemic pregnant patient in cardiac failure.
c) Pituitary failure Choice of transfusion - (NEETIDNB Pattern)
d) Hypothalamic failure a) Platelets b) Packed cells
2275. NST, what is seen except- (NEETIDNB Pattern) c) Whole blood d) Exchange transfusion
a) Variability b) Acceleration 2290. Hydops fetalis is due to - (NEETIDNB Pattern)
c) Time period d) Oxytocin a) Rh mismatch b) Hyperproteinemia
2276. Post menopausal HRT decreases incidences of which c) Placental hypoplesia d) All of the above
malignancy- (NEETIDNB Pattern) 2291. Caput succedaneum indicates that fetus was alive till-
a) Breast b) Colorectal a) Immediately after birth (NEETIDNB Pattern)
c) Ovarian d) Endometrium b) Till 2-3 days after birth
2277. Most common malignant ovarian tumor - c) 2-3 weeks after birth
a) Dysgerminoma (NEETIDNB Pattern) d) 2-3 months after birth
b) Germ cell tumour 2292. CVS change in pregnancy- (NEETIDNB Pattern)
c) Serous cystadenocarcinoma a) Slight right axis deviation in ECG
d) Yolk sac tuomour b) Slight left axis deviation in ECG
2278. Least common presentation of twins - c) Diastolic murmur
a) Both vertex (NEETIDNB Pattern) d) Pulse rate is decreased
b) Both breech 2293. 5 month pregnant female, which ofthe following is
c) Both transverse true - (NEETIDNB Pattern)
a) 50% have soft systolic murmur
d) First vertex and 2nd transverse
b) Cardiac output is reduced
2279. Which is not feature of HELLP syndrome -
c) Systemic vascular resistance is increased
a) Hemolysis (NEETIDNB Pattern)
d) Increase in CVP
b) Elevated liver enzymes
2294. Hydronephrosis is seen in which stage of Ca cervix-
c) Low platelet count
a) 2a b) 2b (NEETIDNB Pattern)
d) Renal failure
c) 3a d) 3b
2280. Menstrual regulation effective upto- (NEETIDNB
2295. Endometrial carcinoma involving cervix, stage is-
a) 14 days b) 21 days Pattern)
a) 1 b) 2 (NEETIDNB Pattern)
c) 4 weeks d) 6 weeks
c) 3 d) 4
2281. Best test to quantify fetomaternal hemorrhage in
2296. Maximum cardiac output in pregnancy is seen at-
Rh isoimmunisation is - (NEETIDNB Pattern) a) 28 weeks b) 32 weeks (NEETIDNB Pattern)
a) UPT b)DCT c) 36 weeks d) 38 weeks
c) Kleinhauser test d) Indirect coombs 2297. 6year old son of pregnant women is suffering from
2282. Most common cause of salpingitis- (NEETIDNB chicken pox. Which of the following is given to
a) Chlamydia b) Mycoplasma Pattern) pregnant women- (NEETIDNB Pattern)
c) Tubercular bacillus d) E. Coli a) Acyclovir
2283. Jacquemeirs sign - (NEETIDNB Pattern) b) Acyclovir + immunoglobin
a) Softening of cervix c) Only immunoglobin
b) Bluish discoloration of anterior vaginal wall d) Vaccination
c) Mucous discharge 2298. MTP by medicine recognised by Govt of India
d) Increased pulsations upto- (NEETIDNB Pattern)
2284. Best to diagnose unruptured ectopic pregnancy - a) 70 days b) 63 days
a) Scopy b) UPT (NEETIDNB Pattern) c) 49 days d) 45 days
c) USG d) Culdocentesis 2299. All are causes of anovulatory amenorrhoea exept-
2285. Most common site of endometriosis - a) PCOD (NEETIDNB Pattern)
a) Ovary b) FT (NEETIDNB Pattern) b) Hyperprolactemia
c) Colon d) LSCS Scar c) Gondal dysgenesis
d) Drugs

2273)a 2274)a 2275)d 2276)b 2277)c 2278)c 2279)d 2280)a 2281)c 2282)a 2283)a 2284)a 2285)a 2286)a
2287)c 2288)b 2289)b ~290)a 2291)a 2292)a 2293)a 2294)d 2295)b 2296)b 2297)c 2298)b 2299)c
GYNAE & OBS [ 108]

2300. After delivery upto which week is known as puperium- 2315. What is false about post menopausal state-
a) 2 weeks b) 4 weeks (NEET/DNB Pattern) a) Low LH (NEETIDNB Pattern)
c) 6 weeks d) 8 weeks b) Low estrogen
2301. Vacuum delivery produces -(NEETIDNB Pattern) c) HighFSH
a) Chingon b) Cephalhematoma d) High androgen
c) Both d) None 2316. DOC for malaria in pregnancy-(NEET/DNB Pattern)
2302. External cephalic version is now a days done at - a) Chloroquin b) Quinine
a) 34 weeks b) 36 weeks (NEETIDNB Pattern)
c) Primaquin d) Artesunate
c) 38 weeks d) 40 weeks
2317. Exclusively Fetal blood loss occurs in-
2303. Antithyroid of choice in pregnancy-
a) Methimazole b) PTU (NEETIDNB Pattern) a) Vasa previa (NEET/DNB Pattern)
c) Carbamizole d) Beta blockers b) Placenta praevia
2304. Weight of fetus at 20 wks gestation - c) Polyhydramnios
a) 150 gms b) 200 gms (NEETIDNB Pattern) d) Oligohydramnios
c) 300 gms d) 400 gms 2318. For uterine prolapse in pregnanacy, Ring pessery
2305. Placenta praevia, false is - (NEETIDNB Pattern) can be inserted upto - (NEETIDNB Pattern)
a) Most common cause of APH a) 12 weeks b) 14 weeks
b) Painful vaginal bleeding c) 16weeks d) 18 weeks
c) Usg is the investigation of choice 2319. Least failure rate is of- (NEETIDNB Pattern)
d) Increased maternal age is a risk factor a) OC pills b) IUDs
2306. Most common cause of pyometra is- (NEETIDNB c) Condom d) DMPA
a) Endometritis b) CA endometrium Pattern) 2320. Mento vertical diameter of fetal skull is -
c) Ca cervix d) Radiation a) 9.5 em b) 10 em (NEETIDNB Pattern)
2307. PAP smear from cervix shows HSIL grade CIN, c) 11.5 em d) 14 em
next step is - (NEETIDNB Pattern) 2321. Enzymes responsible for conversion of androgen
a) Coagulation b) Conization into estrogen are all except -(NEETIDNB Pattern)
c) Colposcopy & biopsy d) Cryosurgery a) Aromatase
2308. pH of vagina in pregnant woman is usually- b) Sulphatase
a) 4.0 b) 4.5 (NEETIDNB Pattern) c) Fumarase
c) 5 d) >5 d) 3 beta hydroxy steroid dehydrogenase
2309. Diagnosis of adenomyosis is made by - 2322. Failure rate of Pomeroy's method of tubal ligation
a) Histopathology (NEETIDNB Pattern) is - (NEETIDNB Pattern)
b) Ultrasound a) 0.2% b) 0.4%
c) MRl c) 0.6% d) 0.8%
d) Laproscopy 2323. Maximum success after reversal of tubal ligation-
2310. Definitive treatment for severe preeclampsia is- a) Cauterization (NEETIDNB Pattern)
a) Mgso4 (NEETIDNB Pattern)
b) Pomeroy's technique
b) Delivery of the baby
c) Clip method
c) Antihypertensive drugs
d) Fimbriectomy
d) Rest
2324. Not a method for delivery of after-coming head of
2311. Treatment for young woman with prolapsed uterus
breech- (NEETIDNB Pattern)
is - (NEETIDNB Pattern)
a) Forceps method
a) Sling operation
b) Anterior colporrhaphy b) Bums and Marshall method
c) Posterior colporrhaphy c) Malar flexion and shoulder traction
d) Manchester operation d) Half hand method
2312. Decidual reaction is due to which hormone - 2325. In a postmenopausal female, which hormone
a) Progesterone b) Estrogen (NEETIDNB Pattern) increases - (NEET/DNB Pattern)
c) IH d) FSH a) FSH b) Estrogen
2313. Min dose of estrogen in OCPs in micrograms- c) GH d) None of the above
a) 10 b) 20 (NEETIDNB Pattern) 2326. Initial drug for ovarian cancer-(NEET/DNB Pattern)
c) 30 d) 40 a) Carboplatin b) Doxorubicin
2314. Prophylactic methergin given for- c) Ifosfamide d) Methotrexate
a) Induction of labour (NEETIDNB Pattern) 2327. If 300 microgram anti D is given to mother ,
b) Induction of abortion amount of blood it will neutralise - (NEETIDNB
c) To stop excess bleeding from uterus a) 30 ml b) 40 ml Pattern)
d) All of the above c) 50ml d) 60 ml

2300) c 2301) c 2302) b 2303) a 2304) c 2305) b 2306) a 2307) c 2308) d 2309) d 2310) b 2311) a 2312) a 2313) b
2314)c 2315)a 2316)a 2317)a 2318)d 2319)a 2320)d 2321)c 2322)b 2323)c 2324)d 2325)a 2326)a 2327)a
GYNAE & OBS [ 109]

2328. Which does not indicate fetal lung maturity- 2340. DOC for cholera in pregnancy is-(NEET/DNB Pattern)
a) Reactive NST (NEETIDNB Pattern) a) Furazolidone b) Tetracycline
b) Gestation 37 weeks c) Doxycycline d) Azithromycin
c) Presence of phosphatidyl choline 2341. GTT in pregnancy is indicated when if fasting
d) LIS ratio blood sugar is above- (NEETIDNB Pattern)
2329. Best test for ovulation- (NEETIDNB Pattern) a) 80 gm/100 rn1 b) 85 gm/100 rn1
a) Serum estrogen b) Serum progesterone c) 90 gm/100 rn1 d) 95 gm/100 rn1
c) Both d) None 2342. Treatment of stage rrm in Ca cervix is -
2330. Tdap vaccine is give in between which weeks of a) Wertheim's hystrectotomy (NEETIDNB Pattern)
pregnancy- (NEETIDNB Pattern) b) Mitra operation
a) 10-16weeks b) 17-22weeks c) Chemoradiation
c) 22-26 weeks d) 27-30 weeks d) Primary radiotherapy
2331. Which is least injured iu gynaecological procedures- 2343. Polyhydramnios- (NEETIDNB Pattern)
a) Ureter at pelvic brim (NEETIDNB Pattern) a) 2000cc b) 1500cc
b) Renal pelvis c) lOOOcc d) 500cc
c) Urinary bladder 2344. Most common cause ofAsherman-(NEET/DNB Pattern)
d) Ureter at infundibulopelvic ligament a) PPH b) Tubercular endometritis
2332. Which of the following cannot be treated by c) Amenorrhea d) Oligomenorrhea
laproscopy- (NEETIDNB Pattern) 2345. Early amniocentesis is done at-(NEETIDNB Pattern)
a) Ectopic pregnancy a) 6-12 weeks b) 12-15 weeks
b) Sterilization c) 15-20 weeks d) 20-25 weeks
c) Non descent of uterus 2346. Most common cause of vulval hematoma -
d) Genital prolapsed a) Episiotomy (NEETIDNB Pattern)
2333. Which offollowing most commonly clinically used- b) Vault rupture
a) Diagonal conjugate (NEETIDNB Pattern) c) Lower segment uterine rupture
b) Ant post diameter of inlet d) Rupture of paravaginal sinuses
c) Transverse diameter of outlet 2347. Which condition internal podalic version is done-
d) Oblique diameter of pelvis a) Transverse lie in 2nd twins
2334. Most suitable type of pelvis in female- b) Breech presentation (NEET!DNB Pattern)
a) Gynaecoid (NEETIDNB Pattern) c) Both
b) Android d) None
c) Anthropoid 2348. Least common type of twins -(NEETIDNB Pattern)
d) Platypelloid a) Diamniotic-dichorionic twins
2335. Active management of3rd stage oflabour is helpful b) Diamniotic -monochorionic twins
in prevention of- (NEETIDNB Pattern) c) Monoamniotic-monochorionic twins
a) Atonic PPH b) Secondary PPH d) Conjoined twins
c) Uterine inertia d) APH 2349. Caput succedenm is said to occur in baby -
2336. Palmer sign is related to - a) Within 24 hrs (NEETIDNB Pattern)
a) Contraction of uterus (NEETIDNB Pattern) b) 2-3 days
b) Dusky hue of ant vaginal wall c) 2-3 weeks
c) Bluish discolouration of ant vaginal wall d) 2-3 months
d) Increased pulsations felt through lateral fornix 2350. All of the following are used in hysteroscopy except-
2337. In a baby born to a diabetic mother all are seen a) C02 b) 0 2 (NEETIDNB Pattern)
except- (NEETIDNB Pattern) c) Normal saline d) Dextrose
a) Malformations b) Macrosomia 2351. OCP decreases risk of- (NEETIDNB Pattern)
c) Growth restriction d) Heart failure a) Cervical ca b) Endometrial ca
2338. Not a cause for secondary amenorrhea- c) Vaginal ca d) Liver carcinoma
a) Pregnancy (NEETIDNB Pattern) 2352. Progesterone only pill not given after -
b) PCOD a) 24 hrs b) 48 hrs (NEETIDNB Pattern)
c) Hyperprolactinaemia c) 72 hrs d) 96 hrs
d) Turner syndrome 2353. Regarding erythroblastosis fetalis all is true except-
2339. Spinbarket is maximum shown in which phase - a) Rh haemolytic disease (NEETIDNB Pattern)
a) Menstrual phase (NEETIDNB Pattern) b) Severe anemia
b) Ovulatory c) Hypoplasia of placental tissue
c) Post ovulatory d) Hypoproteinaemia
d) Pre follicular

2328)a 2329)b 2330)d 233l)b 2332)c 2333)a 2334)a 2335)a 2336)a 2337)d 2338)d 2339)b 2340)d 234l)d
2342)c 2343)a 2344)a 2345)c 2346)a 2347)a 2348)d 2349)a 2350)b 235l)b 2352)a 2353)c
GYNAE & OBS [ 110]

2354. Regarding HCG all are true except - 2366. Antihypertensive contraindicated in pregnancy -
a) It is a glycoprotein (NEETIDNB Pattern) a) Labetalol (NEETIDNB Pattern)
b) Chemically similar to LH b) Hydralazine
c) Secreted by syncytiotrophoblasts of placenta c) Methyl dopa
d) Inhibits adrenal steroidogenesis d) ACE inhibitors
2355. Vasectomy reversal in primary infertility is done 2367. Strongest support of uterus is-
within how many years of vasectomy- a) Mackenrodt's ligament (NEETIDNB Pattern)
a) 5yrs b) 6yrs (NEETIDNB Pattern) b) Broad ligaments
c) 7yrs d) 8yrs c) Round ligaments
2356. Hegar sign is seen in how many weeks- (NEETI
d) Uterosacral ligaments
a) 6-10 weeks b) 10-14 weeks DNB Pattern)
2368. Consumption coagulopathy is seen with -
c) 14-18 weeks d) 18-22 weeks
a) Abruption placenta (NEETIDNB Pattern)
2357. Implantation occurs on which day of menstrual
b) Placenta previa
cycle- (NEETIDNB Pattern)
a) 6th day b) lOth day c) Placenta accrete
c) 15th day d) 20th day d) Retained placenta
2358. Indication of classical caesarean section - 2369. For hormonal assessment, sample is taken from-
a) Cervical cancer (NEETIDNB Pattern) a) Anterior wall (NEETIDNB Pattern)
b) Contracted pelvis b) Lateral wall
c) Non re-assuring FHR c) Posterior wall
d) None d) Fornix
2359. Location of Bartholin cyst is -(NEETIDNB Pattern) 2370. Vaginal emphysematous bulla, which is false-
a) Junction of anterior 2/3rd and posterior l/3rd of a) Presents with vaginal discharge
labium majus b) Leads to ulceration (NEETIDNB Pattern)
b) Junction of anterior l/3rd and posterior 2/3rd of c) Usually occur in pregnant patients
labium majus d) May occur secondary to genital tract infections
c) Junction of anterior and posterior halves of 2371. Most common congenital defect is- (NEETIDNB
labium majus a) Cataract b) PDA Pattern)
d) None of the above c) ASD d) Phenylketonuria
2360. Vaginal pH before puberty is - 2372. Not a tocolytics - (NEETIDNB Pattern)
a) 7 b) 6 (NEETIDNB Pattern) a) Diazepam b) Mag sulphate
c) 4.5 d) 5 c) Indomethacin d) Terbutaline
2361. Carcinoma cervix screening are all except- 2373. Strawberry cervix is seen in-(NEETIDNB Pattern)
a) PAP smear (NEETIDNB Pattern) a) Trichomonas b) Chlamydia
b) Liquid based cytology c) Gonococcal infection d) Candida
c) CT and MRI 2374. Ballard score is used asses -(NEETIDNB Pattern)
d) Acetowhite areas a) Brain development of child
2362. TB uterus all is true except - b) Gestational age of child
a) Mostly secondary (NEETIDNB Pattern) c) Lung maturation of the child
b) Increase incidence of ectopic pregnancy d) Viability of the child
c) Involvement of endosalpinx 2375. After pregnancy, blood volume normalises after-
d) Most common is ascending infection a) 1 week b) 2 weeks (NEETIDNB
2363. Lactating mother OCP of choice - c) 3 weeks d) 4 weeks Pattern)
a) Progestin only pil (NEETIDNB Pattern) 2376. Quadruple test does not include- (NEETIDNB
b) Combined OCP a) MSAFP b) Total hCG Pattern)
c) Estrogen only pill c) PAPA d) Inhibin A
d) None 2377. Cervical cancer 4mm deep invasion and 7 mm
2364. Clotting factors change in pregnancy - spread, stage is- (NEETIDNB Pattern)
a) Fibrinogen level is increased a) IA b) IAl
b) Platelet level is increased(NEET/DNB Pattern) c) IA2 d) IB
c) Factor XII level is decreased 2378. Main hormone acting upon uterus to initiate labour-
d) Factor XI level is increased a) Oxytocin b) Estrogen (NEETIDNB
2365. Maximum amount of02 consumption increased in c) Progesterone d) Cortisole Pattern)
pregnancy is- (NEETIDNB Pattern) 2379. Viable fetus age limit is - (NEETIDNB Pattern)
a) 20% b)30% a) 120 days b) 140 days
c) 40% d) 50% c) 210 days d) 240 days

2354)d 2355)a 2356)a 2357)d 2358)a 2359)a 2360)a 2361)c 2362)d 2363)a 2364)a 2365)c 2366)d 2367)a
2368)a 2369)b 2370)b 237l)c 2372)a 2373)a 2374)b 2375)b 2376)c 2377)c 2378)a 2379)b
GYNAE & OBS [ 111]

2380. True about dysgerminoma - 2396. Side effect of progesterone only pill are all except-
a) Highly aggressive (NEETIDNB Pattern) a) Irregular bleeding (NEET/DNB Pattern)
b) Managed conservatively in young girls b) Amenorrhea
c) Usually seen in old patients c) Decreased lactation
d) Secrets make sex hormones d) Weight gain
2381. Least chance of cord prolapse is seen in- 2397. Depot DMP, all are true except_
a) Vertex b) Breech (NEET!DNB a) Dysmenorrhea (NEETIDNB Pattern)
c) Transverse d) Compound Pattern) b) Dyslipidemia
2382. Which ofthe following is considered normal in pregnancy-
a) Platelets count 80000 (NEET/DNB Pattern) c) Do not prevent STDs
b) WBC 12000 d) Can be given in breast cancer
c) Total protein lOOgm 2398. Hyperstimulation syndrome, true is-
d) Fibrinogen level150mg/dl a) Usually seen in late pregnancy (NEETIDNB
2383. Chicken pox in first trimester, treatment b) hCG is the treatment of choice Pattern)
is - (NEETIDNB Pattern) c) Raised LH is responsible in PCOD
a) VZlG b) Oral acyclovir d) Vascular permeability is decreased
c) Varicella vaccine d) All of the above 2399. AID is the term used to describe the artificial
2384. Obstetric cholestasis is characterized by - insemination achieved by using sperms of-
a) Manifests usually in 1st trimester a) Husband b) Donor (NEETIDNB
b) Most common cause of jaundice in pregnancy c) Both d) None Pattern)
c) Pruritus is the main symptom 2400. Sheehan's syndrome is- (NEETIDNB Pattern)
d) Recurrence rate is low (NEETIDNB Pattern) a) Pituitary necrosis b) Pituitary adenoma
2385. Drug of choice for ovulation inductin- c) Adrenal necrosis d) Adrenal adenoma
a) Clomiphene b) FSH (NEET/DNB Pattern) 2401. True for PCOD is- (NEETIDNB Pattern)
c) IH d) Hcg a) Increased FSH b) Increased LH
2386. Polyhydraminos in 34 weeks is defined as -
a) 800ml b) 1500 ml (NEETIDNB c) Increased SHBG d) Increased FSHILH ratio
c) 1000 m1 d) 2000 ml Pattern) 2402. A patient comes with 6 weeks amenorrhoea and
2387. Complete mole is_ (NEET/DNB Pattern) features of shock- (NEETIDNB Pattern)
a) 46XY b) 46:XX a) Ectopic pregnancy b) H. Mole
c) 45XY d) 45 XX c) Twin pregnancy d) None of the above
2388. Ca cervix lliB is treatment of choice is-(NEETIDNB 2403. Ca cervix staging done by -(NEETIDNB Pattern)
a) Radiotherapy b) Chemotherapy Pattern) a) CT b) MRI
c) Chemoradiation d) Surgery c) Clinical fmdings d) Histopathology
2389. Post menopausal endometrial thickness- 2404. Sheehan's syndrome is due to-
a) 1-3 mm b) 3-5 mm (NEET/DNB a) Ovarian necrosis (NEETIDNB Pattern)
c) 5-7mm d) 6-8 mm Pattern) b) Hypothalamus necrosis
2390. Torsion of ovary is most commonly seen c) Pituitary necrosis
in- (NEETIDNB Pattern) d) Thyroid necrosis
a) Dermoid cyst b) Granulosa cell tumour 2405. Most effective in detecting neural tube defect-
c) Dysgerminoma d) Yolk sac tumour a) AFP b) MRI (NEETIDNB Pattern)
2391. Pomeroy technique is for- (NEETIDNB Pattern) c) CT d) Ultrsound
a) Tubal ligation b) Laprscopy 2406. Recurrent abortion not due to -
c) Hysteroscopy d) Minilaparotomy a) Chromosomal defects (NEETIDNB Pattern)
2392. For gestational sac is seen by TVS at- b) TORCH infection
a) 5 weeks b) 6 weeks (NEETIDNB c) Luteal phase defects
c) 7 weeks d) 8 weeks Pattern) d) Poorly controlled diabetes
2393. Choriocarcinoma diagnosis is by- (NEETIDNB 2407. Partogram is not used to monitor-
a) USG b) CT Pattern) a) cervical dilatation (NEETIDNB Pattern)
c) MRI d) X-RAY b) uterine contractions
2394. Chemotherapy for choriocarcinoma, DOC c) descent of head
is - (NEETIDNB Pattern) d) fetal lung maturity
a) Methotrexate b) Cyclophosphamide 2408. AFP is increased in which ovarian tumor -
c) Cisplatin d) Doxorubicin a) Choriocarcinoma (NEET!DNB Pattern)
2395. Ina34-36weekpregnancy,whatisAmnioticfiuidindex- b) Granulose cell tumour
a) 15 b) 20 (NEETIDNB Pattern) c) Sarcoma
c) 25 d) 30 d) Endodermal sinus tumour
2380)b 2381)a 2382)b 2383)b 2384)c 2385)a 2386)d 2387)b 2388)c 2389)a 2390)a 2391)d 2392)a 2393)a
2394)a 2395)c 2396)c 2397)d 2398)c 2399)b 2400)a 2401)b 2402)a 2403)c 2404)c 2405)a 2406)b 2407)d
2408)d
GYNAE & OBS [ 112]

2409. Lichen sclerosis of vulva false is - 2424. OC pills decrease risk of- (NEETIDNB Pattern)
a) It is infective (NEETIDNB Pattern) a) Stroke b) CVD
b) Usually occurs in old age c) Endometrial ca d) Hepatic adenoma
c) May b associated with autoimmune diseases 2425. Induction/Trial of labour is contraindicated in -
d) Due to decreased estrogen a) Heart disease of Pregnancy
2410. Which antibiotic can be safely used in pregnant b) Hypertensive disease of pregnancy
women- (NEETIDNB Pattern) c) Abruption placenta (NEETIDNB Pattern)
a) Tetracycline b) Erythromycin d) IUGR
c) Isoniazid d) Chloremphenicol 2426. Oassical C section indicated in-(NEETIDNB Pattern)
2411. treatment of stage llB of cervix is - (NEETIDNB a) CA Cervix b) Central placenta praevia
a) Chemoradiation b) Hysterectomy Pattern) c) Failed induction d) Fetal distress
c) Radiation d) Chemotherapy 2427. Not given in pregnancy- (NEETIDNB Pattern)
2412. Diabetes mellitus in pregnancy is diagnosed by- a) ENALAPRIL b) Labetalol
a) Fasting blood glucose (NEETIDNB Pattern) c) Hydralazine d) Nifedipine
b) 50 mg oral GTT 2428. Minipill, side effect is - (NEETIDNB Pattern)
c) 100 mg oral GTT a) Break through Bleeding b) PID
d) 150 mg of oral GTT c) Endometrial CA d) Thromboembolism
2413. Implantation normally occurs in- (NEETIDNB 2429. Latschko's operation- (NEETIDNB Pattern)
a) Ampulla b) Body of uterus Pattern) a) VVF b) Prolapse
c) Cervix d) Ovaries c) Vesicouterine fistula d) Uretrll.rovaginal fistula
2414. Normal sperm motility normal according to WHO 2430. Regeneration of Endothelium after menstruation
is - (NEETIDNB Pattern) takes about- (NEETIDNB Pattern)
a)> 20% b)> 30% a) 2 days b) 5 days
c) >40% d)> 50% c) 10 days d) 15 days
2415. Transmission oflllV infection from infected mother 2431. Tocolytics can be given in -(NEETIDNB Pattern)
to child occurs most commonly during-(NEETIDNB a) Placenta Praevia b) Placenta accerta
a) Antenatal stage b) Before delivery Pattern) c) Pre term labour d) Eclampsia
c) During labour d) During lactation 2432. Therapeutic level of magnesium for treatment of
2416. Early amniocentesis is done at- (NEETIDNB pre-eclampsia should be- (NEETIDNB Pattern)
a) 5-10 weeks b) 10-15 weeks Pattern) a) 2-3 meq/1 b) 4-7.5 meq/1
c) 15-20 weeks d) 20-24 weeks c) 7.5-10meq/l d) 10-15 meq/1
2417. Not a support of uterus- (NEETIDNB Pattern) 2433. Max estrogen Level in Menstrual Cycle is-(NEETIDNB
a) Urogenital diaphragm a) 100-200pg/ml b) 200-300 pg/ml Pattern)
b) Pubocervicalligament c) 300-400 pg/ml d) 400-500 pg/ml
c) Perineal body 2434. Methotrexate is used in Ectopic Pregnancy when-
d) Levator ani muscle fibers a) Patient is Hemodynamically Stable
2418. Commonest cause of PPH is-(NEETIDNB Pattern) b) hCG level is >4000iu/ml (NEETIDNB Pattern)
a) Uterine atony b) Traumatic c) Tubal diameter <5 em
c) Retained tissues d) Blood coagulopathy d) When there is intrabdominal hemorrhage
2419. Radical hysterectomy is named after- 2435. Dye used in Hysterosalpingography-(NEET/DNB
a) Wertheim's b) John clark (NEETIDNB a) Conray 420 b) Renografm-60 Pattern)
c) Meigs d) Mitra Pattern) c) Toluidine blue d) None of the above
2420. Drug not used commonly for menorrhagia - 2436. Ergometrine is contraindicated in -
a) Methergin b) Clomiphene (NEETIDNB a) Third stage of labour with heart disease
c) Danazole d) NSAIDS Pattern) b) Third stage uterine bleeding
2421. Age group for dysgerminoma is - (NEETIDNB c) Both (NEETIDNB Pattern)
a) 10-20 yrs b) 30-40 yrs Pattern) d) None
c) 50-60 yrs d) >60 yrs 2437. mcc ectopic pregnancy- (NEETIDNB Pattern)
2422. What is seen in luteal phase - (NEET/DNB
a) IUCD b) PID
a) Increased progesterone levels Pattern) c) POP d) Tubal surgery
b) Decreased progesterone levels 2438. Which surgery uses rectus sheath to prevent
c) Decreased estrogen level
prolapsed - (NEETIDNB Pattern)
d) None of the above
a) Khanna operation b) Purandare operation
2423. Partial mole, not true is:-
c) Shirodkaroperation d) Le Forte repair
a) Fetus is present (NEETIDNB Pattern)
2439. DOC for eclampsia is - (NEETIDNB Pattern)
b) Diffuse trophoblastic hyperpiesia
a) Methyl dopa b) Labetalol
c) Theca lutein cysts uncommon
c) Mag. sulphate d) Hydralazine
d) Uterine size is less then date
***
2409)a 2410)b 24ll)a 2412)c 2413)b 2414)d 2415)b 2416)c 2417)d 2418)a 2419)a 2420)a 2421)a 2422)a
2423)b 2424)c 2425)a 2426)a 2427)a 2428)a 2429)a 2430)a 2431)c 2432)b 2433)d 2434)a 2435)b 2436)a
2437) b 2438) b 2439)c
su
SURGERY

WOUNDS, TISSUE REPAIR & SCARS 13. A patient with grossly contaminated wound presents
12 hours after an accident. His wound should be
1. Keloid scars is made up of- managed by - (UPSC 96)
a) Dense collagen b) Loose fibrous tissue a) Thorough cleaning and primary repair
c) Granulamatous tissue d) Loose areolar tissue b) Thorough cleaning with debridement of all dead
2. Primary closure of incised wounds must be done and devitalised tissue without primary closure
within •••••••••• - (KERALA 87) c) Primary closure over a drain
a) 2hrs b) 4hrs d) Covering the defect with split skin graft after
c) 6 hrs d) 12 hrs cleaning
e) 16hrs 14. Delayed wound healing is seen in aU except-(AP 96)
3. The tensile strength of wound reaches that of a) Malignancy b) Hypertension
normal tissue by- (PGI 88) c) Diabetes d) Infection
a) 6 weeks b) 2 months 15. The foUowing statement about keloid is true-
c) 4 months d) 6 months a) They do not extend into normal skin
4. The worst position for scars is- (PGI 88) b) Local recurrence is common after excision
a) Back b) Shoulder c) They often undergo malignant change
c) Sternum d) Abdomen d) They are more common in whites than in blacks
5. The best scars are seen in .......... - (PGI 88) 16. Fibroblast in healing wound derived from-
a) Infants b) Children a) Localmesenchyme b)Epithelium (PGI98)
c) Adults d) Very old people c) Endothelial d) Vascular fibrosis
6. Patient has lacerated untidy wound of the leg and 17. In the healing of a clean wound the maximum
attended the casualty after 2 hours. His wound immediate strength of the wound is reached by-
should be- (AIIMS 84) a) 2 - 3 days b) 4 - 7 days
a) Sutured immediately c) 10- 12 days d) 13- 18 days
b) Debrided and sutured immediately 18. Management of an open wound seen 12 hrs. after
c) Debrided and sutured secondarily the injury- (AIIMS 87)
d) Cleaned and dressed a) Suturing b) Debridement and suture
7. FoUowing are required for wound healing except- c) Secondary suturing d) Heal by granulation
a) Zinc b)Copper (AJ93) 19. Deglovinginjuryis- (KERALA2K)
c) Vitamin C d) Calcium a) Surgeon made wound b) Lacerated wound
8. Woundhealingisworstat- (Al93) c) Bluntinjury d)Avulsioninjury
a) Sternum b) Anterior neck e) Abrasive wound
c) Eyelid d) Lips 20. The best cure rate in keloids is achieved by-
9. When is the maximum coHagen content of wound a) Superficial X- ray therapy (UPSC 2001)
tissue- (PGI 81, ROHTAK 87) b) Intralesional injection of triamcinolone
a) Between 3rd to 5th day c) Shaving
b) Between 6th to 17th day d) Excision and radiotherapy
c) Between 17th to 21st day 21. A clean incised wound heals by- (Delhi 92)
d) None of the above a) Primary intention b) Secondary intention
10. If suture marks are to be avoided, skin sutures c) Excessive scarring d) None of the above
should be removed by- (JIPMER 81, AMC 89) 22. In treatment of hand injuries, the greatest priority
a) 72 hours b) 1 week is- (AI 96)
c) 2 weeks d) 3 weeks a) Repair of tendons
11. Whatistrueaboutkeloids- (JIPMER95) b) Restoration of skin cover
a) It appears immediately after surgery c) Repair of nerves
b) It appears a few days after surgery d) Repair of blood vessels
c) It is limited in its distribution 23. During the surgical procedure- (AIIMS 83)
d) It is common in old people a) Tendons should be repaired before nerves
12. Keloid is best treated by- (UPSC 95) b) Nerves should be repaired before tendons
a) Intrakeloidal injection of triamcinolone c) Tendons should not be repaired at the same time
b) Wide excision and grafting d) None is true
c) Wide excision and suturing 24. 'Limb salvage' primarily depends on- (AJIMS 97)
d) Deep X-ray therapy a) Vascular injury b) Skin cover
c) Bone injury d) Nerve injury

1) a 2)c 3)None 4)c 5)d 6)c ?)None 8)a 9)c lO)b ll)b 12)a 13)b 14)b 15)b
16)a 17)d 18)b 19)d 20)d 21)a 22)b 23)b 24)a
SURGERY [ 184]

25. In an open injury toileting and debridement, muscle c) Most common site is perineum followed by trunk
viability is detected by- (PGI 03) and extremities
a) Colour of the muscle b) Muscle size d) Surgical debridement is mandatory
c) Muscle function d) Muscle contractility 38. True about wound healing- (PGI June 09)
e) Punctate bleeding spots on cut edge a) Infected wound heal by primary intention]
26. All of the following favour postoperative wound b) Deep dermal wound heal by scar formation
dehiscence except- (Karnat 05) c) Wound contraction is found in healing by
a) Malignancy secondary intention
b) Vitamin B complex deficiency d) More intense nfamantory response in primary
c) Hypoproteinaemia intention
d) Jaundice 39. Which of the following is a scoring system for
27. The tensile strength of the wound starts and severity of wound infection, and is particularly useful
increases after- (MAHE 05) for surveilance and research- (UPSC-11 09)
a) Immediate suture of the wound a) Apgar score
b) 3 to 4 days b) Glasgow scoring system
c) 7-lOdays c) Southamptom grading system
d) 6months
d) ASA classification
28. Prevention of wound infection done by-
40. Which one of the following organisms is not
a) Pre-op shaving (PGI June 05)
associated with synergistic gangrene -(UPSC-II 09)
b) Pre-op antibiotic therapy
a) Escherichia b) Staphylococcus
c) Monofilament sutures
d) Wound apposition c) Clostridium d) Peptostreptococcus
29. Abbey-Estlander flap is used in the reconstruction of- 41. Trueaboutchronicwound- (PGI Nov 09)
a) Buccal mucosa b) Lip (AI 05) a) Found in DM
c) Tongue d) Palate b) Always require surgical treatment
30. The Vitamin which has inhibitory effect on wound c) May associated with vascular compromise
healing is - (MAHE 05) d) Monofilament sutures prevent infection
a) Vitamin-A b)Vitamin-E e) Any wound that does not heal within 3 month
c) Vitamin-C d) VitaminB-complex 42. Condition associated with Panniculitis is/are-
31. Elective cholecystectomy is - (APPG 06) a) Pancreas cancer (PGI Nov 09)
a) Clean contaminated b) Clean b) Chronic pancreatitis
c) Dirty d) Contaminated c) Acute pancreatitis
32. In a sutured surgical wound, the process of d) Pancreatic divisum
epithelialization is completed within- (UPSC 07) e) Posttraumatic pancreatitis
a) 24 hours b) 48 hours 43. Which of the following is a nonabsorbable suture?
c) 72 hours d) 96 hours a) Polypropylene b) Catgut (Maharashtra 10)
33. Which one of the following surgical procedures is c) Polyglactin d) Polydioxone
considered to have a clean-contaminated wound? 44. Hypertrophic scar is characterized by the following,
a) Elective open cholecystectomy for cholelithiasis except- (UP SC II 10)
b) Herniorrhaphy with mesh repair a) It is non-familial
c) Lumpectomy with axillary node dissection b) It outgrows the wound area
d) Appendectomy with walled off abscess c) It involves the flexor surface
34. Fibroblast in healing wound is derived d) It is not related to the race
from- (UPSC-II 08) 45. Match List-1 with List-IT and select the correct
a) Local mesenchyme b) Epithelium answer using the code given below the Lists-
c) Endothelium d) Vascular fibrosis List-! (Agent) List-II (Name/Example)
35. Reactionary hemorrhage occurs within- A. Polymeric film 1. Benzoyl benzoic acid
a) l-2 days b) 2-7 days (DPGEE 08) B. Debriding agent 2. Vandase streptokinase
c) < 24 hours d) After 7 days C. Biological membrane 3. Opsite tegaderm
36. Which of the following is not absorbable suture- D. Enzymatic agent 4. Porcine skin amnion
a) Catgut b)Polyamide (APPG08) Code: (UP SC Ill 0)
c) Polygalactyl d) Polyester A B C D
37. All ofthe following statements about necrotizing a) l 2 3 4
fascitis are true, except- (A/09) b) 3 1 4 2
a) Infection of fascia and subcutaneous tissue c) 3 1 2 4
b) Most commonly caused by Group A beta d)l 2 4 3
hemolytic streptococci

25)a,d 26)b 27) b 28) b,c,d 29) b 30) b 31) a 32) b,c 33) a 34)a 35)c 36)d 37)c 38)b,c
39)c 40)a 41) a,c,d,e 42) a,c,d 43) a 44) b 45) b
SURGERY [ 185]

46. TrueaboutsurgicalwoundsN (PGINov.10) 58. Following fistulous conditions give rise to


a) No antibiotics required in clean surgery maximum fluid and electrolyte imbalanc~(AIIMS 85)
b) Incision of abscess is done in contaminated wound a) Distal ileal b) Gastric
c) Spillage of stomach content converts a clean/ c) Duodenal d) Sigmoid
contaminated case to a contaminated case 59. The minimum amount of proteins needed for
d) In clean/contaminated wounds infections rate is 10% positive nitrogen balance is- (PGI 85)
e) Hernia repair is contaminated wound a) 20-30gm/day b)35-40gm/day
47. Tidy wounds inflicted by sharp instruments and c) 50 gm/day d) 60 gm/day
containing no devitalised tissues are expected to heal 60. Highest concentration of potassium is seen in-
by- (UPSCII11) a) Jejunum b) ileum (AIIMS 92)
a) Secondary healing b) Primary healing c) Duodenum d) Colon
c) Formation of contracture d) SkingrattmLg 61. Deficiency of following elements is seen with
hyperalimentation except- (JIPMER 93)
a) Calcium b) Phosphates
c) Zinc d) Magnesium
62. Hypochloremic alkalosis is a complication of-
a) Congenital pyloric stenosis (Delhi 89, 91)
b) Vomiting
c) Haematemesis
FLUID & ELECTROLYTE d) Aspirin intoxication
63. The disadvantage of elemental diets in children
50. Complication of total parenteral nutrion include- (AIIMS 81, PGI 86)
include N (NIM 86, AIIMS 87) a) Hypertonic dehydration b) Lower caloric input
a) Hyperglycemia b) Hyperkalemia c) Dumping syndrome d) High nitrogen input
64. 20 mEq (mmol) of potassium chloride in 500 ml of
c) Hyperosmolar dehydration d) Azotemia
5% dextrose solution is given intravenously to
e) All of the above
treat- (PGI 81 AIIMS 84)
51. The highest concentration of potassium is in -
a) Metabolic alkalosis b) Respiratory alkalosis
a) Plasma b) Isotonic saline (AIIMS 85)
c) Metabolic acidosis d) Respiratory acidosis
c) Ringer lactate d) Darrow's solution 65. Haemaccel contains- (PGI 81, AIIMS 84)
52. Which does not occur in the first week of IV a) Albumin b) Degraded gelatin
hyperalimentation- (AI 89) ~) Calcium d) Sodium
a) Weight gain b) Uremia 66. In the immediate post operative period, body
c) Jaundice d) Ketosis potassium is- (JIPMER 86, AMU 86)
53. Pitting edema indicates an excess of .... litres of a) Exchanged with calcium
fluid in tissue spaces- (PGI 88) b) Exchanged with magnesium
a) 2.5 b)3.5 c) Retained in body
c) 4.5 d)5.5 d) Excreted excessively
54. In patients subsisting entirely on parenteral 67. Low molecular weight dextran is contra indicated in
fluids, there is weight loss of .••• daily N (PGI 88) a) Foetal distress syndrome (AIIMS 81, PGI 81)
a) 50gm b) 150 gm b) Cerebrovascular accident
c) 200gm d)250gm c) Electrical bums
55. Condition which does not cause metabolic acidosis - d) Thrombocytopenia
a) Renal failure (AIIMS 84) 68. The commonest cause of metabolic alkalosis
b) Ureterosigmoidostomy is - (Karn. 94)
c) Pancreatic or biliary fistula a) Cancer stomach b) Pyloric stenosis
d) Pyloric stenosis c) Small-bowel obstruction d) Diuretics
56. Albumininfusionforparenteraluseisrestrictedbecause- 69. Commonest cause of metabolic alkalosis in
a) It is costly (AIIMS 84)
surgical pateientis- (JIPMER 95)
a) Antacid therapy
b) Carcinogenic
b) Gastric outlet obstruction
c) Does not raise oncolic pressure
c) Hyperventilation due to head injury
d) All of the above d) Steroid treatment
57. Following TPN, one expects weight gain after- 70. Hyponatremia in multiple myeloma is -(Kerala 95)
a) 2 days b) 7 days (AIIMS 84) a) True b) Relative
c) 4 weeks d) 6 weeks c) Absolute d) Pseudo

46)a,c,d 47)b 48)a 49)c 50)a,b,d 51)d 52)a 53)c 54)b 55)d 56) a 57)b 58)c 59)d
60)d 61)None 62)a,b 63)a,c 64)a 65)b 66)d 67)d 68)b 69)b 70)d
SURGERY [ 186]

71. During nutritional assessment of a surgical 82. Water content in infant- (Orissa R)
patient, the status of muscle protein is indicated a) 60-70% b)75-80%
by which one of the following parameters- c) 80-90% d)>90%
a) Serum albumin (UPSC 95) 83. Insensible daily water loss is- (PGI 88)
b) Triceps skinfold thickness a) 500-600 m1 b) 800-1000 m1
c) Mid-anncircumference c) 1000-1500ml d)2000ml
d) Hblevel 84. Sodium content of one litre of isotonic saline is -
72. C.V.P. (Central Venous Pressure) and pulmonary a) 140mEq b) 154mEq (PGI 88)
wedge pressure give an accurate assessment of all c) 40mEq d)70mEq
the following except- (UPSC 95) 85. TPN may be complicated by- (AIIMS 80,81)
a) Tissue perfusion b) Volume depletion a) Obstructive jaundice b) Hyperosteosis
c) Volume overload d) Myocardial function c) Hypercalcemia d) Pancreatitis
73. In a case of acute trauma best guidline for quick 86. Blood loss during major surgery is best estimated
replacement of fluids is- (AIIMS 94, A196) by- (PG/99)
a) Pulse b) Hb a) Visual assessment
c) Urine Output d) C.V.P. b) Suction bottles
74. All of the following are indication of total parentral c) Transesophageal USG Doppler
nutrition except- (AIIMS 95) d) Cardiac output by thermodilution
a) Post-operative ileus 87. PersistentvomitinginGO.O.causes- (PG/02)
b) Enterocolic fistula a) Hyponatremic hyperchloremia occur
c) Acute pancreatitis b) Hypematremia without J.ed Cl- alkalosis
d) Entenocuatneous fistnla c) Hypokalemic metabolic alkalosis
75. In a patient with multisystem trauma, the d) Paradoxical aciduria
presence of hypotension along with elevated centr:al 88. In IV hyperalimentation, we give- (PGI 02)
venous pressure is suggestive of- (UPSC 97) a) Hypertonic saline b) Fats
a) Upper airway obstruction c) Amino acids d) Dextrose
b) Major abdominal bleed e) LMW dextran
c) Cardio-pulmonary problem 89. A postoperative patient with pH 7.25, MAP (mean
d) Spinal cord injury arterial pressure) 60 mm ofHg treated with-
76. Following TPN, weightloss is seen- (Orissa 99) a) LV. sodium bicarbonate (PGJ03)
a) Up to 7 days b)7-1Qth day b) Onlynormalsaline
c) 10-15th day d) 15th day onwards c) Fluid therapy with CVP monitoring
77. Which of the following is the best method to assess d) fluid restriction
the adequacy of replacement- (AIIMS 2K) 90. For a patient of Gastric outlet obstruction, the OPD
a) Decrease in thirst fluid management is- (PG/03)
b) Increase in urine output a) Normal saline
c) Blood pressure b) Hypertonic saline
d) Increased Pa 0 2 c) Na bicarbonate to counteract aciduria
78. In a person who has fasted for 5 days all are seen d) Hypotonic saline without potassium
except- (AIIMS98) e) Normal saline with potassium
a) GH levels decreased 91. Best vein for total parenteral nutrition is -(Mahara
b) Glucose tolerance decreased a) Sibcoavoarn vein b) Femoral vein 02)
c) Immunoreactive insulin decreased c) Brachial d) Saphenous
d) Free fatty acids(plasmal) increased 92. Critical pH in Mendelson syndrome- (Orissa 04)
79. Content ofNa+ in ringer lactate is---- meq/1- a) 2.5 b)3.0
a) 154 b) 12 (TN 99) c) 3.5 c)4.0
c) 130 d) 144 93. The commonest cause of water intoxication in
80. Body water content in percentage of body weight surgical patients is due to- (COMEDK 05)
is lowest in- (Orissa 98) a) Colorectal wash with plain water
a) Well-builtman b)Fatwoman b) Syndrome of inappropriate secretion ofADH
c) Wellnourishedchild d)Fatman c) Irrigation during transurethral resection of prostate
81. Fructose is not used in IV infusion as it cause- d) Excessive infusion of 5% glucose
a) Irritability (AIIMS 89) 94. Complications ofTPN- (PGI June 05)
b) Mental retardation a) Aspiration pneumonia b) Hypokalemia
c) Increased erythrocyte protoporphyrin c) Hypoglycemia d) Hyperglycemia
d) Increased urinary coproporphyrin e) Hyponatremia

7l)c 72)a 73)c 74)None 75)c 76)a 77)b 78)a 79)c 80)b 8l)None 82)b 83)b 84)b
85)c 86)b 87) c,d 88) b,c,d 89) c 90) a,e 91) a 92)a 93)d 94)b,c,d
SURGERY [ 187]

95. Recognized complication frequently a/w enteral 105. The complications of prolonged parenteral
feeding- (PGI June 05) hyperalimentation may include the following
a) Constipation b)Diarrhea except- (UPSCII 11)
c) Aspiration pneumonia d) Hypoglycemia a) Cholestatic jaundice b) Hyperphosphataemia
e) Hypematremia c) Hyperosmolar acidosis d) Hyperarnmonaemia
96. Consider the following- (UPSC 07)
1. Arginine 2. Nucleotides (RNA and DNA) BLOOD TRANSFUSION
3. Fatty acids 4. Glutamine
Which of the above immuno-nutrients are required 106. Halflife offactor 8 is- (PGI 88)
more in the conditions of stress ? a) 4 hours b) 8 hours
a) 1 and 3 only b) 2 and 4 only c) 34 hours d) 48 hours
c) 1, 2 and 3 only d) 1, 2, 3 and4 only 107. Massive transfusions results in- (PGI 88)
97. A patient undergoes a prolonged and complicated a) DIC b) Hypothermia
pancreatic surgery for chronic pancreatitis. Most c) Hypercalcemia d) Thrombocytopenia
preferred route for supplementary nutrition in this
108. ncompatible blood transfusion is diagnosed in a
patientwouldbe- (AI08)
patient under deep anaesthesia by- {TN 90)
a) Total parentral nutrition
a) Increased capillary blood loss
b) Feeding gastrostomy
b) Persistent fall ofB.P.
c) Feeding Jejunostomy
d) Oral feeding c) Allergic dermatitis
98. Parenteral nutrition is not used in- (PGI June 08) d) Increased B.P.
a) Enterocutaneous fistula b) Bums 109. Stored plasma is deficientin- (PGI79, DNB 90)
c) Crohn's disease d) Ileus a) Factors 7 and 8 b) Factors 2 and 5
e) Pancreatitis c) Factors 5 and 8 d) Factors 7 and 9
99. True aboutTPN- (PGI June 08) e) None of the above
a) Carbohydrate forms about 40% of energy source 110. Rosenthal's syndrome is seen in deficiency offactor-
b) In abdominal injury early parenteral nutrition a) II b)V (AIIMS81,DNB91)
should be started. c) IX d)XI
c) Proteins forms 60% of energy source 111. Which of the following is better indicator of need
d) Lipids form 20% of energy source fortranfusion- (AIIMS 80, UPSC 87)
100. Which ofthe following is not a complication of Total a) Urine output b) Haematorcrit
Parenteral Nutrition- (AIIMS Nov 08) c) Colour of skin d) Clinical examination
a) Metabolic bone disease 112. Blood platelets in stored blood do not remain
b) Essential fatty acid deficiency functional after ....... hrs.- (PGI 81, 88, AIIMS 86)
c) Congestive cardiac failure a) 24 b)48
d) Hypophosphatemia c) 72 d)96
101. Complications oftotal parenteral nutrition include 113. Cryoprecipitate is a rich source of- (PGI 79, AIIMS
all except- (COMED 09) a) Thromboplastin b)FactorVIII 85)
a) Hyperglycaemia
c) FactorX d)FactorVII
b) Hypo glycaemia
114. The maximum life of a transfused R.B.C. is-
c) Hypochloraemic alkalosis
a) Onehour b) One day (Jipmer80,
d) Electrolyte abnormalities
102. Which of the following is not an important cause of c) 15 days d) 50 days DNB 89)
hyponatremia- (Manipal 09) e) 100 days
a) Gastric fistula 115. Massive blood transfusion is defined as- (PGI 95)
b) Excessive vomiting a) 350mlin5min b)500mlin5min
c) Excessive sweating c) 1 Litre in 5 min d) Whole blood volume
d) Prolonged Ryle's tube aspiration 116. How long can blood stored with CPD-A-
103. In a patient of total parenteral nutrition daily a) 12 days b)21 days (JIPMER93)
monitoring is done with- (DPG 10) c) 28 days d) 48 days
a) Body weight b) Plasma protein 117. Massive transfusion in previous healthy adult male
c) CBC d) Plasma osmolality can cause hemorrhage due to- (PGI 98)
104. Which of the following nutrients are not included in a) Increased tPA
Total Parental Nutrition - (AI 11) b) Dilutional thrombocytopenia
a) Lipids b) Carbohydrates c) Vitamin K deficiency
c) Proteins d) Fibers d) Decreased Fibrinogen

95)b,c 96)d 97)c 98)b 99)d lOO)c lOl)b 102)d 103)a 104)d 105)b 106)b 107)a,b,d
108)a,b 109)c llO)d lll)b 112)a 113)b 114)d 115)d 116)c 117)b
SURGERY [ 188]

118. Mismatched blood transfusion in anaesthesia 130. In Glasgow Coma Scale all of the following are
present as- (PGI 2000) considered except- (Jipmer 03)
a) Hyperthermia & hypertension a) Eye opening b) Verbal response
b) Hypotension & bleeding from site of wound c) Motor response d) Sensory response
c) Bradycardia & hypotension 131. The first permanent molar erupts between- (Karn
d) Tachycardia & hypertension a) 8- 10 years b) 6-7 years 03)
119. Platelets can be stored at- (AIIMS Nov 05) c) 11-12 years d) 12- 14 years
a) 20-24°C for 5 days b) 20-24°C for 8 days 132. Differentiating features between sepsis and trauma
c) 4-8° C for 5 days d) 4-8° C for 8 days are- (PGIJune06)
120. Which one of the following blood fractions is stored a) iEnergyrequirement b) Catabolism
at- 40•C?- (UPSC 06) c) Insulin resistance d) Fluid loss
a) Cryprecipitate b) Iluman albumin 133. IV fluid replacement (volume & rate) in a trauma
c) Platelet concentrate d) Packed red cells patient is determined by- (PGI June 06}
121. With reference to fresh frozen plasma (FFP), which a) Urine output b) Chest condition
one of the following statements is not correct? c) CVP d)BP
a) It is used as volume expander (UPSC-II 08) e) BloodHb%
b) It is stored at -40"C to -50"C 134. All of the following are indicators of adequacy of
c) It is a source of coagulation factors pre-operative resuscitation except-
d) It is given in a dose of 12-15 ml!k:g body weight a) C-reactive protein level (DELHI PG Mar. 09}
122. Stored blood which has been preserved in a blood b) Consciousness level
bank is deficient in which of the following c) Urine output
coagulation factors- (UPSCII 11) d) Hematocrit level
a) II only b) II and VII 135. Mangled Extremity Severity Score (MESS) includes
c) V and VIII d) IX and X all ofthe following except? (AIIMS May 11)
a) Shock b) Ischemia
SHOCK c) Neurogenic injury d) Energy of injury
123. Features of hypovolemic shock are all except- INFECTION
a) Oliguria b)Bradycardia (NIMHANS86)
c) LowB.P d) Acidosis 136. Golden period for treatment of open wounds is
124. Blood clot the size of a clenched first is roughly .••.. hours- (AIIMS 86, 88)
equal to- (PGI 88) a) 4 b)6
a) 250ml. b)350ml. c) 12 d)24
c) 500ml. d) 600 ml. 137. Hypotension in a case of gas gangrene is best
125. Following is the most important factor in the
treated by- (JIPMER 87)
management of shock- (AIIMS 84)
a) Ringer Lacatete b) Normal saline
a) Blood pressure
c) Plasma d) Whole blood
b) Cardiac output
c) CVP to 8 em of water 138. Commonest form of actinomycosis is -(JIPMER 87,
d) Deficiency of effective circulation a) Faciocervical b)Thoracic PGI88)
126. Cortisol level returns to normal..•.....•.. after c) Right iliacfossa d) Liver
haemorrhage- (AIIMS 91, Orissa 98) 139. Regarding tuberculous lymphadenitis which is
a) 2 weeks b) 10 days correct-
c) 7 days d) 3 days a) Seen in children and young adults
127. One of the following is earliest indication of b) Seen in the aged
concealed acute bleeding- (AI 95) c) History of contact or drinking infected milk
a) Tachycardia b) Postural HT d) Mostly cervical
c) Oliguria d) Cold clammy fingers e) All are the correct
128. The most important cause of the death in septic 140. Whichisnottrueofcarbuncle- (JIPMER86}
shock is- (JIPMER 80) a) Infective gangrene of subcutaneous tissue
a) DIC b) Respiratory failure b) Caused by staphylococcus
c) Renal d) Cardiac c) Diabetics are more prone
129. Shock is clinically best assessed by- (PGI 97) d) Caused by streptococcus
a) Urine output b)CVP e) Pencillin and excision of necrotic tissue treatment
c) BP d) Hydration of choice

118)b 119)a 120)a 12l)a 122)c 123)b 124)c 125)d 126)d 127)a 128)d 129)a 130)d 13l)b
132)None 133)a 134)a 135)c 136)a 137)a 138)a 139)a,c,d 140)d
SURGERY [ 189]

141. The sensitivty ofCasoni's testis- (AP 84 157. Which ofthe following about yaws is incorrect-
a) 500/o b) 60% KERALA 87) a) Caused by treponema pertunae (PG/ 88)
c) 75% d) 90% b) Spread by direct contact
e) 95% c) Sexully transmitted
142. Sardonic grin is associated with- (AIIMS 87) d) Penicillin is used as treatment
a) Rabies b) Tetanus 158. Serum pox is seen among .............. players -(PGI 88)
c) Bell's palsy d) Hemiplegia a) Football b) Hockey
143. Tetanus is caused by- (PGI 88) c) Rugby d) Chess
a) CL Tetani b) CL Welchii 159. Which of the following is incorrect regarding
carbuncle- (PGI 88)
c) Cl. edematiens d) CL Septicum
a) Staphylococcal infection
144. Commonestcauseofcellulitisis- (PGI 88)
b) Diabetes present
a) Staphylococcus b) Streptococcus
c) Males more common
c) E. coli d) Hemophilus
d) Common before age of 40
145. Foaming liver is seen in- (PGI 88) 160. Following are true of erysipelas except (AIIMS 84)
a) Organophosphorus Poisoning a) Streptococcal infection
b) Actinomycosis b) Contagious and infectious
c) Gas gangrene c) Margins are raised
d) Anthrax d) Common in tropics
146. Erysipelas is caused by- (PGI 88) 161. Chronic thick walled pyogenic abscess may be due
a) Staph aureus b) Staph albus to the following except- (AIIMS 84)
c) Strep pyogenes d) Hemophilus a) Presence of a foreign body
147. Hyperbaric oxygen is useful in- (PGI 88) b) Prolonged antibiotic therapy
a) Tetanus b) Gas gangrene c) Virulent strains of organism
c) Frostbite d) Vincents angina d) Inadequate drainage
148. Commonestformofanthraxis- (PGI 88) 162. Following may be premonitary symptoms of
a) Wool sorters disease b) Alimentary type tetanus except- (AIIMS 84)
c) Cutaneous type d) None of the above a) Sleeplessness b)Anxious expression
149. Malignantpustuleoccursin- (PG/88) c) Urinary incontinence d) Headche
a) Melanoma b) Gas gangrene 163. Ampicillin prophylaxis is given in- (PGI 86)
c) Ovarian tumour d) Anthrax a) Rectal surgery b) Splectonomy
150. Actinomycosis is sensitive to- (PGI 88) c) Head and neck surgery d) Biliary surgery
a) Streptomycin b) Nystatin 164. Treatment of spreading streptococcocal cellulitis
is- (PGI 86)
c) Pencillin d) Lodox- uridine
a) Erythromicin b) Pencillin
151. Globi is seen in .............. leprosy- (PGI 88)
c) Tetracycline d) Chloramphenicol
a) Tuberculoid b) Lepromatous
165. A patient with a fistula and chronic pus discharge
c) Border line d) Borderline tuberculoid
from lower face and mandible is most commonly
152. Which of the following parts of the body is not suffering from- (KERALA 89)
affected by leprosy- (PGI 88) a) Dental cyst b) Vincent's angina
a) Testes b) Ovary c) Ludwigs angina d) Actinomycosis
c) Nasal mucosa d) Axilla 166. Multiple fistula in ano commonly occurs in-(TN 91)
153. Leonine facies is seen in .....••...••... leprosy -(PG/ 88) a) Tuberculosis b) Gonococcal protocolitis
a) Tuberculoid b) Bordedfline c) LGV d) Colloid carcinoma of rectum
c) Lepromatous d) Borderline tuberculoid 167. Meical management of Hydatid disease is indicated
154. Most commonly affected peripheral nerve in in- (AI92)
leprosy is- (PG/ 88) a) Pregnancy b) Infected Hydatid cyst
a) Ulnar b) Radial c) Moribund patients d) Multiple pertoneal cyst
c) Medial d) Lateral Popliteal 168. Thymus gland abscess seen in congtenital syphilis
155. Moth eaten alopecia is seen with- (PGI 88) is called - (PGI 80, AIIMS 86)
a) Leprosy b) Syphilis a) Pouchier's abscess b) Politzeri abscess
c) Fungal infection d) Cylindroma c) Douglas abscess d) Dubois abscess
156. Moon's molars seen with- (PGI 88) 169. The lllV virus can be transmitted by the following
a) Syphilis b) Leprosy routes, except- (KARN 94)
c) Amyloidosis d) Actinomycosis a) Homosexual contact b) Intact skin
c) Matemofoetal d) Needle prick

14l)d 142)b 143)a 144)b 145)c 146)c 147)b,c 148)c 149)d 150)c 15l)b 152)b 153)c 154)a
155)b 156)a 157)c 158)c 159)d 160)b,d 16l)c 162)c 163)d 164)b 165)d 166)a,c 167)c 168)d 169)b
SURGERY [ 190]

170. The high risk groups for transmission ofHIV virus 182. Non surgical infections in surgical patients-
include the following except- (KARN 94) a) UTI (PGI 04)
a) Homosexuals b) Haemophiliacs b) LRTI
c) Children ofHN mothers d) Health care workers c) Superficail thrombophlebitis
171. Treatment of contaminated wound in Gas Gangrene d) Wound cellulites
is- (JIPMER 95) e) Clostridium difficile a diarrhea
a) Debridement of wound 183. False regarding carbuncle is- (Kerala 04)
b) Systemic penicillin a) Staphylococus infection
c) Metronidazole Administration b) Painful condition
d) Peroxide dressings c) Males more commonly affected
172. A carbuncle is treated by- (UPSC 95) d) Common before age of 40
a) Incision and drainage 184. Universal (standard) precautions to be observed by
b) Cruciate incision and deroofing surgeons for the prevention of hospital acquired HIV
c) Antibiocs alone infection include the following except- ( UPSC 05)
d) Wide excision a) Wearing gloves and other barrier precaution
173. Painless effusions in joints in congenital syphilis b) Washing hands on contamination
is called- (AI 95) c) Handling sharp instruments with care
a) Clutton's joints b) Banton's joints d) Pre-operative screening of all patients ofHIV
c) Charcot'sjoints d) Synovitis 185. Regarding antibiotics true are- (PGI June 06)
174. A mentally retarded child aged 12 years has a) No prophylaxis for clean contaminated surgery
multiple, painful, discharging shiny white lesions b) No prophylaxis for gastric ulcer surgery
around the anus. Which of the following is the c) Prophylaxis for colorectal surgery
most probable diagnosis- (UPSC 97) d) Local irrigation with antibiotic contraindicated
a) Lupus vulgaris b) Carcinoma when systemic antibiotics given
c) Syphilitic condyloma d) Haemorrhoids 186. Period of onset in tetanus refers to the time between-
175. A boil is due to staphylococcal infection of -(UP 97) a) First injury to spasm (Karnataka pgmee 06)
a) Hair follicle b) Sweat gland b) First symptom to spasm
c) Subcutaneous tissue d) Epidermis c) First spasm to death
176. Incubation period of gangrene is- (ORRISA 98) d) Trismus to laryngeal spasm
a) 1 -3 days b) 4- 6 days 187. All the following are true regarding R5 strain of
c) 6-8 days d) More than 8 days HIV except- (Karnataka pgmee 06)
177. In AIDS, lymphadenopathy is most often due to- a) Utilizes CCRS Co-receptor
a) 1B (PGI 97) b) Predominates in late stages ofHIV
b) Lymphoma c) Transmits HIV efficiently
c) Non specific enlargement oflymph node d) Infects microglial cells
d) Kaposi's sarcoma 188. Preferred time for prophylactic antibiotic
178. Round worm causes following except- (PGI 97) administration for surgery- (PGI June 09)
a) Gall stone b) Cholangitis a) 1 day before surgery
c) Hemobilia d) Pancreatitis b) At the time of induction of anaesthesia
179. True about cellulitis oflower limb- (PGI 2000) c) I.V. during surgery
a) Infection of skin & subcutaneous tissue d) I.M. before 6 hr
b) Fever & malaise are common e) Orally given
c) Margins are distinct 189. In a surgical patient, the causes of non-surgical
d) External wound always present infection are all except- (DPG 10)
e) Inv. site is red & hot a) Lower respiratory tract infection
180. Fallman 's balanitis is caused by- (Kerala 03) b) Wound infection
a) Trichomonas b) Candida c) Clostridium difficile diarrhea
c) H. Ducreyl d) None d) UTI
181. When do we have to start antibiotics to prevent 190. False about cellulitis- (PGI Nov. 1 0)
post-operative infection? (Jipmer 03) a) Caused by Strep. pyogenes
a) 2 days before surgery b) Causes SIRS
b) After surgery c) Localized infection
c) 1 week before surgery d) Abscess if any should be managed conservatively
d) 1 hour before surgery & continue after surgery e) I & D of abscess should be done

170)d 171)a,b 172)a,d 173)a 174)c 175)a 176) a,b 177) c 178) None 179) a,b,c,e 180) d 181) d
182)a,b,e 183)d 184)d 185)c 186)b 187)b 188)b 189)b 190)c,d
SURGERY [ 191 ]

203. An Isograft indicates transfer of tissues between -


a) Unrelated donors (AI 93)
b) Related donors
c) Monozygotic twins
d) From the same individual
204. Investigation of choice in the early phase of renal
transplant- (Kerala 97)
a) NP b) Retrograde cystourethrogram
c) Ultrasonogram d) CT scan
205. Hyperacute graft rejection is caused by- (Rohtak
a) Preformed antibodies b) T- lymphocytes 97)
c) Macrophages d) B- lymphocytes
e) Mast cells
206. Infection in renal transplant patient is usually
caused by- (Rahtak 98)
a) CMV b)IDV
c) Herpes d) Salmonella
TRANSPLANTATION e) Pneumococcus
207. Mostcommontypeofrenaltransplantationinlndiais-
194. Commonest complication ofimmunosuppression a) Allograft b)Autograft (Al99)
is~ (NIMHANS 86, JIPMER 87, Al88) c) Isograft d) Xenograft
a) Malignancy b) Graft rejection 208. Most important HLA for organ transplantation
c) Infection d) Thrombocytopenia and tissue typing~ (MAHE 98)
195. Highest chance of success in renal transplant is a) HLA-A b) HLA-B
seen when the donor is the -(NIMHANS 86, JIPMER c) HIA-C d)HIA-D
a) Identical twin b) Father 87) 209. HLA matching is not necessary in which of the
c) Mother d) Sister following organ transplantation- (JIPMER 02)
e) Husband a) Liver b)Bonemarrow
196. Commonest indication for liver transplantation c) pancreas d) Kidney
in infant is~ (JIPMER 87, AI 89) 210. Renal transplantation is most commonly done in-
a) Alcoholic cirrhosis b) Biliary cirrhosis a) Chr. glomerulonephritis (PGI 97)
b) Bilateral staghom calculus
c) Primary hemochromatosis d) Biliary atresia
c) Horse shoe kidney
197. Principal cause of death in renal transplant
d) Oxalosis
patients~ (AIIMS 86, PGI 86, UPSC 88)
211. Steroids are used in transplantation ~ (TN 03)
a) Uremia b} Malignancy
a) To prevent graft rejection
c) Rejection d) Infection b) To prevent infection
198. Following drugs are known immunosuppressive c) To speed up recovery
agent except- (ATIMS 84) d) To enhance immunity
a) Prednisolone b) Cephalosporin 212. Which ofthe following drugs is not a part ofthe
c) Azathioprine d) Cyclosporin- A 'Triple Therapy' immunosuppression for post-renal
199. Transplantation of kidney from mother to son is transplant patients? (AI 06)
an example of- (AIIMS 84) a) Cyclosporine b) Azathioprine
a) Autograft b)Allograft c) FK506 d) Prednisolone
c) Isograft d) Xwnograft 213. Trasplantation of which one of"the following organs
200. Site of transplantation in Islet cell transplant for is most often associated with hyper-acute rejection?-
diabetes mellitus - (PGI 84) a) Heart b) Kidney (UPSC06)
a) Forearm muscles c) Lungs d) Liver
b) Pelvis
c) Thigh PLASTIC & RECONSTRUCTIVE
d) Injected into the portal vein
SURGERY, SKIN LESIONS
201. Graft from sister to brother is- (JJPMER 90)
a) Isograft b)Allograft
214. Cock's peculiar tumour is-(UPSC 86,NIMHANS 87,
c) Autograft d) Heterograft a) Papilloma Kerala 87, TN 90)
202. Amputateddigitsarepreservedin- (Al92) b) Infected sebaceous cyst
a) Cold saline b) Cold Ringer Lactate c) Cylindroma
c) Plastic bag in ice d) Deep freezer d) Sqaumous cell carcinoma

19l)d 192)d 193)c 194)c 195)a 196)d 197)d 198)b 199)b 200)d 201)b 202)c 203)c 204)c
205)a 206)a 207)a 208)d 209)a 210)a 211)a 212)c 213)b 214)b
SURGERY [ 192]

215. Cause ofpersistance of a sinus or fistulae includes- 229. Skin flap is used in all except- (AIIMS 89)
a) Foreign body (JIPMER 86) a) Bone b) Tendon
b) Non dependentt drainage c) Burn wound d) Cartilage
c) Unrelieved Obstruction 230. Free skin graft is rejected on- (AJIMS 89)
d) Presence of malignancy a) Muscle b) Fat
e) All of the above c) Deep fascia d) Dermis
216. Premalignant conditions of the skin include- 231. Kaposisarcomaisseenin- (A/91)
a) Leukemia b)Lymphoma
a) Bowen disease b) Pagel's disease of nipple
c) AIDS d) Cytomegalovirus infections
c) Leukoplakia d) Solar keratosis (JIPMER 86)
232. Which of the following is a regressing tumour-
e) All ofthe above a) Portwine stain b) Strawberry angioma {Al91)
217. Melanoma should be excised with a margin of- c) Venous angioma d) Plexiform angioma
a) 2cm b) 5 em (UPSC 88) 233. Sebaceous cyst occurs in all except- (TN 91)
c) 7cm d) lOcm a) Face b) Scrotum
218. Hidradenitis suppuragtive is found to occur c) Scalp d) Palm and soles
in- (JIPMER 86, AIMS 87) 234. Skin graft for facial wounds is taken from-(AIIMS 92)
a) Axilla b) Circumanal a) Medial aspect of thigh b) Cubital fossia
c) Scalp d) Groin c) Groin d) Post auricular region
219. The best dressing is- (PGI 88) 235. Spontaneous regression is seen in all except-(AJ93)
a) Opsite b) Amnion a) Salmon patch b) Small Cavernous hemangioma
c) Tulle grass d) Skin c) Portwine stain d) Strawberry angioma
220. The term universal tumour refers to- (PGI 88) 236. The best skin graftfor open wounds is- {Al93)
a) Adenoma b)Papilloma a) Isograft b) Homograft
c) Fibroma d) Lipoma c) Allograft d) Autograft
221. Hydrocele is a type of. .... cyst- (PGI 88) 237. Dercum's disease is commonest in the- (PGI 80,
a) Retention b) Distension a) Face b)Arm AJIMS 84)
c) Exudation d) Traumatic c) Back d) Thigh
222. Sebaceous cyst does not occur in the ...• - {PGI 88) 238. Margins of squamous cells carcinoma is -(JIPMER
a) Scalp b) Scrotum a) Inverted b) Everted 81,Delhi 86)
c) Back d) Sole c) Rolled d) Undermined
223. Commonest site for rodent ulcer is- (PGI 88) 239. Salmon patch usually disappears by age- (PGI 80, 81,
a) Inner canthus b) Outer canthus a) One mouth b) One year UPSC 89)
c) Angle of mouth d) Cheek c) Puberty d) None of the above
224. Squamous cell carcinoma can arise from-{PGJ 88) 240. Most severe form of malignant malenoma is-
a) Long standing venous ulcers a) Superficially spreading (Kerala 94)
b) chronic lupus vulgaris b) Nodular infiltrating type
c) Rodent ulcer c) Those arising in lower limb
d) All of the above d) Those in choroid
225. The best results in treatment of capillary nevus 241. Calcifying epethelioma is seen in- (JIPMER 95)
have been achieved by- (AIIMS 84) a) Dermatofibroma b) Adenoma sebaceum
a) Full thickness skin graft b) Dermabrasion c) Pyogenic granuloma d) Nevo cellular nevus
c) Tatooing d) Argon laser treatment 242. Besttreatmentofstrawberryangiomais-(.DPMER95)
226. Full thickness skin graft can be taken from the a) Sclerosants b) Corticosteroids
following sites except- (AJIMS 87) c) Masterly inactivity d) Excision
a) Elbow b) Back to neck 243. Cock's pepculiar tumour is- (UPSC 95)
c) Supraclavicular area d) Upper eyelids a) Secondary to osteomylitis of skull bones
b) Secondary to infected sebaceous cyst
227. Keratoacanthoma is- (AIIMS 85)
c) Secondaries in skull bones from cancer thyroid
a) A type ofbasal cell carcinoma
d) Epithelioma of scalp
b) Infected sebaceous cyst
244. Basal cell carcinoma is common in - {Al9 5)
c) Self healing nodular lesion with central ulceration
a) Head and Neck b) Legs
d) Pre-malignant disease
c) Chest d) Upper limbs
228. Acanthossis nigricans is seen with- (PGI 88)
245. Marjolin's ulcer is- {TN 95)
a) Colonic carcinoma a) Squamous cell carcinoma from scar
b) Freckle b) Adenoma of scar
c) Squamous cell carcinoma of skin c) Tuberculous ulcer
d) Carcinoma breast d) Amoebic ulcer

215)e 216)e 217)a 218)a,b,d 219)d 220)d 22l)c 222)d 223)a 224)d 225)d 226)None 227)c
228) a,d 229) c 230) None 231) c 232) b 233) d 234) d 235) b,c 236) d 237) c 238) b 239)b 240)b
24l)None 242)c 243)b 244)a 245)a
SURGERY [ 193]

246. Boil can occur at all sites except- (TN 95) 258. Worst prognosis in Melanoma is seen in the subtype-
a) Pinna b) Skin a) Superficial spreading (Kerala 2001)
c) Scalp d) Palm b) Nodular Melanoma
247. The organism causing destruction of skin grafts c) Lentigo Maligna Melanoma
is- (PGI 95) d) Amelanotic Melanoma
a) Streptococcus b) Staphylococcus 259. Lentigo maligna is most commonly seen in-
c) Pseudomonas d) Clostridium a) Face b) Trunk (AJIMS OJ)
248. Which ofthe following characterise a Demoid Cyst- c) Sole d) Leg
a) Chessy material 260. The subdermal plexus forms the vascular basis for-
b) Presence of columiner epithelium a) Randomised flaps (JJPMER 2002)
b) Axial flaps
c) Tooth
c) Mucocutaneous flaps
d) Air
d) Vasciocutaneous flaps
249. All are features of Gummatous ulcer except-(AP 96)
261. Least malignant melanoma is- (Kerala 2001)
a) Punched out edges b) Syphylitic in nature a) Lentigo maligna b) Superifcial spreading
c) Wash leather slough d) Erythematous base c) Nodular d)Amelanotic
250. Eleven month old child presents with erythematous 262. Regarding hemangiomas following are true-
lesion with central clearing which has been a) Salmon patch disappears after the age of one
decreasing in size- (AI 97) b) Port wine stain present throughout life
a) Strawberry angioma c) Salmon patch-on forehead midline and over occiput
b) Nevus d) all are correct
c) Portwine stain 263. True about marjolins ulcer is- (PGJ 97)
d) Cavernous haemangioma a) Ulcer over scar b) Rapid growth
251. II are true statement about malignant melanoma c) Rodent ulcer d) Painful
except- (AI97) 264. Commonest cancer in burn scar is- (PGJ 97)
a) Clark's classification used for prognosis a) Sq. cell Ca b) Fibrosarcoma
b) Women have better prognosis c) Adenoa Ca d) Adeno-squamous Ca
c) Acral lentigenous have better prognosis 265. Prognosis of melanoma depends on- (PGI 98)
d) Limb perfusion is used for local treatment a) Stage
252. Graft is not taken up on the following-(AJJMS 96) b) Depth of melanoma on biopsy
a) Fat b) Muscle c) Duration of growth
c) Deep fascia d) Skull bone d) Site
253. Prognosis of malignant melanoma depends 266. In pigmented basal cell carcinoma, treatment of
on- (JJPMER 98) choice is - (PGJ 98)
a) Grade of tumor b) Spread of tumor a) Chemotherapy b) Radiotherapy
c) Depth of invasion d) Metastasis c) Cryosurgery d) Excision
254. Man sustained an injury with loss of skin cover 267. Diagnostic procedure for basal cell Ca- (PGI 98)
exposing bone oflOxlO ems. The best treatment is- a) Wedge biopsy b) Shave
a) Full thickness graft (AJIMS 99, 96, c) Incisional biopsy d) Punch biopsy
b) Pedicle graft MAHE 05) 268. Split skin graft can be applied over- (PGI 99)
c) Amnion a) Muscle b) Bone
d) Split thickness skin graft c) Cartilage d) Eyelid
255. Skin graft survival in the first 48 hrs is dependent 269. Which is/are not absorbable suture- (PGI 2000)
on- (AJIMS 99) a) Vietyl b) Catgut
a) Random connection between host & donor capillaries c) Polyethylene d) Polyacramide
b) Plasmatic imbibition e) Polypropylene
c) Saline in dressing 270. Trophic ulcers are caused by- (PGJ 02)
d) Development of new blood vessels a) Leprosy b) Buerger's disease
256. Skin grafting is not done in infection with-(MP 2K) c) Syringomyelia d)DVT
a) Pseudonmonas aeroginosa e) Varicose veins
b) Staph. Aureus 271. True about Marjolins ulcer- (PGJ 03)
c) Beta hemolytic streptococci a) Develops in long standing scar
d) E. coli b) Sq cell Ca develops
257. Frost bit is treated by- (AMC 2K) c) Slow growing lesion
a) Rapid rewarming b) Slow rewarming d) Also know as Baghdad sore
c) IV pentoxyphiline d) Amputation e) CommoninBlackraces

246)d 247)a 248) a,c 249) d 250) a 251)c 252)d 253)c 254)b 255)b 256)c 257)b 258)d 259)a
260)a 261)a 262)d 263)a 264)a 265) a,b,d 266) d 267) a 268) a 269) c,d,e 270) a,c 271) a,b,c
SURGERY [ 194 J

272. Chronically lymphoedematous 6mb is predisposed 283. In the Clatkets level of tumor invasion for malignant
to all of the following except- (AI 04) melanoma level 3 refers to- (COMED 06)
a) Thickening of the skin a) All tumar cells above basement membrane
b) Recurrent soft tissue infections b) Invasion into reticular dennis
c) Matjolin's ulcer c) Invasion into loose connective tissue of papillary
d) Sarcoma dermis
273. Treatment for pyoderma gangrenosum is- (JH 03) d) Tumorcellsatjunctionofpapillaryandreticulardennis
a) Steroids b) I.V. antibiotics 284. Mob's Micrographic excision for basal cell
c) Surgery+ antibiotics d) Surgery alone carcinoma is used for all of the following except -
27 4. Which ofthe following materials for implants will evoke a) Recurrent Tumour (Karnataka 06)
least infiammatory tissue response- (SGPGI 04) b) Tumor less than 2 em in diameter
a) Polypropylene b) Bovine collagen c) Tumors with aggressive histology
c) Polygiactin d) Cotton d) Tumors with perineural invasion
275. Malignant melanoma most often develops 285. Split skin grafts in young children should be
from- (SGPGI 05) harvestedfrom- (UPSC07)
a) Hairy naevus b) Junctional naevus a) Buttocks b) Thigh
c) Intradermal naevus d) Blue naevus c) Trunk d) Upper limb
276. The best cosmetic results for large capillary (port 286. Chronic Burrowing nicer is caused by- (AI 07)
wine) hemangiomas are achieved by- (UPSC 05) a) Microaerophilic streptococci
a) Excision and split-thickness skin b) Peptostreptococcus
b) Laser ablation c) Streptococcus viridans
c) Cryosurgery d) Streptococcus pyogenes
d) Tattooing 287. Basal cell carcinoma spread by- (MAHE 07)
277. Which one ofthe following is not included in the a) Lymphatics b) Haematogenous
treatment of malignant melanoma- (UPSC 05) c) Direct spread d) None of the above
a) Radiation b) Surgical excision 288. Wolfe grafts is- (UP 07)
c) Chemotherapy d) Immunotherapy a) Full th.ickness skin grafts
278. The most common malignancy found in Marjolin's b) Partial thickness skin grafts
ulcer is - (AI 06) c) Split-skin grafts
a) Basal cell carcinoma d) Pedicled flap
b) Squamous cell carcinoma 289. Marjolin ulcer- (PGI June 07)
c) Malignant fibrous histiocytoma a) Cain matjolin's is squamous cell ca
d) Neurotrophic malignant melanoma b) Chronic venous insufficiency
279. The tumor, which may occur in the residual breast c) Basal cell carcinoma
or overlying skin following wide local excision d) arise from base of the ulcer
and radiotherapy for mammary carcinoma, is- 290. The commonest clinical pattern of basal cell
a) Leiomyosarcoma (AI 06) carcinoma is- (Corned 08)
b) Squamous cell carcinoma a) Nodular b) Morpheafonn
c) Basal cell carcinoma c) Superficial d) Keratotic
d) Angiosarcoma 291. Which one ofthe following is not a wound closure
280. Dacron vascular graft is- (AI 06) technique? (UPSC-II 08)
a) Nontextile synthetic b) Textile synthetic a) Partial thickness skin graft
c) Nontextile biologic d) Textile biologic b) Composite graft
281. ThetypeoftnmourisMarjolin'sulcer-(PG/June06) c) Vascular graft
a) Squamous cell carcinoma b) Adenocarcinoma d) Musculo-cutaneous graft
c) Basal cell carcinoma d) Fibrosarcoma 292. Which of the following is a non-absorbable sutrue?
e) Mixed tumour a) Polypropylene b)Vicryl (AI08)
282. Which one of the following statements about Mesh c) Catgut d) Polydioxanone
Skin Grafts is not correct?- (UPSC 06) 293. Abbe-EstlanderFlapis used for- (AI 08)
a) They permit coverage oflarge areas) a) Lip b) Tongue
b) They allow egrees offluid collections underth.e graft) c) Eyelid d) Ears
c) They contract to the same degree as a grafted 294. Abbey estender nap is based on- (Aiims May 08)
sheet of skin) a) Lingual artery b) Facial artery
d) They "take" satisfactorily on a granulating bed) c) Labial artery d) Internal maxillary artery

272)c 273)c 274)a 275)b 276)b 277)a 278)b 279)d 280)b 28l)a 282)c 283)d 284)b 285)b
286)a 287)c 288)a 289)a,b,c 290)a 29l)c 292)a 293)a 294)c
SURGERY [ 195]

295. Chronic Burrowing ulcer is caused by- 304. A Wolfe graft is a- (UPSC II 11)
a) Microaerophilic streptococci (AIIMSMay 08) a) Partial thickness skin graft
b) Peptostreptococcus b) Pinch skin graft
c) Streptococcus viridans c) Small full thickness skin graft
d) Streptococcus pyogenes d) Pedicle graft
296. True about malignant melanoma- (PGI June 08)
a) Lymphatic spread BURNS
b) Lymph node biopsy is done always
c) Block dissection to be done when sentinel node 305. Exposure treatment is done for burns of the-
is involved a) Upperlimb b)Lowerlimbs (JIPMER87)
d) Microsatellitism c) Thorax d) Abdomen
297. 'Take in' of split skin graft occurs when - e) Head&neck
a) Tight dressing is applied (PGI Dec 08) 306. Rule of nine to estimate surface area of a burnt
b) Excessive discharge from wound patientwasintroducedby- (AIIMS 84)
c) ~ hemolytic streptococcus infection is present a) Mortiz kaposi b) Alexander wallace
d) Wound bed not vascularised c) Joseph lister d) Thomas barclay
298. True statement (s) regarding hypertrophic scar- 307. Deep skin burns is treated with- (AIIMS 91)
a) Grow beyond wound margin (PGI Dec 08) a) Split thickness graft
b) More common in female b) Full thickness graft
c) Notfamilial c) Amniotic membrane
d) Rarely subsides d) Synthetic skin derivatives
e) Not race related 308. In burns heat loss is by/ due to- (PGI 80,
299. The most common malignancy found in Marjolin's a) Dilatation of veins DELHI 80,92)
ulcer is- (DELHI PG Feb. 09) b) Shock
a) Basal cell carcinoma c) Exposed area by evaporation
b) Squamous cell carcinoma d) None ofthe above
c) Malignant fibrous histiocytoma 309. The cold water treatment of burns bas the
d) Neutrophic malignant melanoma disadvantageng that it increase the chances of-
300. Match Iistl (Characteristic feature) with list II (Scar a) Pain b)Exudation(PGI81,AIIMS83)
type) and select the correct answer using the codes c) Infection d) None of the above
given below the lists- (UPSC-II 09) 310. An adult whose both lower limbs are charred along
List I (Characteristic List II (Scar type) with genitalia bas ---burns -(PGI 80, AIIMS 84)
feature) a) 18% b) 19%
1. Notfamilial l.Bumscar c) 36% d)37%
2. Kundah 2. Hypertrophic scar 311. Pus in burns form in- (PGI 79, DELHI 89)
3. Lower Sileru 3. Tattooed scar a) 2-3 Days b) 3-5 days
4. Particles or dirt, 4. Keloid scar soot c) 2-3 weeks d) 4
implanted in wound 312. One offolowing is not seen in severe burns -(AI 96)
Codes: a) Hypovolemia b) Sepsis
A B c D c) Duodenal ulcer d) Hyperthemia
a) 3 1 4 2 313. Undue restlessness in a patient during the
b) 2 4 1 3 immediate post bum period is often a manifestitation
c) 3 4 1 2 of- (Karn 95)
d) 2 1 4 3 a) Hypoxia b) Hypovolemia
301. Full thickness graft can be obtained from all oftbe c) Hyperkalemia d) Anxiety
following except- (AIIMS Nov 09) 314. Calculate the percentage of burns on the bead,
a) Axilla b)Groin ueck and face in a child of oue year- (A.L 88)
c) Supraclavicular area d) Elbow a) 100/o b) 16%
302. Cock's peculiar tumor is- (AIIMS Nov 1 0) c) 13% d)15%
a) Basal cell CA b) Squamous cell CA 315. All requires hospitalization except- (AI 91)
c) Ulcerated sebaceous cyst d) Cylindroma a) 5% Bums in children
303. Tumor with perineural invasion- (AIIMSNov 10) b) 10% Scalds in children
a) Adenocarcinoma b) Adenoid cystic Ca c) Electrocution
c) Basaal cell ca d) Squamous cell ca d) 15% Deep bums in adults

295)a 296) a,b,c,d 297) None 298) c,e 299) b 300)b 301)a 302)c 303)b 304)c 305)e 306)b 307)a
308)c 309)c 310)d 311)a 312)d 313)d 314)None 315)a
SURGERY [ 196]

316. Late deaths in burns is due to- (PGI 99) 327. A third degree cirumferential burn in the arm and
a) Sepsis b) Hypovolemia forearm region, which of the following is most
c) Contractures d) Neurogenic important for monitoring- ( U.P.P. G.ME.E. 04)
317. Burns with vesiculation, destruction of the epidermis a) Blood gases
and upper dermis is- (PGI 99) b) Carboxy-oxygen level
a) 1st degree b) 2"d degree c) Macroglobiunria cryoglobinuria
c) 3ro degree d) 4th degree d) Peripheral pulse and circulation
318. Trueaboutburns- (PGI 2000) 328. In 3rd degree burns, all are seen except-
a) Hyperglycemia is seen in early bums a) Vesicles are absent (UPPGMEE04,
b) Child with bums should have damp dressing
b) Painful PG/99)
c) Leathery skin
c) Chemical powder bums should be kept dry
d) Reddish due to Hb infiltration
d) 3rd degree burns are painfull
329. IV rules for burns- (MAHE 05)
319. Head & neck involvement in burns in infant is- a) % body surface area X weight in pounds X 4 =
a) 9-l/o b)1S% (PGI 2000) Volume in ml
c) 27% d)32% b) % body surface area X weight in Kgs X 4 =
320. Metabolic derangements in severe burns are all Volume in Lts
except- (PGI 2000) c) % body surface area X weight in Kgs X 5
a) tcortico steroid secretion Volumeinml
b) Hyperglycaemia d) % body surface area X weight in Kgs X 4 =
c) t secretion ofHCl Volume inml
d) Neutrophil dysfunction 330. In a patient with the burn wound extending into the
321. Superficial burns; true is/are- (PGI OJ) superficial epidermis without involving the dermis
a) Always requires skin grafting would present with all of the following except-
b) Dry & inelastic a) Healing of the wound spontaneously without scar
c) Blisterformation formation (SGPGI 05)
d) Painless b) Anaesthesia at the site of bums
e) Can be healed within 7 to 10 days c) Blisterformation
322. True statement about burn resuscitation -(PGI 03) d) Painful
a) Colloid preferred in initial24 hrs 331. What is the most important aspect of management
b) Colloidpreferredifbumtareais> 15%oftotalBSA of burn injury in the first 24 hours? (UPSC 07)
c) Half ofthe calculated fluid given in initialS hrs. a) Fluid resuscitation b) Dressing
d) Diuretics should be given to all pt of electric bum. c) Escharotomy d) Antibiotics
323. A burn patient is referred when- (PGI 04)
332. Treatment of burns includes- (PGI June 08)
a) No bandage to head & neck
a) 10% superficial bum in child
b) Immediate application of ice cold water
b) Scald in face
c) Superficial bums without blister-no need of dressing
c) 25% superficial bum in adult
d) Escharotomy done for peripheral circumscribed
d) 25% deep burn in adult lesions
e) Buminplam 333. Most common cause of death due to burns in early
324. The ideal temperature of water to cool the burnt period is? (APPG 08)
surface is- (UPSC02) a) Sepsis b) Hypovolemic shock
~19 ~1~ c) Both d) None
c) go d) 6° 334. A 50 year female has 2 deep burn involving 45% total
325. Indications for specialist referral in burns - bodysurfacearea(fBSA). Regarding her management
a) > 20% superficial bum in adult (PGI 04) which ofthe following statement (s) is/are true-
b) Only palms a) Give rapid normal saline infusion (PGI Dec 08)
c) Scalds on head and face b) Half of the calculated fluid should be given in
d) 10% bums in infants initialS hr
e) 10% deep bums in adult c) 91itre ofRinger 's lactate should be given in first24 hr
326. The best guide to adequate tissue perfusion in the fluid d) Urine output should be maintained at 25-30 ml/hr
management of a patient with burns, is to ensure a e) CVP line should be inserted
minimum hourly urine output of- (Kam 04) 335. What are true regarding 20 deep burn-(PG/Dec 08)
a) 10-30ml b)30-50ml a) Blanch on pressure b) Erythema
c) 50-70ml d)70-100ml c) Dry white colour d) Painless
e) Predispose to hypothermia

316)a 317)b 31S)a 319)b 320)c 32l)c,e 322)c,d 323)b,d,e 324)a 325)All 326)b 327)d 32S)b 329)d
330)b 33l)a 332) a,c,d 333) b 334) b,c,e 335) c,e
SURGERY [ 197]

336. True about thermal bum injury- (PG!June 09) 346. Intermittent claudication at the level of the hip
a)Outermost layer is zone of stasis indicates- (PGI 87)
b)Middle layer is zone ofhypermia a) Popliteal artery occlusion
c)inner layer is zone of coagulation b) Bilate iliac artery occlusion
d)Hypermia is due to vasodilation c) Common femoral occlusion
e)Zone of stasis is associated with vascular damage d) superficial femoral artery occlusion
337. Fever in burnt patient is cause by- (PGI June 09) 347. The artery commonly involved in cirsoid aneurysm
a) Septicemia b- (Pill~
b) Due to hypermetabolism a) Occipital b) Superficial temporal
c) Toxin released by dead tissue c) Internal carotid d) External carotid
d) Dead product 348. Plusating tumours include all except- (PGI 88)
338. Which ofthe following statement(s) is/are about a) Bone sarcoma
postbnrn neck contracture- (PGI June 09) b) Osteoclastoma
a) Occur because of conservative management of c) Secondaries from hyper nephromas
deep bum d) Secondary from prostate
b) Treated by flaps 349. Preferred material for femoro popliteal bypass-
c) Oblliteration of cervicomental angle a) Dacron b) PTFE (PGI 89)
d) Dental abnormalities may be present c) Saphenous vein d) Gortex
e) Never develop in deep dermal bum 350. Mostcommoncauseofaneurysm ofabdominalaorta
339. A tumour arising n a bums scar is likely to be- is - (AI 96, Jipmer 80, Delhi 89)
a) Basal cell carcinoma (COMED 09) a) Trauma b) Atherosclerosis
b) Squamous cell carcinoma c) Syphilis d) Cystic medial necrosis
c) Malignantmelanoma 351. The most common complication of an aortic
d) Fibrosarcoma aneurysm size 8 em is - (Delhi, PG 96)
340. Feverinburntpatientiscausedby- (PGINov09) a) Rupture b) Intramural thrombosis
a) Septicemia c) Embolism d) Calcification
b) Due to hypermetabolism 352. Management of a cause of iliac artery embolism
c) Decreased sweating requires- (JIPMER 81, UPSC 86)
d) Release of pyrogens from dead product a)
Embolectomy
e) Dehydration b)
Injection of vasodilators
341. An adult whose both lower limbs are charred along c)
Hypotensive therapy
with genitalia has........bums- (DPG 10) d)
Sympathectomy
a) 18% b) 19% 353. Intermittent-claudicatin is caused by- (TN 89)
c) 36% d)37% a) Venous occlusion b) Arteria insufficiency
c) N erual compression d) Muscular dystrophy
ARTERIAL DISORDERS 354. Burger's disease is seen in - (PGI 88)
a) Only male b) Age less than 40
342. Maximum tourniquet time for the upper limb is- c) Age more than 40 d) Smoker
a) 112 hour b) 1 hr (JIPMER 87) 355. In the abdomen, aneurysms of the .... commonly
c) 1-1/2 hrs. d) 2 hrs occur next only to the aorta .... - (PGI 88)
e) 2-112 hrs a) Internal iliac artery
343. Thromboembolism after pelvic surgery is usually b) External iliac artery
from the veins- (AI 89) c) Splenic artery
a) iliac b) Calf d) Inferior mesentric artery
c) Femoral d) Pelvic 356. CongenitalA-V fistulas in the thigh will be
344. Most common cause of death in patients with associated with all except- (PGI 89)
Burger's disease is - (AIIMS 87) a) Increased cardiac output
a) Gangrena b)Puhnonaryembolism b) Increased skin tempreture
c) Myocardial infarction d) Carcinoma lung c) Gigantism oflimb
345. Which of the following best responds to d) Superficial venous engorgement
sympathectomy- (JIPMER 86) 357. In extraperitoneal approach, to left sympathectomy
a) Burger's disease b) Hyperhydrosis the following may be injured-
c) Raynaud's disease d) Acrocyanosis a) Ureter b) Gonadal vessels
c) A+B d)NC

336)b,e 337)a,b 338)a,b,c,d 339)b 340)a,b 34l)d 342)c 343)c 344)c 345)b 346)b 347)b 348)d 349)c
350)b 351)a 352)a 353)b 354)a,b,d 355)c 356)b 357)c
SURGERY [ 198]

358. Lumbar sympathectomy is indicated in- (TN 90) 369. A useful through temporary improvement in a
a) Intermittent claudication patient's ischaemic foot can be attained by giving
b) TAO with skin changes intravenously- (PGI 79, Delhi 84)
c) Burger's disease a) I 0% Mannitol b) I 0% Dextrose
d) Raynaud's disease c) Dextran 40 d) Dextran I 00
359. Commonest site of throboangitis obliterans is - 370. Diabetic gangrene is due to- (Kerala 94)
a) Femoral artery b) Popiteal artery (AI 90) a) !schema
c) iliac artery d) Pelvic vessels b) Increased blood glucose
360. Theatmentof acute femoral embolus is- (AIIMS 91) c) Altered defence by host and neuropathy
a) Warfarin
d) All of the above
b) Heparin 371. All are true about Embolic Arterial occlusion except-
c) Immediate embolectomy a) No previous history (JIPMER 95)
b) Muscles are unaffected
d) Embolectomy after 5 days bed rest
c) Pulse is absent
361. Ganglion which is spared in Lumbar sympathetomy
d) Anaesthesia is present
is- (JIPMER 92)
372. All are true about Raynauds phenomena except-
a) Ll b)L2
a) Exposure to cold aggravate (Kerala 95)
c) I3 d)L4
b) Spasm of vessels
362. Vessels most commonly involved in thrombo c) More common is females
angitis obliterans- (AIIMS 92) d) Atherosclerosis of vessels
a) ilio-femoral 373. The commonest site ofiodgement of a pulmonary
b) Aorto-iliac embolus is in the territory at- (UPSC 95)
c) Femoropopliteal a) Rt.lower lobe b) Rt. upper lobe
d) Arterior and Posterior tibial c) Lt.lower lobe d) Lt. upper lobe
363. Following are used in treatment ofBuergers disease 374. Kaposi sarcoma is commonly seen in- (AMU 95)
except- (AI 93) a) Upper limbs b) Lower limbs
a) Trental b) Anticoagulation c) Head and Neck d) Trunk
c) Sympathectomy d) Antiplatelets 375. Which ofthe following causes meximum bleeding-
364. For aortic graft the best material available a) Partial arterial severing (PGI 95)
is- (JIPMER 81, Delhi 79, 92) b) Complete arterial severing
a) Dacron b) Artery c) Artery caught between fractured ends of bones
c) Vein d) None d) Intimal tear
365. In a lumbar sympathectomy the sympathetic 376. In a 40 years old male thrombus in the common
chain in its usual position is likely to confused with femoral artery is because of- (AIIMS 97)
the- (PGI 81, AIIMS 80, 82) a) Atheroma
a) Ureter b) Psoas minor b) Thrombangits obliterans
c) Genitofemoral nerve d) Ilioinguinal nerve c) Reynauds disease
e) Lymphatics d) Abdominal mass
366. Bullet wounds near major blood vessels should be 377. One ofthe following is not indicated for arterial
explored only if- (PGI 81, AMC 85) leg ulcer - (PGI 96)
a) The extremity is cold a) Debridement b) Elevation of limb
b) The fingers or toes are paralysed c) Head end of bed is raised d) Low dose aspirin
c) The pulse is weakened 378. Not used as graft material in peripheral vascular
d) There in no pulse disease- (PGI 97)
e) In all cases regardiess of physical findings a) Dacron graft b) Vein
367. A knitted Dacron artery graft- (PGI 99, AIIMS 84) c) PIFE d)PVC
a) Is not porous 379. The commonest cause of arterio-venous fistula is-
b) Is eventually dissolived by tissue reaction a) Penetrating injury (MP 97, AI 88)
c) Never gets infected b) Congenital
d) Can be easily incised and the opening resutured c) Neoplasmic invasion of an artery and adjacent vein
368. The sequence of symptoms in pulmonary embolism d) Aneurysm of the artery eroding a vein
is- (JIPMER 89, DNB 90) 380. AV fistula leads to all except- (AIIMS 98)
a) Fever, pain, dyspnoea a) Sinus tachycardia b) Increased preload
b) Fever, dyspnoea c) Cardiac arrythmias d) Increased cardiac output
c) Dysponea, pain, haemoptysis 381. Commonest peripheral aneurysm is- (SCTIMS 98)
d) Dysponea, cough, purulent sputum a) Popliteal b)Femoral
c) Carotid d) iliac

358)b,c 359)None 360)c 36l)a 362)d 363)b 364)a 365)b,c,e 366)c,d 367)d 368)c 369)c 370)d 37l)b
372)d 373)d 374)b 375)c 376)a 377)b 378)d 379)b 380)c 38l)a
SURGERY[199]

382. Popliteal aneurysm-AU are true except- 395. True statement ofBrerger's desease is/are-(PGJ 04)
a) Presents as a swelling (SCTIMS 98) a) Small and mdeium sized vessels involved
behind the knee b) Commonly involves upper limb than lower limbs
b) Presents with symptoms due to complication c) Common in male
c) Surgery is indicated in case of complication d) Common in female
d) Uncommon among peripheral aneurysm 396. In which one of the following conditions Dactylitis
383. Graft used in infra inguinal by pass is -(Jipmer 2K) CANNOT be seen- (UPSC 02)
a) PTFE b) Dacron a) Sickle- cell anemia b) Beta thalassemia
c) Autologous vein d) Autologous artery c) Congenital syphilis d) Tuberculosis
384. True about Erytbrocyanosis except- (A.P 96) e) Sarcoidosis
a) Affects young girls 397. Complications arising out of A- V fistula done for
b) Cold peripheries renal failure include the following except-
c) Palpable pulses a) Infection (Jipmer 03)
d) Ulceration & gangrene of fingers b) Thrombosis
385. Diabetic gangrene is due to AlE- (TN 86) c) High output cardiac failure
a) Vasospasm d) Necrosis of the distal part
b) Atherosclerosis 398. Not seen in sudden onset peripheral arterial
c) Peripheral neuritis occlusion - (Kerala 04, PGI 98, 2K)
d) Increased sugar in blood a) Pain b)Rubor
386. Pseudo aneurysms are most commonly due- {Jipmer c) Pallor d) Anesthesia
a) Atherosclerosis b) Trauma 93) 399. Drug used for Burger's disease- · (MAHE 05)
c) Congenital deficiency d) Infections a) Xanthinol micotinate b) Propranolol
387. Bilateral pulseless disease in upper limbs in caused c) GIN d) All the above
by- (PGI97)
400. Pseudoarterial aneurysm in drug abuser's seen in-
a) Aortoarteritis b) Coarctation of aorta a) Radial b) Brachial (PGI June 05)
c) Fibromuscular dysplsia d) Buerger's disease c) Femoral d) Carotid
388. AV fistula causes- (PGI 98)
e) Pedal
a) J, Diastolic b) t Venous return
401. Syndrome of internal iliac artery occlusion
c) J, Venous congestion d) t Systolic filling
manifested by- (PGI June 05)
389. Nicoladoni branham sign is- (PGI 98)
a) Pain in calf
a) Compression cause bradycardia
b) Compression cause tachycardia b) Absent pulse at the dorsalis pedis artery
c) Hypotension c) Intermittent claudication
d) Systolic filling d) Gangrene
390. Abdominal Aneurysm is characterized by all 402. The most common cause of peripheral limb
except- (PGI 2000) ischaemia in India is- (AIIMS NOV 05)
a) Elective surgery complication should be < 5% a) Trauma b) Altherosclerosis
b) Emergency surgery complication< 10% c) Buerger's disease d) Takayastu's disease
c) Rarely asymptomatic before rupture 403. Lumbar sympathectomy is of value in the
d) Bigger the size it is more prone to rupture management of- (AI 05)
391. Dissection of which artery is seen in pregnancy- a) Intermittent claudication
a) Carotid artery b) Aorta (PGI 2000) b) Distal ischaemia affecting the skin of the toes
c) Coronary A d) Femoral artery c) Arteriovenous Fistula
392. Buerger's disease affects all except- (PGI 01) d) Back pain
a) Small arteries b) Small veins 404. The most common cause of acquired arteriovenous
c) Medium size arteries d) Multiparity fistuala is- (AI 06)
e) First pregnancy after 30 years a) Bacterial infection b) Fungal infection
393. Buerger's disease is associated with- (PGI 02) c) Blunt trauma d) Penetrating trauma
a) Smoking b) Poor nutrition 405. Etiopathogenesis of diabetic foot include the
c) Alcohol d) Prolonged standing following except- (UPSC 07)
e) Superficial thrombophlebitis a) Myelopathy b) Osteoarthropathy
394. TrueregardingAVfistulais- (PGI02) c) Microangiopathy d) Infection
a) Leads to cardiac failure 406. Fogarty's catheter is used for- (UPSC 07)
b) Causes local gigantism a) Drainage of urinary bladder
c) Can cause ulcers b) Parenteral hyperalimentation
d) Cause excess bleeding on injury c) Removal of embolus from blood vessels
e) Closes spontaneously d) Ureteric catheterisation

382)d 383)c 384)c 385)a 386)b 387) a 388) b 389) a 390) b,c 391) b 392) d,e 393) a,e 394) a,b,c,d
395) a,c 396) b 397)d 398)b 399)a 400) c,d 401) b,c 402) b 403) b 404) d 405) a 406) c
SURGERY [ 200]

407. Pseudoaneurysms in IV drug abusesrs seen 417. Popliteal artery pulsations are difficult to feet because-
commonlyin- (PGI June 07) a) It is not superficial (AI 09)
a) Brachial artery b) Radial artery b) It does not cross prominent bone
c) Femoralartery c) It is not superficial and does not cross prominent bone
408. Treatment of femoral artery naeurysm- d) Its pulsations are weak
a) Ultrasound guided compression of the neck of 418. Intermittent claudication is defined as - (AI 09)
aneurysm (PGI June 07) a) Pain in muscle at rest only
b) Thrombin injection b) Pain in muscle on first step
c) Bypass graft repair c) Pain in muscle on exercise only
409. Which one of the following is not a symptom of d) Pain in muscle on last step
atherosclerotic occlusive disease at the bifurcation 419. A patieutwithcriticallower limb ischemia presents with-
of aorta (Leriche syndrome)? (UPSC-11 08) a) Intermittent claudication (AI 09)
a) Claudication of the buttock and thigh b) Intermittent claudication and gangrene
b) Claudication of the calf c) Rest pain and ischemic ulcers
c) Sexual impotence d) Intermittent claudication and ischemic ulcers
d) Gangrene localized to the feet 420. Lumbar sympathectomy is of value in the
410. A young male presented with dyspnea, bleeding and managementof- (AI09)
petechial hemorrhage in the chest after 2 days a) Intermittent claudication
following fracture shaft of the femur right side. Most b) Distal ischemia affecting the skin of the toes
likely cause is - (UP 08) c) Arteriovenous fistula
a) Air embolism d) Back pain
b) Fat embolism 421. Sympathectomy is indicated in all following
c) Pulmonary thromboembolism conditions except- (AI 09)
d) Amniotic fluid embolism a) Ischemic ulcers b) Intermittent claudication
411. Aortic transaction true about- (PGI June 08) c) Anhidrosis d) Acrocyanosis
a) Most commonly A/w deceleration injury 422. Pseudoclaudication is caused by- (AI 09)
b) High morality a) Femoral artery stenosis
c) Surgery difmitive treatment b) Popliteal artery stenosis
d) Aortography gold standard c) Lumbar canal stenosis
412. Psuedoaneurysm in I.V. drug abusers is commonly d) Radial artery stenosis
found in- (PGI Dec 08) 423. Best material for below inguinal arterial graft is-
a) Radial artery b) Brachial artery a) Saphenous vein graft (upside- down) (AI 09)
c) Femoral artery d) External iliac artery b) PTFE
e) Dorsalis pedis artery c) Dacron
413. Acute vascular ischemia mainfests as- d) Teflon
a) Pulselessness b) Paralysis (PGI Dec 08) 424. Commonest part of subclavian artery to be affected
c) Flushing d) Anaesthesia by stenosis is - (AI 09)
e) Coolness a) First part b) Second part
414. A drug abuser developed pseudoanerysm. Wbat of c) Third part d) Equally affected
the following is/are should include in the treatment 425. The most common site of rupture of abdominal aortic
modalties- (PGI Dec 08) aneurysm is- (AI 09)
a) Ligation of involved vessel & wide surgical a) Laterally into the left retroperitoneum
debridement b) Laterally into the right retroperitoneum
b) Exposure & ligation with subsequent c) Posteriorly into the posterior retroperitoneum
revascularisation d) Anteriorly into the peritoneum (Intraperitonial)
c) Direct interposition graft with synthetic material 426. TrneaboutKasabachsyndrome- (PGI Nov 09)
in groin for revascularisation a) May be due to complication of portwine stain
d) Direct interposition graft with autogenous graft b) Coagulopathy occurs
e) Selective revascularisation using remote c) Due to complication of hemangioma
iliofemoral bypass d) Thrombocytopenia present
415. Most common site of peripheral aneurysm- 427. Most common artery involved in pancreatic
a) Femoral artery b) Radial artery (AIIMS Nov 08) pseudoaneurysm- (PGI Nov 09)
c) Popliteal artery d) Brachial artery a) Gastroduodenal artery
416. The Hunterian Ligature operation is performed b) Inferior pancreatico-duodenal artery
for- (AIIMS Nov 08) c) Gastric artery
a) Aneurysm b) Varicose veins d) Splenic artery
c) A V fistulas d) Acute arterial ischemia e) Hepatic artery

407)c 408)a,b,c 409)d 410)b 411)a,b,c,d 412)a,b,c 413)a,b,d,e 414)a 415)c 416)a 417)c 418)c 419)c
420)b 421)c>b 422)c 423)a 424)a 425)a 426)b,c,d 427)d
SURGERY [ 201 ]

428. Most common cause of abdominal aortic aneurysm AMPUTATION


is- (AI 10)
a) Atherosclerosis b) Trauma 438. Re-implantation time for lower limb is-
c) Syphilis d) Vasculitis a) 6 hrs b) 4 hrs (Kerala 97)
429. The deficiency of all the following factors increases c) 8 hrs d) 10 hrs
the incidence of thrombus formation except- 439. Stump pain is relieved by- (Kerala 97)
a) Lipoprotein A b) Protein-C (DPG 10) a) Continuous tapping over the stump
c) Anti-thrombin III d) Protein-S b) Warming up the stump
430. A patient presents with claudication in both buttocks c) Using steroids
and has impotence. The clinical examination reveals d) Using analgesics
bruit over lower abdomen. What is the clinical 440. For replantation surger, the detached digit or limb
is best preserved in cold - (UPSC 2K)
diagnosis? (UPSCII 10)
a) Glycerol b) Distilled water
a) Bilateral iliac artery occlusion
c) Hypertonic saline d) Isotonic saline
b) Aortoiliac occlusion
441. During reconstruction of an amputated limb which
c) Bilateral iliofemoral occlusion
of the following is done first? (AIIMS Nov 10)
d) Bilateral femoropopliteal occlusion
a) Arterial repair b) Venous repair
431. The normal ankle brachial pressure index (ABPI) c) Fixationofthe bone d) Nerve anastomoses
is 1.0. A value of 0.8 suggests- (UP SC II 10)
a) Goodflow VENOUS DISORDERS
b) Some degree of arterial obstruction.
c) Impending gangrene 442. Earliest sign of deep vein thrombosis is(AIIMS 87)
d) Presence of collaterals a) Calf tenderness b) Rise in temperature
432. True aboutischemic rest pain- (PGI May 10) c) Swelling of calf muscle d) Homan's sign
a) More in night 443. Injection sclerotherapy for varicose veins is by
b) MC in calf muscle using- (PGI 88)
c) Increse upon elevation of limbs a) Phenol b) Absolute alcohol
d) Relieved by dependent position c) 70% alcohol d) Ethanolamine oleate
e) Often associated with trophic changes 444. White leg is due to- (TN 90)
433. All are true about intermittent claudication except- a) Femoral vein thrombosis and lymphatic obstruction
a) Most common in calf muscle (PGI May 10) b) Deep femoral vein thrombosis
b) Pain is positional c) Lymphatic obstruction only
c) Atherosclerosis is important predisposing factor d) None of the above
d) Relieved by rest 445. All of the following are seen in deep vein thrombosis
434. True regarding management of diabetic foot- except- (AI 90)
a) Strict diabetic control (PGI May 10) a) Pain b) Discolouration
b) Venous system is commonly involved c) Swelling d) Claudication
c) Topical antibiotics are used 446. The following is the commonest site for venous
ulcer- (AIIMS 91)
d) Early amputatio should done
a) Instep of foot b) Lower 113 leg and ankle
e) Diabetic ulcers are tropic ulcers
c) Lower 2/3 ofleg d) middle 113 ofleg
435. When a patient suffers from critical limb ischemia,
447. The most important perforator of the Lower limb
the ankle-brachial pressure index (ABPI) is less than-
is between- (AI 92)
a) 1.0 b)0.9 (UPSC II 11) a) Long saphenous and posterior tibial vein
c) 0.7 d)0.3 b) Short saphenous and posterior tibial vein
436. Which is NOT true about femoral artery c) Short saphenous and popliteal vein
cannulation- (AJIMS May 11) d) Long saphenous and femoral vein
a) Common femoral artery is cannulated 448. Pulsating varicose vein in a young adult is due to-
b) Single wall puncture is indicated in those with a) Arteriovenous fistula (AIIMS 92)
normal coagulation profile b) Sapheno femoral incompetence
c) Femoral artery is catheterised at medial third of c) Deep vein thrombosis
femoral head d) Abdominal tumour
d) Seldinger technique is used both for femoral artery 449. Which is not used in treatment of Superficial venous
and vein thrombosis- (AIIMS 92)
,~~~¢#~~~:!1'-l~~', a) Immediate anticoagulation
b) Rest and elevation
c) Analgesics
d) Treat assosiated malignancy

428)a 429)a 430)b 43l)b 432)All 433) b 434)a,c,e 435)d 436)b 437)b 438)c 439) None 440) d
44l)c 442)a 443)d 444)b 445)d 446)b 447)d 448)a 449)a,b
SURGERY [ 202]

450. Most common complication of varicose vein 461. Which of the following test is used to detect
strippingis- (JIPMER 78, AIIMS 79,92) perforator incompetence in varicose- (JIPMER 2K)
a) Infection b) Haemorrhage a) Trendelenberg test b) Fegan's test
c) Ecchymosis d) Thromboembolism c) Morissey's test d) Homan's test
451. An operated case of varicose veins has a 462. The deficiency of all of the following factors increases
recurrence rate of- (AIIMS 80, AP 89) the incidence of thrombus formation except -(UPSC
a) About 10% b) About 25% a) LipoproteinA b)Protein-C 2K)
c) About 50% d) Over 60% c) Anti - thrombin III d) Protein - S
452. What is acceptable in the management offemoral 463. The most common vein to get thrombosed is
vein thrombosis- (AIIMS81, PG/86) the- (AIIMS99)
a) Bed rest and spiral elastic bandages a) Long saphenous b) Short saphenous
b) A venogram c) Both d) Posterior tibial
c) Thrombectomy 464. Brodie-Trendlenburg test demonstrates-
d) Embolectomy a) Mid -thigh perforation (ORR/SA 98)
e) A mob in udin umbrella inserted into the vein b) Deep vein thrombosis
453. Operations for varicose veins are best c) Sapheno --femoral incompetence
accomplished by- (PG/81, AIIMS 84, 86) d) Calf perforators
a) Stripping 465. An intern was doing saphenous cannulation for a
b) Multiple subcutaneos ligatures burns pareint. Then the jpatient developed sudden
c) Subfascialligatures onset of pain along the medial border of the
d) Division and ligation at the superficial venous correponding foot. Which nerve must have been
system accidentally ligated- (AIIMS 2K)
454. A 60-years old male has been operated for carcinoma a) Suralnerve b)Deepperonealnerve
of caecum and right hemicolectomy has been done. c) Saphenous nerve d) Genicular nerve
On the fourth post- oprative day, the patient develops 466. In DVT all are seen except (CMC 2001)
fever and pain in the legs. The most important a) High fever b) Increased temperature at site
clinical entity one should lookfor is- (UPSC 96) c) Pain d) Tenderness
a) Urinary tract infection 467. An obese patient develops acute oedematous lower
b) Intravenous line infection limb following a Pelvic surgery. Deep vein
c) Chest infection thrombosis is suspected . The most useful
d) Deep vein thrombosis investigation in this case would be- (UPSC 2002,
455. All of following may be predisposing factors for a) Doppler imaging Jipmer 98, 92, PG/97,
deep vein thrombosis except- (AIIMS 95) b) Fibrinogen uptake AIIMS 92}
a) Oral contrceptives b) Nephrotic syndrome c) Venography
c) Sickle cell anemia d) Thrombocytosis d) Plethysmography
456. The duration of heparin therapy in deep vein 468. In a patient on anticoagulant therapy, the INR is
thrombosis is- (CUPGEE 96) maintainedat- (UPSC2002)
a) 7- 10 days b) 15-20 days a) 1.5 to 2.5 times the normal
c) 3-4 days d) 1 month b) 2.5 to 3.5 times the normal
457. Cocket&Dodd'soperationisfor- (AP96) c) 3.5to4.5timesthenormal
a) Saphenofemoral flush ligation d) 4.5 to 5.5 times the normal
b) Subfascialligation 469. In diabetic ulcer, following site is involved-(PG/97)
c) Deep vein thrombosis a) Heel b) Head of metatarsal
d) Diabetic foot c) Webs d) Tips of toes
458. In obstruction of inferior vena cava there is -(A/97) 470. For prophylaxis of deep vein thrombosis used is-
a) Prominent thoraco epigastric vein a) Warfarin (PG/97)
b) Caput medusa b) Heparin
c) Hemorrhoids c) Pneumatic shock garment
d) Esophageal varices d) Graded stocking
459. Most common site for venons thrombosis -(JIPMER 471. Which is true regarding Trendelenburg
a) Popliteal vein b) Soleal vein 98) operation- (PG/01)
c) Femoral vein d) Internal iliac vein a) Sripping of the superficial varicose vein
460. Deep vein thrombosis is cansed by all except- b) Flush ligation of the superficial varicose vein
a) Lower limb trauma (AIIMS 98) c) Ligation of the perforators
b) Hip and pelvic surgery d) Ligation of small tributaries at the distal end of
c) Subungual melanoma superficial varicose vein
d) Cushing's syndrome e) Ligation of short saphenous vein

450)c 451)a 452) a,c 453) a 454)d 455) c 456) a 457) b 458) a 459)b 460)c 461)a 462)a 463)d
464)c 465)c 466)a 467)a 468)b 469)a,b,d 470)b 471)b
SURGERY [ 203]

4 72. For which of the following venous abnormality is b) 90% are on the left side
surgery indicated- (PGI OJ) c) Asymptomatic cases require surgery
a) Deep vein incompetence with DVT d) Femoral catheterization with spermatic vien
b) Deep vein incompetence without DVT ablation is done in recurrence
c) VAricosity> 3 em 482. Which one ofthe following is the investigation of
b) Varicosity< 3 em choice for suspected deep vein thrombosis of the
e) Saphenofemoral incompetence lower extremity? (UPSC 07)
473. Migratory tbrombopblebitis is seen most commonly a) Radioactive labelled fibrinogen uptake
with- (PGI 02) b) Ascending contrast phlebography
a) Pancreatic ca b) Testicular ca c) D-dimer estimation
c) Gastric ca d) Breast ca
d) Duplex ultrasonography
e) Liverca
483. With reference to varicocele, which one of the
47 4. Brodie-Tredenlenburg test is positive in- (PGI 02)
following is not true ofit? (UPSC 07)
a) Sapheno-Femoral incompetence
b) Perforator competence above knee a) Varicosity of cremestric veins
c) Deep vein incompetence b) Left side is affected usually
d) Perforator competence below knee c) Feels like a bag of worms
e) svr d) May lead to infertility
475. Varicose veins are seen in- (PGI 02) 484. All are done for a case ofdeep vein thrombosis except-
a) DVf b) Superficial venous thrombosis a) Thrombolytic therapy b)Bandage (MAHE07)
c) AVfistula d) Prolonged standing c) Heparin d) Bed rest
e) Obesity 485. Cocket & Dodd's operation is performed for which
476. Gold standard diagnostic test in varicose veins is one of the following? (UPSC-II 08)
a) Photoplethysmography (Jipmer 03) a) Sapheno femoral flush ligation
b) Duplex imaging b) Sub fascial ligation
c) Ultrasonography c) Deep vein thrombosis
d) Radio - labeled fibrinogen study d) Diabetic foot
477. The initial therapy of documented deep venous 486. Which of the following conditions disappear
thrombosis in a post operative case is - spontaneously in tlrst year of life? (APPG 08)
a) Subcutaneous heparin therapy (Karnataka 03) a) Port wine stain b) Naevus flammeus
b) Intravenous heparin therapy c) Salmon's patch d) Strawberryhaemangioma
c) Thropmbolytic therapy with urokinase 487. The most common cause of superficial
d) Aspirin therapy thrombopblebitis is- (AJ 09)
478. Which one ofthe following reagents is not used as a) Intravenous catheters/infusion
sclerosant in the treatment of bleeding varices-
b)DVf
a) Ethyl alcohol b) Ethanolamine oleate (ICS 05)
o) Varicose veins
c) Phenol d) Sodiummorrhuate
d) Trauma
479. Siteofdiabeticforulcer- (PGIJune05)
a) Medial malleolus b) Lateml malleolus 488. 'SEPS' is a procedure used for- (AI 09)
c) Heel d) Head of metatarsal a) Veins b) Arteries
e) Head of toes c) Lymphatics d)AVfistula
480. Which ofthe following statements is true regarding 489. Under normal conditions venous retllling occurring
fatembolism- (AJIMSNov05) through arterial inflow alone takes _ _ __
a) Most patients with major trauma involving long when the limb at rest- (Maharashtra 10)
bones have urinary fat globules a) 5-7 seconds b) 8-10 seconds
b) All patients with urinary fat globules develop fat c) 11-15 seconds d) 20-30 seconds
embolism 490. Trendelenburg's operation is done for-
c) Peak incidence of respiratory insufficiency for a) Varicocele (UP SC Ill 0)
pulmonary fat embolism is around day 7 after b) Primary varicose veins
injury c) Deep vein thrombosis with varicose veins
d) Heparin as an anticoagulant decreases mortality d) Arteriovenous fistula
and morbidity in fat embolism syndrome 491. True about venous ulcer- (PGI May 10)
481. Which of the following is true about varicocele a) Always stripping done
except? (Manipal 06) b) Always examine deep venous system
a) Incompetent valves of testicular vein are c) Biopsy should be taken from chronic ulcer
responsible for varicocele d) A/w Klippel- Trenaunary syndrome

472)b,c,e 473)a 474)a 475)a,c,d,e 476)b 477)b 478)a 479)c,d,e 480)a 48l)c 482)d 483)a 484)d
485)b 486)c 487)a 488)a 489)d 490)b 49I)b,c,d
SURGERY [ 204]

492. Match List I with List ll and select the correct answer 502. Finding the cause of unilaterallympoedema ofthe
using the code given below the Lists- (UPSC II 11) leg includes- (JIPMER 78, 79, PGI 85)
Listl Listll a) Taking a family history
(Name ofthe perforators) (Site of Perforators) b) Looking for chronic infection in the foot
a) Boyd's 1. 5 em above the c) Looking for early malignant disease of the testis
calcaneus d) Looking for filariasis
b) Dodd's 2.Midthigh e) Performing a casoni test
c) Cockett's 3. Below the knee 503. Treatment of Acute lymphangitis requires-
d) Bassi's 4. Above the ankle a) Antibiotic and rest (JIPMER 81, AMC 84)
Code: b) Immediate lymphangiography
a) 4 3 1 2 c) Immediate multiple incisions
b) 3 2 1 4 d) No special treatment
c) 3 2 4 1 504. Total dose of radiation in Hodgkins dosease
d) 4 3 2 1 is- (JIPMER95)
a) 500-lOOOrad b) 1000-2000rad
LYMPHATIC SYSTEM c) 3000- 5000 rad d) 5000-7000 rad
505. The most important prognostic indicator of
493. Malignant ceO in Hodgkin's lymphoma is- (AIIMS Hodgkin'slymphomais- (TN95)
a) Reed sternberg cell b) Lymphocytes 85) a) Lymphocytic predominance histology
c) Histiocyte d) Reticulum cells b) Visceral involvement
494. Chronic lymphedema predisposes to all c) Hepatic involvement
except- (PGI 89) d) Involvement of spleen
a) Lymphangiosarcoma b) Marjolins ulcer 506. Diagnosis of Hodgkin's disease is confirmed by-
c) Recurrent infections d) Thickening of skin a) CT scan (PGI 97)
495. Commonest cause of unilareral pedal edema in b) Bone marrow biopsy
india is- (AI 90) c) Lymph node biopsy
a) Filariasis b) Post traumatic d) Lymphangiography
c) Post irradiation d) Milroy's disease 507. Gradellymphedemameans- (JIPMER2K)
496. All are true about congenital lymphedema except- a) Pitting edema upto the ankle
a) It is bilateral (AI 91) b) Pitting edema upto the knee
b) Involve lower limb c) Non-pitting edema
c) Almost always manifests before puberty d) Edema disapearing after overnight rest
d) Acute lymphangitis may occur 508. Lymphovenous anastomosis is done for- (PGI 97)
497. The commonest cause for lymphedema of upper limb a) Filarial lymphoedema
is- (AI 91, 92) b) Lymphoid cyst
a) Filariasis b) Congenital c) Cystic hygroma
c) Neck surgery d) Post mastectomy irradiation d) Malignant lymphoedema
498. Milroys disease is- (JIPMER 92) 509. Trueaboutlymphangioma- (PGI03)
a) Edema due to filareasis a) It is a malignant tumour
b) Post cellulitic lymphedema b) It is a congenital sequestration of lymphatic
c) Congenital lymphedema c) Cystic hygroma is a lymphangioma
d) Lymphedema following surgery d) Laser excision is done
499. Investigation of choice in detecting smaU para-aortic e) Sclerotherapy is commonly done
lymph node is- (JIPMER 92) 510. The most common site oflymphangiosarcoma is
a) Ultra sound scan b) CT scan a) Liver (UPSC04)
c) Lymphangiography d)Arteriography b) Spleen
500. Milroys disease is lymphedema which is-(AMU 85) c) Post mastectomy edema of arm
a) Familial b) Follows filariasis d) Retroperitoenum
c) Follows erysipelas d) A sequele to white leg 511. Commonest cause ofA/C Lymphadenitis in India-
501. Lymphangiography of the leg is performed by- a) Barefoot walking (MARE 05)
a) An injection of sodium diatrizoate (Hypaque) b)lB
subcutaneously between the toes c) Staphylococcal skin infection
b) Injecting sodium diatrizoate retrogradely under d) Lymphoma
pressure into a small vein on the dorsum of the foot 512. All of the following soft tissue sarcomas have a
c) Dissecting lymphatics through an incision on the propensity for lymphatic spread except-(AiimsNov 05)
dorsum of the foot a) Neurofibrosarcoma b) Synovial sarcoma
d) The use of an infusion pump c) Rhabdomyosarcoma d) Epitheloid sarcoma

492) c 493) a 494) b 495) a 496) None 497) a 498) c 499) b 500) a 501) c 502) a,b,d 503) a 504) c
505)None 506)c 507)d 508)a 509)b,c 510)c 511)c 512)a
SURGERY [ 205]

513. A 40 year old man presented with a flat 1 em x 1 em 522. Initial treatment in a management oftrauma-
scaly, itchy black mole on the front of thigh. a) Airway b) IV fluids (PGI 96)
Examination did reveal any inguinal lymphadenopathy. c) Fracture stabilisation d) Do not shift
The best course of management would be- 523. Death in blunt trauma chest is due to- (AIIMS 98)
a) FNAC of the lesion (UPSC 07) a) Rupture oesophagus
b) Incision biopsy b) Tracheobronchial injury
c) Excisional biopsy c) Pulmonary contusions
d) Wide excision with inguinal lymphadenectomy d) Chylothorax
514. In India, what is the most common cause of unilateral 524. A 12 hour old bullet injury to the left colon is ideally
lymphoedema oflower limb? (UPSC 07)
treated by- (AIIMS 99)
a) Primary closure without drainage
a) Lymphoedema tarda
b) Primary closure with drainage
b) Carcinoma of penis with metastatic nodes
c) Resection of affected segment with upper segment
c) Filariasis
colostomy and lower segment as a mucus fistula
d) Tubercular lymphadenopathy d) Primary repair with loop colostomy
515. Lympho-venous anastomosis is done for- 525. What is the management of a case of bullet injury to
a) Filariallymphedema (UPSC-11 09) left side of the colon presenting at 12 hours after
b) Lymphoid cyst incident- (AIIMS 2K)
c) Cystic hygroma a) Primary closure
d) Malignant lymphedema b) Proximal colostomy and bringing out the distal
516. All are true about cystic hygroma except- (DPG 10) part as a mucus fistula
a) Pulsatile c) Resection and primary anastomosis
b) May cause respiratory obstruction d) Proximal defunctioning colostomy
c) Common in neck 526. Which one of the following veins should be avoided
d) Present in birth for intravenous infusion in the managment of
517. Trueaboutprimarylumphoedema- (PG!Nov.JO) abdominal trauma- (UPSC 2001)
a) Lymphangiosarcoma may occur a) Cubital b) Cephalic
b) AIW Milroy's disease c) Long saphenous d)Extemaljugular
c) Onset b/w 2-35 years indicates lymphoedema 527. Which one of the following is not a part of the
tarda Revised Trauma score- (UPSC 2001)
d) Onset> 35 years indicates praecox variety a) Glasgow coma scale
e) Prevalence is 2% b) Systolic blood pressure
c) Pulse rate
TRAUMA d) Respiratory rate
528. In severe injury, first to be maintained is -(PGI 97)
518. Treatment of choice for stab injury caecum- a) Hypotension b) Dehydration
a) Caecostomy (AI 89) c) Airway d) Cardiac status
b) Ilea-transverse anastomosis 529. MC abdominal organ injured in blunt trauma
c) Transverse colostomy abdomen is- (PGI 99)
a) Spleen b) Liver
d) Sigmoid colostomy
c) Pancreas d) Stomach
519. Blunt injuries to the abdomen- (JIPMER 82,80)
530. Investigation of choice for blunt trauma abdomen in
a) May cause peritonitis
instable patient- (PGI 2000, AIIMS 87)
b) Rarely need urgent laparotomy a) X-rayabdomen b)USG
c) May cause intestinal obstruction c) Diagnostic Peritoneal lavage d) MRI
d) May cause gastroduodenal ulceration e) CT scan
520. Haemostasis in scalp wound is best achived by- 531. A 30 year old female comes with hypovolemic shock
a) Direct presure over the wound (AIIMS 79, after blunt trauma of the abdomen. An emergency
b) Catching and crushing the PGI 85) USG of abdomen shows splenic tear. Which of the
bleeders by haemostats following is to be done- (PGI OJ)
c) Eversion of galea aponeurotica a) CECT of the abdomen
d) Coagulation ofbleeders b) Diagnostic lavage of peritoneal cavity before
521. In penetrating injury of abdomen commonly proceeding
affected is - (AIIMS 94) c) Monitor patient to assess for progression
a) Liver b) Large bowel d) Immediate surgery
c) Duodenum d) Small intestines e) Chest X-ray

513)c 514)c 515)a 516)a 517)a,b 518)b 519)b 520)a 52l)d 522)a 523)b 524)c 525)b 526)d
527)c 528)c 529)a 530)b 53l)d
SURGERY [ 206]

532. In traumatic transection ofthe femoral artery and d) Splenectomy


vein, which among the following should be done- e) X-ray abdomen and aspiration
a) Femoral artery repair with vein ligation (PGI 01) 540. A person following a road-traffic accident presented
b) Repair of artery and vein emergency with lacreration of iliac On examination
c) Ligation of femoral artery there was swelling of inguinal region and distal
d) Below knee amputation pulsation was felt; internal iliac artery was normal,
e) Repair of artery with contralateral sympathectomy common iliac and external femoral artery was
533. A patient sudden headache and vomiting & normal but common femoral vein is transected.
unconciousness, diagnosis is- (PGI OJ) Treatment of choice is- (PGJ 03)
a) Subarachnoid h'ge b)lntracerebralh'ge a) Vein repair with continuity
c) Subdural h' ge d) Extradural h'ge b) Sclerotherapy
534. Commonly injured in blunt abdominal injury is- c) Ligation offemoral artery and vein
a) Midileum b)Prox.jejunum (PGIOJ) d) Amputation below knee
c) Midjejnum d)Distalileum 541. Which one of the following is not a principle
e) Ileocecaljunction followed in the management of missile injuries-
535. In traumatic injury to common femoral vein & a) Excision of all dead muscles ( UPSC 04)
external femoral artery, which among the following b) Removal of foreign bodies
shouldbedone- (PGJ03) c) Removal of fragments of bone
a) Ligation of both artery & vein d) Leaving the wound open
b) Repair of artery & vein 542. Seat belt causes injury to- (Manipal 04)
c) Below knee amputation a) Duodenum
d) Repair of artery and contralateral sympathectomy b) Head injury due to wind screen
e) Sclerotherapy c) Thorax d) All
536. After sustaining trauma, a pt. developed persistent 543. All of the following are used for gradation of coma is
hypotension with CVP 16 mm H 20 with BP 90/60 'Glasgow coma scale' except- (SGPGJ 05)
mmHg & pulse 140/min, diagnosis is- (PGJ 03) a) Eye opening b) Motor response
a) Cardiac tamponade b) Pericardia! effusion c) Verbal response d) Heart rate
c) Tension pneumothorax d) Lung laceration 544. Mild head injury is having glassgow coma scale of-
e) Puhrtonary embolism a) 3-5 b) 5-8 (SGPGJ05)
537. Apt developed haemoperitoueum following RTA, c) 8-10 d) 10- 15
with BP 90/60 & pulse 140/min which of the 545. According to the Glasgow Coma Scale (GCS), a
following to be done- (PGJ 03) verbal score of !indicates- (AI 05)
a) DPL to be done a) No response
b) Liver is the MC organ to rupture b) Inappropriate words
c) USG is better than CT Scan c) Incomprehensible sounds
d) X-ray to be taken in supine position d) Disoriented response
e) Urgent surgery to be done 546. Which of the following is nota componentofGlasgow
538. A pt. after sustaining RTA, developed# left shaft of Coma Scale? (AI 06)
femur with guarding & rigidity in the abdomen. a) Eye opening b) Motor response
Followingisto be done- (PGJ03} c) Pupil size d) Verbal response
a) X-ray ofleft lower limb & USG abdomen 547. Total score in Glasgow Coma Scale of a conscious
b) Start IVF, Ryle's tube & catheterization person is- (AI 06)
c) Stabilize the # & monitor the pt and do surgery if a) 8 b) 3
necessary later on c) 15 d) 10
d) Grouping & cross matching of two unit of blood 548. Protein metabolism after trauma is characterized
e) Stabilize the# only bythefollowingexcept- (UPSC07)
539. A 30 yrs old gentleman after sustaining road traffic a) Increased liver gluconeogenesis
accident present in emergency with BP 100/60 b) Inhibition of skeletal muscle breakdown by
mmHg, Pulse 120 min and C.T. shows splenic interleukin I and tumour necrosis factor
laceration at inferior border. after 2 units of blood c) Increased urinary nitrogen loss
transfusion, patients conditions are - BP 120170 d) Hepatic synthesis of acute phase reactants
mmHg and pulse 84/min; the next line of 549. Abdominal compartment syndrome is characterized
managementis- (PGJ03, JCS98) bythefollowingexcept- (UPSC 07)
a) Laparotomy a) Hypercarbia and respiratory acidosis
b) Splenorrhaphy b) Hypoxia due to increased peak inspiratory pressure
c) Continue the conservative treatment and take c) Hypotension due to decrease in venous return
subsequent measures on monitoring the pt. d) Oliguria due to uretic obstruction

532) a,b 533) a 534) b,e 535) b 536) a,c 537) c,e 538) a,b,d 539) c 540) a 541) c 542) a 543)d 544)d 545)a
546)c 547)c 548)b 549)d
SURGERY [ 207]

550. Preferred incision for abdominal exploration in 559. True aboutabdominal compartment syndrome-
Blunt injury abdomen is- (AI 07) a) .J, Cardiac output (PGI Dec 08)
a) Always Midline incision b) .J, Urine output
b) Depending upon the organ c) .J, Pulmonary capillary wedge pressure
c) Transverse incision d) .J, Venous return
d) Paramedian e) .J, Systemic vascular resistance
551. A patient sustained Traumatic injury to major 560. Trueaboutdiaphragmaticinjury- (PGI Dec 08)
abdominal vessels. It has been planned to explore a) Advise diagnostic laproscopy
the suprarenal Aorta, the Caeliac Axis, the superior b) Chest X-ray is useful
mesentric artery, and the left renal artery. What c) Conservative management is done in most cases
maneurve for exposure is recommended- (AI 07) d) Late complication is herniation ofabdominal content
a) Cranial visceral Rotation e) All penetrating diaphragmatic injury must be
b) Caudal visceral Rotation repaired via chest
c) Left medial visceral Rotation 561. A person after pelvic fracture, could not pass urine.
d) Right medical visceral Rotation On examination bladder is not palpable. Wbat is
552. Best prognostic factor for head injury is- (AI 07) probable diagnosis- (PGI Dec 08)
a) Glasgowcomascale b)Age a) Posterior urethra repture with retension of urine
c) Mode ofinjury d) CT b) Rectourethral injury
553. Base of the skul fractrue presents with c) Intraperitoneal rupture of bladder
invovlement of the petrous temporal bone, which d) Extraperitoneal rupture ofbladder
ofthefollowingimportantsign- (UP 07) 562. A man comes to emergency with stab injury to left
a) Subconjuctive haemotoma Dank. He has stable vitals. Wbat would be the next
b) CSF rhinorrhoea step in management- (AIIMS Nov 08)
c) Raccon eyes a) CECf b) Diagnostic peritoneallavage
d) Battle sign c) Laprotomy d) Laproscopy
554. Diagnosis of traumatic rupture of diaphragm- 563. A man with blunt injury abdomen after road side
a) Laparoscopy (PGI June 07) accident has a blood pressure ofl00/80 mm Hg and
b) Chest X ray a pulse rate of120 hpm.Airway has been established
c) Diagnostic peritoneal lavage and respiration has been stabilized. Next best step
555. What is the first step taken in case of multiple in managementis- (AI 09)
injuries offace and neck? (UPSC-II 08) a) Immediate blood transfusion
a) Blood transfusion b) IV fluids b) Blood for cross matching & IV fluids
c) Reconstruction d) Maintenance of airways c) Ventilate the patient
556. A male patient with blunt trauma abdomen is d) Rush the patient to the OT
hemodynamically stable. What is the next line of 564. Which of the following is used to def'me Penetrating
management- (AI 08) Necklnjury? (AIIMSMay09)
a) Observation a) 2 em depth of wound
b) Further imaging of abdomen b) Injury to vital structures
c) Exploratory laparotomy c) Breach of platysma
d) Laproscopy d) Through and through wound
557. A young patient presents with a massive injury to 565. Glasgow coma scale include- (PGI June 09)
proximal duodenum, head of pancreas and distal a) Eye opening b) Verbal response
common bile duct. The procedure of choice in this c) Motor response d) Sensory response
patient should be- (AI 08) e) Swallowingreflex
a) Roux-en- Y anastonmosis 566. True regarding presentation (s) of thoracic duct
b) Pancreaticoduodenectomy (Whipple's operation) injury- (PGI June 09)
c) Lateral tube jejunostomy a) Electrolyte disbalance b) Lymphedema
d) Retrograde jejunostomy c) Lymphopenia d) Dehydration
558. True about blunt abdominal trauma with splenic e) Chylothorax may be present
rupture- (PGI June 08) 567. True about indication of celliotomy in blunt trauma-
a) Kehr's sign-discoloration around umplicus a) Peritoneal air on imaging (PGI June 09)
b) Spleen is most common organ to be involved b) Severe hypotension
c) Splenectomy is treatment of choice for splenic c) Grade I spleen damage
rupture d) Grade IT liver damage
d) Cullen's sign seen e) patient with positive diagnostic lavage

550)a 551)c 552)a 553)d 554)a,b 555)d 556)b 557)b 558)b 559) a,b,d 560) a,b,d 561) c,d 562)a
563)b 564)c 565) a,b,c 566) a,c,d,e 567) a,b,c
SURGERY [ 208]

568. What is the investigation of choice in a patient with 577. A man with blunt abdominal trauma with h/o pelvic
blunt abdominal trauma with hematuria- fracture has presented to ER. He has passed only
a) Ultrasonography of abdomen (UPSC-II 09) few drops of blood per meatus and no urine in the
b) Contrast enhanced tomography past 8 hrs. His bladder is palpable per abdomen.
c) Intravenous urogram Which of the following is correct? (AIIMS May 10)
d) Retrograde urogram a) Anuria due to hemorrhagic shock
569. Traumatic diaphragmatic injury except- b) Urethral injury
a) Left side rupture d/t weak left hemidiaphragm at c) Ureteral rupture leading to extravasation of urine
point of entry of embryonic origin (PGI Nov 09) in retroperitoneum
b) Most commonly due to trauma d) Bladder rupture
c) Smaller tear -heal spontaneously & surgery is 578. According to the Bismuth/Strasberg classification
not required 'Cystic blow out' is classified as- (AI 10)
d) Abdominal approach is the most favoured a) TYPEA b)TYPEB
e) Mask ventilation is encouraged in pt. with c) TYPEC d)TYPED
massive visceral herniation 579. Trauma and Injury Severity score (IRISS) includes-
570. All are true about Flail chest, except- a) GCS+BP+RR b)RTS+ISS+Age (AI 10)
a) Fracture ofatleast 3 ribs (AIIMSNov 09) c) RTS+ISS+GCS d)RTS+GCS+BP
b) If overlapping of fractured ribs with severe 580. A 27 year old patient presented with left sided
displacement is seen then patients are treated abdominal pain to the emergency room; 6 hours
surgically with open reduction and fixation after an RTA. He was hemodynamicaly stable and
c) Pa02<40 with Fi02>60 treated with intubation and FAST postive. Contrast Enhanced CT (CECT) scan
PEEP showed grade III splenic laceration. What will be
d) Paradoxical movement may not be seen in the most appropriate treatment- (AI 10)
conscious patients a) Splenectomy
571. In a patient with head injury, eye opening is seen b) Splenorrhaphy
with painful stimulus, localizes the pain and there c) Splenic artery emblization
is inappropriate verbal response. What would be the d) Conservative management
score on Glasgow coma scale- (AIIMS Nov 09) 581. A 27 year old patient presented with left sided
a) 8 b) 9 abdominal pain to the emergency room, 6 hours after
c) 10 d) 11 an RTA. He is hemodynamically stable and FAST
572. Blunt injury abdomen, patient was hemodynamically positive. A CECT (contrast enhanced CT) scan shows
stable, next investigation- (AIIMS Nov 09) a contrast blush along with a grade III laceration.
a) X -ray abdomen b) Barium swallow What will be the most appropriate management-
c) FAST d)DPL a) Splenectomy (AI 10)
573. Commonest organ injured in blast injury is- b) Spenorrhaphy
a)Lung b)Liver (AIIMSMay10) c) Splenic artery embolization
c) Spleen d) Pancreas d) Conservative management
574. In blunt trauma abdomen what should be the approach 582. A lady presented in the emergency department with
fordoing laparotomy- (AIIMSMay 10) a stab injury to the left side of the abdomen. She was
a) Depends on organ injured hemodynamically stable and a contrast enhanced CT
b) Always midline incision scan revealed a laceration in spleen. Laparoscopy
c) Always transverse was planned however the patient's p02 suddenly
d) Depends upon type of injury dropped as soon as the pneumoperitoneum was
575. Best prognostic factor for head injury is- created. Whatisthemostlikelycause- (Al10)
a) Glassgow coma scale (AIIMS May 10) a) Gaseous embolism through splenic vessels
b) Age b) Injury to the left lobe ofthe diaphragm
c) Mode of injury c) Inferior vena cava compression
d) CT findings d) Injury to colon
576. In order to expose the coeliac axis, left renal artery, 583. Which of the following grading methods is used to
superior mesenteric artery and abdominal aorta in evaluate the prognosis/outcome after subarachnoid
a case of trauma, which of the following is performed- hemorrhage- (AI 10)
a) Left medial visceral rotation (AIIMS May 1 0) a) Glassgow coma scale
b) Right medial visceral rotation b) Hess and Hunt scale
c) Cranial visceral rotation c) Glasgow-Blatchford bleeding score
d) Caudal visceral rotation d) Intracerebral hemorrhage score

568)b 569)c,e 570)d 57l)c 572)c 573)a 574)b 575)a 576)a 577)b 578)a 579)b 580)d 58l)c
582)a 583)b
SURGERY [ 209]

584. Initial treatment in the management of trauma is- 597. In a case with infection in heel space-
a) Aitway maintenance b) IV Fluids (DPG 10) a) The infection is due mostly to infection of pad of
c) Fracture stabilization d) Do not shift fat in the heal (PGI 80, AMU 85)
585. Treatment of Simple rib fracture includes foUowing b) The usual cause is treading of a thorn or any other
except· (Maharashtra 10) sharp pointe object in it
a) Analgesics b) Physiotherapy c) Oedema around the ankle may be present
c) Early ambulation d) Strapping of chest d) Drainage through incision on plantar aspect of
heel should be done
ORTHOPAEDICS 598. Psoas spasm not seen in - (PGI 96)
a) Septic arthritis hip joint b) Lumbar TB
586. Healing is slow below the Knee joint because of· c) TB ofhipjoint d) All ofthe above
a) Poor vascularity (JIPMER 91) 599. Dentigerous cyst arises from- (UPSC)
b) Increased movement a) The root of a caries tooth
c) Weight bearing b) The periosteum of the fractured mandible
d) Decreased subcutaneous Fat c) An unerupted permanent tooth
587. In hand injuries firstto be repaired is- (A/95) d) The sequestrum of osteomyelitis of mandible
a) Bone b) Tendon 600. Porters tip deformily is seen in- (UPSC 89)
c) Muscle d)Nerve a) Klumpke's paralysis b) Radial nerve injury
588. Bony metastasis in children is commonest with • c) Ulnar nerve injury d) Erbs paralysis
a) Wilm's tumour b) Neuroblastoma (PGI 89)
601. Ameloblastoma of mandible most commonly arises
c) Lymphoma d) Rhabdomyosarcoma
from- (AIIMS 2001)
589. Reparative granuloma of Jaw is treated by-
a) Site of second molar b) Above the alveolus
a) Antibiotics (AIIMS 90)
c) Canine d) Last unerupted molar
b) Wedge resection
602. Fatembolismisseenin- (JIPMER 91)
c) Resection and bone grafting
a) Prolonged labour b) Fracture clavicle
d) Curettage
c) Sickle cell anemia d) CVC liver
590. Clickingjawisdueto- (KERALA 90)
603. Adventitious bursa is- (MAHE 98)
a) Lax temporomandibular joint
b) Fracture mandible a) Found normally over any joint
c) Fracture maxilla b) Due to degeneration of connective tissue over a joint
d) Dislocation of symphysis mentii c) Found over bony prominences
591. Commonest site for chondroma in vertebral canalis- d) Can turn into malignancy
a) Thoracic b) Cervical (.JJPMER92) 604. Prolapsed intervertebral disc (PID) is most common
c) Sacro- coccygeal d) Lumbar at- (PGI 2000)
592. Treatment of choice in giant cell reparative a) C5C6 b) D10D11
granuloma is- (PGI 81, AIIMS 9 2) c) C2C3 d) L 3L4
a) Curettage b) Wide excision e) L5S1
c) Excision d) Radiotherapy 605. Prophylactic antibiotic (systemic) therapy is
593. The following bone tumour may cause dural deposits indicated in- (PGI 04)
without causing bony changes -(PGI 79,AIIMS 81,85) a) Hip, knee joint prosthesis
a) Hodgkin's lymphoma b) Multiple myeloma b) Extermities vascular surgery
c) Secondararies d) Fibrous dysplasia c) SmalVLarge bowel surgery
594. Riding Breeches deformity is collection of fat on- 606. Which nerve repair is with good prognosis-
a) The medial aspect of upper arms (PGI 81 a) Ulnar nerve b) Radial nerve (Kerala 03)
b) On the medial thighs AIIMS 84) c) Sciatic nerve d) Median nerve
c) Over the femoral trochanters of the thighs 607. A 64 year old hypertensive obese female was
d) Drooping buttocks undergoing surgery for fracture femur under general
595. What is the percentage ofpatients who have undergone anaesthesia. Intra- operatively her end tidal carbon
amputation which show ''phantom limb" pain- dioxide decreased to 20 from 40 mm ofHg, foUowed
a) About 1% b)About5% (AIIMS80,DNB by hypotension and oxygen saturation of85%. What
c) About 10% d) 35% 90) could bethemostprobablecause- (AI 03)
596. FoUowing rui operation for hallux valgus, the most a) Fat embolism b) Hypovolemia
satisfied patients are those who had- (AIIMS 80,81, c) Bronchospasm d) Myocardial infection
a) An associated hammer toe AMU 89) 608. The movement at the following joint permits a
b) Pain person to look towards the right or left- (AI 04)
c) Metatarsus primus varus a) Atlanto- occipital joint b)Atlanto- axial joint
d) An associated bunionette c) C2 - C3 joint d) C3 - C4 joint

584)a 585)d 586)a 587)a 588)b 589)d 590)a 591)b 592)a 593)a 594)c 595)d 596)b 597)d
598)b 599)c 600)d 601)a 602)None 603)c 604)a,e 605)a,b,c 606)b 607)a 608)b
SURGERY [ 210]

609. The management offat embolism includes all the 619. The commonest extradural tumous is- (NIMHANS
following except- (AI 04) a) Neurofibroma b) Glioma 86)
a) Oxygen c) Meningioma d) Metastasis
b) Heparinization 620. Patient with a history of fall presents weeks later
c) Lowmolecularweightdextran with headache and progressive neurological
d) Direct pressure and elevation deterioration. the diagnosis is- (AI 89)
610. Principles of treatment of Pyogenic infection of a) Acute subdural hemorrhage
skeletal system are- (PGI June 05) b) Extradural hemorrhage
a) Analgesics b) Antibiotics c) Chronic subdural hemorrhage
c) Steroids d) Non-operative d) Fracture skull
e) Chemotherapy 621. Commonestsite ofmeningocele- (AJ89)
611. 'Tennis elbow' is characterized by- (MAliA 05) a) Lumbosacral b) Occipital
a) Tenderness over the medial epicondyle c) Frontal d) Thoracic
b) Tendinitis of common extensor origin 622. In causalgia, the nerve most commonly affected
c) Tendonitis ofcommon flexor origin area- (AIIMS 86, 88)
d) Painful flexion and extension a) Radial and ulnar b) Median and sciatic
612. An 8 year old boy pres~nts with a gradually c) Radial and peroneal d) ilioinguinal and sural
progressing swelling and pain since 6 months over 623. Plexiform neurofibromatosis commonly affects
the upper tibia. On X-ray, there is a lytic lesion with the .... nerve- (PGI 88)
sclerotic margins in the upper tibial metaphysis. a) 7ili b)5ili
The diagnosis is- (MAliA 05) c) 6th d) 8ili
a) Oeteogenic sarcoma b) Osteoclastoma 624. The treatment of post traumatic epilepsy is -
c) Brodie's abscess d) Ewing's sarcoma a) Mannitol infusion (AIIMS 84)
613. Allen's test is useful in evaluating- (AI 06) b) Immediate corticosteroids
a) Thoracic outlet compression c) Long term anticonvulsants
b) Presence of cervical rib d) Long term corticosteroids
c) Integrity ofpalmar arch 625. Following are the features of raised intracranial
blood flow tension except- (AIIMS 84)
a) Altered sensorium b) Papilloedema
c) Convulsions d) Tachycardia
626. Facial nerve palsy is seen in the following fracture
a) Anterior cranial fossa (AIIMS 84)
b) Middle cranial fossa
c) Cranialvault
NERVOUS SYSTEM d) Posterior Cranial Fossa
627. Lowest incidence of cerebral tumours is seen
615. Signs of cerebral compression are all except- in - (AIIMS 85)
a) Bradycardia (UPSC 86, KERALA 87 a) Occipital b) Frontal
b) Hypotension JIPMER 88) c) Temporal d) Parietal
c) Papilloedema 628. Commonest orbital tumour causing exophthalmos
d) Vomiting is- (AIIMS 87)
616. Surgery is not useful in- (JIPMER 81, DNB 89) a) Glioma b) Meningioma
a) Cerebral oedema b) Depressed fracture c) Hemangioma d) Neuroblastoma
c) Extral dural haemorrhage d) Subduralhaemorrllage 629. Musculoskeletal abnormality in neurofibromatos
617. In a patient with head injury black eye associated is - (PGI86)
with subconjunctival haemorrhage occurs when a) Hypertrophy oflimb b) Scoliosis
there is- (UPSC 87) c) Cafe au lait spots d) Pseudo arthrosis
a) Fracture of floor and anterior cranial fossa e) All
b) Bleeding between the skin and galea aponeurotica 630. Dumbell tumour is seen in- (Kerala 89)
c) Haemorrhage between galoea aponeurotica and a) Meningioma b) Neurofibroma
pericranium c) Epidendymoma d) Thymoma
d) Fracture of greater wing of sphenoid bone 631. Scalp hemmorrhage is best controlled by- {TN 89)
618. Plexiform neurofibromatosis commonly affects .•. a) Electrocautery b) Pressure
a) Facial nerve (JIPMER 87) c) Ligation of vessels d) Eversion of galea
b) Trigeminal nerve 632. Tinnels sign indicates- (Kerala 91)
c) Peripheral nerve a) Atrophy of nerves b) Neuroma
d) Glossopharyngeal nerve c) Injury to nerve d) Regeneration of nerves

609)d 610)b 61l)b 612)c 613)c 614)b 615)b 616)a 617)a 618)b 619)d 620)c 62l)a 622)b
623)b 624)c 625)d 626)b 627)a 628)a 629)e 630)b 63l)b 632)d
SURGERY[211]

633. ·The most important clinical finding in a case of 644. Commonest cause of carpal tunnel syndrome is -
headinjuryis- __ (JIPMER91) a) Malunited co lie's fracture (Al95)
a) Pupillary dilatation b) Rheumatoid arthritis involving flexor retinaculum
b) Level of consciousness c) Myxoedema
c) Focal neurological deficit d) Pregnancy
d) Fracture skull 645. An elderly man who has had a trival history of head
634. Management of extradural hemorrhage is- injury three months ago, develops headache and
a) Antibiotics (AJIMS 93) turns unconscious. On examination, he is found to
b) Immediate evacuvation have fixed left dilated pupil and right hemiplegia.
c) Evacuvation after 24 hrs. What is the most likely diagnosis- (UPSC 96)
d) Observation a) Contusio-cerebril
635. Lumbar sympathectomy is done for.: (JIPMER 9 3) b) Extradural hematoma
a) Gangrene foot b) Rest pain c) Chronic subdural hematoma
c) lnterinittent Claudications d) Skin Ulcerations d) Brain abscess
636. Bilateral phrenic nerve palsy is caused by- 646. Sympathectomy is not useful in- (AIIMS 96)
a) Carcinoma bronchus (PGI 80, Delhi 79, 93) a) Causalgia b)Raynaud's disease
b) Polio c) Rest pain d) Intermittent claudication
c). Medullary carcinoma thyroid 647. In Neuroblasotmas the most common presentation
d) paget's disease is- (AI98)
637. The nerve ofKuntzis an important landmark in- a) Lytic lesion in skull with suture diasthesis
a) Lumbar sympathectomy (PGI 79, Delhi 84) b) Lung metastasis
b) Cervicodorsal sympthectomy c) Renal invasion
c) Obturator neurectomy d) Secondaries in brain
d) Splanchnicetomy 648. All produce carpal tunnel syndrome except- (Kerala
e) Hemiormaphy a) Colle's # b) Acromegaly 98)
638. Features ofextradural haemorrhage include all except- c) Addisons disease d) Hypothyroidism
a) Severe hypotension (AJIMS82, AP 87) 649. Most common cuases ofsub-dural haematoma-
b) Deteriorating consciousness a) Middle meningeal artery tear (SCTIMS 98)
c) Fixed dilated pupil on the same side b) Rupture of superior cerebral vein
d) Fracture line crossing the temporal bone c) Internal carotid artery tear
639. A dome shaped skull with a high forehead in the d) None of the above
infancts with slight hydrocephalus (Olympian 650. Commonest spinal tumour is- (SCTIMS 98)
brow) is seen in- (JIPMER 81, AMU 88) a) Meningioma b) Ependynoma
a) Marasmus b) Congenital syphilis c) Neurofibroma d) Neuroblastomas
c) Rickets d) Arnold chiari syndrome 651. In patient of head injuries with rapidly increasing
640. Mter an open injury, the optimum time for nerve intracranial tension without haematoma, the drug
suture is- (AJIMS 78, 79, PGI 85) of choice for initial management would be- (UPSC
a) Immediately b) Within one month a) Lasix b) Steroids 2K)
c) 1-2 month d) 2-4 month c) 20% Mannitol d) Glycine
e) When wound is free from infection 652. Neurofibromatosis presents as all of the following
641. Brain abscess may be due to following- (AJIMS 81, . except- (UPSCOJ)
a) Chronic suppurative otitis media AMC 85) a) Elephantiasis neuromatodes
b) Chronic lung abscess b) Plexiform neuroma
c) Trauma c) Von Recklinghausen's disease
d) Any of the above d) Lymphadenovarix
642. Immediate surgery is indicated in- (Kerala 95) 653. Which of the following is primary neurogenic
a) Extradural b) Subdural tumour - (JIMPER 90)
c) Intracerebral d) Brain laceration a) Meninginoma b) Glioblastoma
643. A one month old female child has swelling over c) Acoustic neuroma d) Neuroblastoma
the back in the sacral region. There is no cough 654. Glioblastoma multiforme may occur in the following
impulse in the swelling. X-ray examination except- (Delhi93)
shows erosion of the coccyx. The most likely clinical a) Cerebrum of adult
diagnosis would be- (UPSC 95) b) Brain stem of child
a) Meningocele b) Lipoma c) Spinalcordofadult
c) Sacro-coccygeal teratoma d) Neurofibroma d) Adrenal medulla of child

633)b 634)b 635) b >d >a 636) b 637) b 638) a 639) b 640) e 641) d 642) a,b 643) c 644) d 645) c 646)d
647)a 648)c 649)b 650)c 651)c 652)d 653)d 654)d
SURGERY [ 212]

655. The parasitic infection capable of producing 668. Most common presentation ofintracranial aneurysm
spinal cord compression is/are -(Al/MS 8l,Delhi 83) is- (PGI98)
a) Leishmaniasis b) Wuchereriasis a) Coarctation of aorta b) Systemic hypertension
c) Echinococcosis d)Amoebiasis c) Hypotension d) Intracranial haemorrhage
656. The following are CNS findings of C0 2 669. Opsomyoclonus is encountered as C/F of -(PGI98)
narcosis - (PGI90) a) Meningioma b) Neuroblastoma
a) Excitement b)IncreasedpHofCSF c) Neurofibromatosis d)Excision
c) Decreased pH of CSF d) Papilledema 670. The CNS tumor present with calcification -(PGJ99)
657. Peripheral nerves can withstand Ischemia upto- a) Oligodendroglioma b) Astrocytoma
a) 30minutes b) 1 hour (JIPMER93)
c) Medulloblastoma d) Phaeochromocytoma
c) 2 hours d) 4 hours
671. Psychiatric symptoms, true except- (PGJ 2000)
658. Commonest intramedullary spinal tumours is-
a) More common with supra than infratentorial tumors
a) Secondaries b)Neurofibroma (PGJ85)
b) More common with slow growing
c) Ependymoma d) None of the above
659. Battle's sign is seen in- (JJPMER 90) c) More with temporal than frontal lobe tumours
a) Fracture middle cranial fossa d) More with brain stem lesions
b) Fracture base of skull 672. TrueaboutHemangiomaofhead &neck-(PGJOJ)
c) Fracture anterior cranial fossa a) Are very common b) Sturge Weber synd
d) All of the above c) High output failure d) Thrombocytopenia
660. Premature filling of veins is a manifestation in 673. Suprasellar calcification with polyuria seen in-
cerebral angiography of- (AJJMS 78, AMU 85) a) Langerhan cell histocytosis (PGJ 02)
a) Trauma b) Medulloblastoma
b) Brain tumour c) Pinealoma
c) Arteriovenous malformation d) Craniopharyngioma
d) Arterial occlusion e) Astrocytoma
661. In normal pressure hydrocephalus seen are all 674. A person developes altered sensorium following
except- (PGI97) headache and vomiting. Diagnosis is- (PGI 02)
a) Convulsion b)Ataxia a) Intracerebral hemorrhage
c) Dementia d) Incontinence b) Subarachnoid hemorrhage
662. Lucid interval is seen in- (PGJ97) c) Hysteria
a) Extra dural haemorrhage d) Botulism
b) Intra dural haemorrhage e) Epilepsy
c) Intra cerebral haemorrhage 675. Neurosurgical treatment of epilepsy usually involves,
d) Subarachnoid haemorrhage
removal of epileptic focus from which lobe- (Kam 03)
663. Esthesio neuroblastoma arises from- (PGI97)
a) Frontallobe b)Temporallobe
a) Olfactory N b) Maxillary N
c) Occipital lobe d) Parietal lobe
c) Opthalmic N d) Nasociliary N
676. All ofthe following conditions are known to cause
664. Earliest feature of sudden increase in ICT in infant
is- (PG197) diabetes insipidus except- (AJIMS 04)
a) Boggy swelling offontanelle a) Multiple sclerosis b) Head injury
b) Papilloedema c) Histiocytosis d) Vrral encephalitis
c) Paralysis 677. Best treatment of subdural haemotoma in a
d) Seizures deteriorating patient- (HPU 05)
665. Neurofibromatosis is associated with - (PGI98) a) By IN Mannitol b) Oxygenation
a) Papillary Ca b) Islet cell tumour c) Use of steroids d) Surgical evacuation
c) Pheochromytorna d) Glucagonoma 678. Which one of the following is the most common site
666. Meningomyelocele patient after being operated ofBerry aneurysm- ( UPSC 05)
developed hydrocephalus due to- (PGI98) a) Vertebro-basilar artery
a) Arnold chiari malformation b) Origin of posterior communicating artery
b) Injury to absorptive surface c) Anterior communication artery
c) Central canal injury d) Anterior choroidal artery
d) Arachnoidal block 679. The most common neurologic abnormality that
667. Extradural haematoma is associated with what% of occurs with head injury is- (Kamat 05)
severetrauma- (PGI98) a) Hemiplegia b) Ocular nerve palsy
a) 36% b) 100/o c) Altered consciousness d) Convulsion
c) 77% d)96%

655)c 656)c 657)None 658)c 659)a,b 660)c 661)a 662)a,b 663)a 664)b 665)c 666)a 667)b 668)d
669)b 670)a 67l)b,d 672)a,b,c 673)d 674)b 675)b 676)a 677)d 678)c 679)c
SURGERY [ 213]

680. Vidian Neurectomy is indicated in- (MAHE 05) 690. Duret haemorrhage is/are seen in- (PGI Dec 08)
a) Glossopharyngeal neuralgia a) Conjunctiva b) Middle ear
b) Trigeminal neuralgia c) Endothelium of heart d) Brain
c) Vasomotorrhinitis e) Lung
d) Atrophic rhinitis 691. A 45 year- male presents with abrupt onset pain,
681. The following methods are used to lower the weakness, loss of contour of shoulder and muscle
intracranial pressure in patients with head injury wasting on sth day of tetanus toxoid immunization.
except- (ICS 05) Likely cause is- (AIIMSMay 09)
a) Hyperventilation b) Administration of Mannitol a) Hysterical
c) Hypothermia d)AdministrationofNifedipine
b) Radial nerve entrapment
682. A person has been brought in casualty with history
c) Brachial plexus neuritis
of road accident. He had lost consciousness
d) Thoracic outlet syndrome
transiently and then gained consciousness but again
692. A patient presents with sudden headache and
became unconscious. Most likely, he is having brain
hemorrhageof- (AIIMSNOV05) vomiting and unconsciousness. The diagnosis is -
-a) Intracerebral b) Sub arachnoid a) Subarachnoid hemorrhage (DELHI PG Feb. 09)
c) Sub dural d) Extra dural b) Intracerebral hemorrhage
683. The earliest manifestation of increased intracranial c) Subdural hemorrhage
pressure following head injury is- (AI 05) d) Extradural hemorrhage
a) Ipsilateral pupillary dilatation 693. In a patient with head injury black eye associated
b) Contralateral pupillary dilatation with subconjuctival hemorrhage occurs when there
c) Altered mental status is- (DELHIPGFeb. 09)
d) Hemiparesis a) Fracture of floor and anterior cranial fossa
684. The most preferred approach for pituitary surgery b) Bleeding between the skin and galea aponeurotica
as the present time is- (AI 06) c) Hemorrhage between galea aponeurotica and peri
a) Transcranial b) Transethmoidal cranium
c) Trans-sphenoidal d) Transcallosal d) Fracture of greater wing of sphenoid bone
685. Which of the following will manifest as 694. In patient of head injuries with rapidly increasing
npatchy meningitis" heamorrhaegica Interna- intracranial tension without hematoma, the drug of
a) Epidural haematoma (Manipal 06) choice for initial management would be -(DELHI PG
b) Subdural haematoma a) Lasix b) Steroids Feb. 09)
c) Subarachonoid haemorrhage c) 20% Mannitol d) Glycine
d) Brain infraction 695. Duret haemorrhage is/are seen in- (PGI Nov. 10)
686. Chronic subdural haematoma refers to collection
a) Conjunctiva b) Middle ear
present for a period of- (Karnataka PGMEE 06)
c) Encardium of heart d) Brain
a) 7 days b) 6 months
e) Lung
c) 1 year d) 21 days
696. Which is an ominous sign in case of severe head
687. Thickened peripheral nerves are seen in all the
following conditions except- injury- (PGI Nov. 10)
a) Hansen's neuropathy a) Development of diabetes insipidus
b) Amyloid polyneuropathy b) Anisocoria
c) Diabetic neuropathy c) New focal deficit
d) Refsum's disease d) Depressed skull fracture
688. Lcid interval is classically seen in- (Corned 07) e) Decorticate posturing
a) Intracerebral haematoma 697. About cranial trauma false is- (AIIMS Nov 10)
b) Acute subdural haematoma a) Raccoon eyes seen in subgaleal hemorrhage
c) Chronic subdural haematoma b) Depressed skull # is associated with brain injury
d) Extradural haematoma at the immediate area of impact
689. A20yearoldmalecometocasualitywithheadinjury. c) Carticocavernous fistula occur in base skull#
Examination reveals normal counsciousness, d) Post traumatic epilepsy seen in 15%
clinically normal and blood in the tympanic 698. A patient with head injury opens eyes of painful
membrane. Most likely cause is- (UP 08) stimulus, used inappropriate words, and localizes
a) Extradural haemorrhage pain.Whatis his GCS score? (AI 11)
b) Sudural haemorrhage a)8 b)lO
c) Intraventricular haemorrhage ~ u ~14
c) Complete basilar fracture

680)c 681)d 682)d 683)c 684)c 685)b 686)d 687)c 688) d 689) d 690) d 691)c 692)a 693)a
694)c 695)d 696) b,e 697) a 698)b
SURGERY [ 214]

699. In a head injury victim, which of the following is the 711. Pierre Robin Sequence includes- (PGI Dec 08)
most common initial manifestation of an increasing a) Glossoptosis b) Airway obstruction
intracranial pressure- (UPSC II 11) c) Cleft lip d) Micrognathia
a) Change in the consciousness level e) Heart anomaly
b) Ipsilateral pupillary dilatation 712. With respect to surgical repair of cleft palate, the
c) Contralateral pupillary dilatation soft palate is first repaired, ideal tune for which is_
d) Hemiparesis a) 12 months b) 9 months (Maharashtra 10)
c) 6 months d) 3 months
CLEFT LIP & CLEFT PALATE 713. A four-month-old baby has cleft lip and palate. How
would you manage the baby? (UP SC II 10)
700. Hynes pharyngoplasty is used to improve a a) Immediate repair of the lip
child's- (JIPMER 81, AMU 89) b) Immediate repair of the lit) and palate
a) Appearance b) Teething c) Delayed repair of the lip and palate until2 years
c) Speech d) Feeding d) Delayed repair of the lip and palate until preschool
701. Rhinoplastyis usually done at the age (years) of- age
until the nose is fully grown-(JIPMER 81AIIMS 86) 714. Trueaboutcleftpalate- (PG!May10)
a) 6 years b) 12 years a) Surgery should be done at 1 year
c) 16 years d) 25 years b) 50% recover speech after operation
702. Cleftpalateisrepairedaround- (JIPMER 87) c) A/w hearing loss
a) Soon after birth b) After one month d) A/w cleft lip in 45%
c) After 3 months d) After 1-1/2 years
703. Pierre Robbin syndrome is- (AIIMS 85) MAXILLO FACIAL IN.JURIES
a) Cleft palate with syndactly
b) Cleft palate with mandibular hypoplasia and 715. Mandible is comonly fractured- (JIPMER 87)
respiratory obstruction a) At the neck of the condyle
c) Cleft lip with mandibular hypoplasia b) Through the angle
d) Cleftlip c) Through the carmine fossa
704. Commonesttypeofcleftlip is- (Aiims 91) d) At the middle
a) Bilateral b) Midline 716. Le Forte ll facial fracture implies- (AIIMS 84)
c) Combined with cleft palate d) Unilateral a) Fracture running through alveolar ridge
705. Rhytidectomy operation involves- (Jipmer 92) b) Fracture running through midline ofthe palate and
a) Correction of nasal defects zygomaticomaxillary suture
b) Removal ofwrinkles in forehead c) Fracture running through zygomatic process of
c) Straightening of curved penis the maxilla floor oforbit root ofnose on one side only
d) Correction of prouting Lips d) Similar to C but on both sides
706. In cleft lip operation all the stitches are removed 717. A 20 year old man is hit on the eye with a ball. On
on- (PGI 81, 82, AIIMS 85, PGI 92, DNB 91) examination there is restriction of lateral and
a) 2nd day b) 4th day upward gaze and diplopia. There is no obvious
c) 1Qihday d) 141hday visible sign of injury to the eye ball, but there is
707. Unilateral clefts are most common on- (PGI 80, some enophthalmos, the likely diagonosis is-
a) Left side b) Right side (PGI 80, a) Zygoma fracture (AIIMS 85)
c) Median d) None of the above b) Maxillary fracture
708. The following is the method for operating cleft c) Blow out fracture of the orbit
lip except- (J/P. 81, AIIMS 86) d) Injury to lateral rectus
a) Le Mesurier's method b) Tennison's method 718. Best treatment of this condition will be -(AIIMS 85)
c) Millard's method d) Wardill's method a) Do nothing and assurance
709. Cleft lip is due to non fusion of- (PGI 2001) b) Explore the orbit
a) Maxillary process with lateral nasal process c) Ophthalmic exercise to correct diplopia
b) Maxillary process with medial nasal process d) Reinsertion oflateral rectus muscle
c) Maxillary process with mandibular process 719. Fracture manidble with edentulous jaw is best
d) All of the above treated with - (U.P.P. G.M.E.E. 04)
710. Unilateral cleft lip is associated with- (PGI 99) a) External fixator b) Minerva-plaster
a) Posterior displacement of alar cartilage c) Interdental wiring d) Intennaxillary elastic traction
b) Columella elongated 720. Best view for mandile is- (UP 07)
c) Always cleft palate a) Antero-posterior b) Lateral
d) Defective sucking c) Oblique d) Orthopentomogram

699)a 700)c 701)c 702)d 703)b 704)c 705)b 706)b 707) a 708) d 709) b 710)a 711)a,b.e
712)c 713)a 714)a,c,d 715)a 716)d 717)c 718)b 719)a 720)d
SURGERY [ 215]

721. Which is the most common type of skull fracture - 732. Commonest type oforalmalignancyis- (AI92, 91
a) Linear fracture (NEETIDNB Pattern) a) Melanoma PGI 88)
b) Depressed fracture b) Squamous cell carcinoma
c) Comminuted fracture c) Adeno carcinoma
d) Pond fracture d) Basal cell carcinoma
733. Which is not true of Carcinoma tongue-(AI/MS 92)
ORAL CAVITY a) Lateral border is involved
b) Cervical lymph node involvement
722. Commonest site of carcinoma tongue- (AI 88) c) Commonly adenocarcinoma
a) Apical b) Lateral borders d) Tobacco chewing is a risk factor
c) Dorsum d) Posterior 1/3 734. Painless ulcer of the tongue is due to- (PGI 79, 80,
e) Faucio lingual a) Dyspepsia b) Syphilis UPSC 87)
723. Dentigenous cyst arises from- (AI 88, JIPMER 87) c) Tuberculosis d) None of the above
a) An unerupted tooth 735. Commonest premalignant condition of oral
b) Apex of an infected tooth cancer- (AI 95, 96)
c) Nasopalatine cysts a) Leukoplakia b) Apthous ulcer
d) Solitary bone cyst c) Syphylitic ulcer d) Erythroplakia
736. Most common type of oral carcinoma is- (AI 96)
e) Multi locular keratocytes
a) Lip b) Cheek
724. Carcinoma tongue less than 2 em is treated by- c) Tongue d) Palate
a) Excision (JIPMER 87) 737. Lymphatic metastasis of buccal carcinoma first
b) Radiotherapy involves........ nodes- (AI 97)
c) Chemotherapy a) Submandibular b) Submental
d) Excision and Radiotherapy c) Jugulodigastric d) Juguloomohyoid
e) Excision and Chemotherapy 738. Anatomical structure injured in excision of
725. Treatment of choice for carcinoma of lip of less ranula- (PGI 96)
than 1 em is- (Kerala 87, AI 90) a) Lingual vein b) Lingual artery
a) Radiation b) Chemotherapy . c) Submandibular duct d) Parotid duct
c) Excision d) Radiation and Chemotherapy 739. Not a carcinogenic for cancer of Head &
726. Predisposing factors for development of oral Neck- (PGI 96)
carcinoma is- (JIPMER 88) a) Alcohol b) Smoking
a) Smoking b)Alcohol c) Tobacco d) Unflourinated water
c) Syphilis d) All of the above 740. 60 year old man present with an ulcer on lateral
727. A full thickness loss of middle one third of the margin of tongue also complains of ear pain, most
upper lip is best reconstruted by- (AIIMS 84) probable diagnosis is- (PGI 96)
a) Naso labial flap b) Cheek flap a) Dental ulcer b) Carcinomatous ulcer
c) Abbey flap d) Estlander's flap c) Tuberculosis ulcer d) Syphilitic ulcer
728. A patient with cheek cancer has a tumour of 2.5 741. Cancrumoris-allaretrueexcept- (PG/97)
em located close to and involving the lower a) Associated with malnutrition
alveolus. A single mobile homolaterallymphnode b) In:flammatory swelling
measuring 6 em is palpable. The TNM stage is - c) Associated with vitamin deficiency
a) T 1 N 1 M0 b)T2 N2 M0 (AIIMS85) d) Treatment is excision and skin grafting
c) T2 N 1 M0 d)T4 N2 M0 742. Patient with T3N2AMO lower alveolar Ca
729. In the reconstruction following excision of requires- (AIIMS 98)
previously irradiated cheek cancer, the flap will be- a) Surgery b) Surgery+ Radiotherapy
c) Radiotherapy d) Chemotherapy
a) Local tongue (AIIMS 85)
743. Trismus in oral cancer patients is severe in whom
b) Cervical
was treated with- (Karnatak 99)
c) Forehead a) Surgery and Radiotherapy
d) Pectoralis major myocutaneous b) Chemotherapy alone
730. Commonest site of carcinoma tongue is- (l'N 89) c) Surgery alone
a) Dorsum b) Ventral aspects d) Not related to treatment
c) Anterior 2/3 lateral aspect d) Tip 744. Impacted wisdom teeth may produce referred pain
731. WhatisRanula- (l'N91) vis- (Orissa 99)
a) Retension cyst of sublingual gland a) Lingual nerve
b) Retension cyst of submandibular gland b) Facial nerve
c) Extravasation cyst of sublingual glands c) Branch of the auriculotemporal nerve
d) Extravassation cyst of submandibular glands d) None of the above

721)a 722)b 723)a 724)a 725)c 726)d 727)c,d 728)None 729)d 730)c 73l)a 732)b 733)c 734)b
735)a 736)a 737)a 738)c 739)d 740)b 741)None 742)a 743)a 744)c
SURGERY [ 216]

745. Areas of carcinoma of oral mucosa can be 755. A 70 years males presented with asymptomatic white
indentified by staining with- (PGI 79, BHU 85) patch on oral cavity following application of the
a) 1 % zinc chloride b) 2 % silver nitrate denture. Treatment of choice is- (UP 08)
c) Gentian violet d) 2% toluidine blue a) Low dose radiotherapy
746. Carcinomaofthe tongue- (JIPMER81, UPSC85) b) Biopsy of the all the tissues
a) Occurs most commonly on the lateral border of c) Ascertaining the denture is fit incompletely
the middle third of tongue d) Antibiotics
b) Metastasized readily to cervical lymph nodes 756. The most common cyst of the oral region
c) Is usually radiosensitive is - (DPGEE 08)
d) Treated surgically should include homolateral a) Dentigerous cyst b) Keratosis cyst
c) Dermoid cyst d) Periapical cyst
neck dissection except for very small lession
757. Which one ofthe following statements is correct in
e) All ofthe above
case of squamous cell carcinoma of the lip -
747. Allaretrueaboutcancrumorisexcept- (PGI97)
a) Lymph node metatases occur early (UPSC-II 09)
a) Associated with malnutrition and vitamin deficiency b) More than 90% of cases occur on the upper lip
b) Follows chronic infection c) Lesion often arises in the areas of persistent
c) Involvesjaw hyperkeratosis
d) Treatment is excision and skin grafting with tubed d) Radiotherapy is considered inappropriate
pedicle graft treatment for these lesions
748. Gum tumour with 2 contralateral mobile lympbnode 758. In a patient of carcinoma tongue, the inmtration of
in cheek comes under- (PGI 99) which muscle causes ankyloglossia- (UPSC-II 09)
a) T 3N 2M0 b) T2N 2M 0 a) Styloglossus muscle b) Genioglossus muscle
c) T 4N 2M0 d) T3N 3M 0 c) Mylohyoid muscle d) Palatoglossus muscle
749. Epulis arises from- (PGI99) 759. Which is not true of Carcinoma tongue? (DPG 10)
a) Enamel b) Root of teeth a) Lateral border is involved
c) Gingiva d) Pulp b) Cervicallymphnode involvement
750. All predisposes to oral cancer except- (PGI 99) c) Commonly adenocarcinoma
a) Erythroplakia b) Leukoplakia d) Tobacco chewing is a risk factor
c) Submucosal fibrosis d) Lichen planus 760. A middle-aged man presents with a lower jaw
751. True statement (s) about oral cancer is/are- swelling. Clinically, there is expansion of the left
a) Most common in buccal mucosa (PGI 04) ramus and the X-ray mandible shows soap bubble
b) Metastasis uncommon appearance. What is the clinical- diagnosis?
c) Respond to Radiotherapy a) Keratocyst (UP SC II 1 0)
b) Adamantinoma
d) Surgery done
c) Aneurysmal bone cyst
e) Syphilis and dental irritation predisposes
d) Odontogenic myxoma
752. Radiographic finding of floating teeth can be seen
761. Which of the following statements about 'Ranula' is
in- (Karnataka 02)
most correct- (AI 11)
a) Ectodermal dysplasis a) It is type of Epulis
b) Cleidocranial dysplasia b) It is a cystic swelling in the floor of mouth
c) Osteopetrosis c) It is a type of thyroglossal cyst
d) Histiocytosis - X d) It is a of mucus retention cyst
753. Carcinoma oflip is characterized by the following ~.iiie.ill'a~e
exceptthat- (UPSC 07)
a) 90% of the lip cancers occur on the lower lip
b) The most common site of origin is the vermillion
border
c) 2 em x 2 em cell carcinomas can be treated by V
shaped excision and primary closure
d) Since lymph noded metastases are common after SALIVARY GLANDS
a radical dissection of neck is mandatory
754. What is the most frequenttooth to be impacted? · 763. Warthins tumour is- (AI 88)
a) Lower third molar (UPSC 07, a) Malignant neoplasm
b) Upperthirdmolar Karn 98) b) Rapidly growing
c) Lower canine c) Gives a hot pertechnetate scan
d) Upper premolar d) Cold pertechnetate scan

745)d 746)e 747)b 748)a,b,c 749)c 750)d 75l)b,c,d,e 752)d 753)d 754)a 755)c 756)d 757)d 758)b
759)c 760)b 76l)b 762)d
SURGERY [ 217 ]

764. The parotid duct is known is~ (AP 84) 775. Best treatment for pleomorphic adenoma-
a) Wharton's duct b) Stenson's duct a) Radiotherapy (JIPMER 95)
c) Duct of santorini d) Duct of wirsung b) Chemotherapy
765. Sjorgen's syndrome refers to disease of- c) Superficial paratidectomy
a) Parotid glands (AIIMS 85) d) Complete parotid resection
b) Thyroid disease 776. In a case of parotid swelling the diagnostic procedure
c) Parathyroid glands of choice- (AIIMS 94)
d) Multiple endocrin e neoplasia a) FNAC b) Superficial parotidectomy
766. Treatment of choice for mixed parotid tumour is- c) Enucleation d) Excision biopsy
a) Enucleation (TN 89) 777. Commonest tumour is parotid gland is- (TN 95,
a) Adenoma AI91)
b) Superficial parotidectomy
b) Pleomorphic adenoma
c) Radial parotidectomy
c) Squamous eel carcinoma
d) Wide excision
d) Warthin's tumour
7 67. Patotid tumour which spreads perineurally is- 778. Stonesaremax:imumin- (AMU95, UPSC 87)
a) Mucoepidermoid carcinoma · (PGI 84) a) Parotid ' b) Submandibular
b) Epidermoid carcinoma c) Pancreas d) Sublingual
c) Carcinoma in pleomorphic adenoma 779. Most of the parotid tumor are managed by-
d) Adenoid cystic carcinoma a) Total parotidectomy (AI 97)
768. Treatment of submandibular salivary gland duct b) Radical parotidectomy
calculi is- (TN 90) c) Superficial parotidectomy
a) Excision of submandicular gland d) Radical parotidectomy & Neck dissection
b) Opening the duct at the frenulum 780. During parotid surgery injury to the nerve which
c) Opening the duct and removal of calculus results in Frey's syndrome is - (PGI 96)
d) Excision of gland and duct a) Auriculotemporal Nerve b) Great auricular Nerve
769. Adenolymphomarefersto-. (TN91) c) Mandibular Nerve d) Buccal Nerve
a) Warthin's tumour b) Pleomorphic adenoma 781. The nerve sacrificed in parotid sugery- (AP 98)
c) Adenocarcinoma d) Adenocystic carcinoma a) Auriculotemporal b) Facial
770. In Submandibular gland surgery. The nerve least c) Buccal d) Cervice facial
likely to be injured is- (JIPMER 93) 782. A bacterial pyogenic parotitis is found most
a) Inferior alveolar nerve commonly in which of the following- (Orissa 99)
b) Hypoglossal nerve a) Mumps
c) Lingualnerve b) Debilitation after major surgery
d) Mandibular branch of facial nerve c) Drug reaction (iodine mumps)
771. Inflammatory enlargement of deep lobe of parotid d) Uveoparotid fever
gland is seen in- (PGI 81, Delhi 92) 783. Commonest site of ectopic salivary tumour-
a) Post pharyngeal wall b) Supratonsillar area a) Tongue b) Cheek (CUPGEE 02,
c) Anterior tonsillar pillar d) Tonsillar fossa/bed c) Neck d) Palate AIIMS85)
772. Mixed parotid tumour arises from- (PGI 80, 784. In surgery of submandibular salivary gland, nerve
a) Epithelium Delhi 87, 88) ofteninvolved- (PGI97)
b) Epithelium+ Mesenchymal a) Hypoglosal b) Glossopharygneal
c) Mesenchymal c) Facial d) Lingual
d) None of the above 785. All are true for plemorphic adenoma except-
773. Most common tumour of minor salivary glands is- a) Arises from parotid (PGI99)
a) Muco epidermoid carcinoma (Keala 94) b) May tum into malignant
b) Mixed tumour c) Minor salivary glands involved
c) Squamous cell carcinoma d) None
d) Epithelioma 786. Mucoepidermoid carcinoma of parotid arises
774. Which of the following group constitue Frey's from- (PG/99)
syndrome- (Karn. 94) a) Secretory cells b) Excretory cells
a) Hyperhydrosis, enophthalmos and miosis c) Myoepithelial cells d) Myofibril
b) Anhidrosis, enophthalmos and miosis 787. Treatment of choice in mixed tumor of parotid gland-
c) Redness and sweating over the auriculotemporal a) Superficial parotidectomy (PGI 2000)
during meal b) Total parotidectomy
d) Pain over the distribution ofteh auriculotemporal c) Radiotherapy
during meal d) Chemotherapy

763)c 764)b 765)a 766)b 767)d 768)c 769)a 770)a 771)d 772)b 773)a 774)c 775)c 776)a
777)b 778)b 779)c 780)a 781)b 782)b 783)b 784) a,c,d 785) d 786) a 787) a
SURGERY [ 218]

788. True statement (s) about saHvary gland tumours- 798. All of the following are anatomical markers for
a) Pleomorphic adenoma can arise in submandibular localization offacial nerve during parotid surgery,
gland (PGI 04) except- (DPGEE 08)
b) Warthim's tumour arises from submandibular a) Posterior belly of Digastric
gland b) Mastoid process
c) Pleomorphic adenoma is most common tumour c) Inferior belly of omohyoid
of sub mandibular gland d) Bony external auditory meatus
d) Acinic cell Ca is most malignant 799. Regarding pleomorphic adenoma of salivary gland
e) Frey's syndrome is due to injury ofauriculotemporal true statement (s) is/are- (PGI Dec 08)
nerve a) Parotid gland is most commonly involved
b) Malignant transformation does not occur
789. Which of the following statement regarding salivary
c) Also called mixed tumour
neoplasm is correct- (SGPGI 05)
d) More commonly found in Men than women
a) 60-80% are malignant
e) Superficial parotidectomy is treatment of choice
b) 80% ofbenign are pleomorphic 800. All of the following statements about
c) Warthin is malignant tumor lymphoepethelioma of the parotid gland are true,
d) Tumors of submandibular gland are common except- (AI 09)
790. In which one of the following conditions the a) Parotid gland is the most common site of
Sialography is contraindicated- (AI 05)
Lyrnphoepethelioma in the Head & Neck region
a) Ductal calculus b) Chronic parotitis b) It is associated with EBV infection
c) Acute parotitis d) Recurrent sialadenitis c) It is highly radiosensitive
791. In which of the following locations, Carcinoid tumor d) It is a type ofsquammous cell carcinoma
is most common- (AI 05) 801. Aciniccellcarcinomaisfoundin- (UPSCIIJO)
a) Esophagus b) Stomach a) Thyroid b) Salivary glands
c) Small bowel d) Appendix c) Breast d) Stomach
792. A Warthin's tumour is- (AIIMS May 2005) 802. True about parotid tumor- (PGI May 10)
a) An adenolyrnphoma of parotid gland a) Facial nerve involvement indicates malignancy
b) A pleomorphic adenoma of parotid b) Pleomorphic adenoma is MC variety
c) A carcinoma of the parotid c) Malignant disease is MC veariety
d) A carcinoma of submandibular salivary gland d) Superficial parotidectomy is the treatment of
793. Mixed tumors of the salivary glands, are- (AJ 06) choice
a) Most common in submandibular gland
b) Usually malignant
c) Most common in parotid gland
d) Associated with calculi 803. Glomus tumour arises from- (AI 91)
794. Sialosis refers to - (Karnataka PGMEE 06) a) Ectoderm b) Endoderm
a) Bilateral parotitis c) Mesoderm d) Neuroectoderm
b) Sjogren's syndrome 804. Commonest manifestation of Ca. nasopharynx-
c) Non inflammatory parotid enlargement a) Nasal blockage (JIPMER 93)
d) Bilateral salivary duct ectasia b) Epistaxis
795. Which of the following nerves lies closest to the c) Squint
Wharthin's duct- (Corned 07, AIIMS 87, Delhi 86, d) Secondaries in neck
a) Hypoglossal b)Lingual PGI 81) 805. Nasopharyngeal carcinoma originates in-
c) Chorda tympani d) Facial a) Posterior pharyngeal wall (AIIMS97)
796. The most common tumor of the minor salivary b) Posterior wall of septum
gland is- (Corned 08) c) Fossa ofRosenmuller
a) Mucoepidermoid carcinoma d) Superior meatus
b) Acinic cell carcinoma 806. The Sucque-Hoyer canal in related to- (AJIMS 81)
c) Adenoid cystic carcinoma a) Post-Mastectomy lyrnphanigiosarcoma
d) Pleomorphic adenocarcinoma b) Leimyous cuits
797. Which ofthe following is not a landmark for facial c) Active junctional nerve
nerve during parotid surgery?- (AI 08) d) Glomus tumour
a) Digastric muscle 807. Which among the following is a feature of fracture
b) Inferior belly of omohyoid ofzygoma- (PGI 99)
c) Tragal pointer a) Diplopia b) Epistaxis
d) Retrograde dissection of distal branch c) Bell's palsy d) Frey's syndrome

788)a,e 789)b 790)c 79l)c 792)a 793)c 794)c 795)b 796)c 797)b 798)c 799)a,c,e 800)a 80l)b
802) a,b,d 803) c 804) d 805) c 806)d 807)a
SURGERY [ 219]

808. Predisposition for carcinoma ethmoid sinus is 821. The brilliantly transilluminant tumour in the neck
by- (PGI2000 maybe- (AI 91)
a) Wood dust b) Thorotrast a) Branchial cyst b) Thyroglossal cyst
c) Vinyl chloride d) Nitrosamines c) Sternomastoid tumour d) Cystic hygroma
809. What is the narrowest part of the respiratory tract 822. Thyroglossal fistula develops due to -(KERALA 91)
in children?- (UPSC 06) a) Developmental anomaly
a) Glottis b) Subglottic area b) Injury
c) Incomplete removal of thyroglossal cyst
c) Trachea d) Carina
d) Inflammatory disorder
810. Which one of the following is the primary defect in
823. Which of the following is not excissed in Radical
Pierre Robin syndrome? (UPSC 06) neck dissection- (AIIMS 93)
a) Micrognathia b) Glossoptosis a) Internal jugular vein· b) Spinal accessory nerve
c) High arched palate d) Cleft palate c) Extamal carotid artery d) Sternomastoid Muscle
811. Dentigerous cyst arises from -(DELHI PG Feb. 09) 824. Secondaries in the neck with no obvious primary
a) The root of a caries tooth malignancyismostoftendueto- (JIPMER 93)
b) The periosteum of the fractured mandible a) Ca. stomach b) Ca. Larynx
c) An unerupted permanent tooth c) Ca. Nasopharynx d) Ca. thyroid
d) The sequenstrum of osteomyelitis of mandible 825. Radical dissection of neck includes all except-
a) Cervical lymph nodes (.llPMER 80, DELHI 89)
NECK b) Sternocleidomastoid
c) Phrenic nerves
812. Structures preserved in functional radical d) Internal jugular vein
dissection ofthe neck- (AIIMS 87) 826. Regarding cervical rib, which statement is correct-
a) Int. jugular vein b) Sternomastoid a) It always connects to the scalene tubercle by a
c) Lymph nodes d) Accessory nerve fibrous band (AIIMS 81, PGI 82)
813. Earliest tumour to appear after bith is-(.llPMER 87) b) It passes through the apex of the supraclavicular
a) Sternomastoid tumour b) Cystic hygroma triangle
c) Branchial cyst d) Lymphoma c) It causes pressure on the ulnar nerve
814. Most common site for thyroglossal cyst is- d) Pain is often located in the forearm
a) Suprahyoid (NIMHANS 86, JIPMER 87) 827. Which is never a cause of thyroglossal fistula-
b) Subhyoid a) Infection of thyroglossal cyst(PGI 81, AIIMS 85)
c) Beneath the foramen ceacum b) Inadequate removal of thyroglossal cyst
d) floor of mouth c) Congenital
e) Level of circoid d) None of the above
815. Cystic compressible, translucent swelling in the 828. The most common site of the internal opening of
posterior triangle of neck- (AI 89) a branchial fistula is atthe- (UPSC 95)
a) Cystic hygroma b) Branchial cyst a) Lateral nasopharyngeal wall
c) Thyroglossal cyst d) Dermoid cyst b) Fossa ofrosemuller
816. Treatmentofcystichygromais- (JIPMER 88) c) Ginfivo -labial sulcus
a) Surgical excision b) Injection of sclerosants d) Tonsiller fossa
c) Irradiation d) Masterly inactivity 829. Which neck swelling moves on swallowing-
817. Sistrunk's operation is used in- (TN 89) a) Submandibular salivary gland
a) Parotid tumour b) Thyroglossal fistula b) Thyroid gland
c) Thyroglossal cyst d) Branchial fistula c) Supraclavicular lymph gland
d) Sternomastoid tumour
818. Adson's test is positive in- (KERALA 89)
830. All are true about cystic hygroma except-
a) Cervical rib b) Cervical spondylosis
a) Pulsatile (AMU 95)
c) Cervical fracture d) Cervical dislocation
b) May cause respiratory obstruction
819. Radical neck dissection involves removal of all
c) Common in neck
except- (PGI 89)
d) Present at birth
a) Omohyoid b) Internal jugular vein
831. In modified radical neck dissection, structure not
c) Submandibular galnd d) Tail of parotid
preserved is - (PGI 96)
820. Which of the following does no move on deglutition-
a) Sternocledomastoid b) Internal jugular vein
a) Subligual dermoid (AI 91)
c) Accessory nerve d) Submandibuar gland
b) Thyroid nodule 832. Hyoid bone is closely associated to which- (AI 98)
c) Pretracheallymphnode a) Bronchogenic cyst b) Thyroglossal cyst
d) Thyroglossal cyst c) Cystic hygroma d) Ranula

808)a 809)b 810)a 8ll)c 812)a,b,d 813)b 814)b 815)a 816)a 817)b,c 818)a 819)a,d 820)a 82l)d
822)c 823)c 824)c 825)c 826) c,d 827) c 828) d 829) b 830) a 831)d 832)b
SURGERY [ 220]

833. Which of the following swellings moves as 842. Trueofcarotidbodytumoris- (PG/02)


protrusion of tongue- (ROmAK 98) a) Non-chromaffmparaganglioma
a) Thyroglossal cyst b) Branchial cyst b) Good prognosis
c) Ranula d) Cyst in hyoid bone c) Rarely metastasises
e) Odontogenous cyst d) Similar to mixed parotid tumor
834. Cystic hygroma- (SCTIMS 98) 843. Thoracic outlet syndrome is characterized by -
a) Should be left alone a) Widevariationofsymptorns (PG/03)
b) Excision of cyst at an early age b) X-ray neck can alone diagnose
c) Spontaneous regression c) Cold exposure & exercise can precipitate
d) Manifests in 2nd - 3rd decade d) Absent radial pulses
835. Commonestsiteofbrancbialcyst- (AI 88) e) Surgery is the treatment of choice
a) Lower 113 of sternomastoid on anterior border 844. Trueaboutcarotidbodytumor- (PGI 04)
b) Lower 113 of sternomastoid on posterior border a) Arises from pharyngeal wall
c) Upper 1/3 of sternomastoid on anterior border b) Arises from baroreceptor cells.
d) Upper 113 of sternomastoid on posterior border c) Commonly seen in high altitude habitants
836. A 75 year old woman underwent neck exploration d) 10% have family history
for hyperthyroidism 5 years ago, and a parathyroid e) FNAC is diagnostic
adenoma was excised. At present she is recovering 845. Excision of Hyoid bone is commonly done us-
from a myocardial infarction 6 weeks ago, and she a) Sublingual dermoid (HPU 05, PGI 88)
is in mild congestive heart failure. her b) Branchial fistula
electrocardiogram shows a slow atrial fibrillation. c) Branchial cyst
Measurment of her serine calcium shows a level of d) Thyroglossal cyst
13.0 mg/dl and urine calcium is 300 mg/24 hrs. 846. Commonest treatment of Branchial cyst- (HPU 05)
Studies suggest a small mass in the paatracheal a) Cystectomy b) Aspiration
position behind the right clavicle. Appropriate c) Excision d) Nothing done
management at this time is- (AI 2002) 847. In which one of the following prineural invasion in
a) Observation and repeat serum calcimnin two months head and neck cancer is most commonly seen-
b) Repeat neck exploration a) Adenocarcinoma (AI 05)
c) Treatment with technetimn- 99 m b) Adenoid cystic carcinoma
d) Ultrasound guided alcohol injection in the mass c) Basal Cell Adenoma
837. Precancerous lesion of the larynx include-(PG/ 87) d) Squamous cell carcinoma
a) Kerarosis laryngis b) Pachyderma laryngis 848. Which of the following best represents 'ranula'-
c) Laryngis sicca d) Scleroma a) A type of epulis (AIIMS May 2005)
838. Trotters triad is seen in - (PGI 85) b) A thyroglossal cyst
a) Angiofibroma c) Cystic swelling in the floor of mouth
b) Nasopharyngeal carcinoma d) Forked uvula
c) Laryngeal carcinoma 849. Which ofthe following statements best represents
d) Growth in fossa ofrosenmuller Ludwing's angina- (AIIMS May 2005)
839. All are true about cystic hygroma except -(PGI 99) a) A type of coronary artery spasm
a) Aspiration is diagnostic b) An infection of the cellular tissues around
b) 50<);0 present at birth submandibular salivary gland
c) Presents as posterior cervical swelling c) Oesophageal spasm
d) Sequstration oflymphatic tissue d) Retropharyngeal infection
840. True about cystic hygroma- (PGI 2000) 850. True about Branchial cyst is- (Aiims May 07)
a) Congeuital sequestration oflymphatics a) Cysts are more common than sinuses
b) Resolves spontaneouly by 5 year of age b) Mostly arises from 2nd branchial system
c) Common in upper 1!3rd oflateral neck c) Causes dysphagia and hoarseness
d) Surgery is the treatment of choice d) Sinus should always be operated
841. In the management of thyroglossal cyst- (PGI 02) 851. True about branchial cyst- (PGI June 07)
a) Central portion ofhyoid excised a) Seen deep to lower 1/3 of sternoleidomastoid
b) Sternothyroid muscle dissected b) Wall consists of lymphoid tissue
c) Isthmusectomy e subtotal thyroidectomy c) filled with straw coloured fluid with cholesterol
d) Strap muscle of neck crystals

833)a 834)b 835)c 836)d 837) a,b 838) b 839) a 840)a,d 84l)a 842)a,b,c 843)a,c 844)c,d 845)d
846)c 847)b 848)c 849)b 850)b 85l)b,c
SURGERY [ 221 ]

852. Structures not removed in radical neck dissection- 862. Brilliantly transilluminant swellings are all
a) X nerve (PGI June 07) except? (AIIMS Nov 10)
b) XI nerve a) Hydrocele b) Meningeocele
c) Tail of parotid c) Lipoma d) Cystic hygroma
d) Parotid and post-auricular nerve 863. Consider the following statements : Branchial cysts-
853. Which one of the following is the most common 1. Are associated with tracks passing between the
tumour to produce metastasis to cervical lymph carotid bifurcation. (UPSC II JJ)
2. usually present in early adulthood.
nodes? (UPSC-11 08)
3. Occur along the lower one-third ofthe anteromedial
a) Glottic carcinoma
border of the sternocleidomastoid muscle.
b) Nasopharyngeal carcinoma 4. Develop from the vestigial remnants of the fourth
c) Carcinoma base of tongue branchial cleft.
d) Carcinoma lip Which of the statements given above are correct-
854. Which vital structure is preserved during radical a) 2,3and4only b)1,3and4only
dissection- (UPSC-II 08) c) 1,2, 3 and 4 d) 1,2 and 3 only
a) Vagus nerve b) Submandibular gland 864. The "Subclavian steal syndrome" occurs due to -
c) Strenocleidomastoid d) Internal jugular vein a) Occlusion/stenosis of the vertebral artery
855. Which of the following is used to define penetrating b) Occlusion/stenosis of the carotid artery
neck injury- (AI 08) c) Occlusion of the subclavian artery proximal to
a) 2 em depth of wound origin of vertebral artery (UPSCII 11)
b) Injury to vital structures d) Occlusion of the subclavian artery distal to origin
c) Breach of platysma of vertebral
d) Through and through wound
856. In which of the following head and neck cancers, is
lymph node metastasis least common-?Wms May 08)
a) Tongue b) Buccal mucosa
c) Hard palate d) Lower alveolus
857. Which is incorrect about cystic hygroma ?
a) Brilliantly translucent (APPG 08)
b) Radiotherapy THYROID GLAND
c) Sclerotherapy with Bleomycin
d) Sclerotherapy with actinomycin 866. Therapy of choice for diffuse toxic goitre in a
858. In Post radical neck dissection shoulder syndrome, patient over 45 years- (JIPMER 87)
allareseenexcept- (AIIMSNov08) a) Surgery
a) Restricted range of movement b) Antithroid drugs
b) Pain c) Radio iodine
c) Shoulder drooping d) Antithyroid drugs ftrst followed by surgery
d) Normal electromyographic ftnding 867. Symptoms of endemic goitre are all except -(JIPMER
859. True about branchial anomaly- (AIIMS Nov 09)
a) Cold intolerance b) Hoarseness 86)
c) Dysphagia d) Heat intolerance
a) Cysts are more common than sinuses
868. What percentage of cold thyroid nodules are
b) For sinuses surgery is not always indicated
malignant- (JIPMER87)
c) Cysts present with dysphagia and hoarseness of a) 70-80% b)50-60%
voice c) 40-50% d) 10-20%
d) Most commonly due to 2nd branchial remnant 869. Complications oftherapy with radioactive iodine
860. True about branchial anomaly- (AIIMS May 10) includes- (UPSC 87, 88)
a) Cysts are more common than sinuses a) Thyroid malignancy b) Hypothyroidism
b) For sinuses surgery is not always indicated c) Leukemia d) Ail of the above
c) Cysts present with dysphagia and hoarseness of 87 0. In thyrotoxicosis which of the following is seen -
voice a) Pretibialmyxedema b)Glycosuria (TN89)
d) Most commonly due to 2nd branchial remnant c) Unilateral exophthalmos d) All
861. About congenital torticollis all are true except- 871. Restrosternal tumour cause- (TN 89)
a) Always associated with breech extraction a) Bluish discolouration of the face
b) Spontaneous resolution in most cases b) Edemaofface
c) 2/3rd cases have palpable neck mass at birth c) Can occur in thyroid tumours
d) Uncorrected cases develop plagiocephaly d) All

852) a,c,d 853) b 854) a 855) c 856) c 857) b 858) d 859) d 860) d 861) a 862) c 863) d 864) c 865) b
866)c 867)None 868)d 869)b 870)d 871)d
SURGERY [ 222]

872. Commonest presenting complaints of medullary 883. All are true about Hashimoto's thyroiditis except-
carcinoma thyroid is- (PGI 84) a) Antithyroid microsomal antibodies (Kerala 95)
a) Diarrhoea b) Dysphagia b) Antithyroid nuclear antibodies
c) Hoarsness d) Flushing c) Anti TSH receptor anitbodies
873. Hypothyroidism with increased TSH levels is seen d) Increased level of thyroid hormones
in- · (PGI 90) 884. A 21 year old woman has 3 em node in the lower
a) Sheehan's syndorme deep cervical chain on the left. The biopsy is
b) Lithium corbonate therapy interpreted as revealing normal thyroid tissue in
c) Post radioiodine ablation a lymph node. The most likely diagnosis is-
d) Endemic goitre a) Subacute thyroiditis (UPSC 95)
87 4. All of the following regarding papillary carcinom b) Metastatic carcinoma thyroid
thyroid is true except- (AI 90) c) Hashimoto's disease
a) Multicentric origin d) Lateral aberrant thyroid
b) Secondaries to lymphnodes 885. The carcinoma of thyroid associated with
c) Slowing growing hypocalcemia is- (AIIMS 94)
d) Bony metastasis in early stage a) Papillary cancer b) Medullary cancer
875. Protein bound iodine measures secretory function c) Follicular cancer d) Anaplastic
ofthyroid in all the following circumstances except 886. Hurthle cells are seen in- (AI95)
a) Nephrotic syndrome (AI 90) a) Hashimoto's thyroiditis
b) Following hemithyroidectomy b) Granulomatous thyroiditis
c) During ampicillin therapy c) Reidel's struma
d) Asthamatics on ephedrine d) Acute thyroiditis
876. Serum calcitonin level is raised in - (AI 91) 887. All of the following are symptoms of
a) Papillary ca. thyroid b) Medullary ca. thyroid hyperthyroidism except- (UPSC 96)
c) Follicular ca. thyroid d) Anaplastic ca. thyroid a) Voracious appetite b) Cold intolerence
c) Emotional distrubance d) Sleeplessness
877. Which malignancy of thyroid is cuased by
888. The treatment of non-functioning thyroid nodule
Radiation- (JIPMER 91)
in a 40-year old male should be - (UPSC 97)
a) Papillary carcinoma b) Anaplastic Ca
a) Radiotherapy b) Cortisone
c) FollicularCa d) MedullaryCa c) Surgical excision d) Large dose of iodine-131
878. Metastasis through lymphatics is common is- 889. Toxic adenoma on scaning appear as- (.liPMER 98)
a) Medullary carcinoma (AIIMS 92, 82) a) Hot nodule b) Cold nodule
b) Papillary carcinoma c) Warm nodule d) Neutral
c) Follicular carcinoma 890. A 56-year old lady is found to have to hard,nodular
d) Anaplastic carcinoma thyroid painless swelling in the thyroid region diagnosis in
879. Malignancy in a multinodular goitre is most often- this case is best established by - (UPSC 97)
a) Anaplastic Ca b)FollicularCa (AI93) a) Ultrasound scan b)F.N.A.C.
c) Medullary Ca d) Papillary Ca c) CT Scan d) Radioactive iodine scan
880. Treatment of choice in cold nodule of thyroid- 891. Two hours after sub-total thyroidectomy for
a) Subtotal thyroidectomy (JIPMER 93) thyrotoxicosis, young woman rapidly becomes
b) Wait and watch agitated and complains of increasing difficulty in
c) 1131 breathing. Her pulse rate rises and central cyanosis
d) Hemithyroidectomy is noticed on examination, her neck is found to be
881. The only reason for operating in case of thyroiditisis- tensely swollen beneath the stiches. The most
a) To prevent cancerous degeneration (JIPMER 79, appropriate managemment in this case would be-
b) Forreliefofpaininneckandear AIIMS86) a) Intranasal oxygen (USPC 97)
c) To overcome pressure on trachea or oesophagus b) Passing an endotracheal tube in the ward
d) To cure the toxic reaction c) Removing sutres from all layers in the ward and
evacuation of the hematoma
e) If there is auto immune reaction
d) Immediate transfer of the patient to the operation
882. Thyroid carcinoma- (PGI 82, UPSC 8 7)
theatre for tracheostomy
a) Is often associated with hypothyroidism
892. Young female with proptosis, intolerance to heat
b) Often produces hyperthyroidism
with increased T3 and T4- (AIIMS 98)
c) Is usually euthyroid a) Diffuse thyroid nodule b) Adenomatous giotre
d) Occurs in toxic nodules c) Reidles struma d) Hashimoto's thyroiditis

872)a 873) b,c,d 874) d 875) a 876) b 877) a 878)b 879)b 880)d 88l)c 882)c 883) b 884) d 885) b
886)a 887)b 888)c 889)a 890)b 89l)c 892)d
SURGERY [ 223]

893. Scabard trachea is seen in- (Karnat 99) 905. Treatment of papillary ca· of Thyroid with bony
a) Thyroid cancer b) Thyroiditis metastasis- (PGI 04)
c) Goitre d) All the above a) Radiotherapy
894. FNAC is useful in all of the following tumors b) Radioiodine
ofthyroidexcept- (AJ95,MAHE95) c) Near total thyroidectomy with radiotherapy
a) Papilary carcinoma b )Anaplastic carcinoma d) Near total thyriodectomy with radioiodine &
c) Thyroiditis d) Follicular carcinoma radiotherapy
895. A newborn with a goiter large enough to cause 906. Indications of thyroidectomy- (PGI 04)
dyspnoea is best treated with- (AIIMS 81, 84) a) Cosmetic b) Neoplastic
a) Sulfonamides b) Tracheostomy c) Myxederma d) Pressure effects
e) Lymphocytic throiditis
c) T3 d) Iodides
907. In pregnancy- (APPGE 04)
896. In retrosternal goiter, most common presenting
a) Radioiodine contraindicated
feature is - (PGI 97)
b) Thiouracil is contraindicated
a) Dysphagia b) Stridor c) Surgery is contraindicated
c) Dyspnoea d) Sup. vena caval syndrome d) None
897. Cervical lymph node involvement in papillary ca 908. A 32 year old male with painless lymph node in
thyroid, best treatment is- (PGI 97) cervical region. Lymph node biopsy shown normal
a) Radio active iodine b) Chemotherapy thyroid gland features. The thyroid is normal on
c) Radical neck dissection d) Steroid palpation clinicaUy. The diagnosis is- (Nimhans 05)
898. A patient comes with single thyroid nodule initial a) Lateral aberrant thyroid
investigation of choice is- (PGI 97) b) PapillarY carcinoma thyroid
a) FNAC b) Thyroid function test c) Follicular carcinoma thyroid with metastatic lymph
c) Radio nuclide scan d) MRI nodes
899. Metastasis in thyroid gland come most commonly d) None of the above
from carcinoma of- (PGI 98) 909. All of the following are early life threatening
a) Testis b) Prostate complications of thyroid operation except-
c) Breast d) Lungs a) Tracheomalacia and collapse ofthe larynx
900. True about papillary carcinoma thyroid is-(PGI 99) b) Wound hematoma with compression of the trachea
a) Comprises 10-15% of all thyroid cancers c) Hypocalcemia (SGPG/05)
b) Cells have intranuclear vacuolation d) Thyroid storm
c) Amyloid deposition seen 910. Carcinoma arising from thyroid follicles-
d) Encapsulated a) Medullarycarcinoma-thyroid (MAHE05)
901. Which of the foUowing is true regarding medulary b) Papillary carcinoma-thyroid
c) Anaplastic carcinoma-thyroid
carcinoma of the thyroid- (PGI 01)
d) Hurthle cell carcinoma-thyroid
a) Arises from parafollicular cells
911. Treatment ofMeduUary Ca of thyroid with lymph
b) Secretes calcitonin
node metastasis- (PGI June 05)
c) Occurs in families a) Subtotal thyroidectomy+ Radioiodine
d) Amyloid in stroma b) Subtotal thyroidectomy+ Radiotherapy
e) Commonly malignant c) N eartotal thyroidectomy+ Radioiodine
902. Which of the following factors contribute to the d) Neartotal thyroidectomy+ Radiotherapy
development of duodenal ulcer- (PGI 01} e) Total thyroidectomy+ Radiotherapy
a) mi b) I25J 912. Compared to follicular Ca, papillary Ca ofthyroid
c) 99 Tc d) 32p have- (PGI June 05)
e) Strontium a) More male preponderance
903. TrueaboutanaplasticCaofthyroid- (PGI 04} b) BIL
a) Common in eiderly c) Local recurrence common
b) Surrounding neck tissues are free d) Increased mortality
c) Lymphatic infillration occurs e) Increased lymph node metastasis
d) p53 mutation 913. In video assisted thoracoscopic surgery for better
904. Indication of surgery in a case of Thyroid sweUing vision the space in the operative field is created by-
is/are - (SGPGI 04) a) Self retaining retractor (MAHA 05)
a) Cosmetic b) Pressure symptoms b) C02 insufflation
c) Myxoedema d) pain c) Collapse of ipsilateral lung
e) Swelling with symptoms d) Rib spacing

893)d 894)d 895)b 896)c 897)c 898)a 899) c 900) d 90 I)All 902) a 903) a,c,e 904) a,b,e 905) d
906)a,b,d 907)a 908)b 909)c 910)d 911)e 912) b,c,e 913) c
SURGERY [ 224)

914. Which is not an indication of thoracotomy- 923. A 20 yr old girl presents with 9 month history of
a) Massive pneumothorax (MAHA 05) neck swelling with thyrotoxic symptoms. On
b) Pulmonary cantusion investigation increased T4 and decreased TSH with
c) Bleeding more than 200 ml/hr. in thoracotomy tube palpable 2 em nodule was found. Next investigation
d) Oesophageal rupture will be- (Aiims May 07)
915. The treatment of choice for Anaplastic carcinoma a) USG b) Thyroid scan
of thyroid infiltrating trachea and sternum will be- c) Radioactive iodine uptake d) CT scan
a) Radical excision (AIIMS Nov 05) 924. This thyroiditis is also known as "Painless
b) Chemotherapy Thyroiditis"- (MAHE 07)
c) Radiotherapy a) Subacute lymphocytic b) Dequervain's
d) Palliative/Symptomatic treatment c) Hashimoto d) Riedel
916. Which of the following is the commonest tumour 925. There was a sudden increases in the size of thyroid
of thyroid- (AIJMS Nov 05, PGI 86) swelling along with pain. Most likely cause is-
a) Anaplastic carcinoma b) Follicular carcinoma a) Hemorrhage within the cyst (Thyroid) (MAHE 07)
c) Papillary carcinoma d) Medullary carcinoma b) Malignant change
917. A 45 year old male presents with 4X 4 em, mobile c) Nodular goiter
right solitary thyroid nodule of 5 months duration. d) Colloid goiter
The patient is enthyroid. The following statements 926. Variant of papillary carcinoma thyroid-
about his management are true except- a) Medullary b) Warthin (PGI June 07)
a) Cold nodule on thyroid scan is diagnostic of c) Columnar cell d) Insular
malignancy e) Diffuse sclerosing
b) FNAC is the investigation of choice 927. Papillary Ca thyroid- (PGI June 07)
c) The patient should undergo hemithyroidectomy a) Most common thyroid cancer
ifFNAC report is inconclusive b) Psammoma bodies seen
d) Indirect laryngoscopoy should be done in the c) Encapsulated
preoperative period to assess mobility of vocal d) Blood borne metastasis is common
cords 928. · Thyroid storm after operation is due to -
918. Complications of total thyroidectomy include all a) Inadequate control of hyperthyroidism
except- (AIIMSMay 2005)
b) Massive bleeding (Corned 07)
a) Hoarseness b) Airway obstruction
c) Recurrent laryngeal nerve injury
c) Hemorrhage d) Hypercalcaemia
d) Postoperative infection
919. Thoracic extension of cervical goitre is usually
929. Thyroid carcinoma with pulsating vascular
approached through- (AIIMS May 2005)
a) Neck skeletal metastasis is- (Corned 07, AI 95)
b) Chest a) Follicular b) Anaplastic
c) Combined cervico-thoracic route c) Medullary d) Papillary
d) Thoracoscopic 930. Pendred's syndrome is due to a defect in -(Corned 08)
920. The following statements about thyroglossal a) Chromosome 7p b) Chromosome 7q
cyst are true, except- (AI 06) c) Chromosome 8p d) Chromosome 8q
a) Frequent cause of anterior midline neck masses in 931. A central midline neck swelling is noted in a 4 yr
the first decade of life old girl posted for tonsillectomy. The swelling is,
b) The cyst is located within 2 em of the midline painless, mobile, and cystic, just below the hyoid
c) Incision and drainage is the treatment of choice bone of size 2 x 1.1 x 1 em. U/S showed a thick walled
d) The swelling moves upwards on protrusion cystic lesion. Management would include-
of tongue a) Surgical removal (AIIMS Nov 07)
921. Which ofthe following is not a histological variant b) Antibiotics
ofthyroid neoplasm? (AI07) c) Percutaneous aspiration
a) Follicular b) Merkel cell d) Chest X-ray
c) Insular d) Anaplastic 932. Vocal cord palsy in thyroid surgery is due to injury
922. All of the following are trne about lymphoma of the to- (UPSC-11 08)
thyroid except- (AI 07) a) Superficial laryngeal nerve
a) More common in females b) Recurrent laryngeal nerve
b) Slow growing c) Ansa cervicalis
c) Clinically confused with undifferentiated tumours
d) Vagus nerve
d) May present with respiratory distress and dysphagia

914)b 915)d 916)c 917)a 918)d 919)a 920)c 92l)b 922)b 923)b 924)a 925)a 926) c,e 927) a,b
928)a 929)a 930)b 93l)a 932)b
SURGERY [ 225]

933. FNAC is useful in all the following types ofthyroid c) Hashimoto's disease
carcinoma except- d) Lateral aberrant thyroid
a) Papillary b) Follicular 943. Sistrunk's operation consists of -(DELHIPG Mar. 09)
c) Anaplastic d) Medullary a) Excision ofhyoid bone and cone of tongue muscle
934. Most common histological type of the thyroid b) Excision of hyoid bone and the cyst
carcinomais- (AI08) c) Excision of central part ofhyoid bone and cone of
a) Medullary type b)Folliculartype tongue muscles upto foramen caecum
c) Papillary type d) Anaplastic type d) Excision of cyst only
935. Complications ofHemithyroidectomy include all of 944. Hashimoto's thyroiditis- True are AlE-
the following except- (AI 08) a) Follicular destruction (AllMS May 10)
a) Hypocalcemia b) Increase in lymphocytes
b) Wound hematoma
c) Oncocytic metaplasia
c) Recurrent laryngeal nerve palsy
d) Orphan Annie eye nuclei
d) External branch of superior laryngeal nerve palsy
945. All of the following are early life threatening
936. Investigation of choice in discrete thyroid swelling
is - (DPGEE 08) complications of thyroidectomy (instead of thyroid)-
a) Isotope scans b) Ultrasonography a) Tracheomalacia and collapse of the larynx
c) Autoantibody titres d) FNAC b) Wound hematoma with compression of the trachea
937. About papillary carcinoma what is/are true- c) Hypocalcemia (DPG 10)
a) Often encapsulated (PGI Dec 08) d) Thyroid storm
b) Prognosis bad 946. Symptoms ofthoracic outlet syndrome are produced
c) Lymph node metastases is common due to compression of nerve roots. -
d) Can metastaize to lung a) C5, C6 b) C6, C7 (Maharashtra 10)
e) Multiple foci of tumour is seen c) C7,C8 d)C8,Tl
938. Regarding anaplastic carcinoma which statements 947. A patient with multinodular goitre suddenly
(s) is/are true- (PGI Dec 08) developed pain and difficulty in breathing. Which of
a) Common in elderly the following types of thyroid carcinoma is most
b) Associated with P53 mutation likely to cause this complication?-(Maharashtra 10)
c) Surrounding neck tissue is usually free a) Papillary carcinoma b) Follicular carcinoma
d) Radiotherapy cures c) Medullary carcinoma d) Anaplastic carcinoma
e) Lymphatic spread occur 948. About papillary carcinoma true statement is/are-
939. Treatment of Medullary Carcinoma thyroid- a) Radiation is a risk factor (PGI May 1 O)
a) Surgery and Radiotheryapy (AIIMS Nov 08) b) Multifocal
b) Radiotherapy and Chemotherapy c) Hematogenous spread is common
c) Surgery only d) Distant metastasis is seen
d) Radioiodine ablation
e) More common in iodine deficient area
940. Which of the following would be the best treatment
949. Features (s) ofmeduallary carcinoma thyroid-
for a 2cm thyroid nodule in a 50 year old man with
a) Arises from parafollicular cells (PGI Nov. 10)
FNAC revealing it to be a papillary carcinoma-
a) Hemithyroidectomy (AI 09)
b) Amylodi stroma is present on histology
b) Subtotal thyroidectomy with modified neck c) 10-26% cases are familial
dissection d) Diarrhoea occurs
c) Near total thyroidectomy with modified neck e) Total thyroidectomy is preformed
dissection 950. In which of the following conditions Radioactive
d) Hemithyroidectomy with modified neck dissection iodine (Irradiation) can be used in grave's disease-
941. A person has fever & pain in thyroid gland. True a) Recurrence b) Age> 40 years (PGI Nov. 10)
statement is/are- (PGI June 09) c) Eiderly d) Pregnant
a) T & t level normal b) i ESR e) Presence of associated co-morbidities
c) iTSH d) It is due to T.B 951. Which ofthe following is true- (AIIMS May 11)
e) Radioactive iodine uptake is ed a) Colloid goitre mostly presents as hyperthyroidism
942. A 21 years old woman has3 cmnode in the lower deep b) Thyroid storm, the clinical features are primarily
cervical chain on the left. The biopsy is interpreted due to increased thyroxine
as revealing normal thyroid tissue in a lymph node. c) Excess calcium intake can lead to hyperthyroidism
The most likely diagnosis is- (DELHI PG Feb. 09) d) Goitre more than 5 percent of population is
a) Subacute thyroiditis endemic goitre
b) Metastatic carcinoma thyroid

933)b 934)c 935) a 936) d 937) c,d,e 938) a,b,e 939) c 940) c 941) b,e 942) d 943) b 944) d 945) c
946)d 947)b 948)a,b,d,e 949)All 950)b,c,e 951)d
SURGERY [ 226]

952. Hashimoto's thyroiditis- True are AlE- 961. Hypoparathyroidism occurs as a result of-
a) Follicular destruction (AIIMS May 11) a) Idiopathic atrophy of parathyroids (TN 91)
b) Increase in lymphocytes b) Following surgery
c) Oncocytic metaplasia c) Thyroditis with secondary atrophy of parathyroids
d) Orphan Annie eye nuclei d) All ofthe above
953. Which of the following would be the best treatment 962. Hypocalcemia in immediate post - op period
for a 2 em thyroid nodule in a 50 year old man with following excission of parathyroid adenoma is due
FNAC revealing it to be a papillary carcinoma- to- (AIIMS 92)
a) Hemithyroidectomy (AIIMS May 11) a) Stress b) Increased uptake by bones
b) Total thyroidectomy with left sided modified neck c) Hyper calciuria d) Increased calcitonin
dissection 963. In parathyroid crisis with sudden elevations of
c) Near total thyroidectomy with radiotherapy calcium over 16 mg /100 cc; the treatment consist
d) Hemithyroidectomy with modified neck dissection of- (AIIMS80, UPSC 84)
954. Treatment ofMedullary Carcinoma thyroid- a) Intravenous vitamin D
a) Surgery and Radiotherapy (AIIMSMay 11) b) Parathyroidectomy for removal of adenoma
b) Radiotherapy and Chemotherapy c) Thyrocalcitonin
c) Surgery only d) Intravenous bicarbonate
d) Radioiodine ablation e) All of the above
955. A well differentiated follicular carcinoma of thyroid 964. Hyper paratbyrordism is characterised by the
can be best differentiated from a follicular adenoma following except- (PGI 80, AMU 88)
by- (AI 11) a) Generalised osteoporosis b) Renal calculi
a) Hurthle cell change c) Hypercalcaemia d) Osteosclerosis
b) Lining of tall columnar and cuboidal cells 965. Palpalation on the costovertebral angle produces
c) Vacularinvasion pain and tenderness in acute adrenal insufficiency.
d) Nuclear features This is- (AIIMS 87,89)
956. A 75-year-old lady is diagnosed of having a thyroid a) Rotch's sign b) Rossolimo's sign
malignancy. Her serum calcium is low. What is the. c) Rogoff's sign d) Osler's sign
most likely explanation? (UPSC I 11) 966. A patient bas hypocalcaemia which was the result
a) She has developed metastasis to the parathyroid of a surgical complication. Which operation could
b) She has Follicular carcinoma it possibly have been- (JIPMER 89, UPSC 86}
c) She has Papillary carcinoma a) Nephrectomy b) Thyroidectomy
She has 1vHouu11w c) Gastrectomy d) Vocal cord tumour biopsy
967. The symptoms of Hyperparathyroidism include-
a) Constipation&muscleweakness (AIIMS81,
b) Anorexia & weight loss PGI 85)
c) Polydipsia & polyuria
d) All of the above
968. Commonest cause offor cusbings syndrome is-
PARATHYROID & ADERNAL GLANDS a) Adrenal adenoma b) Carcinoma (Kerala 95)
c) Hyperplasia d) Atrophy
958. Commonest cause for hyperparathyroidism is- 969. A known patient with renal stone disease developed
a) Single adenoma (AI 89) pathological fractures along with abdominalpain and
b) Multiple adenoma certain psychiatric symptoms. He should be
c) Single gland hyperplasia investigated for- (UPSC 96)
d) Multiple gland Hyperplasia a) Polycystic kidney
959. Most common cause of hypercalcemic crisis is- b) Renal tubular acidosis
a) Parathyroid adenoma (AIIMS 87) c) Hyperparathyroidism
b) Parathyroid hyperplasia d) Paget's disease ofbone
c) Carcinoma breast 970. Not seen in Neuroblastoma is- (UP 96)
d) Pagest's disease a) Diarrhoea b) Proptosis
960. Commonest symptom of pheochromocytoma c) Splenomegaly d) Bone involvement
is - (UPSC 88) 971. In pbaeocbromocytoma, the urine will contain-
a) Palpitation b) Headache a) VMA b) HIAA (CUPGEE 96}
c) Sweating d) Dyspnoea c) Both d) None

952)d 953)b 954)c 955)c 956)d 957)b 958)a 959)c 960)b 961) b >a 962) b 963) b,c 964) d 965) c
966)b 967)d 968)c 969)c 970)c
SURGERY [ 227]

972. False statement about pheochromocytoma- 983. MEN-2AincludesAIE- (PG/04)


a) I0% are bilateral (A/97) a) Ganglioneuromas
b) Arises from chromaffin cells b) Cutaneous Lichenoid amyloids
c) Extra adrenal tumor -increased nor adrenaline levels c) Mutation in RET in chromosome 10
d) Increased VMA levels in urine d) Parathyroid adenoma
973. Opsoclonus- Myoclons is a phenomenon seen in- e) Adrenal adenoma
a) Wilms tumor b) Neuroblastoma (PGI 97) 984. Hypoparathyroidism following thyroid surgery
c) Meningioma d) Cortical tuberculoma commonly occurs within- (AIIMS 03)
974. Mostcommonpresentationof hyperparathroidism a) 24 hours b) 2- 5 days
. is - (AIIMS 97) c) 7- 10 days d) 2-3 weeks
a) Gall stone 985. Treatment of hypertensive crisis in
b) Subtle neurological and psychatric symptoms pheochromocytoma- (PGI June 05)
c) Abdominal pain a) Phentolamine b)Hydralazine
d) Swelling in the neck c) Guanethidine d) Salmeterol
975. A patient presented with episodic hypertension e) Methohexitone
and headache, with a thyroid nodule the 986. Indication for surgery in a case of adrenal
investigation of choice is- (AIIMS 97) incidentaloma- (MAHE 07)
a) Urinary 5- HIAA a) Size> 5 em b) Bilateral adrenal metastasis
b) Urinary catecholamines +aspiration of nodule c) Functional tumor d) All ofthe above
c)ESR 987. Incidentaly findings, in CT scan, a 3cm adrenal mass,
d) Thyroid scan which of the foUowing is not done- (UP 08}
976. Hypocalcaemia is a feature of all the following a) Adrenalectomy
except- (UPSC 2K) b) Dexamethasone suppression test
a) Chronic renal failure c) Measurement of catecholamines
b) Hypoparathyroidism d) Midnight plasma cortisol
c) Pseudo hypoparathyroidism 988. Indication for surgery in a case of adrenal
d) Total thyrodectomy incidentaloma- (Manipal 08)
977. After thyroidectomy pt. developed stridor within a) Size> 5 em b) BIL adrenal metastasis
2 hrs. All are likely cause of stridor except- c) Functional tumor d) All ofthe above
a) Hypocalcemia (AIIMS OJ) 989. Which one ofthe following is not aCT feature of
b) Recurrent laryngeal nerve palsy Adrenal adenoma? (AIIMS Nov 10)
c) Larynogomalacia a) Low attenuation
d) Wound hematoma b) Homogeneous density and well defined borders
978. Hypertension is seen in aU except- (PGI 97) c) Enhances rapidly, contrast stays in it for relatively
a) Adrenal tumors b) Phaeochromocytoma longer time and washes out late
c) Conn's syndrome d) Neuroblastoma d) Calcification is rare
979. Neuroblastomas- good prognositc factor is-(PGI 2K)
a) N-myc amplication b) RAS oncogene BREAST
c) Hyperdiploidy d) Translocations
980. Young female presents with hypertension e in 990. Retromammary abscess arises from-
VMA >14 mg/dl, associated with- (PGI 02) a) Tuberculous rib (Jipmer 86,87, Kerala 87)
a) Modullary Ca Thyroid b) Infected hematoma
b) Von Rippel Lindau disease c) Chronic empyema
c) Sturge weber syndrome d) All of the above
d) Grav's disease 991. Most malignant type of carcinoma breast is-
e) Neurofibromatosis a) Paget's disease (NIMHANS 86, JIPMER 87)
981. Primary hyperparathyroidism caused by -(PGI 02) b) Anaplastic carcinoma
a) Pararhyroid hyperplasia b) Adenosis c) Scirrhous's carcinoma
c) MEN 1 d) Thyrotoxicosis d) Atrophic Scerrhous carcinoma
e) CRF e) Mastitis carcinomatosa
982. TrueaboutMEN-1- (PGI 04) 992. In radical mastectomy the structures preserved
a) I VMA in urine b) t Calcitonin are all except- (JIPMER 97)
c) Hypergastrinemia d) Hyperprolactinaemia a) Axillaryvein b) Cephalicvein
e) J.. Ca2+ c) Nerve to serratus anterior d) Pectoralis minor

971)a 972) None 973) b 974) c 975) b 976) None 977) a 978) None 979) c 980) a,b,c,e 981) a,b,c 982) a,c,d
983)a 984)b 985)a 986)d 987)a · 988)d 989)c 990)d 991)e 992)d
SURGERY [ 228]

993. Estrogen receptor studies in carcinoma breast is 1007. Greenish discharge from the nipple is suggestive
doneon- (JIPMER 87) of- (UPSC 88)
a) Blood b) Urine a) Carcinoma b) Duct papilloma
c) Tumour tissue d) Ovary c) Duct carcinoma d) Fibrodenosis
994. Secondary deposits from carcinoma breast is 1008. Fibroadenoma of the breast are- (TN 89)
commonest in~ (AI 89) a) Fixed mass b) Diffuse mass
a) Lung b)Liver c) Multiple duffuse mass d) Solitary mobile mass
c) Brain d) Bone 1009. Breast examination is done yearly in patients with-
995. Risk factor for carcinoma breast- (A/89) a) Multiple fibroadenoma (PGI 88)
a) Fibroadenoma on one side b) Farnilyhistoryofca. breast
c) Carcinoma cervix
b) Sister dead from cancer breast
d) Endometrial carcinoma
c) Jewishorigin
1010. Carcinoma breastis seen in- (PGI 88)
d) All
a) Nulliparity
996. Bleeding per nipple is seen in (A/89) b) Positive family history
a) Duct papilloma b) Pagets c) High socioeconomic status
c) Fibroadenoma d) Carcinoma breast d) All
997. Paeu d' Orange is due to- (AI 89) 1011. In fibroadenosis, the highest risk of malignancy
a) Infiltration of cooper's ligament is seen in- (AIIMS 91)
b) Subcutaneous lymphatic involvement a) Fibrosis b) Epitheliosis
c) Infiltration of the nipple c) Asenosis d) Cyst formation
d) Infiltration of tumour to chest wall 1012. Treatment of choice in pagets disease of nipple
998. Commonest site for carcinoma of the breast- is- (AIIMS91)
a) Lowerinner quadrant b) Nipple a) Mastectomy b) 5-FU ointment
c) Upper inner quadrant d) Upper outer quadrant c) Irradiation d) Antibiotic cream
999. Gynaecomazia may be seen in patients with all 1013. In Radical mastectomy, which is spared-
except- (UPSC 88) a) Pectoralismajor (AIIMS92)
a) Cimetidine therapy b) Cirrhosis ofliver b) Pectoralis minor
c) Klinefelter's syndrome d) Turner's syndrome c) Axillary lymph nodes
1000. Bloody nipple discharge is usually associated d) Supraclavicular lymph nodes
with- (AIIMS 87) 1014. Which one predisposesto Breast cancer-
a) Lactation b) Galactocele a) Adenosis b)Fibrosis (AIIMS92)
c) Early lactation d) Intraductal papilloma c) Blue domed cysts d) Epitheliosis
1001. Acute mastitis commonly occurs during -(UPSC 86, 1015. A blood stained discharge from the nipple means
a) Pregnancy b)PubertyJIPMER88) that the patient may have- (JIPMER 86, UPSC 86)
c) Lactation d) Infancy a) Papillomaofthenipple b) Fibroadenoma
1002. Peau de orange is seen in- (PG/88) c) Duct papilloma d) Duct ectasia
a) Carcinoma breast b) Fibroadenoma 1016. Treatment of cysto sarcoma phylloids is -
c) Chronic abscess d) Mondor's disease a) Radicalmastectomy (Kerala 94, 87,
1003. Cracked nipple may be- (AIIMS84) b) Incisssional biopsy AI 90)
a) Due to syphilitic chancre c) simplemastectomy
b) Cause of retention cyst d) Modified radical mastectomy
c) Pagets disease of nipple 1017. Post operative radiotherapy in breast is given for-
d) For runner of breast abscess a) To prevent metastasis (JIPMER 95)
1004. Blood stained nipple discharge suggest-(AI/MS 84) b) Ablation of remnant of cancer tissue
a) Fibroadenosis b) Fibroadenoma c) To prevent recurrence
c) Duct papilloma d) Plasma cell mastitis d) Prevents distal metastasis
1005. Reconstruction of the breast following total 1018. Complication of post mastectomy lymph
mastectomy for cancer is done ideally by using- accumulation is- (JIPMER 95)
a) Distant tube pedicvle (AIIMS 84) a) Metastases of cancer b) Recurrence
b) Opposite breast c) Lymphosarcoma d)Pain
c) Trapezius myocutaneous flap 1019. Trueaboutpagets disease of the nipple is-
d) Latissmus dorsi myocunaneous flap a) Always there is underlying carcinoma (Kerala 95)
1006. The commonesy type of breast cancer is- b) Often bilateral eczema of nipple seen
a) Ductal b) Comedo (AIIMS 85, PGI 84} c) Histology reveals giant cells
c) Lobular d) Sarcoma d) Highly malignant

993)c 994)d 995)b,c 996)a,d 997)b 998)d 999)d lOOO)d lOOl)c 1002)a,c 1003)d 1004)c 1005)d 1006)a
1007)None 1008)d 1009)b,d lOlO)d lOll)b 1012)a 1013)d 1014)d 1015)c,d 1016)c 1017)c 1018)c 1019)a
SURGERY [ 229]

1020. StageT1N0M 0 in breastcancerindicates-(UPSC95) 1028. A mobile, variegated large lump in the breast of
a) Tumor more than 2 ems in diameter without axillary a 20-year old female is most Ukely to be due to-
nodes and absent metastasis a) Medullary carcinoma (UPSC 97)
b) Tumour 2 ems or less without axillary nodes and b) Inflarmnatory carcinoma
no metastasis c) Cystosarcoma phylloides
c) Tumour fixed to the chest wall without axillary d) Lobular carcinoma
nodes or distant metastasis 1029. Prognosis of Breast cancer is best determined by-
d) Tumour fixed to the underlying muscle with no a) Axillary node involvement (AIIMS 97,
nodes in axilla but with distant metastasis . b) Skininfiltration Kerala 89)
1021. Paget's disease of nipple is treated by- (Al95) c) Size ofthe tumour
a) Radiotherapy d) Estrogen receptor status
b) Biopsy and simple mastectomy 1030. In patey's mastectomy the step not done-
c) Radical mastectomy a) Nipple & areola removed (PGJ 96)
d) Chemotherapy b) Sorrounding normal tissue of tumour is removed
1022. Which one of the following statements is true of c) Pectoralis major removed
cystosarcoma phylloides- (UPSC 96) d) Pectoralis minor removed
a) It is a malignant tumour 1031. 40 years old female with a 2cms nodule in the breast
b) It often metastasises to axillary nodes & a proved metastatic node in the axilla, treatment
c) It is usually bulky and may fungate through the k- ~m~
skin a) Quadrantectomy
d) It is treated by radical mastectomy b) Mastectomy with local radiotherapy
1023. All of following cause gynecomastia except -(A196) c) Patey's with adjuvant chemotherapy
a) Klinefelters syndrome b) Cirrhosis d) Halstedt's operation with tacoxifen
c) Leprosy d) Undescended testis 1032. Single most prognostic factor in breast
1024. All of following may be causes of massive carcinoma- (Al98)
enlargement of breast except- (AIIMS 95) a) Age of patient b) Lymph node status
a) Elephantiasis c) Site of primary d) Invasion
b) Diffuse hypertrophy 1033. In radical mastectomy which of the following is
c) Schirrous carcinoma preserved- (AP 97)
d) Cystosarcoma phylloides a) Pectoralis minor b) Pectoralis major
1025. Of the following the most significant risk factor c) Cephalicvein d)None
for developing breast cancer is- (KARNAT96) 1034. Paget's disease ofthe nipple is- (AP 97)
a) The presence of sclerosing adenosis a) Infection b) Dermatitis
b) Nulliparity c) Neoplasia d) Hypopigmentation
c) Atypical lobular hyperplasia 1035. Following are true of Paget's disease of breast
d) Atypical ductal hyperplasia except- (Karnat 98)
1026. An adolescent boy present himself with bilateral a) Usuallybilateral
prominence of the breasts and wants the breasts b) Associated intraductal carcinoma
to be removed. Which one of the following c) Prognosis good in absence of lump
incisions would be ideal- (UPSC 97) d) Treatment simple mastectomy with axillary
a) Radial incision clearance
b) Incision along the areolar margin 1036. Regarding breast carcinoma all are true except-
c) Submammary incision a) Prevalence 21.2/100,000 population (AIIMS 98)
d) Elliptical incision b) Median age of presentation 42 yrs
1027. A 50-years old woman complains of intermittent c) More common in muslims
bleeding from the left nipple over the past 3 months. d) Family history is present
No mass is palpable, but a bead of blood can be 1037. Pre-menstrual fullness in breasts in 21years old
expressed from the nipple. The ideal procedure in unmarried female is- (AIIMS 98)
this case would be- (UPSC 97) a) Galactocele b) Fibroadenoma
a) Cytological examination of discharge and if no c) Fibroadenosis d) Breast cancer
malignant cells, to be kept under careful observation 1038. Greenish discharge of nipple is seen in- (AIJMS 98)
b) Segmental excision of breast a) Duct ectasia b) Duct Papilloma
c) Microdochotomy c) Fibroadenoma d) Ca breast
d) Simple mastectomy

1020)b l02l)b 1022)c 1023)d 1024)c 1025)c,d 1026)b 1027)c 1028)c 1029)a 1030)c 103l)c 1032)b
1033)c 1034)c 1035)a 1036)c 1037)c 1038)a
SURGERY [ 230]

1039. A distressing complication of radical mastectomy 1049. 4 em breast nodule with ipsilateral mobile LN in
is- (ORRISA 99) axilla staging- (PGI 2000)
a) Paralysis of the fifth finger of the hand a) T2N 1M0 b)T2N 2M 0
b) Oedemaofthearrn c) T 1N 1M0 d) T3N 2M 1
c) Loss of sensation of the medial side of the arm 1050. Conservative surgery in breast cancer is not to be
d) Frequent skin infections of the hand on the donein- (PGI01)
affectecd side a) Low socio-economic status
1040. Breast conservative surgery is done in all except- b) Age > 40 years
a) Young patients (UP 2K) c) Multicentricity
b) Ductal carcinoma in situ d) Lymph node involvement in axilla
c) Lobularcarcinoma e) Family history of breast cancer
d) Infiltrating ductal carcinoma 1051. Breast cancer is epidemiologically commoner with-
1041. A 45 years old lady presents with a lump in her a) Smoking b) Nulliparity (PGI 01)
right breast. The lump is 4 ems in diameter with c) Oral contraceptives d) Multiparity
evidence of cutaneous oedema (peau d' orange), e) First pregnancy after 30 years
not fiXed to pectoralis major muscle. The axillary 1052. Bleeding from nipple is seen in- (PGI 01)
lymph nodes are not enlarged. What is the status a) Fibroadenoma b) Duct ectasia
ofT in T.N.M. classifocation. (ICS 2K} c) Ductal papilloma d) Chronic breast abscess
a) T 1 >2 b) T2 is 2-5 e) Ca. breast
c) T3 >5 d) T4 is any size 1053. AboutMondor'sdisease- (PGI02)
1042. Moderately increased risk for invasive breast a) Superficial thrombophlebitis
carcinoma is associated with which of the following- b) Lymphatic infiltration tumour cell
a) Sclerosing adenoma (Kerala 2K) c) Cord like apperance of subcutanous veins
b) Apocrine metaplasia d) Occurs all over the body
c) Duct Ectasia 1054. CaBreaststagelandllmanagedby- (PG/02)
d) Atypical ductal hyperplasia a) Total mastectomy
e) Fibroadenoma b) Modified radical mastectomy
1043. Statistically important risk factors identified for c) Lumpectomy and axillary clearance
breast cancer- (PGI 2002) d) Lumpectomy axillary clearance and radiotherapy
a) Early age at marriage 1055. Which of the following indicate Ca breast -(PGI 02)
b) Females of non vegetarian diet a) Serons discharge b) Recentretractionofnipple
c) Those who have not breast fed their children c) Ulceration of nipple d) Cracked nipple
d) Smoking e) Cellular atypia
1044. Paget's disease of breast following are true except- 1056. Breast conservation surgery not indicated -(PGI 02)
a) Treated by simple mastectomy (PGI 97) a) Large pendular breast
b) Represents underlying malignancy b) SLE
c) Presents as eczema c) Diffuse microcalcification
d) Cytology diagnostic d) Bilateral carcinoma
1045. Blood stained discharge from nipple is indicative e) Family history
of- (PGJ97) 1057. Breast carcinoma is seen in women who-(PGI02)
a) Intra alveolar papilloma b) Duct ectasia a) Consume fatty food
c) Paget's disease d) LobularCa b) Have early menopause
1046. Peaud' orange is due to- (PGI98) c) Smoke
a) Skin involvement d) Have multiple sex-partners
b) Contraction ofligaments e) Did not breastfeed their children
c) Lymphatic permeation 1058. True about adjuvent therapy in Breast Ca -(PGI 03)
d) Bacterial infection a) Prognosis better if given in young female
1047. 4 em breast tumour with 2 ipsilateral involved b) - survival by 20 %
lymphonodescom under- (PG199) c) Nodal status positive gives good result
a) T3N 1 b) T 3N 2 d) Hormone receptor positive gives good result
c) T2N 1 d) T4N 1 e) Not associated with- survival
1048. All are indicators of malignancy in a mammography 1059. Lymphatic drainage of breast- (PGI 03)
except- (PGI 99) a) Axillary b) Supraclavicular
a) Nodular cacification b) Speckled margin c) Internal mammary d) Mediastinal
c) Attenuated architecture d) Irregular mass e) Celiac

l039)b 1040)c l04l)d 1042)d 1043)c l044)d 1045)b l046)c l047)c 1048)a l049)a 1050)a,c 1051)a,b,e
1052) b,c,e 1053) a,c 1054) b,d 1055)d,e 1056) a,b,c I 057) a,e 1058) a,b,c,d 1059) a,b,c
SURGERY[231]

1060. Breast lump is best diagnosed by- (PGI 03) 1069. Commonest cause ofblood discharge from the nipple
a) FNAC b) Bilateral mammography is - (SGPGI 05)
c) USG d) CECT of breast a) Duct carcinoma
e) Chest X-ray b) Duct papilloma
1061. True statement (s) about nipple discharge is/are c) Chronic cystic mastitis
a) Mammography is diagnostic (PGI 04) d) Cystosarcoma phyllodes
b) Cone excision done in single intraductal tumour 1070. With reference to mammography, which one of the
c) Mammography done when duct papilloma is >4.5cm
following statements is correct- (UPSC 05)
d) Red discharge indicate malignancy
a) A baseline study should be done for all women at
e) Blue-black discharge indicate duct ectasia
1062. True about screening mammography- (PGI 04) age30
a) Indicated in 50-70 years of age b) It uses less radiation energy than a chest X-ray
b) Mortality reduced by 30% c) It should be part of the regular follow up of a woman
c) Radiation due to mammography can cause following therapy for unilateral breast cancer
carcinoma d) It provides an effective substitute for biopsy of
d) MRI is better than mammography suspicious lesions
e) USG is better than mammography 1071. Treatment of choice for Phylloides tumor-(MAHE 05)
1063. In a mammogram all ofthe following are features a) Excision b) Radical mastectomy
of carcinoma breast except- (UPSC 02) c) Radiotherapy d) Chemotherapy
a) A solid lesion with ill defined adge or stellate 1072. True aboutlymphatic spread ofCa breast -(PG!June
configuration a) Axillary nodes are most commonly involved 05)
b) True microcalfication b) Internal mammary nodes are also involved
c) Areas ofmacrocalcification c) If supraclavicular L.N. is involved, then it is N3
d) Increased skin thickness d) Axillary nodes are treated by surgical resection
1064. Flap commonly used in breast reconstruction is-
1073. True about duct ectasia- (PGI June 05)
a) Serratus anterior b)1RAM (TN03)
a) AIW smoking
c) Flap from arm d) Delto pectoral flap
1065. Use oftamoxifen in carcinoma of breast patients b) puckering of skin over the breast seen
does not lead to the following side effects- c) Abscess formation seen
a) Thromboembolic events (AIIMS 03) d) Treated by mastectomy
b) Endometrial carcinoma 1074. Breast cancer is conservatively treated by following
c) Cataract methods- (PGI June 05)
d) Cancer in opposite breast a) It is based on relative size of breast tumor
1066. A 75 year old hypertensive lady has a 2x2 em b) Radiotherapy is given
infiltrating duct cell carcinoma in the subareolar c) Chemotherapy is given
region. There are no palpable lymph nodes and d) Surgery is best modality of treatment
distant metastases. However, she had been treated 1075. Features, which are evaluated for histological
for pulmonary tuberculosis 20 year ago. The best grading of breast carcinoma, include all of the
cause of management for her would be- (UPSC 04) following except- (AIIMS Nov 05)
a) Modified radical mastectomy followed by a) Tumour necrosis b) Mitotic count
radiotherapy c) Tubule formation d) Nuclear pleomorphism
b) Modified radical mastectomy followed by 6
1076. In Patients with breast cancer, chest wall
cyctes of chemotherapy
involvement means involvement of any one of the
c) Breast conservation surgery followed by
radiotherapy following structures except- (AIIMS Nov 05)
d) Modified radical mastectomy followed by a) Serratus anterior b) Pectoralis Major
hormone therapy c) Intercostal Muscles d) Ribs
1067. Indian file pattern is seen in breast cancer 1077. A blood stained discharge from the nipple indicates
type- (APPGE 04) one of the following- (AI 05)
a) Intraductal b) Infiltrating lobular a) Breast abscess b) Fibroadenoma
c) Infiltrating ductular d) Infiltrating ductular c) Duct Papilloma d) Fat Necrosis of Breast
1068. The most frequently used procedure for diagnosing 1078. Inpatehyprocedure- (SGPG/05)
palpable breast masses is- (SGPGI 05) a) Removal ofboth pectoralis major & minor
a) Core-cutting needle biopsly b) Removal of pectoralis minor and preservation of
b) Fine needle aspiration pectoralis major
c) Excisional biopsy c) Pectoralis minoris divided
d) lncisional biopsy d) Both (major & minor) are preserved

1060)a 1061)a,b,d,e 1062)a,b,c 1063)c 1064)b 1065)d 1066)d 1067)b 1068)b 1069)b 1070)c 1071)a
1072)All 1073) a,b,c 1074) a,b,d 1075) a 1076) b 1077) c 1078) b
SURGERY [ 232]

1079. Treatment of hormone dependent fungating 1088. Surgery for periductal mastitis- (MAHE 07)
carcinoma ofbreast with secondaries in the lung in a) Hadfield's operation
a female patient aged 30 years is- (MAHE 05) b) Patey's mastectomy
a) Simple mastectomy followed by oophorectomy c) Modified radical mastectomy
b) Radical mastectomy followed by oophorectomy d) None of the above
c) Adrenalectomy 1089. Thue about breast carcinoma in men-
d) Lumpectomy followed by castration a) Estrogen receptor positive (PG1 June 07)
1080. The most important prognostic factor in breast b) Associated with gynaecomastia
carcinoma is- (AI 06) c) Radiotherapy contraindicated due to close
a) Histological grade of the tumor proximity to chest wall
1090. Aromatase inhibitors inCa breast- (PG1 June 07)
b) Stage of the tumor at the time of diagnosis
a) Letrozole b) Anastrozole
c) Status of estrogen and progesterone receptors
c) Exemestane d) Tamoxifen
d) Over expression ofp-53 tumour suppressor gene
1091. Consider the following statements regarding
1081. In which of the following types of breast carcinoma, Paget's disease of the breast- (UPSC-11 08)
would you consider biopsy of opposite breast- 1. It is a malignant disease
a) Adenocarcinoma-poorly differentiated (AI 06) 2. Diagnosis can be established by scrape cytology
b) Medullary carcinoma 3. Lymph nodes involvement is an associated clinical
c) Lobular carcinoma feature
d) Comedo carcinoma 4. Treatment of choice is simple mastectomy
1082. Tylectomyliterallymeans- (DNB 06, 91) Which ofthe statements given above is/are correct-
a) Excision of a lump b) Excision of a breast a) 1, 2 and4 only b) 1, 2 and 3 only
c) Excision ofLN d) Excision of skin c) 3and4only d)l,2,3and4
1083. Which sutures are preserved in Scanlon's modified 1092. A 25 year old female complains of discharge of blood
raical mastectomy- (Karnataka pgmee 2006) from a single duct in her breast The appropriate
a) Lateral pectoral nerves treatmentis- (AI 08)
b) Level III nodes in axilla a) Radical excision b) Microdochectomy
c) Pectoral fascia c) Radical mastectomy d) Biopsy to rule out carcinoma
d) Nipple and the areola 1093. Thue about treatment of early breast carcinoma-
1084. In which of the following types of carcinoma breast, a) Aromatase inhibitors are replacing tamoxifen in
comedo growth pattern is seen? (Karn 06) premenopausal women (Aiims May 08)
a) Ductal carcinoma in situ b) Postmastectomy radiation therapy is given when
b) Medullary catcinoma 4 or more lymph nodes are positive.
c) Lobular carcinoma in situ c) Tamoxifen is not useful in post-menopausal women
d) Infiltrating lobular carcinoma d) In premenopausal women, multidrug chemotherapy
1085. The following is a maker of Paget's disease of the isgiven in selected patients.
mammary gland- (AI 07) 1094. Treatment for periductal mastitis- (Manipal 08)
a) S-100 b)HMB45 a) Hadfield's operation
c) CEA d) Neuron specific enolase b) Patey's mastectomy
1086. AU of the following statements about Gynaecomastia c) Modified radical mastectomy
are true except- (AI 07) d) Radical mastectomy
a) Subcutaneous mastectomy is the initial treatment 1095. Prognosis ofbreast carcinoma is best determined by?
of choice a) Oestrogen/progesterone receptors (APPG 08)
b) Seen in liver disease b) Axillary lymph node status
c) There may be estrogen I testosterone imbalance c) Clinical assessment
d) Can be drug induced d)CT
1087. 30 yrs female presented with unilateral breast 1096. Distressing Complication after modified radical
cancer associated with axilary lymph node mastectomy? (APPG08)
enlargement Modified radicalmastectomywas done, a) Lymphedema b)Axillaryvein thrombosis
further treatment plan will be- (Aiims May 07) c) Seroma d) Death
a) Observation and followup 1097. Thueaboutgalactorrhoea- (PG1 Dec 08)
b) Adriamycin based chemotherapy followed by a) Always BIL
tamoxifen depending on estrogen/progesterone b) Found in pregnancy & lactation
receptor status c) Associated with prolactinoma & other endocrinopathies
c) Adriamycin based chemotherapy only d) Surgery is done
d) Tamoxifen only e) Hypothyroidism can cause galactorrhoea

1079)a 1080)b 1081)c 1082)a 1083)a 1084)a 1085)c 1086)a 1087)b 1088)a 1089)a,b 1090) a,b,c 1091)d
1092)b 1093)b 1094)a 1095)b 1096)a 1097)c,d,e
SURGERY [ 233]

1098. Ipsilateral supraclavicular lymphnodes are positive 1108. Which oneofthefollowingis NOT a correct statement
in a patient of Ca Breast. Stage is -(AIIMS Nov 08) with reference to locally advanced carcinoma breast-
a) II b)illB a) It constitutes the bulk of patients of carcinoma
c) illC d)N breast in India (UPSC-II 09)
1099. A women noticed mass in her left breast with bloody b) Patients are staged as T 3 or T4 with any N, without
discharge. Histopathology revealed duct ectasia. distant metastasis (M 0)
Treatment is- (AIIMS Nov 08) c) Neoadjuvant chemotherapy downgrades the
a) Simple mastectomy b) Microdochotomy disease
c) Lobectomy d) Hadfield operation d) Radical mastectomy is the treatment of choice
1100. Triple assessment for CA Breast includes- (AI 09)
1109. In a patient with breast cancer, the following are
a) History, Clinical examination and mammogram
poor prognostic factors except- (UPSC-I! 09)
b) History, Clinical examination and FNAC
a) High grade
c) USG, MammogramandFNAC
b) Absence of epidermal growth factor receptor
d) Clinical examination, Mammogram and FNAC
1101. Moderately increased risk of invasive Breast c) Aneuploid status
carcinoma is associated with which of the following d) Age less than 35 years
benign lesions of the breast- (AI 09) 1110. Lymph node first involved inCa breastis/are-(PGI
a) Sclerosing adenosis a) Axillary b)Intemalmammary Nov09)
b) Atypical lobular hyperplasia c) Supraclavicular d) Contralateral axillary
c) Apocrine metaplasia 1111. Paget's disease of breast, true are- (PGI Nov 09)
d) Squammous metaplasia a) Intraductal carcinoma b) Mastectomy needed
1102. A well differentiated follicular carcinoma of thyroid c) Malignant d) Bilateral
can be best differentiated from a follicular adenoma 1112. Breast conservative surgery is done in all except-
by- (AJ09) a) Young patients (DPG 10)
a) Hurthle cell change b) Ductal carcinoma in situ
b) Lining of tall columnar and cuboidal cells c) Lobularcarcinoma
c) Vascular invasion d) Infiltrative ductal carcinoma
d) Nuclear features 1113. In patients with breast cancer, chest wall involvement
1103. Blood stained nipple discharge is seen in- (PGI June means involvement of any one of the following
a) Breast abscess b) Fibroadenoma 09) structures except- (DPG 10)
c) Ductal papilloma d) fat necrosis a) Serratus anterior b) Pectoralis'Major
1104. True regarding male breast Ca- (PGI June 09) c) Intercostal Muscles d) Ribs
a) MC lobular type 1114. Mondor'sdiseaseis- (Maharashtra 10)
b) Estrogen receptor positive a) Superficial thrombophlebitis ofbreast
c) WO gynaecomastia may be present b) Fat necrosis in breast
d) Paget's disease of nipple is more common in male c) Postradiation breast and arm edema
than female d) Skin infection over the breast
e) Undescended testes is a risk factor 1115. In carcinoma breast, adjuvant radiotherapy is
1105. The most important prognostic factor in carcinoma indicated after modified radical mastectomy in all of
breastis- (DELHI PG Feb. 09) the following, except- (UP SC II 10)
a) Sizeoftumour a) ER, PR hormone receptor negative tumour
b) Skin involvement b) Tumour size more than 5 em
c) Involvement of muscles c) Positive margins
d) Axillary gland involvement d) More than four positive axillary lymph nodes
1106. Rare histological variants of carcinoma breast with 1116. Which of the following does not alter the 'T' stage in
better prognosis include all except- breast cancer? (UP SCI! 10)
a) Colloid carcinoma (DELHI PG Mar. 09) a) Peaud'orange
b) Medullary carcinoma b) Skin ulceration
c) Inflammatory carcinoma c) Pectoral muscle involvement
d) Tubular carcinoma d) Serratus anterior muscle involvement
1107. Which type of breast cancer is most likely to be 1117. Treatment with herceptin in breast cancer is
bilateral- (UPSC-II 09) indicated for- (UP SC II 10)
a) Infiltrating duct carcinoma a) ER receptor +ve tumours
b) Paget's disease of breast b) PRreceptor+vetumours
c) Lobular carcinoma of breast c) K: 67 stain +ve tumours
d) Medullary carcinoma ofbreast d) Tumours with over-expressed C-erb B-2 protein

1098)c 1099)d llOO)d llOl)b 1102)c 1103)c 1104)b,c,e 1105)d 1106)c Il07)c 1108)d 1109)c lllO)a,b
llll)a,b,c 1112)c 1113) b 1114)a 1115)a 1116)c 1117)d
SURGERY [ 234]

1118. True about leiomyosarcoma breast- (PGI Nov. 10) ''"'fi0·w.~llt,()fJor•~pt/'{;,. · ·.: . . .··(NEJi/E!13ffil8:·Eatte~ilj
a) Axillary lymph mode dissection is mandatory ··":' ''}Jt)>Radiother~pyr .•> • "
b) Well encapsulated · . b)Neo.:adjuvant cliem?tb.erapy
c) Follow up not required .· · c)'-~vfadifi~.4r:adi~~lTastectomy
d) Mastectomy is mainstay treatment '1.1~;;,;1•f(l)~~lema!1Ste~0irly<r•·r .· ·r··;·;. • .,.,, ··• ., .. ,,•. ,;.;
e) Metastasize by lymphatic channel ~iil~.}~·"Jli~~ ~'••sm~u.t: ;fol'·~el\.'0B,r~ttst. ifiell(~~sall
1119. Risk factor for CA breast- (PGI Nov. 10)
a) OCP
c) Nulliparity
b)BRCA-1 mutation
d) Multiparity
<('ii;.:~.~;~~~!~ksett~x:nrrination'r' ~ti:&f~·~;tte~)
e) Family history J'it•r•· d)•NiaJmri'
< ·...•...• ·.•·.·· ••• .• ••...•..·•
fi\11~ llhuut HERZ!il¢1l'~y~ie .·
1120. BRCA 1 gene is located on?
a) Chromosome 13
(AIIMS May 11)
b) Chromosome 11 Q~~·>.:~:·.· ·····:..· f1:f~JS'ttz?n1!~if~im;·X'
c) Chromosome 17 d) Chromosome22
1121. Gynaecomastia may occur in tbe following conditions
except- (UPSC I 11)
ahti~6d.ie~".,
/·· ,_._. _-:-- ___ ":. ____ : _;:;.;}iJ?_:.¥A_ --~-'i).i:K:~<-:_:_>,;-~,
a) Physiologic response to puberty o»:. ~ast cancer . <;,. >.......·.•;.;.~:
b) Primary hypogonadism l' ' ll < ·~~ .·· .. ·..· rlij.~'v~;b: ·•1iiic•:•

c) Primary aldosteronism a) i\Pa.Jl.dLa~er,a,J ...... •. · • ;.f?(!;ttf;rn)


d) Choriocarcinoma testis l?).t\_PanltM~qh~~~r
1122. A patient has a 6-cm breast tumour with mobile, . ·•·. ,c)Qr~(}:-Ralldaland.. l#te,rW:.vi~~>,i , >

clinically positive, ipsilateral axillary lymph nodes •· ·~)·€taniP;~cau<Ialag~M~~j~4~t~mi9bi~9¥~·L ••.. ••.••


and no evidence of distant metastasis. The stage of . . . :rf~lq~~t. . ~~i~.~f'p~,~~ r ....... · .....•.

the breast cancer is- (UPSC II 11) ~~;·)pj<ir9 .. ' c~~(),n~·~Jia~sis)Je~t


a) Stage I b) Stage II b ;7i'N1l.ij . .. ~~~gement~ ......·.···• (NE$T!!Jf1B!N~~t~)
c) Stage III a d) Stage III b .· • • a}~f~ast .~IJJ1S~rvative 'slll;gery• ' · • <
1123. Following radical surgery for the carcinoma of b~~S:irn!l~~~asrecwmy·• .;,, • :: x c,.
breast, reconstruction of the breast can he • ••. .• .•. c}~M.
.. ~.difi~4 r&li~l mast~ctofny•: " . ' ...
performed by using tbe following procedures except- ~'>.(;;·-;-::/:-;;41~~(t!~~~r~p~;::-,·_. _ -,,; :-_--.-_}--_:'::~:J(:.:~:---·<:-_~~->- _·;.: :-:,:--._-_._:_:~/> .>'-__ -:_-.c_:;-;
a) Silicon implants (UPSC II 11) ~~rJ~~~ter1wA~~~lrl~Yi'li~ea~tt~r~~t\~fmcfftil1£fs·4on¢
b) Latissimus dorsi flap (LD flap) ··;~l. i·ily-:•:~p~~·;:? .·;· ·. ·...·· · •.· ·• :yp.fEF/r!EfHIJJ}atteril).
c) Transversus abdorninis muscle flap (TRAM flap) .• ·. arP~ctdfal1s:m~j?rmyocufarte01iSflap . .
d) Deltopectoral flap , :·. .· ·~) Deltoid myoctitaneous fl!lp" • · ·
11~tJ['A ··~JiEpr
• c) Serr~tus anteriot .9ap
·. •. ·• .·. d) Latissiluus tibisi n,iy~cutarieo~s flap •·
tp3~·.····~p~a.ti~ntP,~esentsWi.tb.br~~stca.• InsJ)~~nshows
.;····. ·•·P~~ud1 o~ang~ appe~r~uice.WJta.~!~>(lle,Tstageof
t~tiln!!t,., , · ... ••·· . . . . .. ·. · • (NEEJ'/!J]VB,Pq.ttem)
.~;t~J·~.~~.·;~;~.;'';r·••··.···~j.p!~~.· ·.· • .. . l· •.•..• ·
1134;· .·.. f~m~lepresentsmth.duet3t~~illsitll4Jfbrellst
. with dif(use.microcaldfic~fi()n~ on magul'~graphy.
SheitlsogivesfaD!iiY:·.mst()I;Y4Jf.·)),reast.c.8.WJtati.s
. b~tpossible m~t1g(lm~nt~. (NEETIDJ:{Jl ])q.ttern) •
g~;:4~''"fl2)"l!~"~t~(<~~<tti:~~·sw$l'Y ... · ~:·. . ;J
'b) ~iniple:.~li$t~c.k>my, ,. · · . ....
•.••• ,. ;Jz9 :Mol$fi~tl tadical mastectom~J
it) RadiQtherapy • . . .• . . . . . .... · ..
1~35; &~djotherapy after•ma,S'tectnmywoultlnl)t. })e
· ·•l'e<Jlliredin which ofthefollo\}'in:~:case scenarios
•; .; .·••lj{;hrea$~ea'".. · • · · · · "(NEET1DN!BP/iuetft)•
•)•: ··ay:lliargetrun()r
b) N!et~stasis ··
.~l' ~il1aty nqdal it\y()lvemenf
. d) PQsitive margins ·. ·.

1118)b,d 1119)b,c,e 1120)c 1121)c 1122)d 1123)d 1124) c 1125) a 1126) b 1127) b 1128) a 1129) c 1130) d
113l)b 1132)d 1133)d 1134)b 1135)b
SURGERY [ 235]

1147. The organism most frequently related to mediastinal


fibrosis is - (AIIMS 81, PGI 85)
a) Actinomycosis b) Histoplasma
c) Hansen bacillus d) Staphylococcus
1148. In a patient of mesothelioma, one often finds-
a) Hypoglycaemia (PGI 79, 80, AIIMS 84)
b) An association with asbestosis
c) Haemorrhagic pleural effusion
d) Clubbing offmgers
e) All of the above
THORAX 1149. Failure of establish adequatedrainagein an empyema
with a bronchopleural fistula present is indicated
1138. A patient with mesothelioma, which ofthe foUowing by- (JIPMER 81, AMC 85)
are seen in- (PDf 87) a) Drainage less than 100 c.c. per day
b) haemorrhagic drainage less than 100 c.c. per day
a) Associated with asbeestos
c) The development ofhaemoptysis
b) Hemorrhagec pleural effusion
d) Continued productive cough with purulent
c) Hypoglycmemia
material
d) All of the above
e) All of the above
1139. Best treatment of tension pneumothorax is-
1150. The immediate treatment of choice in most cases
a) Immedisate letting out of air (AIIMS 89) of flail chest is- (PGI 81, AMC 86)
b) Wait and watch a) Analgesics b) Positive pressure ventilation
c) Rib resection c) Tracheostomy d) Breathing excercises
d) Underwater drainage 1151. Meig's syndrome consists of the foUowing except-
1140. Commonest posterior mediastinal tumour a) Ascites (Karn. 94)
is- (UPSC 85, NIM. 87, AIIMS 87, Keral87) b) Hydrothorax
a) Lung cyst b) Neurofibroma c) Benign ovarian tumour
c) Dermoid d) Thyroid d) Malignant ovarian tumour
e) Thymic tumour 1152. Most common cause of cold abscess of chest waD is-
1141. The treatment of choice in severe flail chest-W 89) a) Pott's spine (TN 95)
a) IPPV b) Strapping b) TB abscesses of chest wall
c) Wiring ' d) Nasal oxygen c) TB or ribs
1142. Spontaneous pneumothorax exceeding ..... % of d) Intercostal lymphadenitis
chest cavity should have a chest tube inserted - 1153. Broncholithiasis means- (Kerala 96)
a) 10 b)25 (AIIMS 84) a) Calcified lymphnodes eroding into bronchus
c) 45 d)60 b) Foreign body calciifed in bronchus
1143. Commonest symptom associated with thoracic c) Lithium deposition in bronchial wall
outlet syndrome is- (PGI 87) d) A hamartoma
1154. What is the 1st thing to be done to a patient with
a) Intermittent claudication
tension pneumothorax- (CUPGEE 96)
b) Pain on radial distribution
a) Insertion of wire bore needle in the inter costal space
c) Pain in ulnar distribution
b) Water seal drainage
d) Gangrene
c) Leave the patient at rest for air to be absorbed
1144. FoUowing is true of eventration of diaphragm- d) None ·
a) It ia a developmental defect (AIIMS 84) 1155. Tenson pneumothorax due to fracture rib is treated
b) Early surgery is treatment by- (PG/96)
c) Defect is usually muscular a) Strapping
d) Diagnosed mostly clinically b) Tube drainage
1145. Emphysema may be found in the following c) IPPV
conditions- (Kerala 98) d) Internal fixation with open reduction
a) Chest injury b) Tracheostomy 1156. Emergency treatment of tension pnenmothorax in
c) Surgical wound d) All the causality Department is- (PGI 96)
1146. In pneumothorax due to blunt injury, treatment a) Chest X-ray
ofchoiceof- (AIIMS92) b) Puncture by wide lose needle
a) Observation b) Pneumonectomy c) Tube drainage
c) Thoracotomy d) Intercostal drainage d) IPPV

1136)d 1137)d 1138)d 1139)a 1140)b 1141)a 1142)b 1143)c 1144)a,c 1145)d 1146)d 1147) b 1148)e ll49)d
1150)a 1151)d 1152)a 1153)a 1154)a 1155)b 1156)b
SURGERY [ 236]

1157. Cause of death in blunt trauma chest is- 1168. One of these muscles is not cut in postero-lateral
a) Pulmonary laceration (AJ/MS 96) thoracotomy- (PGI98)
b) Rupture of oesophagus a) Serratus anterior b) Pectoralis major
c) Rupture thoracic duct c) Latissimus dorsi d) Rhomboidus major
d) Tracheo bronchial rupture 1169. The most common primary tumour of mediastinum-
1158. Commonest lesion in middle mediastinum- {AIIMS a) Lymphoma b) Teratoma {Jipmer 92)
a) Congenital cyst b) Lipoma 96) c) Neurogenic tumour d) Thymoma
c) Aneursym d) Neurogenic tumours 1170. Most common mediastinal tumour is- (Karn 94)
1159. All are seen in Thoracic outlet syndrome except- a) Lymphoma
a) Mass in the Neck (PG197)
b) Neurofibroma
b) Wasting of foream muscles
c) Thymic tumour
c) Adson's test positive
d) Secondaries from bronchial carcinoma
d) Pallor
1171. Valvoplasty done in foUowing except- (PG197)
1160. Thymectomycause- (TN98)
a) Failure of rejection of transplanted organs a) Coarctationofaorta b)PS
b) Myastheina gravis c) MS d)AS
c) Autoimmune disorders 1172. Regarding pectus excavatum aU are true except-
d) None of the above a) Gross CVS dysfunction (PG197)
1161. Tension pneumothorax results in- (JCS 98) b) Decrease in lung capacity
a) Alkalosis c) Cosmetic deformity
b) Increased cardiac output d) Depression in chest
c) Decreased venous return 1173. The ideal treatment for hemothorax of blood loss
d) All of the above greater than 500 ml/hour- (PGI99)
1162. Traumatodiapbragm,trueis- (AIIMS98) a) Wait and watch
a) Usuallybilateral b) Needle aspiration
b) Left side more common c) Intercostal tube
c) Conservatively managed d) Open thoracotomy with ligation of vessel
d) Results in pneumothrorax usually 1174. Absolute contraindications of heart transplantation
1163. Thoracotomy is indicated for all except-(AJIMS 98) a) HIV infection (PG12000)
a) Esophageal laceration b) Age > 60 years
b) Bleeding > 200 m1lhr c) Irreversible pulmonary hypertension
c) Massive air leak d) Significant pulmonary vascular disease
d) Pulmonary contusions e) Malignancy
1164. In congenital diaphragmatic hernia all are seen except- 1175. Abouthemothorax- (PGI02)
a) Common on left side (JIPMER 99) a) Seen in choriocarcinoma
b) Abdominal distension b) Supine posture is better than erect posture
c) Can be detected antenatally c) Needle aspiration may be needed for diagnosis
d) Heart beat shifted to right d) Thoracotomy is always done
1165. Morgagnihernia- {AP 98) 1176. Treatment of rib fracture- (PGI02)
a) Hernia between the costal and sternal part of the a) Immediate thoracotomy b) IPPV
diaphragm c) Analgesics d) Strapping
b) Hernia through the pleuriperitonial canal e) ICWSD
c) Hernia through the lumbar triangle 1177. Rx ofHaemorrhoids- (PG103)
d) Hernia through inguinal canal a) Banding b)Ligation
1166. About diaphragmatic hernia aU are true except- c) Cryocoagulation d) Coagulation
a) Most commonly right sided & (MP 2K) e) Photocoagulation
anteriorly 1178. Posterior mediastinal tumours- (PGI 03)
b) Unilateral pulmonary agenesis a) Neuroblastoma b) Bronchogenic cyst
c) Mediastinal shift c) Neuroenteric cyst d) Lymphoma
d) Gastric volvulus commonly associated e) Anterior thoracic meningioma
1167. Which of the foUowing is the best site for intercostal 1179. True about Flail chest - (PG104)
drainage in a case of pleural effusion- (AI 02) a) #of3or4ribs
a) Second intercostal space midclavicular line b) Chestwall moves inwards during inspiration
b) Fifth intercostal space just lateral to vertebral column c) Mechanical Ventilation always needed
c) Fifth intercostal space in midclavicular line d) Mediastinal shift
d) Seventh intercostal space in midaxillary line e) Ultimately leads to respiratory failure

1157)d 1158)a 1159)b 1160)a 116l)c 1162)b 1163)d 1164)b 1165)a 1166)a 1167)d 1168)b 1169)c 1170)d
1171) a 1172) a 1173) d 1174) a,c,e 1175) a,b,c 1176) b,c 1177) a,b,c,e 1178) a,b,c,d 1179) a,b,e
SURGERY [ 237]

1180. Anterior mediastinal tnmors is/are- (PGI 04) 1189. Tumours of anterior mediastinum include the
a) Thymoma b) Aortic aneurysm following except- (UPSC 07)
c) Bronchogenic Cyst d) Lymphoma a) Thymoma b) Lymphoma
e) Bochdalek hernia c) Germ cell tumour d) Schwanoma
1181. Empyema necessitities is defined as so when- 1190. Steering wheel injury on chest of a young man reveals
a) Plural empyema is under pressure (UPSC02) multiple fractures of ribs and paradoxical movement
b) Pleural empyema has ruptured into bronchus with severe respiratory distress. X-ray shows
c) Pleural empyema has ruptured into the pericardium pulmonary contusion on right side without
d) Pleural empyema is showing extension to the pneumothorax. What is the initial treatment of
subcutaneous tissue choice? (UPSC 07)
a) Immediate internal fixation
1182. In a patient with one episode of Spontaneous
b) Endotracheal intubation and mechanical ventilation
pneumothorax, Which is advised- (Jharkand 03)
c) Thoracic epidural analgesia and 02 therapy
a) Stop diving b) Stop smoking
d) Stabilization with towel clips
c) Stop flying d)All
1191. Which of the following is not a complication of
1183. The four points of probe placement in Focused surgery for thoracic outlet syndrome-
Abdominal Sonogram for trauma (FAST) in blunt
a) Pneumothorax (Aiirns May 07)
thoraco-abdominal trauma are- (Kamataka 04)
b) Brachial plexus injury
a) Epigastrium (R) hypochondrium, (L) Lower
c) Lymphocutaneous fistula
chest, hypogastrium
d) Long thoracic nerve injury
b) Epigastrium, (R) and (L) Hypochondria, (R) Iliac
1192. Most common tumor of anterior mediastimum is-
fossa
a) Thymoma b) Lymphoma (MAHE 07)
c) Epigastrium, (R) and (L) Lumbar regions,
c) Germ cell cumor d) Thyroid tumors
hypogastrium
1193. The commonest anterior mediastinal tumour is-
d) Hypogastrium (R) and (L) Lumbar regions, (R)
a) Aneurysm of descending aorta (Corned 07)
lower chest
b) Neurogenic tumour
1184. Treatment of choice for Tension pneumothroax is- c) Thymoma
a) Immediate IC tube drainage (SGPGI 05)
d) Bronchogenic cyst
b) Continuous aspiration by needle
1194. Heimlich valve is used for drainage of- (Corned 08)
c) Intermittent aspiration by needle
a) Pneumothorax b) Hemothorax.
d) Thoractomy with repair ofleakage c) Emphyema d) Malignant pleural effusion
1185. TRUE regarding management of traumatic
1195. The following are indications for performing
pneumothoraxis- (SGPG/05)
thoracotomy after blunt injury of the chest, except-
a) Immediate ICD tube insertion a) 1000 ml drainage after placing an intercostal tube
b) CT-scan should be done to confirm pulmonary leak
b) Continuous bleeding through intercostal tube of
c) Intermittent needle aspiration
more than 200 mllhour for three or more hours
d) Sealed
c) Cardiac tamponade (UPSC-II 08)
1186. Treatment of choice for medullary carcinoma of
d) Rib fracture
thyroid to- (AIIMS May 2005)
1196. What is the treatment of choice in severe flail chest?
a) Total thyroidectomy b) Partial thyroidectomy
a) IPPV b) Strapping (UPSC-11 08)
c) F31 ablation d)Hemithyroidectomy
c) Wiring d) Nasal Oxygen
1187. Which of the following statement is true regarding
1197. Decision regarding surgery in a case of hemothorax
subclavian steal syndrome- (AIIMS Nov 05)
due to blunt trauma chest should be based on-
a) Reversal ofblood flow in the ipsilateral vertebral artery
a) Chest symptoms (AI 08)
b) Reversal ofblood flow in the contralateral carotid artery b) Hemodynamic status
c) Reversal ofblood flow in the contralateral vertebral
c) Nature of chest tube output
artery
d) X-ray fmding
d) Bilateral reversal ofblood flow in the vertebral arteries
1198. Most common tumor in the posterior mediastinum
1188. Amanis broughttocasualtywhometwithRTA.He
is- (AI 08)
sustained multiple rib fractures with paradoxical
a) Neurofibroma b) Teratoma
movement of chest. Management is- (PGI June 06)
c) Lymphoma d) Bronchogenic cyst
a) Tracheostomy
1199. True about chest trauma- (PGI June 08)
b) Consult cardiothoracic surgeon
a) ECG done in all cases alw sternal fracture
c) Strapping ·
b) Under water seal drainage if alw pneumothorax
d) Intermittent positive pressure ventilation
c) X-ray chest investigation of choice
e) No intervention required
d) Urgent surgery needed in all cases

1180)a,b,d 118l)d 1182)d 1183)c 1184)a 1185)b 1186)a 1187)a 1188)d 1189)d 1190)b 119l)c 1192)a
1193)c 1194)a 1195)d 1196)a 1197)b>c 1198)a 1199)a,b,c
SURGERY [ 238]

1200. Most common posterior mediastinal tumour is- 1210. Blood stained sputum may be the only symptom
a) Dermoid b) Neurofibroma (DPGEE 08) in- (KERALA90
c) Lipoma d) Lymphoma a) Bronchietasis b) Carcinoma bronchus
1201. Thoracic outlet syndrome is primarily diagnosed c) Adenoma brounchus d) Pulmonary T.B.
by- (AJ09) 1211. Intralobar sequestration of lung is commonest in
a) Clinical Evaluation b) CT scan the- (JIPMER81 UPSC89)
c) MRI d) Angiography a) Apical segment of upper lobe
1202. Which one of the following statements is NOT b) Medial segment of middle lobe
correct regarding thoracic inlet syndrome- c) Lateral basal segmentoflower lobe
a) Radial nerve is involved (UPSC-I! 09) d) Posterior basal segment oflower lobe
b) Resection of first rib is effective treatment 1212. Extralobar bronchogenic cysts may communicate
c) Physiotherapy and position exercises relieve with the following except- (JIPMER 80,AMU 89)
symptoms a) Oesophagus b) Stomach
d) Neurological signs and symptoms are common c) Brounchus d) None of the above
1203. Anentityofpneumothoraxwhichleadstobuildupof 1213. Complication toPEEPincludeallexcept-(PG/81, UPSC
positive pressure within the hemithorax, leading to a) Pulmonary oedema b) Emphysema 87)
collapse oflung, flattened diaphragm, mediastinal c) Cardiogenic shock d) Pneumonia
shift to contralateral side and eventually the venous 1214. The greatest incidence ofbronchopleural fistula
return to the right side of heart is compromised is is follwing- (PGI 80, DELHI 88)
known as- (UPSC-I! 09) a) Segmental resection b) Lobectomies
a) Primary spontaneous pneumothorax c) Pneumonectomies d) Thorocotomy
b) Tension pneumothorax 1215. The most common type of bronchogenic
c) Haemo-pneumothorax carcinoma is- (AJIMS 82, DNB 89)
d) Secondary spontaneous pneumothorax a) Epidermoid b) Anaplastic
c) Alveolar d) Bronchoalveolar
1204. The organism most frequently related to mediastina
1216. Most common symptom ofbronchial adenoma is-
fibrosis is - (DPG 1 0)
a) Chest pain b) Cough (Al96)
b) Histoplasma
c) Recurrent hemoptsis d) Weight loss
d) Stat>hvl,oco,:cus
1217.. The lung tumour responding best to radiotherapy-
a) Small cell anaplastic (CUPGEE 96)
b) Squamous cell CA
c) AdenoCA
d) All respond equally well
1218. The first step when doing a penumononectomy for
cancer of the bronchus is to- (UP 97)
a) Ligate the pulmonary vein
b) Ligate pulmonary artery
c) Divide the brounches
d) Perform lymphnode clearance
1219. In small cell carcinoma of the lung one of the
following is not seen - (KERALA 97)
a) Hypercalcemia b) Hyponatremia
c) Watery diarrhoea d) Hypokalemia
1220. True about bronchial adenoma- (AJ98)
LUNG a) 10-15%ofalllungtumour
b) Mostly malignant
1208. Bronchogenic carcinoma which produce c) Recurrent haemoptysis
paraneoplastic syndrome- (KERALA 87, AI 88) d) Peripherally located
a) Squamous cell carcinoma b) Oat cell carcinoma 1221. Sequestrated lung is suplied most commonly
c) Adeno carcinoma d) Large cell carcinoma by- (Kerala 98)
1209. Commonest symptom of carcinoma bronchus a) Bronchial arteries b) Descending aorta
is - (AP 84, KERALA 87) c) Subclavian artery d) Intercostal arteries
a) Hemoptysis b) Dyspnoea 1222. Diagnosis oflung sequestration by -(JIPMER 2K)
a) CT b) Angiography
c) Cough d) Wheezing
c) MRI d) X-ray
. e) Pain

1200)b 1201)a 1202)a 1203)b 1204)b 1205)c 1206)a 1207)b 1208)b 1209)c 1210)c 121l)d 1212)c 1213)b
1214)b 1215)a 1216)c 1217)a 1218)b 1219)c 1220)c 122l)b 1222)a
SURGERY [ 239]

1223. Which one of the following types ofbronchogenic 1235. The most important consideration in a patient with
carcinoma is most likelyto cavitate?- (UPSC 96, borderline pulmonary function requiring lung
a) Adenocarcinoma AIIMS 89) resection is- (Karnataka 03)
b) Bronchoalveolar carcinoma a) The amount of nonfunctioning lung tissue to be
c) Oat cell carcinoma removed
d) Squamous cell carcinoma b) The amount offunctioning lung tissue to be removed
1224. Pseudochylous pleural effusion is most often seen c) Experience of the surgical team
d) Elevated pulmonary artery pressure
in- (AIIMS78, PGI 81)
1236. All are been in Pancoast syndrome except-
a) T.B. b)Lymphoma
a) Brachial plexus involvement (HPU 05)
c) Ca.lung d) Filariasis b) Dyspnoea
1225. IVC fdteris used in following except- (PGI 97) c) Clubbing
a) Massive emboli b) Negligible size of emboli d) Myasthenia gravis
c) Repeatedemboli d)None 1237. ~ost common sets of ~etastases of carcinoma
1226. In pulmonary embolism, fibrinolytic therapy is bronchi- (HPU 05)
responsible for- (PGI 97) a) Liver+ Bones b) Prostate
a) Riskofhaemorrhage b) Prognosis good c) Kidney d) Breast
c) Massive emboli d) All of the above 1238. The most common histological type seen in
1227. In pancoast tumor, following is seen except-(PGI 98) branchogenic carcinoma is- (UPSC 05, MP 97)
a) Homer's syndrome b) Rib erosion a) Small cell carcinoma b) Large cell carcinoma
c) Haemoptysis d) Pain in shoulder and arm c) Squamous cell carcinoma d) Adenocarcinoma
1228. Complication of empyema are AlE- (PGI 99) 1239. Clinical manifestations ofbronchogenic carcinoma
a) Empyema necessitans b) Bronchopleural fistula include the following except- (ICS 05)
c) Osteomyelitis d) Pneumonia a) Hoarseness of voice due to involvement of left
1229. A foreign body completely obstructing the right main recurrent laryngeal nerve
bronchuscauses- (PGI99) b) Homer's syndrome
a) Decreased ventilation perfusion ratio (VRF) c) Diaphragmatic palsy due to infiltration of phrenic
b) Increased ventilation in left lung nerve
c) Perfusion doubles in right lung d) Gastroparesis due to vagal involvement
d) Increased VF ratio in right lung 1240. All of the following statements regarding bronchial
1230. Least common site oflung abscess is- (PGI 99) cysts are true except- (MAHA 05)
a) Left upper lobe b) Left lower lobe a) Seen in mediastinum b) 50-70% occur in lung
c) Right upper lobe d) Right low lobe c) Are connnonly infected d) Multilocular
1231. All are elaborated by small cell carcinoma lung, 1241. Which needle is used for pleural biopsy-(Corned 07)
except- (PGI 2000) a) VmSilvermann's b)Abram's
a) ADH b)ACTH c) Abraham's d) Osgood's
c) 5-Hf d) Noradrenaline 1242. The most common benign tumor of the lung is-
1232. ~ostdiagnosticofpulmonaryemboliis-(PGI2000) a) Hamartoma b)Alveolaradenoma (Comed08)
a) Segmental perfusion defects corresponding to c) Teratoma d) Fibroma
abnormal chest X -ray 1243. A 80 years old male presented with lung abscess in
b) Multiple segmental perfusion defect with normal left upper zone. Best treatment of modalities is -
a) Antibiotics according to organisms (UP 08)
chest X-ray and ventilation scan
c) NormalchestX-ray b) Surgical drainage
c) Tube thoracostomy
d) Tenting of diaphragm
d) Wait and watch
1233. Hoarseness secondary to bronchogenic carcinoma
1244. All are true regarding mesothelioma except?
is usually due to extension ofthe tumour into-
a) Bilaterally symmetrical (AIIMS May 11)
a) Vocal cord (UPSC 02)
b) Associated with asbestos exposure
b) Superior laryngeal nerve
c) Histopathalogy shows biphasic pattern
c) Left recurrent laryngeal nerve
d) Occurs in late middle age
d) Right vagus nerve
1245. Inhaled foreign bodies are more likely to get lodged
1234. A young man with tuberculosis presents with in the right main bronchus because- (UPSC II 11)
massive recurrent hemoptysis. For angiographic a) Left main bronchus is wider and shorter
treatment which vascular structure should be b) Left main bronchus is narrow and wide
evaluated first- (AIIMS 03) c) Right main bronchus is shorter, wider andnearly vertical
a) Pulmonary b) Bronchial artery d) Right main bronchus is shorter, narrower and lies
c) Pulmonary vein d) Superior vena cava horizontally in relation to trachea

1223)d 1224)c 1225)b 1226)a 1227)c 1228)d 1229)a 1230)a 1231)d 1232)b 1233)c 1234)b 1235)d 1236)d
1237)a 1238)d 1239)d 1240)b,c,d 1241)b 1242)a 1243)a 1244)a 1245)c
SURGERY [ 240]

HEART & PERICARDIUM (C.V.S.) 1257. Mallory weiss syndrome often occurs in- (PGI 87)
a) Patient who abuse analgesics
1246. Most common congenital defect associated with b) Children
pulmonary stenosis is- (AIIMS 85) c) Pregnant females
~ASD ~VSD d) Patients with hiatus hernia
c) Patent foramen ovale d) PDA 1258. Hiatus hernia is treated by- (TN 89)
1247. Unsucessful external cardiac massage is due to a) Surgery when medical treatment has failed
all except- (AIIMS 84) b) Nissen's fundoplication
a) Intrinsic cardiac failure b) Hypovolumia c) Medical treatment only
c) Tension pneumothorax d) Cardiac tamponade d) None of the above
1248. Tausing- Blalock shunt is used in- (AIIMS 92) 1259. Which structure corsses the esophagus at 25 em
a) TAPVD b)VSD from the incisor teeth- (PGI 88)
c) PDA d) Tetralogy of fallot a) Arch of aorta b) Bifurcation of trachea
c) Lt. bronchus d) Left subclavian artery
1249. Fasle about mitral valvotomy for mitral stenosis is-
1260. Commonest benign tumour of the eosophagus-
a) Is best performed between the age of20-50 years
a) Leiomyoma b)Papilloma (PGI 88)
b) Should be considered when the symptoms are c) Adenoma d)Hemangioma
aggravated by pregnancy (AMU 86, 91) 1261. WhichdrugisusedforAchalasiacardia- (TN90)
c) Is indicated in persistent congestive heart failure a) Nifedipine b) Propranolol
d) Gives good results when the mitral valve is immobile c) Atenolol d) Bethnecol
1250. Which nerve is easliy injured at PDAligation- 1262. All are true of achalasia cardia except-
a) Phrenic (PGI 85, JIPMER 85, a) Dysphagia (J/PMER 90)
b) Sympathetic trunk: AIIMS 85) b) Aspiration pneumonitis
c) Cardial nerves c) Mecholyl test is hyposensitive
d) Recurrent laryngeal d) X-ray fmding of dilated esophagus with a narrow
1251. Rashkind's balloon septostomy is done in- end
a) Coarctation of aorta (PGI 81, DNB 90) 1263. Heller's operation is done in- (A190)
b) Patent ductus arteriosus a) Achalasia cardia b) Pyloric stenosis
c) Transpostion of great vessels c) Hirschsprung's disease d) Crohn's disease
d) Fallot's tetralogy 1264. Gastroesophageal reflux is best diagnosed by-
1252. Most common artery used for coronary artery a) Bariumswallow b)Bariummeal (Al/MS92)
bypass graftis- (Rohtak 97) c) Endoscopy d) Ultrasound
a) Int. Mammary artery b) Intercostal artery 1265. Barrets ulcer is due to- (J/PMER 93)
c) Radial artery d) Dorsalis pedisartery a) Ischemia b) Ectopic gastric mucosa
e) Brachial artery c) Bile reflux d) Reflux esophagitis
1253. Blood loss is least in trauma to which chamber of 1266. Radiologic feature suggestive ofAchalasia cardia is-
heart- (JIPMER 99) a) Absence of gastric air bubble (AIIMS 79, PGI 78)
a) Right atrium b) Right Ventricle b) Air fluid level in mediastinum
c) LeftAtrium d)LeftVentricle c) Sigmoid esophagus
d) All the above
OESOPHAGUS 1267. Which figures-are very important to the
oesophagoscopist- (PGI 81, AIIMS 86)
1254. About achalasia cardia all are correct except-
a) 10,20 and 30 b) 15,25 and40
a) Mostly in women (AP 84, Kerala 86, 88, JIP, 98)
c) 20,30and40 d) 25,35 and45
b) Dilated esophagus narrowing to a point
e) 25, 35 and 50
c) Heller's operation treatment of choice
1268. The aim ofpreventing reflux oesophagitis by repairing
d) Not premalignant condition
hiatus hernia is achieved by -(AIIMS 79,Rohtak 87)
1255. During esophagoscopy, the area in the esophagus
a) Bringing the stomach inferior to diaphragm
mostly perforated is- (PGI 88)
b) Reconstitution of the angle ofHill
a) At aortic arch
c) Repair of defect in diaphragm
b) Mid esophagus
d) All of the above
c) At cricopharyngeus muscle
1269. Mter surgical treatment of carcinoma Esophagus,
d) At esophagogastric junction
replacement of esophagus is done with- (AIIMS 81,
1256. Commonest cause of esophageal perforation is-
a) Left colon b) Right colon AP 89)
a) Acid ingestion b) Hyperemesis (PGI 87)
c) Stomach d) Jejunum
c) Instrumentation d) Carcinoma infiltrating
e) All of the above

I246)a I247)b 1248)d 1249)d 1250)d 1251)c 1252)a 1253)d 1254)a,d 1255)c 1256)c 1257)a,c,d 1258)a,b
1259)a,c 1260)a 1261)a 1262)c 1263)a 1264)None 1265)b,d 1266)d 1267)b 1268)d 1269)e
SURGERY [ 241 ]

1270. Maximum dilatation of esophagus occurs in- 1280. Investigation of choice for reftux oesophagittis -
a) Carcinoma at gastro esophageal junction a) Endoscopy (JIPMER 98, PGI 98,
b) Achalasia cardia (Kerala 94) b) Barium swallow COMED 07)
c) Stricture at lower end c) 24 hrs acid output
d) CRUST syndrome d) Echocardiogram
1271. True about carcinoma eosphagus is- (Kerala 94) 1281. Not true aboutp;ummer vinson syndrome is-
a) Most common site is lower end a) Occures in elderly males (AIIMS 97)
b) Both adeno and squamous cell carcinoma occur b) Post cricoid Web
c) Commonest histology is adenocarcinoma c) Predispose to hypopharynx malignancy
d) More common in females d) Koilonychia
1272. Second swallowing in barium meal studies is 1282. Common site for esophageal perforation is-
foundin- (JIPMER95) a) Thoracic esophagus above arch (AIIMS 96)
a) Pharyngeal pouch b) Achalasia cardia b) Below arch
c) Scleroderma d) Reflux esophagitis c) Cervical esophageal
1273. A 55-year old male has retero-sternal discomfort d) Esophageal gastric junction
unrelated to physical exertion. Pain gets worse 12a3. Thecommonestcauseofhemldmlesisandmalena-(1N98)
after lying down there is partial relief with a) Oesophageal varies b) Chronic peptic ulcer
antacids. The most likely diagnosis is - (UPSC 96) c) Acute peptic ulcer d) Carcinoma of stomach
a) Ischaemic heart disease 1284. 3rd constriction of the oesophagus is at the level of-
b) Carcinoma oesophagus a) Left bronchus crossing the aorta (CMC 98)
c) Achalasia cardia b) Where it pierces the diaphram
d) Hiatus hernia c) Junction of oesophagus and stomach
1274. Feature of Achalasia cardia (cardiospasm) include d) Cricopharynx
all of the following except- ( UPSC 96) 1285. Regarding adenocarcinoma oesophagus which is
a) Increasing difficulty in swallowing, more for true- (A198)
liquids than for solids a) Common in upper 1/3
b) Regurgitant vomiting b) Caused by barrets oesophagus
c) Dilated end elongated oesophagus c) Common in children
d) Filling defect and "rat-tail" deformity on barium study d) Most common type of carcinoma
1275. In mallory Weiss syndrome, where is the mucosal 1286. Following are radiological evidence of Achalasia
tear located- ( CUPGEE 96) cardia except- (Karnat 98)
a) Smooth narrowing of oesophagus
a) Gastric cell anaplastic
b) Dilated tortous oesophagus
b) Squamous cell CA
c) Absence of air in the fundus
c) Gastro-oesophagal junction
d) Exaggerated peristalsis
d) Near gastric pylorus 1287. Following Ivor lewis operation for esphageal
1276. Dysphagia lusorum is due to obstruction due to- cancer oflower 1/3rd, death is due to- (AIIMS 98)
a) Vascular rings b) Psychogenic (AP 96) a) Myocarditis b)Anastomolic leak
c) Foreign body d) None c) Pulmonary atelectasis d) Flap necrosis
1277. In majority of patient with eosophagealleaks in 1288. The pharyngeal diverticulum is a protrustion of
thoracic cavity ofless than 12 hours duration, the mucosa between- (UPSC 2K)
treatment of choice is- (UPSC 97) a) The two parts of inferior constrictor muscle of
a) Primary closure, drainage and antibiotics the pharynx
b) Early oesophagogastrorstomy b) The two parts of middle constrictor muscle of
c) Exclusion and diversion of continuity the pharynx
d) Total oesophagectomy and gastric pull up c) The two parts of the superior constrictor mucle
1278. All statement of Barret's esophagus are true except- of the pharynx
a) Metaplasia with intestinal columnar cells d) Cricopharyngeal and posterior part of suprahyoid
b) Metaplasia with gastric fundus cells (AIIMS 97) membrane
c) Carcinoma in situ formation 1289. Peptic oesophagitis- (UPSC 2K)
d) Increased incdence of sqiunaous cell carcinoma a) Is effectively demonstrated by barium swallow
1279. Adenocarcinoma of oesophagus is seen b) Is always associated with hiatus hernia
in- (AIIMS 96) c) Can be readily confirmed by oesophagoscopy
a) Barret's oesophagus b) Achlasia cardia d) Is associated with the production of higher than
c) Scleroderma d) Menetriers diesease normal amounts of gastric acid

1270)b 127l)a,b 1272)a 1273)d 1274)d 1275)c 1276)a 1277)a 1278)d 1279)a 1280)c 128l)a 1282)c
1283) b 1284) b 1285) b,d 1286) d 1287) b 1288) a 1289) None
SURGERY [ 242]

1290. Best flap for eosphagus repair- (CMC Vellore) 1301. During sclerotherapy (by endoscopy), following are
a) Colon b) Stomach complications except- (PGI 98)
c) Jejunum d) Latismus dorsi a) Hepatic encephalopathy b) Perforation
1291. In eosphageal perforation all are seen except- c) Stenosis d) Fibrosis
a) Pain b) Bradycardia (UP 2K) 1302. Ca oesophagus is characterized by following except-
c) Fever d) Hypotension a) Adenocarcinoma (PGI 98)
1292. Which of the following statements regarding b) Middle i/3 affected
plummer Vinson syndrome is not true-(Kerala 2k) c) Dysphagia is characteristic
a) It is otherwise known as Patterson-Brownkelly d) Pernicious anemia often present
disease 1303. Connective tissue disorder which is associated with
b) Iron deficiency anaemia is probably the basic cause gastroesophageal reflux is- (PGI 99)
c) It occurs exclusively in men a) Sl.E b) Scleroderma
d) Achlorhydria is usually present c) Behcet's syndrome d) Dermatomyositis
e) Prone to post circoid cancer 1304. About mallory weiss sydrome true is- (PGI 99)
1293. A patient presents with dysphagia of 4 weeks a) Gastrointestinalreflux
durattion. Now he is able to swallow liquid food b) Obesity
only. Which of the following is the one investigation c) Tumour at gastroesophageal junction
to be done- (PGI 2K) d) Seen in alcoholics
a) Barium studies are the best in this case 1305. Esophageal carcinoma is not predisposed
b) Upper GI endoscopy is to be done by- (PGI99)
c) CT Scan is needed a) Achalasia b) Scleroderma
d) Esophageal manometry c) Corrosive intake d) Barret's esophagus
e) USGabdomen 1306. True about sliding esophageal hernia in all cases-
1294. Length of alimentary Canal from Oral Cavity to a) Esophagus always shor (PG/ 99)
gastroesophageal Junction is- (AJIMS 92) b) Cardia goes through hiatus
a) 15Cm b)25Cm c) Cardia and fundus goes through hiatus
c) 30Cm d)40Cm d) Peritoneal sac goes with paraesophageal hernia
1295. Motility in the upper third of the oesophagus is 1307. The commonest type of tracheo-esophageal fistula is-
decreased in except- (DNB 89) a) Proximal end blind; distal end communicating
a) Pseudobulbular palsy b) Chagas disease with trachea (PGI 99)
c) Myasthenia gravis d) Scleroderma b) Distal end; blind proximal end communicating
e) Cricopharyngeal carcinoma with trachea
1296. Cork screw oesophagus is seen in- (PGI 97) c) Both ends blind
a) Diffuse oesophageal spasm d) Both ends open
b) Achalasia cardia 1308. Investigation of choice for dysphagia forsolids -
c) Ca. oesophagus a) Barrium swallow b) Endoscopy (PGI 2K)
d) Pulsion diverticulum c) X-ray chest d) C.T. scan
1297. Mallory weiss syndrome in partial thickness 1309. Patterson Brown Kelly syndrome is characterised
rupture occurs at- (PGI 97) by all except- (PG/2K)
a) Gastric cardia a) Lower oesophageal web
b) Oesophagus mucosa b) Iron deficiency andema
c) Gastro oesophageal junction c) Common in adult female
d) Gastroduodenal junction d) Premalignant
1298. Investigation of choice in upper GI bleeding is- 1310. In achalasia cardia, true is- (PGI 2K)
a) Baswallow b)X-ray a) Pressure at distal end i with no peristalis
c) Endoscopy d) Ultrasound b) Low pr. at LES with no periotalins
1299. In dysphagia lusoria true is- (PGI 97) c) Pressure> 50 mm Hg with peristalis
a) Esophagela web 1311. Reflux esophagitis is prevented by- (PG/ 01)
b) Aberrant vessels a) Long intrabdominal esophagus
c) Oesophageal stricture b) Increased intraabdominal pressure
d) Oesophageal carcinoma c) Right crus of diaphragm
1300. Schatazki's ring is- (PGI 98) d) Increased intrathoracic pressure
a) Mucosal ring ata sq-columnar in 1312. In ca esophagus predisposing factors- (PG/ 02)
b) Muscular ring a) Tylosis b) Smoking
c) Dysphagia is the symptom c) Lye stricture d) Reflux erophagitis
d) Inflammatory structure e) Plummervilson syndrome

1290)b 1291)b 1292)c 1293)b 1294)d 1295)d,e 1296)a 1297)a 1298)c 1299)b 1300)a,c 1301)a 1302)d
1303)b 1304)d 1305)b 1306)b 1307)a 1308)b 1309)a 1310)a 1311)a,b,c 1312)All
SURGERY [ 243]

1313. True about Ca esophagus- (PGJ 03) 1323. True about Barrets esophagus- (PGJ 04)
a) MCinmiddle 1J3rd a) Prone to adenocarcinoma
b) Adenocarcinoma is common variety b) Squamous to columnar metaplasia
c) Carcinoma develops at the achalasia segment c) Squamous carcinoma
d) Smoking is a risk factor d) Lump in the throat
e) Endoscopy is the investigation of choice 1324. Which one of the following treatment modalities
1314. A 60 yrs old pt presenting with dysphagia of 6 wks is not used for management of acute blood loss
duration with solid foods now can swallow only due to ruptured esophageal varices- (UPSC 04)
liquids. Investigations done to diagnose here- a) Endoscopic sclerotherapy
a) CXR b)BaSwallow(PG/03) b) Endoscopic band ligation
c) Endoscopy d)USG
c) Octreotide
e) CTScan
d) Propranolol
1315. TrueaboutBarret'sesophagus- (PG/03)
1325. Amyl nitrate inhalation test is used to detect-
a) Premalignant
b) Predispose to sq. cell Ca a) Carcinoma esophagus (COMEDK 05)
c) Can be diagnosed by seeing under endoscope b) Achalasia cardia
d) Biopsy is necessary to diagnose c) Oesophageal diverticulum
e) Stricture may be present in high esophagus d) Transeoesophageal fistula
1316. TrneaboutupperGLbleedding (PG/03) 1326. An old man presenting to the Emergency following
a) Malaena is the only symptom about of prolonged vomiting with excessive
b) Bleeding occurs beyond the ampula ofvater haematemesis following alcohol ingestion is likely
c) Endoscopy can best diagnose it to suffer from- ( UPSC 05)
d) Peptic ulcer is the MC cause a) Mallory-Wesis syndrome
e) -edBUN b) Oesophageal varices
1317. Lower oesophageal sphinter is relaxed by -(PGJ 03) c) Gastric cancer
a) Alcohol b) Coffeine I Tea d) Bleeding disorder
c) Diazepam d) Antacid 1327. Complications of"retlux esophagitis" -(MAHE 05)
1318. True about Barrett's oesophagus- (PGI 03) a) Stricture b) Schatzki 's ring
a) Sequence of prolonged G.E.R. c) Barrett's esophagus d) All of above
b) It is premalignant 1328. Predisposing factors of Ca esophagus-(PGJJune 05)
c) Lower oesophageal mucosa is replaced by a) Tylosis b) Achalasia
intestinal type of epithelium c) Barrett's esophagus d) Hiatus hernia
d) Varocose veins are seen e) Corrosive ingestion
e) Predisposes to squamous cell Ca of esophagus
1329. Malignant tracheo-esophageal fistula best treated
1319. Intermittent dysphagia is caused by- (PGI 04)
with- (PGJJune 05)
a) Stricture
a) Radiotherapy b) Chemotherapy
b) Reflux esophagitis
c) Stenting d) Tube
c) Achalasia cardia
d) Pharyngeal diverticulm e) Surgical correction
e) Diffuse esophageal spasm 1330. Boerhove's syndrome, true about- (PGJ June 05)
1320. Dysphagia lusoria is due to- (PGI 2000) a) Iatrogenic
a) Oesophageal webs b) Silent manifestation
b) Abnormalities of Arch of Aorta c) Presented with acute chest pain
c) Aortic aneurysm d) Treatment is surgical
d) Enlarged left atrium 1331. Which of the following surgical approach was first
1321. LES sphineter is relaxed by- (PGJ 02) described by Orringer for the management of
a) Nitrates b) HL blockers carcinoma esophagus- (J & K 05)
c) Morphine d) Atropine a) Tranhiatal
e) Calcium channel blockers b) Thoracoscopic
1322. True about achalasia cardia is- (PGI 02) c) Left thoracoabdominal
a) Dysphagia is a presenting symptom d) Right thoracoabdominal
b) The cause is the absence of Auerbach's plexus 1332. The most common complication seen in hiatus
c) Esophagectomy is the treatment hernia is- (AI 05)
d) Motility-improving agents are used in treatment a) Oesophagitis b) Aspiration pneumonitis
e) Barium swallow shows irregular filling-defect in c) Volvulus d) Esophageal stricture
lower esophagus

1313) a,b,d,e 1314)All 1315) a,c,d,e 1316) c,d,e 1317) a,b,c 1318) a,b,c 1319)d,e 1320) b 1321) a,d,e 1322) a,b,d
1323)a,b 1324)d 1325)b 1326)a 1327)d 1328)a,b,c,e 1329)c 1330)c,d 133l)a 1332)a
SURGERY [ 244]

1333. Features of Barrett's esophagus are -(PGI June 06) 1343. Mallory-Weiss tear occurs at- (UPSC-11 08)
a) Metaplasia a) Gastro-oesophageal juction
b) Always gastric type of epithelium b) Duodeno-jejunal flexure
c) Adenocarcinoma more common c) Ileo-caecal junction
d) Present as patchy or ring involvement d) Colo-rectaljunction
1334. Regarding oesophagoscope true are -(PGI June 06) 1344. Commonest site of oesophagus carcinoma is-
a) Incisor acts as fulcrum a) Upper2/3 b) Middle 1/3 (UP08)
c) Lower 1/3 d) Crico-esophageal junction
b) Compresses posterior tongue
1345. Early stage ofcarcinoma esophagus is diagnosed by-
c) Lifts epiglottis
a) Bariummeal b) Trans oesophageal USG
d) Tip is in pyriform fossa
c) MRI d) Fluoroscopy (UP 08)
e) Inserted from right side 1346. Increasing difficulty in swallowing both for solids
1335. Which is the most reliable diagnostic method for & liquids in a women with bird's beak appearance in
staging the oesphageal cancer?- ( UPSC 06) X-ray seen in- (PGI June 08)
a) MRI b) Endoscopic ultrasound a) Achalasia cardia b) Carcinoma
c) CT scan d) Thoracoscopy c) Reflux esophatitis d) Barret's esophagus
1336. The most common type ofTracheo-Oesophageal e) Esophagitis
Fistula is - (MAHE 07) 1347. 30 years ofwomen comes with dysphagia for both
a) Esophageal atresia without tracheoesophageal solid & liquids & barium swallow shows Parrot beak
fistula appearance. On esophageal manometry, increased
b) Esophageal atresia with proximal LES pressure. Management includes -(PGIJune 08)
tracheoesophageal fistula a) Nitrates b) Cachannel blockers
c) Esophageal atresia with distal tracheoesophagea c) Botulinum toxin d) Myotomy
fistula 1348. True about hiatus hernia- (PGI June 08)
a) Surgery indicated in all symptomatic cases of para-
d) Esophageal atresia with proximal and distal fistula
esophageal
1337. Boerhaave'ssyndromeisdueto- (UP 07)
b) Para-esophageal type is more complicated
a) Drug induced esophagus perforation
c) Para-esophageal type is common type
b) Corrosive injury d) Common in infants
c) Spontaneous perforation 1349. The defect in achalasia cardia is present in-
d) Gastro-esophageal reflux disease a) Myenterich plexus of auerbach (Manipal 08)
1338. Esophageal carcinoma is predisposed by- b) Meissner's plexus
a) Achalasia b) Scleroderma c) Kesselbach's plexus
c) Corrosive intake d) Barret's esophagus d) Mesentric plexus
1339. Causes of esophageal carcinoma- (PGI June 07) 1350. The most common type of tracheoesophageal fistula
a) Tylosis b) Reflux esophagitis is - (Manipal 08)
c) Lye ingestion a) Esophageal atresia without tracheoesophageal fistula
1340. All are features of Plummer-Vinson syndrome b) Esophageal atresia with proximal
except- (Comed 07) tracheoesophageal fistula
a) Oesophageal web b) Iron deficiency c) Esophageal atresia with distal tracheoesophageal
c) Achalasia cardia d) Dysphagia fistula
d) Esophageal atresia with proximal and distal
1341. Transhiatal esophagectomy was planned for
tracheoesophageal fistula
adenocarcinoma of lower end of esophagus. The
1351. Best test to diagnose gastroesophageal reflux
approach would be in the following order-
disease and quantify acid output is- (AJIMS Nov 08)
a) Abdomen-Neck (AIIMS Nov 07) a) Esohagogram b) Endoscopy
b) Abdomen-Thorax-Neck c) Manometry d) 24 hr pH monitoring
c) Neck-Thorax-Abdomen 1352. Non progressive contraction of esophagus are-
d) Abdomen-Thorax a) Primary b) Secondary (AI 09)
1342. When a 'spontaneous perforation' ofthe oesophagus c) Tertiary d) Quaternary
occurs as a result of severe barotrauma while a 1353. All of the following statements about Zenker's
person vomits against a closed glotts, what is the diverticulum aretrneExcept- (AI 09)
condition known as- (UPSC-ll 08) a) Acquired diverticulum
a) Maltory-Weiss syndrome b) Lateral X-rays on Barium swallow are often diagnostic
b) Plummer Vinsion syndrome c) False diverticulum
c) Kartergener syndrome d) Out pouching of the anterior pharyngeal wall, just
d) Boerhaave syndrome above the cricopharyngeus muscle

1333) a,c,d 1334) b,d,e 1335) b 1336) c 1337) c 1338) a,c,d 1339) a,b,c 1340) c 1341) a 1342) d 1343) a 1344) a
1345)a 1346)a 1347)a,b,c,d 1348)a,b 1349)a 1350)c 135l)d 1352)c 1353)d
SURGERY [ 245]

1354. Most common cause of esophagitis is-(AIIMS May 09) 1364. Barrett's esophagus shows- (Al/MS May 1 0)
a) Alcohol b) Smoking a) Intestinal dysplasia
c) Spicy and hot food d) Esophageal reflux b) Intestinal metaplasia
1355. Not a predisposing factor for Ca esophagus- c) Squamous cell metaplasia
a) Diverticula (Al/MS May 09) d) Columnar cell metaplasia
b) Human papilloma virus 1365. Dysphagia which is more for liquids than for solids
c) Mediastinal fibrosis in a 25-year-old female strongly suggests the
d) Caustic ingestion possibility of- (UPSC-I 10)
1356. Which of the following statement(s) is/are true about a) Pseudobulbar palsy
trauma injury- (PGI June 09) b) Achalasia of oesophagus
a) Seat belt can cause pancreato- duodenal injury c) Carcinoma of oesophagus
b) Late death is caused by sepsis d) Corrosive induced oesophageal stricture
c) Damage control surgery (DCS) is used to control 1366. A 45-year-old man presented with regurgitated food
major bleeding & to prevent contamination of particles eaten several days earlier. He has foul
peritoneal cavity. smelling breath and occasional dysphagia for solid
d) In DCS the abdomen is closed in layer to prevent fooD. What is the probable diagnosis? (UPSC-I 10)
evisceration a) Gastric outlet obstruction
e) In DCS, laprotomy is decided based on patient b) Zenker's diverticulum
prognosis c) Scleroderma
1357. Trueaboutoesophagealinjury- (PG/June 09) d) Diabetic gastroparesis
a) Barium swallow is diagnostic 1367. Dysphagia lusoria is a condition which results from-
b) Treatment is primary repair a) Oesophageal atresia (UP SC II 10)
c) MC after penetrating injury b) Aberrant right subclavian artery
d) Mortality increased if repair delayed beyond 24 hr c) Oesophageal web
e) Can lead to medasitinal collection d) Corrosive stricture
1358. Commonest benign tumour of the esophagus- 1368. True about Schatzki's ring- (PGI Nov. 1 0)
a) Leiomyoma b) Papilloma (DELHI PG Feb. a) Has skeletal muscle
c) Adenoma d)Hemangioma 09) b) Located at lower esophagus
1359. A patient present with dysphagia of 4 weeks duration. c) Causes dysphagia
Now he is able to swallow liquid food only. Which of d) Contain all layers of oesophagus
the following is the one investigation to be 1369. Non progressive contraction of Esophagus are-
done? (DELHIPGFeb. 09) a) Primary b) Secondary (Al/MS May 11)
a) Barium studies b) Upper GI endoscopy c) Tertiary d) Quaternary
c) CT scan d) Esophageal manometry 1370. A 30-year-old woman complains of suprasternal
1360. The site of mucosal tear in Mallory-Weiss syndrome dysphagia only for, solids. She also has a long-
is - (COMED 09) standing untreated menorrhagia leading to iron
a) Stomach b) Small intestine deficiency anaemia. What is the most probable cause
c) Colon d) Oesophagus forherdysphagia? (UPSCI 11)
1361. Heller's operation is done for - (COMED 09) a) Pharyngeal pouch b) Carcinoma oesophagus
a) Achalasia cardia b) Pyloric stenosis c) lusoria d) Patterson-Kelly SVIJtdro1me
c) Hirschsprung's disease d) Crohn's disease
1362. True about mallory weiss tear- (PGI Nov 09)
a) Hammanmediastinum
b) Esophagus perforates at all layers
c) Auscultation- Crunching sound
d) Symptoms improves after conservative Mx
e) Longitudinal tear at junction ofesophagus & stomach
1363. True about Boerhoeve's syndrome- (PGI Nov 09)
a) MC at lower l/3rd
b) Hematemesis is early symptoms
c) Acute chest pain
d) Surgically treated
e) Operation is done after 24 hr

1354)d 1355)c 1356)a,b,c,e 1357)All 1358)a 1359)b 1360)a 1361)a 1362)d,e 1363)a,c,d 1364)b 1365)b
1366) b 1367) b 1368) b,c 1369) c 1370) d 1371) b 1372) b 1373) c
SURGERY [ 246]

STOMACH & DUODENUM 1385. In pyloric stenosis the following changes occur-
a) Hypokalemic hyponatremic alkalosis (AIIMS 84)
1374. The most usuful method to diagnose cuse of upper b) Hyperkalaemia
GI haemorrhage is- (PGI 80, UPSC 84) c) Acidosis with hyponartremic alkalosis
a) Barium study b) Celiac angiography d) Hyperchloremic acidosis
c) Gastric biopsy d) Endoscopy 1386. Commonest stomach tumour which bleeds -(PGI88)
1375. All of the following are common cuases of a) Adenocarcinoma b) Squamous carcinoma
haematemisis except- (AIIMS 82, DNB 88) c) Leiomysarcoma d) Fibrosarcoma
1387. Regarding acute dilation of stomach which is
a) Chronic peptic ulcer b) Pernicious anemia
incorrect- (PGI 88)
c) Esophageal varices d)Carcinomaofstomach
a) Occurs with fracture femur
1376. UncommoncauseofupperGibleed- (PGI96)
b) Occurs with plaster
a) Varices b) Erosive gastritis c) Resolves spontaneosly without treatment
c) Peptic ulcer d) Carcinoma stomach d) Hypophophatemia is to be avoided
1377. Commonest site of peptic ulcer 1388. Commonest operation done for peptic ulcer with
perforation- (UPSC 86, KERALA 86, AI 86) gastric outlet obstruction is- (KERALA 89)
a) Anterior aspect of the ftrst part of duodenum a) Truncal vagotomy with pyloroplasty
b) Posterior aspect of the 1st part of duodenum b) Highly selective vagotomy with pyloroplasty
c) Greater cuvature ofthe stomach c) Truncal vagotomy with gastrojejunostomy
d) Lesser curvature of the stomach d) Gastrojejunostomy
e) Anterior apect of 2nd of duodenum 1389. Which of the following regarding hypertrophic
1378. Treatment of Zollinger Ellison syndrome- (AI 88) pyloric stenosis is correct- (PGI 89)
a) Total gastrectomy with removal of tumour a) More in female infants
b) Partial gastrectomy b) 4-6 months after birth
c) Excision of tumour alone c) Billious vomiting
d) Hypochromic Metabolic alkalosis
d) H2 receptor antogonist
1390. The treatment ofHodgkin's disease of stomach is-
1379. Curlings ulcer is seen in- (AI 88)
a) Gastric resection (Karnataka 89)
a) Burns patients
b) Gastric resection and chemotherapy
b) Patients with head injuries c) Puurely medical
c) Zollinger Ellison syndrome d) None of the above
d) Analgesic drug abuse 1391. Commonest cause of death in peptic ulcer patients
1380. Most common site for carcinoma stomach is- is- (AI 90)
a) Prepyloric (KERALA 87, AP 85, UPSC 88, a) Perforation b) Hemorrhage
b) Body of stomach KERALA 94) c) Pyloric stenosis d) Maliganancy
c) Fundus 1392. All of the following predispose to gastric
d) Lesser curvature carcinoma except- (AI 90)
1381. Operability in carcinoma stomach is indicated by a) Achlorhydria b) '0' blood group
all except- (PGI 86) c) Pernicious anaemia d) Post gastrectomy
a) Involvement of omental nodes 1393. Linitis plasticais commonly seen in- (AI91)
b) Involvement oflymphnodes at the celiac axis a) Ca. stomach b) Sarcoidosis
c) Lymphnode at porta hepatis c) Lymphoma d) Leiomyosarcoma
d) Solitary metastatic nodule in the liver 1394. G-cells are present mostly in- (TN 91)
a) Fundus p) Cardia
e) Krukenberg tumour
c) Pyloric antrum d) Body
1382. Percentage of patient with perforated peptic ulcer
1395. Precancerous condition ofthe stomach is -(Kerala
who show free gas under the diaphragm- a) Lipoma b)Linitisplastica 91)
a) 1000/o b) 75% c) Atrophic gastritis d) Hyperacidity
c) 50% d)90% 1396. The commonest earliest complication of TV and
1383. Duodenal blowout following a Bilroth gastrectomy GJis- (AIIMS91)
most commonly occurs on the.........day- (AIIMS 87) a) Stomal obstruction b) Panalytic ileus
a) Jrd b) 2ru1 c) Gastricleak d)AnastomoticHemorrhage
c) 6th d) 19th 1397. Malignant transformation is commonly seen in-
1384. Which of the following does not occur in chronic a) Stomal ulcer (AIIMS 91)
pyloric obstruction- (BHU 88) b) Gastric ulcer
a) Acid urine b)Alkalineurine c) Chronic Duodenal ulcer
c) Hypokalaemia d) Hypochloremia d) Post bulbar ulcer

1374)d 1375)b 1376)d 1377)a 1378)c 1379)a 1380)d 138l)e 1382)b 1383)None 1384)b 1385)a 1386)c
1387)c 1388)c 1389)d 1390)d 139l)b 1392)b 1393)a 1394)c 1395)c 1396)d 1397)None
SURGERY [ 247]

1398. Hour glass stomach is seen in- (AIIMS 92) 1411. The operation of choice in congentital pyloric
a) Gastric carcinoma b) Gastric ulcer stenosis is- (AIIMS 80, DELH/83,
c) Gastric Lymphoma d)Corrossive Strictures a) Pyloroplasty AP 89, Jipmer81, PG/97)
1399. Most common benign tnmour of the stomach is- b) Pyloromyotomy
a) Adenoma b)Lipoma (PG/93) c) Gastro-jejunostomy
c) Hamartoma d) Leiomyoma d) Gastro duodenostomy
1400. Gastric Acid secretion is inhibited by- (AI 93) 1412. Prensenting symptom of carcinoma stomach is-
a) Gastrin b)AlkalinepH a) Bleeding b)Obstruction(.41IMS81,AP90)
c) Vagal stimulation d) Somatostatin c) Perforation d) Weight loss
1401. All the following predispose to Carcinoma 1413. Investigation of choice in peptic ulcer perforation
stomach- is - (KERALA 94)
a) Menetriers disease b) Chronic gastritis a) USG b)X-Rayabdomen
c) Gastric polyp d) Gastric erosion c) Paracentasis d) CT scan
1402. Erosive gastritis commonly occurs at -(JIPMER 93) 1414. About 6-8 hours after peptic perforation the
a) Body b) Fundus disappearance of abdominal wall rigidity is due to -
c) Lesser curvature d) Antrum a) Cessation of acid secretion in the stomach
1403. Endoscopy is useful in diagnosis of peptic ulcer in b) Revival from initial shock (UPSC 9 5)
following sutnations except -(JIPMER 79, PG/80) c) Dilution of acid in the peritonial cavity
a) Post bulbar ulcer b) Stomal ulcers d) Fatigue of reflex arc.
c) Giant duodenal ulcer d) Duodenal erosion 1415. Best prognosis in carcinoma stomach is- (AI 95,
1404. Deverticulum of the stomach- (PGI 79, UPSC 86) a) Superficial spreading AIIMS 89, 92)
a) Pain is the main symptom b) Ulcerative
b) Usually at cardiac end c) Lintis Plastica
c) Usually on posterior surface d) Polypoidal fungating
d) Inversion is the satisfactory treatment 1416. What complication commonly occurs in anterior
e) All of the above
duodenalulcer- (TN95)
1405. Anemia is greater in which of the following gastric
a) Bleeding b) Penetration
resection- (PGI 81, AMU 85)
c) Perforation d) Stricture formation
a) Billorthll
1417. A person who had undergone gastrojejunotomy
b) Billorthl
suddenly develops severe diarrhoea. Which should
c) Both of the above are equal
d) Neither of the above be suspected- (1N95)
1406. The most frequent symptom of gastric
a) Gastric carcinoma b) Tb abdomen
diverticulum is- (AIIMS 81, PG/82)
c) Gastrojejunocolic fistula d) Gastric amoebiasis
a) Epigastric pain b) Haematemesis 1418. A 35- year old male who had chronic duodenal ulcer
c) Vomiting d) Pain relieved by food for the last six years presents with worsening of
1407. All of the following may be featnres of a silent symtomps loss of periodicity of symptoms pain on
carcinoma of the body of the stomach except- risingin the morning sense of epigastric bloating
a) Obstructive jaundice (J/PMER 80, AMU 89) and post- prandial vomiting. The most likely cause
b) Ascites oftheworseningofhis symptomsisthedevelopmentof-
c) Dysphagia a) Posterior penetration (UPSC 96)
d) Krukenberg's tumours b) Gastric outlet obstruction
1408. In congenital pyloric stenosis the defect usually c) Carcinoma
lies in- antrum- (A/IMS 78, PGI 83) d) Pancreatitis
a) Nerve fibres 1419. All of the following are indications for surgery in
b) Circular muscle fibres a case of duodenal ulcer except- (UPSC 96)
c) Longitudinal muscle fibres a) Acute perforation of ulcer
d) Mucosa b) Pyloric stenosis
1409. The following are the complications of trichobezoars c) Massive haemorrhage
except- (Al/MS 89, AMU 89) d) Typical periodicity
a) Haematemesis b) Perforation and peritonis 1420. The operation wherein the stnmp of the stomach is
c) Obstruction d) Malignancy durectly anastomosed to the stnmp ofthe duodenum
1410. If the mucosa was accidentally opended at is called- (KARNAT96)
operation (Ramstedt) it is wise not to feed the a) Polya gastrectomy
child orally for- (AIIMS 70, AMU 90) b) Hoffmeister gastrectomy
a) 12 Hours b) 24 Hours c) Billrothl gastrectomy
c) 48 Hours d) 1 week d) Billrothe II gastrectomy

1398)b 1399)a 1400)d 1401)d 1402)a,b 1403)c 1404)b,c 1405)a 1406)a 1407)c 1408)b 1409)d 1410)b
141l)b 1412)d 1413)b 1414)c 1415)a 1416)c 1417)c 1418)b 1419)d 1420)c
SURGERY [ 248]

1421. Which ofthe following is not true of Curling's ulcer- 1432. True about DIPS is all except- (AIIMS 99)
a) Seen in burned patients (KARNAT 96) a) Males commonly affected
b) Are solitary penetrating ulcer b) Commonly presents at 3-8 weeks
c) Are shallow multiple erosions c) X-Ray shows gastric dilation and gasless small
d) Has also been described in children after head intestines
injury or craniotomy d) USG is of no use in diagnosis
1422. Trosier's sign is- (AP 96) 1433. Gastric carcinoma involving the anrtum with lymph
a) Rt. supraclavicular lymph node enlargment node involvements. The pancreas, liver, peritoneal
b) Lt. supraclavicular Lymphnode enlargement cavity are normal. Most appropriate surgery is-
c) Carpopedal spasm a) Total radical gastrectomy
d) Migrating Thrombophlebitis b) Palliative gastrectomy
1423. Maximal reduction in gastric acidity is achieved c) Gastrojejunostomy
by- (UPSC 97, d) None of the above
a) Truncal vagotomy and pyloroplasty PGI 96} 1434. Electrical pacemaker of stomach is situated in-
b) Truncal vagotomy and antrectomy a) Fundus (Karnataka 01)
c) Partial gastrectomy b) Body
d) Highly selective vagotomy c) IncisuraAngularis
1424. Investigation of choice for an early gastric carcinoma- d) Gastro oesophageal junction
a) Endoscopic ultrasound (AIIMS 97) 1435. H. pylori causes- (TN 2001)
b) Stained endoscopic biopsy a) Type A Gastritis b) Type B gastritis
c) Barium meal c) Autommine d) Allergic Gastritis
d) CTscan 1436. Highest pickup for gastro uejunocolic fistula
1425. Early gastric carcinoma uncludes all except- is by- (J & K 2001)
a) Invasion of Mucosa a) Baswallow b)Bameal
b) Invasion of Submucosa c) Ba enema d) Ba follow through
1437. Post -Vagotomy diarrhoea can be effectively
c) Invasion of muscularis propria
managed by- (UPSC 2002)
d) Dysplasia
a) Steroids b) Thyroxin
1426. Function of thick gastric mucusa is- (AT 97)
c) Somatostatin analogue d) Parathormone
a) Protects epithelium b) Neutralises HCL
1438. Trichobezoaris- (HPU 2001)
c) Traps foreign particles d) None of the above a) Hair ball in stomach in psychiatric patient
1427. Prognosis in a case of duodenal perforation is b) Tumours of stomach
determined by all except- (PGI 96) c) TB ofbowel
a) Age of the petient b) Duration of history d) Collection of worms in stomach
c) Basal pneumonia d) Peritonitis 1439. Histologic examination of the lesion in stomach
1428. Gastrojejunostomy is an example of -(JIPMER 98) reveal fat -laiden cells, likely cause is -(AIJMS 2001)
a) Clean contaminated wound a) Lymphoma
b) Clean uncontaminated wound b) Post gastrectomy
c) Unclean uncontaminated wound c) Signet- cell carcinoma stomach
d) Unclean contaminated wound d) Atrophic gastritis
1429. 'DUODENAL BLOWOUT' is- (AP 97) 1440. Minimum amount ofGI bleed to cause melena is
a) Perforation of duodenal ulcer a) lOml b)40ml (PGI 78, AIIMS 79)
b) Latrogenic c) 60ml d) 115ml
c) Complication of partial gastrectomy 1441. The most sensitive test to detect GI bleeding is -
d) Due to trauma a) Selective angiography (PGI 79, AIIMS 81)
1430. Dumping syndrome occurs due to- (AI 99) b) Radio labelled, erythrocyte scanning
a) Small stomach c) 1131 Fibrinogen studies
b) Hyperosmolar load in intestine d) Stool for occult blood
c) Vagolytic action 1442. Most Severe degree of alkalosis occurs in
d) Excesive intake of food obstruction of- (MAHE 98)
a) Cardiac end b) Pylorus
1431. A patient with ANTRAL Ca repeatedly vomits.
c) Lleocaecal region d) Colon
Not seen is - (AIIMS 98)
1443. In gastrectomy following occurs except- (PGI 97)
a) Acidosis b) Hypokalemia
a) Calciumdeficiency b) Steatorrhoea
c) Hyponatremia d) Hypochloremia c) Fe. deficiency d) Fluid loss

142l)b,d 1422)b 1423)b 1424)b 1425)c 1426)a 1427)c 1428)a 1429)c 1430)b 1431)a 1432)d 1433)d
1434)a 1435)b 1436)c 1437)c 1438)a 1439)c 1440)c 144l)b 1442)b 1443)d
SURGERY[249]

1444. Antenatal double bubble appearance on ultrasound 1455. Congeital pyloric stenosis causes- (PGI 02)
is due to- (PGI 97) a) Billous vomiting
a) Diaphragmatic hernia b) Duodenal atresia b) Non-billious vomiting
c) Volvulus d) Intussusception c) Projectile vomiting
1445. Precancerous condition of Ca stomach is -(PGI 97) d) Non projectile vomiting
a) Peptic ulcer b) Chronic gastric atrophy e) Forceful
c) Achalasia cardia . d) Curling's ulcer 1456. Trueofearlygastriccarcinoma- (PGI02)
a) Invasion of mucosa and sub-mucosa with
1446. Deficiency of the abdominal muscle is associated
neighbouring lymphanode
with- (PGI99)
b) Invasion of mucosa and submucosa irrespective
a) Eagle-Barrette syndrome to L.N. spread
b) Christopher syndrome c) Radical Gastrectomy perferred
c) Megacystitis d) Endoscopic removal oflesions
d) Megaureter e) Conservative gastrectomy
1447. True about stomach carcinoma- (PGI 2000) 1457. Predisposing factors for gastric ca are- (PGI 02)
a) Weightloss is commonest feature a) Atrophic gastritis b) Hyperplastic polyp
b) Secondaries are most commonly seen in c) Adenomatous polyp d)Achlorhydria
peritoneum & omentum e) Animal fat consumption
c) Lymphatic & hematogenous spread are rare 1458. Hypertrophic pyloric stenosis, true about -(PGI 03)
d) Barrium meal is diagnostic a) Common in females
e) Radioresistant b) Present after 3-5 wks of birth
1448. Artery to bleed in duodenal ulcer h 'ge -(PGI 2000, c) USG can't diagnose
a) Spleenic a PGI 88, 99, AI 89, UP 96) d) Alkalosis
b) Gastroduodenal a e) Surgery is the TOC
c) Left gastric a 1459. TrueaboutCastomach- (PGI03)
d) Sup. mesenteric a a) Weight loss is the MC presentation
1449. Risk factor for carcinoma stomach are all b) Mesenteric & peritoneal spread is early
except- (PGI 2000) c) Prognosis depends upon lymph node involvement
a) Blood group A b) Post gastrectomy d) Spreads through haematogenous & lymphatic
c) Old peptic ulcer d) Atrophic gastritis routes
1450. Treatment of perforated peptic ulcer includes - 1460. Thickened gastric folds are found in- (PGI 03)
a) i.v. fluids (PGIOJ) a) Lymphoma b) Menetrier's disease
b) Drainage ofparacolic gutter c) Carcinoma d) Eosinophilic gastritis
c) Immediate surgery e) Giardiasis
d) Antacids 1461. What is correct about duodenal ulcer- (Kerala 03)
e) i.v. pantoprazole a) 25% will occur if H. pylori is not eradicated -
1451. The investigation for duodenal ulcer is- (PGI OJ) b) Magnesium containing drugs may cause
constipation
a) Hypotonic duodenography
c) Bismuth is not used for long terms
b) Barium meal
d) None
c) Barium swallow
1462. H. pylori infection causes carcinoma by which
d) Upper GI endoscopy
mechanism- (TN03)
e) Abdominal X-ray erect view
a) Production of nitrosamines
1452. Predisposing factors for stomach carcinoma-
b) Gastric metaplasia
a) Gastriculcer (PGIOJ)
c) Increasing acid secretion
b) Pernicious anemia & achlorhydria d) Causing mutation
c) Hiatus hernia 1463. Hpylori- (PGI04)
d) Atrophic gastritis a) G- ve nonflagellate spiral organism
1453. Which of the following factors contribute to the b) Obligate parasite on gastric mucosa
development of duodenal ulcers- (PGI OJ) c) Does not infect duodenal mucosa
a) Lysolecithin b) Gastric acid d) Antibiotics not effective
c) Alcohol abuse d) Prostaglandins e) Causes hypergastinemia induced peptic ulcer
e) Smoking 1464. Duodenal adenocarcinoma- (PGI 04)
1454. Duodenal stricture is caused by- (PGI 02) a) Most common small bowel carcinoma
a) Amoebiasis b) T.B. b) Periampullary carcinoma
c) Ca pancreas d) Crohn's disease c) Jaundice & anemia- most common symptom
e) Giardiasis d) Local resection - curative

1444)b 1445)b 1446)a 1447)e 1448)b 1449)a,c 1450)a,c,e 1451)a,b,d 1452)b,d 1453)b,c,e 1454)b,c,d
1455)b,c,d,e 1456)b,c,d 1457)a,c,d 1458)b,d,e 1459)a,c,d 1460)a,b,c,d 146l)d 1462)b 1463)b 1464)a,c
SURGERY [ 250]

1465. Gastric malignancy is predisposed with-(Kerala 04) 1475. Features of pyloric stenosis- (PGI June 06)
a) Blood group 0 b) Intestinal metaplasia a) Hypokalemic alkalosis
c) Gastric hyperplasia d) Duodenal ulcer b) Peristalsis right to left
1466. Ramstedts operation is done for- (TN 04, AIIMS 87) c) Commonly caused by carcinoma stomach
a) Congenital hypertrophic pyloric stenosis d) Retention vomiting present
b) Breast carcinoma e) Commonly females involved
c) Thyroglossal cyst 1476. Characteristic of H. Pylori is/are- (PGI June 06)
a) Doesn't affect normal duodenal mucosa
d) Carcinoma stomach
b) Important cause for gastric ulcer
1467. In which one of the following conditions is gas under
c) Protozoa
diaphragm not seen- (UPSC 05) d) Antibiotics not useful
a) Perforated duodenal ulcer 1477. Eradication ofhelicobacter pylori has been proved
b) Typhoid perforation to beneficial in which of the following disorders of
c) After laparotomy the stomach- (COMED 06)
d) Spontaneous rupture of oesophagus a) Low grade malt lymphoma
1468. The structures removed, while carrying out radial b) Erosive gastritis
gastrectomy for a 2 x 2 em antral adenocarcinoma, c) Carcinoma stomach
wonld include the following except- (UPSC 05) d) Gastroesophageal disease
a) Distal 2/3 of stomach with centimeter cuff of 1478. In the Forest classification for bleeding peptic nlcler
duodenum with a visible vessel or pigmented protuberance is
b) Lesser and greater omentum classified as - ( COMED 06)
c) Lymph nodes along left and right gastric common a) F1 b)Flla
hepatic and splenic arteries c) Fll b d) Fll c
d) Spleen 1479. Stress nlcers is caused by all ofthe foUowing except?
1469. The most commonly practiced operative procedure a) Burns (APPG 06)
for a perforated duodenal ulcer is- (Karnat 05, b) Catisol therapy
a) Vagotomy and pyloroplasty SGPGI 05) c) Penicillin therapy
b) Vagotomy and antrectomy d) Pulmonary insufficiency
c) Vagotomy and perforation closure 1480. True about Trichobazoers except- (Manipal 06)
a) It is caused by Trichuris
d) Graham's omentum patch repair
b) It is a psychiatric manifestation
1470. Lesser curvature anterior seromyotomy is indicated
c) Balls of hairs in the stomach
in- (MAHE05)
d) Pulling the hair & sucking of hair is usually seen
a) Gastric ulcer b) Gastric CA
1481. With reference to duodenal ulcers, consider the
c) Duodenal blowout d) Duodenal uncer
following statements- (UPSC 07)
1471. Eradication of H.pylori has been proved to be
I. They occur most often in the second part of
beneficial in the following condition except-
duodenum
a) Duodenal ulcer (ICS 05)
2. Infection with H. pylori and NSAID-induced injury
b) Gastric ulcer account for majority of duodenal ulcer
c) LowgradeMALTlymphoma 3. Malignant duodenal ulcers are extremely rare
d) Hypertrophic gastritis 4. Exradication of H. pylori has greatly reduced the
1472. TrueaboutgastriccancerareAIE- (PGIJune05) recurrence rates in doudenal ulcers
a) Incidence increasing worldwide Which of the statements given above are correct?
b) Incidence of upper gastric cancer increasing a) 1, 2 and 3 only b) 2, 3 and 4 only
c) Surgically correctable c) 1 and 4 only d) I, 2, 3 and 4
d) Prognosis depends on the depth of the lesion than 1482. Which of the foUowing statements related to gastric
the size of the lesion injury is not true ? (AI 07)
1473. All of the following are indications for surgery in a) Mostly related to penetrating trauma
gastric lymphoma except- (AIIMS Nov 05) b) Treatment is simple debridement and suturing
a) Bleeding c) Blood in stomach is always related to gastric injury
b) Perforation d) Heals well and fast
c) Residual disease following chemotherapy 1483. Metabolic abnormality seen in congenital
d) Intractable pain hypertrophic pyloric stenosis is- (Aiims May 07)
14 74. The most common cause of gastric outlet obstruction a) Hypochloremic hypokalemic metabolic alkalosis
in India as- (AI 06) b) Hyperchlonemic hypokalemic metabolic alkalosis
a) Tuberculosis b) Cancer of stomach c) Hypochloremic hypokalemic metabolic acidosis
c) Duodenal lymphoma d) Peptic ulcer disease d) Hyperchloremic hypokalemic metabolic acidosis

1465)b 1466)a 1467)d 1468)c 1469)d 1470)d 1471)d 1472)a 1473)d 1474)b 1475)a 1476)a,b 1477)a 1478)b
1479)c 1480)a 148I)b 1482)c 1483)a
SURGERY [ 251 ]

1484. A patient comes with hematemesis and malena. On 1493. Veins involve in stomach varices are -(PG1 Dec 08)
the upper GI endoscopy there was no significant a) Coronary vein
finding. 2-days later the patient rebleeds. Next line b) Short gastric vein
ofinvestigation is- (Aiims May 07) c) Right Gastroepiploic vein
a) Emergency angiography d) Left gastroepiploic vein
b) Repeat upper GI endoscopy e) Left gastric vein
c) Enteroscopy 1494. Most common site of Curling's ulcer-
d) Laprotomy a) lleum b) Stomach (A11MS Nov 08)
1485. In a highly selective vagotomy, the vagal supply c) Duodenum d) Esophagus
is severed to - (Corned 07) 1495. Bariatric surgical procedures include-
a) Proximal two-thirds of stomach
a) Gastric Banding (A11MS Nov 08)
b) Antrum
b) Gastric Bypass
c) Pylorus
c) Biliopancreatic diversion
d) Whole of stomach
1486. Which of the following anaemias is a risk factor for d) Ileal Transposition
the development of gastric carcinoma- (Corned 07) 1496. Double Bubble sign is seen with- (AIIMS May 09)
a) Pernicious anaemia a) Pyloric stenosis b) Duodenal atresia
b) Megaloblastic anaemia c) Ileal atresia d) Esophageal atresia
c) Aplastic anaemia 1497. True statement (s) regarding peptic ulcer disease -
d) Haemolytic anaemia a) Anterior ulcer bleeds more commonly
1487. Prepyloric or channel ulcer in the stomach is b) Posteriorly perforated ulcer is always management
termed as- (Corned 08) conservatively (PG1 June 09)
a) type 1 b) type 2 c) Anti-H. pylori drugs must be included in the
c) type 3 d) type 4 treatment regime
1488. Which one of the following types of stomach cancers d) H.pylori is known to increase incidence of gastric
carries the best prognosis- (UPSC-11 08) malignancies
a) Superficial spreading type e) Increase acid production is prerequisite for gastric
b) Ulcerative type ulcer
c) Linitis plastica type 1498. All are true about gastric Ca except -(PG1 June 09)
d) Polypoidal type a) H-pylori association is present
1489. Treatment of choice in peptic ulcer grade m is- b) D2 gastrectomy include total gastrectomy
a) Vagotomy only (UP 08) c) Surgical non curatice lesion should not be resected
b) Vagotomy and antrectomy d) Pt under total gastrectomy should be given vit B 12
c) Vagotomy and pyloroplasty e) Haematemesis present in majority of patients
d) Highly selective vagotomy 1499. Posterior perforated ulcer on pyloric antrum cause
1490. An otherwise normal female presents with symptoms .abcss formation in- (PG1 June 09)
of flatulent dyspepsia. She was started on proton a) Greater sac b) Lesser sac
pump inhibitors, which controlled her symptoms. c) Pouch of morrison d) Omental bursa
The next step in management of this condition should e) Right subphrenic
be- (AI 08) 1500. Which one of the following is NOT a feature of
a) Immediate laparoscopic cholecstectomy congenital hypertrophic pyloric stenosis-(UPSC-1 09)
b) Laparotomy after 1 or 2 months a) More common in males b) Projectile vomiting
c) Wait & watch c) Diarrhea d) Malnutrition
d) Endoscopic retrograde cholangiopancreatography 1501. True about gastric stump carcinoma-
1491. Metabolic abnormality seen in gastric outlet a) Enterogastric reflex is the cause (PG1 Nov 09)
obstruction is- (Aiirns May 08) b) Prognosis good after surgery
a) Hypochloremic, hypokalemic acidosis c) It is always adenocarcinoma in nature
b) Hypochloremic, hypokalemic alkalosis d) Diffuse type is only variety
c) Hyperchloremic, hypokalemic acidosis 1502. True regarding gastric antral vascular ectasia
d) Hypochloremic, hyperkalemic alkalosis (GAVE)- (PG1 Nov 09)
1492. FalseaboutGastriclymphomais- (AiimsMay08) a) Capillary lesion
a) Stomach is the most common site b) Arterial type
b) Asosciate with H. pylori infection c) Difficult to manage because of multiplicity & size
c) Total gastrectomy with adjuvant chemotherapy is d) Best treatment argon plasma coagulation
the treatment of choice e) Associated with chronic renal failure & iron
d) 5 yr survival rate after treatment is 60%. deficiency anaemia

1484)d 1485)b 1486)a 1487)c 1488)a 1489)b 1490)c 1491)b 1492)c 1493)All 1494)c 1495)d 1496)b 1497)d
1498)e 1499)b 1500)c 150l)a 1502)a,b,e
SURGERY [ 252]

1503. Increased risk of gastric carcinomias associated with- 1511. All of the following about gastrointestinal carcinoid
a) Intestinal metaplasia (PG/ Nov 09) tumors are true, except- (AI 10)
b) Polyp hyperplastic or adenoma a) Small intestine and appendix account for almost
c) Atrophic gastritis 60% of all gastrointestinal carcinoid.
d) Corossive antral stricture b) Rectum is spared
e) Eosinophilic gastritis c) 5 year survival for carcinoid tumors is> 60%
1504. Treatment of high lying ulcer near gastro- d) Appendicial carcinoids are more common in
oesophagealjunction is/are- (PGI Nov 09)
females than males
a) Pouchet procedure
1512. All of the following are primarily restrictive
b) Kelling- Madlenger operation
c) Csendes procedure operation for morbid obesity, except- (AI 10)
d) Total gasrectomy a) Vertical band gastroplasty
e) Vagotomy & pyloroplasty b) Switch duodenal operation
1505. Incorrect about gastric ulcer- (PGI Nov 09) c) Rouxen Y operation
a) Most common on lesser curvature d) Laparoscopic adjustable gastric banding
b) 75%H.pylorirelated 1513. A 35-year old male who bad chronic duodenal ulcer
c) Type IV ulcer most common type for the last six years presents with worsening of
d) Treatment is primaryly medical symptoms, loss of periodicity of symptoms, pain on
e) 30% GU are associated with malignancy rising in the morning, sense of epigastric bloating
1506. In a case of hypertrophic pyloric stenosis, the and post-prandial vomiting. The most likely cause
metabolic disturbance is- (AllMS Nov 09) of the worsening ofhis symptoms is development of-
a) Respiratory alkalosis a) Posterior penetration (DPG 10)
b) Metabolic acidosis b) Gastric outlet obstruction
c) Metabolic alkalosis with paradoxical aciduria c) Carcinoma
d) Metabolic alkalosis with alkaline urine
d) Pancreatitis
1507. A robust male baby with vigorous feeding and
1514. H. pylori infection is associated with increased risk
immediate vomiting at 2 months of age. Most
probable diagnosis is- (AIIMS Nov 09) of- (Maharashtra 10)
a) Paralytic ileus a) Proximal gastric cancer
b) Hirschsprung's disease b) Carcinoma ofbody and pylorus of stomach
c) Brain tumor c) Carcinoma of duodenum first part
d) Congenital hypertrophic pyloric stenosis d) Carcinoma involving gastroesophageal junction
1508. A 65 years old patient of coronary artery disease 1515. Barium meal contrast radiography is better than
was on Aspirin for 2 years. He now complains of endoscopy for diagnosing which of the following?
black stools. Abdominal examination is normal. a) Large hiatus hernia (rolling type)
What is the most probable diagnosis?(AJ/MS Nov 09) b) Linitus plastica (Maharashtra 10)
a) Ileocecal TB b) Ca colon c) Chronic gastric volvulus
c) Esophageal varices d) Duodenal ulcer d) All of the above
1509. Congenital hypertrophic pyloric stenosis associated 1516. What is the most common cause of gastric outlet
with- (A.IIMS May 10) obstruction in a 4-week-old baby? (UP SC Ill 0)
a) Hypokalemic acidosis b) Hypokalemic alkalosis a) Annular pancreas
c) Hyperkalemic acidosis d) Hyperkalemic alkalosis
b) Foreign body
1510. An adult presented with bemetemesis and upper
c) Congenital hypertrophic pyloric stenosis
abdominal pain. Endoscopy revealed a growth at the
pyloric antrum of the stomach. CT scan showed d) Ouodenal atresia
growth involving the pyloric antrum without 1517. The transition between the stomach and duodenum
infiltration or invasion into surrounding structures is marked by- (UP SC 1110)
and no evidence of distant metastasis. At laparotomy a) VeinofMayo
neoplastic growth was observed to involve the b) Hepatoduodenalligament
posterior wall of stomach and the pancreas extending c) Gastroduodenal artery
6cm up to tail of pancreas. What will be the most d) Incisura
apropriate surgical management- (AI 10) 1518. Dnmpingsyndromecanoccnrafter- (UPSCJJJO)
a) Closure of the abdomen a) Billroth-II operation
b) Antrectp,u amd vagotomy b) Heller's operation
c) Partial gastrectomy+ Distal pancreatectomy c) Whipple's operation
d) Partial gastrrectomy + Distal pancreatectomy+ d) Nissen fundoplication
splenectomy

1503)a,b,c 1504)a,b 1505)c,e 1506)c 1507)d 1508)d 1509)b 1510)c 151l)b 1512)b 1513)b 1514)b 1515)d
1516)c 1517)a 1518)a
SURGERY [ 253]

1519. A 3 week old patient presenting with vomiting Which of the statements given above are correct?
and failure to thrive is found to have pyloric a) 1 and 2 only b) 2, 3 and 4 only
stenosis. What should be the next step of c) 1 and 3 only d) 1,2 and 4 only
management? (AIIMSMay 11)
a) Its emergency so do pyloromyotomy immediately
b) Fluid resuscitation may be delayed
c) Correction of electrolyte disturbances
d) Cardiopulmonary resuscitation
1520. Best test to diagnose gastroesophageal reflux
disease and quantify acid output is -(AIIMS May 11)
a) Esohagogram b) Endoscopy
c) Manometry d) 24 hrpH monitoring
1521. In a male after laparoscopic cholecystectomy,
specimen is sent for histopathology which shows
carcinoma gallbladder stage T1a. Appropriate
managementis- (AIIMSMay 11) LIVER
a) Conservative and follow up.
b) Extended cholecystectomy 1528. For bleeding varices ofthe oesophagus the common
c) Excision of all port sites operation operations is/are- (AIIMS 81, DNB 89)
d) Radiotherapy a) Portocaval shunt b) Gastrectomy
1522. A thirty year old male presents to the Emergency c) Splenectomy d) All of the above
1529. In Budd chiari syndrome the occlusion is at
Department with symptoms of epigastric pain
the- (Kerala 87, AP 86, Karn. 89)
radiating to back that wakes him up at nigh and is
a) rvc b)Renalvein
relieved by consuming food. He gives history of
c) Hepatic vein d) Spelnic vein
similar pain in the past which was diagnosed as
1530. Line ofsurgical divison ofthe lobes of the liver is-
perforated duodenal ulcer and treated with ometal
a) Falciform ligament to the diaphragm (AIIMS 85,
patch surgery on two occasions. Pain before and
b) Gall bladder bed to IVC 87)
after surgery has been contolled with proton pump c) Gall bladder bed on the It curs of diaphragm
inhibitors and analgesics. The likely diagnosis on d) One inch to the left of falciform ligament to the IVC
this occasion is- (AI 11) 1531. Honeycombliverisseenin- (JIPMER 87)
a) Duodenal Ulcer b) Gastric Ulcer a) Micronodular cirrhosis
c) Atrophic Gastritis d) Chronic Pancreatitis b) Dubin Johnson's syndrome
1523. Which of the following statements about Gastric c) Actinomycosis
carcinomaaretrue- (AI 11) d) Hydatidosis
a) Squammous cell carcinoma is the most common 1532. The sengstaken tube must maintain a pressue of
histological subtype ... to stop bleeding from varices- (JIPMER 87)
b) Often associated with Hypochlorhydria/ a) 20mmHg b)25mmHg
Achlorhydria c) 35mmHg d)45mmHg
c) Occult blood in stool is not seen 1533. Following resection of 2/3 of the liver, regeneration
d) Highly Radiosensitive tumor is complete within- (BHU 87)
1524. A 44-year-old man presents to the hospital a) 2-3 months b)8-10weeks
emergency with sudden onset pain in the c) 4-6 months d) 4-5 weeks
epigastrium and three episodes of vomiting in the 1534. What percentage of the blood flow to the liver is
last two hours. His last vomitus was slightly brownish snpplied by the hepatic artery- (UPSC 86,
in colour, lie is a known case of ankylosing a) 900/o b)20% JIPMER88)
spondylitis requiring regular medication for back c) 40% d)60%
pain. Which one ofthe following tests would best 1535. Most common nodule found in the liver is -(PGI87)
help diagnose the patient? (UPSC Ill) a) Hepatoma b) Hamartoma
a) Upper GI endoscopy b) Ultrasound abdomen c) Hemangioma d) Cho1angioadenoma
c) Ryle's tube aspiration d) H. pylori serology 1536. Spontaneous ruptureoftheliveroccursin-(BHU88)
1525. ConsiderthefoDowing statements in respect ofcongenital a) Hepatoma b) Portal hypertension
hyper trophic pyloric stenosis- (UPSC II11) c) Spherocytosis d) Secondary deposits
1. The condition is more common in males. 1537. Multiple liver secondaries are most common in
2. The investigation of choice is ultrasonography. the following cancers- (AIIMS 84)
3. Hypertrophy is maximal in the prepyloric region. a) Head of pancreas b) Stomach
4. The condition presents within the first week ofbirth c) Gall bladder d) Periampullary

1519)c 1520)d 1521)c 1522)a 1523)b 1524)a 1525)d 1526)a 1527)b 1528)a 1529)c 1530)b 1531)c 1532)c
1533)c 1534)b 1535)c 1536)a 1537)c
SURGERY [ 254]

1538. Multiple Liver secondaries are most common in 1549. True treatment regarding hepatic amoebiasis-
followingpersons- (AIIMS 84) a) More common in females (PGI 96)
a) Overweight b) Undeweight b) Multiple lesions
c) Normal weight d) Short and shunted c) Mostly treated conservatively
1539. Liver malignancies can be studied by- (PGI 88) d) Jaundice is common
a) Radionurclear study b) Ultrasound 1550. Criteria for assessing prognosis of systemic portal
c) Cf d) Laprascope shunt is by- (AIIMS 96)
e) All a) Serum albumin b) Resistant ascites
1540. Bud chiari syndrome is due to thrombosis of- c) Serum billirubin d) Type of shunt
a) lnfrarenaliVC (PGI 88) 1551. All are complications of hydatid cyst in the liver
b) RenalpartofiVC except- (AP 97)
c) Superior mesentric vein thrombosis a) Jaundice b) Suppuration
d) Hepatic veins c) Cirrhosis d) Rupture
1541. Contra indications of porta systemic shunting 1552. Which one of the following is not a treatment of
include- (PGI 88) gastroesophageal variceal haemorrhage-
a) Serum albumin less than 3 mg% a) Sclerotherapy (UPSC OJ)
b) Massive ascites b) Sengstaken tube
c) Significantjuandice c) Transjugular intrahepatic portacaval shunt
d) All of the above d) Gastric freezing
1542. Drug induced portal hypertension is seen 1553. With couniard's nomenclature which one of the
with- (PGI 78, AIIMS 79) following segments of liver has an independent
a) VitaminA toxicity b) Methotrexate vascularisation- (UPSC 02)
c) Aldomet d) Valproic acid a) Segment I b) Segment II
1543. Which is the commonest incidentaloma detected c) Segment IV d) Segment VIII
in the liver- (Karn. 94) 1554. In hepatocellular carcinoma, factor important is-
a) Focal nodular hyperplasia a) Alcoholic hepatitis (PGI 97)
b) Haemangioma b) Schistomiasis
c) Hepatocellular adenoma c) Cirrhosis
d) Hydatid cyst d) Fasciolepsis buski infestation
1544. A 40 year old male presents with a painless cystic 1555. True about TIPSS- (PGI 98)
liver enlargement of four years duration without a) It is a type of portocaval shunt
fever or jaundice. The most diagnosis is-(UPSC 96) b) It is intrahepatic shunt
a) Amoebic liver abscess b) Hepatoma c) Performed by passing endoscopes
c) Hydatid cyst of liver d) Choledochal cyst d) Most suitable for patient going for liver transplant
1545. A 20 years old football player received a hard 1556. Liver transplant is not possible for- (PGI 98)
kick in the epigastrium. A large cystic swelling a) Liver b) Pancreas
appeared in the epigastrium two weeks later. The c) Small intestine d) Cornea
most likely diagnosis is - (UPSC 96) 1557. In non hemolytic jaundice, urobilinogen is seen
a) Hydatic cyst ofliver in- (PG/99)
b) Amoebic liver abscess a) Obstructive jaundice b) Hepatic fibrosis
c) Pseudopancreatic cyst c) Fatty liver d) Infective hepatitis
d) Haematoma of rectus sheath 1558. True about carcinoma liver- (PGI 2000)
1546. Regarding hepatic artery ligation which statement a) AFP increased in 70% cases
is false - (Karnat. 96) b) Resection possible in few cases only
a) The best rusults are obtained incase ofhaembilla c) USG guided biopsy is good for diagnosis
b) Not useful in primary hepatoma d) Aflatoxin & thorotrast are not risk factor
c) Can cure secondary carcinoma 1559. Trueaboutamoebicliverabscess- (PGI 2000)
d) Must be coverd by massive antibiotic administration a) Frequently associated with diarrhea at presentation
1547. Liver biopsy is done through 8th midaxillary line b) Most common in right lobe liver
toavoid- (Al97) c) Can rupture into pleural cavity
a) Lung b) Pleural cavity d) Almost never respond to metronidazole
c) Subdiaphragmatic space d) Gall bladder 1560. Trueaboutpyogenicliverabscess- (PGIOJ)
1548. In child's criteria partial encephalopaty billirubin a) Single & large abscess
2.5 mgldl albumin 3 gm/dl controlled ascites b) X-ray features are diagnostic
indicates - (PGI 96) c) Serology is confirmatory investigation
a) Grade A b) GradeB d) Systemic complaints, fever & jaundice common
c) GradeC d) More information needed e) Liver enzyme abnormalities are common & severe

I538)a 1539)e 1540)d 154l)d 1542)a 1543)b 1544)c 1545)c 1546)b,c l547)a 1548)b 1549)c 1550)a,b,c
15Sl)c l552)d 1553)a 1554)a,c l55S)b,d 1556)c 1557)d 1558)a,b,c 1559)b,c 1560)a
SURGERY [ 255]

1561. TrueaboutHepatocellularcarcinoma- (PG/02) 1571. The Couinaud's segmental nomenclature is based


a) Most common tumour of liver on the position of the- (AI 04)
b) Resectable only in 1% cases a) Hepatic veins and portal vein
c) >70% of cases shows -edAFP b) Hepatic veins and biliary ducts
d) USG guided aspiration biopsy is used for c) Portal vein and biliary ducts
diagnosis d) Portal vein and hepatic artery
1562. Surgical lobes ofliver are divided on the basis of- 1572. Vascular inflow occlusion of the liver is by-
a) Hepatic artery b) Hepatic vein (PGI 02) a) Clamping the hepatic artery
c) Bile ducts d) Portal vein b) Occluding the portal vein
c) Clamping the hepatic veins
e) Central veins
d) The pringle manoeuvre
1563. True about hepatocelluar ca is- (PGI 02)
1573. Which of the following liver tumour has a propensity
a) Associated with HBV and HCV
to invade the portal or hepatic vein- (AIIMS 04)
b) Cirrhosis is predisposing factor
a) Cavernous haemangioma
c) Fibrolamellar variety is not associated with b) Hepatocellular carcinoma
cirrhosis c) Focal Nodular hyperplasia
d) Low propensity of vascular invasion d) Hepatic adenoma
e) Alcoholic cirrhosis does not predispose to HCC 1574. A 17 year old boy is admitted to the hospital after a
1564. Portal hypertension following portal vein road traffic accident. Per abdomen examination is
thrombosis are guided by- (PGI 03) normal. After adequate resuscitation, his pulse rate
a) t in splenic pulp pressure is 80/min and BP is 110170 mmHg. Abdominal CT
b) t in portal vein pressure reveals 1 em deep laceration in the left lobe of the
c) tin hepatic vein pressure liver extending from the dome more than halfway
d) Portal vein doppler study through the parenchyma. Appropriate management
1565. Two most important clinical features of primary at this time would be- (UPSC 05)
biliary cirrhosis- (PGI 03) a) Conservative treatment
a) Generalised pruritus b) Jaundice b) Abdominal exploration and packing ofhepatic wounds
c) Fatigue d) Clubbing c) Abdominal exploration and ligation ofleft hepatic
e) Hematemesis artery
1566. Tumour markers of Hepatocellular carcinoma- d) Left hepatectomy
a) AFP b)CEA (PG/03) 1575. The operation that precipitates portoasystemic
c) Hffi d)CA-19-9 encephalopathy is- (MAllE 05)
e) CA-125 a) Splenorenal shunt
b) Siguira operation
1567. Trueabouthydatidcystofliver- (PG/04}
c) Talma-Marison Operation
a) Caused by E. granulosus
d) Portacaval anastomosis
b) In 80% cases cyst is single
1576. Which one of the following is not correct with regard
c) Blood culture helpful
to amoebic Hver abscess- (ICS 05)
d) Surgery is the only treatment. a) It usual occurrence is in the right lobe
1568. Child-pugh criteria doesnot includes- (PGI 04) b) Patient is toxic
a) Encephalopathy b) Ascites c) Surgical drainage is always indicated
c) Albumin d) Creatinine d) Extension of abscess from liver to pericardium is
e) ALT the most dreaded complication
1569. TrueaboutHepatocellularCarcinoma- (PGI04) 1577. TrueaboutHydatiddisease- (PGIJune05)
a) Most prevalent malignancy a) MC site is lung
b) Common liver tumor b) Pre-op Albendazole followed by surgery is the
c) HBV predisposes treatment of choice
d) Patient present in early stage c) 20% saline is scolicidal
e) Prognosis good d) Puncute, aspiration, injection & reaspiration
1570. During surgical exploration for hydrated cyst of (PAIR) done in hepatic hydatids
the liver, any of the following agents can be used 1578. Indication oflivertransplantationAIE-
as scolicidal agent except- (UPSC 04) a) Biliary atresia (PGI June 05)
a) Hypertonic sodium chloride b) Sclerosing cholangitis
b) Formalin c) Hepatitis A
c) Cetrimide d) Cirrhosis
d) Povidone Iodine e) Fulminant hepatic failure

156l)c,d 1562)a,c,d 1563)a,b,c 1564)a,b,d 1565)a,c 1566)a 1567)a,b 1568)d,e 1569)c 1570)d 157l)a 1572)d
1573)b 1574)b 1575)d 1576)c>b 1577)b,c,d 1578)c
SURGERY [ 256]

1579. A patient presented to emergency ward with massive 1589. The minimum amount of normal perfused liver
upper gastrointestinal bleed. On examination, he has parenchyma to be left intact when a hepatic
mild splenomegaly. In the absence of any other resection is planned is- (Comed 07)
information available. Which of the following is the a) 100/o b) 20%
most appropriate therapeutic modality- c) 500/o d) 75%
a) Intravenous propranolol (AIIMSNov 05) 1590. The earliest symptom in primary biliary cirrhosis
b) Intravenous vasopressin is - ( Comed 07)
c) Intravenous pantoprazole a) Jaundice b) Pruritus
d) Intravenous somatostatin c) Melanosis d) Vomiting
1580. The following is ideal for the treatment with 1591. Normal portal vein pressure is- (Comed 08)
injection of sclerosing agents- (AI 05) a) <3mmHg b)3-5mmHg
a) External hemorrhoids c) 5-lOmmHg d) lOto 12mmofHg
b) Internal hemorrhoids 1592. In Couinaud classification, segment IV of liver
c) Prolapsed hemorrhoids is - (AIIMS Nov 07)
d) Strangulated hemorrhoids a) Caudate lobe b) Quadrate lobe
1581. All of the following modalities can be used for in c) Right lobe d) Left lobe
situ ablation of liver secondaries, except- (AI 06) 1593. A surgeon excises a portion of liver to the left ofthe
a) Ultrasonic waves b) Cryotherapy attachment ofthe falciform ligament. The segments
c) Alcohol d) Radio-frequency that have been resected are- (AI 08)
1582. Indications for fine needle aspiration in liver abscess a) Segment laand4 b) Segment 1 and4b
are- (PGI June 06) c) Segment 2 and 3 d) Segment 1 and 3
a) Recurrent 1594. Auxiliary orthotopic liver transplant is indicated for-
b) Left lobe a) Metabolic liver disease (Aiims May 08)
c) Refractory to treatment after 48-72 hrs b) As a standby procedure until finding a suitable
d)> 10 ems size donor
e) Multiple c) Drug induced hepatic failure
1583. Best test to differentiate between medicals and d) Acute fulminant liver failure for any cause
surgical jaudice is? (APPG 06) 1595. Indication of aspiration in liver abscess AlE-
a) Akaline phosphatase a) Left lobe abscess (PGI June 08)
b) Bilirubin and serum enzymes b) Deep & less than 5 mm size
c) Enzymes c) Multiple
d) Ultrasound d) Recurrent
1584. TIPSS involves percutaneous creation of a shunt 1596. Treamentofhydatidcyst? (APPG08)
between- (Karnataka pgmee 2006) a) Excision of cyst
a) Portal vein and vena cava b) Percutaneous drainage
b) Portal vein and hepatic vein c) Conservative managment
c) Hepatic vein and vena cava d) None
d) Portal vein and hepatic artery 1597. Central dot sign is seen in? (AIIMS Nov 08)
1585. Contraindications to major hepatic resection for a) Primary sclerosing cholangitis
metastatic disease include all of the following except- b) Liver Hamartoma
a) Total hepatic involvement (Karn 06) c) Caroli's disease
b) Advanced cirrhosis d) Polycystic liver disease
c) Extrahepatic tumor involvement 1598. Right hepatic duct drains all, except-
d) Jaundice from extrinsic ductal obstruction a) Segment! b)Segmentiii (AIIMSMay09)
1586. All are used in treatment of amoebic liver abscess c) Segment V d) Segment VI
except- (MAHE 07) 1599. Falseabouthepaticduct- (AIIMSMay09)
a) Diloxanide furoate b) Chloroquine a) Left hepatic duct formed in umbilical fissure
c) Metronidazole d) Emetine b) Caudate lobe drains only left hepatic duct
1587. "Crumbled egg appearance" in liver seen in- c) Right hepatic duct formed by V and VIII segments
a) Hepatic adenoma (UP 07) d) Left hepatic duct crosses IV segment
b) Chronic amoebic liver abscess 1600. Not true about choledochal cyst is -(AIIMS May 09)
c) Hydatid liver disease a) Associated with anomalous junction of the
d) Haemangioma pancreatic and biliary duct.
1588. Charcot's triad- (PGI June 07) b) Type 2 is most common
a) Pain b)Fever c) Surgical removal is the treatment of choice
c) Jaundice d)Anaemia d) If ruptures can cause biliary peritonitis

1579)c 1580)b 158l)a 1582)d 1583)d 1584)b 1585)d 1586)a 1587)c 1588)a,b,c 1589)b 1590)b 1591)c
1592)b 1593)c 1594)d 1595)b,c,d 1596)b 1597)c 1598)b 1599)b 1600)b
SURGERY [ 257]

1601. Liver biopsy is done through 8th midaxillary line to 1609. Following is true about Focal nodular hyperplasia
avoid- (DELHI PG Feb. 09) (FNH)except- (Maharashtra IO)
a) Lung b) Pleural cavity a) Unusual benign tumor ofliver
c) Sub diaphragmatic d) Gall bladder b) Commonly symptomatic and occurs as multiple
1602. Which of the following liver tumors always merit liver nodules
surgery ? (DELHI PG Mar. 09) c) More common in females than in males
a) Hemangioma d) Contains hepatocytes and Kupffer cells
1610. The percutaneous PAIR therapy used in the
b) Hepatic adenoma
treatment of uncomplicated hepatic hydatid cyst can
c) Focal nodular hyperplasia
be associated with the following complications,
d) Peliosis hepatic except- (UPSCII IO)
1603. In patients with cirrhosis of the liver the site of a) Urticaria b) Anaphylaxis
obstruction in the portal system is in the-(Manipal 09) c) Hypotension d)Vorniting
a) Hepatic vein b) Post sinusoidal 1611. All ae true about amoebic liver abscess except-
c) Extra hepatic portal vein d) Sinusoids a) Metronidazole is mainstay of treatment
1604. In Budd-Chiari syndrome, the site of venous b) Multifocal abscess can not be treated by aspiratio
thrombosis is- (Manipal 09) c) More common in left side (PGI May IO)
a) Infra-hepatic inferior vena cava d) More common in female
b) Infra-renal inferior vena cava 1612. False about hydatid cyst ofliver- (PGI Nov. I 0)
c) Hepatic veins a) Hepatic resection is never done
d) Portal vein b) Laparoscopic aspiration of cyst is performed
1605. A 20-year-old man presents with repeated episodes c) Most commonly located in the right liver
ofhaematemesis. There is no history of jaundice or d) Mostly asymptomatic
liver decompensation. On examination, the e) Most common causative organism is echinococcus
granulosus
significant findings include moderate splenomegaly
1613. Not an indication for PAIR treatment in hydatid
and presence of esophageal varices. There is no
cyst- (PGI Nov. I 0)
ascites or peptic ulceration. The liver function tests a) Size> 5 em b) Multiloculated
are normal. What is the most likely diagnosis - c) Cyst in lung d) Recurrence after surgery
a) Extrahepatic portal venous obstruction e) Perforated cyst
b) Non-cirrhotic portal fibrosis (UPSC-I 09) 1614. False about hepatic duct-
c) Cirrhosis a) Left hepatic duct formed in umbilical fissure
d) Hepatic venous outflow tract obstruction b) Caudate lobe drains only left hepatic duct
1606. Consider the following with reference to the c) Right hepatic duct formed by V and VIII segments
management of portal hypertension SURGERY: d) Left hepatic duct crosses IV segment
1. Infusion of vasopressin (UPSC-II 09) 1615. Surgeon excises a portion of liver to the left of the
2. General resuscitation attachment of the falciform ligament. The segments
3. Devascularisation procedure that have been resected are- (AI 11)
4. Endoscopic sclerotherapy a) Segment laand4 b) Segment 1 and4b
What is the appropriate sequence in the line of c) Segment 2 and 3 d) Segment 1 and 3
management in the event of massive variceal bleeding 1616. The prognostic indicators in a cse of portal
hypertension include- (UPSC II 11)
in portal hypertension ?
1. Serum globulin 2. Serum albumin
a) 1,4,2,3 b)2, 1,4,3
3. Serum bilirubin 4. Ascites
c)4,2,1,3 d)3,2,1,4
Select the correct answer using the code given
1607. In orthotropic liver transplantation, which is the best below- (UPSC II 11)
way to get bile drainage in donor liver- (AI I 0) a) 1,2and3 b)1and4only
a) Donor bile duct with recipient bile duct or Roux en c) 2, 3 and 4 d) 3 and 4 only
Y choledochojejunostomy 1617. A 40-year old alcoholic male complains of acute pain
b) Donor bile duct with duodenum of recipient in the epigastrium associated with vomiting for the
c) Donor bile duct with jejunum of recipient last 10 days. On clinical examination, he is found to
d) External drainage for few days followed by have a mass in the epigastrium. The most likely
choledochojejunostomy diagnosis is- (UPSC II 11)
1608. In Budd Chiari syndrome, the occlusion is at the- a) Perforated peptic ulcer with sub-hepatic abscess
a) NC b) Renal vein (DPG I 0) b) Pseudopancreatic cyst
c) Hepatic vein d) Splenic vein c) Carcinoma of head of the pancreas
d) Hepatoma in left lobe ofliver

160l)a 1602)b 1603)d 1604)c 1605)b 1606)b 1607)a 1608)c 1609)b 1610)None 1611)c,d 1612)a 1613)d,e
1614)b 1615)c 1616)c 1617)b
SURGERY [ 258]

SPLEEN 1628. Splenectomy is least useful in- (All India 89)


a) Congenital elliptocytosis
1618. Most common complication post splenectomy is- b) Thalasemia major
a) Formation of pseudo cyst (UPSC 86, c) Congenital spherocytic anaemia
b) Injury to tail of pancreas JIPMER 88) d) Hereditary nonspherocytic haemolytic anaemea
c) Left lower lobe atelectasis 1629. Splenectomyisindicatedin- (AIIMS 87)
d) Perforation of stomach a) Sickle cell anemia
1619. Most common post operative complication of b) Hereditary spherocytosis
splenectomy is- (BHU 88) c) Hemoglobin C disease
a) Left side pleural effusion d) Hodgkins lymphoma
b) Subphrenic abscess 1630. Removal of senescent RBC from circulation by
c) Left lower lobe atelectasis spleen is called- (PGI 79,JIPMER 80)
d) Pancreatic fistula a) Culling b) Pitting
1620. Accidental small splenic rupture is treated with- c) Filtering d) Phagocytosis
a) Catgut sutures (PGI 88) 1631. Hypersplenism is seen in- (PGI 97)
b) Silk sutures a) Pancytopenia b) Thrombocytopenia
c) Omental patch c) Leukopenia d) Polycythemia
d) Catgut suturing with omental patch 1632. After splenectomy, most common infections
e) Splenectomy is- (PG/97, AIIMS 84)
1621. The spleen contains about. ..... % of the total blood a) Pneumococcus b) Staphylococcus
volume- (PGI 85) c) Esch. coli d) Pneumocystis carinii
a) 1 b) 2 1633. Most common malignancy affecting spleen is -(PGI
c) 5 d) 7 a) Angiosarcoma b)Hamartoma 97)
1622. The most reliable radiological evidence of splenic c) Secondaries d) Lymphoma
rupture- (PGI 85, Jipmer 86, 87). 1634. Post splenectomy sepsis is common in -(PGI 2000)
a) Obliteration of psoas shadow a) ITP b) Thalassemia
b) Elevation ofleft diaphragm c) Hereditary spherocytosis d) Trauma
c) Indendation of gastric fundic shadow 1635. Infections encountered after splenectomy-
a) H. influenza b) Streptococcus pneumoniae
d) Fracture ribs of the left lower side
c) Klebsiela d) Neisseria (PGI June 05)
e) Obliteration of splenic outline
1636. In splenic injury conservative management is done
1623. Most common complication of splenectomy is-
in- (MAHE05)
a) Pancreatic leak (AIIMS 92)
a) Hemodynamically unstable
b) Pulmonary complications
b) Young patients
c) Pneumococcal peritonitis
c) Shattered spleen
d) Hemorrhage
d) Extreme pallor and hypotension b
1624. Splenosis means- (PGI 79, AIIMS 85)
1637. Positive kehrs sign is- (Manipal 06)
a) Infection of spleen a) Haemopertionium b) Acute cholecystisi
b) Presence of acessory spleens c) Acute pancreatitis d) Amoebic abscess
c) Rupture of spleen and distribution of its tissue on 1638. An evidence that splenectomy might benifit a patient
peritoneum with idiopathic thrombocytopenic purpura includes
d) Non-functioning spleen which of the following? (UPSC 07)
1625. Splenic vein thrombosis is best treated by- a) A significant enlargement of the spleen
a) Splenectomy (USPC 97) b) A high reticulocyte count
b) Porta-caval shunt c) Patients age less than five years
c) Spleno-renal shunt d) An increase in platelet count on corticosteroid therapy
d) Mesenterico-caval shunt 1639. What is the most common malignancy affecting
1626. All of the following are features of splenic rupture spleen- (UPSC-11 08)
except- (TN 99) a) Angiosarcoma b) Hamartoma
a) Obliteration of psoas shadow c) Secondaries d) Lymphoma
b) Obliteration of splenic outline 1640. A patient with ITP has a platelet count of 50000 and
c) Elevation ofleft diaphragm is being planned for splenectomy. What is the best
d) Obliteration of colonic air bubble time for platelet infusion in this patient - (AI 08)
1627. One of the following does not cause increase in a) 2 hours before surgery
the size of spleen in later stages- (MAN/PAL OJ) b) At the time of skin incision
a) Sickle cell anaemia b) Cirrhosis c) After ligating the splenic artery
c) Infectious mononucleosis d) Hairy cell leukaemia d) Immediately after removal of spleen

1618)c 1619)c 1620)d 1621)b 1622)e 1623)b 1624)c 1625)a 1626)d 1627)a 1628)d 1629)a,b,d 1630)a
1631)a,b,c 1632)a 1633)d 1634)a,b 1635)a,b,c 1636)b 1637)a 1638)d 1639)d 1640)c
SURGERY [ 259]

1641. During splenectomy in ITP, platelet infusion is given- 1652. A 40 year old lady is found to have a symptomatic
a) Immediately after ligating splenic vein(DPGEE 08) galls sotnes. The treatment of choice is-(AJ/MS 85)
b) Immediately after removal of spleen a) Observation b) Cholecystostomy
c) After incision c) Cholecystectomy d) Fat free diet
d) Next day of surgery 1653. The following can be done in obstructive
1642. All of the following are true about OPSI jaundice- (PGI 88)
(opportunistic post splenectomy infection) except - a) Vitamin K injections b)Vitamin C injections
c) Dehydration therapy d) Extarnal drainage
a) Develop 1-5 year after splenectomy(PGI June 09)
1654. Bile is concentra in the gall bladder to.•••••times -
b) Max risk is within 1 yr of splenectomy a) 5 b) 10 (PGI 88)
c) Begin with headache, myalgia & fever c) 20 d)50
d) May present with severe septic shock 1655. The components of Saints triad are- (PGI 88)
e) Usually not respond with antibiotic Treatment a) Gall stones b) Mesentric cysts
1643. Mostcommoncystsofthespleenare- (Al10) c) Hiatus hernia d) Diverticulosis
a) Hydatid cyst b) Dermatoid cyst 1656. In cholangitis, the organism mostly responsible
c) Pseudocyst d) Lymphangioma is- (PGI88)
1644. A patient presents with fever for 3 weeks. On a) E. coli b) Streptococcus
examination he is observed to have splenomegaly. c) E. Histolytica d) Clostridium
Ultrasonography reveals a hypoechoic shadow in 1657. The gall bladder is capable of distending •••••• ml.-
spleen near the hilum. Gram negative bacilli are a) 10 b) 20 (PGI 88)
isolated on blood culture. Which of the following is c) 40 d)50
the most likely causative organism- (AI 10) 1658. Association of carcinoma gall bladder with gall
a) Cytomegalovirus b) Toxoplasmosis stones is about- (PGI 85)
c) Salmonella d) Lymphoma virus a) 50% b)70%
c) 90% d)20%
1645. Most common infections after splenectomy are-
1659. Best investigative modality for gall bladder-
a) Capsulated bacteria (DPG 10)
a) CXXJ b) PTC (PGI 85)
b) Uncapsulated bacteria c) Ultrasound d) Intravenous cholangiogram
c) Gram positive sepsis 1660. Triad of Jaundice, chills and fever occur in -(Al90)
d) Gram negative bacteria a) CBD stones b) Post cholecystectomy
1646. Management of grade 3 splenic trauma in a stale c) Acute hepatitis d) Pancreatitis
child- (PGINov. 10) 1661. Which does not contribute to Enterobiliary
a) Embolization b) Partial splenectomy Fistula- (JIPMER 91)
c) Total splenectomy d) Conservative a) Duodenal ulcer b) Gall stones
c) Gastric ulcer d) Carcinoma gall bladder
GALL BLADDER & BILE DUCT 1662. After a routine cholecystectomy, malignancy is
detected in the gall bladder, the next line of treatment
1647. Following tumours have the best prognosis - is- (AIIMS91)
a) Carcinomagallbladder (AIIMS 84) a) Right partial hepatectomy
b) Carcinoma pancreas b) Chemotherapy via hepatic artery
c) Hepatoma c) Hepatic wedge resection
d) Cholangiocarcinoma of the CBD d) Regular follow up
1648. Commonest typeofgallstoneis- (Kerala 87) 1663. Commonest association seen in carcinoma gall
a) Cholesterol stone b) Pigment bladder- (AIIMS 91)
c) Mixed d) All are equally common a) Peritoneal deposits
b) Duodenal infiltration
1649. Percentage of gall stones which are radio opaque -
c) Secondaries liver
a) 10% b)20% (JIPMER86)
d) Cystic node involvement
c) 30% d)50% 1664. Incidence of gall stone disease is increased in all
e) 80% of the following except- (AI 92)
1650. Which of the following is not a component of a) Truncal vagotomy b) Jejunal resection
Saint's triad- (PGI 86) c) ileal resection d) Right Hemicolectomy
a) Galljstones b)Diverticulicoli 1665. Strawberry gallbladder is seen in- (JIPMER 81)
c) Crohn's disease d) Hiatus hernia a) Gangrene of gallbladder
1651. Common bile duct stones will manifest all except- b) Porcelain gall baldder
a) Distended gall bladder b) Jaundice (AI 89) c) Adenomatosis
c) Itching d)Clay coloured stools d) Cholesterolosis

1641)a 1642)b 1643)a 1644)c 1645)a 1646)a,d1647)c 1648)c 1649)a 1650)c 1651)a 1652)c 1653)a,d 1654)a,b
1655)a,c,d 1656)a 1657)d 1658)c 1659)c 1660)a 1661)c 1662)d 1663)c 1664)a,b,d 1665)d
SURGERY [ 260]

1666. Cholangiocarcinoma histologically resembles- 1678. The most common malignancy after
a) Sq. cell type (A1IMS 79, Delhi 79, 92) cholecystectomy is of- (DELHI PG 96)
b) Colloid cell type a) Colon b) Stomach
c) Schirrhous type c) Pancreas d) ileum
d) Columnar cell type 1679. Following are true about choledochal cyst except
1667. Which ofthe following is the commonest site for a) Presents mostly in childhood (AIIMS 96)
impaction of the gall stones- (JIMPER 81,87 b) Presents jaundice abdomen, mass abdomen pain
c) Congenital weakness of gall bladder wall
a) Common hepatic duct AIIMS 84)
d) Treatment is cystojejunostomy
b) Ampulla of vater
1680. After exploration of common bile duct, the T-
c) Retroduodenalportion of common bile duct Tube is removed on which of the following
d) Supradodenal portion of common bile duct days- (Karnat 96)
1668. Gall stones do not contain- (JIPMER 81, AMU 86) a) 3 postop.day b) 4 postop.day
a) Oxalate b) Cholesterol c) 12 postop.day d) 6 postop.day
c) Phosphate d) Carbonate 1681. An ultrasound examination shows dilated
1669. Graham Cole test refers to- (AIIMS 81, AP 89) intrahepatic biliary channels with a small gall
a) Oral cholecystography bladder. The most likely possibility is -(Karn 96,94)
b) Intravenous cholangiography a) Gall baldder stone
c) Pre-oprerative cholangiography b) Pancreatic calculus
d) Post-operative cholangiography c) Common bile duct stone
e) Tomography d) Carcinoma of the head of the pancreas
1670. Gall stones are composed of all except -(Kerala 94) 1682. The most common cuase of suppurative cholangitis
a) Oxalates b) Bile salts is- (UP 97)
c) Choesterol d) Bile pigments a) Stone in common bile duct
1671. Inability to visualise gall bladder in OCG is due to- b) Cancer ofthe ampulla of vater
a) Malabsorption (Kerala 94) c) Choledochal cyst
b) Poor functioning of gall bladder d) Empyema of gall bladder
c) Non functioning ofliver 1683. All of the following increase the risk of gallbladder
d) All carcinoma except - (AIIMS 97)
1672. Investigation of choice in gall bladder stone a) Chronic pancreatitis b) Choledochal cyst
is - (Kerala 94) c) Chloronchis d) Ulcerative colitis
a) USG b)X-rayabdomen 1684. Investigation of choice of case of obstructive
c) (XX} d) Intravenous cholangiogram jaundice- (AI 97)
1673. Cholesterosis- (Karn 94) a) USG b)PainX-ray
a) Disease of defective metabolism of choline c) CT scan d)ERCP
b) Concerned with epithelial tumours of brain' 1685. Treatment of choice of choledochal cyst- (PGI 96)
a) Hepatic transplantation
c) Diffuse deposition of cholesterol in mucosa of
b) Cystojejunostomy
gall bladder
c) Excision of cyst with hepatojejunostomy
d) Disease concerned with obstructive jaundice
d) Gastrojejunostomy
1674. Cholesterol gall stones are due to- (JIPMER 95)
1686. After surgery, a small left over stone in the CBD
a) Decreased motility of Gall bladder
is best treated by- (Kerala 97)
b) Hypo secretion of bile salts
a) ERCP
c) Hyper cholesterolemia
b) ESWL
d) All of the above
c) Heparinised saline through T tube
1675. best suture for common bile duct is- (JIPMER 95)
d) Repeat surgery
a) Synthetic absorbable synthetic
1687. 'STRAWBERRYGALLBLADDER'is- (AP97)
b) Synthetic non-absorbable a) Adenoma gall bladder
c) Non-synthetic absorbable b) Acute cholecystitis
d) Non-synthetic non-absorbable c) Cholesterosis
1676. Contra indication for laproscopic cholecystectomy d) Carcinoma gall bladder
is all except- (Kerala 9 5) 1688. Treatment of choice of a CBD stone of 2.5 ems
a) Shrunken liver b) Previous laprotomy size, diagnosed 2 years after cholecystectomy-
c) Emphysema d) Obese individual a) Choledochotomy and T tube (AI 99)
1677. Charot's triad is seen in- (AI 96) b) Dormia basket
a) Acute pancreatitis · b) Hemobilia c) Transduodenal sphincterotomy
c) Stone in bileduct d) Chronic cholecystitis d) Supraduodenal choledochotomy

1666)c 1667)b 1668)a 1669)a 1670)a 167l)d 1672)a 1673)c 1674)d 1675)c 1676)a 1677)c 1678)b 1679)c
1680)c 168l)c 1682)a 1683)a,b,c,d 1684)a 1685)c 1686)a 1687)c 1688)c
SURGERY[261]

1689. In a post-cholecystectomy patient, who develops 1699. Features of healthy gallbladder on laprotomy are-
bile leakage due to CBD TEAR 5 days pot-op, with a) Typical "sea-green" colored (PGI 2000)
NOT-TUBE INSERTED atthe time surgery ideal b) Wall in thin & elastic
treatmentis- (AIIMS98) c) Cannot be emptied
a) ERCP procedure with stent d) Not easily visible
b) Hepaticojejunostomy 1700. Which ofthe following statements is true regarding
c) Primary repair after reopening abdomen cholangitis- (PGI 01)
d) Ultrasound guided insertion of drain a) Increased leucocyte count
1690. CholangiographyviaT-tube done after how many b) Increased transaminases
days of cholesystectomy- (TN 99) c) Increased bilirubin
a) 1-5 days b) 5-9 days d) Increased alkaline phosphatase
c) 10-14days d) 15-20days e) Association with fever and chills
1691. Gall stones- (Orissa 98) 1701. Gallstones may be complicated by which among the
a) Are about twice as common in men as in women following- (PGI 01)
b) There is an increased incidence of stones in a) Pancreatitis b) Choledocholithiasis
-diabetics c) Acute cholecystitis d) Carcinoma stomach
c) About 80-90% of gall stones are radio-opaque e) Carcinoma pancreas
d) Are usually more than 50 mm in diameter 1702. Theatmentfor common bile duct stone is y -(PGI 01)
1692. Regarding stones in gall bladder the following are a) ESWL
true except- (Kerala 2k) b) Exploration of bile duct and recovery of stones
a) Mixed stones are common in the west c) Bile duct stenting
b) In Saints Triad diverticulosis of colon and d) Nasobiliary drainage
hiatus hernia coexist e) Percutaneous drainage
c) Is a risk factor in the development of GB carcinoma 1703. Acalculous cholecystitis is caused by- (PGI 01)
d) 90 % of gall stones are radio-opaque a) Diabetes mellitus b) Total parenteral nutrition
e) A mucocele of GB is cuased by a stone impacted c) Tuberculosis d)Anemia
in the Hartmann's pouch e) Malignancy
1693. The substance usedofOCG is- (PGI 87 Kerala 88) 1704. Which of the following are histopathological features
a) Iapanoic acid of Extra hepatic Biliary Atresia- (PGI 01)
b) Sodium diltrozite a) Bile lakes
c) Meglurnine iodothalamate b) Hepatocyte ballooning degeneration
c) Marked bile duct proliferation
d) Biligraffin
d) Fibrosis of hepatic duct
e) Dianosil
e) Parenchymal cholestasis
1694. Dye used in IV cholangiography is- (PGI 86)
1705. Better prognostic factor for operation ofbiliary duct
a) Dianosil b) Conray obstruction in newborn are- (PGI 01)
c) Biligraffin d) Myodil a) No passage of bile
1695. What is more appropriate for diagnosis of CBD b) Size of ductule > 200 micron
stones- (PGI 97) c) Weight of baby> 3 kg
a) Ultrasonography b)ERCP d) Preterm baby
c) OCCJ d) IV cholangiography e) Age of8 weeks
1696. All are seen in hemobilia except- (PGI 99) 1706. A patient having multiple Gall stones and shows 8
a) Shock b) Colicky pain mm dilation and 4 stones in CBD, best treatment
c) Melena d) Jaundice modalities are- (PGI 02)
1697. Which among the following does not lead to pigment a) Cholecystectomy with choledecholithotomy at
gallstones- (PGI 99) same setting
a) TPN b) Clonorchis sinensis b) ESWL
c) Hemolytic anemia d) Alcoholic cirrhosis c) Cholecystectomy and wait for ERCP
1698. The treatment of gall stone ileus is - (PGI 99) d) Sphicterotomy and then cholecystectomy
a) Cholecystectomy alone e) Cholecystectomy and after 14 days sphincterono
b) Removal of obstruction done
c) Cholecystectomy, closure of fistula and removal 1707. CBD stone may present with- (PGI 02)
of stone by enterotomy a) Increased bilirubin b) iWBC count
d) Cholecystectomy with closure of fistula c) i livers enzymes d) Fever with rigors
e) Hepatomegaly

1689)a 1690)c 1691)b 1692)d 1693)a 1694)c 1695)b 1696)a 1697)a 1698)b 1699)a,b 1700)All 1701)a,b,c
1702)All 1703) a,b,c,e 1704) c,d,e 1705) b,e 1706) a,d 1707) a,b,c,d
SURGERY [ 262]

1708. True about gall stones- (PG/ 02) 1716. A 69 year old male patient having coronary heart
a) More common in female disease was found to have gall bladder stones white
b) Gall stones, haitus hernia, CBD stones form undergoing a routine ultrasound of the abdomen.
Saints triad There was no history of colic or jaundice at any
c) Limely bile precipitated time. What is the best treatment advice for such
d) Lithotripsy always done a patient for his gall bladder stones- (A//MS 04)
1709. RegardingCagallbladder- (PG/02) a) Open cholecystectomy
a) Squamous cell ca is the most common b) Laparoscopic cholesystectomy
b) Presentwithjaundice c) No surgery for gall bladder stones
c) Good prognosis d) ERCP- aridremoval of gall bladder stones
d) Gallstones predispose
1717. A retained stone impacted in distal common bile
e) 65% survival after surgery
duct is seen on T- tube cholangiogram. What is
1710. On 7tb postoperative day after laparoscopic
the best management of stone- (UPSC 04)
cholecystectomy, pt. developed rt. upper abdominal
pain and 10 em X 8 em collection. Treatment consists a) Dissolution therapy
of- (PG/03) b) Operativeremoval
a) Immediate laparotomy c) Endoscopic sphincterotomy and stone extraction
b) Percutaneous drainage d) No active treatment is required
c) Laparotomy & surgical exploration of bile duct 1718. Clinical features of choledochal cyst in adult are-
and T tube insetion a) Pain, lump and intermittent jaundice (UPSC 04)
d) Laparoscopic cystic duct ligation and b) Pain, fever and intermittent jaundice
percutaneous drain c) Pain, lump and progressive jaundice
e) Roux-en-Y loop hepatojejunostomy d) Pain, fever and progressive jaundice
1711. Charcot's triad consists of- (PGI 03) 1719. The ideal treatment of stenosis of sphincter of Oddi
a) Biliary colic b) Jaundice is - (SGPGI 04)
c) fever with rigor d) Hypotension a) Transduodenal sphincteroplasty
e) Pancreatitis b) Endoscopic sphincterotomy
1712. Bile ductopenia seen in- (PG/03) c) Choledochojejunostomy
a) GVIID b) Alcoholic hepatitis d) Choledochoduodenostrny
c) Autoimmune hepatitis d) Cirrhosis 1720. Laproscopic cholecystectomy is largely preferred
e) Sclerosing cholangitis for all of the following reasons to conventional
1713. On 5th postoperative day after laparoscopic laparotmy except- (SGPGI 04)
cholecystectomy, a 50 years old lady presented with a) Decrease pain
rt. upper quadrant pain with fever and 12 em b) Decreased incidence of bile duct injuries
subhepatic collection on CT and ERCP shows cystic
c) Smaller scar
duct leak. The best management is- (PGI 03)
d) Decreased stay in hospital
a) Immediate laparotomy
1721. "Limey bile" is- (Karnataka 04)
b) Percutaneous drainage of fluid
c) Leparatomy and surgical exploration of bile duct a) Present in the CBD
and T-tube insertion b) Thin and clear
d) Laparoscopic cystic duct ligation & percutaneous c) Like toothpaste emulsion in the gall bladder
drain d) Bacteria rich
e) Roux en loop hepatojejunostomy 1722. The treatment of choice for a mucocele of
1714. Vanishing bile duct syndrome is seen in- (PG/ 03) gallbladder is- (AIIMS 04)
a) Chronic viral hepatitis a) Aspiration of mucous
b) Sarcoidosis b) Cholecystectomy
c) Lymphoma c) Cholecystostomy
d) Non-cirrhotic portal fibrosis d) Antibiotics and observation
e) Alcoholism 1723. A 30-year old lady is found to have gall stones. She
1715. A patient presenting with HJO diarrhea for several is asymptomatic and has never had any jaundice or
years with recent onset pruri tus & t Alkalline dyspeptic symptoms is the past. The best course of
Phosphatase, normal SGOT/PT & abnormality; the management for her would be- (UPSC 05)
diagnosis is- (PGI 04) a) Dissolution therapy
a) Hodgkins Lymphoma b) Sclerosing cholangitis b) Extra corporeal lithotripsy
c) Chronic Hepatitis d) Autoimmune Hepatitis c) Cholecystectomy
e) virald Hepatitis d) To wait till she becomes symptomatic

1708)a 1709)b,d 1710)b,d,e 1711)a,b,c 1712)a 1713)b,d 1714)b 1715)b 1716)c 1717)c 1718)c 1719)a
1720)b 172l)c 1722)b 1723)d
SURGERY [ 263]

1724. Which of the following factors in bile juice is 1731. All ofthe following are true for patients of ulcerative
responsible for preventing the precipitation of colitis associated with primary sclerosing
cholesterol and formation of gallstones- (ICS 05) cholangitis (PSC), except- (AI 05)
a) High alkaline condition a) They may develop biliary cirrhosis
b) High concentration of bicarbonates b) May have raised alkaline phosphatase
c) Bile salts c) Increased risk ofhilar Cholangiocarcinoma
d) Bile pigments d) PSC reverts after a total colectomy
1725. True about primary sclerosing cholangitis are AlE- 1732. The initial investigation of choice for a post
a) Fibrosing cholangitis ofbile duct (PGI June 05) cholecystectomy biliary stricture is-(AIIMS May 05)
a) Ultrasound scan of theabdomen
b) Periductal onion skin appearance
b) Endoscopic cholangiography
c) Cirrhosis never occurs
c) Computed tomography
d) Jaundice seen
d) Magnetic resonance cholangiography
e) A/W ulcerative colitis
1733. The treatment of choice for an 8 mm retained
1726. Factors a/w gall bladder Ca- (PGI June 05) common bile duct (CBD) stone is- (AIIMSMay 2005)
a) Chronic cholesteral stone b) Hyperlipidemia a) Laparoscopic CBD exploration
c) Chronic gall bladder disease d) Hepatitis b) Percutaneous stone extraction
e) Porcelain gall bladder c) Endoscopic stone extraction
1727. A 50 year old lady with history of jaundice in the d) Extracorporeal shock wave lithotripsy
past has presented with right upper quadrant 1734. Which of the following is not an indication for
abdominal pain. Examination and investigations cholecystectomy- (AIIMS May 2005)
reveal chronic calculous cholecystitis. The liver a) 70-year old male with symptomatic gallstones
function tests are within normal lilmits and no b) 20-year old male with sickle cell anaemia and
ultrasound examination the common bile duct is not symptomatic gallstones
dilated. Which of the following will be the procedure c) 65-year old female with a large gallbladder polyp
of choice in her- d) 55-year old with an asymptomatic gallstone
a) Laparoscopic cholecystectomy (J & K 05) 1735. A patient of post-cholecystectomy biliary stricture
b) Open choledocholithotomy with CBD exploration has undergone an ERCP three days ago. Following
c) ERCP ±cholecystectomy followed by laparoscopic this she has developed acute cholangitis. The most
cholecystectomy likely organism is- (AI 06)
d) Laparoscopic cholecystectomy followed by ERCP a) Escherichia colli
± choledocholithotomy b) Bacillus fragilis
1728. Most common type ofcancer gall bladder in a patient c) Streptococcus viridans
with gallstones- (APPGE 05)
d) Pseudomonas aerogenosa
1736. Which one of the following statementds incorrect
a) Adenocarcinoma b) Squamous carcinoma
regarding stones in the common bile duct ? (AI 06)
c) Adenocarcinomad) None
a) Can present with Charcot's triad
1729. Regarding bile duct injuries following
b) Are suggested by a bile duct diameter > 6 mm of
Cholecystectomy which ofthe following statement ultrasound
is false- (AIIMSNov 05) c) ERCP, sphincterotomy and balloon clearance is
a) The incidence following open cholecystectomy now the standard treatment
is in the range of0.2- 0.3% d) When Removed by exploration of the common
b) The incidence rate following Laparoscopic bile ducts the T-tube can be removed after 3 days
Cholecystectomy is three times higher than the 1737. Biliary stricture developing after laparascopic
rates following Open cholecystectomy choice-cystectomy usually occurs at which part of
c) Untreated cases may develop secondary biliary the common bile duct- (AI 06)
cirrhosis a) Upper b) Middle
d) Routine use of 'open' technique of laparoscopic c) Lower d) All side with equal frequency
port insertion has resulted in a decline in the 1738. Absolute indication for choledochotomy -(PGIJune
incidence of postlaparoscopic cholecystectomy a) Gallstone ileus b) Gallstone pancreatitis 06)
bile duct injuries c) Fever d) Jaundice
1730. Acalculous cholecystitis can he seen in all the e) Palpable CBD stone
following conditions except- (AIIMS Nov 05) 1739. Which of the following are causes for cholecystitis
a) Enteric fever b) Dengue haemorrhagic fever except? (APPG 06)
c) Leptospirosis d) Malaria a) Estrogen b) OCP
c) Diabetesmellitus d) Obesity

1724) c 1725) c 1726) a,c,e 1727) d 1728) a 1729) d 1730) d 1731) d 1732) c 1733) c 1734) d 1735) a 1736) d
1737) a 1738) e 1739) c
SURGERY [ 264]

1740. Sentinel node of gall bladder is- (Manipal 06) 1750. A 45 year old female presents with symptoms of acute
a) Virchow's nodes b) Iris nodes cholecystisis. On USG there is a solitary gallstone
c) Clouquet node d) Lymphnode oflund ofsize 1.5 em. Symptoms are controlled with medical
1741. The procedure of choice for elective removal ofCBD management Which of the following is the next most
stones for most patients is- (Karn 06) appropriate step in the management of this patient-
a) Open choledocholithotomy a) Regular follow up (AI 08)
b) Endoscopic choledocholithotomy b) IV Antibiotics
c) Laparoscopic choledocholithotomy c) Laparoscopy cholecystectomy immediately
d) Percutaneous choledocholithotomy d) Open cholecystectomy immediately
1742. The predominant constituent of the pale yellow 1751. Most common type of choledochal cyst? (APPG 08)
gall stones in the gall bladder is- (Comed 07) a) Type 1 b) Type 2
a) Mucin glycoprotein b) Calcium carbonate c) Type 3 d) Type 6
c) Cholesterol d) Calcium phosphate 1752. Which is nottrue about PBC? (APPQ08)
17 43. Haemobilia is characterized by all except-(Comed 07) a) No increase in risk of hepatocellular carcinoma
b) Often asyptomatic
a) Jaundice b)Biliarycolic
c) Elevated IgM
c) Melaena d)Fever
d) Positive antimitochondrial antibody
1744. The Reynold's pentad of fever, jaundice, right
1753. Bile strictures are seen in- (PGI Dec 08)
upper quadrant pain, septic shock and mental a) Cholelithiasis b) Malignancy
status change iu typical of- (Comed 08) c) Post operative d) Worm infestation
a) Cholangitis b) Hepatitis e) Acute pancreatitis
c) Cholecystitis d) Pancreatitis 1754. In a male after laparoscopic cholecystectomy,
1745. In a survey, many children are examined and were specimen is sent for histopathology which shows
found to have urogenital abnormalities. Which carcinoma gallbladder stage lb. Appropriate
congenital anomaly is associated with increased risk management is- (AIIMS Nov 08)
ofbladdercarcinoma? (AIIMS Nov 07) a) Conservative and follow up.
a) Medullary sponge kidney b) Bladder exstrophy b) Extended cholecystectomy
c) Unilateral renal agenesis d) Double ureter c) Excision of all port sites
1746. Common buleductinjuriesaremostcommonlyseenin- d) Radiotherapy
a) Radical gastrectomy (UPSC-II 08) 1755. Most common site of cholangiocarcinoma?
b) Penetrating injuries of abdomen a) Distal biliary duct b) Hilum (AIIMS Nov 08)
c) ERCP & sphincterotomy c) Intrahepatic duct d) Multifocal
d) Laparoscopic cholecystectomy operation 1756. Risk factors for malignant change in an
1747. A 88 years male patients presented with end stage asymptomatic patient with a gall bladder polyp on
renal disease with coronary artery block and ultrasound include all of the following, Except-
metastasis in the lungs. Now presents with acute a) Age> 60 years (AI 09)
cholecystitis, patients relatives needs treatments to b) Rapid increase in size of polyp
do something- (UP 08) c) Size of polyp> 5 mm
d) Associated Gall stones
a) Open cholecystectomy
1757. Which of the following statements about acalculous
b) Tube cholecystostomy
cholecystitis is incorrect? (DELHIPGMar. 09)
c) Laproscopic cholecystectomy
a) Manifestation of disturbed microcirculation in
d) Antibiotics then elective cholecystectomy critically ill patient
1748. A 60 year old females presented with hematuria and b) Prolonged parenteral nutrition can be causative
diagnosed transitional cell carcinoma of bladder c) It is life threatening condition
stage T 1• N1M0• Best treatment modalities is- d) Cholecystectomy is not indicated
a) Transurethral resection (UP 08) 1758. Which of the following is not a component of
b) Transurethral resection and intravesical chemo- Reynolds' Pentad in toxic cholangitis?
immunotherapy a) Right upper quadrant pain (DELHI PG Mar. 09)
c) Total cystectomy and pelvic lymphadenectomy b) Confusion
d) Systemic chemotherapy c) Septic shock
174 9. In cholecystecomy, fresh plasma should be given - d) Markedly elevated transaminases
a) Just before operation (UP 08) 1759. Saccular diverticulum of extrahepatic bile duct in
b) At the time of operation choledochal cyst is classified as- (COMED 09)
c) 6 hours before operation a) Type I b) Type II
d) 12 hour after operation c) Type III d) Type IV

1740)d 1741)b 1742)c 1743)d 1744)a 1745)b 1746)d 1747)b 1748)b 1749)a 1750)c 1751)a 1752)a
1753)a,b,c,d l754)c 1755)b 1756)c 1757)d 1758)d 1759)b
SURGERY [ 265]

1760. In the management of symptomatic benign prostatic 1769. True about Biliary cystic adenocarcinoma-
hyperplasia with finesteride the period of trial a) CT scan is used for diagnosis (PGI Nov 09)
required for determining a satisfactory response is- b) CA 19-9 is elevated
a) I Month b) 2 Months (COMED 09) c) Intrahepatic location
c) 4 Months d) 6 Months d) Extrahepatic location
1761. The most common complication of transurethral e) Cornmoninmale
resection of the prostate is- (COMED 09) 1770. True about cystic duct stump stone are AlE-
a) Erectile dysfunction a) Stone cause of postop pain {PGI Nov 09)
b) Retrograde ejaculation b) Re-cholecystectomy is the definite treatment of
c) Urinary incontinence choice
c) ERCP is the investigation of choice to diagnose
d) Urethral stricture disease
d) Basket extraction is the Mx of choice
1762. Which of the following is NOT associated with
e) Oral Ursodeoxycholic acid relieves symptoms
elevation of prostatic specific antigen -(COMED 09)
remarkably
a) Acute prostatitis 1771. Risk factor for cholangiocarcinoma all except-
b)BPH a) Chronictyphoidcarrier (AIIMSNov09)
c) Prostatic intraepithelial neoplasm b) Chronic ulcerative colitis
d) Prostatic needle biopsy
c) Parasitic infestation
1763. Which one of the following is NOT a common fea~e d) Choledocholithiasis
ofbile duct stone- (UPSC-II 09) 1772. Laparoscopic cholecystectomy done in a patient with
a) Obstructive jaundice
cholelithiasis. Pathology report shows
b) Itching
adenocarcinoma with invasion of muscular layer.
c) Clay colored stools
CT was normal. Further t/t is- (AIIMS Nov 09)
d) Distended gallbladder
a) Wait and regular follow up
1764. A young patient develops high grade fever with chills b) Wedge hepatic resection with lymph node
and rigors, mild jaundice and acute pain in the upper
dissection
abdomen following cholecystectomy. On examination, c) Excise all port sites
she was jaundiced, toxic, haemodynamically stable d) Radiotherapy '
and having vagne fullness upper abdomen. What is
1773. A patient with ITP has a plalelet count of50,000 and
the most probable diagnosis- (UPSC-11 09) is being planned for splenectomy. What is the best
a) Localised collection ofbile in peritoneal cavity
time for plateletinfusion in this patient- (AI 10)
b) Iatrogenic ligation of common bile duct
a) 2 hours before surgery
c) Duodenal injury b) At the time of skin incision
d) Acute pancreatitis c) After ligating the splenic artery
1765. The following are complications of gallstone d) Inunediately after removal of spleen
except- (UPSC-II09) 1774. All the following increase risk for
a) Hemobilia b) Cholangitis
cholangiocarcinomaexcept- (DPG 10)
c) Biliary enteric fistula d) Acute pancreatitis
a) Ulcerative colitis b) Gall stones in CBD
1766. Consider the following- (UPSC-11 09) c) Sclerosing cholangitis d) Chlonorchis
1. Cholesterosis 1775. An ultrasound examination shows dilated
2. Adenomyomatosis
intrahepatic biliary channels with a small gall
3. Polyposis bladder. The most likely possibility is- {DPG 10)
4. Cholelithiasis
a) Gall bladder stone
To which of the above does cholecystoses refer to:
b) Pancreatic calculus
a) 1,2and3 b)Iand3only c) Common bile duct stone
c) 2, 3 and4 d) 1 and 3 only d) Carcinoma of the head of the pancreas
1767. A 50-year-old diabetic patient with asymptomatic
1776. 'Strawberry gall bladder' is seen in-
gallstone (>3 em) will be best treated by- a) Cholesterosis (Maharashtra 10)
a) Early surgery (UPSC-11 09) b) Porcelain gall bladder
b) Bile salt treatment
c) Necrosis of gall bladder
c) ESWL d) Gall bladder carcinoma
d) Waiting till it becomes symptomatic 1777. Primary sclerosing cholangitis is associated with-
1768. Rigler's triad includes- (PGI Nov 09) a) Ulcerative cholangitis (Maharashtra 10)
a) Pheumobilia b) Ectopic stone
b) Retroperitoneal fibrosis
c) Cholangitis d) Intestinal obstruction c) Reidel's thyroiditis
e) Biliary stenosis
d) All of the above

1760)d 176l)b 1762)d 1763)d 1764)b 1765)a 1766)a 1767)a 1768)a,b,d 1769)All 1770)b,e 1771)d 1772)b
1773)c 1774)b 1775)c 1776)a 1777)d
SURGERY [ 266]

1778. Pneumobilia can be seen in- (UP SC II 10) 1789. The following constitute the Saint's triad except-
a) Mirizzi's syndrome b) Acute pancreatitis a) Gall stones b) Jaundice (UPSC II 11)
c) Gallstone ileus d) Carcinoma gallbladder Hiatus hernia Colonic diverticulosis
1779. Which one of the following is not a component of
Charcot's triad? (UP SC II 10)
a) Pain b) Fever
c) Vomiting d) Jaundice
1780. T/t of CBD stone includes- (PGI May 10)
a) Endoscopic papillotomy b) ERCP
c) Ursodeoxy cholic acid d) Hepatojejunostomy
e) Choledochotomy
1781. Not included in Charcot's triad? (AIIMS Nov 10)
a) Fever b)Pain
c) Jaundice d) Gallbladder lump
1782. Organism associated with fish consumption and also
causes carcinoma gallbladder- (AJIMSNov 10)
a) Gnathostoma
b) Anglostrongyloidosis cantonensis
c) Clonorchis sinensis
d) H. dimunata
1783. On abdominal ultrasound gall bladder shows diffuse
wall thickening with hyperechoic nodules at neck
and comet tail artifacts. The most likely diagnosis
will be-
a) Adenomyomatosis
b) Adenocarcinoma of gall bladder
c) Xanthogranulomatous cholecystitis
d) Chelesterol crystals PANCREAS
1784. Risk factors for malignant change in an
asymptomatic patient with a gall bladder polyp on 1795. Which one is not Ranson's prognostic criteria in
ultrasound include all ofthe following, except - acute pancreatitis- (AI 88)
a) Age> 60 years (AIIMS May 11) a) Age over 55 years
b) Rapid increase in size of polyp b) Blood glucose more than 200 mgs %
c) Size ofpolyp> 5 mm c) WBC more than 16000/mm3
d) Associated Gall stones d) Serum calcium more than 8 mg%
1785. Most common site of cholangiocarcinoma- (AJIMS 1796. The treatment of annular pancreas is {AliMS 85)
a) Distal biliary duct b) Hilum May 11) a) Gastrojejunostomy
c) Intrahepatic duct d) Multifocal b) Duodeno duodenostomy
1786. The technique oflaproscopic cholecystectomy was c) Duodenojejunostomy
first described by? (AJIMS May 11) d) Gastroduodenostomy
a) ErichMuhe b)PhillipMoure 1797. Most common cause of acute pancreatitis
c) KurtSemm d)EddieReddick is - (JIPMER 87)
1787. Survivalin unresectable GB ca is? (AJIMS May 11) a) Biliary tract diseases b) Alcoholism
a) 4-6 months b) 8-10 months c) Post traumatic d) Idiopathic
c) 1 yr d) 12-24 months 1798. Treatment of pseudocyst of pancreas
1788. Regarding laproscopic cholecystectomy, which of include- (JIPMER 87)
the following statements is correct- (UPSCII 11) a) Cystogastrostomy b) Cystojejunostomy
a) It is primarily done for cholecystitis in the third c) Cystocolostomy d) Excision of the cyst
trimester of pregnancy e) All of the above
b) It is associated with higher rate ofbile duct injuries 1799. Whipple's triad in insulinoma includes-
than open cholecystectomy a) Hypoglycemia below 45 mg% (JIPMER 86)
c) It is contraindjcated in acute cholecystitis b) An attack of hypoglycemia in the fasting stage
d) It is safer than open cholecystectomy in patients c) Symptoms relieved by glucose
with cardiorespiratory disease d) all of the above

1778) c 1779) c 1780) a,b,c,e 1781) d 1782) c 1783) a 1784) c 1785) b 1786) a 1787) a 1788) b 1789) b 1790) d
1791)d 1792)d 1793)c 1794)d 1795)d 1796)b,c1797)a 1798)a,b,d 1799)d
SURGERY [ 267]

1800. Most common cause of pancreatic pseudocyst 1810. Serious complication in pancreatic pseudocyst
formation is- {AIIMS 86, 87) include all ofthe following except- (UPSC 97)
a) Trauma b) Pancreatitis a) Intracystic haemorrhage
c) Neoplastic d) None of the above b) Secondary infection
1801. Pain reliefin chronic pancreatitis can be obtained c) Calcification in the cyst wall
by destruction of- (UPSC 87, 88) d) Rupture of the cyst
a) Celiac ganglia 1811. All are true about pseudopancreatic cyst except-
a) Common after acute pancreatitis - {AI 98)
b) Vagus nerve
b) Present as adbomen mass
c) Anterolateral column of spinal cord
c) Increased serum amylase
d) None of the above d) Mostly present in head of pancreas
1802. Which of the following about acute pancreatitis 1812. A chronic alcoholic with epigastric pain was
is true- {PGI 88) submitted to ERCP which showed chain oflakes'
a) Pain last for several hours appearnace of main pancreatic duct treatment of
b) Serum amylase levels correlates with severity of choice is - (MAHE 98)
attack a) Lateral pancreaticojejunostomy
c) Common in alcoholics b) Distal pancreaticojejunostomy
d) Low serum calcium levels indicate good prognosis c) pancreatectomy
1803. The complication least likely to occur in a d) Proximal pancreaticojejunostomy
pseudocyst ofthe pancreas is- {Kerala 90) 1813. In Whipple's operation all are removed except-
a) Hemorrhage b) Torsion a) CBD b) Portal vein {AIIMS 98)
c) Infection d) Carcinomatous change c) Head of pancreas d) Duodenum
1804. Commonest type of pancreatic carcinoma is- 1814. Most sentitive investigation of pancreatic carcinoma
a) Acinarcellcarcinoma {Al92) is- ~m~
a) Angiography b)ERCP
b) Ductal adenocarcinoma
c) Ultrasound d) CT scan
c) Islet cell carcinoma
1815. Whipples triad is seen in- {PGI 97)
d) Mucinous carcinoma a) Islet cell tumour b) Ca head pancreas
1805. Treatment of congenital cyst of head of c) Argentaffmoma d) Cholangiocarcinoma
pancreas- {PGI 83, AMC 86) 1816. Inverted "3" sign seen in- {PGI 97)
a) Total excision a) Ampullary Ca b) Insulinoma
b) Partial excision c) Ca. head pancreas d) Ca. stomach
c) Marsupialization 1817. Treatment of annular pancreas is- {PGI 97)
d) Observe and medical treatment a) Excision of head of pancreas
1806. In Mucoviscidosis of the pancreas the commonest b) Duodenojejunostomy
defect is in the- {JIPMER 80, AIIMS 89) c) Excision of tail of pancreas
a) Jejunum b)ileum d) Any of the above
c) Ascending d) Descending colon 1818. Treatment ofpseudopancreatic cystis- {PGI97)
1807. Leukocyticinfiltraton in islet cells ofpancrease a) Endoscopic drainage
is characterstically seen in some cases of- b) Percutaneous drainage
a) Juvenile diabetes {PGI 81, AMU 87) c) Internal drainage (surgical)
d) Pancreatectomy
b) Diabetic ketosis
1819. Which is not a feature of pancreatic ascites-
c) Systemic mucoviscidosis
a) Lowprotein {PGI99)
d) Haemorrhage pancreatic necrosis b) Somatostatin is the drug of choice
1808. All are seen in chronic calcific pancreatitis except c) Communication with pancreatic duct in 80%
a) Diabetis mellitus {Kerala 96) d) Raised amylase levels
b) Fat malabsorption 1820. True about pancreatic carcinoma- {PGI 2000)
c) Hypercalaemia a) Head is the most common site
d) Recurrent adbominal pain b) Pain is the most common symptom
e) lncreasd incidence of pancreatic carcinoma c) Obstruction ofbile & pancreatic secretion is common
1809. All are features ofpesudopancreatic cyst, except d) 80% cases respond well to resection
a) Follows acute pancreatitis {AI 97) 1821. Investigation to diagnose carcinoma head of
b) Lined by false epithelium pancrease are/is- {PGI 2K)
c) May regress spontaneously a) Hypotonic duodenogram b) X-ray abdomen
d) Treatment of choice is percutaneous aspiration c) lJS(} d) Endoscopy
e) C.T. Scan

1800)b 1801)a 1802)a,c 1803)d 1804)b 1805)c 1806)b 1807)a 1808)c 1809)d 1810)c 1811)d 1812)a 1813)b
1814)b 1815)a 1816)c 1817)b 1818)All 1819)a 1820)a,c 1821)a,c,d,e
SURGERY[268]

1822. Common cause (s) of acute pancreatitis is/are- 1834. Investigation done inCa pancreas are- (PGI 03)
a) Alcohol b) Gall stone (PGI 2K) a) Hypotonic duodenography b) X-ray abdomen
c) Primary billiary cirrhosis d) L-asparginase c) Endoscopy d) C.T. Scan
1823. True about acute pancreatitis ale (PGI 2K) e) Ba-fallowthrough
a) Gallstones & alcohol are common 1835. True about acute pancreatitis- (PGI 04)
b) Sr. amylase levels increase after 72 hrs. of attack· a) Alcohol is the MC cause
c) Can cause secondary pleural effusion b) 20-25% needs surgical treatment
d) Can present with shock c) CECT is preffered over USG in diagnosis
1824. Which of the following is the most common site for d) Serum Amylase is diagnostic
the carcinoma of pancreas- (PGI 2000) e) Grey Turner sign seen around the umpilicus
a) Head b) Ampulla 1836. The following conditions are indicative of surgery
c) Body d) Tail in acute pancreatitis- (UPSC 04)
1825. Commonest complication of pancreatic a) Acute fluid collection
pseudocyst (PGI 2000, AJIMS 84) b) Persistent pseudocyst pancreas
a) Rupture b) Infection c) Pancreatic abscess
c) Pressure on viscera d) Hemorrhage d) Infective pancreatic necrosis
1826. Qlldstandardtestininsulinoma- (PGI 2000) 1837. Increased amylase levels in pleural fluid are seen
a) CT scan b) Ultrasound in - (BIHAR 03)
c) MRI d) Arteriography a) Malignancy b) Pancreatitis
1827. Trueaboutpseudocystofpancreas- (PG/01) c) Oesophageal rupture d)All
a) Is a true cyst 1838. Major compHcation ofcysto gastrostomy for pseudo
b) Common with penetrating trauma abdomen pancreatic cyst is- (COMEDK 05)
c) Treated with cystogastrostomy if size> 2 em a) Infection b) Obstruction
d) Can present from mediastinum to pubis c) Fistula d) Haemorrhage
e) Contains a wall of fibrous granulation tissue 1839. All are features seen in Chronic Pancreatitis except-
1828. True aboutZES (gastrinoma)- (PGI OJ) a) Chronic persistent pain (HPU 05)
a) Gastrin levels> 1000 pg/ml b) Diarrhoea, Steatorrhoea
b) BAO (Basal acid output) > 15 meq/hr c) Calcification
c) Somatostatin is inhibitor ofHCL secretion d) Paralytic ileus
d) Omeprazole is helpful 1840. The preferred bypass procedure in case of non
e) Secretin increases gastrin secretion in Zollinger resectable carcinoma ofhead of pancreas is-
Ellison Syndrome a) Cholecystojejunostomy (MAHE 05)
1829. True about pancreatic Ca- (PG/02) b) Cholecystogastrostomy
a) M.C. symptom is pain c) Choledochoduodenostomy
b) Ca head of pancreas causes obstruction of both d) Choledochojejunostomy
pancreatic and billiary duct 1841. Gastrinomaa!w- (PG!June05)
c) Smoking predisposes a) Diarrhoea
d) Family H/0 familial adenomatous polyposis b) Constipation
syndrome c) Achlorhydria
1830. Annular pancreas true about- (PG/02) d) Multiple superficial gastric ulcers
a) Complete or partial rotation failure e) Gastric atrophy
b) Congenital 1842. Insulinoma is most commonly located in which part
c) Normal histology of the pacreas- (MAHA 05)
d) Interstromal cell proliforation a) Head b) Body
e) Surgical correction should be done c) Tail d) Equally distributed
1831. True increase in islet cell- (PGI 02) 1843. Pancreatitis that has good prognosis- (APPGE 05)
a) Nesdioblastoma b)TypeiiDM a) Gallstone pancreatitis
c) Insulinoma b) Post operative pancreatitis
1832. Pancreatitis caused by- (PG/02) c) Alcoholic pancreatitis
a) Ascending chlongitis b) ledPTH d) Chronic pancreatitis
c) CBD stone d) Alcohol 1844. All of the following are features of Zollinger Ellison
e) Starvation syndrome except- (AIIMSNov 05)
1833. Acute pancreatitis is caused by- (PG/02) a) Intractable peptic ulcers
a) Gallstones b) Alcohol b) Severe diarrhoea
c) Starvation d) Hypercalcemia c) Beta cell tumours of the pancreas
e) Hypertriglyceridemia d) Very high acid output

1822)a,b 1823)b 1824)a 1825)b 1826)b 1827)d,e 1828)All 1829)b,c 1830)a,b,c,e 183l)a,c 1832)b,c,d
1833)a,b,d,e 1834)a,c,d 1835)c 1836)b,d 1837)d 1838)d 1839)d 1840)a 1841) a 1842)d 1843)a 1844)c
SURGERY [ 269 ]

1845. All the following can be used to predict severe acute 1856. Which of the following is the most common non-
pancreatitis except- (AIIMS Nov 05) alcholic cause of acute pancreatitis- (Corned 07)
a) Glasgow score ~ 3 a) Thiazides b) Hypercalcaemia
b) APACHE II score~ 9 c) Hyperlipidaernia d) Gall stones
c) CT severity score~ 6 1857. Which of the following organs is the most common
d) C - reactive protein < 100 site of origin of the tumour associated with the
1846. True in Pancreas trauma- (PGI June 06) Zollinger-Ellison syndrome- (Corned 07)
a) Solitary involvement common a) Duodenum b) Lymph nodes
b) Blunt injury usual cause c) Spleen d) Pancreas
c) Always surgery needed 1858. The commonest cause of acute pancreatitis is-
d) Amylase i in 90% cases a) Biliary calculi b) Alcohol abuse (Corned 07)
e) HRCT is investigation of choice c) Infective d) Idiopathic
1847. Commonly seen pancreatic endocrinal 1859. Grey Turner's sign (flank discoloration) is seen
tumour- (PGI June 06) in- (Corned 08)
a) lnsulinoma b) Somatostatinoma a) Acute pyelonephritis b) Acute cholecystitis
c) Gastrinoma d) Glucagonoma c) Acute pancreatitis d) Acute peritonitis
e) Vipoma 1860. Which one of the following is the management of
1848. What is the most common surgical complication pancreatic abscess- (UPSC-II 08)
following whipple procedure? (UPSC 07) a) Needle aspiration b) Gastro-cystostomy
a) Disruption of pancreatic anastomosis c) External drainage d) Jejuno-cystostomy
b) Biliary peritonitis 1861. Carcinoma of pancreas associated with- (UP 08)
c) Disruption of gastric anastomosis a) Hypoglycemia
d) GI bleeding b) Syndrome of inappropriate secretion of ADH
1849. Vascular complications of acute pancreatitis include c) Erythropoisis is due to erythropoietin
the following except- (UPSC 07) d) Hypercalcemia
a) Splenic vein thrombosis 1862. A chronic alcoholic presents with abdominal pain
b) Splenic artery aneurysm radiating to the back that responds to analgesics. At
c) Gastroduodenal artery aneurysm evaluation the pancreatic duct was found to be dilated
d) Middle colic artery thrombosis and stones were noted in the tail of pancreas. The
1850. Most common complication after ERCP is- most appropriate management is- (AI 08)
a) Acute Pancreatitis (Aiirns May 07) a) Pancreatic tail resection
b) Acute cholangitis b) Pancreatico jejunostomy
c) Acute cholecystitis c) Percutaneous removal of stone
d) Duodenal perforation d) Medical management
1851. Severity of acute pancreatitis correlate with levels 1863. A chronic alcoholic presents with repeated episodes
of all ofthe following except- (Aiirns May 07) of severe intractable abdominal pain. At evaluation
a) Glucose b) Amylase the pancreatic duct was found to be dilated and stones
c) Transaminase d) Calcium were noted in the tail of pancreas. The most
1852. Most common tumour of pancreas is- (UP07, appropriate managementis- (AI 08)
a) Adeno-carcinoma TN 89) a) Pancreatic tail resection
b) Squamous cell carcinoma b) Pancreatico Jejunostomy
c) Adeno-squamous cell carcinoma c) Percutaneous removal of stone
d) Duct cell adeno carcinoma d) Medical management
1853. Best prognosis in carcinoma of pancreas in the 1864. Zollinger Ellison syndrome true about AlE-
region of- (UP 07) a) Surgery is to be done (PGI June 08)
a) Head b) Tail b) Exocrine tumour
c) Body d) Periampullary c) Endocrine disorder
1854. "Chain oflakes" appearance seen in- (UP 07, 05) d) Secretory diarrhoea seen
a) Acute pancreatitis e) Metastasis seen
b) Chronic pancreatitis 1865. Which ofthe following statements about Pancreatic
c) Carcinoma pancreas Carcinoma is not true- (A/09)
d) Strawberry gall bladder a) Mutation in P53 gene is associated in 75% of cases
1855. Cullen's sign - (UP07) b) Hereditary Pancreatitis significantly increases the risk
a) Bluish disclolouration of the flanks c) Median survival in locally advanced (stage III)
b) Bluish discolouration in the umbilicus disease is 3 - 6 months
c) Migratory thrombophelebitis d) Five year survival after curative
d) Subcutaneous fat necrosis pancreaticoduodenectomy is 15 - 20%

1845)d 1846)d,el847)a 1848)c 1849)d 1850)a 1851) b 1852)d 1853)d 1854)b 1855)b 1856)d 1857)a 1858)a
1859)c 1860)c 186l)a 1862)d 1863)b 1864)b 1865)c
SURGERY [ 270]

1866. The advantage of bladder drainage over enteric 1876. Treatment of choice for annular pancrease is-
drainage after pancreatic transplantation is better a) Division ofpancreas (AI 10)
monitoring of- (AI 09) b) Duodenoduodenostomy
a) HBA 1C levels b) Amylase levels c) Duodenojejunostomy
c) Glucose levels d) Electrolyte levels d) Roux-en-Y loop
1867. Sister Mary Joseph nodule is most commonly seen 1877. A lady presented with recurrent attacks of giddiness
with- (AIIMSMay09)
and abdominal pain since three months. Endoscopy
a) Ovarian cancer b) Stomach cancer
was normal. Her fasting blood glucose was 40 mg
c) Colon cancer d) Pancreatic cancer
% and insulin levels were elevated. CT abdomen
1868. Commonest complication of Pseudocyst of the
pancreas is- (DELHI PG Feb. 09) showed a well defined 8 mm enhancing lesion is the
a) Commonest into peritoneum head of pancreas, with no other abnormal findings.
b) Rupture into colon What should be the treatment plan for this patient-
c) Hemorrhage a) Whipple's operation (AI 10)
d) Infection b) Enucleation
1869. Which of the following types of pancreatitis has the c) Enucleation with radiotherapy
best prognosis- (Manipal 09) d) Administration of streptozotocin
a) Alcoholic pancreatitis 1878. Gold standard testforforinsulinoma-
b) Gall stone pancreatitis a) 72 hr fasting test (AIIMS May 11)
c) Post operative pancreatitis b) Plasma insulin levels
d) Idiopathic pancreatitis c) C-peptide levels
1870. Consider the following prognostic parameters of d) Low glucose levels< 30 mg/dl
acute pancreatitis- (UPSC-II 09) 1879. Which of the following statements about Pancreatic
1. Rise in blood urea nitrogen over 5 mg/dl Carcinoma is not true? (AIIMS May 11)
2 Hematocrit decrease over l 0% a) Mutation in P53 gene is associated in 75% of cases
3. Base deficit more than 4 mmolllit b) Hereditary Pancreatitis significantly increases the
4. Blood glucose over 10 mmolllit risk
Which of the above parameters are important during c) Median survival in locally advanced (stage III)
initial48 hours ? disease is 3-6 months
a) 1 and 2 only b) 2 and 3 only d) Five year survical after curative
c) 1, 2 and3 d) 1, 3 and4 pancreaticoduodenectomy is 15-20%
1871. WhichofthefollowingisNOTanindicationforsurgical 1880. Not true about mucinous cystadenoma pancreas-
intervention in acute pancreatitis- (UPSC-II 09) a) Microcystic adenoma (AIIMS May 11)
a) Acute fluid collection b) Pancreatic necrosis b) Lined by columnar epithelium
c) Pancreatic abscess d) Diagnostic dilemma c) Pre malignant
1872. True about gastrinoma are all except- (PGI Nov 09) d) Focus of ovarian stroma in it
a) Located mostly in pancrease 1881. A lady present with three day history of epigastric
b) Unusallocated ulcer
pain radiating to back. Serum amylase levels were
c) 90% beningn
observed to be normal while USG abdomen reveals
d) Hypersecretion of gastric acid
gall bladder stones and an enlarged pancreas. CT
1873. Middle segment pancrectomy avoided in-
scan was done which clinched the diagnosis. Which
a) Cystadenoma (PGI Nov 09)
of the following is the most likely diagnosis -(AI 11)
b) Tumors of head of pancreas
a) Acute Cholecystitis b) Acute Pancreatitis
c) Tumors of tail of pancreas
c) Acute Appendicitis d) Acute Peritonitis
d) Tumor of neck of pancreas
1874. All are true for annular pancreas except- 1882. A fifty year old lady presents with two year history
a) Upper GI series is IOC (PGI Nov 09)
of recurrent abdominal pain with radiation to her
b) Duodenal obstruction present back. Pain is severe in intensity, and refractory to
c) ERCP is done simple analgesics. Ultrasound and Contrast
d) Non - rotation of gut Enhanced CT scan (CECT) confirmed the diagnosis
e) Treatment is division of ring and showed a dilated pancreatic duct. Which of the
1875. All are true about insulinoma except- following is the likely recommended surgical
a) Usually asymptomatic & need no treatment procedureofchoice? (AI 11)
b) Usually small & multiple (PGI Nov 09) a) Vagatomy withAntrectomy
c) Diazoxide & octreotide reduce insulin synthesis b) Vagotomy with Gastrolejunostomy
d) Most common site is pancreas c) Whipple's Procedure
e) 90% are Benign d) Longitudinal Pancreatico Jejunostomy

1866)b 1867)b 1868)d 1869)b 1870)d 1871)d 1872)a,c 1873)b,c1874)e 1875)a,b1876)c 1877)b 1878)a 1879)c
1880)a 188l)b 1882)d
SURGERY [ 271]

1883. Zollinger-Ellison syndrome is characterised by the 1895. Most common cause of generalised peritonitis is
following except- (UPSC I 11) a 40 year old adult male is- (AIIMS 9 2)
a) Profound gastric hypersecretion a) Enteric perforation
b) Large diarrhoea with occasional steatorrhoea b) Ruptured liver abscess
c) Hypocalcaemia c) Duodenal Ulcer perforation
d) Hvpergastrinaemia d) Perforated Ca stomach
1884. In acute pancreatitis, surgery is indicated in which 1896. Correct about subphrenic abscess is-
one of the following conditions- (UPSC II 11) a) Rarely chest symptoms (PGI 81,Delhi 86)
a) Acute fluid collection b) Toxaemia
b) Acute pseudocyst c) Rarellytoxaemia
c) Sterile pancreatic necrosis d) No sign and symptoms
d) Infected pancreatic necrosis 1897. Odourless peritoneal fluid is noticed in-
1885; Most common complication ofac!lt~ano cbrpnlc a) Perforatedpetic ulcer (AMC 81, AP 89)
, iTpancr¢atiti~ is:.- . . • .··.· . (NEET/DN]JC8atte'l:if)t, b) Perforated ileum
a) Portal:veiP:thr.ombosi$. Q).Panor~atie iibscess c) Perforated appendix
c) PstmdQCYSt · . . , . d)Fan<ifeatip:l1e~t)J.a8s d) T.B. Peritonitis
~8~~~, M,9,stcoiJJ.Wo~,~;~~f!s~9fJ!~()nie.pall~t:~titi~ - :. ~· ~ l' i 1898. The commonest cuase of acute mesentric adenitis
· ~) q~tH!g,nes ·...... <~>, :;.;(/;{J];E1Yfll'fB.:·P.<~ttern) is- (JIPMER81,AMU87)
. .·t b):'frgpic~ pancreatiti~. · · · a) Tuberculosis b) Brucellosis
' c),fan~J;.~1f.S divisil.lJil: ·•· · c) Pneumococcal infection d) Idiopathic
. < ylf)Alcohtif ·. . .
!< • • . , . • . •• ;:f:.,..• l" :, ...,:.~ 1899. Idiopathic retroperitoneal fibrosis involves first
1887~~~a,ttis,£om1Ilon .in b~fJl B:CU.t~T•f!jl ~~ro~ic of all- (AIIMS 90, DNB 90)
pan,creatitis? .· (flEETIJ)f.IB Eqttern) a) Stomach b) Duodenum
' ·~t'?~en~~pyst · 1>)!\~f!c~sfqtina,~o~ · c) Kidneys d) Ureter
ctD~C · ·· ······ _c!)~C§l~cJ.l§qri13SS
e) Urethra
PERITONEUM 1900. Which mesentric cyst whose removal entrails
removalsofpartofgut- (TN95)
1888. Commonest type ofmesentric cyst is- (JIPMER 87) a) Chylolymphatic cyst b) Enterogenous cyst
a) Enterogenous b) Chylolyrnphatic c) Dermoid d) All
c) Dermoid d) Urogenital remnant 1901. The most favoured treatment for a pelvic abscess
1889. Primary peritonitis with pneumococcus is in cul-de-sac is- (Delhi 96)
associated with- (UPSC 86, JIPMER 88) a) Laparotomy b) Colpotomy
a) Lymphomas b) Nephrotic syndrome c) External I & D d) Antibiotics
c) Carcinoids d) None of the above 1902. Intra-abdominal calcificaition in a plane X-ray
1890. Treatment of pouch of Douglas abscess is- abdomen is most often seen in- (Karn 9 5)
a) Laparotomy (UPSC 87, 88) a) Meconium ileus
b) Posterior colpotomy b) Meconium peritonitis
c) Antibiotics c) Meconium plug syndrome
d) Extraperitoneal drainage d) Necrotising enterocolitis
1891. Generalised diffuse peritonitis bas been compared 1903. Best investigation for air in peritoneal cavity is -
to second and third degree burns of- (AIIMS 84) a) USG b) Laparotomy ( CMC 98)
~13% ~30% c) Laparoscopy d) X-ray abdomen-erect view
c) 45% d)60% e) CT scan
1892. Early surgery is indicated in- (fGI 86) 1904. "Peritoneal mice" is- (AP 97)
a) Amoebiasis peritonitis b) Biliary peritomitis
a) Pseudomyxoma peritonel
c) Typhoid peritonitis d)All
b) Appendices epiploicae
1893. Which of the following is not characteristic of
c) Peritoneal seedings of tumour
visceral pain - (fGI 89)
a) Poor localisation b) Diffuse in nature d) Endometriosis
c) High threshold d) Very rapid adaptation 1905. The "Mesenteric cyst" - (AP 97)
1894. Most common site for intra abdominal abscess a) Moves parallel to the mysentery
following laparotomy is- (AIIMS 92) b) Moves peJ;Pendicular to the mesentery
a) Sub hepatic b) Subphrenic c) Is secondary tumour
c) Pelvic d) Paracolic d) Is fixed and immobile

1883)c 1884)d 1885)c 1886)d 1887)a 1888)b 1889)b 1890)b 1891)c l892)b,cl893)d 1894)a 1895)c 1896)b
1897)a 1898)d 1899)d 1900)b 1901)b 1902)b 1903)d 1904)b 1905)b
SURGERY [ 272]

1906. Ormond's disease is- (Kerala 98) 1917. Pseudochylous ascites occurs in -(PGI80, JIPMER 81)
a) Idiopathic lymphadenopathy a) Cirrhosis b) Hyperlipidemia
b) Retractile testis c) Filariasis d) Malignant ascites
c) Idiopathic retroperitonial fibrosis 1918. Localised idiopathic fibrosis is seen in all of the
d) Idiopathic mediastinitis following except- (UPSC 2001)
1907. Gas most suited for laproscopy is- (Rohtak 97) a) Riedel's struma b) Hypertrophic scar
a) Air b) Nitrogen c) Sclerosing cholangitis d) Panniculitis
c) CD d) Oxygen
1919. True about mesenteric cysts is- (PGI 97)
e) Carbon dioxide
a) Enterogenous cyst is commonest type
1908. Advantage of carbon dioxide in laproscopy are all
excpet- (Rohtak 97) b) Recurrence is common after enucleation
a) Non-irritant b) Non-inflammable c) Mensentric cyst are usually multiple
c) Minimally absorbed d) No tissue reaction d) Enucleation is the treatment of choice of
e) None chylolymphatic cyst
1909. Colopotomyisdonetotreat- (AP 98) 1920. In pneumoperitoneum following are seen except-
a) Ischeorectal abscess b) Pelvic abscess a) Hypertension b)Bradycardia (PGI97)
c) Appendicular abscess d) Perianal abscess c) Tachycardia d) Hypercapnia
1910. Which one of the following statements regarding 1921. Treatment of ruptured pneumo-peritoneum, as a
Meconium peritonitis is NOT correct- (UPSC 2K) result of colonoscopy in a young patient is -(PGI 98)
a) It is a septic peritonitis a) Temporary colostomy b) Closure+ lavage
b) It develops in later intra-uterine life or during or c) Permanent colostomy d) Symptomatic
just after delivery 1922. Meconium peritonitis occurs- (PGI99)
c) This condition should always be considered when a) Just before birth b) Just after birth
a baby is born with tense abdomen c) Before and after birth d) Due to birth trauma
d) Plain X-ray abdomen of this condition reveals
1923. Mucinous peritoneal fluid is found in- (PGI 2000)
calcification on liver and spleen
a) Carcinoma stomach b) Leimyosarcoma
1911. The following are true regarding ascites except-
a) Only when the amount of fluid (Kerala 2k)
c) Teratoma ovary d) Mucocele of appendix
present exceeds 1500 ml. It can be recognised 1924. Mucinous ascites is seen in - (PGI 2000)
clinically a) Stomach Ca b) TB
b) Shifting dullness is absent when there is a very c) Nephrotic syndrome d) Cirrhosis
large accumulation of fluid 1925. In which of the following conditions is air under
c) In Cirrhosis there is obstruction to the venous both sides of diaphragm visualized· (PGI 01)
outflow of the liver due to obliterrative fibrosis a) Perforated Meckel's diverticulum
of the intra hepatic venous bed b) Uterine rupture following illegal abortion
d) A transudate has a protein content of greater than c) Perforation of duodenal ulcer
30 gms. Ofprotein per litre d) Liver abscess
e) In Meig's syndrome it is associated with pleral e) Ruptured appendix
effusion and solid fibroma of ovary 1926. Peritonitis with air under both diaphragmatic domes,
1912. Mucinous peritoneal fluid- (PGI 2K) possibility is/are - (PGI 01)
a) Carcinoma stomach b) Leimyo sarcoma a) Duodenal perforation
c) Teratoma ovary d) Mucocele of appendix
b) Perforation of appendix
1913. Malignant change in lipoma is most common is-
c) Meckle's perforation
a) Thigh b) Nape of neck (2000 R)
c) Retroperitoneum d)Back d) Liver abscess rupture
1914. Thecommonestorganismseeninperitonitisis-(TN01) e) Uterine perforation (during procedure for abortion)
a) Escherichia coli b) Clostridium welchii 1927. Trueaboutpseudomyxoma peritonei- (SG PGI04)
c) Staphylococci d) Klebsiella a) Seen in male only
1915. Retractile mesentertis may be seen in-(Nimhans 86) b) Cytoreductive surgery needed
a) Ormonds's disease b) Gardner' syndrome c) Always appendicectomy needed
c) Turner's syndrome d) Down's syndrome d) Radiation therapy given
1916. Regarding primary peritonitis which of the e) Locally malignat tumor
following is correct- (JIPMER 88) 1928. Pseudomyxomaperitone- (PGJ04)
a) Caused by pneumococcus a) Males more common than females
b) Often oxygen b) Appendix should be removed in all cases
c) Steroids c) Chemotherapy is effective
d) No treatment is necessary d) Not malignant

1906)c 1907)e 1908)c 1909)b 1910)a 19ll)a,d 1912)a,d 1913)c 1914)a 1915)a 1916)a 1917)d 1918)b
1919)d 1920)b 1921)b 1922)c 1923)a 1924)a 1925)a,b,c,e 1926)a,c 1927)b,c,e 1928)b,c
SURGERY [ 273]

1929. When gas filled cysts are found in subserosa or 1939. Sub acute spontaneous peritonitis in cirrhosis
submucosa ofsmall intestine or colon, it is called- patients the polymorphonuclear cells are- (UP 08)
a) Pneumatosis cystoids intestinalis (AMU 05) a) More than 200 cells/cumm
b) Crohn's disease b) More than 300 cells/cumm
c) Ulcerative colitis c) More than 400 cells/cumm
d) Mesenteric cyst d) More than 500 cells/cumm
1930. Layers which are penetrated with trochar & cannula 1940. All are true about pseudomyxoma peritonei except-
in production of pneumoperitoneum are- a) Associated with ovarian tumours (DPGEE 08)
a) Skin & superficial fascia (PGI June 05) b) Appendicitis
c) Yellowjellycollectionoffluid
b) Deep fascia
d) Common in male
c) Rectus abdominis
1941. Most common site ofintraperitoneal abscess is-
d) Transversus abdominis
a) Right superior intraperitoneal space(DPGEE 08)
e) Rectus sheath
b) Right inferior intraperitoneal space
1931. Apart from Escherichia coli, the other most common c) Left superior intraperitoneal space
organism implicated in acute suppurative bacterial d) Left inferior intraperitoneal space
peritonitis is- (AI 06) 1942. Most common site of intra peritoneal abscess?
a) Bacteroides b) Klebsiella a) Morrison's pouch (APPG 08)
c) Peptostreptococcus d) Pseudomonas b) Omental bursa
1932. Commonest site from where pseudomyxoma c) Pelvic region
peritonei arise - (PGI June 06) d) Left subhepatic pouch
a) Ovary b) Appendix 1943. Pseudomyxoma peritonei arises from-
c) Pancreas d) Stomach a) Carcinoma ovary (PGI Dec 08)
e) Colon b) Ovarion cyst
1933. Commonest sites ofintraabdominal abscess- c) Ovarian dermoid
a) Subphrenic (PGI June 06, Jipmer 86, 87) d) Adenocarcinoma colon
b) Paracolic e) Mucocele of appendix
c) Pelvis 1944. Not associated with fat necrosis- (PGI June 09)
d) Retroperitoneal a) Liposuction b) Radiotherapy
e) Between loops c) Mammoplasty d) Carcinoma breast
1934. Which of the following complication is not seen with e) Following trauma
peritonitis - (Manipal 06) 1945. Infection ofall the following structures can be cause
a) Renal failure b) Residual abscess PsoasAbscessexcept- (DELHIPGMar. 09)
c) Endoxic shock d) Bone marrow suppression a) Vertebrae b) Appendix
1935. A 25 years old female presents with pyrexia for ten c) Hip joint d) Ribs
days, develops acute pain in periumblical region 1946. Which ofthefollowingstatements about preoperative
spreading all over the abdomen. What would be the optimization for perforation peritonitis is incorrect?
most likely cause? (UPSC 07) a) High technology intensive care unit environment
a) Perforation peritonitis due to intestinal tuberculosis is required (DELHI PG Mar. 09)
b) Generalised peritonitis due to appendicular b) Patients may require large volume of crystalloid
perforation infusion
c) TYPhoid enteric perforation and peritonitis c) Hypovolemia and sepsis contribute to tissue
d) Acute salpingo-oophoritis with peritonitis underperfusion
1936. Most pathognomic in pelvic abscess is- (UP 07) d) Base deficit >6 is marker of significant metabolic
a) Constipation b) Mucopurulent discharge acidosis
c) Loose stool d) Bleeding 1947. The most common intraperitoneal abscess following
1937. All are true aboutPseudomyxoma peritnonei except- peritonitis is- (UPSC-I/ 09)
a) Commoninmale (UP07) a) Subphrenic b) Pelvic
b) Associated with ovary tumours c) Paracolic d) Interloop
c) Yellow jelly collection of fluids 1948. Regarding Abdominal cocoon all statements are true
d) Appendicetal adeno carcinoma except- (PG!Nov09)
1938. 'Gas' in the tissue should be differenciates with- a) Common in young girl
a) Pseudomyxoma peritonei (UP 07) b) Associated with liver fibrosis
b) Pseudomonas infection c) Fibrosis of small bowel & stomach
c) Clostridium nouyi d) Chronic peritonitis is seen
d) Non clostridial infection e) Seen in tropical & subtropical region

1929)a 1930)a,c,d,e 193l)a 1932)b 1933)c 1934)None 1935)c 1936)b 1937)a 1938)c 1939)a 1940)d
194l)b 1942)c 1943)a,b,d,e 1944)b,c,d 1945)d 1946)a 1947)b 1948)b
SURGERY[274]

1949. The commonest site of an intraperitoneal abscess- 1957. Thumb printing is characteristic of -(AIIMS 86,88,
a) Pelvis (Maharashtra 10) a) Crohn's disease b)Ischemiccolitis PGI85)
b) Subphrenic space c) lGV d) Ulcerative colitis
c) Periappendicular 1958. Ulcerative colitis almost always involves the ... -
d) Paracolic gutter a) Caecum b) Sigmoid (AIIMS 87)
1950. A patient with stab injury to anterior abdomen c) Right colon d) Rectum
presents with a tag of omentum protruding through 1959. Procedure of choice in ulcerative colitis with acute
the abdominal wall near the umbilicus. On evaluation perforation is- (AIIMS 86, 88)
he is hemodynamically stable and shows no signs of a) Defunctioning ileostomy
peritonitis. Initial management of patient should b) Closure of perforation
involve- (A/11)
c) Proximal diversion colostomy
a) FAST
d) Total colectomy and ileostomy
b) Exploratory Laparotomy
1960. Commonest site for colonic diverticula
c) Local Wound Exploration and Suturing
d) CECT Abdomen is- (AIIMS 87, AI 90, PGI 89)
1951. A patient with abdominal injury presents to the a) Ascending colon b) Sigmoid colon
emergency department with signs of signs of c) Transverse colon d) Descending colon
peritonitis and shock. Airway and Breathing were 1961. Diagnostic ofHirschsprungs disease is-
secured and IV fluids were started with 2large bore a) Bariumenema (JIPMER87,PGI98)
cannulas. The next line of management should be- b) Rectal examination
a) FAST (AI 11) c) Manometry
b) Exploratory Laparotomy under general anesthesia d) Rectal biopsy
c) Insertion of abdominal drain followed by 1962. Small intestine is sutured by- (PGI 88)
laparotomy a) Non absorbable in 2layers
d) Laproscopy b) Absorbable in 2 layers
1952. Consider the following findings with reference to a c) Inner absorbable and outer non absorbable
diagnostic peritoneal lavage (DPL) in a case of d) Inter non absorbable and outer absorbable
abdominal trauma- (UPSC II 11) 1963. The following is true ofmesentric artery thrombosis-
1. 10 m1 of gross blood on aspiration a) Mostely embolic in nature (PGI 88)
2. W.B.(c) count more than 500/cu mm b) Shock is present
3. Amylase level more than 175lU/dL c) Extensive infarction of whole of small bowel and
4. R.B.(c)countmorethan 100,000/cumm cecum is seen
The criteria for a positive DPL are: d) Good prognosis
a) 1 and 2 only b) 3 and4 only 1964. Resection of the terminal ileum results in-(PGI 88)
c) 1, 2 and3 only 3 and4
't9~ii$J:j'''·.~,·--·~, -· a) Vitamin B 12 defeciency
b) Iron defeciency
c) Increased incidence of gall stones
d) Steatorrhoea
1965. Small intestinal tuberculosis can cause- (PGI 88)
SMALL & LARGE INTESTINE a) Diarrhoea b) Constipation
c) Stricture d) Malabsorption
1954. Complications of Meckel's diverticulum 1966. The commonest congenital gastrointestinal
include- (NIMH.87, Kerala 86) anomaly in an infant is- (TN 90)
a) Hemorrhage b) Intussusception a) Malrotation b) Congenital atresia
c) Strangulation d) All are correct c) Congenital megacolon d) None of the above
1955. Which sulphonamide is used for the treatment of 1967. Typhoid perforation occurs during- (AIIMS 89)
ulcerative colitis- (AI 88) a) Ist week b) 2nd weeks
a) sulphamethiazole b) Sulphathalazole c) 3m weeks d) 4th weeks
c) Sulphaguanidine d) Salazopyrin 1968. Tenesmus occurs in lesions of- (TN 90)
1956. In acute diverticulitis of the colon, the a) ileum b) Right side of colon
sigmoidoscopic finding is- (Kerala 87) c) Descending colon d) Sigmoid colon
a) Mucosa is inflammed 1969. The following are complications of ulcerative
b) Minute diverticuli seen colitis except- (AI 90)
c) Saw toothed appearance a) Peptic ulceration b) Arthritis
d) Sigmoidoscope cannot be passed beyond 15 em c) Sclerosing cholangitis d) Toxic megacolon

1949)a 1950)d 1951)b 1952)d 1953)b 1954)d 1955)d 1956)a 1957)b 1958)d 1959)d 1960)b 196l)d 1962)c
1963)a,b,c 1964)a,c 1965)a,c,d 1966)b 1967)c 1968)d 1969)a
SURGERY [ 275]

1970. Sulfonamide useful in treating ulcerative colitis is- 1983. The appearance of anastamotic leakage following
a) Sulfadiazine b) Sulfasalazine(AJ9/) a low colonic anastomosis most often mainfests -
c) Sulfamethoxazole d) Sulfadimidine a) 1-4 days b) 5-lO days (PG/ 80,AIIMS 85)
1971. All are true about villous adenoma of intestine c) 11-15 days d) 16-20days
except- (A/91) e) None of the above
a) Watery diarrhoea 1984. The most likely cause for suture leak after colon
b) Hypercalcemia and hyper phosphatema resection and anastomosis for cancer sigmoid is-
c) hypokalemia
a) Subclinical malnutrition (PG/81, Delhi 83)
d) Prolapse of the mass
b) Infection of anastomotic site
1972. Which is trne regarding Hirschsprungs disease-
a) More in females (AIJMS 91) c) Mechanical disruption due to colonic pressure
b) Presentation within 3 days and contractility
c) Regular dilatation is effective d) Ischaemia
d) Rectal biopsy diagnostic 1985. The operative treatment in Hirschsprung's disease
1973. Treatment of choice in multiple intestinal is only undertaken when child- (PG/79, TN 89)
strictures of segment of Jejumum is- (AJJMS 92) a) Is 2 years of age
a) Resection and end to end anastomosis b) Is at least 8 kg in weight and thriving
b) Nobles procedure c) Has no distension of abdomen
c) Stricturoplasty d) Has failed to respond to conservative treatment
d) End to side anastomosis 1986. Mass in right iliac fossa can be- (Kerala 94)
1974. Which is true of Ulcerative colitis- (AIIMS 92) a) ileoceccal T.B. b) ileocecal neoplasm
a) Sting sign ofkantor positive c) Amoeboma d) All
b) Skip lesions are seen 1987. Diverticular disease is not common in- (Kar. 94)
c) Rectum is always invlolved a) Colon b) Jejunum
d) Fistulas are common c) Duodenum d) Stomach
1975. Most common indication for laparotomy in 1988. False regarding invlovement in crohn's disease-
intestine T.B is- (PGJ 93) a) Anorectal area (AIIMS 94)
a) Peritonitis b) Intestinal obstruction b) Rectum
c) Doubtful diagnosis d) Lower GI bleeding c) Small intestine with Rt. colon
1976. The area ofthe colon which is used visulalized by d) Large intestine alone without involvement of
barium studies- (AIIMS 80, PG/81) small intestine
a) Sigmoid b) Hepatic flexure 1989. Skip transmural lesions with tuberculoid
c) Splenic flexure d) Caecum granulomaseenin- (AIIMS94, 91)
1977. The most frequent congenital anomaly ofthe GI
a) Ulcerative colitis b) Crohn's
tract is- (PG/80, AIIMS 80,81)
c) Tuberculosis d) Amoebiasis
a) Imperforate anus b) Meckel's diveticulum
1990. A 60 years old man presents with acute onset of pain
c) Malrotation d) Duodenal atresia
in lower abdomen followed by repeated rectal bledding
1978. Prophylactic polypectomy is done is -(JIPMER 81)
Ex. amination regealed pulse rate oflOO minute, BP
a) Peutz Jegher's syndrome b) Gardner's syndrome
c) Familial polyposis d) None of the above 160/96 mm of Hg and a localised tenderness in the
left hypochondrium. Stools examination reveales only
19379.The incidence of carcinoma of the caecum in
relation to colonic adenocarcinoma is- (PG/79, a few pus cells and sigmoidoscopywas normal. Which
a) 2% b) 4.5% AIIMS 82) one ofthe following is the most likely diagnos-
c) ZOO/o d) 44 % a) Idiopathic ulcerative colitis (UPSC 96)
e) 62% b) Bacillary dysentery
1980. Following procedures (except one) are done for c) Ischaemic colitis
correction ofHirschprung's disease- d) Amoebic colitis
a) Duhamel's b) Soave's (JIPMER 81, 1991. Following is least common about angiodyplasia of
c) Swenson's d) Bayar's AllMS 87) colon- (A/96)
1981. In intestine, lipoma is commonest in- a) Involvementofcecum
a) Rectum b) Sigmoid colon (AllMS 82, b) Involvementofrectumin 50% of cases
c) Caecum d)ileum PG/85) c) Affecting age group> 40 yrs.
1982. Internal fistulas with colonic diverticulitis are d) Cause of trouble some lower GI. hemorrhage
mostoften- (PG/81, 89) 1992. When rectal washouts are given to Hirshsprung's
a) Colovesical b) Coloenteric disease, the following flifluid is used- (Karn 95)
c) Colocolonic d) Colovaginal a) 5% dextrose b) Normal saline
e) Coloureteral c) Soap solution d) Tap water

1970)b 197I)b 1972)b,d 1973)c 1974)c 1975)a,b 1976)b,c 1977)b 1978)d 1979)None 1980)d 198l)d
1982)a 1983)b 1984)d 1985)b 1986)d 1987)d 1988)b 1989)b 1990)c 1991)b 1992)b
SURGERY [ 276]

1993. PeriampuUay carcinoma feature is- (AP 96) 2005. Which of the foUowing is true about carcinoma
a) Early jaundice b) Late jaundice colon- (AIIMS 2K)
c) Hemolytic jaundice d) No jaundice a) Lesion on the left side of the colon presents with
1994. Best treatment for Remission of Acute ulcerative features of anemia
colitis is - (AliMS 97) b) Solitary metastasis in liver is not a contraindication
a) Sulphasalazine b) Prednisolone for surgery
c) Aminosalicyclic acid d) NSAIDS c) Mucinous carcinoma has a good prognosis
d) Duke'sAstage should receive adjuvant chemotherapy
1995. All are features of congenital megacolon
2006. The following regarding colostomy are true
except- (A/97)
except - (Kerala 2K)
a) Large bulky stools b) Tight anal ring a) A colostomy is an artificial opening made in Large
c) Pseudodiarrhoea d) Failure to thrive Bowel to divert the faces to the exterior
1996. Best investigation for carcinoma colon (Kerala 97) b) Temporary colostomy is established to defunction
a) Barium enem b) Colonic biopsy an Anastomosis
c) Cliniclexamination d) Colonscopy c) Permanent colostomy is formed after the resection
1997. The best investigation for colorectal carcinoma- of Rectum by the abdominoperineal technique
a) Exfoliative cytology (Kerala 97) d) Double barreled colostomy is commonly done
b) Air contrast barium enema nowadays
c) Ultrasound e) Colostomy hernia is a common complication
d) Colonoscopy and biopsy 2007. Most common indication for operation in
1998. In ulcerative colitis, the inflammatory process is tuberculosis of intestine is - (Kerala 2001)
usuallyconfinedto- ~PSC) a) Obstruction b) Perforation
a) Mucosa alone c) Mass abdomen d) GI symptoms
b) Mucosa and submucosa 2008. Antiperistalsis is seen in- (AIIMS 9 1)
c) Mucosa, submucosa and muscularis a) Distal colon b) Jejunum
d) Mucosa submucosa, muscularis and serosa c) Proximal Colon d) ileum
1999. On colonoscopy which of the foUowing is highly 2009. Guaiac test is used for- (PGI 82)
malignant- ~IPMER9~ a) Pentosuria b) Fructosuria
a) Single pedunculated polyp c) For occult blood in stool d) Pancreatitis
b) Multiple flat polyp about hundreds 2010. Commonest site of tuberculosis of the intestines-
c) Multiple pedunculated polyp a) Stomach b) ileum (AI 89)
d) Solitary flat polyp c) Jejunum d) Colon
2000. Management of Ca descending colon in a 75 year 2011. Commenst malignancy of the smaU instestineis-
old male is- (AIIMS 98) a) Adenocarcinoma b) Lymphoma (PGI 89)
a) Total colectomy b) Left hemicolectomy c) Carcinoid d)Leiomyosarcoma
c) Defunctional colostomy d)Harmann'sprocedure 2012. Typhoid perforation is diagnosed by- (AIIMS 81,
2001. Hirschprung's disease is treated by- (AP 98) a) Plain X-ray of abdomen in erect posture
a) Colostomy b) Rectal examination BHU 8 7)
b) Excision of aganglionic segment c) Gastric aspiration
c) Colectomy d) Barium enema
d) Sodium chloride wash 2013. Commonest site involved in lleocaecal T.B.
2002. The commonest site of perforation during a) Intestinal wall b) Lymph node
c) Mesentery d) Intestinal mucosa
colonoscopy is- ~PSC 2K)
2014. Investigation of choice for small intestine tumor-
a) Caecum b) Hepatic flexure
a) Bameal follow through (JIMPER 98)
c) Splenic flexure d) Sigmoid colon
b) Echo
2003. Not true about hyperplastic tuberculosis -~P 2K)
c) X-ray abdomen
a) Most common site is ileo-caecal region
d) CT scan with contrast
b) Presents as mass in right iliac fossa
2015. Ulcerative colitis progressing to malignancy is
c) Surgery is the treatment of choice
characterized by following except- (PGI 97)
d) Barium studies are characteristic : a) Risk increases with the time
2004. True about carcinoma colon is- ~p 2K)
b) Prognosis worsens
a) Left side present with anemia c) Prognosis depends on period
b) Solitary liver metastasis is not a contradindication d) Forms pseudopolyps
to surgery 2016. Absence of myenteric ganglion is seen in -(PGI 97)
c) Most common site is ascending colon a) Crohn's disease b) Ulcerative colitis
d) Right side are usually stenosing variety c) Hirschprung's disease d) Intussusception

1993)a 1994)b 1995)a 1996)b 1997)d 1998)b 1999)b 2000)d 200l)b 2002)d 2003)c 2004)b 2005)b 2006)d
2007)a 2008)a 2009)c 2010)b 2011)a 2012)a 2013)d 2014)a 2015)d 2016)c
SURGERY [ 277]

2017. True about colonic organisms is- {PGI 98) 2028. Not true about malignancy arising from Ulcerative
a) Distal ileum 103- 105 org colitis is- (PG/99)
b) Colon 10 10 -IOil org a) Takes atleast 10 years to develop
c) First org in new born is colifonns and streptococcus b) Left sided is more common
d) Chyme in jejunum contains many bacteria c) Associated with dysplasia of the rest of the colon
2018. lnchildrenMCtypeofpolypis- {PG/98) d) Younger age of onset is associated with increased
a) Juvenile polyp chance of carcinoma
b) Solitarypolyp 2029. Paralytic ileus is seen in- {PG/99)
c) Familial polyposis a) Spinal cord injury b) Hypocalcemia
d) Multiple adenomatous polyp c) Hypermagnesernia d) Uremia
2019. In ulcerative colitis, Ca arises from- (PGI 98)
2030. True regarding cancer colon- (PGI 2000)
a) Pseudopolyps
a) Obstructive features are more common with right
b) Dysplastic sites
colon
c) Familialpolyposis
d) Multiple adenornatous polyp b) 40% cases at presentation show liver metastasis
2020. Trne about duplication of intestine is- {PGI 98) c) Resection possible in 25% cases
a) Spherical type is MC d) More common in AIDS patients
b) Tubular type is attached longitudinally with bowel 2031. Painless lower GI bleed is seen in child
c) Spherical cyst communicates with lumen with- (PGI 2000)
d) All of the above a) Meckle's diverticulum b) Rectal polyp
2021. Correct about diverticulosis is- {PGI 98) c) Anal fissure d) Ac. appendicitis
a) Arise from diverticula of colon 2032. True about enterocutaneous fistula- (PGI 2000)
b) Common in ileum a) High output fistula drains 500 ml/day
c) Protrusion of mucosa through muscular layers at b) Malignancy is most common cause
point of blood vessel c) Fluid & electrolyte loss can occur
d) All of the above d) No skin damage
2022. TrneaboutappendicnlarruptureisAIE- {PG/99) 2033. Trne about mesenteric venous thrombosis-
a) Common in extremes of age a) Peritoneal signs are always present (PGI 2000)
b) Common in people with Feclith obstruction b) Invariably involves long length ofbowel
c) Early antibiotics prevent rupture c) I. V. Heparin is the treatment of choice
d) Appendicectomy is done always in presence of d) Surgery can lead to short-bowel syndrome
rupture 2034. All are premalignant except- (PGI 2000)
2023. Which is true about intestinal tuberculosis - a) Adenomatous polyps b) Juvenile polyps
a) Common site is appendix {PGI 99) c) Familial polyposis coli d) Peutz-jegher synd
b) Causes intestinal perforation 2035. Pt with recurrent diarrhoea, pseudopolyp, lead pipe
c) Commonly associated with pulmonary TB appearance on Ba enema has- (PGI 2000)
d) Caused by mycobacterium TB a) Ulcerative colitis
2024. Fecal soiling in children is most commonly due to- b) Crolm's disease
a) Hirschsprung's disease {PG/99) c) Irritable bowel syndrome
b) Chronic constipation d) Short bowel syndrome
c) Rectal atresia 2036. Ulcerative colitis involves- {PGIOJ)
d) None of the above a) Serosa b) Lamina propria
2025. Which among the following is not pre-malignant- c) Mucosa d) Circularis mescle
a) Ulcerative colitis {PGI 99) e) Submucosa
b) Peutz-jeghers syndrome 2037. Risk of malignancy in ulcerative colitis is more in-
c) Villous adenoma a) Onset in childhood {PGIOJ)
d) Familial adenomatoses polyopsis b) Extensive involvement of colon
2026. In extensive small bowel resection, all are seen c) Disrupted architecture with crypt abscesses
except- {PG/99) d) Pseudopolyps
a) Gastric hyposecretion e) Recurrence after treatment
b) Decreased protein absorption 2038. True about Hirschsprung's disease- (PGIOJ)
c) Gall stones increased a) Pathology of myenteric plexus of Auerbach
d) Oxalate stone increased b) Blood in stools
2027. Whichismostmalignant- {PG/99) c) May involve small intestine rarely
a) Villous adenoma b) Tubulovillous adenoma d) Involved segment of intestine is dilated
c) Tubular adenoma d) Familial polyposis e) Present only in infant & children

2017)b 2018)a 2019)b 2020)a,b 2021)a,c 2022)c 2023)b,c,d 2024)b 2025)b 2026)a 2027)d 2028)b 2029)a,d
2030)None 2031)a,b 2032)a,c 2033)d 2034)b,d 2035)a 2036)c,e 2037)a,b 2038)a,c
SURGERY [ 278]

2039. True about left sided colon carcinoma- (PGI OJ) 2049. After Hemicolectomy, on the 6th postoperative day
a) Anemia b) Obstruction pt. developed serous discharge from the wound.
c) Melena d) Feculant Following are to be done- (PGI 03)
e) Sigmoid spared a) Dressing of the wound only
2040. True about Meckel's diverticulum is/are -(PGI OJ) b) StartiVF
a) Bleeding PR c) Do urgent Laparotomy
b) Can be a leading point of intussusception d) Do Ba-enema to see for anastomotic leak
2050. True statements about inflammatory bewel disease
c) Multiple diverticuli common
(IBD)- (PG/03)
d) Visualised best by barium meal
a) Surgery is the 1st management
e) Almost never requires active treatment b) Pouch operation is done for:fuhninantulcerative colitis
2041. TrueaboutCacolon- (PG/02) c) 2% case ofchrons disease undergo malignant changes
a) l/3rd of cases shows liver metastasis d) Ulcerative colitis don't undergo malignant changes
b) Obstruction common in sigmoid colon 2051. True about mesenteric vein thrombosis- (PGI 03)
c) CEA is useful for prognostic importance a) Peritoneal signs always present
d) King's staging in done b) Thrombobectomy is always done
e) Alternate bowel habit is common inCa left colon c) Heparin is given
2042. Dietery factors associated with colon carcinoma- d) Surgery can lead to short bowel synd.
a) Highfiber b)Lowfiber (PG/02) 2052. True about small bowel tumour- (PGI 03)
c) Smoked fish d) High fat intake a) Commonly located in duodenum
e) Japanese are common to develop Ca colon b) Lymphoma is common
2043. Turcot's syndrome is associated with- (PGI 02) c) Adenocarcinoma has good prognosis
a) Duodenal polyps d) Palliative surgeries are done even in presence of
b) Familial adenomatous polyposis metastasis
e) Adjuvant chemotherapy is given
c) Brain tumors
2053. A 60 years old man suffering from left colon Ca
d) Villous adenoma
presented with acute Lt. colonic obstruction the
e) Hyperplastic polyps treatmentis- (PGI 03)
2044. Carcinomaofrightcolonpresentsas- (PG/02) a) Primary resection and Hartman's procedure
a) Anemia b) Defunctioning colostomy
b) Mass in RIF c) Rt. hemicolectomy
c) BleedingPR d) Resection ofwhole Lt bowel and end to end anastomosis
d) Alternate constipation and diarrhea e) Conservative treatment
e) Presents with obstruction 2054. Which of the following polyps is not premaliganant-
2045. Predisposing factors for colon ca are- (PGI 02) a) Juvenile polyposis syndrome (PGI 03)
a) Animal fat consumption b) Peutz jegher's syndrome
b) Familial adenomatous polyposis c) Ulcerative colitis
c) Ulcerative colitis d) Familial polyposis coli
d) Crohn's disease 2055. True about neoplastic colorectal polyps- (PGI 03)
e) 1B a) Sessile polyps > 1 em is malignant
2046. Hirchprung's disease- (PGI 02) b) MC site is colon and rectum
c) Adenomatous polyp is premalignant
a) Is seen in infants and children only
d) Tubular adenoma is malignant
b) Absence of ganglia in involved segement
e) Pseudpolyps are premalignant
c) The involved segment is the dilated colon 2056. Appendicitis is diagnosed by- (PGI 03)
d) Bleeding PR is a presenting feature a) TC andDLC b)X-rayabodmen
e) Surgery is used in therapy c) USG d) Colour doppler
2047. Which of these are associated with increased risk e) Baenema
of colorectal ca- (PGI 02) 2057. A 70 yrs old man presenting with acute pain
a) More intake of animal fat b) Aspirin abdomen, pulsating abdominal mass with shock; the
c) Ulcerative colitis d) Amoebic colitis managementis- (PG/03)
e) Polyps a) 6 unit blood transfusion
2048. When acute appendicitis is suspected, it can be b) CT scan of abdomen to diagnose the aneurysm
confirmedby- (PG/02) c) Shift the patient to 0. T. immediately and
a) Clinicalexamination b)USG exploration of abdomen
c) CT scan d) Blood counts d) Laparotomy
e) Upper GI endoscopy e) Angiography to diagnose the aneurysm

2039) b 2040) a,b 2041) a,b,c,e 2042) b,d,e 2043) b,c 2044) a,b 2045) a,b,c,d 2046) b,e 2047) a,c,e 2048) a,b,c
2049) a 2050) None or c 2051) c,d 2052) d 2053) a,b,d 2054) a,b,c,e 2055) a,b,c 2056) c 2057) a,c,d
SURGERY [ 279]

2058. True about dietary factors in colorectal carcinoma- 2068. Rotation of sigmoid volvulus occurs- (AMU 05)
a) Low fiber diet (PGI 03) a) Clockwise
b) High fiber diet and high fat intake b) Anticlockwise
c) Low fiber, high fat intake c) Initially clockwise later anti clockwise
d) Smoked fish d) Either clockwise or anticlockwise
e) High fiber diet 2069. Carcinoma of the colon develops in all patients with-
2059. In case elective surgery of Ca sigmoid colon, a) Juvenile polyposis (COMEDK 05)
whichofthefollowingshould be done- (PGI04) b) Hamartomatous polyp
a) Mechanical bowel wash c) Inflammatorypolyps
b) Broad spectrum antibiotic given 48 hours before d) Familial adenomatous polyposis
operation 2070. In a patient with spontaneous biliary enteric fistula
c) Broad spectrum antibiotic at the time of operation the most common site of communication with gall
2060. Trueaboutduodenaladenocarcinoma- (PGI04) bladderis- (UPSC05)
a) Commonest small intestinal tumor a) Duodenum b) Jejunum
b) Arises from periampullary region c) lleum d) Transverse colon
c) Jaundice & anaemia found 2071. The most useful investigation for profuse lower
d) 5 yr survival is 5% gastrointestinal bleeding is- (UPSC 05)
e) Surgery is only curative a) Proctosigmoidoscopy
2061. Commonest complication ofMeckel's diverticulum b) Colonoscopy
is - (Kerala 03, AI 91) c) Double contrast barium enema
a) Bleeding b) Perforation d) Selective arteriolgraphy
c) Diverticulitis d) Intussusception 2072. "Cobblestone" appearance seen on colonoscope is
2062. Which ofthe following organisms produces signs characteristic of the foUowing disease -(MAHE 05)
and symptoms that mimic acute appendicitis- a) Crohn's disease b) Ulcerative colitis
a) Enteropathic Escherichia coli (Karnataka 03) c) TB colitis d) Irritable bowel syndrome
b) Enterobius vermicularis 2073. The fold ofTreves is- (MAHE 05)
c) Trichomonas hominis
a) The fold of mucous membrane projecting into the
d) Yersinia enterocolitica
lumen ofthe rectum
2063. Which one of the following conditions is diagnosed
b) The iiieo-appendicular fold of peritoneum
by T99 Pertechnetate Scintigraphy? ( UPSC 04)
a) Pharygeal diverticulum c) The fold of mucous membrane around the papilla
b) Duodenal diverticulum ofVater
c) Meckel's diverticulum d) The fold of peritoneum over the inferior mesenteric
d) Colonic divertiCulum vein
2064. Which of the following is true about carcinoma of 207 4. Signe-de-Dance is- (MAllE 05)
colon- (SGPGI 05) a) Empty right iliac fossa in intussusception
a) Left colon tumors usually present with anaemia b) Pincer shaped appearance in barium enema in
which is severe & unyielding to treatment intussusception
b) Mucinous carcinoma has excellent prognosis c) Tenderness at the McBurney's Point
c) Duke 'A: tage tumours need adjuvant chemotherapy d) Passing of large quantities of urine in
d) Solitary superficial hepatic metastasis is not a hydronephrosis
contraindication for resection 2075. Common surgical indication in chronic
2065. All are true about Gardner's syndrome except- infiammatory bowel disease- (PGI June 05)
a) Protein losing enteropathy (SGPGI 05) a) Obstruction b) Malignancy
b) Always in 5th decade of life c) Massive bleeding
c) Small intestine polyp 2076. True about smaU bowel diverticula- (PGI June 05)
d) Malignancy is common a) Contains all the layers ofbowel wall
2066. Rectal polyp most commonly presents as-(SGPGI 05) b) Seen in terminal ileum
a) Obstruction b) Bleeding c) Radiographically not visualized
c) Infection d) Changes into malignancy d) Surgical treatment is not required
2067. Toxic megacolon is seen in- (SGPGI 05, NIM 86,
2077. Surgical treatment of ulcerative colitis -
a) Chronic non specific ulcerative colitisAIIMS 87,
a) Done in late cases only (PGI June 05)
b) Crohn's disease 94)
b) Done in cases where medical treatment fails
c) Colonic diverticulosis
c) Pouch surgery done
d) Hamartomatous polyp

2058)a,c,d 2059)a,c 2060)b,c,e 2061)a 2062)d 2063)c 2064)d 2065)b 2066)b 2067)a 2068)a 2069)d 2070)a
2071)b 2072)a 2073)b 2074)a 2075)a,b,c 2076)a,b 2077)All
SURGERY [ 280]

2078. Mr. C.P. Patel is having carcinoma colon ofleft side, 2088. Surgical indications in inflammatory bowel diseases
comes to emergency department with obstruction. are AlE- (PGI June 06)
What will be the best treatment to be given- a) Obstruction
a) Hartman's procedure (MAHA 05) b) Perianal complication
b) Defunctioning colostomy c) Extraintestinal complication
c) Lieotransverse anastomosis d) Stricture
d) Transverse colostomy e) Malignancy
2079. Peutz..Jegher syndrome-all are ture except- 2089. ffirschpruug's disease true are- (PGI June 06)
a) Pigmentation oflips & oral mucosa (APPGE 05) a) Sometimes found in adult
b) Polyposis of jejunum b) Dilated segment involved
c) Radiotherapy is treatment of choice c) Auerbach's plexus absent
d) Malignancy occurs rarely d) Sometimes involve small intestine
2080. Snow storm ascites is seen in- (APPGE 05) e) Bleeding PR is usual presentation
a) Meconium ileus 2090. Meckel's diverticulum, true are AlE -(PGI June 06)
b) Hirschsprung disease a) Has three layers with artery
c) Ileocaecal tuberculosis b) Heterotopic epithelium in 50-60% cases
d) Pseudomyxoma peritonei c) Always gastric epithelium
2081. The term "Left sided appendicitis" as popularly d) May present with hemorrhage
called is nothing but- (APPGE 05) e) When accidentally seen in laparotomy always
a) Diverticulitis b) Ascending colitis resected
c) Descending colitis d) Typhilitis 2091. Features ofBiunttrauma abdomen maximum injury
2082. The diagnosis of congenital megacolon is confirmed is to- (PGI June 06)
by- (AIIMSNov05) a) Proximalileum b)Proximaljejunum
a) Clinical features b) Barium enema c) Midilium d) Ileocaecaljunction
c) Rectal biopsy d) Recto - sigmoidoscopy e) Colon
2083. All of the following extraintestinal manifestations
2092. Features of left side Ca colon are- (PGI June 06)
of ulcerative colitis respond to colectomy except-
a) Anemia b) Obstruction
a) Primary sclerosing cholangitis (AIIMS Nov 05)
c) Altered bowel habit d) Melena
b) Pyoderma gangrenosum
e) Feculentvomiting
c) Episcleritis
2093. Commonly undergoing malignant transformation
d) Peripheral arthralgia
is/are - (PGI June 06)
2084. The most common facial abnormality seen in
a) FAP b) Crobn's disease
Gardener's syndrome is- (AIIMS Nov 05)
a) Ectodermal dysplasia b) Odontomes c) Ulcerative colitis d) Enteric colitis
c) Multile osteomas d) Dental cysts e) Juvenile polyposis
2085. All of the following are modalities of therapy for 2094. Inflammatory bowel disease found in children-
hepatocellular carcinoma except- (AIIMS Nov 05) a) Ulcerative colitis b) Tropical sprue (PGI June
a) Radiofrequency ablation c) Crobn's disease d) Celiac disease 06)
b) Transarterial catheter embolization e) Cystic fibrosis
c) Percutaneous acetic acid 2095. Small intestinal biopsy is diagnostic in -
d) Nd Yag laser ablation a) Whipple's disease (PGI June 06)
2086. Gardener's syndrome is a rare hereditary disorder b) Abetalipoproteinemia
involving the colon. It is characterized by- (AI 05) c) Celiac disease
a) Polyposis colon, cancer thyroid, skin tumours d) Agammaglobulinemia
b) Polyposis isj~pituitmyadenomaandsk:intumours 2096. Carcinoma sigmoid colon with obstruction
c) Polyposis colon, osteomas, epidermal inclusion Management includes- (PGI June 06)
cysts and fibrous tumours in the skin a) Hartmann's procedure
d) Polyposis of gastrointestinal tract, b) Resection and end to end anastomosis with
cholangiocarcinoma and skin tumours proximal colostomy
2087. A new born presented with bloated abdomen shortly c) Proximal colostomy with distal ligation
after birth with passing of less meconium. A full- d) Sub-total colectomy
thickness biopsy rectal biopsy finding is most likely 2097. All of the following is seen after massive small bowel
tobepresent- (AI05) resection, except- (APPG 06)
a) Fibrosis of submucosa a) Increase in length of the bowel
b) Hyalinisation of the muscular coat b) Villous hypertrophy
c) Thickened Muscularis propria c) Mucosal hyperplasia
d) Lack of ganglion cells d) Decrease in surface area per cell

2078)a 2079)c 2080)a 208l)a 2082)c 2083)a 2084)c 2085)d 2086)c 2087)d 2088)c 2089)a,c,d 2090)c,e
2091)b,d 2092)b,c 2093)a,c 2094)a,c 2095)a,b,d 2096)All 2097)a
SURGERY [ 281 ]

2098. Which one of the following gastrointestinal 2110. What is most common type of carcinoma of the right
disorders predisposes to urolithiasis· (UPSC 07) colon- (UPSC-II 08)
a) Peutz- Jegher's syndrome a) Stenosing b) Ulcerative
b) Short bowel syndrome c) Tubular d) Fungating
c) Familial polyposis coli 2111. Most common cause of colonic fistula in India at age
d) Ulcerative colitis of27 years- (UP 08)
2099. All the following polyps are premalignant except- a) Crohn's disease b) Ulcerative colitis
a) Juvenile polyposis syndrome (AI 07) c) Tuberculosis d) Carcinoma rectum
b) Famillial polyposis syndrome 2112. Metabolic abnormality seen in large colorectal
c) Juvenile polyp villous adenoma- (Aiirns May 08)
d) Peutz jeger syndrome a) Hypokalemic metabolic alkalosis
2100. Which oftbe following bas least malignant potential- b) Hypokalemic metabolic acidosis
a) Juvenile polyps in Juvenile (AI 07) c) Chlorine sensitive metabolic acidosis
polyposis syndrome d) Chlorine resistant metabolic alkalosis
b) Hamartomatous polyps in peutz Jaghers syndrome 2113. Treatment of carcinoma left colon with acute
c) Adenomatous polyps in Familial colonic polyposis obstruction- (PGI June 08)
d) Adenomatous polyps in HNPCC a) Harman's procedure
2101. Colonic diverticulosis is best diagnosed by- b) Left colectomy with anastomosis
a) Colonoscopy (Aiirns May 07) c) Proximal colostomy
b) Nuclear scan d) Extended rt. colectomy with ileoanal anastomosis
c) Barium enema e) Primary anastomosis should never be attemped
d) CT scan 2114. String sign ofkantor seen in? (APPG 08)
2102. Which of the following colonic polyps is not a) Chrons discose b) Ulcerative colitis
premalignant· (Aiims May 07) c) Both d) None
a) Juvenile polyps 2115. What is epicolic node? (APPG 08)
b) Hamartomatous polyps associated with Peutz- a) Node draining colon b) Adjacent to aorta
Jegher's syndrome c) Epitracheal node d) None
c) Villous adenoma 2116. Most common presentation of Meckels
d) Tubular adenomas diverticulum? (APPG 08)
2103. Most common site for carcinoid tumor in the a) Lower GI bleeding b) Upper GI bleeding
abdomen- (MAHE 07) c) Diarrhorea d) Abdominal pain
a) Appendix b)Liver 2117. True about Hirschprung's disease- (PGI Dec 08)
c) Intestines d) Pancreas a) Aganglionic segment is contracted not dilated
2104. Most common cause of lower gastro intestinal b) Descending colon is most common site of
bleeding is· (UP 07) aganglionosis
a) Diverticulitis b) Colorectal carcinoma c) Barium enema is diagnostic
c) Angiodysplasia d) Anal fissure d) Is seen in infants & children only
2105. Meckel's diverticulum is patent of· (UP07) e) Barium enema show calcification
a) Vitello intestinal duct b) Urachus 2118. Most common cause of acute mesenteric ischemia
c) Oganic diaphragm d) None is - (AIIMS Nov 08)
2106. Crohn's disease can be seen in- (Corned 07) a) Arterial thrombosis b) Venous thrombosis
a) Jejunum only b) Colon only c) Embolism d) Non occlusive disease
c) Terminal ileum and right side d) Mouth of anus 2119. Based on Epidemiological studies, which of the
2107. The Mechel's diverticulum is situated within following bas been found to be most protective against
about ___ em from the ileocecal valve- carcinoma colon- (AI 09)
a) 25 b)60 (Corned07) a) High fiber diet b) Low fat diet
c) 75 d) 100 c) Low selenium diet d) Low protein diet
2108. Cobble stone appearance is seen in- (Corned 07) 2120. All of the following genes may be involved in
a) Ulcerative colitis b) Crohn's disease development of carcinoma of colon Except- (AI 09)
c) Appendicitis d) Carcinoma rectum a) APC b) Beta- Catenin
2109. Thecommonestgastricpolypis- (Corned08) c) K - ras d) Mismatch repair genes
a) Hyperplastic polyp 2121. Most important prognostic factor for colorectal
b) Inflammatory polyp carcinoma is- (AI 09)
c) Adenomatous polyp a) Site oflesion b) Stage oflesion
d) Part of familial polyposis c) Age of patient d) Lymph node status

2098)b 2099)c 2100)b 2101)c 2102)a 2103)a 2104)a 2105)a 2106)d 2107)b 2108)b 2109)a 2110)b 2111)a
2112)b 2113)a,b,c,d 2114)a 2115)a 2116)a 2117)a 2118)c 2119)a 2120)None 2121)b
SURGERY [ 282]

2122. Parastomal hernia is mostfrequently seen with- 2132. Which of the following statements aboutCrohn's
a) End colostomy b) Loop colostomy (AI 09) disease is incorrect? (DELHI PG Mar. 09)
c) End Iliostomy d) Loop Iliostomy a) Granuloma present frequently
2123. Most common cause of death in Crohn's disease is b) It is separate and distinct form ulcerative colitis
dueto- (AIIMSMay09) c) Cigarette smoking is a risk factor
a) Sepsis d) Rectum spared in 50% patients with large bowel
b) Thromboembolic complication involvement
c) Electrolyte disturbance 2133. The short bowel syndrome is characterized by all of
d) Malignancy thefollowingexcept- (Manipal 09)
2124. Skip lesions are seen in- (AIIMSMay09) a) Diarrhea b) Hypogastrinemia
a) Ulcerative colitis b) Crohn's disease c) Weight loss d) Steatorrhoea
c) Typhoid d) Tuberculosis 2134. The following disorders are predisposing conditions
2125. True regarding Barium study of ileocaecal for carcinoma of the colon except- (UPSC-II 09)
tuberculosis- (PGI June 09) a) Ulcerative colitis
a) String sign b) Goose neck sign b) Villous adenoma
c) Rt sided obstruction d) Pulled up caecum c) Familial polyposis coli
e) Sterllin sign d) Peutz-Jeghers syndrome
2126. True about virtual colonscopy (VC)- (PGI June 09) 2135. Which ones is commonly a/w crohn's disease-
a) Have better image than conventional colonoscopy a) Cologastric b) Coloureteric (PGI Nov 09)
b) VC is performed by CT & MRl c) Colovesical d) Coloduodenal
c) Take over time than conventional colonoscopy e) Colovaginal
d) easy to take tissue sample 2136. All are true about Ulcerative colitis except-
e) Helpful in pathology outside colon a) All layer are involved (PGI Nov 09)
2127. True about diverticulitis- (PGI June 09) b) Malabsorption
a) Occur at any age c) Backwash ileitis in 10-15%
b) Often incidental finding at operation d) Mesalazine maintains the disease in remission
c) Young patient has more aggressive disease e) Rarely recurs after surgery
d) Left sided colon involvement is more common 2137. Hirschsprung's disease is due to- (AIIMS Nov 09)
e) Operation is not for all patients a) Loss of ganglion cells in the sympathetic chain
2128. Which ofthe following statement about edematous b) Atrophy of longitudinal muscles
bowel is correct? (DELHI PG Mar: 09) c) Failure of migration of neural crest cells from
a) Stapled Anastomosis is less prone to leak than cranial to caudal direction
hand-sewn ones d) Malformed taenia coli
b) Continuous monolayer Anastomosis gives best 2138. lleocecal tuberculosis is associated with-
result a) Megaloblastic anemia (AIIMS Nov 09)
c) As bowel edema subsides, knots become loose b) Iron deficiency anemia
d) Bowel edema results from vigorous resuscitation c) Sideroblastic anemia
2129. Which of the following about acute diverticulititis d) Normocytic normochromic anemia
is incorrect? (DELHI PG Mar: 09) 2139. Which of the following colonic polyps has no risk
a) Sigmoid is the commonest site for malignancy- (AIIMS Nov 09)
b) Peri-colic abscess can occur a) Juvenile polyps
c) Fistulization is an emergency b) Hamartomatous polyps associated with Peutz-
d) Conservative treatment may be successful in Jegher's syndrome
severe attack c) Juvenile polyposis syndrome
2130. The commonest cause of significant lower d) Famillial adenomatous polyposis syndrome
gastrointestinal bleed in a middle aged person with 2140. Which ofthe following colonic polyps has no risk
unknown reason is- (DELHI PG Mar. 09) for malignancy- (AIIMSMay 10)
a) Sigmoid diverticula b) Angiodysplasia a) Juvenile polyps
c) Ischemic colitis d) Ulcerative colitis b) Hamartomatous polyps associated with Peutz-
2131. Which of the following statements about sigmoid Jegher's syndrome
volvulus is incorrect? (DELHI PG Mar: 09) c) Juvenile polyposis syndrome
a) More common with laxative abuse d) Famillial adenomatous polyposis syndrome
b) Non-operative treatment has no role 2141. PeutzJeghers polyps present most commonly in-
c) Recurrence rate around 40% a) Rectum b) Colon (AIIMS May 1 0)
d) Sigmoid resection is defmitive treatment c) Esophagus d) Jejunum

2122)a 2123)d 2124)b 2125)All 2126)a,b,e 2127)d 2128)c 2129)c 2130)a 2131)b 2132)d 2133)b 2134)d
2135)a,c,d,e 2136)a,b 2137)c 2138)a 2139)a 2140)a 2141)d
SURGERY [ 283]

2142. All of the following genes may be involved in 2152. Child presents with recurrent abdominal pain and
development of carcinoma of colon Except- bilious vomiting. Condition was diagnosed by barium
a) APC b)Beta-Catenin(AIIMSMay 10) follow through. Surgery was done, - mesentric
c) K-ras d) Mismatch Repair Genes widening, appendicectomy, cutting the Ladd's band.
2143. Early post-operative complication of ileostomy in the What is the diagnosis? (AIIMS Nov 10)
post-operative period- (AIIMS May 10) a) Recurrent Caecal volvulus
b) Mal rotation
a) Obstruction b) Necrosis
c) Recurrent appendicitis
c) Diarrhea d) Prolapse
d) Stricture TB
2144. Best investigation to diagnose Meckel's
2153. A 50 yr. old male presents with obstructive symptoms.
diverticulum? (Maharashtra 10) Biopsy of stomach reveals Gastrointestinal stromal
a) X-ray abdomen b) Ultrasonography tumor(GIS1). MOst appropriate marker for GIST is-
c) Barium study d) Tc 99m Pertectinate scan a) CD34 b) CD 117 (AIIMSMay 11)
2145. Which one ofthe following is not a premalignant c) CD30 d) CD 10
condition for colon cancer? (UP SC II 10) 2154. All of the following genes may be involved in
a) Familial adenomatous polyposis coli development of carcinoma of colon Except-
b) Villous adenoma a) APC b)Beta-Catenin(AIIMSMay 11)
c) Ulcerative colitis c) K-ras d) Mismatch Repair Genes
d) Hamartomatous polyps 2155. Which of the following is NOT true about FAP?
2146. True about Hirschprung's disease- (PGI May 10) a) ARinheritance (AIIMSMay 1!)
a) Aganglionic segment is contracted not dilated b) Screening done by sigmoidoscopy
b) Descending colon is most common site of c) Polyps develop in late adulthood
aganglionosis d) Epidermal cysts & osteomas may occur
2156. Based on Epidemiological studies, which of the
c) Barium enema is diagnostic
following has been found to be most protective against
d) It is seen in infants & children only
Carcinoma Colon? (AIIMS May 11)
e) Barium enema show calcification
a) High fiber diet b) Low fat diet
214 7. Investigation for acute abdomen includes - c) Low selenium diet d) Low protein diet
a) USG (PGI May 10) 2157. Which ofthe following is the terminal group oflymph
b) Multidetector CT node for colon? (AIIMS May 11)
c) Contrast enhanced CT a) Paracolic b) Epicolic
d) X-ray c) Preaortic d) Ileocolic
e) Echocardiography 2158. Gold standard investigation for recurrent
2148. Colonic disease can be diagnosed by all gastrointestinal stromal tumor is-(AIIMS May 11)
except- (PGIMay10) a) MRI b)MIBG
a) Virtual colonoscopy b)Baenema c) USG d)PETCT
c) Baswallow d) Ba follow through 2159. Most important prognostic factor for colorectal
e) Enteroclysis carcinoma is- (AIIMS May 11)
2149. All are true statement about Meckel's diverticulum a) Site oflesion b) Tumour size & characteristics
except- (PGINov.10) c) Age of patient d) Lymph node status
a) Occurs in 2% of population 2160. Most common cause of acute mesenteric ischemia
is - (AIIMS May 11)
b) Perforation occurs
a) Arterial thrombosis b) Venous thrombosis
c) Common on antimesenteric border
c) Embolism d) Non occlusive disease
d) Contains ectopic gastric tissue 2161. Multiple Cutaneous Sebaceous adenomas are seen
e) Diarrhoea very common in- (AI 11)
2150. Features(s) of Jejunal diverticula is/are- a) Gardner's syndrome b) Turcot's syndrome
a) 1' Folate absorption (PGI Nov. 10) c) Muir Torre syndrome d) Cowden syndrome
b) J.. Ferritin absorption 2162. A 70-year-old patient complains of recent change in
c) J.. B 12 absorption bowel habits. His stool examination is positive for
d) Urea breath test occult blood and he has anaemia. Which one of the
e) Steatorrhoea following tumour markers would be appropriate to
2151. TrueaboutGISTallexcept- (AIIMS Nov 10) test in him? (UPSC I 11)
a) Most common in duodenum a) CA-125
b) Necrosis and ulceration present b) Prostatespecific antigen
c) PET is used to assess response to therapy c) Carcino Embryonic Antigen
d) Well circumscribed d) Neuron-specific enolase

2142)b 2143)b 2144)d 2145)d 2146)a 2147)a,b,c,d 2148)c,d,~ 2149)e 2150)c,e2151)a 2152)b 2153)b 2154)b
2155)a 2156)a 2157)c 2158)d 2159)d 2160)c 2161)c 2162)c
SURGERY [ 284]

2163. The following statements regarding Meckel's 2174. Which is true about Intusssusception- (AI 91)
diverticulum in adults are true except- (UPSC II 11) a) Common in neonates
a) It is a remnant of omphalomesenteric duct b) Fever always present
b) It usually presents on the anti-mesenteric border c) Not associated with tumors of intestine
of small intestine d) Usually relieved by barium enema
c) Bleeding is a common complication 2175. The treatment of choice in duodenal atresia-
d) It should be removed if detected incidentally a) Gastrojejunostomy (PGI 79, AI 89)
during an abdominal exploration b) Duodenojejunostomy
2164. The most common type ofintussuception is- c) Bishop Koop procedure
a) Multiple b) Colocolic (PGI 81, AP 89) d) Duodenoduodenostomy
Ileoileal Ileoilecolic
2176. Best investogation of acute mechanical intestinal
obstruction is- (DELHI 89, 90)
a) X-Ray abdomen b) Baruin enema
c) Proctosigmoidoscopy d) Flatus tube
2177. The first to appear in a cause of acute intestinal
obstruction is- (PGI 82,DNB 90)
INTESTINAL OBSTRUCTION a) Constipation b) Colicky pain
c) Vomiting d) Distension
2166. Commonest site of intestinal atresia is in the - 2178. The least common type ofintussuception is-
(AP 86, PGI 86, NIMHANS 86, PGI 85, UPSC 87, a) Multiple b) Colocolic (PGI 81, AP 89)
a) Duodenum b) Jeiunum Jipmer 91) c) Ileoileal d) Ileoilecolic
c) Lleum d)Colon 2179. Henock Schnolein Purpura may rarely
2167. Predisposing factors for sigmoid volvulus are - cause- (JIPMER 80,81,AIIMS 90)
a) Band of adhesion (KERALA 97) a) lntussuception b) Volvulus
b) Long pelvic meso colon c) Atrial fibrillation d) Hernia
c) Narrow attachment of pelvic mesocolon 2180. Bilious vomitting is seen in all except- (Kerala 94)
d) Loaded pelvic colon a) Jejunal atresia b) Volvulus of small intestine
e) All of the above c) Djuodenal atresia d) Pyloric stenosis
2168. Most common cause of colonic obstruction 2181. Paralytic ileus is caused by- (Kerala 94)
is - (AMI 86, PGI 86, UPSC 88) a) Peritonitis
a) Volvulus b) Hernia b) Hyperkalemia
c) Adhesions d) Neoplasm c) Acute intestinal obstruction
2169. In obstruction of the large gut rupture occurs at d) Head injury
the....... - (PGI 89) 2182. Acute mechanical large bowel obstruction should
a) Cecum b) Ascending colon be operated early because- (UPSC 9 5)
c) Transverse colon d) 'bescending colon a) Electrolyte imbalance due to third space loss
2170. Commonest cause ofintussusception is- (TN 90) b) Septicaemia from absorption of bowel contents
a) Submucous lipoma c) Early gangrene and perforation
b) Meckel's diverticulum d) Respiratory embarrassment to massive abdminal
c) Hypertrophy of submucous peyer's patches distension
d) Polyp 2183. In an infant born with intestinal obstruction due
2171. Commonest cause of colonic obstruction in to meconium ileus, one should suspect -(UPSC 95)
neonates is- (TN 90) a) Hirschsprung's disease b) Mucoviscoidosis
a) Meconium ileus b) Aganglionic colon c) Imperforate anus d) Bile duct atresia
c) Ileal atresia d) Volvulus 2184. Cause of gaseous distension of abdomen in acute
2172. Intestinal obstruction due to meconium occurs intestinal obstruction is- (AIIMS 95, A/95)
in- (TN90) a) Diffusion of gases from blood into intestinal lumen
a) Cystic fibrosis b) Mucoviscidosis b) Swallowed atmospheric air
c) Hurschsprung disease d) Ileal atresia c) Bacterial fermentation of food matter
2173. Water loss is severe if intestinal obstruction occurs d) Disgestion process & released gases
at- (JIPMER 90) 2185. Primary feature of small intestinal obstruction -
a) First part of duodenum a) Fever (AP 96)
b) Third part of duodenum b) High peristalsis with colic
c) Midjejunum c) Abdominal distension
d) lleum d) Empty rectum

2163)d 2164)d 2165)b 2166)a 2167)e 2168)d 2169)a 2170)c 2171)b 2172)b 2173)a 2174)d 2175)d 2176)a
2177)b 2178)a 2179)a 2180)d 2181)a 2182)c 2183)b 2184)b 2185)b
SURGERY[285]

2186. Acute mechanical large bowel obstruction should 2197. Major causes of distension in intestinal obstruction
be operated early because oftbe risk of- a) Gas produced by coliform organism (PGI 98)
a) Respiratory embarrassment due to abdominal b) Swallowed air
distension c) Diffuse from arterial surface
b) Electrolyte imbalance from vomiting (UPSC 97) d) Not known
c) Septicaemia from bowel contents 2198. Prologed Post-op ileus is best treated by- (PGI 98)
d) Closed-loop obstruction and caecal perforation a) Long tube insertion
b) Calcium pentonthenate
2187. Commonest site of ischemic colitis- (PG/97)
c) Laporotomy and exploration
a) Hepatic flexure b) Splenic flexure d) Peristaltic stimulants
c) Transverse colon d) Sigmoid colon 2199. Fluidlevelsarenotvisiblein- (PG/98)
2188. An elderly male with history of IHD and a) Meconeum ileus b) Intussusception
cerebrovascular disease presented with abdominal c) Colon pouch d) Duodenal obstruction
pain, and bloody stools, the likely diagnosis will be- 2200. Spastic ileus is seen in - (PGI 99)
a) Ulcerative colitis (AIIMS 97) a) Porphyria b) Retroperitoneal abscess
b) Crohns c) Hypokalemia d) MI
c) Acute mesentric ischaemia 2201. Recurrent pain abdomen with intestinal obstruction
d) Malignancy and mass passes per rectum goes in favour of-
2189. Treatment of choice of acute obstruction due to left a) Internal herniation b) Stricture (PGI 99)
sided colonic carcinoma in a 70 year old male is- c) Strangulated hernia d) Intussusception
a) Left sided colectomy (AIIMSC 97) 2202. What are the features of colonic obstruction-
b) Total colectomy a) No passage of gas absolutely (Obstipation)
c) Hartman's procedure b) No passage of stools absolutely (PGI 2K)
d) Defunctionung colostomy c) Distention of abdomen
d) Mild fever initially
2190. In colonic obstruction immediate treatment is
e) Fecal vomitus
requiredto prevent- (PGI 96)
2203. Recurrent obstruction, mass per rectum and
a) Caecal rupture b) Water imbalance diarrhoea in child- (PGI 2000)
c) Abdominal distension d) Severevomitting a) Intussusception b) Rectal prolapse
2191. Best way to diagnose lower intestinal obswtruction c) Internal hernia d) Haemorrhoids
a) Pain abdomen (PGI 96) 2204. In intestinal obstruction, investigations needed are-
b) Abdominal distension a) Bariumswallow (PG/01)
c) Profuse vimitting b) Intestinal barium meal
d) Multiple air gas shadows on X - ray c) Stomach barium meal
2192. For intestinal obstruction immediate operation d) ErectX-rayabdomen
should not be done in case of- (CULCUITA 2K) e) Supine X-ray abdomen
a) Post - op adhesion b) Appendix perforation 2205. Features of intussusception are- (PGI 01)
c) Volvulus d) Obstructed hernia a) Pincer sign b) Target sign
2193. A Neonate is brought with history of not having c) Dove sign d) Coiled spring sign
passed meconium on examination there is no anal e) Dance sign
opening but a dimple. Investigation of choice is- 2206. Malrotation presents as- (PGI 02)
a) Mass abdomen b) Bleeding PR
a) X-ray erect posture (JIPMER 90)
c) Billions vomiting d) Haematomesis
b) X-ray supine posture
2207. Acute intestinal obstruction is characterized by-
c) Gastrograffm study a) Vomiting is common in duodenal obstruction
d) Invertogram b) Pain after each attack of vomiting is characteristic
2194. What is intussuscepiens- (PG/97) of ileal obstruction (PGI 03)
a) The entire complex of intussusception c) In colonic obstruction distension is common than
b) The entering layer vomiting
c) The outerlayer d) X-ray erect posture is diagnostic
d) The process of reducing the intussusception e) Colicky pain to steady pain indicates strangulation
2195. Most common cause of acute intestinal obstruction- 2208. 30years old lady presented with acute pain abdomen,
a) Adhesions b) Carcinoma (PGI 97, 88) constipation and vomiting suspecting acute intestinal
c) Int. hernia d) Lymphoma obstruction. The investigation of choice for the
2196. Definitive treatment of sigmoid volvulus is-(PGI 97) patieutis- (PG/03)
a) Surgical correction b) Colectomy a) X-ray abdomen erect posture b) Ba enema
c) Enema d) Endoscopic correction c) USG d) CT scan

2186)d 2187)b 2188)c 2189)c 2190)a 219l)d 2192)a,b 2193)d 2194)c 2195)a 2196)b 2197)b 2198)c
2199)a 2200)a 2201)d 2202)a,b,c 2203)a 2204)b,c,e 2205)b,d,e 2206)a,c 2207)a,c,d 2208)a
SURGERY [ 286]

2209. Paralytic ileus is characterized by all except- 2219. True about Ogilive's syndrome are all except-
a) No bowel sounds on auseultation (SGPGI 05) a) It is caused by mechanical obstruction of the colon
b) No passage of flatus b) It involves entire I part of the large colon
c) Gas filled loops of intestine with multiple fluid c) It occurs after previous surgery (AIIMS Nov 07)
levels d) It occurs commonly after narcotic use
d) Loops of intestine are not seen d/t loss of peristalis 2220. A new born girl not passed meconium for 48 brs,
2210. Intestine get strangulated most commonly in which has abdominal distention and vomiting. Initial
space- (SGPGI 05) investigation of choice would be- (AIIMSNov07)
a) Omental bursa b) Paraduodenal space a) Manometry
c) Rectouterine space d) Subphrenic b) Genotyping for cystic fibrosis
2211. Gall stone causes intestinal obstruction when it gets c) Lower GI contrast study
impacted in which part of the intestine commonly- d) Serum trypsin immunoblot
a) Proximal ileum b) Distal ileum 2221. Which one of the following is the treatment of gall
c) Duodenum d) Jejunum (SGPGI 05) stone ileus- (UPSC-11 08)
2212. The most common cause of perforation of the distal a) Cholecystectomy alone
ileum in India is- (UPSC 05) b) Removal of obstruction
a) Tuberculosis b) Typhoid c) Cholecystectomy, closure of fistula and removal
c) Amoebiasis d) Regional enteritis of stone by enterotomy
2213. Borchart's triad of acute epigastric pain, violent d) Cholecystectomy with closure of fistula
retching and inability to pass a nasogastric tube is 2222. Which of the following statement about volvulus is
seen in patients with - (J & K 05) false- (AI 08)
a) Achalasia cardia a) More common in psychiatric patients
b) Acute gastric volvulus b) Sigmoid volvulus is more common than caecal
c) Jejunogastric intussusception volvulus
d) Hiatus hernia c) Lower GI scopy is contraindicated in sigmoid
2214. The most common site ofintestinal obstruction in volvulus
gallstone ileus is- (AIIMSMay 2005) d) Volvulus of caecum is managed by conservative
a) Duodenum b) Jejunum methods
c) lleum d) Sigmoid colon 2223. A new born child has not passed meconium for 48
2215. Features of intestinal obstruction : clinically/ brs. What is the diagnostic procedure of choice ?
investigation by- (PG1 June 06) a) USG b) Contrast enema (AI 08)
a) Abdominal distension b) Vomiting c) cr d)MRI
c) Fluid level in X-ray> 4 d) Localized tenderness 2224. Claw sign seen in? (APPG 08)
e) Diarrhea a) Intussception b) Volvulus
2216. With reference to Hirschsprung's disease, which c) Both d) None
one of the following statements is correct? 2225. Complete Rx ofintussception indicates ?(APPG 08)
a) It is initially treated by colostomy (UPSC 07) a) Free passage ofbarium in the terminal ileum
b) In the neonatal period, it is best confirmed by b) Passage of faceus and flatus along with barium
barium enema c) Improvement of clinical condition
c) It is associated with high incidence of genito- d) None
urinary tract anomalies 2226. All statements about adult instussusception are true
d) It is characterised by the absence of ganglion cells except- (PGINov09)
in the transverse colon a) Idiopathic & more enteric rather than colonic
2217. Acute pseudo-obstruction of the colon known as- b) Lead point present in majority of cases
a) Sjogren's syndrome (UP 07) c) Resection of bowel is adequate for large bowel
b) Gardner's syndrome intussusception
c) Ogilivie's syndrome d) Hydrostatic reduction with barium or air are done
d) Peutz-jegher's syndrome if bowel is not completely obstructed
2218. Sigmoid volvulus rotation occurs- (UP 07, 05) 2227. "Red currant jelly" stool in an infant is
a) Clockwise characteristic of- (UPSC-1 I 0)
b) Anticlock wise a) Acute gastroenteritis b Rectal piles
c) Both clock wise and anti clock wise c) Intussusception d) Fissure-in-ano
d) Axial in direction

2209)d 2210)a 2211)b 2212)a 2213)b 2214)c 2215)a,b2216)a 2217)c 2218)b 2219)a 2220)c 2221)b 2222)c
2223) b 2224) a 2225) a 2226) a 2227) c
SURGERY [ 287]

2228. Consider the following radiological signs- 2237. Mucocele of the appendix is- (AI 89)
1. Claw sign in barium enema (UP SC II 1 0) a) Benign tumour b) Low grade malignancy
2. Apple core sign in barium enema c) Retention cyst d) Infective process
3. Multiple fluid levels with absent caecal gas in plain 2238. ApatientwitbCrohn'sdiseasewasopenedforandan
skiagram of abdomen inflammed appendix found The treatment ofchoice is-
4. Single large fluid and air level in plain skiagram of a) Appendicectomy (PGI 88)
abdomen b) ileocolic resection and anastamosis
Which of the above signs is/are suggestive of c) Close the adbomen and start medical treatment
ileocolic intussusception? (UP SC II 10) d) None of the above
a) 1 and 2 only b) 1 and 3 only 2239. Which of the following present as acute abdomen-
c) 3 only d) 1, 2 3 and4 a) Acute intermittent porphyria (Karnataka 89)
2229. True about Intnssusceptions in children - b) Tabes
a) Most common variety is ileocolic (PGI May 1 0) c) Pneumonitis oflower lobe
b) A/w pathological lead point d) All
c) May be seen after viral infection 2240. Oschner sherren regime is used in management
d) Canbe relieved by barium enema of- (JIPMER92)
e) Surgery is always indicated a) Appendicular abscess b) Chronic appendicitis
2230. True statement about treatment ofintussnsceptions- c) Appendicular mass d) Acute appendcitis
a) Air enema (PGI May 1 0) 2241. When the rectum is inflated with air through a
b) Saline enema rectal tube, pain and tenderness occur in the right
c) Baenema iliac fossain case of appenidicitis. This is known
d) Hydostatic reduction as- (AIIMS 81, AMU 85)
e) Colonoscopy is always done to confirmdiagnosis a) Aaron's sign b) Battle's sign
2231. Trueabontcolonicvolvulus- (PGIMay 10) c) Bastede's sign d) Me Burney's sign
a) Most common in caecum 2242. The frequent mechanism in perforation of
b) Common in psychiatric pt. appendixis- (DNB 89, 91)
c) Bird's beak sign a) Impacted faecolith
d) May present as intestinal obstruction b) Tension gangrene due to the accumulating
2232. Emergency operation done in cases of- secretions
a) Volvulus (PGINov.10) c) Necrosis oflymphoid patch
b) Obstructed hernia d) Retrocaecal function
c) Appendicular perforation with paralytic ileus 2243. Acute appendicitis is due to- (AIIMS 90, AMU 90)
d) Toxic megacolon a) Faecoliths
e) Colonic perforation b) Worms of ileo-caecal region
2233. The cecum is found to be placed below the stomach c) Streptococcal infections
and in the midline. Which of the following d) Abuse of puragatives
abnormality most have taken place while rotation of e) None of the above
the gut? (AIIMS Nov 10) 2244. All of the following are early complications arising
a) Non rotation b) Mal rotation after appendicectomy for acute appendicitis except-
c) Reversed rotation d) Mixed rotation a) ilues (AIIMS 81, Rohtak 88)
b) Sterility
VERMIFORM APPENDIX c) Intestinal obstruction
d) Pulmonary complications
2234. Earliest symptoms in acute appendicitis is - 2245. All are usuful in acute appendicitis excpet-
a) Pain b) Fever (JIPMER 86, 87, a) Antibiotics b) Analgesics (Kerala 94)
c) ~miting d)Riseofpulserate Kerala87) c) IV Fluids d) Purgation
2235. Commonest position ofthe appendix- (.liPMER 87) 2246. Which of the following statments is not true of
a) Retrocecal b) Preileal McBurney's incsion- (Kam 94)
c) Postileal d) Pelvic a) Most suitable if the diagnosis of appendicitis is
e) Subcecal definite
2236. Surgery for carcinoid of the appendix- (AIIMS 85) b) If it is converted into a muscle cutting incision it
a) Righthemicolectomy is called Rutherford Morison's incision
b) Appendicectomy c) Inguinal hernia is a sequlae of the incision
c) Limitedresectionofthe Rt. Colon d) The incision can be extended upwards or
d) Rt. Hemicolectomy with 6 inches of the ileum also downwards

2228) b 2229) a,b,c,d 2230) a,b,c,d 2231) b,c,d 2232)All 2233) b 2234) a 2235) a 2236) a,b 2237) a,b,c 2238) c
2239)d 2240)c 2241)c 2242)b 2243)a,b 2244)b 2245)d 2246)d
SURGERY [ 288]

2247. A Gridiron incision becomes a Rutherford RECTUM


Morison's incision is extended by- (Karn 94)
a) Splitting the muscles laterally 2257. Prognosis for carcinoma rectum is best assessed by-
b) ·cutting the muscles laterally a) Site of tumour (AllMS 87, Karn. 89)
c) Cutting the muscles medially into the rectus sheath b) Histological grading
d) Incising vertically along the rectus muscle c) Size of tumours
2248. During appendicectomy if it is noticed that base d) Duration of the symptoms
of appendix is inflammed than further line of 2258. Earliest symptoms of carcinoma rectum- (AI 88)
treatment is- (PG196)
a) Pain b) Alteration of bowel habits
a) No appendicectomy b) No hurrying of stump
c) BleedingPR d) Tenesmus
c) Hemicolectomy d) Caecal resection
2249. A 25 year old man presents with 3 days history of 2259. Villouspolypofrectummanifest- (AI89)
pain in the right lower abdomen and vomitings. a) BleedingPR
patient's general condition is satisfactory and b) Mucus diarrhoea with hypokalemia
clinical examination reveals a tender lump in right c) Prolapserectum
iliac fossa. The most appropriate management in d) Obstruction
this case would be- (JCS 98) 2260. Recurrent prolapse of the rectum in children is
a) Immediate appendicectomy treated by- (JIPMER 87, 88)
b) Exploratory laprotomy a) Thiersh wiring b) Digital reposition
c) OschnerSherrenregimen c) Excision d) Ripstein's operation
d) External drainage 2261. In villous papillomas ofthe rectum which is lost-
2250. An appendicular fistula is least likely to heal if- a) Na+ b) Mg+ (TN 89)
a) The stump was sutured with vicryl (Kerala 98) c) K+ d)All
b) There is stenosis/narrowing of the sigmoid colon 2262. Arterior resection is contra indicated in the
c) Superaded infection following- (PGJ90)
d) None · a) Age more than 60 years
2251. What should be done for an incidental carcinoid b) Undifferentiated carcinoma
of2.5 ems size detected in the appendix- (AJJMS 99) c) Melanin in liver
a) Righthemicolectomy d) Cancer is less than 5 em from anorectal margin
b) Appendicectomy 2263. Resting Tone ofRectum is decreased in all except
c) Appendiccectomy followed by yearly HIAA a) Micturation (AJ91)
estimation b) Retained feaces in the rectum
d) Total colectomy c) Prolapserectum
2252. Diffuse peritonitis following appendicitis is usually d) Trauma involving the perineum
seen - (JCS 2K) 2264. Best procedure in mid rectal Carcinoma is-
a) When appendicular perforation occurs early a) Abdominoperineal resection (AllMS 92)
(within 24 hours) b) Anterior resection
· b) When perforation occurs late (after 24 hours) c) Perineal loop
c) Particularly in non-obstructive appendicitis d) Transverse Colostomy
d) When antibiotics are withheld 2265. The best surgical management for villous adenoma
2253. All of the following signs are not seen in acute of the rectum is- (PGJ81, AP 88)
appendicitis except- (TNOJ) a) Local resection oflesion
a) Rovsing's b) Murphy's sign b) Repeated sigmoidoscopy
c) Boa's sign d) Mack wen's sign c) Abdominoperineal resection
2254. Confirmation of appendicitis - (PGI June 07)
d) Electrolyte infusion and chemotherapy
a) Clinical examination b) USG abdomen
2266. Treatment of carcinoma rectum 5 em from anal
c) X-ray abodmen d) Raised 1LC/DLC
verge without nodal metastasis is- (Kerala 94)
2255. In a case of retrocecal appendicitis which movement
a) Abdominoperineal resection
aggravates pain- (AIJMS Nov 07)
b) Radiotherapy
a) Flexion b) Extension
Medial rotation Lateral rotation c) Endoscopic resection
d) Chemotherapy
2267. Sitz Bath consists of which of the following-
a) Patient bathed in normal saline (Karan. 96)
b) Bathed in molten wax
c) Sits in a basin containing warm antiseptic lotion
d) Sits in a basin conatining molten wax

2247)b 2248)b 2249)c 2250)b 2251)a 2252)a 2253)a 2254)~b2255)b 2256)a 2257)b 2258)c 2259)b 2260)d
2261) c 2262) c,d 2263) b 2264) b 2265) a 2266) a 2267) c
SURGERY [ 289]

2268. Rectal polyps usually present with- '(UP 97) 2280. Rectal incontinence is due to involvement of-
a) Obstruction b) Perforation a) External anal sphincter (PGI June 06)
c) Bleeding d) Malignant change b) Internal anal sphincter
2269. Solitary rectal ulcer all are true except-(AJ/MS 97) c) Ischiococcygeus
a) 20% have multiple ulcers d) Pubococcygeus
b) Most common site is midline posterior 2281. Which of the following is more aggressive rectal
c) Half ofpatients give history of digital examination carcinoma- (ManipalO~
d) Commonest cause of rectal prolapse a) Adencarcinoma
2270. In carcinoma of anus distal margin of clearance b) Secondary mucoid carcinoma
ofanalcanalofatleast ............ - (CMC 2001, c) Signet ring carcinoma
a) 2cm b)5cm AJ90) d) Squamous cell carcinoma
c) 4cm d)7cm 2282. True about solitary rectal ulcer syndrome is all/
2271. An AIIDS patients presents with fistula-in-ano. except- (Aiims May 07)
His CD4 count is below 50. What is the treatment a) Increased muscle layer proliferation
of choice- (MAN/PAL OJ) b) Crypt distortion
a) Seton b) Fistulectomy c) Lamina propria infiltration with lymphocyte
c) None d) Both d) Subepithelial fibrosis
2272. Muscle concerning with rectal continence 2283. Hemangioma of the rectum- (PGI June 07)
is- (PG/86, UPSC 86, NIMHANS 87, Kerala 88) a) Common tumour
a) Ext. sphincter b) Int. sphincter b) Fatal haemorrhage seen
c) Puborectalis d) Sacrococcygeous c) Ulcerative colitis like symptoms seen
e) iliococcygeous 2284. A punch biopsy shows carcinoma rectum with fixed
2273. Sphincter saving surgery for rectal malignancy is mass. X-ray chest normal, which of the following is
notdonein- (PG/01) LEAST useful investigation- (UP 08)
a) Age over 50 years a) Rigid protoscope b) Barium enema
b) Lymph node involvement c) CT chest d) MRI-abdomen and pelvis
c) Infiltration oflamina propria 2285. Management of rectosigmoid obstructive carcinoma
d) More than 4 em from anal verge in elderly frail- (PGI Nov 09)
e) High grade tumor a) Colostomy
2274. Abdomino-perineal resection is done in colorectal b) Abdomino-perineal resection (APR)
Caon the basis of- (PG/02) c) Resection & primary anastomosis
a) Age of patient b) Distance from anal-verge d) Hartmann procedure
c) Fixity oftomor d) Hepatic metastasis e) Laserrecanalisation
e) Extentoftumor 2286. A young male patient presents with complete rectal
2275. Rectal adenoma is associated with- (Jipmer 03) prolapse. The surgery of choice is- (AI 10)
a) Familial polyposis coli b) Hypokalemia a) Abdominal rectopexy
c) Intussusception d) Hemorrhoids b) Delerome's procedure
2276. Delorme's Procedure is used for- (SGPGI 04) c) Anterior resection
a) Rectal prolapse b) Solitary rectal ulcer d) Goodsall's procedure
c) Rectal bilharziasis d) Proctalgia fugax 2287. A young office executive, on tour, presents with
2277. The length ofa standard proctoscope is- (TN 04) bright red painless bleed since 7 days and abdominal
a) 4 inches b) 6 inches pain. External and perrectal examinations are
c) 8 inches d) 3 inches normal. As attending general practioner what will
2278. Which type of malignancy is found in anorectal be your next step- (AIIMS Nov 10)
fistula- (PGI June 05) a) Proctoscopy
a) Sq. cell Ca b) Transitional cell Ca b) Do a barium enema
c) Adeno Ca d)ColumnarCa c) Refer to surgeon for sigmoidoscopy
2279. TrueaboutrectalCa- (PGIJune05) d) Refer to gastroenterologist for colonoscopy
a) MC type adenocarcinoma
b) Surgery is the treatment of choice ANAL CANAL
c) Surgical treatment indicated inspite of hepatic
metastasis 2288. Commonest type ofAno rectal abscess is-
d) APR done in lesions of upper zone a) Ischio rectal b) Submucous (AIIMS 85)
c) Pelvi-rectal d) Perianal

2268)c 2269)b,c 2270)a 227l)a 2272)c 2273)d 2274)b 2275)b 2276)a 2277)a 2278)a,c 2279)a,b,c
2280)All 228l)b 2282)c 2283)b,c 2284)a 2285)a,b,e 2286)a 2287)a 2288)d
SURGERY [ 290]

2289. Sentinel pile indicates- (AJIMS 87) 2303. It can be stated that the superior haemorrhoidal
a) Carcinoma rectum b) Internal haemorrhoids veins- (AJIMS84)
c) Perianal fistula d) Anal fissure a) Drain into the inferior mesentric vein
2290. Cancer ofthe anus is commonly- (JIPMER 87) b) Have no valve
a) Adenocarcinoma b) Squamous carcinoma c) Leave the anal canal at the pactinate line
c) Melanoma d) Sarcoma d) Cause external haemorrhoids
2291. Below the pectineal line the lymphatic spread is 2304. The increased incidence of infection in the Ischio
to .....nodes- (PGI 89) rectal fossa is due to- (AI 89)
a) Superficial inguinal b) Internal iliac a) Absence of deep fascia
c) External iliac d) Para aortic b) Proximity to anus
2292. Rectal examination should not be done- c) Poor blood supply
a) Anal fissure (JIPMER 90) d) Presence of fibrofatty tissue
b) Fistual in ano 2305. Anal carcinoma is most commonly carcinoma of
c) Prolapsed piles with bleeding type- (PGI 97)
d) Anal stenosis a) Adeno carcinoma b) Epidermoid
2293. Treatment of Choice in 2nd degree piles c) Mixed d) None of the above
is - (AJIMS 92) 2306. True about melanoma of the anal canalis -(PGI 99)
a) Cryosurgery b) Sclerotherapy a) Present usually as anal bleeding
c) Banding d) Surgery b) AP resection gives better result than local excision
2294. The treatment of choice in fistula in ano- (Jipmer c) Local recurrence at the same site after resection
a) Anal dilatation b) Fissurotomy 93) d) Radiosensitive
c) Fistulectomy d) Fistulotomy 2307. Most common complication following
2295. Internal sphincterotomy is the treatment of choice hemorrhoidectomy is- (Kerala 04, AJIMS 92)
for- (JIPMER8l,AMC86)
a) Hemorrhage b) Infection
a) Piles b) Fistula c) Fecallmpaction d) Urinary retension
2308. A 1 em x 1 em squamous cell carcinoma of anal
c) Fissure-in-ano. d) Carcinoma
canal is best treated initially by- (UPSC 05)
2296. Internal sphincter of rectum is formed by-
a) Abdominoperineal resection
a) Levator ani (AIIMS 79, Delhi 96)
b) Localized resection followed by irradiation
b) Puborectalis
c) Proximal colostomy followed by interstitial
c) Longitudinal muscle fibres condensation
irradiation
d) Circular muscles fibres condensation d) Chemo-radiotherapy
2297. Treatmentofprimarypilesis- (Kerala 94)
2309. The treatment of choice for the management of
a) Surgery b) Sclerotherapy carcinoma of the anal canal is- (AI 06)
c) No treatment d) Analgesics a) Abdominoperineal resection
2298. Best investigation to diagnose piles is- (Kerala 94) b) Primary radiotherapy
a) Proctosigmoidoscopy b) Barium enema c) Combined radio-and chemotherapy
c) Ultrasound d) Proctoscopy d) Neoadjuvant chemotherapy and local excision
2299. Ideal treatment for squamous cell carcinoma of 2310. The treatment of choice for squamous cell anal
Anal canal is- (AIIMS 97) cancer is- (AI 06)
a) Pelvic exentration a) Abdominoperennial resection
b) Chemotherapy and radiotherapy b) Laser fulgaration
c) Chemotherapy alone c) Chemoradiotherapy
d) Abdomino perineal resection d) Platinum-based chemotherapy
2300. Five-day self subsiding pain is diagnostic of- 2311. Which of the following statement is true about
a) Anal fissure (AP 97) external haemorrhoids? (AI 07)
b) Fistul-in-ano a) Painful, below the dentate line
c) Thrombosed external hemorrhoids b) Painless, above the dentate line
d) Thrombosed internal hemorrhoids c) Rubber band ligation is the treatment of choice
2301. Invertogram is taken after- (2000 R) d) May be associated with a skin tag
a) 2 hrs b) 4 hrs 2312. The following statement about Pilonidal sinus is
c) 6 hrs d) 8 hrs true- (AIO~
2302. The following are true of haemorrhoids except- a) More common in females
a) They are arteriolar dilatations (JIPMER 01) b) Mostly congenital
b) They are common causes of painless bleeding c) Prognosis after surgery is poor
c) They cannot be per rectally palpated d) Treatment of choice is surgical excision of sinus
d) They can be banded tract

2289)d 2290)b 229l)a 2292)a 2293)c 2294)d 2295)c 2296)d 2297)a,b2298)d 2299)b 2300)c 230l)c 2302)a
2303)a,b 2304)c 2305)b 2306)a 2307)d 2308)d 2309)c 2310)c 23ll)a 2312)d
SURGERY [ 291 ]

2313. Most common cause of anorectal abscess is- 2322. Treatment of choice in desmoid tumours is-
a) Inflammation of anal gland (MAHE 07) a) Irradiation (AIIMS 85)
b) Folliculitis b) Wide excision
c) Inflammation of rectal mucosa c) Local excision
d) Rectum d) Local excision following radiation
2314. IDgh or low fistula in a no is termed according to its 2323. Recurrent fibroma refers to Desmoid tumor
internal opening present with reference to - arising in - (PGI 88)
a) Anal canal b) Dentate line (UPSC-II 08) a) Uterus b) Scar tissue
c) Anorectal ring d) Sacral promontary c) Ovary d) Muscle
2315. AnaJnssure best diagnosed by- (AI 08) 2324. Meckel's diverticulum is (JIPMER 81, Rohatak 87)
a) Anoscopy a) Ductus venosus
b) History and superficial clinical examination b) Vitello intestinal duct
c) PRexamination c) Left umbilical vein
d)USG d) Obliterated umbilical artery
2316. Which of the following statements is true for Nigro's 2325. Slid ding constituent of a large direct hernia is-
regimen - (AI 08) a) Bladder b) Sigmoid colon (AI 88)
a) It is a regimen for anal canal neoplasm c) Caecum d)Appendix
b) It incorporates chemotherapy with radiation as an 2326. Commonesttypeofherniais- (JIPMER 87)
alternative to surgery a) Femoral b) Inguinal
c) Has the advantage of preserving continence c) Ventral d) Epigastric
d) All of the above e) Hiatus
2317. A new born baby presents with absent anal orifice 2327. Commonest type of hernia in a female is-
and meconuria. What is the most appropriate a) Femoral b) Inguinal (AIIMS 85)
management? (AI 08) c) Ventral d) Hiatus
a) Transverse colostomy 2328. Burst abdomen most commonly occurs on tbe-
b) Conservative a) 2nd day b) 3rd day (JIPMER 86)
c) Posterior Saggital Anorectoplasty c) 7th day d) 9th day
d) Perenial V-Yplasty e) 5th day
2318. Most common cause of anorectal abscess is- 2329. Most important step in the repair of an indirect
a) Inflammation of anal gland (Manipal 08) inguinal hernia is- (JIPMER 87)
b) Folliculitis a) Herniotomy
c) Inflammation of rectal mucosa b) Narrowing of the internal ring
d) Rectum c) Bassini's repair
2319. All of the following are true in management of d) Transfixation of the neck of the sac
Haemorrhoidsexcept- (DELHIPGMar. 09) 2330. The covering over an omphalocele is- (PGI 87)
a) Excisional surgery is cornerstone a) Skin b)Amnioticmembrane
b) Fiber supplementation is effective c) Chorionic membrane d) None of the above
c) Improvement in bowel function is helpful 2331. The superficial inguinal ring is a defect in-
d) Ligation with rubber bands effective a) Transversalis fascia (UPSC 88)
2320. Trne about treatment ofhemorrhoids-(PG/May 10) b) Internal oblique
a) Band ligation c) External oblique
b) 5% phenol in almond oil is used as sclerosant d) External oblique aponeurosis
c) May be resolved by diet modification 2332. Femoral hernia is characteristically ••• the pubic
d) Hemorrhoidectomy is TOC tubercle- (1N 89)
a) Lateral and below b) Medial and above
HERNIA c) Lateral and above d) Medial and below
2333. Causes of recurrent hernia- (PGI 88)
2321. Rgarding desmoid tumour which is not correct- a) Absorbable sutures b) Sliding hernia
a) Often seen below the umbilicus (JIPMER 87) c) Missed sac d) Infection
b) Unencapsulated 2334. In hernia-en-glissade, the most common content
c) More common in women is- (AIIMS89)
d) Metastasis does not occur a) Omentum b) Urinary bladder
e) Highly radiosensitive c) Cecum d) Sigmoid colon

2313)a 2314)c 2315)b 2316)d 2317)a 2318)a 2319)a 2320)All 2321)e 2322)b 2323)b 2324)b 2325)a
2326)b 2327)b 2328)c 2329)b 2330)b 2331)d 2332)a 2333)a,b,c,d 2334)d
SURGERY [ 292]

2335. Vestigeal remnants of vitello intestinal duct 2347. A patient is advised to avoid strenous activity
include- (JIPMER 90) following herniorrhaphy for a period of-
a) Meckel's diveticulum a) One day b) One week (AMU 86, 91)
b) Enterocyst c) 3 weeks d) 6 weeks
c) Enteroteratoma at umbilicus 2348. On an average, the distance between femoral ring
d) All and saphenous opening (length offemoral canal) is-
2336. Exomphalos is a disease involving- (Keala 91) a) 1.25cm b)2.50cm(AJIMS81, UPSC87)
a) Umbilicus b) Cervix c) 3.75cm d)5.00cm
c) Abdominal wall d) Urinary bladder 2349. Patent vitello-intestinal duct should preferably be
2337. Which of the following does not predispose to operated at- (PGI 80,81, AMU 89)
abdominal wall dehiscence except- (JIPMER 92) a) Birth b) 6 months of age
·a) Faulty technique c) 12monthsofage d) 3 yearsofage
b) Malignancy 2350. Which age group most often presented with
c) Raised intra abdominal pressure jaundice due to Omphalitis in infants is- (PGI 81,
d) Old age a) At birth b) 24-72 hours Kerala 90)
2338. Strangulation is most common with which hernia- c) 1-3 weeks d) 3-6 weeks
a) Femoral b) Inguinal (A/93) 2351. The patent vitello-intestinal duct most often
c) Obturator d) Epigastric discharges- (JIPMER 78, 79, TN 89)
2339. Treatment of choice of umbilical adenoma in a new a) Mucus b) Pus
born in- (PGI 80, Delhi 81,93) c) Urine d) Faeces
a) Occlusion with a coin b) Strapping 2352. Treatment of strangulated hernia is- (Kerala 94)
c) Surgery d) Masterly inactivity a) Observation b) Immediate surgery
2340. "Raspberry tumour" is another name for- c) Manual reduction d) Analgesics
a) Umbilical fistula (PGL BHU 85) 2353. Which is the 1st sign of strangulation of ingunial
b) Umbilical granuloma hernia- (CUPGEE 96)
c) Umbilical adenoma a) Tense b) Tender
d) Meckel's diverticulum c) Irreducible d) Redness
2341. Hernia with hydorcele is •••• hernia- (PG/79, 2354. The hernia which often stimulates a peptic ulcer
a) Gibbon's b) Fruber's AJIMS84) is - {Karn. 96, 94)
c) Dobson's d) Leobel's a) Umbilical hernia
2342. In Moore's classification of omphalocole b) Fatty hernia of the Linea alba
(examphalos), type I unbilical defect is less than c) Incisional hernia
•••• em.- (PGI 81, AIIMS 86) d) Inguinal hernia
a) 0.5 b)2.5 2355. Preservation of illioinguiual nerve is an important
c) 3.5 d)4.5 step inguinal hernia operation while- (UPSC 97)
2343. Henia into pou,ch of Douglas is ••.•• hernia- a) Incising the sub-cutaneous tissue
a) Bedard's b) Bochdaleks (AIIMS 81, b) Incising the extra oblique aponeurosis
c) Blandin's d) Berger's PGI 84) c) Incising the cremasteric fascia
2344. The treatment of choice from inguinal hernia in d) Isolating the sac
infants is- (AMU 85) 2356. While operating for obstructed inguinal hernia
a) Herniotomy b) Herniorrhapy the sac is opened at- (AJIMS 96)
c) Truss d) Hernioplasty a) Fundus b) Neck
2345. Exomphalos major should be operated at- (PGI 81, c) Body d) Base
a) Birth b) 3 months of age DNB 91) 2357. Raspberry tumour is- (JIPMER98)
c) 1 year d) 3 years a) Neoplastic b) Inflamatory
2346. In examination of the patient for a hernia, it is c) Traumatic d) Congenital
usefulto realize that- (AIIMS 81, PGI 83) 2358. Hernia is prevented by- (ROHTAK98)
a) An impulse if often much better seen than felt a) Scarpa's fascia b) Transversalisa fascia
b) The internal abdominal ring lies 1.25 em. above c) External oblique d) Lacunar ligament
the midpoint ofpoupart's ligament e) Rectus sheath
c) The external abdominal ring lies 1.25 em above 2359. Spigelian hernia occurs- (A/99)
and external a) Para umbically b) Supra umbically
d) None of the above c) Infraumbically d) At lumbar areas

2335)d 2336)c 2337)a,b,c 2338)a 2339)None 2340)c 2341)a 2342)b 2343)d 2344)a 2345)a 2346)b2347)None
2348)a 2349)b 2350)d 235l)a 2352)b 2353)a 2354)b 2355)b 2356)c 2357)d 2358)None 2359)c
SURGERY [ 293]

2360. Inguinal and femoral hernia are differentiated by 2371. True about inguinal hernia- (PGI 03)
relation with- (AIIMS 98) a) It is more common in female
a) Pubic tubercle b) Femoral artery b) Rt sided is more common than Lt side
c) Inferior epigastic artery d) Pectineal line c) Direct hernia is less likely to undergo strangulation
2361. Truss cannot prevent progression of which type d) Femoral hernia is more common in female
ofinguinal hernia- (CUPGEE 99) 23 72. True statment (s) about indirect inguinal hernia-
a) Sliding b) Littres a) 25% is bilateral (PGI 04)
c) Indirect d) Direct b) In children, if inguinal (indirect) hernia is present
2362. Ventral hernia is alan- (AMC 99) in on side, then processus vaginalis is intact on
a) Incisional hernia b) Umbilical hernia other side
c) Femoral hernia d) Inguinal hernia c) In bubonocele sac lies in the inguinal canal
2363. Mayo's operation is done for- (2000 R) d) Equal incidence in male and female
a) Spigelian hernia b) Femoral hernia 2373. In a case of strangulated hernia management
c) Richer's hernia d) Umbilical hernia is - (PGI June 06)
2364. Hernia through foramen of bochdalek true a) USG-abdomen b) X-ray abdomen
is- (PG/98) c) Aspirate contents d) Immediate surgery
a) Congeniatal hernia b) In asymptomatic e) IVfluids
c) Seen executive in males d) Least common 2374. Which is one ofthe following is not performed in
2365. True about femoral hernia is- (PGI 98) Lichtenstein tension free hernioplasty? (Karn 06)
a) Occurs exclusively in females a) High ligation of indirect hernia sac
b) Pregnancy is common cause b) Mesh sutured to the conjoint tendon and inguinal
c) Doesn't strangulate ligament
d) In males associated with cryptochidism c) Conjoint tendon sutured to inguinal ligament
2366. Spigelian hernia- (PGI 2000) d) Spermatic cord is placed in two tails of the internal
a) Lateral to rectus abdominis ring
b) Obturator internus 2375. In the treatment of femoral hernia Lockwood's
c) Lies above int. oblique
operation refers to- (Karnataka pgmee 06)
d) Common in men
a) Low operation
2367. Which of the following is not to be done in
b) High operation
strangulated inguinal hernia- (PGI OJ, 03)
c) Inguinal operation
a) X-ray abdomen
d) Laparoscopic surgery
b) USG scrotum
c) Aspiration of sac for confirming diagnosis 2376. Umbilical hernia in a child- indication for sruger
d) Immediate surgery is required is - (MAHE 07)
e) Hypovolemia should be corrected with i./v. fluids a) Failure to disappear by 3 years
2368. Which of these would you like to do for a case of b) >2cmsize
strangulated hernia- (PGI 02) c) Symptomatic
a) X-ray abdomen d) Ail ofthe above
b) USG abdomen 2377. The sac contains only a portion of the
c) Aspiration of contents of sac circumference of the intestine is called-(UP 07, 05)
d) Correction ofhypovolemia a) Richter's hernia b) Littre's hernia
e) Prepare OT for urgent surgery c) Spigelian hernia d) Lumber hernia
2369. True about hernia- (PGI 03) 2378. True statement regarding direct inguinal
a) Extra-abdominal hernia is common hernia- (UP 08)
b) Direct hernia usually acquired a) Most common direct inguinal hernia in women
c) Strangulation is common in femoral hernia b) Direct hernia medial to inferior epigastric artery
d) Direct hernia is acquired in old age c) Repair ofthe transvers alis fascia and the internal
e) TOC for indirect inguinal hernia is surgery ring
2370. About hernia, false statements- (PGI 03) d) Descends downwards and inwards towards the
a) In children, indirect inguinal hernia is treated scrotum
medically
2379. Most useful investigation in sliding hernia in
b) In Richter's hernia, absolute constipation seen
female- (UP08)
c) Indirect inguinal hernia is the MC type
a) Fluroscopy b)Barium-meal
d) Deep inguinal ring is lateral & above the puplic
c) Palpation method d) Ultrasound
tubercle

2360)a 2361)a 2362)a 2363)d 2364)a 2365)b 2366)a 2367)a,b,c 2368)d,e 2369)All 2370)a,b,d 237l)b,c,d
2372)b,c 2373)d,e 2374)c 2375)a 2376)d 2377)a 2378)b 2379)b
SURGERY [ 294]

2380. Internal oblique, external oblique and transverses 2391. What is oliguria- (JIPMER 87)
is retracted laterally in- (Aiims May 08) a) Excretion ofless than 300ml in 24 hrs
a) Classic renal approach b) Excretion ofless than 500ml in 24 hrs
b) Laparoscopic approach c) Excretion ofless than 300 ml. in 12 hrs
c) Spigelian hernia d) Excretion ofless than 100 ml. in 24 hrs
d)----- 2392. Normal capacity of the renal pelvis is- (AJIMS 87,
2381. Which one of the foDowing is the treatment of choice a)7ml. b)lOml CMC86)
in a child with inguinal hernia- (UPSC-II 09) c) 15ml d)20ml
a) Bassin's repair b) Shouldice operation 2393. Acute onset of anuria in elderly men- (AI 89)
c) Herniotomy alone d) Lichtenstein repair a) Bilateral infarction of kidneys
2382. Thue about obturator hernia in adults- b) Obstructive urinary disease
a) More common in space oflorentz (PGI Nov 09) c) Acute tubular necrosis
b) Common in female d) Acute cortical necrosis
c) Chronic constipation risk factor 2394. Urinary incontinence results from all except-
d) Surgical treatment should be done a) Neurogenic bladder (AMU 88)
e) May present with intestinal obstruction b) Vesico vaginal fistula
2383. False aboutparaduodenal hernia- (PGI Nov 09) c) Ectopic ureter
a) Congenital d) Rectovesical fistula
b) Found in fossa ofKolb 2395. Urinary cytology is a useful screening test for the
c) Found in fossa ofLandzert dignosis of- (AIIMS 84)
d) Common in right side a) Renal cell carcinoma b) Wilm's tumour
2384. Most important prognostic factor in congenital c) Urothelial carcinoma d) Carcinoma prostate
diaphragmatic hernia- (AIIMS Nov 09) 2396. Urinary tract infection exists when the bacterial
a) Pulmonary hypertension b) Size ofhernia count in 1 mi. of midstream specimen of urine is-
c) Timing of surgery d) Gestational age a) 100 b) 1000 (AIIMS84)
2385. Most common site ofMorgagni hernia -(AIIMS May c) 10' d) 105 orover
a) Right anterior b) Right posterior 10) 2397. Majority of primary infection of the urinary tract
c) Right anterior d) Left posterior are caused by- (AIIMS 84, AP 85)
2386. Strangulation most commonly occurs a) Strep fecalis b) E. coli
in hernia- (Maharashtra 10) c) Proteus d) Pseudomonas
a) Ritcher's b) Spigelian 2398. Isotope Renogram- (Karn. 94)
c) Femoral d) Sliding a) Study of renin mechanism
2387. What is the most common hernia in females? b) Contrast study of kidneys, ureter and bladder
a) Femoralhernia (UPSCII 10) c) Utilised in mapping the anatomy of kidneys
b) Spigelian hernia d) Graphic representation of radioactivity ofkidneys
c) Inguinal hernia 2399. On exertion urine stream increases in- (AP 96)
d) Obturator hernia a) Prostate enlargement b) Marion's disease
2388. True statement about inguinal hernia- c) Post. urethral valves d) Urethral stricture
a) Pain indicate obstruction (PGI May 10) 2400. The most reliable urine specimen is obtained by-
b) Direct common in young male a) Urethral catheterization (UPSC 05)
c) Spigelia hernia occur through rectus sheath b) Catheter aspiration
d) Direct is more common c) Midstream voiding
e) More common on left side d) Suprapubic aspiration
~~~?,~ ·.~~·· tijle:t\~;li~ffii~ ccul'.,ifJg,"·~·h •· 2401. An elderly male presents with one episode of gross
: ; ....· . ··. ·.. X •...·. •F(JJ:June 2000) haematuria. All fo the following investigations are

' <''~~~~,~~~',!'I'~>Nct:;
recommended for invastigating this patient except-
a) Cystoscopy · (AI 07)
b) Urine microscopy for malignant cells
;'a)"·'~o:Ugli.Hrre~atwaM o~·mgljtitlaliiianat.·· z· •
;<" •.· •·' c) Urinetumormarkers
d) Intravenous pyelogram
URINARY TRACT (General) 2402. A 60 yr old smoker came with a history of painless
gross hematuria for one day. Most logical
2390. Acute urinary retention in a male child may be due to- investigation would be- (AI 07)
a) Prostatic radiotherapy (AIIMS 87) a) Urine routine
b) Urethral stricute b) Plain X ray KUB
c) Hysteria c) USGKIJB
d) Meatal ulcer with scabbing d) Urine microscopy for malignant cytology

2380)? 2381)c 2382)b,c,d,e 2383)d 2384)a 2385)a 2386)c 2387)c 2388)a 2389)b 2390)d 2391)a 2392)a
2393) b 2394) d 2395) c 2396) d 2397) b 2398) d 2399) d 2400) d 2401) c 2402) d
SURGERY [ 295]

2403. A new born presents with discharge ofnrine from 2413. Dormia basketing is used for removal of renal
the nmbilicns for 3 days. Diagnosis is- (UP 08) calculi in the- (JIPMER 85)
a) Meckel'sdiverticulum b) Mesenteric cysts a) Pelvic ureteric juction b) Upper 1/3 of ureter
c) Urachal fistula d) Omphalocele c) Middle 1/3 ofureter d) Lower 1/3 of ureter
2404. A new born presents with mid anterior abdominal 2414. EarliestsymptomofWilm'stumour(JJPMER86)
wall defect with characteristic spontaneous a) Hematuria b)Pyrexia
disappearance at age of 4 years- (UP 08) c) Abdominal mass d) Metastases
a) Patent urachus b) Omphalocele 2415. Renal tuberculosis is characterised by- (TN 89)
c) Ectopia vesicae d) Umbilical hernia a) Loinpain
2405. Most common cause of umblicus does not seperate b) Painful hematuria
at age of2 years- (UP 08) c) Sterile acid pyuria
a) Raspbery tumour d) Constitutional symptoms
b) Leukocyte adhesion deficiency 2416. The triad ofWilm's tumour is- (PGI 88)
c) Patent urachus a) Hematuria b) Mass abdomen
d) Umblical granuloma c) Pain d) Fever
2406. A six yr. old female presents with constipation and e) Weight loss
urinary reten~on. On examination a presacral mass 2417. Polycystic kidneys can be associated with (PGI 84)
is noted. Most probable diagnosis is - a) Cysts in liver lungs b) Coarcctation of aorta
a) Pelvic neuroblastoma (Aiims May 08) c) Berry aneurysms d) All
b) Rectal duplication cyst 2418. A patient with alkaline urine which is cloudy with
c) Sacrococcygeal teratoma plenty of pus cells is suffering from ......infection -
d) Anterior sacral meningocele a) E. coli b) Proteus (Kerala 89)
c) T.B. d) None
KIDNEY & URETER 2419. Ideal approach for renal malignancy is- (AIIMS89)
a) Transperitoneal
2407. Radio-luscent renal stones are composed of- b) Retroperitoneal
a) Cysteine b) Calcium phosphate (PGI 85) c) Lumbar incision
c) Uric acid d) Xanthine d) Abdominothoracic incision
2408. Commonest type of cancer of the renal pelvis and 2420. What is the following is Radiolucent- (AI90)
upper ureter is- (NIMHANS 87) a) Calcium phophae b) Calcium oxalate
a) Transitional cell carcinoma c) Uric acid d) Cysteine
b) Adenocarcinoma 2421. AU are true about Wilm's tnmonr except- (AI97)
c) Squarmous eel carcinoma a) Fever and weakness are clinical features
d) Nephroblastoma b) Arises from Primitive cells
2409. Spider leg appearance in IVP is suggestive of- c) Haematuria almost always present
a) Renal cyst (AP 84, Kerala 86, 88, d) It presents as abdominal mass
b) Renal carcinoma JIPMER 87) 2422. Polycystic kidney may be assocaited with cyst in
c) Renal Tb allthesitesexcept- (AI91)
d) Hydronephrosis a) Lung b) Liver
e) Chronic renal failure c) Pancreas d) Brain
2410. Percentage of renal stones which are radio 2423. Good prognosis in Wilm's tumour is seen in-
opaque- (JIPMER 86) a) 2-5 years b) Less than 1 year (Kerala 91)
a) 100/o b)25 c) Male child d) Female child
c) 37% d)75% 2424. Treatment of choice in polycystic kidney
e) 90% is - (Kerala 91)
2411. Staghorn calculus is made out of- (UPSC 97) a) Removal of cyst b) Nephrectomy
a) Oxalate b) Phosphate c) Dialysis d) Renal transplant
c) Uric acid d) Cystine 2425. Commonest presentation of bilateral ureteric
2412. Most common congenital anomaly of the upper stones- (AIIMS91)
renaltractis- (JIPMER 87) a) CRF b)UTI
a) Duplication of renal pelvis c) Pain d) Hematuria
b) Duplication of ureter 2426. Incidence of Renal ectopia is- (AI 92)
c) Ectopic ureteric orifice a) 1:100000 b) 1:75000
d) Congenital megaureter c) 1:10,000 d) 1:1000

2403) c 2404) d 2405) b 2406) d 2407) c,d 2408) a 2409) a 2410) e 2411) b 2412) a 2413) d 2414) c 2415) c
2416)a,b,d 2417)a,c 2418)b 2419)a 2420)c 242l)c 2422)d 2423)b 2424)None 2425)c 2426)d
SURGERY [ 296]

2427. The incidence of Liver cysts in childhood 2438. Most common of congenital abnormality of
polycystic kidney disease is - (AI 92) upper renal tract- (PGI 79, AIIMS 81, 92)
a) 5% b)lO% a) Duplication of Pelvis b) Duplication of ureter
c) 18% d)50% c) Horse shoe kidney d) Post caval ureter
2428. All are trne regarding Wilm's tumour except- 2439. Inadvertent surgical occlusion of the ureter leato-
a) Pre Operative use of Actinomycin D (AIIMS 92) a) Complete renal atrophy (PGI 80,
b) Post Operative Radiotherapy b) Hematuria AIIMS 81, 87)
c) Good prognosis in Infants c) Renal failure
d) Neuroblastoma is the commonest differential d) Hydronephrosis
diagnosis e) Hypertension
2429. Which does not happen in Unilateral Renal 2440. The following are the complications of
Trauma- (AIIMS92) haemodialysis except- (JIPMER 81, AIIMS 86)
a) Hypertension b) Uraemia a) Hypotension b) Peritonitis
c) Clot formation d) Perinephric haematoma c) Hypertension d) Bleeding tendency
2430. Following is trne of Pyonephrosis except- 2441. The risk of rupture in renal artery aneurysms is -
a) Commonly associated with Renal calculi a) Less than 1% b) 5% (AMU81,
b) Always unilateral (AIIMS 92) c) 200/o d)75% JJPMER 81)
c) Is a complication of Hydronephrosis e) None of the above
d) Follows acute pyelonephritis 2442. Treatment of choice of ureteric colic is- (UPSC
2431. Which is false regarding Ureteric stones- a) Nitrites b) Pethidine 82,84)
a) Urine is always infected (AJIMS 92) c) Adrenaline d) Regitine
b) Should be removed immediately 2443. Renal tuberculosis originates in the- (JIPMER 81,
c) Source is always the kidneys a) Renal papilla PGI 86)
d) Pain is referred to tip of penis in intramural stones b) Renal medulla
2432. Which is false ofHosr shoe Kidney- (AIIMS 92) c) Afferent tubules
a) Spider like appearance in IVP d) Efferent arteriole of glomerulus
b) Ureteral Obstruction is common 2444. Flower 'Vase' pattern of the pelvis in an intravenous
c) Lower calyx is reversed urogram is seen in - (PGI80,AIJMS 84, Karn 96)
d) Heminephrectomy improves function a) Polycystic kidney b) Renal carcinoma
2433. In Renal injury following blunt injury to abdomen c) Horse shoe kidney d) Ectopic kidney
whichisnotdone- (AIIMS 92) 2445. Epidermoid carcinoma of renal pelvis is usually
a) Prophylactic nephrectomy associated with- (Karn. 94)
b) Diagnostic peritoneal lavage a) Multiple papilomas b) Pelvic calculus
c) lVP c) Tuberculosis of kidney d) Filariasis
d) Exploratory laparotomy 2446. "Golf-hole" ureter is seen in- (Karn. 94)
2434. All are true of Aberrant renal artery except- a) Ureteric calculus
a) Bilateral (PGI 93) b) Ureteral polyp
b) Leads to hydronephrosis c) Tuberculosis of ureter
c) Comon in females d) Retroperitonial fibrosis
d) More common on left side 2447. Auto nephrectomyisseenin- (JIPMER 95)
2435. Subcapsular nephrectomy is indicated is -(PGI 93) a) Sickle cell anemia b) Renal T.B.
a) Perinephric abscess c) Sarcoidosis d) Lymphoma
b) Hydronephrosis 2448. In Renal cell carcinoma which is true(JIPMER 95)
c) Pyonephrosis a) Hypercalcemia b) Polycythemia
d) Solitary adenocarcinoma c) Cushings syndrome d) All
2436. Most common route of infection of Kidneys in 2449. Commonest site of metastasis ofWilm's tumour is-
T.B. is- (All India 93) a) Bones b) Lungs (AIIMS 94)
a) Direct invastion b) Hematogenous c) Liver d) Brain
c) Lymphatic d) Ascending 2450. Regarding renal cell carcinoma what is wroug-
2437. Renal collar to prevent spread of malignancy a) Most common is females (AJIMS 94)
from kidney is put around- (JIPMER 93) b) Invades renal vein
a) Aorta b)NC c) May be associated with varicocle
c) Renal vein d) Renal artery d) Arises from proximal convulated tubule

2427)d 2428)a 2429)b 2430)b 2431)a,b 2432)a,d 2433)a 2434)a 2435)c 2436)b 2437)c 2438)a 2439)a,d
2440)b,c 2441)e 2442)None 2443)a 2444)c 2445)b 2446)c 2447)a 2448)d 2449)b 2450)a
SURGERY [ 297]

2451. False regarding angiomyolipoma of kidney is- b) At pelvic brim


a) Is associated with tuberous sclerosis(AJJMS 94) c) At the lelve of gonadal vessels
b) Treatment is neprectomy d) Intramural portion of the ureter
c) May cause hypertension 2464. Pseudo kidney is- (J & K 2001)
d) Usually presents with pain a) Thickened bowel loop on USG
2452. True about renal trauma is- (AI 95) b) Hydronephrosis
a) Urgent IVP is indicate c) Unascended kidney
b) Exploration ofkidney to be done in all causes d) Undescended testes
c) Lumbar approach to kidney 2465. Drug of choice for renal colic- (Kerala 98)
d) Renal artery aneurysms are common a) Methadone b) Pentazocine
2453. Commonest metastasis in Renal CeO Carcinoma c) Voveran d) Piroxican
is to- (AIIMS 97) 2466. Ectopic ureter may be frequendy associated with-
a) Lung b)Bone a) Oliguria (JIPMER8J,AMU89)
c) Adrenal d) Liver b) Dysuria
2454. Not true about polycystic kidney disease is- c) Bilateral hydroureter
a) Autosomal dominant (AIIMS 97) d) paradoxical incontinence
b) Progress to CRF 2467. Earliest and often the only presentation of TB
c) Proteinuria is common, but seldom exceeds 2g kidney is- (AIIMS 78,AMU88)
d) Excision of the cyst is the treatment of choice a) Increased frequency b) Pain
and it reverses the renal function c) Hematuria d) Renal calculi
2455. Rena trauma is best treated by- (UPSC) 2468. Renal function failure is best evaluated by-
a) Observation and supportive measures a) DTPA b) DMSA
b) Early drainage and perirenal haematoma c) Iodohippurate d)MAG-3
c) Heminephrectomy 2469. IVP is done using- (PGI 85)
d) Nephrostomy a) Conray 240 b) Conray 3080
2456. The most common neoplasm ofkidneyis-(Kan. 98) c) Conray 480 d) conray 540
a) Wilm's tumour 11) Renal cell carcinoma 2470. In a patient who was has acid-fast bacilli in the urine-
c) Renal adenoma d) Haemagioma a) Calcification of the bladder is common
2457. Best method of confirm treatment ofTB kidney- b) Bladder diseases is associated with extensive
a) NP b) CT (AIIMS 98) renal disease (AIIMS 83)
c) Renal angiogram d) Ultrasound c) Ureteric involvement causes shortening of the
2458. Stone at lower end ofureterremoved by -(AIIMS 98) ureters
a) Diuretics b) Endoscopic removal d) Renal disease can produce changes indentical to
c) Lithotripsyin situ d) Antispasmodics reflux nephropathy
2459. Ectopic ureter occurs in all sites except(AJJMS 98) e) Ureteric calculi calculi are commonly present
a) Bladderneck b) Seminal vesicle 2471. Which of the foUowing is diagnostic of the Rim
c) Prostate d) Bulbar urethra sign in a nephrogram- (AMU 88)
2460. Most severe pain in ureteric stone is seen in cases a) Phelonephritis
of- (CUPGEE 99) b) Hypernephroma
a) Oxalate stones b) Triple phosphate c) Polycystic kidney
c) Cystine stone d) Uric and stone d) Severe hydronephrosis
2461. Unilateral hydronephrosis is due to- (AMC 99) 2472. In Genitourinary TB, True is- (PGI 98)
a) Bladder neck contracture b) Stricture urethra a) Sterile pyurias consistent finding
c) Carcinoma of prostate d) Ureterocele b) AFB in early morning sample is always positive
2462. During investigation of hydonephrosis, isotope c) MC site is pelvis
renogram is useful mainly in- (UPSC 2K) d) Commonest cause of pyelonephritis
a) Detecting vesicoureteric reflux 2473. Urinary ascites is due to- (PGI 98)
b) Anatomical definition a) Injury to bladder during birth
c) Distinguishing between non-obstructed system b) Ureteric obstruction
d) Identifying ectopic kidney tissue c) Cong. urethral atresia
2463. A patient was admitted with complaints of d) Urethral valves
ureteric stones. He was on treatment with i.v. 2474. Left loin nephrectomy, Str. not cut is- (PGI 98)
fluids and analgesics. Suddenly he developed a) Trapezius b) Seratus inf. post
radiating pain to the pubic area and medial aspect c) Lat. dorsi d) Internal oblique
of the thigh. The stone is coming down. What is 2475. The resting uretric pressure in mm- (PGI 99)
the most probable site oflodgment of the stone- a) 5-7 cmofHP b) l5-30cmof~O
a) At renal pelvis (AIIMS 2K) c) 7-lOcmof~O d)0-5 cmofHP

245l)b,c,d 2452)a 2453)a 2454)d 2455)a 2456)c 2457)a 2458)b 2459)d 2460)a 246l)c,d 2462)c 2463)b
2464)a 2465)c 2466)d 2467)a 2468)All 2469)None 2470)b,c,d 2471)d 2472)a 2473)All 2474)a
SURGERY [ 298]

2476. Renal cell carcinoma histopathologicaly showing 2485. The neonatal kidney achieves concentrating
'perinuclear halo' & "Plant like"structure ability equivalent to adult's kidney by- (Al 04)
imalignant cells- (PGI 2000) a) One year of age
a) Clear cell tumor b) Papillary Ca b) Eighteen months of age
c) Collecting duct. Ca d) Oncocytoma c) Three to six months of age
2477. Which of the following statements is true regarding d) Just before puberty
kidney tumors- (PGI OJ) 2486. A 3 year old girl presents with recurrent UTI. On
a) Mutated VHL gene is associated with clear cell USG shows hydronephrosis with filling defect and
carcinoma negative shadow of bladder with no ectopic
b) Extreme hyperdiploidy occurs orifice- (U.P.P.GME.E. 04)
c) Extreme hypodiploidy occurs a) Vesicoureteric reflex
d) Renal papillary carcinoma has defect in b) Hydronephrosis
chromosome 8 c) Ureterocele
e) Oncocytoma has defect in chromosome 11 d) Sacrococcygeal teratom
2478. ll'ue ofautosomal dominant polycystic kidney disease 2487. An absent kidney is found in- (AMU 05)
I and II respectiely- (PGI 02) a) 1:200 individuals b) 1:700 individuals
a) Chromosomes 16 and 5 b) 16 and 4 c) 1:1400 individuals d) 1:5000 individuals
c) 11 and5 d) 11 and4 2488. All are features of Hypernephroma except-
e) 4and5 a) Persistent pyrexia (HPU 05)
2479. The substances present in the gall bladder stones b) Hematuria ·
or the kidney stones can be best identified by the c) Polycythemia
following techniques- (AI 03) d) Lower pole involvement
a) Flourescence spectroscopy 2489. Forty eight hours after sustaining a blunt abdominal
b) Electron microscopy injury, a 15 year old boy presents with hematuria
c) Nuclear magnetic magnetic resonance and pain in the left side of abdomen. On examination,
he has a pulse rate of96/minute with a BP of110170
d) X- ray diffraction
mmHg. His Hb is 10-8 gm% with a PCV of31 %.
2480. A 10mm calculus in the right lower ureter
The blood urea is 32 mg%.Abdominal examination
associated with proximal hydroureteronephrosis
revealed tenderness in left lumbar region but no
is best treated with - (AI 03)
palpable mass. The most appropriate investigation
a) Extracorporeal shockwave lithotripsy
to diagnose and find the extent of renal injury would
b) Antegrade percutaneous access be-
c) Open ureterolithtomy a) Sonographic evaluation of abdomen (UPSC 05)
d) Ureteroscopic retrieval b) Intravenous pyelography ·
2481. Which of the following is not a contraindication c) Contrast enhanced computed tomography
for extra corporeal Shockwave Lithotripsy d) MR urography
(ESWL) for renal calculi? (AIIMS 03) 2490. Ectopic ureter opening is not located in -(MAHA 05)
a) Uncorrected bleeding diathesis a) Bulbar urethra b) Prostatic urethra
b) Pregnancy c) Seminal vesicle d) Bladderneck
c) Ureteric stricture 2491. Which ofthe following is not a preferred site for
d) Stone in a calyceal diverticuhi.m planning vascular access for maintenance
2482. Unilateral small smooth kidney is seen in- hemodialysis? (AI 06)
a) Reflux nephropathy (Karnataka 03) a) Nondominant extremity
b) Labar infarction b) Upper limb
c) Renal artery stenosis c) Radiocephalic AV fistula
d) Chronic glomerulonephritis d) Saphenofemoral fistula
2483. A 67 year old man presented initially after a single 2492. Oxalate stones are found in- (PGI June 06)
episode of gross painless haematuria. An a) Ethylene glycol b) Ethanol
excretory urogram demonstrated a 1.5 mm round c) Diethylglycol d) Methyl alcohol
filing defect in the right lower renal infundibulum. 2493. In a patient suspected to be suffering from
The best study to obtain next is - (Karnataka 03) vesicoureteric reflex, which one of the following
a) Urinary cytology b) Cystoscopy radiological investigations may confirm the
c) Ultrasonography d) Retrograde pyelography diagnosis ? (UPSC 07)
2484. Polycystic disease of the kidney may have cysts in a) Intravenous urography
all the following organs except- (AI 04) b) Micturating cystourethrography
a) Lung b)Liver c) Pelvic ultrasound
c) Pancreas d) Spleen d) Antegrade pyelography

2475)b 2476)None 2477)a,c 2478)b 2479)d 2480)d 248l)d 2482)c 2483)c 2484)d 2485)a 2486)c 2487)c
2488)d 2489)c 2490)a 2491)d 2492)a 2493)b
SURGERY [ 299]

2494. Stone which is resistant to lithotripsy- 2504. What percent of cases with blunt injury to kidney
a) Calcium oxalate (Aiims May 07) require surgical exploration- (Manipal 08)
b) Triple phosphate stone a) 200/o b) 90%
c) Cystine stone c) 500/o d) 70%
d) Uric acid stone 2505. Whatis column ofbertini in kidney? (APPG 08)
2495. Frnit Juice which helps in preventing UTI- a) Renal tumour
a) Cranberry b) Raspberry (Aiims May 07) b) Tongue like papillary projection
c) Orange d) Grape c) Caliculus
2496. "Spider leg" deformity in excretory urogram d) None
2506. A lady present with UIL hydronephrosis on USG She
occurs in- (UP 07, AI91)
is asymptomatic. Ureteropelvic drainage is normal.
a) Hydronephrosis b) Polycystic kidney
What is/are treatment modality for her-
c) Uretrocele d) Renal agenesis
a) Pyleroplasty (PGI Dec'08)
2497. Commonest cause of ureteric injury during
b) Analgesia SOS
surgical operation is- (UP 07, 06)
c) Underobservation
a) Abdomino-perineal resection d) Follow up by USG
b) Hysterectomy 2507. Renal calculi associated with proteus infection-
c) Prostectomy a) Uric acid b) Triple phosphate (AI 09)
d) Colectomy c) Calcium oxalate d) Xanthine
2498. Ureteric colic characterized by all except- (UP 07) 2508. Which of the following is the Post- Chemotherapy
a) Acute onset based staging system in Wilm's tnmor- (AI 09)
b) Stillness of the patients a) National Wilm's tumor staging system (NWTSG)
c) Responds to antispasmodics b) International society ofPediatric Oncology (SlOP)
d) Radiates to the groin c) AJCCTNM
2499. The common organism isolated in d) Chadwick
Emphysematous pyelonephritis is- (Corned 08) 2509. Most important prognostic indicator for Renal cell
a) E.Coli b) Proteus carcinoma- (AIIMS May 09)
c) Pseudomonas d) Klebsiella a) Nuclear grade b) Histological type
2500. A 23 year old male who is otherwise normal c) Size d) Pathological staging
complains of mild pain in his right iliac fossa in a 2510. Which one of the following is the treatment of choice
waveform pattern which increases during the night for lower ureteric stone- (UPSC-II 09)
and he becomes exhausted and is admitted in the a) Endoscopic removal b) Diuretics
hospital. On examination there is mild hematuria. c) Drug disslution d) Laser
Urine examination reveals plenty of RBCs, 2511. Renal calculi associated with Proteus infection -
50WBCs/hpf. Urine pH is 5.5. Most likely diagnosis a) Uricacid (AIIMSMayJO)
is - (AIIMS Nov 07) b) Triple phosphate
a) Glomerulonephritis b) Ca-Urinary bladder c) Calciumoxalate
c) Ureteral calculus d) Cystitis d) Xanthine
2501. Persistent foetal lobulation of adult kidney is due 2512. Wilm's tnmor associated with AlE -(AIIMS May I 0)
to- (AIIMSNov07) a) Hemihypertrophy
a) Congenital renal defect b)Aniridia
b) Obstructive uropathy c) Hypertension
c) Intrauterine infections and scar d) Bilateral polycystic kidney disease
d) Is a normal variant 2513. A 60 yr old chronic smoker presents with painless
2502. The typical appearance of "spider leg" on excretory gross hematnria of 1 day duration. Investigation of
urography is seen in- (UPSC-II 08) choice to know the cause ofhematnria-
a) Hydronephrosis a) USG (AIIMS May 10)
b) Polycystic kidney b) X-rayKUB
c) Medullary sponge kidney c) Urine routine
d) Renal cell carcinoma d) Urine microscopy for malignant cytology cells
2503. Ureteric colic due to stone is caused by- (AI 08) 2514. Which of the following stones is hard to break by
a) Stretching of renal capsule due to back pressure ESWL.- (AI 10)
b) Increased peristalsis of ureter to overcome the a) Calcium oxalate monohydrate
obstruction b) Calcium oxalate dihydrate
c) lrriation of intramural ureter c) Uricacid
d) Extravasation of urine d) Struvite

2494)c 2495)a 2496)b 2497)b 2498)b 2499)a 2500)c 250I)d 2502)b 2503)b 2504)a 2505)b 2506)b,c,d
2507)b 2508)b 2509)d 2510)a 25ll)b 2512)d 2513)d 2514)a
SURGERY [ 300]

2515. A 50 year old female is admitted with abdominal pain 2523. Stone which is resistant to Hthotripsy-{All.MS Nov 10)
and anuria. Radiological studies revealed bilateral a) Calcium oxalate b) Triple phosphate stone
impacted ureteric stones with hydronephrosis. Urine c) Cystine stone d) Uric acid stone
analysis showed RBCs with pus. cells in urine. 2524. A 24 yr old man presented with retroperitoneal,
Serum creatinine level was 16 mgldl and urea level necrotic, heterogenous enhancing mass on CT near
was 200 mmol/1 which of the following should be the the hilum of the left kidney. What is the most
immediatetreatment- (AI IO) probable diagnosis- (AIIMS Nov 10)
a) Hemodialysis a) Metastatic germ cell tumour
b) 'J' stent drainage b) Metastatic melanoma
c) Lithotripsy
c) Lymphoma
d) Ureteroscopic removal of stones
d) Metastatic transitional cell tumour
2516. What compllcation should one expect when PCNL
2525. Which of the following is not associated with Renal
is done through 11th intercostals space - (All 0)
cell carcinoma- (AIIMS May 11)
a) Hydrothorax b)Hematuria
c) Damage to colon d) Ramnants fragments a) Polycythemia b) Amyloidosis
2517. Which of the following is the most common renal c) Cushing's syndrome d) Hypertension
vascular anomaly- (AI I 0) 2526. Renal Calculi associated with proteus infection-
a) Supernumerary renal arteries a) Uric Acid b) Triple phosphate
b) Suprnumerary renal veins c) Calciumoxalate d) Xanthine (AI 11)
c) Double renal arteries 2527. A 55 year old smoker presents with history of five
d) Double renal veins episodes of macroscopic hematuria each lasting for
2518. First autologous renal transplantation was done by- about 4-5 days in the past five years. Which ofthe
a) Hardy b) Kavosis (AI 10) following investigations should be performed to
c) Higgins d) Studor evaluate the suspected diagnosis- (AI 11)
2519. MEN 2A is associated with following except- a) Urine microscopy and cytology
a) Medullary Thyroid Carcinoma (Maharashtra I 0) b) X-rayKUB
b) Pituitary tumor c) Ultrasound KUB
c) Pheochromocytoma d) DTPAscan
d) Hyperparathyroidism 2528. A 40 year old patient with a single kidney presents
2520. Renal carcinoma with solitary lung secondary is with a solitary exophytic mass of 4 em localized at
best treated by- (UP SC II 10) its lower pole which amongst the following is the
a) Radiotherapy b) Surgery best recommended management option- (AI 11)
c) Chemotherapy d) Immunotherapy a) Partial Nephrectomy
2521. Match List-1 with List-11 and select the correct
b) Radical Nephrectomy
answer using the code given below the Lists-
c) Radical Nephrectomy with dialysis
List-I (Aetiology) List-II (Type of cyst)
d) Radical Nephrectomy with immediate Renal
A. Hamartoma 1. Enterogenous
Transplantation
B. Failure of 2. Urachal
connections of 2529. A 25-year-old hypertensive man with a history of
tubular elements recurrent calcium-containing renal calculi presents
C Persistence of 3. Polycystic kidney to the emergency department with excruciating
normal vestigial flank pain and blood in the urine. This patient is
remnants likely to have which one of the following underlying
D. Duplication 4. Lymphatic cyst disorders? (UPSC I 11)
Code: (UPSCIIlO) a) Chronic proteus infection
A B C D b) Factor VIII deficiency
a) 4 3 1 1 c) Hyperaldosteronism
b) 4 3 1 2 d) Hyperparathyroidism
c) 3 4 1 2
d)3 4 2 1
2522. Indications of percutaneous nephrostomy-
a) Stoneremoval {PGINov. 10)
b) Ureteral obstruction
c) Anterograde renography
d) Renal tumor resection
e) Ischemic renal failure

2515)b 2516)a 2517)a 2518)a 2519)b 2520)b 252l)a 2522)a,b,c,d 2523)c 2524)a 2525)c 2526)b 2527)a
2528)a 2529)d 2530)a 253l)b
SURGERY[301]

URINARY BLADDER 2544. Bladder cancer most constantly shows which-


a) Dysuria (AIIMS 81, Rohtak 86)
2532. Commonest cause for pulsion diverticulum of the b) Urgency
urinarybladderis~ (JIPMER 86) c) Increased frequency
a) Benign enlargement of prostate d) Haematuria
b) Fibrous prostate e) Persistent & recurrent urinary infection
c) Contracure ofbladder neck 2545. Treatmentof'Thimblebladder'is~ (JIPMER81,
d) Stricture urethra a) Anti-tubercular treatment· Delhi 84)
2533. Most malignant carcinoma ofthe bladder is~
b) Corticosteroids
a) Malignant villous tumour (AP 85)
b) Solid tumour c) ileocystoplasty
c) Carcinomatous ulcer d) Anti-tubercular drugs + steroids
d) Adenocarcinoma 2546. 'Kiss cancer' of the urinary bladder
2534. Most reliable investigation in bladder rupture is is- (AIIMS 80,81, UPSC 88)
a) NP (JIPMER 87) a) Highly malignant b) Malignant
b) Cystoscopy c) Benign d) Pre-malignant
c) Retrograde cystogram 2547. In extraperitonial bladder rupture urine escapes
d) Catheterisation into- (AIIMS 94, AI 93)
2535. Following is the commonest tumour of urinary a) Peritonium
bladder- (AIIMS 84, PGI 97, Jipmer 86, b) Perivesical space
a) Papilloma AI 89) c) Below urogenital diaphragm
b) Adenocarcinoma d) Groin
c) Transitional cell carcinoma 2548. About transitional cell carcinoma of bladder all
d) Squamous cell carcinoma of following are correct except- (Delhi, PGI 86)
2536. Following may predispose to carcinoma bladder a) Most common site is fundus
except- (AIIMS 84) b) Prognosis is excellent if muscle layer is invaded
a) B-Nephthylamine b) Smoking c) Exposure to industrial carcinogens predisposes
c) Bilharziasis d) Tuberculous cystitis to it
2537. It is true of carcinoma of the urinary bladder that d) Most of carcinomas are flat, solid & deeply
a) It usually occurs in childhood (AIIMS 85) infiltrating
b) Occurs more often in aniline dye workers 2549. Earliest symptom is carcinoma bladder is-
c) It is located most frequently in the trigone a) Frequency b) Pain (AIIMS 96)
d) Papillary formation is rare c) Burning d) Haematuria
2538. Which is not seen in complete ecotopic vesicae- 2550. Which is a normal finding in cystometry- (PGI 97)
a) Umbilicalhemia (IN89) a) Absence of systolic detrussor contraction
b) Visible ureterovesical efflux b) Residual volume of75 ml
c) Hypospadias c) Leakage on coughing
d) Waddling gate d) First sensation of urination at 300 ml
2539. The commonest bladder sone is- (PGI 84) 2551. 'Teardorp'bladderisfoundin- (Karn. 98)
a) Triple phosphate b) Xanthine a) Tuberculosis b) Neurogenic bladder
c) Uric acid d) Cysteine c) Pelvic hematoma d) None of the above
2540. The following is true about bladder stones~ 2552. In ureterosigmoidostomy all occure
a) Girls more than boys (PGI 84) except- (JIPMER 98)
b) Treatment is lithopexy a) Hyponatremia b) Hyperkalemia
c) Always forms in kidneys and passes down to c) Hyperchloremia d) Acidosis
bladder 2553. Regarding STONES all are true, except-
d) Usually asymptomatic a) Uric acid is a dropped stone (AIIMS 98)
2541. Bladder tumours mostly arises from- (AI 91)
b) Obstruction to urinary tract is a common cause
a) Mucosa b) Submucosa
c) Jackstone calculus is formed in alkaline pH due
c) Muscularis Mucosa d) Seorsa
to urea - splitting organisms
2542. Thimble bladderis seen in- (AI 91)
d) Staghorn calculus is seen in alkaline urine
a) Acute tuberculosis b)Chronic tuberculosis
2554. Cystoscopic fmdings in TB bladder are all except~
c) Neurogenic bladder d) Schistosomiasis
a) Cobblestone mucosa (PGI 97)
2543. All are precancerous for carcinoma bladder
b) Thimble bladder
except- (AI 91)
a) Tuberculosis bladder b) Aniline dyes c) Golfholeureter
c) Schistosomiasis d) Chronic ulcer d) Whitish efflux from the ureteric holes

2532)c 2533)b 2534)c 2535)c 2536)d 2537) b 2538)c 2539)None 2540)b 2541)a 2542)b 2543)a,d 2544)d
2545)a,c 2546)c 2547)b 2548)a,b,d 2549)d 2550)a 2551)c 2552)a,b 2553)c 2554)d
SURGERY [ 302)

2555. Sq. cell tumor of urinary bladder is due to -(PGI 97) 2566. Secondary vesical calculus refers to stones formed
a) Stone b) Schistosomiasis due to- (Karnataka PGMEE 06)
c) Chr. cystitis d) Diabetes mellitus a) Hypercalciuria b) Injury
2556. Tear-drop bladder is seen in- (PGI 99) c) Infection d) Migrating from
a) Tuberculosis 2567. A 50 year old man known to have diabetes presents
b) Runner's ulcer with, poor urinary stream, hesitancy, difficulty in
c) Perivescial hemorrhage with rupture micturation and failure to void completely. What is
d) Perivesical hemorrhage without rupture the diagnosis ? (AI 07)
2557. Transitional cell carcinoma of bladder is associated a) BPH b) Autonomic neuropathy
with- (PG/01) c) UTI d) Atonic bladder
a) Schistosomiasis b) Naphthylamine 2568. A 50 year old man, known to have diabetes, presents
c) Smoking d) Tuberculosis of bladder with poor urinary stream, increased frequency of
2558. Squamous cell ca of urinary bladder is predisposed micturation and hesitancy. What is the most likely
toby- (PG/02) diagnosis - (AI 07)
a) Urolithiasis b) Persistent Urachus a) BPH b) Autonomic neuropathy
c) Schistosomiasis d) Polyp
c) UTI d) Atonic bladder
e) Smoking 2569. A man falls astride a penetrating object. He develops
2559. True about transitional cell Ca of urinary bladder-
retention of urine, perineal hematoma and bleeding
a) Smokingpredisposes (PG/03)
from urinary meatus. The nature ofinjury would be-
b) Schistosoma infection predisposes
a) Rupture of membranous urethra (UPSC II 1 0)
c) Aniline dye workers
b) Rupture of bulbar urethra
d) Radiation
c) Intraperitoneal rupture of bladder
2560. Post micturation dribbling is due to- (AMU 05)
d) Extraperitoneal rupture of bladder
a) Detrusor overactivity
2570. A lady who presented with hematuria was found to
b) Dribbling decreases in case of urethral stricture
have Stage ll Transitional Cell Carcinoma of bladder.
c) Collection ofUrine in U shaped curve ofbulb ofpenis
d) Neurogenic bladder Which of the following statements about
2561. Which one of the following is not an intravesical management of her condition is true- (AI 11)
chemotherapeutic agent- (UPSC 05) a) Cystoscopic falguration is the standard treatment
a) MitomycinC b)BCG b) 70% chance of requiring cystectomy in 5 years
c) Epirubicin d) Thiotepa afterTUR
2562. A patient Ramu presents with hematuria for many c) History of smoking is not a risk factor
days. On investigations be is found to have renal d) There is no role of chemotherapy
calculi, calcifications in the wall of urinary bladder 2571. In a patient with stage I (confined to mucosa and
and small contracted bladder; most probable cause submucosa) transitional cell carcinoma of the
is - (MAHA 05) urinary bladder, the recommended treatment is-
a) Schistosomiasis b)Amyloidosis a) Trans urethral resection ofbladder tumour (TURBT)
c) Tuberculosis d) Ca urinary bladder b) Radical cystectomy (UPSC II 11)
2563. Which of the following is the most effective c) Radiation therapy
intravesical therapy for superficial bladder d) Systemic chemotherapy
cancer- (AIIMSNOV05) ~$1~· PeryJ~~~~·u·~~h~sis.~ ris~tac(orJ()~""'····•··.
a) Mitomycin b) Adriamycin •.·····. •·•· · · .•.·• . ~)z>ftat!It1~0Jial~c~lh~~pfblaqijtfJ'c1 .. •.,i;(Jfcfi6TIDNJ{
c) Thiotepa d)BCG :>,?:;~:, y,;,h) Acl.en~carcmmna o:t:~lad!i¢r: ,,,, '' :. ':, )eattern)
2564. Urinary catheterisation indicated in case of acute · c)Sq1Jamouscellca,~f;qlad;4et .· ·
retention of urine in following conditions - i· .· . ,,, :d}hl;ti{fi€ferentiated:ca:bfblad(fet ... / .• ··.·. ·····"
a) Stricture b) Rupture (PGI June 06) Z573.• J)i.versionofurinarybladder is best,don~lat-
c) Postoperative d) Carcinoma prostate ·a) Jejunum h)Jleurn(JYl.EEJJ!JJ.NiJPiiitem):
e) Before appendicectomy in acute appendicitis •'·· •. • A;) Ce<:um:~~·:~ ·~;•rj; .d)Cdlootc ;J:;,• ~J .,;,.:Jti: . ; ..
2565. Consider the following conditions-
1) Urinary flow rate less than lOcc I second PROSTATE & SEMINAL VESICLE
2) Residual volume of urine> 1OOcc
3) Serum level of prostatic specific antigen> 10 mmOl 2574. Marion's disease is due to- (AIIMS 81, DNB 90)
/litre a) Muscular hypertrophy of internal sphincter of
4) Trabeculated urinary bladder urinary bladder
Which ofthe above are indications ofTRUP for BliP? b) Fibrosis of the neck of bladder
a)l,2and3 b) 2,3and4 c) Vesiculardiverticula
c) 1,2 and 4 d) 1, 3 and 4 d) Vesicular calculi

2555) a,b 2556) c 2557) a,b,c 2558) a,c 2559) a,b,c,d 2560) c 2561) b 2562) a 2563) d 2564) c,d 2565) c 2566) c
2567)b 2568)a 2569)b 2570)b 2571)a 2572)b 2573)b 2574) a
SURGERY [ 303]

2575. Medical castration is effected by- (KERALA 90) b) Mesentric ischemia


a) Diethylstibesterol b) LH RH analogues c) Bladder rupture
c) Gossypol d) Hanovan d) meteorism
2576. T.U.R. (trans urethral resection) syndrome is due 2588. A patient presents with complains of sciatica. On
to- (UPSC 95) radioligical examination there was sclerotic
a) Hyponatremia b) Hypokalemia lesions on his skum. Which of the following is most
c) Hypovolaemia d) Hypoxia likely to be elevated in this patient - (AIIMS 2K)
2577. 80 year old man underwent transurethral a) CEA
prostatectmy biopsy revealed foci of adenocarcinoma. b) Prostate specific antigen
Next line of management- (AI89, AIIMS 87, 2K) c) Alkaline phosphatase
a) Radiotherapy b) Hormonal therapy d) Alpha 1 antitrypsin
c) Surgery d) No further treatment 2589. Which is not used in carcinoma prostate -(PGI 97)
2578. Complication which commonly accompanies a) Estrogen b) Progesterone
acute prostatitis- (AIIMS 87) c) Cyproterone acetate d) Flutamide
a) Epidymitis b) Orchitis 2590. Absence offructose in semen indicates- (PGI 98)
c) Seminal vesiculitis d) Sterility a) Obstruction to seminal vesicles
2579. Grade I benign prostate with outflow obstruction b) Obstruction at prostatic urethra
is best treated witb- (AIIMS 84) c) Vas deferens obstruction
a) Retropubic prostatectomy d) Testicular failure
b) Transurethral resection 2591. In prostatic metastasis, the site most commonly
c) Transvesical pristatectomy involved is- (PGI 99)
d) Androgen therapy a) Obturator nodes b) Perivesical nodes
2580. Complimentary operation done at the time of c) Pre-sacral nodes d) Paraaortic nodes
prostatectomey is- (UPSC 88) 2592. Which one of the following is an absolute
a) Vasectomy b) Circumcission indication for surgery in cases of benign prostatic
c) Hernia repair d) All of the above hyperplasia- (AI 03)
2581. In carcinoma prostate with matastasis which is a) Bilateral hydroureteronephrosis
raised - (TN 89) b) Nocturnal frequency
a) ESR b)Alkalinephosphatase c) Recurremt urinary tract infection
c) Acid phosphatase d) Bilirubin d) Voiding bladder pressures> 50 em of water
2582. Secondary deposits from prostatic carcinoma is 2593. Semen analysis of a young man who presented
commonest in- (PGI 80, AI 89, AP 89) with primary infertility revealed low volume,
a) Bone b) Kidney fructose negative ejaculate with azoospermia.
c) liver d)Brain Which one of the following is the most useful
2583. Indication for surgery in Benign prostatic imaging modality to evaluate the cause of his
hypertrophy are all except -(JIPMER 80, AI89,AP 85) infertility- (AI 03)
a) Prostatism b) Chronic retention a) Color duplex ultrasonography of the scrotum
c) Hemorrhage d) Enlarged prostate b) Transrectal ultrasonography
2584. Most reliable indicator of carcinoma of prostate c) Retrograde urethrography
recurrence after surgery- (JIPMER 95) d) Spermatic venography
a) Prostate specific antigen b) Acid phosphatase 2594. Which one of the following lasers is used treatment
c) Alkaline phosphatase d)CEA-15 of benign prostatic hyperplasia as well as urinary
2585. Which is the earliest symptom of benign calculi - (AI 03)
hypertrophy of prostate- (Karnat 96) a) C02 laser b) Excimer laser
a) Frequency b)Haematuria c) Ho: YAG laser d) Nd: YAG laser
c) Incontinence d) Strangury 2595. Screening of prostate Ca commonly done by-
2586. 70 year old man with Ca.prostate with osteoblastic a) DRE(Digitalrectalexam) b)USG(PGIJune05)
secondaries in pelvis and lumbar vertebra showed c) MRl d)PSA
well differentiatedAdeno Carcinoma prostate on e) CTScan
needle biopsy. He is idealy treated by- (MAHE 98) 2596. The following statements regarding fmasteride are
a) Radicalprostectomy b)TURP trueexcept- (A/05)
c) Radiation d) Hormonal manipulation a) It is used in the medical treatment of benign
2587. A patient was undergoing trans Urethral resection Prosstatic hypertrophy (BPH)
of Prostate. He was under spinal anesthesia using b) Impotence is well documented after its use.
Bupivacaine. He developed preumbilical pain after c) It blocks the conversion of dihydrotestosterone
30 minutes of surgery. The probable cause conld be- to testosterone
a) Effect ofBupivacaine waning (AIIMS 2K) d) It is a 5-a reductase inhibitor

2575) a,b2576) a 2577) d 2578) c 2579) b 2580) None 2581) b 2582) a 2583) d 2584) a 2585) a 2586) d 2587) c
2588)b 2589)b 2590)a 259l)a 2592)a 2593)b 2594)c 2595)d 2596)c
SURGERY [ 304]

2597. Which of the following is the most troublesome 2606. Which of the following is true about prostate cancer
source of bleeding during a radical retropubic screening? (APPG 08)
prostatectomy· (AI05) a) Digital screening along with PSA is additive
a) Dorsal venous complex b) Prostate cancer is common among young males
b) Inferior vesical pedicle c) Tumor markers are diagnosed
c) Superior vesical pedicle d) Bleeding per rectum in earliest manifestation of
d) Seminal vesicular artery disease
2598. McNeal's peripheral Zone in prostate gland is the 2607. Hot flush is not associated with- (PGI Dec 08)
seat of- (Karnataka PGMEE 06) a) Medical castration
a) Benign Hypertrophy b) Cancer b) Surgical castration
c) Prostatitis d) Calculi c) Ketoconazole therapy
2599. 50 year old male with positive family history fo d) Androgen receptor blockade
prostate cancer has come to you for a screening e) Radical prostatectomy
test. The most sensitive screening test to pickup 2608. Gleason score: all are true except- (AIIMS Nov 08)
prostate cancer is- (AI 07) a) Used for grading prostate cancer
a) DRE b) Scores range from 1-10
b)PSA c) Higher the score, poorer the prognosis
c) DRE+PSA d) Helps in planning management
d) Endorectal coil MRI with T 1Wand T2W images 2609. Assessment of patient with Prostatism include all
2600. 60 yr old male undergoes TURP.After 3 days patient except- (DELHIPGMar. 09)
develops altered senstiveness and drowsiness. Most a) Rectal examination-
probable diagnosis is- (Aiirns May 07) b) Serum prostate specific antigen
a) Hypematrernia c) Pressure flow urodynamic studies
b) Hyponatremia d) Transrectal ultrasound scanning
c) Stroke 2610. 50 year old male with positive family history of
d) Meningitis due to spinal anaesthesia prostate cancer has come to you for a screening
2601. Carcinoma ofthe prostate arise from- (UP 07, 05) test. The most sensitive screening test to pickup
a) Central zone b) Peripheral zone prostate cancer is- (AIIMS May I 0)
c) Transitional zone d) Periurethral zone a) DRE
2602. The most important use of transrectal b)PSA
ultarsonography (TRUS) is for- (Corned 07) c) DRE+PSA
a) Screening for Ca prostate d) Endorectal Coil MRI with Tl Wand T2W images
b) Distinguishing prostate cancer from BPH 2611. What is the optimum duration antibacterial
c) Systematic prostate biopsy in suspected prostate treatment of acute bacterial prostatitis?(UPSC-1 I 0)
cancer a) 3 days b) 4-6 weeks
d) Guiding transurethral resection of prostate cancer c) 7-14days d)2-14days
2603. The most common complication of transurethral 2612. Screening of prostate Ca is done by -(PGI Nov. I 0)
resection of prostate (TURP)- (Corned 07) a) Transrectal USG b) Digital rectal exam
a) Erectile dysfunction c) PSA d) CT scan
b) Retrograde ejaculation e) Urodynamic study
c) Urinary incontinence 2613. In follow up ofBPH, most important indication of
d) Urethral stricture disease surgery is - (AIIMS Nov I 0)
2604. Which one of the following is used as an irrigation a) Prostate size >75gm
solution during transurethral resection of the b) Single episode of UTI requiring 3 day antibiotics
prostate- (Corned 08) c) Cannot use medication due to hypertension
a) 1.5% glycine b) Physiological saline d) Bilateral hydronephrosis
c) Ringer's lactate d) 5% dextrose 2614. 60 yrs old male with poor stream of urine, post void
2605. Trans rectal ultrasonogram in evaluation of residual urine is 400ml, b/1 hydronephrosis and
carcinoma prostate most useful for- (AI 08) prostate weighing 70 g. His urea is 120 and creat
a) Taking guided biopsy 3.5. Ideal "next immediate" step- (AIIMS Nov I 0)
b) Identifying seminal vesicle invasion a) Catheterize with Foley catheter
c) Nodal sampling b) bll PC Nephrostomies
d) Measuring in extent of invasion c) CT to rule out ca
d) MRI pelvis

2597)a 2598)b 2599)c 2600)b 2601)b 2602)c 2603)b 2604)a 2605)a 2606)a 2607)c,e2608)b 2609)c 2610)c
2611)b 2612)b,c 2613)d 2614)a
SURGERY [ 305]

2615. Which of the following is the most common cause of 2625. Circumcision is done in a child in which of the
delayed urinary tract obstructive symptoms after following conditions- (TN 91)
TURP- (AI 11) a) Phimosis b) Recurrent balanitis
a) Stricture of the Navicular fossa c) Paraphimosis d) All of the above
b) Stricture of the membranous urethra 2626. Commonest late complication of traumatic
c) Stricture of the bulb of urethra rupture of urethra is- (JIPMER 92)
d) Bladder neck stenosis a) Diverticulam b) Retrograde ejaculation
2616. During TURP, surgeon takes care to dissect above c) Stricture d) Choradee
theverumontonumtopreventinjuryto- (AI 11) 2627. Which is not true ofHypopadias- (AIIMS 9 2)
a) External Urethral Sphincter a) Chordee is reversed after 5 years
b) Urethral crest b) Glandular type needs no treatment
c) Prostatic Utricle c) Circumcission should not be done
d) Trigone of bladder d) Surgical correction has good results in infancy
2617. Gleason score: all are true except- (AIIMS May 11) 2628. Rupture of membranous Urethra occurs more
a) Used for grading prostate cancer commonly due to- (AIIMS 92)
b) Scores range from 1-1 0 a) Thin unsupported wall b) Fixity of Urethra
c) Higher the score, poorer the prognosis c) Angulation d) Proximity to bladder
2629. Rupture of bulbar urethra is associated with all
the following except- (AIIMS 92)
a) Retention ofUrine b) Pelvic fracture
c) Urethral hemorrhage d) Perineal hematoma
2630. Posterior urethral valves are commonly situated
URETHRA & PENIS a) Above verumontanum (JIPMER 79, Delhi 93)
b) At verumontanum
2619. Features of carcinoma penins are all except- c) Belowverumontanum
a) Circumcision soon after (PGI 85, Kerala 87) d) Bladderneck
birth provides total immunity 2631. Penis is curved in downward direction in all types
b) Metastatic to inguinal nodes ofhypospadius except- (PGI 80, AIIMS 80)
c) Surgery treatment of choice a) Glandular b) Coronal
d) Transitional cell carcinoma c) penile d) Perineal
2620. Commonest type of hypospadias is- (JIPMER 87) 2632. Epispadias in relation to hypospadias-
a) Glandular b) Coronal a) Is more common (PGI 81, UPSC 88)
c) Penile d) Penoscrotal b) Less common
e) Perineal c) Occures with the same frequency
2621. In case of pelvic fracture with urethral injury, d) Is difficult to treat
the most important first step in management is- 2633. In children persistent priapism may result due to
a) Repair in injured urethra (AIIMS 84) a) Thrombosis of venous plexus (PGJ 81,
b) Fixation of pelvic fracture b) Leukaemia AIIMS 86)
c) Treatment of shock and haemorrhage c) Wtlm'stumour
d) Splinting urethra with catheters d) Trauma
2622. Posterior urethral valve are commonly observed 2634. Which type of hypospadias usually does not
in - (AIIMS 84) require any treatment- (JIPMER 80, AMU 89,
a) Boys b)Girls a) Penile b) Penoscrotal Assam 96)
c) Adult males d) Adult females c) Coronal d) Glandular
2623. Commonest site for post gonococcal stricture 2635. In severe hyposadias the possibility of an intersex
urethra is - (AIIMS 87, Jipmer 87) problem is settled by· (AIIMS 81, AMU 87)
a) Penoscrotal Jn. b) Bulb a) Careful inspection of genitals
c) Distal spongy urethra d) Membranous urethra b) Biopsy for gonadal tissue
2624. Narrowest part of the urethra is-(PGI 88,Jipmer 87, c) Karyotyping
a) Intenal meatus AI 89) d) Hormone assay
b) Membranous urethra 2636. Bleedig penile ulcer is seen in all except -
c) Bulbous urethra a) Syphilis b) LGV (Kerala 94)
d) External meatus c) Chanchroid d)Granulomainguinale

2615)d 2616)a 2617)b 2618)c 2619)d 2620)a 262l)c 2622)a 2623)b 2624)d 2625)d 2626)c 2627)a
2628)b 2629)b 2630)c 2631)a 2632)b 2633)b 2634)d 2635)c 2636)a
SURGERY [ 306]

2637. The cause of death from cancer penis in the 2648. Hypospadias true is/are- (PGI 02)
terminal stages is usually by- (Karn 94) a) Spatulated penis
a) Inanition b) Ventral hood
b) Secondaries liver c) Meatal stenosis
c) Lung secondaries d) Opens on ventral aspect of penis
d) Bleeding form femoral or iliac vessels 2649. True about congenital short urethra- (PGI 03)
2638. Treatment of fracture pelvis with rupture urethra a) Urethra is short
b) Opening is always ventral
is- (Kerala 95)
c) Prepuce deficient ventrally
a) Supra pubic cystostomy
d) Splitting of the two secrotum in the midline
b) Explore and correct the fracture, repair urethra e) Spatulated penis
c) Catheerisation 2650. Urinary retention in child is most commonly caused
d) Urethrogram to access injury by- (PG/03)
2639. All of following are seen in hypospadias except- a) Metal scab with ulceration
a) Ectopia vesicae b) Hooded prepuce b) Post.urethral valve
c) Chordee · d)Infertility (AI96) c) Urethral stricture
2640. Palable fibrous plaque on dorsa penile shaft d) Epispadius
indicates- (Karn 95) e) Congenital short penis
a) Paget's disease b) Potter's syndrome 2651. Posterior urethral valve is diagnosed by- (PGI 03)
c) Prehn's sign d) Peryonie's disease a) MJ b)MCU
2641. The following are complocations of stricture c) CT scan d) X-ray pelvis
urethra except- (Karnat 96) e) USG
a) Perurethral abscess b) Inguinal hernia 2652. True about Hypospadiasis- (PGI 03)
c) Hydronephrosis d) Papilloma ofbladder a) Meatal stenosis
b) Dorsal hood
2642. Commonest cuase of urethral stricture in a young
c) Proximal opening of meatus
person is - (Karnat 96)
d) Lateral curvature
a) Trauma b) Gonococcal e) Urethral stricture common
c) Syphilis d) Tuberculosis 2653. Catheterisation should not be done in case of acute
2643. If a patient with a suspected fracture of the pelvis retention of urine due to- (PGI 03)
has some bleeding form the urethra and is unable to a) Benign prostatic hyperplasia (BPH)
pass urine- (UPSC 97) b) Ca prostate
a) He should be encouraged to pass urine after c) Stricture urethra
beign given antibiotics and analgesics d) Rupture urethra
b) He should be immediately catheterised in the ward e) Post operative retention
c) A hot water bottle should be given followed by 2654. Urine extravasation occurs in the following case
injection of carbachol of penile urethral rupture, EXCEPT- (Jipmer 03)
d) He should be prepared for surgery and a) Ischiorectal fossa
catheteristion attempted in the O.T. b) Scrotum
2644. Which of the following urethral anomaly is the c) Abdominal wall
mostcommon- (TN99) d) Below superficial fascia of penis
2655. There is a high risk of renal dysplasia in-
a) Hypospadias b) Pinhole meatus
a) Posterior urethral valves (AIIMS 03)
c) Epispadias d) Stricture urethra
b) Bladder extrophy
2645. Circumcission is included in management ofCa c) Anorectal malformation
penis at- (PGI 98) d) Neonatal sepsis
a) Glans b) Prepuce 2656. Indications of circumicision are all except-
c) Glanduloprepucial d) Shaft of penis a) Chronic balanoposthitis (Mahara 02)
2646. About Ectopia vesicae, following is true except- b) Jew religion
a) CA bladder may occur (PGI 98) c) Carcinoma penis
b) Ventral curvature of penis d) Paraphimosis
c) Incontinence of urine 2657. Which of the following is not a content of the
d) Visible uretero - vesical efflux pudendal canal- (Kerala 04)
2647. True about hypospadias is all except- (PGI 0 1) a) Pudendal nerve
a) Bifid scrotum b) Meatal stenosis b) Internal pudendal artery
c) Mental Retardation d) Spatulated glans c) Internal pudendal vein
e) Dorsal chordee d) Nerve to obturator internus

2637)d 2638)a 2639)a 2640)d 264l)d 2642)a 2643)d 2644)a 2645)b 2646)b 2647)a,c 2648)c,d 2649)a
2650)a 2651)b 2652)a,b,c 2653)c,d 2654)a 2655)a 2656)c 2657)d
SURGERY [ 307]

2658-.For treatment of the ectopic-vesicle, which of the 2667. Epispadias is associates with? (AI 08)
following bone is divided to reach the site - a) Bifid pubic symphysis b) Chordee
a) Pubic rami b) Iliac bone ( UPPGMEE 04) c) Anal atresia d) Intestinal obstruction
c) Ischium bone d) Symphysis 2668. A young man gets into a fight after taking bear and
2659. In hypospadias all are seen except- ( UPPGMEE 04, is kicked by the lower abdomen. There was pelvic
a) Hooded penis PGI99) fracture. Blood at meatus. Most likely cause is -
b) Dorsal chordee a) Rupture of membranous urethra (Manipal 08)
c) Spatulated glans b) Bulbar urethral injury
d) Meatal stenosis c) Bladder rupture
d) Kidney laceration
2660. The recent treatment of short bridle passable
2669. All are features of membranous urethral injury
stricture of urethra in the penile and bulbous
except- (Manipal 08)
urethra is - (MARE 05)
a) Blood at meatus b) Retension of urine
a) Internal urethrotomy with Thompson-Walker's
c) Pelvic fracture d) All of the above
urethrotome 2670. Smegma is secreted by- (DPGEE 08)
b) Optical internal urethrotomy a) Tyson gland b) Brenner gland
c) Syme's operation c) Cowper's gland d) Bartholin's gland
d) Wheelhouse operation 2671. All are indications for penile angiography except-
2661. During urethral catheterization in male patients, a) Painful priapism (AJIMS May 09)
resistance is encountered at the following sites b) Peyronie's disease
except- (ICS 05) c) Erectile dysfunction
a) Base of navicular fossa d) Arterio-venous malformation
b) Mid-penile urethra 2672. Most frequent causes of acute retention of urine
c) Urogenital diaphragm include all except- (DELHI PG Mar. 09)
d) Bulbomembranous junction a) Meatal ulcer with scabbing in children
2662. A patient is brought to the hospital with history of b) Haemorrhoidectomy
RTA8 hours back. A few drops ofblood were noted at c) Hemiorraphy
the external urethral meatus. He has not passed d) Fecal impaction
urine and his bladder is palpable per abdomen. What 2673. Most common site or urethral carcinoma in men is-
is the probable diagnosis- (AI 07) a) Bulbomembranous urethra (AI 10)
a) Ureteral injury with extravasations of urine in the b) Penile urethra
retro peritoneum c) Prostatic urethra
b) Urethral injury d) Fossa Navicularis
267 4. The Grayhack shunt is estab]jshed between-
c) Rupture bladder
a) Corpora cavernoso and corporo spongiosa
d) Anuria due to hypovolemia
b) Corpora cavernosa and saphenous vein
2663. Ayound man gets into a flight after taking beer and
c) Corpora cavernosa and dorsal vein (AI 10)
iskicked by the lower abdomen. There was pelvic
d) Corpora cavernosa and glans
fracture. Blood at meatus. Most likely cause is- 2675. Priapism in a young male could occur because of-
a) Rupture to membranous urethra (MAHE 07) a) Testicular cancer (UP SC II 10)
b) Bulbar urethral injury b) Carcinoid tumour of appendix
c) Kidney laceration c) Leukaemia
2664. All the features of membranous urethral injury d) Penile cancer
except- (MAHE 07) 2676. True about hypospadias- (PGI May 10)
a) blood of meatus b) Retention of urine a) Defect seen in ventral penis
c) Pelvic fracture d) All of the above b) Always a!w chordee
2665. All are true about Peyronie's disease except- c) A/w hooded prepuce
a) Selfli:rniting UP 07, UP 06) d) Circumcision should be avoided
b) Medial treatment is effective 2677. A five year old child presents with ballooning of
c) Association with Dupvytren's contracture prepuce after micturation. Examination of penis
d) Calcified plaques reveals prepucial adhesions which the following
2666. Optical urethroplasty is done in- (UP 07) the best treatment- (AI 11)
a) Congenital stricture of urethra a) Adhesiolysis & dilatation
b) Hypospadias b) Circumcision
c) Epispadias c) Dorsal slit
d) Testiculartumour's d) Conservative management

2658)b 2659)b 2660)b 2661)a 2662)b 2663)a 2664)d 2665)b 2666)a 2667)b 2668)a 2669)d 2670)a 267l)a
2672) d 2673) a 2674) b 2675) c 2676) a,c,d 2677) b
SURGERY [ 308]

2688. Disseminated seminoma is treated by- (PGI 88)


a) CT or RT and orchidectomy
b) onlyRT
c) OnlyCT
d) Retroperitoueallymphnode dissection
2689. Regarding spermatocele which is correct -(PGI 88)
a) Occurs in head of epidydimus
b) Barley water fluid in appearance
c) Tender
d) Contain spermatozoa
2690. Testicular tumour can simulate- (PGI 88)
a) Hydrocele
b) Hematocele
c) Acute epidydimo orchitis
d) Chronic orchitis
2691. In seminoma testis the treatment is- (PGJ 89)
a) Surgery b) Radiotherapy
c) Chemotherapy d) Stibesterol
2692. Which of the following is the differential diagnosis
of cancer testes - (PGI 90)
a) Hydrocele. b) Hematocele
c) Spermatocele d) Granulomatous orchitis
2693. Fournier's gangrene occurs in the- (JIPMER 90)
a) Toes b) Scrotum
c) Fingers d) Muscles
2694. Testicular cancer is common in- (AI 91)
a) Ectopic testis b) Undescended abdominal testis
c) Atropic testis d) Anteverted testis
2695. Differential diagnosis of acute funicultitis with a
small inguinal swelling is - (TN 91)
TESTIS & SCROTUM a) Undescended testes
b) Acute orchitis
2683. Ectopic testis is found in all location except-
c) Lymphadentis
a) Lumbar b) Perineal (AIIMS 87)
c) Intra abdominal d) Inguinal d) Small strangulated inguinal hernia
2684. Which is not true regarding varicocele- (AI 88) 2696. Orchitis without epididymitis is seen in- (Al92)
a) Testicular veins involved a) Gonorrhoea b) Tuberculosis
b) More common on the right side c) Syphilis d) Chlamydia infection
c) May be the first feature of a renal tumaur 2697. Which of the following closely mimics testicular
d) Feels like a bag of worms malignancy- (JIPMER 81, DELHI 79,93, PGI 93)
2685. Commonest testicular malignancy is- (AI 89) a) Hydrocele b) Haematocele
a) Seminoma b) Teratoma c) Spematocele d) Cyst of epididymis
c) Choriocarcinoma d) Embryonal cell carcinoma 2698. The life of preserved semen for artificial
2686. Most malignant testicular tumour- (AIIMS 87) insemination is- (PGI 82, DNB 90)
a) Embryonal cell carcinoma a) One year b) Two year
b) Seminoma c) Five year d) Ten year
c) Choriocarcinoma e) Fifty year f) None of the above
d) Teratoma 2699. Incompletlty descended testis is commonest
2687. It is true about seminoma testis that~ (AJIMS 85) on- (JIMPER78,79,PGJ85)
a) It commonly occurs in the 6th decade a) Right side b) Left side
b) An undescended testes is more liable to develop c) Both side d) Right sided only
this tumour 2700. Complication of ectopic testis is- (KERALA 94)
c) Histologically it resembles dysgerminoma of the a) Seminoma b) Atrophy
ovary c) Torsion d) All
d) It is radioresistant

2678)a 2679)b 2680)c 268l)d 2682)b 2683)a,c 2684)b 2685)a 2686)c 2687)b,c 2688)a 2689)a,b,d
2690)a,b,c 2691)a,b,c 2692)a,b,c 2693)b 2694)b 2695)d 2696)c 2697)b 2698)d 2699)a 2700)c
SURGERY [ 309]

2701. Torsion of the testis can be distinguished from 2711. Hydrocele is labelled 'vaginal'when it is-
epididymoorchitis by- (KARN 94) a) Limited to sorotum (AIIMS 96)
a) Elevation of the testis relieves the pain in torsion b) Upto inguinal canal
b) Elevation does not relieve the pain in torsion c) Conununicating into coelomic cavity
c) Fever is more in torsion d) Upto deep inguinal ring
d) Absence of tenderbess in torsion 2712. Treatment of a large hydrocele in an infant-
2702. A 5-year old male child has been brought with a a) Repeated aspirations (KERALA 97)
complaint that there is only one testis in the b) Ligation of sac at the opening if ingunal canal
scrotum. On examination, it is found that the c) Herniotomy
testis on the opposite side is felt in the inguinal d) Eversion of sac
2713. Cause of hydrocele is infants- (CMC 98)
canal. The patient should be advesed- (UPSC 96)
a) Pateint processus vaginalis
a) Orchiopexy
b) Patent gubernaculum
b) To wait till puberty c) Impaired drainage
c) Orchidectomy d) Epididymal cyst
d) Administration of androgens e) Infection
2703. Seminoma testis is seen- (ASSAM 96) 2714. 58 years old male presenting with acute onset of
a) Always in undescendent testis varicocle on left side most prbobable cause -
b) Always bilateral a) Ca testes (CMC 98)
c) Occurs in younger b) Epididymitis
d) Occrus in elderly c) Inguinallymphnodes
2704. Testis tumor is associated with secondary d) Cakidney
hydrocele in- (KARN 95) 2715. Varicocele ofpampiniform plexus of veins has all
a) 1% of cases b) 10% of cases the following characteristics except- (MP 97)
c) 20% of cases d) 30% of cases a) Negative transillumination test
2705. In differential diagnosis of epididymo-orchitis & b) Reducible
torsion it is important that- (AIIMS 95) c) Cough impluse is present
a) Elevation, of testis in torson relieves pain d) Frequently on right side
b) Elevation of testis in epididymo orchitis relieves 2716. Most common cause of surgically treatment male
pain infertility is - (MAHE 98)
c) Tenderness is characteristic of torsion of testis a) Varicocele b) Cryptorchidism
d) Fever is characteristic of epididymo orchitis c) Stricture urethra d) Epidydimitis
2706. The lymph nodes first involved in cancer of the 2 717. Orchidopexy in an undescended testes decreases
skin ofthe scrotum are- (KARNAT 96)
all except- (AIIMS 99)
a) Tumour incidence b) Epididymoorchitis
a) Superfical injguinal b) External iliac
c) Torsion d)Avoidsexualambiguity
c) Para aortic d) Gland of eloquent
2718. A 25 years old male who had sex recently presented
2707. Subcapsular orchiectomy is done for cancer of-
with painful testes and the pain was not relieved on
a) Testes b) Prostate (AP 96) elevation of the testes • Find the diagnosis from the
c) Penis d) Urethra following- (AIIMS 99)
2708. Which one of the following statement is true of a) Epididymo-orchitis b) Torsion testis
undescended testis- (UPSC 97) c) Testicular malignancy d) Pyocele
a) Usually descends spontaneously at puberty 2719. Dermoid arises from- (PGI 97)
b) Orchipexy to be done of no descent by puberty a) Pluripotent cell b) Totipotent cell
c) Has a higher incidence of malignancy c) Ectoderm d) Mesoderm
d) Maintains normal sperm production 2720. In testicular ca, investigation not done is -(PGI 98)
2709. Ligation of cord in orchidectomy for treatment a) Aortography b) CT
of testicular tumor is done at- (PGI 96) c) Biopsy d) SerumAFP
a) External ring b) Internal ring 2721. Seminoma correspond to- (PGI 99)
c) Base of scrotum d) Just above epididiymis a) Choriocarcinoma b) Dysgerminoma
2710. After the division oftesticular vein for treatment c) Granulosa tumor d) Leutal cyst
of varicose vein, venous dreainage oftestes is by- 2722. True regarding epispadias is- (PGI 99)
a) Cremasteric veins (PGI 96, Jipmer 80, a) Always associated with dystrophy of bladder
b) Dorsal vein of penis Delhi 90) b) Penile
c) Pampiniform pleus c) Conunon congenital anomaly
d) Internal pudendal vein d) Urethra opens on ventral side of penis

270l)b 2702)a 2703)c 2704)b 2705)b 2706)a 2707)b 2708)c 2709)b 2710)a 27ll)a 2712)c 2713)a 2714)d
2715)d 2716)a 2717)a 2718)b 2719)b 2720)c 272l)b 2722)a
SURGERY [ 310]

2723. The treatment of metastatic testicular carcinoma is 2732. All ofthe following abnormalities are predisposing
a) Bleomycin, Etoposide, Cisplatin (PGI 99) causes for torsion ofthe testis except-( COMED 06)
b) Vinblastine, Etoposide, Cisplatin a) Inversion of testis
c) Doxorubicin, 5-FU mercaptopurine b) Low investment of tunica vaginalis
d) Methotrexate, 5-FU, Vincristine c) Between 10 to 25 years of age
2724. Germ cell tumours oftestis are- (PGI OJ) d) Seperation of the epididymis
a) Semmoma b) Teratoma 2733. Which of the following is not seen in testicular
c) Leydig cell tumour d) Gynandroblastoma carcinoma? (APPG 06)
e) Rhabdomyoma a) Abdominal lump
2725. Predisposing factor of Testicular germ cell tumor- b) Epidydimo orchitis
a) Cryptorchidism (PGI 03) c) Inguinal lymphadenopathy
b) Testicular feminization syndrome d) Hydrocele
c) Klinefelter's syndrome 2734. Orchidopexy for undescended testis is recommended
d) Radiation at which age? (Karnataka - PG MEE- 2006)
e) Trauma a) 3 months b) 6 months
2726. Testicular tumour markers are- (PGI 03) c) 2 years d) 5 years
a) b-HCG b) AFP 2735. Ideal age for orchiopexy for cryptorchidism is-
c) LDH d) CA-125 a) At 6-10 yrs b) At 1-2 yrs (Aiims May 07)
e) CEA c) At puberty d)< 1 yr of age
2727. Stage-II testicular teratoma is treated by- 2736. All are features ofFournier's'Gangrene except-
a) Orchidectomy+ RPLND (AMU 05) a) Testicles are involved (MAHE 07)
b) Orchidectomy+ Chemotherapy b) Obliterative arteritis seen
c) Orchidectomy c) Hemolytic streptococci, isolated
d) Radiotherapy d) Necrotizing fascitis
2728. Testicular tumour most sensitive to radiation 2737. High inguinal orchidectomy specimen showed
is- (UPSC05) tumor testis with involvement of epididymis
a) Teratoma b) Lymphoma without vascular invasion; Stage is- (MAHE 07)
c) Mixed germ cell tumour d) Seminoma a)T 1 b)T2
2729. A 16 -year old boy presents with acute onset pain c)T3 d)T4
in the left testis. The following statements about 2738. A 30 year old male patient presents with sudden
his management are true except- (AIIMS Nov 05) onset swelling and pain over the right hemiscrotum.
a) The patient should be prescribed antibiotics and On examination the scrotum is redened and tender.
asked to come after a week
Which of the following statement about the affecting
b) Colour flow Doppler will be very useful in
condition is not true? (AI 08)
diagnosis
a) Probable diagnosis is torsion
c) Scrotal exploration should be done without
delay if doppler is not available b) The right testis is likely to ride high in the scrotal
d) If left testis is not viable on exploration, patient compartment.
should undergo left Orchidectomy and right c) If torsion confirmed, trest with antibiotics and
orchidopexy. analgesics and perform corrective surgery
2730. All ofthe following clinicopathologic features are immediately
seen more often in seminomas as compared to non- d) If torsion confirmed, treat with antibiotics and
seminomatous germ cell tumors of the testis except- analgesics and perform corrective surgery after
a) Tumors remain localized to (AIIMS May 2005) 14 days
testis for a long time 2 739. Most common testicular tumor in prepubertal adults
b) They are radiosensitive is - (Aiims May 08)
c) They metastasize predominantly by lymphatics a) Yolk sac tumor b) Embryonal cell Ca
d) They are often associated with raised levels of c) Semillorna d) Teratoma
serumAFP and HCG 2740. All are features ofFournier's gangrene except-
2731. Which of the following statements in true regarding a) Testicles are involved (Manipal 08)
testicular tumors? (AI 06) b) Obliterative arteritis seen
a) Are embryonal cell carcinomas in 95% of cases c) Haemolytic streptococi, E.coli, staphylococci, Cl.
b) Bilateral in up to 10% cases Welchi can be isolated
c) Teratomas are more common than seminomas d) Necrotising fascitis
d) Usually present after 50 years of age

2723) a 2724) a,b 2725) a,b,c,e 2726) a,b,c 2727) a 2728) d 2729) a 2730) d 2731) b 2732) b 2733) c 2734) c
2735) d 2736) a 2737) a 2738) d 2739) a 2740) a
SURGERY [ 311 ]

2741. High inguinal orchidectomy specimen showed 2751. Which of the following is not true about testicular
teratoma testis with involvement of epididymis; stage tumor? (Maharashtra 1 0)
is- (Manipal 08) a) About 95% are embryonal tumors
a)T 1 b)T2 b) 10% are bilateral
c) T3 d) T4b c) Pulmonary metastases suggest that the tumor is
2742. Which is false about hydrocele? (APPG 08) teratoma
a) Almost always fluid is transudate d) Lymphatic spread is to retroperitoneal and
intrathoracic lymph nodes
b) Get above the swelling
2752. Surgery for undescended testis is recommended
c) Testis is separate from swelling
atwhatage- (AI ll)
d) Obscures inguinal hernia a) 6 months b) 12 months
27 43. Operative managements of varicocele are indicated c) 24 months d) 36 months
in which ofthe following condition (s) -(PGI Dec 08) 2753. Surgery for undescended testes should be
a) Ipsilateral testis small size performedbeforewhatage? (AI 11)
b) Oligospermia on semen analysis a) 6 months b) 12 months
c) Grade - 3 varicocele (large size) c) 24 months d) 36 months
d) Signs or symptoms present
e) Subclinical presentaion NEOPLASIA
2744. Which of the following is false about testicular
tumor- (AIIMSMay09) 2754. Which of the following tumours are sugrically
a) Pure seminoma is more aggressive than curative- (PGI 88)
nonseminoma a) Pheochromocytoma b) lnsulinoma
c) Glucogonoma d) Appendicularcarcinoid
b) Seminoma is radiosensitive
e) All
c) Seminoma can be treated by orchidectomy with
2755. Commonest intra-abdominal tumour below 2 years-
radiotherapy
a) Neuroblastoma b) Wilm's tomour (PGI 85)
d) Seminoma spreads commonly through lymphatics c) Hepatoblastoma d) Lymphoma
27 45. Orchidopexy for cryptorchidism is done at the age 2756. Commonest intra-abdominal tumour between 2-
of- (AIIMSNov09) 5 years - (PGI 86)
a) 1 to 2 years b) 5 to 6 years a) Neuroblastoma b) Wilm's tomour
c) Puberty d) Neonatal period c) Hepatoblastoma d) Lymphoma
27 46. A male with azoospermia found to have normal FSH 2757. Lipoma becomes malignant commonly at which
& testosteron levels & normal size testes. Probable site- (JIPMER 88)
cause is- (AIIMS Nov 09) a) Subcutaneous b) Retro-pertioneal
a) Vas obstruction c) Sub-aponeurotic d) Intermuscular
b) Kallman syndrome 2758. Calcifying epithelioma is also known as-(AJ/MS 86)
c) Undescended testis a) Pilomatrixoma b) Myoblastoma
d) Klinefeltor's syndrome c) Calcinosis cutis d)Dennato:fibromalenticulare
2747. All of the following are true about cryptorchidism, 2759. Notaneuroglialtumor- (Kerala 95)
a) Shwanoma b) Astrocytoma
except- (AIIMS Nov 09)
c) Medulloblastoma d)Apendymoma
a) Cryptorchidism is a risk factor for testicular tumor
2760. Pseudolymphoma is seen in- (PGI 97)
b) Seminoma is the most common tumor
a) Sjogren's syndrome
c) Contalateral testis is also at risk b) SIE
d) Orchidopexy reduces the risk of malignancy c) Mixed connective tissue disease
2748. Orchidopexy for cryptorchidism is done at the age d) Behcet's syndrome
of- (AIIMSMay IO) 2761. Opsomyoclonus is seen in- (PGI 97)
a) 1 to 2 years b) 5 to 6 years a) Neuroblastoma b) Meningioma
c) Puberty d) Neonatal period c) Medulloblastoma d) Wilms tumor
2749. Lord'splicationisdonefor- (AI 10) 2762. Most favorable prognosis after radiotherapy is in-
a) Inguinal hernia b) Testicular cancer a) Melanoma b) Teratoma (PGI 97)
c) Hydrocele d) Testicular varices c) Seminoma d) Desmoid
2750. Best time for surgery of undescended testis is- 2763. Mosthaemorrhagictumoris- (PGI98)
a) Justafterbirth (AI 10) a) Leiomyosarcoma stomach
b) 6 months of age b) Ca stomach
c) 12 months of age c) Adeno Ca GB
d) 24 months of age d) Ca pancreas

2741)a 2742)c 2743)a,b,c:d 2744)a 2745)a 2746)a 2747)d 2748)a 2749)c 2750)c 2751)a 2752)a 2753)b
2754)a,b,d 2755)a 2756)a 2757)b 2758)a 2759)a 2760)a 2761)a 2762)c 2763)a
SURGERY [ 312]

2764. Pancoast tumour is seen with cancer of- (PGI 99) 2776. CA-125 associated with- (PG/02)
a) Apical lobe oflung b) Lingual lobe a) Colon Ca b) Breast Ca
c) Thyroid d) Pyriform fossa c) Ovarian Ca d) Bronchogenic ca
2765. All are recognised tumour markers except-(PGJ 99) e) Pancreatic ca
a) BetaHCG b)Beta2microglobulin 2777. CEA is associated with- (PG/02)
c) Alpha fetoprotein d) Acid phosphatase a) Adenocarcinoma of colon b) Pancreatic ca
27 66. Erythema chronicum migrans is seen in- (PGI 99) c) Neuroblastoma d) Ovarian ca
a) Lyme's disease b) Glucagonoma e) Prostatic ca
c) Gastrinoma d) Phaeochromocytoma 2778. Psammomabodyfoundin- (PG/03)
2767. Cancers associated with excess fat intake are/is- a) Papillary Ca thyroid b) Follicular Ca thyroid
a) Breast b) Colon (PGI 2000) c) Medullary Ca thyroid d) Anaplastic Ca thyroid
c) Prostate d) Lung e) Lymphoma thyroid
e) Thyroid 2779. 'Irue statement about soft tissue sarcoma is/are-
2768. Trueaboutkeratoacanthoma- (PGI2000) a) Liposarcoma is rare (PGI 04)
a) Benign tumor b) Incisional biopsy is needed when size> Scm
b) Malignant skin tumor like squamous cell carcinoma c) ENSC is diagnostic
c) Treatment same as for squamous cell carcinoma d) TNM staging done
d) Easy to differentiate from squamous cell Ca. e) Radio sensitive
histologically 2780. 'Irue about Sential Lymph node biospy- (PGI 04)
e) Treatment is masterly inactivity a) Special OT is requried
2769. Featuresofcarcinoidare- (PGI 2000) b) Blue dyes injected
a) Wheezing b) Cyanosis c) Contraindicated if axillary LN is involved
c) Flushing d) Mitral valve involvement d) It is done to avoid inadvertent axillary LN biopsy
e) Clubbing e) Radioactive dye is used
2770. Gleason's staging is done in- (PGI 2000) 2781. Lynch II syndrome is associated with which ofthe
a) Ca prostate b) Ca Pancreas following carcinoma- (PGI 04)
c) Ca kidney d) Ca Cx a) Ovary b) Colon
2771. 5 H. indole acetic acid in urine is seen in- c) Breast d) Thyroid
a) Carcinoid (PGI 2000) 2782. Buschke- Lowenstein tumor is- (TN03)
b) Pheochromocytoma a) Molluscum contagiosum
c) Hirschsprung's disease b) Condyloma lata
d) Wilm's tumor c) Giant condyloma accuminata
2772. True about Carcinoid is all except- (PGI 2000) d) Metastasise
a) Common in distal3/4 appendix 2783. Which of the following is not an oncological
b) Carcinoid sydrome is due to malignant emergency- (AIIMS 03)
transformation a) Spinal cord compression
c) Distal2/3 T.O.C. Apendicectomy b) Superior venacaval syndrome
d) Of all the carcinoid tumors in GIT 45% in appendix c) Tumorlysis syndrome
2773. Usesoftumormarkerare- (PG/01) d) Carcinoma cervix stage- III "B" with pyometra
a) Screening of a cancer 2784. Which one of the following is frequent cause of
b) Follow up of a cancer patient, esp. for knowing serum alpha feto protein level greater than 10
about recurrence times the normal upper limit- (UPSC 04)
c) Confmnation of a diagnosed cancer a) Seminoma
d) For monitoring the treatment of a cancer b) Metastaic carcinoma ofliver
277 4. Erythropoietin secreting tumor (s) - (PGI 01) c) Cirrhosis ofliver
a) Cerebellar hemangioblastoma d) Oat cell tumour oflung
b) Hepatoma 2785. Kaposi's Sarcoma- (SGPGI 04)
c) Renal cell Ca a) Does not occur in non HIV positive persons
d) Adrenal adenoma b) Has increasing incidence among AIDS patients
e) Fibromyomaofuterus c) No GI bleeding
2775. 'fine about soft tissue sarcoma- (PGI 02) d) Uncommon among homosexual HIV Positive
a) Lymphatic spread 2786. Which one of the following is rare site for
b) Enlarged size metastasis- (SGPGI 04)
c) Pseudoencapsulated a) Vertebrae b) Skull
d) Spread though musculoaponeurotic plane c) Pelvis d) Forearm and leg bones

2764)a 2765)b 2766)a 2767)a,b,c 2768)a,e 2769)a,c 2770)a 277l)a 2772)b,d 2773)a,b,d 2774)a,b,c,e 2775)b,c,d
2776) c 2777) a,b,d 2778) a 2779) a,b,d 2780) b,d,e 2781) a,b 2782) c 2783) d 2784) b 2785) b 2786) d
SURGERY [ 313]

2787. All are seen in carcinoid syndrome- (HPU 05) 2800. Screening increases life span in which ca -
a) Diarrhoea b) Constipation a) Breast b) Colon (PGI June 07)
c) Liver metastasis d) 5-HT secretion c) Prostate d) Lung ca
2788. In which of the following tumors alpha fetoprotein 2801. The commonest site of lymphoma in the
is elevated- (AIIMS Nov 05) gastrointestinal system is- (Comed 07)
a) Choriocarcinoma b) Neuroblastoma a) Small bowel b) Stomach
c) Hepatocellular carcinoma d) Seminoma c) Large intestine d) Oesophagus
2 789. In which ofthe following disease, the overall survival 2802. Most common location of spinal tumors-
is increased by screening procedure- (AI 05) a) Intramedullary (AIIMS Nov 07)
a) Prostate cancer b) Lung cancer b) Intradural extramedullary
c) Colon cancer d) Ovarian cancer c) Extradural
2790. Which one of the following soft tissue sarcomas d) Equally distributed
frequently metastasizes to lymph nodes- (AI 05) 2803. Radiotherapy is the treatment of choice for which
a) Fibrosarcoma one of the following tumors? (UPSC-I! 08)
b) Osteosarcoma a) Verrucous carcinoma b) Malignant melanoma
c) Embryonal Rhabdomyosarcoma c) Marjolin's ulcer d) Rodent ulcer
d) Alveolar soft part sarcoma 2804. What is the treatment of choice in desmoid tumors-
2791. The prognosis of rhabdomyosarcoma is likely to be a) Irradiation (UPSC-II 08)
poorifthesiteofthetumouris- (AI 06) b) Wide excision
a) Orbit b) Para testicular c) Local excision
c) Extremity d) Urinary bladder d) Local excision following radiation
2792. Fine needle aspiration cytology is not suitable 2805. Most common site for carcinoid tumor-
for diagnosing- (AI 06) a) Appendix b) Liver (Manipal 08)
a) Tubercular lymphadenitis c) Stomach d) Rectum
b) Papillary carcinoma thyroid 2806. Which is not having underlying malignancy?
c) Plasmacytoma a) Paget disease of bone (APPG 08)
d) Aneuryrnal bone cyst b) Paget disease of nipple
2793. Spontaneouslyregressingtumoursare-(PGIJune06) c) Paget disease of vulva
a) Malignant melanoma b) Neuroblastoma d) Paget disease of anal region
c) Ewing's sarcoma d) Wilm's tumour 2807. Glomus tumor is seen in- (AIIMS Nov 08)
2794. All are correctly matched except- (PGI June 06) a) Liver b) Adrenals
a) BRCAI- Lung c) Pitutary d) Finger
b) BCL2 Apoptosis 2808. Concomitant chemoradiotherapy is indicated in all
c) Ch - 16 - Philadelphia chromosome ofthefollowingExcept- (AI 09)
d) APC - colon a) Stage ill B Ca cervix
2795. AD the following skin conditions are considered as skin b) T2 No Mo anal cancer
markers for internal malignancy except- (Karn 06) c) T2 No Mo Glottic cancer
a) Acanthosis nigricans b) Dermatomyositis d) Tl N2 Mo Nasopharyngeal cancer
c) Bullous pemphigoid d) Pemphigus vulgaris 2809. Smoking may be associated with all of the following
2796. Carcinoembryonic antigens (CEA) is increased in cancer's Except- (AI 09)
all except- (Aiims May 07) a) CaLarynx b)CaNasopharynx
a) Lung cancer b) Breast cancer c) CaBladder d) Ca Esophagus
c) Colon cancer d) Osteogenic sarcoma 2810. Dye used in chromoendoscopy for detection of
2797. Tumor marker for primary hepatocellular cancer- (AIIMS May 09)
carcinoma are all except- (Aiims May 07) a) Gentian violet b) Toluidine blue
a) Alpha feto protein b) Alpha 2 macroglobulin c) Hemotoxiline and eosine d) Methylene blue
c) PIVKA-2 d) Neurotensin 2811. Reparative granuloma ofJaw is treated by-
2798. All are precancerous except- (MAHE 07) a) Antibiotics (DELHI PG Feb. 09)
a) Lichen planus b) Verrucous carcinoma b) Wedge resection
c) Submucous fibrosis d) Leukoplakia c) Resection and bone grafting
2799. Dukes classification is used for- (UP 07) d) Curettage
a) Pancreas carcinoma 2812. The following bone tumour may cause dural deposits
b) Gastric carcinoma without causing bony changes- (DELHI PG Feb. 09)
c) Urinary bladder carcinoma a) Hodgkin's lymphoma b) Multiple myeloma
d) Colo-rectal carcinoma c) Secondaries d) Fibrous dysplasia

2787)b 2788)c 2789)c 2790)c 279l)c 2792)d 2793)a,b 2794)a,c 2795)d 2796)d 2797)b 2798) b 2799)d
2800)a,b,c 280l)b 2802)c 2803)d 2804)b 2805)a 2806)a 2807)d 2808)c 2809)None 2810)d 2811)d 2812)a
SURGERY [ 314]

2813. Carcinoma in which surgery is rarely indicated· 2823. A well differentiated follicular carcinoma ofthyroid
a) Osteosarcoma (PGI Nov 09) can be best differentiated from a follicnlar adenoma
b) Wilm's twnor by- (AIIMSMay 10)
c) Neuroblastoma a) Hurthle cell change
d) Rhabdomyosarcoma b) Lining of tall columnar and cuboidal cells
e) Hodgkin'slymphoma c) Vascular invasion
2814. Frozen section is/are used for· (PGINov09) d) Nuclear features
a) Enzyme 2824. Psammoma bodies seen in AlE- (AJIMSMay 10)
b) Intraop Histopathological examination a) FollicularCAofthyroid
c) Fat b) Papillary CA of thyroid
d) Acid fast bacilli c) Serous cystadenocarcinoma of ovary
e) To check surgical margin in tumor surgery d) Meningioma
2815. Common tumor of Posterior mediastinum are· 2825. Sister Mary Joseph nodnle is most commonly seen
a) Lymphoma (PGI Nov 09) with- (ADMSMay10)
b) Neuroblastoma a) Ovarian cancer b) Stomach cancer
c) Neurogenic tumors c) Colon cancer d) Pancreatic cancer
d) Thymoma 2826. Which of the following is NOT true about FAP?
e) Bronchogenic cyst a) ARinheritance (AJIMSMay10)
2816. MC germ cell tumor is· (PGINov09) b) Screening done by sigmoidoscopy
c) Polyps develop in late adulthood
a) Embryonal teratoma b)Dennoid
d) Epidermal cysts & osteomas may occur
c) Rhabdomyosarcoma d) Seminoma
2827. Smoking may be associated with all of the following
e) Sertoli cell CA
cancers, except- (AIIMSMay 10)
2817. A 50 yr. old male presents with obstructive symptoms.
a) CaLarynx b)CaNasopharynx
Biopsy of stomach reveals Gastrointestinal stromal c) CaBladder d) Ca Esophagus
tumor (GIS1). Most appropriate marker for GIST 2828. Neoadjuvant chemotherapy is used in all except-
is· (AJIMSNov 09) a) Esophageal Ca b) Breast Ca (DPG 10)
a) CD34 b)CD 117 c) Thyroid Ca d) Lung non-Small cell Ca
c) CD30 d) CD 10 2829. All of the following hereditary conditions predispose
2818. Lynch syndrome is associated with ~(AJIMS Nov 09) to CNS tumors, except- (DPG 10)
a) Endometrial ca, Colon ca & Ovarian ca a) Neurofibromatosis 1 and 2
b) Breast ca, Colon ca & Ovarian ca b) Tuberous sclerosis
c) Breast ca, Endometrial ca & Ovarian ca c) Von-Hippel-Lindau syndrome
d) Breast ca, Stomach ca & Colon ca d) Xeroderma pigmentosum
2819. Alpha feto protein is increased in- (AIIMS Nov 09) 2830. A 60-yearold male was diagnosed as carcinoma right
a) Hepatoblastoma b) Neuroblastoma lung. On CECT chest there was tumour of 5 x 5 em
c) Seminoma d) Renal cell carcinoma in upper lobe and another 2 x 2 em size tumour
2820. Desmoid tumor, treatment is- (AJIMS Nov 09) nodule in middle lobe. The primary modality of
a) Local excision treatmentis- (DPG 10)
b) Wide excision a) Radiotherapy b) Chemotherapy
c) Wide excision with radiotherapy c) Surgery d) Supportive treatment
d) Radiotherapy 2831. Most common site of minor salivary gland tumor?-
2821. Presence of nephroblastomatosis in a biopsy a) Pharyngeal cavities (Maharashtra 10)
b) Paranasal sinuses
specimen from Wilm's tumor ofleft kidney indicates
c) Retromolararea
high possibility of- (AIIMS Nov 09)
d) Nasal cavity
a) Denys-Drash syndrome
2832. Pseudoachalasia may be caused by-(Maharashtra 10)
b) Mutation in insulin like growth factor
a) Gastric adenocarcinoma b) Lung carcinoma
c) Increased risk of tumor in right kidney c) Pancreatic carcinoma d) All the above three
d) Lymph node metastasis 2833. Hypertension with hypokalemia and a 7-cm
2822. Tumors associated with infective etiology are all suprarenal mass suggests the diagnosis of-
except- (AIIMS Nov 09) a) Cushing's syndrome (Maharashtra 1 0)
a) Nasopharyngeal ca b) Adrenal carcinoma
b) Hepatocellular ca c) Hypernephroma
c) Non-small cell carcinoma lung d) Pheochromocytoma
d) Gastric ca

2813)e 2814)a,b,c,e 2815)b,c,e 2816)d 2817)b 2818)a 2819)a 2820)b 2821)c 2822)c 2823)c 2824)a 2825)b
2826) a 2827) None >b 2828) c 2829) d 2830) c 2831) c 2832) d 2833) b
SURGERY [ 315 J

2834. Mostcommonsiteofcolorectalmalignancy? (MH 10) 2847. Burning epigastric pain is due to- (TN 90)
a) Caecum b) Splenic flexure a) Vomiting b) Reflux esophagitis
c) Sigmoidcolon d)Rectum c) Duodenal ulcer d) Gastric ulcer
2835. Serum calcitonin may be elevated in -(UP SC 111 0) 2848. Intractable peptic ulceration with renal stones
a) Islet cell tumour of pancreas occurs in- (JIPMER 90)
b) Choriocarcinoma of ovary a) Zollinger Ellison syndrome
c) Carcinoid tumour of appendix b) Parathyroid adenoma
d) Medullary carcinoma of thyroid c) Milk, alkali syndrome
2836. 'Double duct' sign is diagnostic of- (UP SC II 1 0) d) MEN I syndrome
a) Periampullary carcinoma 2849. Embolotherapy using heat contrast is done in-
b) Gallbladdercarcinoma a) Renal tumours (TN 91)
c) Klatskin's carcinoma b) A-Vmalformation
c) Varicosa veins oflowr limb
d) Varicocele of testes
2850. Peyronies disease affects the- (TN 91)
a) Breast b) Vagina
c) Scrotus d) Penis
2851. Which is a non absorbable suture- (JIPMER 91)
a) Polypropylene b) Polyglycollic acid
c) Viccyl d) Collagen
MISCELLANEOUS 2852. Removal ofL1 Ganglion in sympathetomy results
in- (JIPMER 91)
2839. All ofthe following are cause of blood in stools in a) Impotence b) Retention of urine
children except- (AIIMS 85) c) Sterility d) Causalgia
a) Meckels diverticulum b) Carcinoma 2853. Gasless abdomen in X-ray is a sign of -(AIIMS 91)
c) Intussusception d) Juvenile polyp a) Acute pancreatitis
2840. Most common coagulopathy noted in surgical b) Necortising enterocolitis
patients is- (AIIMS 86, 88) c) Ulcerative colitis
a) Thrombocytopenia b)Afibrinogenemia d) Intussusception
c) Fibrinolysis d) Factor VIII defeciency 2854. 5-Fu is the chemotherapeutic agent of choice for
2841. Leucine aminopeptidase is elevated obstructions of- all except- (PGI 9 3)
a) Ureter (BHU 88, Assam 96) a) Ca Breast b) Ca stomach
b) Urethra c) Ca pancreas d) Ca colon
c) Common bile duct 2855. Gastric tetany is due to- (PGI 93)
d) Spermatic cord a) Increased intestinal acidity
2842. Bee venon can be neutralised by applying- (PGI 88) b) Vagal Hyperactivity
a) Soda bicarbonate b) Vmegar c) Increased Calcium Sequestration
c) Lemonjuice d)DiluteHCL d) Decreased Calcium absorption
2843. The most dangerous injury is- (PGI 88) 2856. Subcutaneous Calcifications are seen in -(JIPMER
a) Snake bite b) Scorpion bite a) Gout b) Hyperparathyroidism 93)
c) Wasp sting d) Human bites c) Oochronosis d) Malignancies
2844. The best site for intramuscular injection is- 2857. Raji cell assays are used to quantitate- (PGI 39,
a) Deltoid (PGI 88) a) Complement levels AIIMS 81)
b) Anterolateral part of thigh b) Immune complexes
c) Upper outer segment of buttocks c) T-cells
d) Upper inner segment of buttocks d) Interferon levels
2845. Hereditary spehrocytosis is transmitted 2858. Courvoisier's law is related to-
as- (PGI 85) a) Jaundice (AIIMS 79, AMC 81, 84)
a) Autosomal dominant b)Autosomal recessive b) Ureteric calculi
c) X-linked dominant d) X-linked recessive c) Portal hypertension
2846. A patients compalints of occasional vomiting of d) The length of skin flap in skin grafting
food particles eaten a few days ago. His wife 2859. In polycythaemia vera the most common
reports that his breath smells foul. The most likely postoperative complication following major
diagnosis is- (Kerala 89) surgery is - (PGI 91, AMUR 87)
a) Pyloric construction b) Carcinoma stomach a) Thrombosis b) Gastric ulcer
c) Carcinoma esophagus d) Achalasia cardia c) Diabetes insipidus d) Haemorrhage

2834)d 2835)d 2836)a 2837) c 2838)a 2839)b 2840)a 284l)c 2842)a 2843)d 2844)c 2845)a 2846)d 2847)b
2848) d 2849) b 2850) d 2851) a 2852) c 2853) c 2854) c 2855) d 2856) b,d 2857) b 2858) a 2859) d
SURGERY [ 316]

2860. Witzelsucht syndrome (i.e. "Pathological Joking") 2871. Secondary amyloidosis occurs in- (JIPMER 80,
is seen in- (JIPMER 88, Al90) a) Chronic osteomyelitis AIIMS 82)
a) Frontal lobe tumours b) Rheumatoid arthritis
b) Parietal lobe tumours c) Leprosy
c) Temporal lobe tumours d) Syphilis
d) IV ventriculars tumours 2872. Presence of trifluroacetic acid (TFA) in urine
2861. Biot's respiration is seen in- (JIPMER 88, AI 90) indicates that volatile anaesthetic agent used was
a) Hypnosedative poisoning a) Halothane (JIPMER 81, AIIMS 87)
b) Appendicitis b) Methoxyflurance
c) Cholecystitis c) Trichloroethylene
d) None of the above
d) Bulbarpoliomyelitis
2873. Arrow headed finger on X-ray is suggestive of-
2862. 'Sterile needle test' helps in differentiating-
a) Acromegaly (PGI 83, 85)
a) Healing proces (JIPMER 81, AIIMS 86)
b) Hyperparathyroidism
b) Depth of burns c) Down's syndrome
c) Degenerative proces d) Sarcoidosis
d) Infection 287 4. Preoperative shaving is idealy done at-
2863. Van Bucbem's syndrome is characterized by all a) Evening before (JIPMER 81, Delhi 86)
except- (PGI 80, Rohtak 86) b) Morning of operation
a) Overgrowth b) Distrotion of mandible c) Just before operation
c) Facial Palsy d) Increased acid phosphatase d) At operation table
2864. The stage of myasthenia gravis that best responds 2875. The most important in assessing Fertility
to thymectomy is- (AIIMS 80,81, PGI 86) potential is- (PGI 80, UPSC 89)
a) Stage I : active a) Sperm count
b) Stage 2 : inactive b) Spermmotility
c) Stage 3 : burnt out c) Spermmorphology
d) Those with thymoma present d) Quantity of ejaculated semen
e) None of the above e) None of the above
2865. Quant's sign (aT-shaped depression in the occipital 2876. A seldinger neeedle is used for- (AIIMS81,
bone)maybepresentin- (JIPMER81, AIIMS84) a) Liver biopsy b) Suturing skin AMC87)
a) Down's syndrome b) Head injury c) Arteriography d) Lymphography
c) FUcke~ d) Scurvy 2877. Newman and Seabrook's operation is used for-
2866. Nezelof's syndrome is recurrent episodes a) Repair of parotid fistula (PGI 79, AIIMS 84)
b) For parotid calculi
of- (JIPMER81,DNB90)
c) Forcarcinomaoftongue
a) Appendicitis b) Cholecystitis
d) For treatment of recurrent chronic parotitis
c) Intestinal obstruction d) Pneumonia
2878. The commando operation is- (JIPMER 80,81,
2867. Oxygen consumption during anesthesia and
a) Abdomino-perineal resection of (UPSC 85)
surgery with mid levels of hypothermia in infants the rectum for carcinoma
less than 12 kg is (35-360C) - (AIIMS 89, DNB 90) b) Disarticulation of the hip for gas gangrene of the
a) Unchanged b) Doubles leg
c) Tripled d) Quadrupled c) Extended radical mastectomy
e) None of the above d) Excision of carcinoma of the tongue, the floor
2868. Hickey-Hare test is used to diagnose-: (PGI 81, of the mouth, part of the jaw and lymphnodes
a) Congenital pyloric stenosis AJIMS 86) enbloc
b) Duodenal atresia 2879. All of the following are perils of prolonged
c) Achiasia cardia antibiotic therapy in intra abdominal sepsis except
d) Diabetes insipidus a) Masking of general signs (PGI 81, AIIMS 86)
2869. Usually employed tecbynique for splanchnic b) Subacute intestinal obstruction
block is- (JIPMER 79,AIIMS80) c) Malignant change
a) Braun's method b) Kappi's method d) Frozen pelviss
c) Wending's method d) None of the above 2880. Emergency thoractomy is only indicated in-
2870. Spontaneous regression of malignant tumour is a) Empyema (JIPMER 81, AMU 86)
seen in- (JIPMER 80, AIIMS 81) b) Pneumothorax
a) Burki~ lymphoma b) Neuroblasoma c) Uncontrolled bleeding
c) Wilm's tumour d) Renal cell carcinoma d) None of the above

2860) a 2861) None 2862) b 2863) d 2864) a 2865) c 2866) d 2867) b 2868) d 2869) a 2870) b,d 2871) a,b,c
2872)a 2873)a 2874)c 2875)b 2876)c 2877)a 2878)d 2879)c 2880)c
SURGERY [ 317]

2881. Aaron's sign is seen in- (AIIMS 81, 86) 2892. Ramsteds operation is for- (Kerala 94)
a) Achlasia cardia a) Duodenal atresia
b) Hiatus hernia b) Volvulous of small intestine
c) Mediastinum emphysema c) Anorectal malformation
d) Acute appendicitis d) Congenital pyloric stenosis
2882. Duhamel operation is done in- (JIPMER81, 2893. Corpora amylaciae is seen in- (Kerala 94)
a) Congenital pyloric stenosis UPSC89) a) Thymus b) Lymph node
b) Hiatus hernia c) Spleen d) Prostate
c) Achlasia cardia 2894; Antibioma is best treated by- (JIPMER 95)
d) Hirschsprung's disease a) Partial resection
b) Complete resection
2883. Bolognini's symptom (a feeling of crepitation
c) Aspiration
occuring from gradual increasing pressure on the
d) Administration of antibiotics
abdomen) is seen in- (PGI 80,81, AIIMS 84)
2895. Antibiotics should be given how long with surgery
a) Congenital pyloric stenosis forprophytaxis- (JIPMER 95)
b) Gastric polyp a) 1 hour before b) 1 hour after
c) Duodenal atresia c) 2 hours before d) 2 hours after
d) Measles 2896. During endotracheal intubation, unilateral breath
2884. First meconium is said to be formed during the -- sounds, no air heard entering the stomach and no
-monthoffoetallife- (PGI 81, AIIMS 84) gastric distension is suggestive of entry of the
a) Second b) Fourth endotracheal tube into- (UPSC 96)
c) Seventh d) Ninth a) Right main bronchus b) Oesophagus
2885. Mauriac's syndrome is characterized by the c) Mid-trachea d) Left main bronchus
following except- (JIPMER 81, AIIMS 86) 2897. Sappey's line denotes a line- (Karn. 95)
a) Diabetes b) Obesity a) Encircling the neck at C6 vertebra level
c) Dwarfism d) Cardiomegaly b) Encircling the trunk just above the umbilicus
2886. Kocher'smanuevermeans- (JIPMER 81) c) Encircling the salphigian tubes
a) Mobilisation ofgall bladder during cholecystectomy d) None of the above
b) Mobilisation of 2nd part of duodenum 2898. Hormonal treatment is given for which of the
c) Mobilisation of pancreas during pancreatectomy following malignancy- (Kerala 96)
d) Mobilisation of ascending colon a) Chorio Carcinoma b) Carcinoma prostate
2887. Pyrexia due to wound infection commonly occurs c) Hepatoma d) Teratoma
after- (PGI 81, AMU 87) e) Granulosa cell tumour
2899. Rovsing sign is seen in- (PGI 95)
a) Third post operation day
a) Acute appendicitis b) Acute cholecystitis
b) Fifth post operation day
c) Pancreatitis d) None
c) Seventh post operation day
2900. A female patients complains of perumbilical pain
d) Second post operation day
& nausea particularly after taking food. The
2888. Catgut is prepared from submucosal layer of the diagnosis is- (CUPGEE 95)
intestine of- (AIIMS 81, DNB 89) a) Meckel's diverticulum
a) Cat b) Sheep b) Peptic ulcer syndrome
c) Human being d) Rabbit c) Lactose intolerance
2889. A cricoid hook is used particularly- (JIPMER 81, d) None
a) In thyrodectomy DNB 89) 2901. In a Kocher's subcostal incision which of the
b) In block dissection ofthe neck following should not be divided- (Karanatak 96, 94)
c) For retractting the superior laryngeal nerve a) Rectus abdominis muscle b) 8th dorsal nerve
d) In tracheostomy c) Oblique muscles d) 9th dorsal nerve
2890. Notapremalignantulcer- (Kerala 94) 2902. Hilton's method of treatment of an axillary
a) Bazin 's ulcer b) Pagets disease of nipple abscess is advised because it- (Karnatak 96, 94)
c) Marjolins ulcer d) Lupur vulgaris a) Protects vital structures
2891. Cullen's sign in seen in- (Kerala 94) b) Ensures adquate drainage
a) Acute cholecystitis c) Hinders the spread of infection
b) Acute pancreatitis d) Allows local instillation of antibiotics
c) acute haemorrhagic pancreatitis 2903. Bisgard treatment is for- (AP 96, SCTIMS 98)
d) Blunt injury abdomen a) Arterial ulcer b) Venous ulcer
c) TAO d) Raynauds phenomenon

2881)d 2882)d 2883)d 2884)a 2885)d 2886)b 2887)b 2888)b 2889)d 2890)a 2891)c 2892)d 2893)d 2894)b
2895)a 2896)a 2897)a 2898)b 2899)a 2900)a 2901)b,d 2902)a 2903)b
SURGERY [ 318]

2904. Which tumour is treated by hormones -(Kerala 96) 2915. A 40 year old male was brought to the hospital with
a) Carcinoma prostrate b) Carcinoma bladder acute pain in the upper abdomen. Pateints was in
c) Carcinoma cervix d) Carcinoma pancreas shock with feeble pulse andtachycardia. There was
e) Carcinoma stomach tenderness present in the epigastrium. There is no
2905. A patient is admitted with sever pain in the blood in the gastric aspirate and the patient felt better
abdomen, nausea, vomiting and fever. The most after aspiration. X-ray abdomen showed no free gas
likely diagnosis is - (UPSC 97) under the diaphragm. Investigations revealed TLC
a) Perforated peptic ulcer b) Intestinal obstruction 13500 serum bilirubin 2.0 mg and serum amylase
c) Acute pancreatitis d) Acute cholecystitis 800 I.U•• The most likely diagnosis is- (UPSC 88)
2906. A 12 month old male child suddenly draws up his a) Acute cholecystitis b) Acute pancreatitis
legs and screams with pain. This is repeated c) Acute appendicitis d) Acute hepatitis
periodically throughout the night interspeersed 2916. A 24 years old male, who bas been having fever for
with periods of quiet sleep. When seen after 12 15 days starts having acute pain and distension of
hours the child looks pale, has just vomited and abdomen. Abdominal examination reveals
passed thin blood-stained stool; there is a mass generalised tenderness with guarding. The most
around umbilicus. What is the most likely diagnosis likely diagnosis is- (UPSC 88)
a) Appendicitis (UPSC 97) a) Acute appendicitis
b) Intussusception b) Acute pancreatitis
c) Gastro-enteritis c) Enteric perforation
d) Roundworm obstruction d) Duodenal ulcer perforation
2907. Commonest tumour among children 1-5 years in 2917. loguinallymphoode enlargement is seen in-(MP 97)
SouthAfricais- (TN 96) a) Seminoma testis b) Malignant melanoma foot
a) Neuroblastoma b) Wilm's tumour c) Cacervix d) None
c) Neurofibroma d) Burkitts lymphoma 2918. Panceatico duodenectomy is the treatment of
2908. In a patient presenting with abdominal trauma, choice for- (TN 99)
fracture Rib and bruise over left hypochondrium a) Duodenal carcinoma b) Pancreatic carcinoma
probable diagnosis is- (PGI 96) c) Gall bladder carcinoma d) Gastric carcinoma
a) Rupture left lobe of liver 2919. A 10 year old female who used to use the swimming
b) Rupture right lobe ofliver pool regularly, comes with a three day history of
c) Splenic rupture vomittiog, fever and adbominal pain. On
d) Rupture stomach examination, abdominal tenderness and guarding
2909. Earliest method of diagnosing pituitary tumour are present. The liver dullness is not obliterated.
a) CT scan (JIPMER 98) Likely diagnosis is- (AIIMS 99)
b) Visual field charting a) Gangrenous intussusception
c) Visual evoked potential b) Perforation
d) X-ray skull c) Spontaneous biliary peritonitis
2910. Most common sarcoma in a child is- (JIPMER 98) d) Primary peritonitis
a) Fibrosarcoma b) Rhabdomyosarcoma 2920. Use of all the following significantly decreases
c) Leiomyosarcoma d) Liposarcoma airborne infection is operating room except-
2911. BCGisusedin tumour therapy- (JIPMER 98) a) Laminar airflow (UPSC 2K)
a) Bladder b) Stomach b) Air-conditioning
c) Oesophagus d) Colon c) mtravioletlight
2912. Regarding CEA-false is- (AIIMS 97) d) Microfilters
a) Prognostic indicator 2921. Vicryl, the commonly used suture material is a-
b) Glycoprotein a) Homopolymer of polydioxanone (UPSC 2K)
c) Elevated in colorectal carcinoma b) Co-polymer of glycolide and lectide
d) Elevated only when there is hepatic metastasis c) Homopolymerofglycolide
2913. In head injury with lesion to the pituitary stalk, d) Homopolymer oflactide
all are seen- (AIIMS 97) 2922. All the following suture materials are non
a) SIADH b) Diabetes inspidus absorbable except- (Kerala 2K)
c) Diabetes mellitus d) Prolactinoma a) Poly amide b) Poly propylene
2914. Randall's plagues causes- (TN 98) c) Poly diaxonone d) Polyethylene
a) Bile stones b) Urinary stones e) Polyester
c) Premalignant lesions d) Bacterial infections

2904)a 2905)c 2906)b 2907) d 2908)c 2909)a 2910)b 2911)a 2912)d 2913)a,b2914)b 2915)b 2916)c 2917)b
2918)a,b 2919)d 2920)b 2921)b 2922)c
SURGERY [ 319]

2923. 45 years executive suddenly develops bematemesis 2935. Nottrue about uretbralinjuries is- (TN 200J)
at home. He is brougbtto thehospital 4 hrs. later, a) Catheterize the pt. immediately
there be again bad about ofhematemesis. Total blood b) Can be associated with fracture pelvis
loss would be around 2litres. Most likely diagnosis c) Bladder injury is associated with post urethral
is- (AJIMS200J) injuries
a) Gastritis b) Oesophagitis d) Blood at the external urethral meatus is an Imp
c) Oesophageal varices d) Duodenal ulcer feature
2924. Psychiatric symptoms, true except- (PGI 2K) 2936. Which of these is not a risk factor for
a) More common with supra than infra tentorial thromboembolism- (TN200J)
tumours a) Myocardial infarction
b) More common with slow growing b) Hypertension
c) More with temporal than frontal lobe tumours c) Estrogen therapy
d) More with brain stem lesions d) Superficial thrmbophlebitis
2925. PDSisabsorbedwithin- (MAHE 200J) 2937. Not true about 'Struvite Stones' is- (SCTIMSOJ)
a) 7 days b) 21 days a) better k/a staghorn calculus
c) 100 days d) 225 days b) These are triple phosphate stones
2926. Bird's beak is seen in- (J & K 200J) c) Common in infected urine
a) Volvulus b) Intussusception d) Usually seen in acidic urine
c) Achalasia d) Ulcerative colitis 2938. Treatment for parathyroid hyperplasia is-
2927. Sentinel node biopsy is done in- (Al2002) a) Removal of all four glands (UPSC 0 J)
a) Carcinoma Lung b) Carcinoma breast b) Calcitonin
c) Liposarcoma d) Carcinoma colon c) Removal of3 112 glands
2928. Local anaeshthetics cannot be used at the site of d) Enlarged glands to be removed
infections because it causes- (MANIPA.L OJ) 2939. Treatment of choice forwarthins tumour is-
a) Spread of infection b) Lowered efficiency a) Superficial parotidectomy (UPSC OJ)
c) Both d)None b) Enucleation
2929. An AIIDS patient presents with fistula - in- c) Radiotherapy
ano.His CD4 count is below 50. What is the d) Injection of a sclerosant agent
treatment of choice- (MAN/PAL OJ) 2940. Treatment of choice for solitary thyroid nodule
a) Seton b) Fistulectomy is- (TN OJ)
c) None d) Both a) Removal of the nodule
2930. Commonest tumour at inner canthus of b) Hemithyroidectomy
eye- (HPU 200J) c) Subtotal thyroidectomy
a) Rodent ulcer b) Dermoid cyst d) Total thyroidectomy
c) Malignant melanoma d) Buruli ulcer 2941. Most common site oflentigo maligna melanoma is-
2931. Steinstrasse is- (UPSC 200J) a) Face b)Legs (PGIOJ)
a) Staining of stones c) Trunks d) Soles
b) Stones because of stress 2942. Most common origin of melanoma is from-
c) Failure ofESWL a) Junctional melanocytes (AMU OJ)
d) Ureteric obstruction due to fragments is ureter b) Epidermal cells
2932. Stereotactic radiosurgery is done for- (Jipmer 02) c) Basal cells
a) Glioblastomamultiforme d) Follicular cells
b) Medulloblastoma spinal cord 2943. Congenital hydrocele is best t/t by- (PGI 0J)
c) Ependymoma a) Eversion of sac b) Excision of sac
d) AV malformation ofbrain c) Lords procedure d) Herniotomy
2933. Not true about bladder stones is- (Jipmer 02) 2944. For carcinoma ofthe prostate the commonest site
a) Rare in Indian children is- (AIIMS OJ)
b) Primary stones are rare a) Anterior zone b) Peripheral zone
c) Small stones can be removed per urethra c) Central zone d) Transitional zone
d) Mixed stones are radioopaque 2945. Splenectomy is not done in- (AIIMS OJ)
2934. Which malignancy would occur to prolonged a) Mylofibrosis
multimodulargoitre- (AMU 200J) b) Sickle cell anemia
a) PapillaryCa b) FollicularCa c) Hereditary spherocytosis
c) Anaplastic Ca d) MedullaryCa d) Splenic abscess

2923)d 2924)d 2925)d 2926)a,c2927)b 2928)c 2929)a 2930)a 293l)d 2932)d 2933)a 2934)b 2935)a 2936)d
2937)d 2938)c 2939)a 2940)b 294l)a 2942)b 2943)d 2944)b 2945)b
SURGERY [ 320]

2946. All are true about long flexor tendons except 2958. True about berry-aneurysm is following except-
a) Flexor digitorum profundus (KERALA 95) a) Associated with familial syndrome (PGI 2000)
inserted to distal phalanx base b) Most common site of rupture is apex which causes
b) Flexor digitorum superficialis attached to the SAH
sides of middle phalanx c) Wall contains smooth muscle fibroblasts
c) Damag to the tendons involves formation of d) 90% occurs at ant. part of circulation at branching
tenoma during repair point
d) Good repair results if tendon sheath is 2959. Immediate surgery is required for- (PGIOJ)
damaged a) Paralytic ileus due to pelvic collection
2947. No man's land in palm corresponds to- (MAllE 98) b) Volvulus of intestine
a) Zone I b) Zone II c) Post-operative adhesions
c) Zone III d) Zone IV d) Intestinal obstruction
2948. Failure of migration of neural crest cells is seen e) Paralytic ileus with appendicitis
in- (KERALA 2001) 2960. AFP is raised in- (PG103)
a) Albinism b) Congenital megacolon a) Prostate Ca b) Hepatic Ca
c) Odontomes d) Adrenal tumour c) LungCa d) BreastCa
2949. Hypothermia is used in all except- (PGI 98) e) ColonCa
a) Cardiac surgery b) Neonatal ischemia 2961. Absorbable sutures are- (PG104)
c) Heat stroke d) Cardiac arrhythmia a) Catgut b) Silk
2950. Millards 'Rule ofTen' includes all except- c) Polyprophylene d) Polyglycolic acid
a) lOkg b) lOweeks (Amu95) e) Vicryl
c) 10 gm% Hemoglobin d) 10 months 2962. True about intra abdominal compartment syndrome-
2951. The commonest symptom post operatively seen is- a) Intraabdominalpressure> 15cmHp (PG/04)
a) Depression (Kerala 97) b) Pneumoperitoneum can produce it.
b) Psychosis c) i Renal blood flow
c) Euphoria d) .! venous return
d) None of the above e) i ce~tral blood volume
2952. The most sensitive qualitative method for detection 2963. Increased intra abdominal pressure is/are
of air embolism is - (PG/79, AIIMS 86) associated with- (PGI 04)
a) Doppler ultra sound technique a) i pul capillary wedge pressure
b) Electro cardiogram b) i venous return
c) Arterial pressure c) i pul inspiratory pressure
d) End expiratory carbon dioxide content d) i renal blood flow
e) Rest oflife e) i cardiac output
2953. Commonestarteryforcannulationis- (PG/97) 2964. What will be your advice to a 60yr moderately obese
a) Radial b)Ulnar patient with h/0 4-6 cup of coffee per day, 4-6 glass
c) Brachial d) Cubital of wine/day with 20 cigarettes per day. He is engaged
2954. Duplication ofGIT most commonly involves- as a salesman in a computer company. His brother
a) Esophagus b) Stomach (PG/97) died of Pancreatic carcinoma- (PGI 04)
c) Duodenum d)Ileum a) Urgent wt. Reduction b) Strict vegetarian diet
2955. Fiberoptic endoscopy is contraindicated in -(PG/98) c) Stop alcohol d) Stop coffee
a) Children e) Stop cigarette smoking
b) Aneurysm of arch of aorta 2965. Which is the baseline investigation in the case of
c) Cervical spondylosis an acute abdomen in this high -tech era-(Kerala 03)
d) Hemoptysis a) Abdomen CT b) Abdomen X- Ray
2956. Fluoroscopy is used for diagnosis of- (PG/98) c) USG d) Colonoscopy
a) LV function b) Diaphragmatic palsy 2966. Which one of the following statements about
c) Valvular clacification d) Pericardia! effusion xeroradiography is correct- (Karnataka 02)
2957. Which is not a feature ofDupuytern's contracture - a) Is a wet technique
a) Recurrenttrauma (PG/99) b) UsesX-rayfilm
b) Genetic predilection c) Uses charged selenium plate
c) Bilateral lesions rare d) Requires more radiation exposure
d) Seen equally in males and females

2946)d 2947)b 2948)b 2949)b,d 2950)d 2951)d 2952)d 2953)a 2954)d 2955)b 2956)b,c 2957)d 2958)c
2959)b,d 2960)b,c 296l)a,d,e 2962)a,b,d 2963)a,c 2964)a,b,e 2965)b 2966)d
SURGERY [ 321 ]

2967. A chest physician performs bronchoscopy in the 2976. Tl and T2 images are a feature of which diagnostic
procedure room of the out patient department. To test- (Karnataka 04)
make the instrument safe for nse in the next patient a) Ultrasound b) C.T. scan
waiting outside, the most appropriate method to c) Duplex scan d) MRI scan
disinfectthe endoscope is by- (AI 03) 2977. Who said: "Skin is the best dressing"? (Karn 04)
a) 70% alcohol for 5 min a) Joseph Lister b) John Hunter
b) 2% gluteraldehyde for 20 min c) James Paget d) McNeill Love
2978. In the acronym "SWELLING" used for the history
c) 2% formaldehyde for 10 min
and examination of a lump or swelling, the letter 'N'
d) 1% sodium hypochlorite for 15 min
standsfor- (Karnataka 04)
2968. Bedsore is an example of- (AI 03) a) Nodes b) Noise (Thrill/bruit)
a) Tropical ulcer b) Trophic ulcer c) Numbness d) Neurological effects
c) Venous ulcer d) Post thrombotic ulcer 2979. Disparity of the bowel ends during and to end
2969. Depressed bridge of nose can be due to any of the anastamosis is corrected by- (Karnataka 04)
following except- (Karnataka 03) a) Cheatle's manoeuvre b) Connell suture
a) Leprosy b) Syphilis c) Lambert suture d) Czemytechnique
c) Thalassemia d)Acromegaly 2980. Which of the following is a delayed absorbable
2970. A 60 year old lady underwent abdominal surgery synthetic suture material- (TN 04)
aud on the 41h post- operative day she was diagnosed a) Chromic catgut b) Vicryl
to have Systemic Inflammatory Response Syndrome c) Silk d) Nylon
(SIRS). What are the features of SIRS ?(UPSC 04) 2981. Lamina dura lining the alveolus is -(Karnataka 02)
a) Normal body temperature and normal respiratory a) Cancellous bone b) Ligament
rate c) Dense cortical bone d) Muscle
b) WBC> 12x 109/Lor<4x 109/L 2982. Low and fixed specific gravity of urine is seen in-
c) Respiratory rate > 24 breaths/minute and heart a) Diabetes mellitus (SGPGI 05)
b) Diabetes insipidus
rate > 90 beats/minute
c) Chronic renal failure
d) Respiratory rate < 10 breaths/minute d) Acute glomerulonephritis
2971. Which ofthe following preservatives is used while 2983. Functional GI disorders can be differentiated from
packing catgut suture? (AI 04) organicGidisordersby- (SGPG/05)
a) Isopropyl alcohol b) Colloidal iodine a) Abdominal pain b) Diarrhoea
c) Gluteraldehyde d) Hydrogen peroxide c) Tenesmus d)BleedingPR
2972. In sickle cell anemia sudden onset of Pancytopenia 2984. In triage green colour indicates- (COMEDK 05)
with hemolysis and no rise of reticulocyte count a) Ambulatory patients
occurs in - (Jipmer 04) b) Dead or moribund patients
a) Sequestration crisis b) Aplastic crisis c) High priority treatment of transfer
c) Hemolytic crisis d) Vasooculusive crisis d) Medium priority or transfer
2973. Suicidal enzyme is- (Jipmer 04) 2985. Lord's aud Jaboulay's operation is done for- (AMU
a) Cyc1oxygenase b) Lipooxygense a) Rectal prolapse b) Fistula in ano 05)
c) Dehydrogenase d) Pyruvate kinase c) Inguinal hernia d) Hydrocele
2974. A 48 year-old woman underwent subtotal 2986. Surgically nsed suture material polydioxanone
thyroidectomy. She has a vague family history of (PDS)- (COMEDK05)
a) Is non absorbable and remains encapsulated
malignant hyperthermia. She develops agitation,
b) Undergoes hydrolysis and complete absorption
restlessness, fever, tremor, shivering, and c) Undergoes phagocytosis and enzymatic
tachypnea. Thyrotoxic crises can be best degradation
distinguished from malignant hyperthermia by d) Is specifically used for heart valves or synthetic
estimating- (Kerala 04) grafts
a) Temperature variation b) Increased CPK levels 2987. Which one of the following is a muscle-splitting
c) IDH d) Mascularrigidity incision - (ICS 05)
2975. Who said these words: To study the phenomenon of a) Kocher's b) Rutherford-Morris
disease without books is to sail an uncharted sea, c) Pfannenstiel d) Lanz
while to study books without patients is not to go to 2988. Aims of abbreviated laparotomy- (PGI June 05)
sea at all- (Karnataka 04) a) Decreased chance of infection
a) Hamilton Bailey b) Sir Robert Hutchison b) Early ambulation
c) SirWilliamOs1er d) J.B. Murphy c) Early wound healing
d) Haemostasis

2967)b 2968)b 2969)d 2970)b,c 2971)a 2972)a 2973)a 2974)b 2975)c 2976)d 2977)a 2978)b 2979)a 2980)b
298l)c 2982)c 2983)d 2984)a 2985)d 2986)b 2987)d 2988)d
SURGERY [ 322]

2989. Orthobaric oxygen is used in- (MAHA 05) arterial wall. The inflammation also extended into
a) Carbon monoxide poisoning the neighbouring veins and nerves. The most
b) Ventilation failure probably diagnosis is- (AIIMS 06)
c) Anaerobic infection a) Takayasu arteritis
d) Gangrene b) Giant cell arteritis
2990. Which of the following catheter materials is most c) Hypersensitivity angiitis
suited for long-term use is- ~ (AI 05) d) Thromboangiitis obliterans
a) Latex b) Silicone 3000. RET proto oncogene mutation is a ballmarkofwhich
c) Rubber d) Polyurethane of the following tumors? (AIIMS 06)
2991. Referred pain from all of the following conditions a) Medullary carcinoma thy:t:Oid
may be felt along the inner side of right thigh, b) Astrocytoma
except- (AI 06) c) Hurthle cell tumor thyroid
a) Inflamed pelvic appendix b) Inflamed ovaries 3001. A 65-year-old miner has lost 7 kgs weight within
c) Stone in pelvic ureter d) Pelvic abscess two months, has presented with cough, and bloody
2992. On the 4th postoperative day oflaparotomy a patient streaked sputum. He was treated for pulmonary
presents with bleeding & oozing from the wound tuberculosis 10 years ago. He also has drooping of
Management is- (PGI June 06) his left eyelid for one month. On physical
a) Dressing of wound & observe for dehiscence examination, there is ptosis of the left eye and
b) IV fluids papillary miosis. Chest X-ray revealed round
c) Send for USG abdomen opacification in the left upper apical lobe. What is
d) Start treatments for peritonitis the most probable diagnosis? , (AIIMS 06)
e) Urgent surgery a) Secondary tyberculosis b)Adencarcinoma
2993. Pelvic exentration is known as- (APPG 06) c) Squamous cell carcinoma d) Asbestosis
a) Miles Operation b) Lyods operation 3002. In which of the following conditions acquired
c) Finch operation d) Brunshwing" operation (secondfary) megacolon is seen? (AIIMS 06)
a) Fissure in-ano
2994. Jack stone calculi is been in which anatomic part-
b) Complete absence ofparasympathetic ganglion cells
a) Prostate b) Kidney (Manipal 06)
c) Absence of sympathetic ganglion cells
c) Ureter d) Bladder
d) Rectal malignancy
2995. In abdominal surgery lembert sutures refers to-
3003. One of the following is not correct in papillary
a) Single layer suturing (Karn 2006)
carcinoma of thyroid- (AIIMS 06)
b) Sero muscular sutures
a) Can be reliably diagnosed using fine needle
c) All coat intestinal suturing aspiration cytology
d) Skin suturing b) Always unifocal
2996. Barrets esophagus is commonly associated with one c) Typically spreads to the cervical lymph nodes
of the following- (AIIMS 06) d) Requires a total thyroidectomy for large tumours
a) Adenocarcinoma 3004. Which one of the following·statements is incorrect
b) Squamous cell carcinoma in regard to stones in the gallbladder? (AIIMS 06)
c) Sarcoma a) Pigment stones are due to increased excretion of
d) Gastrointestinal stromal tumor conjugated bilirubin
2997. Which of the following brain tumours is highly b) Are considered a risk factor for the development
vascular in nature? (AIIMS 06) of gallbladder carcinoma
a) Glioblastoma b) Meningiomas c) 10% of gallstones are radio-opaque
c) CP angle epidermoid d) Pituitory adenomas d) A mucocele ofthe gallbladder is caused by a stone
2998. A 40-year old female patient presented with impacted in Hartmann's pouch
dysphagia to both liquids and solids and 3005. A 45-year-olf gentleman has undergone truncal
regurgitation for 3 months. The dysphagia was non- vagotomy and pyloroplasty for bleeding duodenal
progressive. What is the most likely diagnosis? ulcer seven years ago. Now he was intractable
a) Carcinoma of the esophagus (AIIMS 06) recurrent symptoms of peptic ulcer. All of the
b) Lower esophageal mucosal ring following suggest the diagnosis of Zolliner Ellison
c) Achalasia cardia syndrome, except- (AIIMS 06)
d) Reflux esophagitis with esophageal stricture a) Basal acid output of 15 meqlhour
2999. A 45-year-old male having a long history of cigarette b) Serum gastrin value of 500 pglml
smoking presented with gangrene ofleft foot. An c) Ulcers in proximal jejunum and lower end of
amputation of the left foot was done. Representative esophagus
sections from the specimen revealed presence of d) Serum gastrin value of 200 pglml with secretin
arterial thrombus with neutrophilic infiltrate in the stimulatiopn

2989)a 2990)b 2991)d 2992)b,e 2993)d 2994)d 2995)b 2996)a 2997)a 2998)c 2999)d 3000)a 3001)c 3002)d
3003) b 3004) d 3005) d
SURGERY [ 323]

3006. The defective migration of neural crest ceUs results C Basal cell carcinoma lymphatics
m- (AIIMSO~ D. Malignant melanoma 3. Prognosis depends
a) Congenital megacolon on thickness
b) Albinism 4. Highly radiosensitive
c) Adrenogenital hypoplasia Code:
d) Dentinogenesis imperfecta A B C D
3007. Which of the foUowing statements is true of primary a) 4 1 2 3
grade IV-V vesicoureteric reftux m young children?
b) 4 2 1 3
a) Renal scarring usually begins (AIIMS 0~
c) 3 I 2 4
in the midpolar regions
b) Postnatal scarring may occur even in the absence d)3 2 l 4
of urinary tract infections 3014. A55-year-oldmalepatientunderwentcbolecystectomy
c) Long-term outcome is comparable in patients for GaD stone calculus. During surgery the patient's
treated with either antibiotic prophylaxis or surgery pulse was irregulary irregular, 160/mm, BP =80/
d) Oral amoxicillin is the choice antibiotic for 50 mm ofHg, temp. 400c On examination a swelling
prophylaxis in the neck was found. Most likely diagnosis is-
3008. Which of the foUowing is the best way of preventing a) Tbyioid storm b) Myocardial infarction
development deep vein thrombosis (DVT) in c) Arrthmias d) Stridor (MAHE 07)
postoperative period? (AIIMS 0~ 3015. Cushing reflex is- (UP 07)
a) Early ambulation b) Physiotherapy a) i Mean arterial pressure with increase intra cranial
c) Prophylactic heparin d) Low dose aspirin pressure
3009. A 48-year-old sports photographer bas noticed a b) i Mean arterial pressure with decrease intracranial
smaU nodule over the upper lip from four months. pressure
The nodule is pearly white with central necrosis, c) i Mean arterial pressure with increase intracranila
telangiectasia. The most likely diagnosis would be-
pressure
a) Basal cell carcinoma (AIIMS 0~
d) J.. Mean arterial pressure with decrease intra cranial
b) Squamous cell carcinoma
c) Atypical melanoma pressure
d) Kaposis sarcoma 3016. Gases for Pneumoperitoneum- (PGI June 07)
3010. Which one of the foUowing statements is mcorrect a)C02 b)N2
regarding Meckel's diverticulum? (AIIMS 0~ c) 0 2 d)Roomair
a) Is found on the anti-mesenteric border of the small e)Np
intestine 3017. Scrambled egg appearance is seen in- (Corned 07)
b) Consists of mucosa without a muscle coat a) Carcinoma stomach · b) Carcinoma galll bladder
c) Heterotopic gastric mucosa can ulcerate and cause c) Pancreatic carcinoma d) Renal carcinoma
a brisk gastrointestinal bleed 3018. Shoulder pain post laparoscopy is due to-
d) A fibrous band between the apex and umbilicus a) Subphrenic abscess (AIIMS Nov 07)
can cause intestinal obstruction b) col retention
3011. A 35-year-old premenopausal patients as recently c) Positioning of the patient
developed a 1.5 em sized pigmented lesion on her d) Compression of the lung
back. Which of the following forms of tissue 3019. FNAC needle size- (AIIMS Nov 07)
diagnosis will you recommend for her? (AIIMS 0~
a) 18-22 b)22-26
a) Needle biopsy b) Trucut biopsy
c) Tl-29 d) 16-18
c) Excision biopsy d) Incisional biopsy
3012. Extensive surgical debridement, decompression or 3020. Triangle ofDoom is bounded by aU oftbe foUowing
amputation may be indicated in the foUowing clinical except- (AIIMS Nov 08)
setting except- (UPSC 07) a) Cooper's ligament b) Vas deferens
a) Progressive synergistic gangrene c) Gonadal vessels d) Peritoneal reflection
b) Acute thrombophlabitis 3021. Nicoldonisignisalsoknownas- (AIIMSNov08)
c) Acute haemolytic streptococcal cellulitis a) Murray sign b) Frei sign
d) Acute rhabdomyolysis c) Darrier sign d) Branham sign
3013. Match listl with list II and select the correct answer 3022. Allopurinol is used morgan preservation as-
using the code given below the lists- (UPSC 07) a) Antioxidant (AIIMSMay 09)
List I ListiT b) Preservative
(Carcinoma) (Characteristic) c) Free radical scavenger
A. Seminoma testis l. Hormone dependent d) Precursor for energy metabolism
B. Carcinoma prostate 2. Does not spread by

3006)a,b 3007)b 3008)c 3009)a 3010)b 3011)c 3012)b 3013)a 3014)a 3015)a 3016)a,c,e 3017)c 3018)b
3019)b 3020)a 302l)d 3022)c
SURGERY [ 324]

3023. True about aprocine gland- (PGI June 09) 3032. The following statements are correct about burst
a) Modified sweat gland abdomen (abdominal dehiscence) except-
b) Modified sebaceous gland a) Peak incidence in between 61h and gth
c) present in axilla & groin postoperative days (UPSC-I1 09)
d) Hidradenitis suppurativa is infection of apocrine b) Manage with nasogastric aspiration and
gland intravenous fluids
3024. Aim of damage controllaparatomy are- c) Cover the wound with sterile towel and perform
a) Provide fascial closure (PGI June 09) emergency surgery
b) Arrest haemorrhage d) Second dehiscence is very common
c) Control contamination 3033. A patient presents to the emergency department with
d) Prevent infection pain and distension of abdomen and absolute
e) Prevent coagulation constipation. What is the investigation of choice-
3025. In surgical stress hormone is increased except- a) Plain X-ray abdomen (erect) (UPSC-11 09)
a) ADH b)ACTH (PGIJune09) b) Ultrasonography
c) Cortisol d) Insulin c) Barium meal follow-through
e) Renin d) Colonoscopy
3026. True aboutAbdominalcompartment syndrome- 3034. Mackler's triad includes- (PGI Nov 09)
a) J, Cardiac output (PGI June 09) a) Vomiting
b) J. Urine b) Subcutaneos emphysema
c) J. Pulmonary capillary wedge pressure c) Lower thoracic pain
d) J. Venous return d) Peripheral cyanosis
e) J. Systemic vascular resistance e) Pleural effusion
3027. Minimum amount ofGI bleed to cause malena is- 3035. Pain in hypogatric region may arise from-
a) lOml b)40ml (DELHIPGFeb. 09) a) Uterus b) Left colon (PGI May 1 0)
c) 60ml d) 115 ml c) Urinary bladder d) Gallbladder
3028. Which of the following is a delayed absorbable 3036. Day care snrgerycan be done in- (PG1Nov.10)
synthetic suture material? (DELHI PG Feb. 09) a) Lateral sphincterotomy
a) Chronic catgut b) Vicryl b) Rhinoplasty
c) Silk d) Nylon c) Orchidectomy
3029. All of the following patients presenting with d) Total thyroidectomy
e) Sub-cutaneous mastectomy
abdominal pain and shock need immediate
3037. AdvantageofMinimalaccesssurgery-(PG1Nov.10)
laparatomy except- (DELHI PG Mar. 09)
a) i Heat loss b) Better Hemostasis control
a) Ruptured ectopic pregnancy
c) Improved vision d) J. in wound pain
b) Haemorrhagic pancreatitis
e) Improved Mobility
c) Rupture abdominal aortic aneurysm
3038. Fogarty's catheter is used for? (AJIMS Nov 1 0)
d) Ruptured liver hemangioma
a) Urethral catheterization
3030. Which of the following statements is incorrect? b) Removal of blood clots from the arteries
a) Gas in biliary free may be seen following c) Bladder drainage
sphicterotomy d)TPN
b) Column of feces in the left colon is normal 3039. Complication (s) of obesity is /are- (PGI May 10)
c) Intestinal obstruction can be one of the a) Venous ulcer b)Pulmonaryembolism
presentation in Gall Stone disease c) 1' Mortality d) Prostate cancer
d) Gas in periphery of liver suggests portal vein gas
3031. Risk factors for the development of critical limb
ischemia include all except- (DELHI PG Mar. 09)
a) Diabetes
b) Hyeprlipidemia
c) Moderate alchol intake
d) Hyperhomocysteinemia

***

3023)a,c,d 3024)b,c 3025)a,d 3026)a,b,d 3027)c 3028)b 3029)b 3030)b 303l)c 3032)d 3033)a 3034)a,b,c
3035) a,b,c 3036) a,b,c,e 3037) c,d,e 3038) b 3039)All 3040) b
c
FORENSIC

SEC· I & II : LAW & MEDICINE, 14. Who can be considered as an expert witness-
PERSONAL INDENTIFICATION, a) Medical men b) Firearm expert
c) Chemical examiner d) All (TN 89)
MEDICOLEGAL AUTOPSY, EXHUMATION
15. To detemine the age of a 16 year old girl which of
1. Coroner's court is present at- (PG/87, Kerala 87, the following is to be rediographed- (PG/88)
a) Bombay b) Delhi UPSC 85) a) Wrist b) Knee
c) Madras d) Bangalore c) Ankle d) Elbow
e) Chandigarh 16. Conduct money is paid to expert witness with
2. In case of death in lock up, the inquest is held by- summons from- (A/90)
a) A Police officer b) Magistrate (PG/86) a) Civil court b) Criminal court
c) Panchayat officer d) District Attorney c) High court d) Sub magistrate
3. Which carries more weigbt in a court oflaw- 17. Ossification centre appearing just before birth
a) Dying declaration (PG/87) is- (AI90)
b) Dying deposition a) Lower end of femur b) Lower end of tibia
c) Botli carry the same weight c) Upper end of humerus d) Scaphoid
d) Both are not significant 18. Super imposition technique is used in - (A/90)
4. The age of a 16 years old female is best determined a) Skull b) Pelvis
by the radiograph of- (PG/87) c) Femur d) Ribs
a) Lower end of radius and Ulna 19. Secondmolareruptsat- (A/91)
b) Upper end ofhumerus a) 6 years b) 12 years
c) Upper end of radius and Ulna c) 18 to 22 years d) 25 to 28 years
d) Xiphisternum 20. Tatoo is useful in identifying body- (A/91)
5. Legal age for marriage for boys in India- a) Living b) Dead
a) 18 b) 19 (PIIMS 87, PGI 86) c) Decomposed d) Burnt
c) 20 d)21 21. Perjuryis- (TN91)
e) 25 a) Willful utterance of falsehood
6. Most common pattern of finger print is- (AI 88) b) An act comitted within court premises
a) Arch b) Loop c) Failure to sign summons
c) Whorl d) Composite
d) Failure to appear as a witness
7. Postmortern staining can occur in some cases
22. Death sentence can be awarded by- (Kerala 91)
before death in - (AIIMS 84)
a) First class magistrate
a) Children b) Old persons
c) Cholera d) All of the above b) Second class magistrate
8. Cephalic index is used for determintion of- c) Session court
a) Race b)Age (AI 89) d) Chiefjudicial magistrate
c) Sex d) Pelvic abnormality 23. Medicolegal autopsy requires the premission
9. Dying declaration can be received by- (AI 89) of- (JIPMER91)
a) Medical officer b) Lawyer a) Police b) Magistrate
c) Police officer d) All c) Reletives d) Medical superintendant
10. Most reliable method of identificaton of a person 24. Best method for identification of a person is
b~- ~ill~ by- (JIPMER91)
a) Scars b) Anthropometry a) Scars b) Blood group
c) Fingerprinting d) Photography c) DNA Analysis d) Finger prints
11. Dying deposition b recored by- (NIMHANS 86) 25. In India, inquest b carried out by all except-
a) Doctor b) Magistrate a) Police b) Coroner (AIIMS 91)
c) Police officer d) Lawyer c) Doctor d) Magistrate
12. The power of Amnesty for capital punishment 26. Rigor mortis is simulated by- (AIIMS 92)
is vested with- (AMJ 88) a) Mummification b) Algor mortis
a) The president b) Supreme court c) Cadaveric spasm d) All of the above
c) High court d) The governer 27. Medicolegal autopsy required the permission of-
13. Pre-auricular sulcus is used for - (AMJ 88) a) Relatives (A/93)
a) Determination of Race
b) Medical superintendent
b) Determination of age
_c) Police
c) Determination of sex
d) Magistrate
d) None of the above

1)a 2)b 3)b 4)a 5)d 6)b 7)c 8)a 9)a IO)c ll)b 12)a 13)c 14)d
15)a 16)a 17)a 18)a 19)b 20)b,c 21)a 22)c 23)a,b 24)d 25)c 26)c 27)c
FORENSIC [ 416]

28. Which oftbe following is true about oath- 40. A girl of 10 years will have---- permanent and
a) Not compulsory (JIPMER 93) -temporary teeth- (PG/79, DNB 90)
b) Compulsory and binds witness for evidence given a) 8, 12 b) 8, 16
c) Holds witness responsible for consequences c) 12, 12 d) 16,8
of evidence 41 Spinal cord is opened from the- (AIIMS 86)
d) Witness is liable for prosecution if he does not a) Frontal approach b) Back side
take oath c) Lateral side d) None of the above
29. Mummification is seen in- (JIPMER 80, Delhi 92) 42. C.S.F.is required to be preserved in-
a) I.U.D. (Intrauternine death) fresh a) Alcohol poisoning (AIIMS 80, BIHAR 89)
b) I.U.D.late b)Arsenic poisoning
c) Both c) Copper poisoning
d) None d) Organophosphorous poisoning
30. Rigor mortis is simulated by- (AIIMS 92) 43. The temperature of the body rises up for first
a) Cadaveric spasm b) Mummification two hours after death. The probable condition
c) Algor mortis d) All of the above include following except- (AIIMS 81, DNB 89)
31. To make a positive identification with the help a) Sun-stoke b) Frost-bite
of a partial finger point, the points of similarity c) Septicemia d) Tetanus
should be atleast- (DNB 89) e) Strychnine poisoning
a) 10 b) 12 44. Multiplying factor of estimating from femur
c) 16 d)20 (males) is- (AIIMS 84)
32. Skull of a male differs from that of a female by a) 3.6-3.8 b)4.1-4.8
all of the following except- (JIPMER 81, Delhi 93) c) 4.9-5.6 d) 5.8-6.2
a) Capacity greater than 1500 c.c 45. The most reliable bones for the purpose of medul
b) Muscular markings over occiput are less marked lary index are the following except- (AP 91)
c) Orbits square a) Humerus b) Tibia
d) Frontal eminence small c) Radius d) Sternum
33. Multiplying factor for estimating stature from 46. Lower end of the femur can help to determine
humerus is (males)- (AllMS 84) the- (PGI 85)
a) 10-12 b)5-6 a) Age b) Height
c) 7-9 d)9-10 c) Weight d) Stature
34. Minimum quantity of blood required to be pre 47. In case of death in lock up, the inquest is held
served for chemical analysis is- (PG/81) by- (PG/86)
a) 2ml b) 10ml a) A police officer b) A magistrate
c) 50ml d) lOOml c) A panchayat officer d) Distric attorney
35. Medical certificate is- (AllMS 85) 48. The age of a 15 year old female is best determined
a) Documentary evidence b) Oral evidence by the radiograph of- (AIIMS 86, PG/8 7)
c) Hearsay evidence d) Parole a) Lower end of Radius and Ulna
36. Eruption oftempm:ary teeth will be completed by- b) Upper end of humerus
a) One to one and half year (JIPMER 81, c) Upper end of Radius and Ulna
b) Two to two and half years Kerala 90) d) Xiphisternum
c) Three to four years 49. Faint lettermarkcanbemadevisibleby-
d) Four to five years a) Infrared photography (AIIMS 86)
37. Colliquative liquefaction is seen within---- after b) Spectrophotometer
death- (AllMS 80, AMU 86) c) Ordinary light
a) Immediately b) 24hours d) X-rays
c) 72 hours d) 1 week 50. Which of the following is best for establishing
38. Union of epiphysis at the age of 17 occurs identity- (Delhi 85)
in- (BIHAR91) a) Anthropomery b) Dactylography
a) Headoffemur b) Headofhumerus c) Hair examination d) Blood
c) Lower end of femur d) All the above 51. Deep inspiration above the water level may cause
39. The enquiry into the circumstances of death is a) No effect on floatation ofbody- (Delhi 84)
called- (TN91) b) Early floatation of the body
a) Homicide enquiry b) Inquest report c) Late floatation of the body
c) Open verdict d) Adjounred verdict d) Late floatation of the body only is sea water

28)b 29)b 30)a 31)c 32)b 33)b 34)b 35)a 36)b 37)d 38)a 39)b 40)d 41)b
42)a 43)b 44)a 45)d 46)a 47)b 48)c 49)a 50)b 51)a
FORENSIC [ 417]

52. A female pelvis differs from male pelvis by all 65. The most common death by suicide is by-
except- (Delhi 86, 87, 88, 89) a) Phenobarbitone (PGI 80, AIIMS 81)
a) Obtuse subpubic angle b) Insecticide
b) Broad greater sciatic foramen c) Hanging
c) Broad lesser sciatic foramen d) Suicidal cut throat
d) Prominent muscle markings 66. The time limit for ordering of exhumation in India
53. Pearson's formula is used for- (PGI 79, AIIMS 83) is- (AIIMS 79, PGI 81)
a) Cephalic index b) Stature a) 1 month b) 1year
c) Race d) Age c) 10 years d) No limit
54. Closure of coronal sutures starts at the age 67. Best specimen ofbone for sex determination is-
of- (AIIMS 85) a) Femur b) Pelvis (Kerala 94)
a) 20years b)21 years c) Skull d) Mandible
c) 30-35 years d) 50-60 years 68. Best method to determine age upto 14 years
55. Pure aryans have which type of skull- is - (Kerala 94)
a) Mesaticephalic (JIPMER 80, AIIMS 85) a) Ossification centres b) Dentition
b) Brachycephalic c) Anthropometry d) Head circumference
c) Dolicocephatic 69. The common type of inquest in India is-
d) None of the above a) Coroner's inquest (Karn 94)
56. "Cardiac polyp" is a term used for postmortem - b) Police inquest
a) Fibrinous clots in heart (AIIMS 81) c) Judicial magistrate inquest
b) Aneurysm d) Medical examiner's inquest
c) Infarcts in heart 70. After post mortem exam body has to be handed
d) Pale patches in heart overtoinvestigating- (JIPMER95)
57. Lungs are preserved in - (AIIMS 85) a) Investing police officer b) Relatives of victim
a) Alcohol poisoning c) Magistrate d) The civil authorities
b) HCN poisoning 71. For exhumation the order should be passed by-
c) Carbon monoxide poisoning a) Chiefjudicialmagistrate (Kerala 95)
d) All of the above b) Executive magistrate
58. A child at the age of7 years has how many teeth- c) Police officer
a) 16 b) 20 (PGI 83, UPSC 81) d) Session judge
c) 24 d)28 72. Leading questions are permitted only in- (AI 95)
a) Examination in chief b) Cross examination
59. The minimum age for the appointment of
c) Dying declaration d) Re-examination
governoris- (AIIMS 79, AMC 81)
73. If angle of mandible is obtuse it means that the
a) 18years b)21 years
bone belongs to- (Delhi 96)
c) 35 years d) 45 years a) Adultmale b)Adultfemale
60. Who can commute a death sentence-(JIPMER 80) c) Young female child d) Elderly person
a) High court b) Supreme court 74. Identical twins may not have- (Karnat 96)
c) President d) Any of the above a) Same DNA finger
61. Crural index is calcutated as-(PGI 80, AIIMS 82) b) Same finger print pattern
a) Length of Radius b) Length of tibia c) Same blood group
c) Length ofHumerus d) Length of ulna d) Same HLA system
62. The sentence awarded by a court may be enhanced 75. Dying deposition is more important than dying
bythe- (AIIMS 83) declaration because itis- (TN 97)
a) Magistrate court b) High court a) Made to a magistrate
c) Higher court d) None ofthe above b) Oath is taken
63. While despatching blood and urine chemical c) Made in presence of accused
analysis sodium flouride is added as preservative d) Cross examination is permitted
in the following concentration- (PGI 80, AMU 86) 76. Best method for identification of human is- (AI 97)
a) 30mg/10ml b)40mg./10ml a) Blood grouping b) Dactylography
c) 50mg.lml d) 100mgl10ml c) Anthropometry d) Gustuffon's formula
64. Exhumation is usually done in the- (AIIMS 80) 77. 100 % sure differentiation of sex is possible by
a) Early morning b) Evening pelvic assessment of all except- (PGI 96)
c) Night d)Anytime a) Subpubic angle b) Obturator foramen
c) Preauricular sulcus d) Greater sciatic notch

52)d 53)b 54)c 55)c 56) a 57)b 58)c 59)c 60)d 61)b 62)c 63)d 64)a 65)c
66)d 67)b 68)b 69)b 70)a 7l)a 72)b 73)c,d 74)b 75)a,d 76) b 77)None
FORENSIC [ 418 ]

78. Post-mortem autopsy is done for- (JIPMER 98) 93. Corpus delicti means- (TN 02)
a) Whole body a) Essence of crime
b) Parts which are injured b) Non decomposed body
c) Parts which have been asked for in inquest c) Identification of dead person
d) Parts which relatives have given permission for d) Decomposed body
79. After postmortem body is handed over to- 94. An order for exhumation can be given by-
a) The authority who has conducted inquest in that a) District collector (AIIMS 81, AP 02)
particular case (JIPEMR 98) b) Additional district magistrate
b) Police station near by c) Sub collector
c) Coroner
d) Any of the above
d) Chiefmagsitrate
95. Subpoena is a kind of- (AIIMS 81, AMU 02)
80. Most reliable method ofindetification of a person
a) Decomposed body tissue b) Designation
is - (Rajasthan 97)
c) Document d) Court tribunal
a) Gustafson method b) Galton method
c) Anthropometry d) Scars e) Indelible ink
81. Sodium Fluride is used for preservation of-(Al 98) 96. Puglistic attitude is seen in - (All India 02)
a) Urine b) Blood a) Drowning b) Protein coagulation
c) Alcohol d) Carbolic acid c) Electrocoagulation d) Hanging
82. Perjury is- (MAHE 98) 97. Two carpal bones are radiologically seen in the
a) Wilful utterance of falsehood wrist X-ray of most ofthe children by the end of-
b) Grevious injury a) 1 year b) 2 years (UPSC 96)
c) Contributory negligence c) 3 years d) 4 years
d) Criminal negligence 98. Atavism is when a child resembles his -(AIIMS 99)
83. First finger loop bureau was established in - a) Parents b) Grandparents
a) London b) Paris (MP 2K) c) Cousin d) Twin
c) Calcutta d) Madras 99. The pubic arch in females is above- (PGJ 88)
84. Rectified spirit not used as preservative in case of- a) 60-70 b)70-75
a) Phenol b) Cyanide (Bardwan 2K) c) 100 d) 130
c) Insectides d) Aluminium phosphide 100. Most specific and sensitive for identification
85. Exhumaion can be done in India- (Bardwan 2K) is- (PG/98)
a) After 7 years b) After 2 years a) Anthropometry b) Dactylography
c) AfterlOyears d)Atanytime c) Skull d) Pelvis
86. The period of mixed dentition is between the 101. Most common type in dactylography is- (PG/98)
ages of- (Orissa 98)
a) Wholrls b) Loops
a) 2-5 years b) 6-11 years
c) Composite d) Arches
c) 12-14years d)l5-17years
102. Sex determination form pelvis possible upto 95%
87. 20 permanent teeth and 8 temproray teeth are
accuracy from following except- (PGJ 98)
seen at the age of- (Kerala 2001)
a) lOyrs b) 11 yrs a) Greater sciatic notch b) Pre auricular sulcus
c) 9 yrs d) 12 yrs c) Obturator foramen d) Sub public arch
88. Cross examination of the prosecution witness is 103. The fetus can be considered viable if the following
doneby- (Manipal OJ) ossification center appears- (PGI 99)
a) Public prosecutor b) Defence counsel a) Calcaneum b) Manubrium stemi
c) Judge d) None c) Lowerendoffemur d) None ofthe above
89. Firstpermanenttooth to arise- (JIPMER 02) 104. Testamentary capacity refers to- (PGJ 99)
a) Incistor b) Canine a) Ability to make a valid will
c) Premolar d) Molar b) Criminal liability
90. Temperature of a dead person is best measured by- c) Right to vote
a) Mouth b)Rectum (TN02) d) Ability to give evidence
c) Armpit d) Groin 105. 99% sex difference can be done by all features of
91. Rigor mortis in a foetus develops after attaining- pelvic bones except- (PGI 2000)
a) 3 months of age b) 4 months of age (TN 02) a) Subpubic angle b) Greater sciatic notch
c) 7monthsofage d) lOmonthsofage c) Preauricular sulcus d) Public rami
92. Cheiloscopy is the study of- (TN 2002) 106. Best bone to assess age between 20-50 yrs-
a) Foot prints b) Palatal prints a) Skull b) Ribs (PGI 2000)
c) Finger prints d) Lip prints c) Sternum d) Symphysis pubis

78)a 79)a 80)b 8l)c 82)a 83)c 84)a 85)d 86)b 87)b 88)b 89)d 90)b 9l)c 92)d
93)a 94)b 95)c 96)b 97)b 98)b 99)c lOO)b lOl)b 102)None 103)c 104)a 105)None 106)a
FORENSIC [ 419 ]

107. Cephalic index is useful for determination of- 119. A boy has 20 permanent and 8 temporary teeth
a) Age b)Sex (PG/03) his age would be- (APPPGE 04)
c) Race d)Religion a) 9years b) lOyears
e) Caste c) 11 years d) 12 years
108. A skull with squared orbit & rectangular palate 120. Corpus delciti means- (TN 04)
with cephalic index 75 with broad nose, probably a) Warrant of arrest b) Body of crime
the race is- (PGI 04) c) Dead body d) Body of a criminal
a) Negroes 121. Exhumationisorderedby- (J&K05)
b) Mongoloids a) Civil Surgeon b) Superintendent of police
c) Europeans c) Circle Inspector d) District magistrate
109. Blackening and tattooing of skin and clothing can 122. Which is not a medical negligence act-(MAHA 05)
be best demonstrated by- (AI 03) a) 37IPC b)304A
a) Luminol spray c) 351IPC d)312IPC
b) Infra red photography 123. The following may be the example of privileged
c) Ultra violet light communicationexcept- (MAHA 05)
d) Magnifying lens a) An impotent person marrying
110. Perjury means giving willful false evidence by a b) A syphilitic taking bath in a public pool
witness while under oath, the witness is liable to c) Engine driver found to be color blind
be prosecuted for perjury and the imprisonment d) Pilot having hypermetropia
may extend to seven years. This falls under which 124. In India, magistrate inquest is done in the following
section ofiPC- (AI 03) cases except- (AI 05)
a) 190 ofPenal Code b) 191 ofPenal Code a) Exhumation cases
c) 192 of Penal Code d) 193 of Penal Code b) Dowry deaths within 5 years of marriage
111. Atthe age of9 number of teeth are- (Mahara 02) c) Murder cases
a)l2 b)l6 d) Death of a person in police custody
c) 20 d)24 125. Medical qualifications awarded by institutions
112. Jurisdiction of land assessment comes outside India and recognized by MCI are registered
under- (Mahara 02) in- (A/06)
a) Supreme court b) Sessions count a) First schedule of Indian Medical Council Act 1956
c) District court d) None of the above b) Second schedule of Indian Medical Council Act
113. "Juvenile court" is presided over by -(Mahara 02) 1956
a) I class woman magistrate c) Part I ofthird schedule oflndian Medical Council
b) II class woman magistrate Act 1956
c) II class male magistrate d) Part II of third schedule of Indian Medical Council
d) None of the above Act 1956
114. The cephalic index of Indian population is between- 126. Powers of a 1"1 class magistrate - (PGI June 06)
a) 70-75 b)75-80 (AI04) a) Fine upto Rs. 5000 & 3 years imprisonment
c) 80-85 d)85-90 b) Fine uptoRs. 3000
115. Dichotomy means- (Manipal 04) 127. Subpnoeais- (Manipal 06)
a) Fee splitting b) Summons a) Summons b) Panchanama
c) Civil wrong d) None c) Suratra d) Review
116. If a patient survives after having given dying 128. Power of chief judicial magistrate is imprisonment
declaration then it stands as - (Jipmer 04) upto- (MAHE 07)
a) Corroborative evidence b) Non valued a) 7 years b) 5 years
c) Valid for 48 hours d) None c) 3 years d) Death sentence
117. "Dry Submarine" in custodial torture injuries 129. Spalding sign is seen in (AI 07)
refers to - (Karnataka 04) a) Drowning b) Maceration
a) Falanga beating, on soles of feet c) Mummification d) Starvation
b) Tying of a plastic bag over the ears 130. Spalding's sign is seen in - (Aiims May 07)
c) Immersing head in contaminated water a) Drowning b) Mummification
d) Suspending the accused by the ankles c) Adipocere d) Maceration
118. Cheiloscopy is the study of prints of- (AIIMS 04) 131. Affiliation refers to- (MAHE 07)
a) Foot b) Fingers a) Paternity dispute b) Divorce case
c) Palate d) Lips c) Rape d) Adultery

107) c 108) a 109) b 110) d 111) d 112) c 113) a 114) a 115)a 116) a 117) b 118)d 119)b 120)bl2l)d
l22)c 123)a 124)c 125)b 126)a 127)a 128)a 129)b 130)d 13l)a
FORENSIC [ 420 ]

132. DNA finger printing doe with all except- 143. Ossification centres appears- (UP 08)
a) WBC b)RBC (UP 07, 06) a) Just before birth
c) Saliva d) Spermatozoan b) After upper and of the femur
133. Exhumation order is given by- (UP 07, 03) c) Upper end of the tibia
a) Coroner's court b) Police officer d) After lower end offemur
c) Magistrate d) All of the above 144. Corporobasal index is useful for determination of-
134. If death of a patient occurs during surgery due to a) Race b) Age (AI 08)
the negligency of the surgeon, then he can be charged c) Stature d) Sex
under- (Corned 07) 145. Finger print bureau was first established in-
a) 299IPC b)300IPC a) India b) England (Aiims May 08)
c) 304AIPC d)304BIPC c) USA d) France
135. There is no criminal responsibility below the age 146. Power of chief judicial magistrate is imprisonment
of- (Corned 07) upto- (Manipal 08)
a) 5 years b)7 years a) 7 years b) 5 years
c) 10 years d) 12 years c) 3 years d) Death sentence
136. The center of ossification of which one of the 147. Contributory negligence if proved is beneficial to-
following is used as medicolegal evidence for fetal a) The patient (DPGEE 08)
viability- (Corned 07) b) The treating doctor
a) Headofthefemur b)Distalendoffemur c) Both of the above
c) Greater trochanter d) Lesser trochanter d) None of the above
137. Nottrue aboutconsentis- (AIIMS Nov 07) 148. A leading question is not asked during-
a) Explaining to the patient the nature ofhis disease, a) Cross-examination (DPGEE 08)
the proposed treatment or procedure, the b) Examination-in chief
alternative procedure, the risks and benefits c) Re-examination
involved in both the proposed and alternative d) All of the above
procedure, the relative chances of success or 149. Contributory negligence is a defence in?
failure of both procedures a) Civil negligence (APPG 08)
b) In cases of emergency consent is not necessary. b) Criminal negligence
c) In criminal cases, the victim cannot be examined c) Ethical negligence
without his/her consent. d) None
150. The test in which weight oflungs is compared with
d) Blanket consent authorizers the attending
body weight is/are- (PGI Dec 08)
physician or surgeon to do what ever he thinks
a) Fodere's Test b) Ploucquet's Test
best for him under the circumstances. c) Raygat's Test d) Wredin Test
138. Sex can be established by examining hair root cells e) Flourence Test
for the presence of- (Corned 08) 151. When two objects come in contact with each other,
a) Davidson body b) Barr body transfer of material takes place in accordance with-
c) Golgi body d) Medullar index a) Me naughten rule (PGI Dec 08)
139. Among the secondary changes in tooth the most b) Locard exchange principle
useful one for age determination is- (Corned 08) c) Rule ofHaase
a) Attrition d) Locard's method
b) Secondary dentine deposition 152. Privileged communication is made between-
c) Root resorption a) Patient and Doctor (AI 09)
d) Root transparency b) Doctor and court ofLaw
140. The most common type of fingerprint is- (UP 08) c) Doctor and relative
a) Loop b)Arch d) Doctor and concerned authority
c) Composite d) Whorl 153. Disease which permanently alters the finger prints
141. Cephalic index determines- (UP 08) is- {AI09)
a) Age b)Sex a) Leprosy b) Tuberculosis
c) Race d) Socio-economic status c) Diabetes d) Dermatitis
142. Medical man is witness box is- (UP 08) 154. Casper's dictum is used for- (AI 09)
a) Common witness a) Identification of body
b) Expert witness b) Estimation of time since death
c) Common and Expert witness c) Establishing cause of death
d) Voluntary witness d) Establishing weapon oflnjury

132)c 133)a,c 134)c 135)b 136)b 137)d 138)b 139)d 140)a 141)c 142)c 143)a 144)d 145)a
146)a 147)b 148)c 149)b 150)b 15l)b 152)d 153)a 154)b
FORENSIC [ 421 ]

155. All of the following are type of fingerprinting except- 167. A teacher slapped a 6th standard student after which
a) Arch b) Loop (PGI June 09) she suffered from 25% hearing loss in left ear. This
c) Composite d) Circle was corrected after a surgery. Which type ofinjnry
e) Whorls is this? (AI/MS Nov 09)
156. In India the age of consent for marriage of a female a) Simple b) Grievous
is- (COMED 09) c) Dangerous d) Serious
a) 15 years b) 16 years 168. Incisional wound on genital is seen commonly in-
c) 18 years d) 20 years a) Accidents b) Suicides (AIIMS May 10)
157. Diaphanous test is the test to detect - (COMED 09) c) Homicides d) Postmortem artifact
a) Pregnancy 169. A woman died within 5 years of marriage under
b) Stoppage of respiration suspicions circumstances. Her parents complained
c) Stoppage of circulation that her in laws used to frequently demand fordowry.
d) Presence of blood Under which of the following sections can a
158. Rokitansky's method is- (DELHI PG Feb. 09) magistrate authorize ofthe case- (AI 10)
a) An autopsy technique for infants a) Section 174CrPc b) Section 176 CrPc
b) Used in heavy metal detection c) Section 304 IPC d) Section 302 IPC
c) Used for estimation of age using skull sutures 170. Which of the following Indian Penal Codes offers
d) Used in brain mapping immunity to doctor if he/she treats a patient without
159. All are true about cadaveric spasm, except- his/her consent in case of an emergency?
a) Appears instantaneously by any method after a) IPC91 b)IPC92 (Maharashtra 10)
death (DELHI PG Feb. 09) c) IPC93 d) IPC94
b) Cannot be produced by any method after death 171. As per Indian Medical Council 2002 it is compulsory
c) Molecular death does not occur to maintain the medical records pertaining to all
d) All the muscle ofthe body are involved indoor patients for a minimum period of__years-
160. First sign of decomposition in a dead body a) 1 year b) 2 years (Maharashtra 10)
is- (DELHI PG Feb. 09) c) 3 years d) 4 years
a) Greanish discoloration of skin over face 172. The age of a 15-year old female is best determined
b) Greanish discoloration of skin over right iliac fossa by the radiograph of- (DPG 10)
c) Greanish discoloration of skin over left iliac fossa a) Lower end of Radius and Ulna
d) Reddish discoloration of skin over right iliac fossa b) Upper end ofhumerus
161. What is the age of a child having 20 permanent and c) Upper end of Radius and Ulna
8 temporary teeth? (DELHl PG Mar. 09) d) Xiphisternum
a) 13 years b) 11 years 173. When a group of muscles of a dead body were in
c) 9 years d) 15 years state of strong connaction innnediately prior to death
162. Culpable homicide not amounting to murder is under and remain so even after death, the condition is
section- (DELHI PG Mar. 09) termedas- (DPG10)
a) 304 b) 304A a) Gas stiffening b) Rigor mortis
c) 302 d)376 c) Cadaveric spasm d) Cold stiffening
163. To compare weight of lung of body which of the 174. Which of the following sections ofiPC are concerned
following test is used - (PGI Nov 09) withdowrydeath? (DPG 10)
a) Cavette test b) Teichman test a) 300 b)302
c) Takayama test d) Fordere test c) 304A d)304B
e) Ploucqet test 175. A cognizable offence signifies- (DPG 10)
a) Imprisonment upto 2 years
164. At what age, a person can be called as major
b) Imprisonment upto 5 years
(According to ''Indian Majority Act")-
c) Arrest without warrant
a) 16 b) 18 (PGI Nov 09)
d) Fine upto Rs. 10,000
c) 21 d)25 176. A cbildhas20 temporary teeth and8 permanent teeth.
e) 12 Whatis the age of the child- (Punjab 10)
165. Cadaveric spasm seen- (PGI Nov 09) a) 10 years b) 11 years
a) After 6 hrs b)After 24 hrs c) 13 years d) 14 years
c)After12hrs d)After48hrs 177. Which carries more weight in a court oflaw-
e) Immediately after death a) Dying deposition {Punjab 10)
166. Section IPC 377 deals with- (AliMS Nov 09) b) Dying declaration
a) Unnatural sex offences b) Rape c) Both carry the same weight
c) Incest d) Adultery d) Both are not significant

155)d 156)c 157)c 158)a 159)d 160)b 161)b 162)b 163)e 164)b,c 165)e 166)a 167)b 168)c
169)b 170)b 171)c 172)a 173)c 174)d 175)c 176) None 177) b
FORENSIC [ 422 ]

178. Determination of sex is done best by which bone- 189. In civil negligence, onus of proof lies on-
a) Femur b)Pelvis (Punjab 10) a) Judicial first degree magistrate (AIIMS May 11)
c) Skull d) Mandible b) Police not below the level of sub inspector
179. A skull with squared orbit & rectangular palate with c) Doctor
cephalic index 75 with broad nose, probably the race d) Patient
is- (Punjab 10) 190. Condition promoting adipocere formation - (AIIMS
a) Negroes b) Mongoloids
a) Dry and hot b)Hotandhumid May 11)
c) Europeans d) Oriental
c) Dry & optimum d) cold and moist
180. Following surgery, a surgeon leaves behind a swab
191. According to MCI act, medical education granted by
in the abdominal cavity. He is liable for punishment
under- (Jiprner 10) Indian universities comes under- (AIIMSMay 11)
a) Criminal negligence a) Schedule I ofMCI act
b) Civil negligence b) Schedule II ofMCI act
c) Res judicata c) Part I of schedule III
d) Novus actus interveniens d) Part II of schedule III
181. The confirmatory testfor blood is- (Corned 10) 192. Dental numbering is done by all, except- (AI 11)
a) Benzidine test a) FDI Two Digit System
b) Luminoltest b) Anatomic and diagrammatic charting
c) Leucomalachite green test c) Palmer Notation
d) Teichmann test d) Aerogram
182. "Declaration of oslo" deals with- (Corned 10) 193. Which Indian law describes dying declaration?
a) Abortion b) Torture a) Sec.39Cr.P.C (Karn 11)
c) Human experimentation d) Consumerism b) Sec. 278 Cr.P.C
183. True about battered baby syndrome- (PGI May 10) c) Sec.32 lEA
a) Posterior rib# may be seen in X- ray d) Sec. 118 lEA
b) Injury at the side of metaphysic is specific 194. With respect to dating of a bone, a bone more than
c) Epiphyseal injury is common
100yearsoldcontain- (Karn 11)
d) Babygram is diagnostic
a) 6 amino acids b) 7 amino acids
e) Me casue of death is extradural hemorrhage
c) 8 amino acids d) 9 amino acids
184. All are true about postmortem staining except-
195. In Polygraph test "GSR" stands for- (Karn 11)
a) Occur immediate after death (PGI May 10)
a) Guilt Score Reaction
b) Common in dependent part
b) Galvanic Skin Reaction.
c) Disappear with rigor mortis
c) Guilt Sensor Reaction
d) Margins are raised
d) Guilt Sensitivity Reaction.
e) Not found in internal organ
196. Algor mortis is- (PUNJAB 11)
185. True about suspected animation- (PGI May 10)
a) Cadaveric spasm
a) Common phenomenon in yogic person
b) Can be voluntary b) Postpartum rigidity
c) Similar to molecular death c) Cooling of the body
d) Similar to brain death d) Post mortem caloricity
e) Person can be revived 197. Species identification is done by- (PUNJAB 11)
186. TrueaboutMoleculardeatb- (PG!Nov.10) a) Neutron activation analysis (NAA)
a) Same as cellular death b) Precipitin test
b) Same as cellular death as indicated by rigor mortis c) Benzidine test
c) Indicated by stopping of heart function and flat d) Spectroscopy
£(X} 198. Most common primary finger print pattern-
d) Brain death a) Loop b) Whorl (Jiprner 11)
e) All cells ofbody die c) Mixed d)Arch
187. Widmark's formula is used in calculating-
a) Quantity of alcohol in body (AIIMS Nov 10} SEC -Ill : MEDICOLEGAL ASPECTS OF
b) Time since death DEATH, INVESTIGATIONS, HANGING,
c) Infliction of injuries STRANGULATION, STARVATION,
d) Stature
188. During autopsy, if spinal cord is to be examined what COLD&HEAT
is the most commonly used approach-?41IMSMay 11} 199. Rigor moritis develops ....after death- (PGI 86, 87)
a) Anterior b) Posterior a) 1/2- 1 hr. b) 1 to 2 hrs
c) Lateral d)Antero-lateral c) 3 to 6 hrs. d) 12 hrs

178)b 179)a 180)a 181)d 182)a 183)a,b,c,d 184)a,c,d,e 185)a,b,e 186)a,b,e 187)a 188)b 189)d
190)b 191)a 192)d 193)c 194)b 195)b 196)c 197)b 198)a 199)d
FORENSIC [ 423]

200. Cadaveric spasm- (PGI 86, 87) 213. Floatation of a body in water is due to- (Kerala 91)
a) Instant in onset a) Flesh being eaten by fishes
b) Confined to a small group of muscle b) Presence of gases
c) Occurs only in voluntary muscles c) Waterlogging of body
d) Indicates that the person was alive at that time d) All of the abve
e) All of the above 214. Honeycombing is seen in liver in cases of-
201. Foamy liver is seen in- (AIIMS 87) a) Cirrhosis b) Rupture (Kerala 91)
a) Putrefaction b) Drowning c) Putrefaction d) Hydatid disease
c) Mummification d)Adipocere 215. Presence of maggots at post mortem indicates-
202. Post mortem staining gets fixed after- (PGI 87) a) Duration since death (JIPMER 91)
a) 2-3 hrs., b) 3-4 hrs., b) Cause of death
c) 5-6 hrs., d) 6-7hrs., c) Mode of death
e) 7-8hrs., d) Posture of body at time of death
203. Mummification occurs when- (PGI 87) 216. Cold stiffening may be associated with-
a) High atmospheric temperture is present a) Trench foot (AIIMS 80, AMC 85)
b) Dry air condition b) Immersion foot
c) Wind is present c) Body rigid, heavy and stiff
d) All are correct d) Any of the above
204. If a body is left undisturbed, for bow long does 217. Colour changes ofputrefacion are first observed
post mortem staining persists- (PGI 87) in the- (AIIMS 86)
a) Few hours a) iliac fossa b) Popliteal fossa
b) Few days c) Cubital fossa d) Arm pits
c) Few months e) Neck
d) Persists till it merges with discolouration of 218. Rigor mortis in India in summer sets
putrefaction in- (JIPMER 80, Delhi 92)
205. What poisoning retards putrefaction- (PGI 87) a)2-3hrs b)18-36hrs
c) 2-5 days d) 1 week
a) Phosphorus b) Arsenic
219. Number of hours since death is calculated by
c) Mercury d) Lead
multiplying the fall in rectal temperature -
e) Copper
a) 0.33 b)0.67 (JIPMER8l,Delhi93)
206. Suspended animation may be seen in- (AIIMS 85)
c) 1.0 d) 1.33
a) Electrocution b) Morphine poisoning
e) 1.5
c) Strangulation d) Rape
220. Defmition of suspended animation is-
207. Hypostasis lastsfor- (PG/88)
a) It is a condition of apparent death when no signs
a) Hours b) Days
of life are discovered as the functions are inter
c) Weeks d) Months
rupted for sometime or are reuced to minimum
208. Cadaveric spasm- (PGI 88)
b) Sign of death are irreversible
a) Immediately after death
c) Only signs of brain death are present
b) After 2 hrs of death
d) None
c) Affect voluntary muscles
221. In putrefaction greenish discolouration is first
d) Affects involuntary muscle
seen in- (TN91)
209. The rate of cooling down of dead bodies in
a) Lion b) Epigastrium
tropical climate is- (PGI 88)
c) Right iliac fossa d) lower limb
a) 0.2°c/hr., b) 0.5°clhr.,
222. Maggots in a dead body do not appear before-
c) 1.0°clhr., d) 1.5°clhr.,
a) 4 hours b) 12 hours (PGI 80, Delhi 87)
210. Last organ to be affected in putrefaction-
c) 24 hours d) 48 hours
a) Prostate b) Uterus (PG/85)
223. The following are correct about cadaveric rigidity
c) Kidney d) Thyroid
in contrast to cadaveric spasm except-
211. Immediate stiffnes postmortem occurs in- (AI 90)
a) Condition is seen after death (AIIMS 80,
a) Cadaveric rigidity b) Cadaveric spasm
b) Affects all mucles of body PGI 84)
c) Rigor mortis d) Algor mortis
c) Helpful in determining time since death
· 212. Mummificationis- (A/91)
d) None of the above
a) Dessication of the body
224. Rigor mortis first starts in- (PGI 81, AIIMS 82)
b) Pugilistic attitude ofthe body
a) Upper eyelids b) Lower eyelids
c) Electric burns
c) Lower limbs d) Fingers
d) Burking

200)e 20l)a 202)e 203)d 204)d 205)b 206) a,b 207) b 208) a,c 209) b 210)b 21l)b 212)a 213)b
214)c 215)a 216)c 217)a 218)a 219)e 220)a 221)c 222)d 223)a 224)a
FORENSIC [ 424]

225. Marbling is noticed by- (AJIMS 84) 241. Rigor mortis occurs first in which voluntary
a) 18 hours b) 36 hours muscle- (JIPMER 02)
c) 48 hours d) 24 hours a) Muscles of eyelids b) Small muscles ofhands
226. In rigor mortis muscle- (Kerala 94) c) Neck muscles d) Face muscles
a) Stiffens b) Shortens 242. Entomology of cadaver helps in finding- (1N 02)
c) Stiffens and shortens d) Stiffens and lengthens a) Time since death b) Mode of death
227. Putrefaction is a- (Karn 94) c) Manner of death d) Identify the disease
a) Perimortem sign of death
243. Putrefaction is facilitated by following except-
b) Immediate sign of death
a) Very high temp b) Free air
c) Early sign of death
d) Late sign of death c) Damp environment d) Shallow grave
228. Mummificationisdueto- (Al95) 244. Maggots in rainy day are seen within- (PGI 97)
a) Putrefaction b) Saponification a) 6 hours b) l-2 days
c) Dessication d) Extreme cold c) 2-4 days d) 5-6 days
229. Adipocere is likely to occur in- (Karnat 96) 245. Rigor mortis in winder season in Northern India
a) Warm humid climate b) Dry hot climate lasts for- (PGI 97)
c) Burial in sandy soil d) All the above a) 24-48HR b)>24HR
230. Post-mortem caloricity is seen with- (AI 97) c) 2-3 days d)> 3 days
a) Strychnine poisoning 246. True about mummification is- (PGI 97)
b) Lead poisoning a) External features are preserved
c) Mercury poisoning b) High atmospheric temperature is needed for it
d) Phosphorous poisoning c) Occurs in dry air
231. First indication of post mortem putrefaction is- d) All of the above
a) Discoloration of right iliac fossa (AI 97) 247. Rigor mortis starts in- (PGI 98)
b) Puffmess offace a) Eyelids b) Heart
c) Bullous lesion c) Voluntary muscles d) Limbs
d) Scaling of skin 248. First internal sign of putrefaction is found-(PGJ 98)
232. Immediate rigidity if a group of muscle without a) Below the liver b) In mediastinum
passing into stage of primary relaxation- (AI 97) c) Below the spleen d) In kidney
a) Cadaveric spasm b) Cadaveric rigidity
249. Suspended animation is defined as- (PGI 98)
c) None
a) Apparent death from which person can be aroused
233. Maggots are seen on the dead body after- (PGI 96)
b) True death from which person can not be aroused
a) One day b) 2 to 3days
c) May last for days or weeks
c) 3- 4 days d) one week
d) Can not be produced voluntarily
234. Rate of cooling helps in determining-(JJPMER 98)
a) Time of death b) Manner of death 250. Postmortem caloricity is- (PGI 98)
c) Place of death d) Cause of death a) i ed temp within 2 hours of death
235. Maggots appear in body in rainy season within- b) Rapid fall in temp after death
a) 24hr b) 1-2day (PG/97) c) i ed temp after 6-8 hours of death
c) 2-3 day d) 3-4 days d) Decreases in temperature followed by rise
236. WhatissuspendedAnimation- (PGI 97) 251. Adipocere formation is seen in- (PGI 99)
a) Animal is suspended in air a) Dead body exposed to air
b) The subject is alive but shows no signs of life b) Dead body buried in damp, clay soil
c) Hibernation c) Burial in dry hot air
d) Deep sleep d) Prolonged immersion in water
237. Suspended animation occurs after- (DIA 99) 252. Trueaboutcadavericspasm- (PGI 2000)
a) Electrocution b) Cyanide poisoning a) Develops immediately after death
c) Bums d) Drowning b) May develop hours after death
238. Putrefraction starts in- (PGI 98) c) Develops only in facial muscles
a) Liver b) Lung d) Involves individuaVsmall group of muscles
c) Brain d) Heart 253. True statement regarding mummification-
239. Rigor mortis first starts in- (PGI 98) a) Time since death (PGI OJ)
a) Heart b) Eye lids b) Facial features are preserved
c) Lung d)Limbs c) Skin is shrunken, dry and brown
240. Rigor mortis starts in- (DNB 2001) d) Implies the body was recovered from a hot and
a) Right iliac fossa b) Left iliac fossa dry place
c) Brain d) Heart e) Modified putrefaction

225)b 226)c 227)d 228)c 229)a 230)a 23l)a 232)a 233)b 234)a 235)b 236)b 237)a 238)a
239)a 240)d 24l)a 242)a 243)a 244)c 245)a 246)d 247)b 248)a 249)a 250)a 251) b,d 252) a,d
253) a,c,d,e
FORENSIC [ 425]

254. Post mortem lividity persists till- (PGI 01) 267. The conclusive post mortem finding of death by
a) 24hours b) 18hours drowning- (PGI 86, 87)
c) 2-3 days d) 48 hours a) Cutis anserina
e) Merges with putrefactive changes b) Presence of water in the middle ear
255. True about adipocere- (PGI 02) c) Edematous lungs
a) Common in dry and cold climate d) Demonstration of diatoms in the organs of the
b) Also called saponification body
c) Sweetish smell
268. White lathery froth is seen in deaths duee to-
d) Occurs due to gradual hydrolysis and
hydrogenation of fats a) Drowning (AI 88)
256. Rigor mortis is simulated by- (PGI 02)
b) Hanging
a) Cold stiffness b) Heat stiffness c) Organophosphours poisoning
c) Tetanus d) Strychnine poisoning d) Mineral acid poisoning
e) Putrefaction 269. Fractures ofthe hyoid bone results from all except-
257. Which is true about Somatic death- (PGI 03) a) Throttling b) Hanging (PGI 86)
a) Cooling of the body c) Choking d) Ligature strangulation
b) Cessation of spontaneous respiration 270. Death of judicial hanging is because of- (AMJ 88)
c) Cessation of circulation a) \Tasovagalshock
d) Flat isoelectric E.E.G b) Occlusion of airway
e) Deep unconsciousness with no response to c) Fracture disclosation of upper cervical vertebra
external stimuli or internal need d) Coma
258. Prolongation of Rigor mortis is seen in- (PGI 03) 271. In starvation, the gall bladder may be- (PGI 85,
a) Lead b) Arsenic a) Atrophied b) Distended (JIPMER 90)
c) Mercury d) Copper c) Show stones d) Nonnal
259. True about rigor mortis- (PGI 03)
272. Hanging is established by- (PGI 85)
a) First appear in chin
a) Fracture of hyoid cartilage
b) It last 18-36 hours in summer
c) Seen immediately after death b) Fracture of thyroid cartilage
d) It disappears in the sequence as it appears c) Staining of saliva
e) It last 24-48 hours in winter d) Ligature mark
260. Rigor mortis first seen in- (PGI 04) 273. Hanging with the feet touching the ground is seen
a) Myocardium b) Upper eye lid in- (TN90)
c) Lower eye lid d) Jaw a) Partial hanging b) Complete hanging
e) Leg c) Homicidal hanging d) Suicidal hanging
261. Post- mortem caloricity is increased in all except- 274. Hyoid fracture is seen in- (JIPMER 90, TN 90)
a) Burns b) Sunstroke (Jipmer 03) a) Hanging b) Strangulation
c) Tetanus d) Septicaemia c) Burking d) Throttling
262. Algor Mortis is- (Jipmer 03) 275. Petechial hemorhages in drowning is seen in-
a) Cadaveric spasm b) Hypostasis a) Subpleural tissutes of lung (JIPMER 90)
c) Cooling of body d) Ethyl alcohol b) Submucosa of trachea
263. Rigor mortis starts at- (TN 03) c) Bronchioles
a) Hands b) Eyes d) Alveoli
c) Heart d) Intestines
276. Paultaf's hemorrhages occurs in which injury-
264. Postmortem lividity is unlikely to develop in a case
of- (A/03) a) Strangulation b) Hanging (A/90)
a) Drowning in well c) Drowning d) Burns
b) Drowning in a fast flowing river 277. Horizontal ligature mark in the neck is seen in-
c) Postmortem submersion a) Throttling b) Hanging (A/91}
d) Drowning in chlorinated swimming pool c) Strangulation by ligature d) Chocking
265. Aseptic autolysis is seen in- (Kerala 04) 278. Emphysemaaqueosumisseenin- (AIIMS 92)
a) Adipocere b) Maceration a) Wet drowning b) Secondary drowning
c) Putrefaction d) Mummification c) Dry drowning d) Immersion syndrome
266. Burking consits of- (PGI 85,87, UPSC 87) 279. In a acute starvation feeling of intense hunger
a) Smothering and Traumatic asphyxia lasts - (AIJMS 79, AMU 86)
b) Choking and drowning a) 12-24hours b)24-36hours
c) Gagging only c) 36-48 hours d) 48-60 hours
d) Traumatic asphyxia only

254) e 255) b,c,d 256) a,b,e 257) b,c,d,e 258) b 259) b,d,e 260) a 261) a 262) c 263)c 264)b 265)b 266)a
267)c,d 268)a 269)c 270)c 271)p,c 272)c,d 273)a 274)d 275)a 276)c 277)c 278)a 279)c
FORENSIC [ 426]

280. Which of the following sign will indicate that the 292. Hanging is defined as- (Kerala 94)
death was antemortem- (Delhi 86, TN 93) a) Suspension of body by a ligature, body weight
a) Cutis anserina acting as constricting force
b) Washerwoman's skin b) Suspension of body by ligature after death
c) Weed and grass in the hands c) Obliteration of air passages by external
d) Cyanosis compression
281. Cafe coronary commonly occurs when a person d) Mechanical interference to respiration.
is - (PGI 79, Bihar 89) 293. Maximum congestion is seen in- (Kerala 94)
a) Intoxicated b) Eating fatty food a) Choking b) Hanging
c) Eating meat d) Eating fish c) Strangulation d) Drowning
282. Accidental drowning is ruled out if- (PGI 80, 84) 294. Oedema aquosum occurs in - (Karn. 94)
a) The body is tied up inside a gunny bag a) Wet drowning
b) Hand and feet are tied by a ligature in way that is b) Dry drowning
inconsistent with suicidal drowning c) Secondary drowning
c) Heavy weights are attached in such a way that d) Immersion syndrome
suicidal drowning is excluded 295. Gettler's test is positive in- (AIIMS 94)
d) All of the above a) Hanging b) Poisoning
283. Froth of drowning can be differentiated from that c) Strangulation d) Drowning
due to other causes by- (AIIMS 86) 296. Dry drowning is defmed as- (Delhi 96)
a) Microscopic detection of mucous a) Drowning in sea water
b) Detecting excess of salt in the froth b) Drowning is cold water, death dut to cardiac
c) Both a and b arrest
d) None of the above c) Drowning of a dehydrated body
284. A method of homicidal smothering and traumatic d) Drowning where death is due to laryngospasm
asphyxia is known is- (PGI 80, Delhi 86) 297. The most specific feature of death due to hanging is-
a) Choking b) Gagging a) Tardieu sports (Delhi 96)
c) Burking d) Overlying b) Ligature mark
285. Hyperventilation before drowning- (PGI 85) c) Fracture of thyroid cartilage
a) Can cause death d) Dribbling of saliva
b) Does not alter the process of drowning 298. Extensive bruising of neck muscles or ecchymosis
c) Can cause delayed death of subcutaneous tissues in neck is characteristic
d) Can cause secondary drowning of- (Delhi 96)
286. Entire absence of fat throughout the body is a) Hanging b) Strangulation
suggestive of- (PGI 90) c) Throttling d) Burking
a) Starvation b) Tuberculosis 299. One of the following is a defmite sign of death
c) Diabetes d) All ofthe above due to drowning- (Karn. 9 5)
287. Commonesttypeofdrowningis- (JIPMER80, a) Sand and mud in the mouth and nostrils
a) Suicidal b) Accidental AIIMS 84) b) Fine lathery froth in the mouth and nostrils
c) Homicidal d) All of the above c) Washerman's hands and feet
288. In frost bite, skin becomes hard and black in about d) Diatoms in bone marrow
a) 3 days b) 7 days (AIIMS 87) 300. In a case of hanging neck ligature marks are
c) 2 weeks d) 4-6 weeks exampleof- (JIPMER 98)
289. Le facies sympathique is seen in- (JIPMER 79, a) Contussion b) Printed abrasion
a) Traffic accident b) Hanging AMU 87) c) Laceration d) Bruise
c) Strangulation d) Myocardial infarction 301. Gettlers test is positive in- (Rajashthan 97)
290. If both water and food are completely with held a) Hanging b) Poisoning
to a person, death usually occurs in -(JIPMER 80, c) Strangulation d) Drowning
a) 1 to 2 days b) 2 to 4 days Kerala 88) 302. The postmortem finding seen in smothering-
c) 5 to 7 days d) 10 to 12 days a) Abrasion on the inner side of the mouth (AP 98)
291. Suicidal drowning differs from homicidal drowning b) Fracture body of Hyoid
by the following except- (PGI 80, AIIMS 82) c) Thyroid fracture
a) Absence of signs of struggle d) Curved marks on the neck
b) Marks of violence on body absent 303. Gettler's test is done in- (MP 98)
c) Untom clothes a) Hanging b) Strangulation
d) None of the above c) Drowning d) Throtling

280)c 281)a 282)d 283)d 284)c 285)c 286)a 287)b 288)c 289)b 290)d 29l)d 292)a 293)c
294)a 295)d 296)d 297)d 298)b 299)d 300)b 301)d 302)a 303)c
FORENSIC [ 427 ]

304. Cafecoronaryisdueto- (MP 98) 316. In fresh water drowning the death occurs within
a) MI b) Asphyxia 4-5 minutes of submersion due to ventricular
c) Strangulaion d) Drowning fibrillation. Which of the following reasons is
305. Lynching is- (AMC 2K) responsible for this- (AIIMS 04)
a) Practised in North America a) Total asphyxia is produced due to fresh water
b) Practised by white people on Negroes b) Laryngospasm causing vagal inhibition
c) Hanging publically on a tree c) Haemoconcentration of blood caused by the
d) All osmotic pressure effect
306. For diatoms test the best site for taking sample is d) Haemodilution, overloading of heart and
a) Lungs (J &K 2001) haemolysis resulting in release of potassium
b) Bone marrow in ulna 317. Which of the following is not seen in drowning-
c) Bone marrow in femur a) Weeds in stomach and lung (Kerala 04)
d) Muscle b) Miosis
307. Atypical arrest due to drowning is- · (PGI 79, c) Wet heavy lungs
a) Cardiac arrest due to drowning AIIMS 83) d) Frothing from mouth
b) Laryngo spasm due to drowning 318. All are types of strangulation except- (Kerala 04)
c) Drowning in unconcious state a) Bansdola b) Lynching
d) All of the above c) Garotting d) Mugging
308. In acute starvation, feeling of intense hunger 319. When skull and photograph offace are available,
lasts - (PGI 80, AMU 89) identity can be established by- (SGPGI 05)
a) 10 to 20 hours b) 20 to 30 hours a) Dactylography
c) 36 to 48 hours d) Beyond 60 hours b) Poroscopy
309. Number of deciduous teeth is- (JIPMER 95) c) Superimposition technique
a) 20 b)24
d) Gustafson's method
c) 28 d)32
320. Foamy liver is seen in- (SGPGI 05)
310. Most common consistent feature after a child has
a) Arsenic poisoning b) Putrefaction
been taken out from pond died from drowing is-
c) Mummification d) Aconite poisoning
a) Water in lungs alveoli (PGI 97)
321. Study of finger printing is- (PGI June 05)
b) Petechial haemorrhages
a) DNA fmger printing b) Dactylography
c) Washer woman skin
c) Gene analysis d) Poroscopy
d) Cyanosis
e) Cheiloscopy
311. Hanging causes large amount of injury to-
322. Medico-legal importance of rigor mortis is in
a) Vertebral artery b) Carotid A (PGI 98)
estimation of- (MAHE 05)
c) Trachea d) Oesphagus
312. Diatom test is to determine death due to- (PGI 98) a) Time of death b) Manner of death
a) Drowing b) Strangulation c) Cause of death d) AU the above
c) Hanging d) Burns 323. Female pelvis can be differentiated from male
313. All are seen in drowing except- · (PGI 99) pelvis by- (APPGE 05)
a) Miosis a) Larger obturator foramen
b) Wet heavy lungs b) Obtuse angle of suprapubis
c) Water and weeds in stomach and lung c) Suprapubic sulcus is prominent
d) Froth from mouth · d) All
314. Features supporting strangulation are- (PGI 01) 324. The minimum age at which an individual is
a) Blood outside vessels in microscopy from skin responsible for his criminal act is- (AI 05)
ofligature site a) 7 years b) 12 years
b) Dribbling of saliva from angle of mouth c) 16years d) 21 years
c) Marked cyanosis of face 325. The most reliable method of identification of an
d) Frothy sputum in mouth and nostrils individual is- (AI 05)
e) Ligature mark a) Dactylography b) Scars
315. A dead body is found, on pressure over the chest c) Anthropometry d) Handwriting
fine leathery froth came out through nostrils and 326. The most common pattern of fingerprint is-
mouth, the mode of death was- (PGI 03) a) Arch b)Loop (A/05)
a) Hanging c) Whorl d) Composite
b) Organophosphorus poisoning 327. The most reliable criteria in Gustafson's method
c) Drowing ofidentification is- (AI 05)
d) Strangulation a) Cementum apposition b) Transparency of root
e) Morphine poisoning c) Attrition d) Root resorption

304)None 305)d 306)c 307)d 308)c 309)a 310)a 311)b 312)a 313)a 314) a,c,e 315) c 316)d 317) b
318)b 319)c 320)b 32l)b 322)a 323)b 324)a 325)a 326)b 327)b
FORENSIC [ 428]

328. Spalding's sign occurs after- (AI 06) 342. In Riger mortis, first appearance occurs at-(UP 08)
a) Birth oflive fetus a) Neck b)Eyelid
b) Death of fetus in uterus c) Face d) Trunk
c) Rigor mortis of infant 343. Which of the following is not a cause of death due to
d) Cadaveric spasm suffocation- (AI 08)
329. Fingerprint Bureau was ftrst established in- a) Choking b) Gagging
a) England b) China (AI06) c) Smothering d) Throttling
c) India d) Singapore 344. Fracture of the hyoid bone and thyroid cartilage with
330. When a surgeon wants to send the autopsy specimen neck muscle bruising suggests a diagnosis of-
for virological examination, it should be preserved a) Strangulation b)Hanging (AI08)
in- (AIIMS 06) c) Choking d) Gagging
a) 50% glycerine 345. Fracture of hyoid bone, thyroid cartilage and cricoid
b) 10% formalin cartilage along with extensive bruising of neck
c) Rectified spirit muscles suggests a diagnosis of- (AI 08)
d) Saturated solution of common salt a) Manual strangulation
331. The fingerprint pattern may he impaired permantly b) Ligature strangulation
in cases of- (AIIMS 06) c) Hanging
a) Eczema b) Scalds d) Choking
c) Scabies d) Leprosy 346. SweatingisNotpresentin- (AiimsMay08)
332. Most reliable criteria in Gustafson method of a) Heat syncope b)Heatcramps
age estimation is- (AIIMS 06) c) Heat stroke d) Heat fatigue
a) Attrition 347. Corpus delicti means- (Aiims May 08)
b) Secondary dentin deposition a) Essence of crime
c) Transparency of root b) Inquest into death
d) Cementum apposition c) Postmortem examination
333. Minimum age for punishment in Railways act? d) Death by asphyxia
a) 2 years b) 5 years (APPG 06)
348. Last organ to be dissected during autopsy in
c) 7 years d) 12 years
asphyxial death is- (Aiims May 08)
334. The rate oftemperature falls after death is -(UP 07)
a) Neck b) Head
a) 0.2-0.3°Cihour b)0.5-0.7°Cihour
c) Abdomen d) Thorax
c) 2- 3°C/hour d) 5- 7°C/hour
349. Dark brown P.M. lividity is seen in- (PGI June 08)
335. The medicolegal importance of postmortem
lividity is all except~ (UP 07)
a) Aniline b) CO
a) Cause of death elsewhere c) Phosphorous d) Hydrocyanide
b) Time since death 350. The rate of temperature fall after death •(DPGEE 08)
c) Manner of death a) 2-3°C/hour b)0.5-0.7°C/hour
d) Position of the body at the death c) 0.2-0.3°C/hour d)5-7°C/hour
336. Delayed rigor mortis occurs with- (UP 07) 351. What is aseptic autolysis called? (APPG 08)
a) Vegetables poisons b) Strychnine a) Decomposition b) Adipocere
c) Opium d) Septicemia c) Mummification d) None
337. Last organ to be dissected during autopsy in 352. Heat Rupture is characterized by- (AI 09)
asphyxial death is- (AIIMS Nov 07) a) Regular margins
a) Neck b) Head b) Irregular margins
c) Abdomen d) Thorax c) Ruptured blood vessels
338. In autopsy, preservative used for vitreous is- d) Ruptured red blood cells
a) Fluoride b)HCl (AIIMSNov07) 353. Incisional wound on genital is seen commonly
c) Phenol d) Xylol in- (AIIMSMay09)
339. Postmortem rigidity first starts in- (Comed 08) a) Accidents b) Suicides
a) Eyelids b) Neck c) Homicides d) Postmortem artifact
c) Upper limbs d) Lower limbs 354. Tentative cuts are seen in- (PGI June 09)
340. In normal conditions of temperature and a) Homicidal wound
atmosphere, tbe rate of cooling of dead body is- b) Suidical wound
a) 1.0°F/hour b) 1.5°F/hour (Comed 08) c) Rail Track accidental wound
c) 2.0°Fihour d) 2.5°Fihour d) Road traffic wound
341. Foamy liver is found in- (UP 08) 355. After death, tache noir is seen in -
a) Drowning b) Putrefaction a) Lungs b) Brain (DELHI PG Feb. 09)
c) Adipocere d) Abortion c) Heart d) Eyes

328)b 329)c 330)b 33l)d 332)c 333)b 334)b 335)c 336)d 337)a 338)a 339)a 340)b 34l)b
342)b 343)d 344)a 345)a 346)c 347)a 348)a 349)c 350)b 35l)a 352)b 353)c 354)b 355)d
FORENSIC [ 429 ]

356. Paltauf's hemorrhage is seen in-(DELmPG Feb. 09) 368. When a doctor shows gross absence of skill and
a) Wet drowning b) Dry drowning care during treatment resulting in serious injury
c) Electrocution d) Suicidal hanging or death ofthe patient, it is called- (Jipmer 11)
357. Heat rupture is characterised by- (AIIMSMay 10) a) Criminal negligence b) Malpractice
a) lrregularmargin b) Clotted blood c) Maloccurence d) Misadventure
c) Regular margin d) Clotted blood vessels
358. Duriug putrefaction, fixation of marbling occurs SEC -IV: MECHANICAL INJURIES,
at- (Maharashtra 10) FIREARMS AND FIREARM INJURIES,
a) 48-72 hours b) 72-96 hours
REGIONAL INJURIES. THERMAL INJURIES
c) 36-48 hours d) 12-24hours
359. Cephalic index is used to detect- (Maharashtra 10) 369. Grevious injury includes which of the following-
a) Age b)Sex a) Incised wound of scalp (PGI 87)
c) Race d) Any of the above b) Lacerated wound of scalp
360. Le Facies sympathique is suggestive of- c) Fracture radius
a) Antemortem hanging (Maharashtra 10)
d) Injury which kept patient away from week for
b) Suicidal hanging lOdays
c) Homicidal hanging
e) Abdominal trauma
d) Postmortem hanging
370. Arborescent marking are seen in bums due to-
361. Which ofthefoDowing is bestfor testing development
a) Electricity b) Phosphorus (PGI 87)
of rigor mortis? (Maharashtra 1 0)
c) Lightening d) UV rays
a) Opening the lower jaw
b) Flexing elbow joint 371. Puglistic attitude in burns is seen in-
c) Flexing knee joint a) Antemortem bums (AIIMS 87, PGI 86)
d) Falling the leg from height b) Post mortem bums
362. Maximum congestion is seen in- (DPG 10) c) Homicidal bums
a) Choking b) Hanging d) Suicidal bums
c) Strangulation d) Drowning e) Both antemortem burns and post mortem bums
363. In freshwater drowning, the death occurs with 4-5 372. Bevelling of the skull is seen in the- (AI 88)
minutes ofsubmersion due to ventricular fibriDation. a) Broad end of the entry point in bullet injury
Which of the following reasons is responsible for b) Narrow end of the entry point in bullet injury
this? (DPG 10) c) Exit point of bullet
a) Total asphyxia produced due to fresh water d) Depressed fracture of the skull
b) Laryngospasm causing vagal inhibition 373. Pugilistic attitude is seen in- (AI 89)
c) Hemoconcentration of blood caused by the a) Bums b) Electrocution
osmotic pressure effect c) Drowning d) Strangulation
d) Haemodilution, overloading of heart and 374. Greasecolorisseenwith ....... Wounds- (A/89)
haemolysis resulting in release of potassium a) Fire arm entry b) Fire arm exit
364. All are components of black gum powder bUster c) Road traffic accident d) Hanging
except- (DPG 10) 375. Joule bum is seen with - (AI 89, TN 90)
a) Sulphur b) Charcoal a) Electrocution b) Scalds
c) Potassium nitrate d) Lead peroxide c) Lighting d) Vitriolage
365. In case of death due to starvation, at autopsy the 376. Which ofthe following constitute grevious injury-
condition of the gall bladder will be- (Corned 10) a) Incised wound of scalp (AI 89, TN 90)
a) Distended with inspissated bile b) Incised wound ofthigh
b) Empty c) Fracture Tibia
c) Filled with mucus d) Laceration of the scalp
d) Filled with purulent fluid 377. Grease collar is seen with - (AIIMS 84)
366. Spanish windlass was practices in Spain as a method a) Electrocution b) Lighting
ofexecution.Itisatypeof- (AIIMS Nov 10) c) Thermal injury d) Bullet entry
a) Bansdola b) Mugging 378. In a lacerted wound the hair bulb is- (AIIMS 84)
c) Garrotting d) Hanging a) Cut b) Crushed
367. 'La facies sympathique' is a condition seen in cases c) Both of the above d) Lacerated
of- (PUNJAB 11) 379. Pugilistic attitude is seen in- (PGI 88)
a) Hanging b) Strangulation a) Antemortem bums b) Post mortem burns
c) Myocardial insufficiency d) Railway accidents c) Drowning d)All

356)a 357)d 358)c 359)c 360)a 36l)a 362)c 363)d 364)d 365)a 366)c 367)a 368)a 369)c
370)c 37l)e 372)c 373)a 374)a 375)a 376)c 377)d 378)b 379)a,b
FORENSIC [ 430]

380. Privation of any member of jointis a •••••..... hurt- 394. On tenth day of a tooth being knocked out, the local
a) Simple b) Grevious (PGI 85) clinical finding will be- (AIIMS 81, Kerala 89)
c) Serious d) Dangerous a) Tooth socket being filled up by tissue
381. More than 5°/o carboxyhemoglobin is indicative of- b) Blood clots in the socket
a) Ante mortem burns b) Drowning (PGI 89) c) Alvoelar process smooth
c) HCN poisioning d) Suffocation d) Fluid blood in the socket
382. Greenish colour in a contussion is due to- 395. The effective range of shot gun is- (AIIMS 79,
a) Hemosiderin b) Haematoidin (JIPMER 90)
a) IOto 15 yards b)20to30yards AMU87)
c) Bilirubin d) Biliverdin
c) 30 to 40 yards d) 60 to 80 yards
383. In a completely charred body at autopsy, medical officer
can say that the burns are antemortem if he can find- 396. False vesication differs from true vesication except-
a) Soot in respiratory passage (JIPMER 90) a) Produced after death (DNB 89)
b) Cyanosis of fmger nails b) Vesicles contain gas and no fluid
c) Congestion of kidney c) No inflammatory or respiratory zone present
d) Pale internal organs d) None of the above
384. Tentative cuts occur in which injury- (AI 90) 397. Bullet is picked up with- (AIIMS 81, ROHTAK 86)
a) Accidental b) Suicidal a) Toothed forceps b) Hands
c) Homicidal d) Fabricated c) Scissors d) Needle
385. Choking is constricting device within the barrel 398. Compressed air is used to fire lead slugs
atthemuzzleendof- (TN 89) in- (PGI81,AMU87)
a) Revolver b) 303 rifle a) Automatic pistol b) Revolver
c) Shot gun d) Semiautomatic pistol c) Air rifle d) Shot gun
386. Which is true about stab wound- (AI 91) 399. Detonator cap is situated at- (AIIMS 80, PGI 85)
a) Breath is maximum a) Top of the bullet b) Side of the bullet case
b) Length is maximum c) Base of the rim d) None of them
c) Depthismaximum 400. Section 302 oflndian penal Code is for-
d) It has wound of entry and exit a) Rape (AIIMS 80, AMC 83)
387. The commonest cause for sudden death is - (AI 91) b) Murder
a) Acute myocarditis b) Pulmonary edema c) Grievous hurt
c) Myocardial infarction d) CCF
d) Attempt to commit suicide
388. Dermal Nitrate test detects- (AIIMS 91)
401. Pellet holes in a firearm wound is seen if distance
a) Nitrite Salt Poisoning b) Blood stains
is- (JIPMER 81, AIIMS 84)
c) Nerve gas poisoning d) Gun powderresidue
a) 60cm b) 100cm
389. Which is a Cognisable offence except- (Al93)
c) 150cm d)200cm
a) Robbery b) Murder
402. Incised-like lacerated wound appears at- (PGI 83,
c) Rape d) Attempted suicide
a) Forehead b) Hand DELHI 87)
390. Postmortem wound best differs from the antemortem
c) Thorax d) Abdomen
wound by- (Delhi 87, 93)
a) Gaps on incising 403. Filigree burns are due to- (AIIMS 79, PGI 81)
b) No clots a) Chemicals b) Lightening
c) Absence of erythema and cellular changes c) Electric shock d) Radiations burns
d) Stain can be removed by washing 404. The proliferation of vascular endothelium is seen
391. Which of the following indicates that the wound in a within- (AIIMS81,AMU86)
hospitalized woman typist hit by lathis admitted for a) 6 hours b) 12 hours
20 days was grevious- (AIIMS 81, Delhi 93) c) 24 hours d) 7 days
a) She is not able to be recognized 405. Smokeless gun powder is composed of- (AIIMS 80,
b) Wound does not heal completely a) K.Mn04 b) HCN 81)
c) Incised wound over scalp c) Nitrocellulose d) Sulphur
d) She is not able to do her daily routines 406. Provisional callus forms in fracture of bone
392. Following are the causes of death in a patient with by- (JIPMER81,AMU86)
burns except- (AIIMS78, ORISSA 91) a) 3 days b) 7 days
a) Shock b) Suffocation c) 14 days d) 30 days
c) Sepsis d)Airembolism 407. 'Pond Fracture' is seen commonly in- (TN 87)
393. Tearing of the skin and subcutaneous tissue is a) Persons who are over 50 year of age
called - (AP 90) b) Young adults
a) Abrasion b) Laceration c) Children
c) Contussion d) Avulsion d) Women

380)b 38l)None 382)b 383)a 384)b 385)c 386) c 387) c 388) d 389) d 390) c,d 391) d 392) d 393) b
394)a 395) c 396) d 397) b 398) c 399) c 400)b 40l)d 402)a,b 403)b 404)d 405)c 406)b 407)c
FORENSIC [ 431 ]

408. Full dispersion is seen in- (PGI 79, AIIMS 81) 422. Joule burn is seen in- (UP 97)
a) Full choked b) Half choked a) Electrocution b) Thermal Injury
c) Unchoked d) All of the above c) Radiation Injury d) Lightning
409. Callus becomes thick and hard like bone 423. In deep incised wounds, Langer's lines determine-
within- (JIPMER 80, AMU 86) a) Direction b) Gaping (MAHE 98)
a) 2 weeks b) 4 weeks c) Shelving d) Healing
c) 6 weeks d) 3 months 424. Damage produced by a bullet fired from a _ __
410. "Fracture a-la signature' is- (TN 89) person in direct proportion- (Orissa 99)
a) A depressed fracture a) Size b) Shape
b) A fissured fracture c) Velocity d) Weight
c) Separation of the suture lines 425. Which is associated with shotgun along-
d) A ring fracture a) Gunpowder b) Primer (Orissa 98)
411. The killing range of a military rifle is about- c) Projectile d) Wads
a) lOOyards b)500yards (AP89) 426. For dowry death punishment is- (Orissa R)
c) 1000 yards d) 300 yards a) 3 year imprisonment+ Rs. 10,000/-
412. Open wound is a- (Kerala 94) b) 5 year imprisonment+ Rs. 15,000/-
a) Contussion b) Abrasion c) 7 year imprisonment+ Rs. 20,000/-
c) Laceration d) Concussion d) 2 year imprisonment+ Rs. 15,000/-
413. "Triage" is associated with- (Jipmer 95) 427. Grevious hurt is punishable under section _ __
a) Warfare injuries b) Road injuries IPC- (Orissa R)
c) Gun shot injuries d) Poisoning resuscitation a) 320 b)354
414. Brush injury is due to- (All India 95) c) 321 d)402
a) Friction b) Electric shock 428. Which of the following sections ofiPC are concerned
c) Scalds d) Bullet injury with dowry death - (Kerala 2K)
415. One of the following is au example of a grevious a) 300 b) 302 .
injury- <Delhi 96) c) 304 d)304-A
a) Scalp laceration e) 304-B
b) Forearm laceration requiring stitches 4 29. Grevious injury includes all the following except-
c) Fracture of radius a) Emasculation (Kerala 2K)
d) Groin injury with loss of 1 litre of blood b) Permanent privation of hearing
416. The most common type of hemorrhage in injury c) Privation of any member/joint
to boxers is- (AIIMS 95) d) Disability to follow one's ordinary pursuits for a
a) Extradural b) Subdural period of a week
c) Intracerebral d) Intraventricular e) Permanent disfigeration of face
417. Crocodile skin appearance is seen in- (Kerala 96) 430. Dermal Nitrate test is done for the detection
a) High voltage burns b) Electric Shock of- (J&K2001)
c) Drowning d) Mununifications a) Gunpowder residue b) Seminal stains
418. Which is not true regarding colour of bruise- c) Blood stains d) Saliva
a) 4th day- bluish black to brown (CUPGEE 96) 431. Gutter fracture is due to- (KERALA 2001)
b) 7-12 days- green a) A sharp weapon b) Fire arm injury
c) l''day-Red c) Blunt weapon d) Serrated edge
d) 2 Weeks- normal 432. Detection of metals (heavy) is done by using all
419. Filigree burns are due to- (AP 96) tests except- (AI 2002)
a) Splashes b) Flame burns a) Harrison Gilory test
c) AC Current d) Lightening b) Spectroscopic examination
420. All are components of black gun powder c) Neutron activation test
except- (AIIMS 97) d) Paraffm test
a) Sulphur b) Charcoal 433. Black gun powder ; ingredient not present is -
c) Potassium nitrate d) Lead peroxide a) Lead peroxide b) Arsenic (PGI 02)
4 21. Ante mortem blister differs from postmortem blister c) Charcoal d) Silver nitrate
by- (PG/96) e) Sulphur
a) Presence of Albumin & Chloride in blister fluid 434. Where does a lacerated wound appear like a incised
b) Gas in blister wound- (AP 02)
c) Dry hard surface of the floor ofblister (punctured) a) Abdomen b) Thorax
d) Absence of hyperemia around the blister c) Hand d) Forehead

408)c 409)d 410)a 411)b 412)c 413)a 414)a 415)c 416)b 417)a . 418) b 419)d 420)d 42l)a
422)a 423)b 424) c,d 425) d 426)c 427)a 428)e 429)d 430)a 431)b 432)d 433) a,b,d 434) c,d
FORENSIC [ 432 ]

435. Gutter fracture result from* (AP 02) 448. Grevious injuries are- (PGI 2000)
a) Injury with a large round object a) Severe bodily pain for 10 days
b) Bullet injuries b) Not able to work for 17 days
c) Automobile accident c) Dislocation of tooth
d) Falling from a height d) Joint dislocation
436. Grevious injury includes- (PGI 02) e) Bone fracture
a) Emasculation 449. Difference between antemortem burns &
b) Loss of 15 days work postmortem burns & vesicles, favouring AM burn
c) Permanent disfigurement vesicles is/are- (PGI 2000)
d) Fracture of bones
a) Tissue beneath vesicle is dry, hard & yellow
437. Mercury fulminate is used in fire arm
b) Protein & chloriedes in vesicle are more
ammunition as- (JIPMER 89, AMC 02)
c) Erythema is present
a) Detonator b) Smokeless powder
d) Fluid in the vesicle is scanty
c) Black powder d) Mixed powder
438. Pugilistic attitude is due to - (PGI 02) 450. Difference between incised wound & heat rupture,
a) Burns favouringheatruptureare- (PGI 2000)
b) Vitriolage a) Margins well defined
c) Flexor group contracture b) Small & multiple wounds
d) Extensor group contracture c) Nerves & vessels around are visible in floor
439. Markings in a projectile occurs in- (A.L 02) d) Seen only over scalp
a) Double barrel gun b) Pistol 451. Use of wadding in a smooth bore gun rifle produces
c) Rifle d) Shot gun all except- (PGI 2000)
440. In blast injury most common organ affected- a) Causes fatal injuries b) Helps in lubrication
a) Eardrum b) Stomach (CUPGE 97) c) Optimum pressure d) Sealing the air
c) Lungs d)Alloftheabove 452. Sites notorious for incised looking wound are all
441. The extent of muscle damage by a bullet depends except- (PGI 2000)
primary on the- (UPSC 97) a) Chest b) Zygoma
a) Size b) Velocity c) Iliac crest d) Shin
c) Shape d) Weight 453. Depressed# of skull results from- (PGI 2000)
442. Lacerated wound looks like an incised wound in the- a) Heavy object with small striking surface
a) Scrotum b) Scalp (PGI 88) b) Heavy object with large striking surface
c) Abdomen d) Thigh c) Fall over the ground
443. Burn (beat) haematoma is seen between -(PGI 97) d) Light object
a) Skull and dura b) Brain and arachnoid 454. Lacerated wound appears as incised wound in
c) Arachnoid and Pia d) Pia and dura
which of the following sites- (PGI 0 1)
444. Pugilistic attitude is most common in woman dying
a) Scalp b) Tibial shin
with- (PGI 97)
c) Anterior abdominal wall d) Anterior thigh
a) Antemortem burns
b) Post mortem burns e) Around the hip
c) Asphyxia 455. Difference between antemortem wounds and
d) Carbon monoxide poisoning postmortem wounds, in favour ofAM wounds is/are-
445. Injury that comes under sec 320 IPC- (PGI 97) a) Are larger (PGI 01)
a) Abrasion over face b) Increased gaping of wounds
b) Nasal bone fracture c) Increased enzyme activity
c) Epistaxis d) Oozing of blood
d) Lacerated wound over scalp 456. Black gunpowder contains- (PGI 02)
446. Antemortem bruise is differentiated from a) Charcoal b)KN03
porstmortem bruise by- (PGI 99) c) Sulfur d) Lead peroxide
a) Well defined margin e) Arsenic
b) Capillary rupture with extravasation blood and 457. Which of the following are grevious injury-
ligature mark a) Emasculation (PGI 03)
c) Yellow colour b) Privation of any member ofjoint
d) Gaping c) Permanent disfiguration of head or face
447. Pugilistic attitude is due to- (PGI 99) d) Injury that heals spontaneously without any damage
a) Coagulation of proteins b) Cadaveric spasm e) Injury causing an absence from work for 15 days
c) Rigor mortis d) Coagulation offats

435) b 436) a,c,d 437) a 438) a 439) b,c 440) d 441) b,d 442) b 443) a 444) a,b 445) b 446) b,d 447) a 448) c,d,e
449) b,c 450) b,e 451) a 452) a 453) a 454) a,b 455) b,c 456) a,b,e 457) a,b,c
FORENSIC [ 433]

458. In antemortem tooth loss or extraction of the 470. Which of the following statements is not correct
alveolus is- (Karnataka 02) regarding diatom- (AI 05)
a) Smooth a) Diatoms are aquatic unicellular plant
b) Sharp and feathered b) Diatoms has an extracellular coat composed of
c) Does not show any injury magnesium
d) sMay have a regular or an irregular appearance c) Acid diagestion technique is used to extract
459. I.P.C section for grevious injury- (Jipmer 03) diatoms
a) Section 420 b) Section 320 d) Presence of diatoms in the femoral bone marrow
c) Section299 d) Section351 is an indication of antemarten inhalation of water.
460. Stellate wound is produced with firearm in- 471. Postmortem caloricity may be seen in all the
a) Contact shot b) Close shot (TN 03)
following causes of death except- (AIIMS May 05)
c) Range within 60 ems d) Distant shot
a) Septicemia
461. In which of the following weapons empty
b) Barbiturates poisoning
cartridge case is ejected after firing- (AIIMS 04)
a) Short gun b) Revolver c) Strychnine poisoning
c) Pistol d) Rifle d) Tetanus
462. A 25 year old person sustained injury in right eye. 472. The ideal place to record temperature in dead
He developed right corneal opacity following the bodyisfrom- (AIIMSMay05)
injury. Left eye was already having poor vision. a) Axilla b)Groin
Corneoplasty of right eye was done and vision was c) Rectum d) Mouth
restored. Medicolegally such injury is labeled 473. When a group of muscles of a dead body were in
as- (Al04) state of strong contraction immediately prior to
a) Grievous b) Simple death and remain so even after death, the condition
c) Dangerous d) Serious is termed as- (AIIMS May 05)
463. True regarding electrocution injuries is all a) Gas stiffening b) Rigor mortis
except- (SGPG/04) c) Cadaveric spasm d) Cold stiffening
a) Direct current produces greater damage as 474. The ideal place to record body temperature in
compared to alternating current dead body is- (Al/MS 06)
b) The injuries are greater when associated with a) Rectum b)Axilla
moist contact c) Mouth d)Groin
c) The injuries are of higher intensity if the body is 475. A dead boy having cadaveric lividity of bluish
partially not earthed green colour.The most likely cause of death is by
d) High voltages throw the victms clear poisioning due to- (AIIMS 06)
464. In Northern India postmortem staining seen a) Hydrocyanic acid b) Hydrogen sulphide
in- (PGI June 05)
c) Oleander d) Sodium nitrite
a) 6 hr after death b) 12 hr after death
476. Paradoxical undressing is seen in - (Manipal 06)
c) 24 hr after death d) 18 hr after death
"a) Exhibitionism b) Fetichism
c) 26 hr after death
465. Goose skin or cutis anserina is seen in -(PG!june 05) c) Hypothermia d) Carbon monoxide poisoning
a) Drowning b) Lightening 4 77. KEYHOLE bullet entry is a feature of- (MAHE 07)
c) Strangulation d) Fire arm injury a) Tandem Bullet b) Ricochet Bullet
466. Ideal tissue for diatom study- (PGI June 05) c) Single Bullet d) Double Bullet
a) Bone marrow of femur b) Lung 478. Most common cause of death in Electrocution is-
c) Liver d) Brain a) Respiratory muscle paralysis (UP 07)
e) Kidney b) Ventricular fibrillation
467. Atypicaldrowningisdueto- (MAHE 05) c) Asphyxia
a) Laryngospasm b) Vagal inhibition d) Regurgitation of fluids
c) Water inhalation d) Electrolyte disturbance 479. 'Molotov cocktail' is- (UP 07)
468. A person is presumed dead as per law if not seen a) Time bound bomb
or heard for by his/her near and dear ones for- b) Mixture of alcohol
a) 5 years b) 7 years (Orissa 05) c) Mixture of drug composition
c) lOyears d) 15 years d) Incendiary bomb
469. 'La facies sympathique' is a condition seen in 438. Fg, FFg FFFg used to describe- (PGI June 07)
cases of- (AIIMS NOV 05) a) Primer b) Cartridge
a) Hanging b) Strangulation c) Black gunpowder d) Wad
c) Myocardial insufficiency d) Railway accidents

458)b 459)b 460)a 461)c 462)a 463)a 464)a 465)a 466)a 467)b 468)b 469)a 470)b 471)b
472)c 473)c 474)a 475)b 476)c 477)b 478)b 479)d 480)c
FORENSIC [ 434]

481. With Reference to an incised wound all the following 493. Hydrocutiorr.U~""· (AIIMS Nov 08)
are true except- (Corned 07) a) Electrocution~ water
a) It has clean cut margins b) Dry drowning '>,
b) Bleeding is generally less than in laceration c) Drowning in cold water
c) Tailing is often present d) Postmortem immersion
d) Length of injury does not correspond with the 494. What is Falanga? (AIIMS Nov 08)
length of the blade a) Beating on soles with blunt object
482. Fracture dislocation of 2nd and 3rd cervical b) Beating on ear with both palms
vertebrae is common in- (Corned 07) c) Beating on the abdomen
a) Partial hanging b) Lynching d) Suspension by wrists
c) Judicial hanging d) Throttling 495. FG,FFG,FFFG••used todiscribe- (PGI Dec 08)
483. Which of these is not a grevious injury? a) Primer b) Cartridge
a) Loss of teeth (AIIMS Nov 07) c) Blackgunpowder d) Wadding
b) Loss of hearing of one ear e) Baseoffrrearms
c) Permanent disfiguration of face. 496. Antemortem Abrasions can be confused With-
d) Abrasion on face. a) Eczema b)Antbitemarks (AI 09)
484. Telefono is... (AIIMS Nov 07) c) Chemical burn d) Joule burn
a) Beating the soles of feet with rod 497. Extensive abrasions are found all over the body of a
b) Beating of the ears pedestrian lying by the road side. What is the likely
c) Forced immersion ofhead until stage of suffocation cause? (AI 09)
d) Tying of a plastic bag over head. a) Primary impact injury
485. Most common site of primary impact injury to b) Secondary impact injury
pedestrians in a motor vehicle accident is••• c) Secondary injury
a) Abdoemn b) Legs (AIIMS Nov 07) d) Postmortem artifact
c) Head d) Chest 498. Stellate wound may be seen in which of the following
486. In tandem bullet, number of bullet/bullets coming bulletentrywounds- (AI 09)
out ofthe gun is/are- (AIIMS Nov 07) a) Contact wound b) Close shot
a) 1 b) 2 c) Distance shot d) Two feet distance
c) 3 d)4 499. Commonest organ to be injured in primary blast
487. In a 6year old child with burns involving the whole injuries is - (AI 09)
of head and trunk, the estimated body surface area a) Lung b)Liver
ofburnsis- (Corned08) c) Spleen d) Skin
a) 44% b) 52% 500. Which of the following skull fractures is called a
c) 55% d)58% 'Motorcyclist's fracture- (AI 09)
488. Vipers snake bites resembles- (UP 08) a) Ring fracture ofbase of skull
a) Sulfuric acid b) Organophosphorus b) Hinge fracture of base of skull
c) Thallium d) Abrus precatorius c) Comminuted fracture of skull
489. Gun powder consists of all except- (UP 08) d) Depressed fracture of skull
a) Sulphur b) Charcoal 501. All ofthe following may cause Traumatic Asphyxia,
c) Potassium nitrate d) Lead nitrate Except- (AI 09)
490. Difference between post burn rupture & incised a) Railway accident
wound- (PGI June 08) b) Road traffic accident
a) Seen in front of the thigb c) Accidental strangulation
b) Intact blood vessels & nerve at floor of the wound d) Stampede in crowd
c) Bleeding from the wound 502. In which part of body a lesser impact causes
d) Small and multiple maximum bruise? (AIIMS May 09)
491. Hingee fracture of skull is seen in accidents a) Face b) Sole
involving- (Manipal 08) c) Back zd)Palm
a) Car driver b) Car passenger 503. Antemortem and postmortem wounds could be
c) Pedestrian d) Motorcycler driger differentiated by all, except- (AIIMS May 09)
492. A bullet which is packed with jacket and opens at a) Everted margins
base is called - (DPGEE 08) b) Blood clots in surrounding tissue
a) Tandem bullet b) Duplex bullet c) Swollen edges
c) Dum Dum bullet d) Souneir bullet d) Sharp edges

481)b 482)c 483)d 484)b 485)b 486)b 487)b 488)d 489)d 490) a,b,d 491) d 492) c 493) c 494) a
495)c 496)b 497)c 498)a 499)a 500)b 501)c 502)a 503)d
FORENSIC [ 435]

504. Destructive power of bullet is determined 516. Arborescent marks or filigree burns are seen
by- (AJIMSMay 09) in- (Maharashtra 1 0)
a) Weight of bullet b) Shape ofbullet a} Radiation b) Electrical burns
c) Size ofbullet d) Velocity of bullet c) Chemical burns d) Lightening
505. The most affected organ or system in a blast under 517. Bevelling of the skuU is seen in the- (DPG 10)
water is- (DEUfi PG Feb. 09) a) Broad end of the entry point in bullet injury
a) Lungs b) Intestine b) Narrow end of the entry point in bullet injury
c) Skin d) Heart c) Exit point ofbullet
506. In countrecoup injuries- (DELHI PG Mar. 09) d) Depressed fracture of the skull
a) Injury is on same side 518. A bullet fired from a gun is not released. It is ejected
b) Injury is on opposite side out with the subsequent shot. It is known
c) Injury is all over the brain as- (DPG 10)
d) None ofthe above a) Dumdum bullet b) Rocketing bullet
507. Blackening found at entry wound in firearm injuries c) Richochet bullet d) Tandem bullet
is due to- (DELHI PG Mar. 09) 519. Rule of Hasse is used for calculating- (Punjab 10)
a) Burnt gun powder b) Smoke a) Time since death b) Age ofFoetus
c) Flame d) Friction and dirt c) Fluid requirement d) Burn area
508. All belong to category of grievous hurt, 520. AU are components of black gun powder except-
except- (DELHI PG Mar. 09) a) Sulphur (Punjab 10)
a) Superficial firearm wound over thigh b) Charcoal
b) Stab injury of abdominal cavity c) Potassium nitrate
c) Hairline fracture of flrst metatarsal d) Nitrocellulose
d) Acid burn over face 521. Incised looking lacerated wounds are seen in -
509. All are suggestive of traumatic bleed, except-
a) Hand b)Thorax (Jipmer 10)
a) Extradural haemorrhage (DELHI PG Mar. 09)
c) Abdomen d) Forehead
b) Subdural haemorrhage
522. Differentiating features of contusion (Vs
c) Subarachnoid haemorrhage
postmortem staining) are- (PGI May 10)
d) Contusion
a) Blusih is color
510. Antemortem bum differ from postmortem bums by
b) Disappear on pressure area
all except- (AIIMS Nov 09)
c) Margin irregular
a) Pus in vesicle
d) Limited to intravascular compartment
b) Vesicle with hyperemic base
e) Extravasation of blood occurs
c) Vesicle containing air
d) lnfarnmatory red line 523. True about pugilistic attitude- (PGI May 10)
511. Motorcyclist fracture among the following is- a) Indicate only antemortem burn
a) Ring fracture (AIIMSMay IO) b) Indicate only postmorten burn
b) Sutural seperation c) Can not differentiate blw ante & post mortem
c) Comminuted fracture of vault of skull. burn
d) Fracture base of skull into anterior & posterior halves d) Occur d/t intense heat
512. Destructive power of bullet is determined by- e) Indicate defence by victim
a) Weight of bullet (Al/MS May 10) 524. False about bruise- (PGI Nov. 1 0)
b) Shape of bullet a) Much darker at point of contact I pressure
c) Size of bullet b) Blood is present in vessel and can be easily
d) Velocity of bullet washed away
513. Primary impact injuries are commonly seen on- c) Seen at I around the area of injury
a) Chest b) Abdomen (AI 10) d) Extravasation of blood occurs
c) Legs d) Head e) Produced by blunt object
514. Which of the following bullets leaves a visible mark, 525. A rickshaw tyre passed on a 8 year old child. Tyre
so that a person can see it- (AI 10) marks were found on the body. This is an exmaple
a) Tandem bullet b) Tracer bullet of- (AIIMS Nov 10)
c) DumDumbullet d)Incendiarybullet a) Percolated bruise b) Imprint abrasion
515. Lightening flash can cause injury by all of the c) Contusion d) Patterned bruise
following, Except- (AI 1 0) 526. 'Sparrow's foot' mark is seen in- (AI 11)
a) Direct effect of electric current a) Gunshot Injuries
b) Super heated air b) Stab injuryofface
c) Expanded and repelled air c) Windshield Glass injury
d) Compressed air pushed in front of the current d) Vitriolage

504)d 505)b 506)b 507)b 508)a 509)c 510)c 51l)d 512)d 513)c 514)b 515)d 516)d 517)c
518)d 519)b 520)d 52l)d 522) a,c,e 523) c,d,e 524) a,b 525) b 526) c
FORENSIC [ 436]

527. Gunpowder and/or Soot on bloodstained garments 538. Spaldings sign is diagnostic of- (Kerala 91)
can be visualized by- (AI 11) a) Prematurity b) Post maturity
a) Magnifying lens b) UV Rays c) Fetal death d) Hydrocephalus
c) Infrared Rays d) X rays 539. Of the following which does not categorises under
528. A Rikshawtire passed over the body of an 8 year old Rape. Sexual intercourse with- (PGI 93)
child leaving distinct markings of the tire treads. a) Wife below 15 yrs
Marking's ofthe Rikshaw Tire over the child's body b) Wife above 20 years
are best defined as - (AI 11) c) Woman below 16 years
a) Contact Abrasions b) Percolated Abrasions d) Woman above 20 years
c) Imprint Abrasions d) Patterned Abrasions
540. Which of the following sign will indicate that
529. 'Fracture-ala signantre' or signature fracture is-
menstrual blood was antemortem- {Delhi 93)
a) Gutter fracture & Contrecoup injury (AI 11)
a) Alkaline
b) Depressed fracture of skull
b) Light pink
c) Ring fracture at foramen magnum
d) Sutural separation c) Does not clot unless mixed with seminal fluid
530. A child is brought to the casualty with reports of d) Shows endometrial and vaginal cells
violent shaking by parents. Most likely injury is? 541. Thefeminineofimpotenceis- (AIIMS81, DNB 91)
a) Long bone fracture (AI 11) a) Rigidity b) Frigidity
b) Ruptured spleen c) Sterility d) Flaccidity
c) Subdural haematoma e) Nymphomania
d) Skull bone fracture 542. The commonest maternal cause of abortion
531. One gram of smokeless gun powder produces- is - (JIPMER 78, DNB 90)
a) 3000-4000ccofgas (Karn 11) a) Tuberculosis b) Syphilis
b) 9000-10,000 cc of gas c) General debility d) Sexual weakness
c) 12,000-l3,000ccofgas 543. Impotence may be pleaded as a ground for all
d) 15,000-16,000ccofgas except- (AIIMS 79, Delhi 93)
532. Filigree burns are seen in- (Jipmer 11) a) Divorce b) Rape
a) Electrocution b) Lightening c) Adultery d) Unnatural sexual offence
c) Antemortem burns d) Postmortem burns 544. The average length of a full term child will be
about- (AIIMS 79, DNB 89)
SEC- V : IMPOTENCE, STERILITY, a) 30cms b) 35 ems
STERILISATION, VIRGINITY, PREGNANCY, c) 40 ems d) 50 ems .
545. The most striking symptom of pregnancy is-
DELIVERY, NATURAL SEXUAL OFFENSES,
a) Enlargementofabdomen (JIPMER80,AMC87)
UNNATURAL SEXUAL OFFENSES, b) Morning sickness
c) Frequency of micturition
533. Bestiality is - (PGI 87) d) Cessation of menstruation
a) Having intercourse with an animal 546. Post mortem in a new born baby is done by opening
b) Cruelty to animals first (AP 91)
c) Cruelty to fellow human a) The skull
d) Beast like behaviour of an Insane b) The chest cavity
534. The Hydrostatic Test need not be performed in c) The abdominal cavity
case of - (AIIMS 84) d) Any of the above according to convenience
a) Infanticide 547. The currently favoured hypothesis suggests that
b) Spongy or crepitant cot-deaths are due to- (PGI 80, ROHTAK 87)
c) Lungs or liver like in consistency a) Allergy to cow's milk protein
d) None of the above b) Laryngeal spasm
535. Spaldings sign is seen with ....... foetus - c) Prolonged sleep apnoea
a) Live born b) Dead born (NIMHANS 86) d) Respiratory infection
c) Still born d) All of the above 548. When any two objects come into contact there is
536. TbeMTPActwasintroducedin- (AMJ 88) always a transference of material from each object
a) 1%1 b) 1971 on to the other. This is caUed -(PGI 80, Kerala 91)
c) 1974 d) 1975 a) Locard's exchange principle
537. Pedophile is having anal intercourse with- b) Corpus delicti
a) Older women b) Children (A./. 90) c) Metallicfouling
c) Homosexual adult d) Hijra d) Rule ofHaase

527)c 528) d >c 529) b 530) c 531) c 532) b 533) a 534) d 535) b 536)b 537)b 538)c 539)b 540)c
54l)b 542)b 543)None 544)d 545)d 546)c 547)c 548)a
FORENSIC [ 437 ]

549. Hymencanberupturedby- (PG/80,BIHAR91) c) Presence of semen in the posterior fornix in a


a) Sol.apith b) Masturbation girl aged 14 years
c) Sanitary tampons d) Horse riding d) Presence of semen in the posterior fornix in a
e) Any of the above married girl aged 16 years
550. Crown-Heel length of five month foetus would be- 561. The type of paraphilia where sexual excitement
a) 16cm b)25cm (AIIMS87) is obtained by wearing clothes of opposite sex is -
c) 30cm d)35cm a) Tribadism b) Voyeurism (AI 96)
551. All the following are indication for doing Medical c) Transvestite d) Exhibitionism
Termination of pregnancy except- (JIPMER 81, 562. Eonism is - (Kerala 96)
a) Pregnancy from rape AMU 88) a) Desire to identify with the opposite sex
b) Substantial risk of delivering seriously b) Intercourse with a lower animal
handicapped baby c) Female homosexualism
c) Very poor socioeconomic position of the family d) Oral sex
d) Injury to physical and mental health of the e) Obtaining sexual gratification by seeing naked
pregnant woman bodies
552. The most important sign of defloration is-(AP 90) 563. Edmund Locard is known for his- (Karnat 96)
a) Enlargednipple a) System of personal identification using the body
b) Dilation of vaginal canal measurements
c) Enlarged breasts b) Finger print study
d) Ruptured Hymen c) Formula for estimation of stature
553. Dowry Death is included in the following section d) Theory of exchange
ofthe Indian Penal Code- (A/IMS 80, AMU 86) 564. Statutory Rape is- (Kerala 97)
a) 304-A b)304-B a) Rape of an insane woman
c) 320 d)498-A b) Rape of another persons wife
554. The most important cause of temporary impotence c) Rape in police custody
d) Rape of under 15 years
is- (PGI 81, AP 89)
565. Legal age by which fetus is capable of independent
a) Overexcitement b)Guilt
existence is - (MAHE 98)
c) Fear d) Aversion
a) 240 days b) 230 days
555. For hydrostatic test to be positive at all stages,
c) 220 days d) 210 days
the following most be present inside lung alveoli
566. Aseptic autolysis is seen in- (AIIMS 98)
a) Residualair (A/IMS84)
a) Maceration b)Muminification
b) Tidal air c) Adipocere d) Putrefaction
c) Exudates and tidal air 567. Gettlers test is positive in- (Rajasthan 97)
d) Anmiotic fluid and tidal air a) Hanging b) Poisoning
556. All ofthe following could be seen in a false virgin c) Strangulation d) Drowning
except- (J/PMER 80, DNB 89) 568. Negligence on the part of the father towards the
a) Flabby breasts son can be punished under section- (Orissa R)
b) Tight and narrow vagina a) 319 b)317
c) Labia separated c) 315 d)318
d) Thick and elastic injured hymen 569. When two objects come in contact, an exchange
557. Sexual perversions include- (AIIMSM 89) takes place. This is known as- (KERALA 2001)
a) Masturbation b) Transvestism a) Galton exchange
c) Fetichism d) All of the above b) Locard exchange principle
558. Which of the following is considered as an "ideal c) Chertillions system
infanticidal poison" (JIPMER 80, Delhi 81) d) Hasse rule
a) Cannabis b) Cocaine 570. Macerated foetus indicates- (AP 88)
c) Opium d) Alcohol a) Dead born b) Still born
559. In India, a boy below years cannot be charged c) Live born d)IUGR
witbrape- (PGI 80, AIMS 81) 571. Which of the following statements about the
a) 12 b) 14 award of capital punishment to a pregnant woman
c) 21 d) No such limit sentenced to death is correct- (Kerala 2K)
560. One ofthe following is a sure sign of rape: a) Until delivery
a) Ruptured hymen in a (JIPMER 80, DNB 89) b) Unti16 months after the birth of the child
girl aged 20 years c) Untill year passes after the birth of the child
b) Presence of semen in the posterior fornix in a d) Unti12 years pass after the birth of the child
girl age 20 years e) None of the above

549)e 550)b 55l)c 552)d 553)b 554)c 555)b 556)b 557)b,c 558)c 559)d 560)c 561)c 562)a
563)d 564)d 565)d 566)a 567)d 568)b 569)b 570)a 57l)b
FORENSIC [ 438]

572. The commonest cause of impotence is- (UPSC 87) 584. Keyhole bullet entry is a feature of - (MAHE 05)
a) Syphilis b) Psychogenic a) Tandem Bullet b) Durn Durn Bullet
c) Leriche syndrome d) Diabetes mellitus c) Ricochet Bullet d) Fragmentation Bullet
573. Which part of the hymen ruptures first-AIIMS 92) 585. In a Anti-mortem burn which is NOT true-
a) Posterior b) Posterolateral a) Line of redness (MAHA 05)
. c) Anterolateral d) Anterior b) Increase of enzymes
574. Paraphilias are all except- (PGI OJ) c) Vesicle formation
a) Bisexuality b) Homosexuality
d) Non albuminus bulla
c) Bestiality d) Frottuerism
586. A cognizable offence signifies- (MAHA 05)
e) Sadomasochism
575. Spalding sign is seen in- (PGI OJ) a) Imprisonment upto 2 yrs
a) Abortion b) Still birth b) Imprisonment upto 3 yrs + Rs. 1000 fine
c) IUD d) Infanticide c) Arrest without warrant
576. Rape- is defined if man has intercourse with d) Only fine upto Rs. 1000
women- (PGI OJ) 587. For dowry death punishment is-
a) of<16yrage b)of<15yrage a) 3 year imprisonment+ Rs. 10,000/- fine
c) Wife of< 15 yrage d) Wife of< 16yrage b) 5 year imprisonment+ Rs. 15,000/- fine
577. In prenatal diagnostic technique Act 1994 which c) 7 year imprisonment+ Rs. 20,000/- fine
one of the following is not a ground for carrying d) 2 year imprisonment+ Rs. 15,000/- fine
out prenatal test- (AI 03) 588. Chop wounds are caused by weapons with-
a) Pregnant women above 35 years a) Sharp cutting edge, heavy (Orissa 05)
b) History of two or more spontaneous abortion or b) Blunt & Heavy
fetal loss c) Sharp cutting edge light
c) When fetal heart rate is 160 per min at fifth and d) Sharp edges
120 per min at ninth 589. Gunshot residue on hands can be detected by-
d) History of exposure to potentially teratogenic drugs a) Phenolphthalein test (AIIMSNOV05)
578. Adeadhornfoetusdoesnothave- (AIIMS03) b) Dermal nitrate test
a) Rigor mortis at birth b) Adipocere formation c) Benzidine test
c) Maceration · d) Mummification d) Hydrogen activation analysis
579. For Sexual intercourse with a prostitute- 590. A dead body is found to have marks like
a) Consent is required (Bihar 03) branching of a tree on front of chest. The most
b) Consent is not required likely cause of death could he due to-
c) May or may not a) Fire-arm (AIIMS NOV 05)
d) None b) Lightening injury
580. MasochismMeams- (Jharkand 03) c) Injuries due to bomb blast
a) Sexual intercourse with dead body d) Road traffic accident
b) Sexual pleasure by contact with articles of
591. In a firearm injury, there is burning, blackening,
opposite sex
toattooing around the wound, along with cherry
c) Sexual pleasure by the suffering of pain
red colour of the surrounding tissues and is
d) Sexual pleasure by self stimulation
cruciate in shape, the injury is - (AI 05)
581. Following is not a feature of self inDicted injury-
a) Close shot entry b) Close contact exit
a) Clothes are cut in incompatible way with number,
c) Contact shot entry d) Distant shot entry
length direction of nature of wounds (SGPGI 05)
b) Characteristic multiple & superficial injuries 592. 'Patterned' abrasion is variety of- (AIIMS May 05)
c) Multiple scars of different edges a) Linear abrasion b) Pressure abrasion
d) Injuries on palmar aspect of hand c) Sliding abrasion d) Superficial bruise
582. Tattooning is seen surrounding the wound of 593. Scab or crust of abrasion appears brown- (AI 06)
entrance from a revolver or pistol, if the weapon a) Between 12-24hours b)Between2-3 days
is discharged up to distance of- (COMEDK 05) c) Between 4-5 days d) Between 5-7 days
a) 10cm b) 30cm 594. 'Fracture-ala signature' is- (AIIMS 06)
c) 50cm d)90cm a) Gutter fracture b) Depressed fracture
583. Antemortem and postmortem blisters can be c) Ring fracture d) Sutural separation
differentiated by- (PGI June 05) 595. A bullet fired from a gun is not released. It is
a) Size ejected out with the subsequent shot. It is known
b) Colour ofblister fluid as- (HP06)
c) Choloride & albumin content a) Durnburn bullet b) Rocketing bullet
d) PM blister is dry c) Richochet bullet d) Tandem bullet

572)b 573)b 574)a 575)c 576)a,c 577)c 578)b 579)a 580)b 58l)d 582)d 583)c 584)c 585)d
586)c 587)b 588)a 589)b 590)b 59l)a 592)b 593)b 594)b 595)d
FORENSIC [ 439]

596. Ring fracture is present around- (Karn 06) 608. What is meant by "Suppositions child" -
a) Obturator foramen a) Child born out of wedlock (PGI Dec 08)
b) Glenoid cavity b) Child fictitiously claimed by a woman
c) Foramenmagnum c) Second born of a twin pregnancy
d) Base of a metacarpal boone d) Child is born after artificial insemination
597. In Forensic science, a comparison microscope is homologous
used for examination of- (Karn 06) 609. Unnnaturalsexual offences is/are- (PGI June 09)
a) Soft tissues b) Bone pieces a) Sodomy b) Buccal coitus
c) Sample of soil d) Bullets c) Rape d) Incest
598. Fresh bruise is coloured- (Karn 06) e) Adultry
a) Red b)Pink 610. Voyeurism is a sexual perversion characterized
c) Blue d) Brown by- (COMED09)
599. Shape of nulliparous cervical canal is- a) Sexual pleasure derived form observing people in
a) Circular b) Transverse (Aiims May 07) sexual acts
c) Spherical d) Longitudinal b) Sexual focus on children
600. 509-IPC deals with- (UP 07) c) Sexual focus on pain
a) Masturbation in public place d) Sexual focus from non living objects
b) Inquest 611. Which oftbe following is not a ground for cmiduction
c) Dowry death MfP? (DELHI PG Mar. 09)
d) Act intended to insult the modesty ofa women a) Contraceptive failure b) Rape
601. Fetishism is a sexual perversion characterized by- c) Unmarried female d) Mother at risk
a) Sexual focus on children {Corned 08) 612. Early signs of pregnancy include all, except:
b) Sexual focus on genital rubbing a) Quickening (DELHI PG Mar. 09)
c) Sexual pleasure from pain b) Amenorrhea
d) Sexual pleasure derived from non living objects c) Morning sickness
602. Puuishmentfor the offence of rape under- (UP 08) d) Detection ofHCG in urine
a) 375 IPC b) 376 IPC 613. All of the following are features of four months of
c) 377IPC d)378IPC intra-uterine life, except- (DELHI PG Mar. 09)
603. Which of the following test is done for detecting a) Length of16 em
vaginal cells from the accused in a case of rape? b) Weight ofl25 gm
a) Lugol's iodine b) Phenolphthalein (AI 08) c) Sex differentiable
c) Toludine blue d) Methylene blue d) Nails upto tips of fingers
604. Which oftbe following best describes the mechanism 614. In a virgin girl, rupture of hymen after sexual
ofinduction oflabor with use of abortion stick- intercourse, occurs at which position-(PGI Nov 09)
a) Stimulation of uterine contraction (AI 08) a) 12'0clock b)6'0clock
b) Oxytocin present in the stick c) 11' 0 clock d)7' 0 clock
c) Uterine necrosis e) 10' 0 clock
d) Menstrual bleeding 615. In Buggery, the victim in called paederasty ifhe is.
605. Affiliation refers to- (Manipal 08) a) Young boy b) Young girl (Maharashtra 10)
a) Paternity dispute b) Divorse case c) Oldwoman d)Oldman
c) Rape d) Adultery 616. Most important sign of defloration?
606. Which of the following is called catamite ? a) Enlarged breast (Maharashtra 10)
a) Passive victim of sodomy anybody (APPG 08) b) Enlarged nipple
b) Passive victim of sodomy young boy c) Roomyvagina
c) Both d) Ruptured hymen
d) None 617. Legal age by which fetus is capable ofindependent
607. What is transvestism? (APPG 08) existence- (DPG 10)
a) Wearing clothes of opposite sex to be known as a) 240 days b) 230 days
of opposite sex c) 220 days d) 210 days
b) Wearing clothes of opposite sex for sexual 618. Rape, even after consent is considered when age of
gratification women is- (PGI May 10)
c) Contact with another person to obtain sexual a) <16yr b)<17yr
satisfaction c) <21 yr d)< 25 yr and in police custody
d) None e)<l8yr 4

596)c 597)d 598)a 599)b 600)d 601)d 602)b 603)a 604)a 605)a 606)b 607)b 608)b 609)a,b
610)a 6ll)c 612)a 613)d 614)b 615)a 616)d 617)d 618)a
FORENSIC [ 440 ]

619. Incest- (Karn 11) 629. The hair examination is important for the
a) Is punishable under Sec. 294 IPC following cases except- (AIIMS 79, PGI 83)
b) Is punishable under Sec. 304A IPC a) To find out its origin
c) Is punishable under Sec. 377 IPC b) To find the accused or victim
d) Is not punishable in India 4 c) In cases of sexual assault
d) In cases of lead poisoning
SEC· VI : FORENSIC PSYCHIATRY 630. The following may be the examples of privilaged
communication except- (JIPMER 80, AIIMS 81)
620. Professional secrecy can be divulged- (AIIMS 85) a) An impotent person marrying
a) If the doctor feels so b) A syphilitic taking bath in a public pool
b) On demand by a court c) Engine driver found to be colour blind
c) Both d) Pilot having hypermetropia
d) None 631. Indian Lunacy Act was passed in -(PGI 79, Delhi 8 7)
621. McNaughten Rule is concerned with- (TN 89) a) 1804 b) 1881
a) Criminal responsibility b) Litigation c) 1916 d) 1921
c) Rape d) Suicide 632. Criminal negligence is punishable under-
622. Which of the following legislation is regarding a) 306IPC b)376IPC
insanity- (JIPMER 90) c) 304-AIPC d)304-BIPC
a) McNaughten Rule b) Curren Rule 633. All offollowing can be used in criminal negligence
c) Durhams Rule d) All of the above except- (All India 96)
623. Law does not consider the following doctrine in a) Resipsa loquitor
a charge of criminal negligence- (PGI 93, AP 91) b) Contributory negligence plea
a) Vicarious laibility c) Law of vicarious liability
b) Contributory negligence d) Novus actus interveniens
c) Resipsa loquitur 634. 20 year old young female comes to a male
d) Novus actus interveniens gynaecologist for pelvic examination- (PGI 95)
624. A quack gives a penicillin injection to a patient The consent for examination is :
who dies due to it. Will the quack be relieved for a) Implied consent b) Written consent
his criminal responsibility by raising a plea of c) Verbal consent d) Informed consent
accident- (PGI 80, ROHTAK 88) 635. Repeated advertisement in newspaper by a
a) Definitely medical practitioner is an example of -
b) Likelyto a) Infamous conduct (JIPMER 98)
c) Never b) Ethical negligence
d) Deserves to be acquitted on the Strength of evidence c) Criminalnegligence
625. An arrested person on request the magistrate for d) Privilaged communication
a medical examination of his body by a registered 636. Death of a patient due to an unintentional act
medical practitioner as per the following provision by doctor, staff or hospital is - (MAHE 98)
in the code of criminal procedure- (JIPMER 79, a) Therapeutic misadventure
a) Section 53 b) Section 54 BIHAR 91) b) Vicarious liability
c) Section 56 d) Section 57 c) Therapeutic privilage
626. Criminal responsibility of a person of unsound d) Diminished liability
mind in India is incorporated in the Indian Penal 637. Medical negligence falls under- (AIIMS 98)
Code- (JIPMER 80, DNB 89) a) Sec. 304A b) Indian contract act
a) Section 302 b) Section 84 c) IPC312 d)IPC351
c) Section 85 d) Section 88 638. Normal courtesy of one doctor towards another
627. Criminal responsibility in Indian constitution is is according to- (AMC 99)
included in IPC section- (PGI 81, KERALA 90) a) Medical etiquette b) Medical ethics
a) 84 b)300 c) Both d) None
c) 302 d)304 639. A doctor should not take fees for issuing
628. Disciplinary control over Registered Medical certificate - (Calcutta 2K)
Practitioners is under- (AIIMS 85) a) To a govt. employee b) Death certificate
a) State Medical Council c) Poor patient d) Psychiatric patient
b) Indian Medical Council 640. In case of malpractice punishment is given by-
c) Director of Medical and Helth Services a) State medical council b) MCI (Calcutta 2K)
d) Health secretary of State Government c) IMA d) High court

619)d 620)b 621)a 622)d 623) b,d 624) c 625)b 626)b 627)a 628)a 629)a 630)a 631)None
632)c 633)a,b 634)a 635)a 636)a 637)a 638)a 639)b 640)a
FORENSIC [ 441 ]

641. Professional death sentence is- (AMC 2K) 653. Presence of semen in the vaginal canal is a proof
a) Imprisonment for whole life of- (J&K05)
b) Rigorous imprisonment a) Rape b) Sexual intercourse
c) Erasing the name from the panel ofRMPs c) Indecent assault d) Attempted rape
d) None 654. MTPActwaspassedintheyear- (MAHE 05)
642. For diagnosis of lunacy a person can be kept a) 1971 b) 1974
under observation at a stretch for- (Orissa 98) c) 1981 d) 1984
a) 2 days b) 7 days 655. Sexual asphyxia is associated with- (MAHA 05)
c) 10 days d) 30 days a) Sadism b) Masochism
643. Me Naughten's rule: (Orissa R) c) Fetichism d) Vouyerism
a) Section 84 b) Section 85 656. Feature of sodomy, conclusive in nature-
c) Section 86 d) Section 87 a) Dilated tender levator ani (Orissa 05)
644. All ofthe following conditions of defence available b) Lax sphincter ani
to a doctor against allegation of negligence except c) Sperms in anus
a) Medical maloccurrence (Kerala 2K) d) Bleeding from anal verge
b) No fee for treatment charged 657. 'Vagitus uterinus' is- (AIIMS NOV 05)
c) Therapeutic misadventure a) An infection of vagina
d) Res Judicata b) An infection of uterus
e) Error judgement c) A cry of unborn baby from uterus
645. Lucid interval is seen in- (NIMHANS 2K) d) Infection of both vagina and uterus
a) Insanity b) Malignant disease 658. Which of following tests in used to detect semen ?
c) Electrical shock d) Syphilis a) Phenolphthalein test b) Reine's test (AI 06)
646. Barr body is seen in- (TN 2001) c) Barberio's test d) Paraffin test
a) Buccalmucosasmear b)RBC 659. Age limit for marriage in Men- (PGI June 06)
c) WBC d) Saliva a) 15 b) 16
647. Euthanasia is - (Kerala 97)
c) 18 d)21
a) Recently recognised in India
e) 25
b) Recognoised in all countries
660. Sexual asphyxia is associated with- (HP 06)
c) Recognized is few countries
a) Sadism b) Masochism
d) illegal world wide
c) Fetishism d) Voyeurism
648. McNaughten rule is applicable to- (PGI 02)
a) Calculate length of fetus b) Estimate height 661. Supposititious child is - (Manipal 06)
c) Insanity d) Dactyglography a) Child born after death of father
e) Detect CO poisoning b) Birth after artificial insemination
649. During an operation if a pair of scissors is left in c) Child born through Surrogate mother
abdomen, it is known as- (Kerala 04) d) Fictitious child
a) Petty's method b) Res ipsa forcepalis 662. Irresistible sexual desire in male is known as-
c) Res ipsa loquitor d) Pharaoh's serpents a) Sadism b)Tribadism (AiimsMay08)
650. A case of head injury requiring urgent c) Satyriasis d) Nymphomania
decompression, when there are no relative present. 663. Legal Responsibility ofinsane is/are all except-
The doctor mnst- a) Rule of Haase (PGI June 09)
a) Operate b) Me Naughten's rule
b) Consent from police is a must c) Curren's rule
c) Wait for relatives d) Durham rule
d) None e) Rule of nine
651. Disputed maternity can be solved by using the 664. All are related with legal responsibility to insane
following tests except- (All India 04) person, except- (DELHI PG Mar. 09)
a) Blood grouping b) HLA typing a) Rule of Hasse b) McNaughten's rule
c) Precipitin test d) DNA finger printing c) Durham'srule d)Curren'srule
652. A 16 years boy is accused as rape to a 12 years 665. For claiming criminal negligence against a doctor,
old girl. By which of the following the boy can case should be filed within - (Jipmer 10)
prove that he is innocent- (PGI June 05) a) 1 year b) 3 years
a) No abrasion in body & genitalia c) 6 months d) 2 years
b) Absence of blood & semen in cloth & genitalia 666. Not a grievous hurt- (PGI May 10)
c) Absence of saliva in cheek a) Emasculation b) Contusion over scalp
d) Age of the boy 16 years c) # of bone d) Hurt which endanger life
e) Presence of smegma in penis & copora granulosus e) Severe bodily pain for 15 days

64l)c 642)c,d 643)a 644)b 645)a 646)a 647)c 648)c 649)c 650)a 65l)c 652) a,b 653) b 654)a
655)b 656)c 657)c 658)c 659)d 660)b 661)b 662)c 663) a,e 664) a 665)d 666)e
FORENSIC [ 442 ]

SEC ·VII : FORENSIC EXAMINATION OF 680. Fragmentation of medulla of scalp hair is a feature
BIOLOGICAL FLUID, STAINS & OTHER of- (All India 96)
MATERIALS, COLLECTION AND a) Negro race b) Monogolians
c) Caucasians d) Aryans
PRESERVATION OF BIOLOGICAL MATERIALS 681. Absent fragmented medula in hair is seen in all
except- (AI 98)
667. If the father bas blood group genotype AO and a) Negroes b) Aryans
the mother's blood group genotype BO. The c) Mongloids d) Caucasians
offspring may have blood group genotype- 682. In Takayama test, which of the following is seen
a) A b)B (AIIMS84) under microscope- (AP 97)
c) AB d) All of the above
a) Yellowish to brownish rhomboid crystals
668. Best test for seminal stains is- (PGI 86)
b) Yellow needle shaped crystals
a) Acid phosphatase test.
c) Pink feathery structures
b) Precipitin test
d) Spherical granules
c) Florence test
683. Benzidine test is positive with the enzyme-(Kerala 94)
d) Alkaline phosphatase test
a) Cytochrome oxidase b) Dehydrogenase
669. Florencetestdetects- (AIIMS85)
a) Bilirubin • b) Urine c) Hydrolase d) Peroxidase
c) Semen d) Blood 684. Species identification is done by- (AIIMS 03)
670. Blood clots after •••••••••••• brs. of death- (PGI 85) a) Neutron activation analysis (N.A.A)
a) 112hrs. b)45minutes b) Precipitin test
c) 114 hrs. d) 1Yz hrs. c) Benzidine test
671. Berberio's test is done for- (PGI 89) d) Spectroscopy
a) Blood b) Urine 685. Hydrogen peroxide is used in all the following
c) Saliva d) Semen chemical tests for blood except- (All India 04)
672. In old blood stained fibre, blood grouping is done a) Benzedine test b) Phenoththalein test
·~ (AI9W c) Orthotoluidine test d) Teichmann's test
a) Benzidine test b) Acid elution test 686. Maximum period of observation for proving
c) Preciptin test d) Hemin cystal test insanity is - (Orissa 05)
673. Acid phosphatase test is used for analysing- a) 10 days b) 14 days
a) Semen b)Blood (AI91) c) 30 days d) 90 days
c) Urine d) Bile 687. Me. Naughten 'slaw used in- (PGI June 06)
674. All are tests for Seminal stains except-(AJIMS 91) a) Insanity b) Alcohol level estimation
a) Takayama test b) Barberio's test c) Druglevel
c) Florence test d) Acid phosphatase test 688. Criminal responsibility of a person of unsound
675. Takayamareagentisusedin- (AI 91) mind comes under which penal code- (APPG 06)
a) Gulacum test b) Castle mayer test a) Section 84 b) Section 85
c) Haemin cystal test d) Haemochromogen test c) Section 88 d) Section 302
676. Wbichiswrongmatch- (AIIMS 83) 689. Best method to prove paternity is- (Kerala 94)
Test for blood Positive colour a) Blood grouping b) Finger prints
a) Benzidine test Blue c) Phenotypic resembelance d) DNA fmger printing
b) Gulacum test Deep blue 690. Consider the following statements- (ICS 98)
c) Castle mayer test Pink purple DNA fingerprint is valuable in
d) Leucomalachite test Pink a) Determination of blood group
677. Trueaboutbumanbairis- (Delhi 92) b) Investigation of crime
a) Medulla is 1!3rd of cortex c) Investigation of autoimmune
b) Cortex is 113n1 of medulla d) Determination of paternity
c) Has scales 691. Puppe'sruleisrelatedto- (CMC 2001)
d) None of the above a) Sequence of bullets
678. In Leucomalacbite test, the positive test is b) Enzyme assay in blisters
indicated by colour- (AIIMS 83, 86) c) Related to age estimation
a) Peacock blue b) Deep purple d) Sex determination
c) Red d) Khaki 692. What is 'DNA' fingerprinting- (PGI OJ)
679. Human hair differs from animal hair by that- a) Identification of a person fingerprinting by using
a) Cuticle has scales (PGI 86, genetic enginering method
b) Medulla is thick Delhi 85, 86,89) b) A record of a persons genetic make up
c) Medulla is broader than cortex c) Identification of persons by genetic analysis
d) Pigment is central d) None of the above

667)d 668)a 669)c 670)a 671)d 672)b 673)a 674)a 675)d 676)d 677)a,c 678)a 679)None
680)a 681)a 682)c 683) d 684) b 685) d 686) c 687)a 688)a 689)d 690)b,d 691)a 692)c
FORENSIC [443]

693. Which is the best method to determine 707. Nalorphin is used as an antidote of- (AP 88)
paternity- (Kerala 01) a) Strychnine b)Opium
a) Finger printing b) DNA fmger printing c) Dhatura d) Cocaine
c) Blood group d)HLA 708. Greenish urine is seen in poisoning with-(AMJ 88)
694. The species orgin of blood can be detected by- a) Carbolic acid b) Nitric acid
a) Benzidine test b) Takayama test (Corned 08) c) Copper sulphate d) Hydrochloric acid
c) Spectroscopy d) Precipitin test 709. Optic atrophy can be caused by ......poisoning-
695. DNA fmgerprinting is done with all except- a) Phosphrous b) Ethyl alcohol (AIIMS 85)
a) RBC b)WBC (DPGEE08) c) Methyl alcohol d) Lead
c) Salvia d) Spermatozoa 710. Preservative used for alcohol poisoning is-
696. Dried semen stain in clothes is identified ~Fmmillin (AJIMS8~
by- (AIIMS Nov 09) b) Saturated solution of sodium chloride
a) UV light b) Spectroscopy c) Methyl alcohol
c) Magnifying lens d) Infra red d) None of the above
697. Sodium fluoride is used for preservation of- 711. Blue line on the gums is seen in chronic
a) Urine d) Blood (DPG 10) .....poisoning- (PGI 85,JIPMER 90)
c) Alcohol d) Carbolic acid a) Hg b)Pb.
c) Ag d)As
POISONS 712. Anaemia, punctate basophilia, constipation blue line
and abdominal colic are characreristic of- (Al86)
698. Neostigmine is beneficial iu symptomatic bites by- a) Opium addiction b) Arsenic poisoning
a) Cobra b)Viper (UPSC05)
c) Mercuric poisoning d) Lead poisoning
c) Scorpion d) Black widow spider
713. In Chronic alcoholism which of the following is
699. Brown disclouration ofthe mucosa of the stomach
seen in - (JIPMER 90)
is seen in poisoiniug due to- (AJIMS 87, PGI 86)
a) Delirium tremens b) Wernike's syndrome
a) Nitric acid b) Sulphuric acid
c) Hydrochloric acid d) Mercury c) Korsokoff psychosis d) All Qfthe above
e) Arsenic 714. Dilated pupil, dry mouth is a features of poisoning
700. Amyl Nitrie is used as an antidote in .•.... dueto- (A/.89)
poisoiuing- (AIIMS 87) a) Morphine b) Organophorous
a) C02 b)CO c) Dhatura d) Phenothiazines
c) Cyanide d) Nitric acid 715. The pupil is constricted in- (A.I 89)
e) Arsenic a) Dhatura b)Opium
701. Complication of Kerosene poisoning- (AI 88) c) Benzo diazepines d) Phenothiazines
a) Pneumonia b) Vomiting 716. Best method of treatment of Methyl alcohol
c) Hemoptysis d) Diarrhoea poisoning is- (JIPMER 91)
e) Acute renal failure a) Calcium gluconate b) Ethyl alcohol
702. Diwali poisoning is due to- (AI 88) c) Amphetamines d) !%Ammonia
a) Phosphorus b) Arsenic 717. The nonpoisonous saltofcyanideis-(JIPMER 91)
c) Mercury d) Lead a) Potassium Cyanide b) Sodium cyanide
e) Copper c) Hydrocyanic acid d) Potassium ferrocyanide
703. Antidote for sodium nitrate poisoning is- (Al88) 718. Tactile hallucinations is seen in chronic poisoning
a) Methylene blue IV b) Egg albumin with- (JIPMER 91)
c) EDTA d) Animal charcoal a) Cocaine b)Opiurn
704. Nux Vomica is- (AP 85,88) c) LSD d) Cannabis
a) Vegetable poison containing atropine 719. Anterior horn cells are affected in- (JIPMER 91)
b) Seed containing strychnine a) Dhatura poisoning
c) Used as a cattle poison b) Srtychnine poisoning
d) Used to induce vomiting in poisoning c) Botulism
705. Brown coloured urine is seen in the- (AJ89) d) None of the above
a) Nitric acid poisoning 720. An overdosage of morphine is treated with-
b) Carbolic acid poisoning a) Nalorphine IV b) Steroids (PG I93)
c) Sulphuric acid poisoning c) IV methadone d) Naloxone IV
d) Hydrochloric acid poisoning 721. Which of the following toxins can be detected in
706. Pin point pupils is seen in .......poisoning- burnt bone- (AI 93)
a) Barbiturate b) Morphine (PGI 86) a) Cyanide b) Arsenic
c) Organophosphours d) Methyl aicohol c) Mercury d) Phospoms

693)b 694)d 695)a 696)a 697)b 698)a 699)b 700)c 701)a 702)c 703)a 704)b,c 705)a 706)b,c
707)b 708)a 709)c 710)d 7ll)a,b,c 712)d 713)d 714)c 715)b 716)b 717)d 718)a 719)b 720)d
721)b
FORENSIC [ 444]

722. The non poisonous Salt of cyanide is-(JIPMER 93) 736. In nux vomica poisoning the following is also
a) Potasium cyanide required to be preserved- (PGI 81,AMU 89)
b) Hydrocyanic acid a) Long bones b)Brain
c) Sodium cyanide c) Muscles d) Skin
d) Potassium ferrocyanide 737. The following is also known as "Knock out drops"
723. Which of the following is non poisonous snake- or"mickyfinn- (AIIMS 81, AMU 89)
a) Viper b)Krait (AIIMS81,JJPMER92) a) Chloroform b) Methyl alcohol
c) Sea snake d) Rat snake c) Chloral hydrate d) Ethylene glyco1
724. True about strychnine poisoning is-(DELHI 86, 92) 738. Ganja is obtained from- (JIPMER 79, UPSC 89)
a) All muscles affected of the same time a) Dried leaves b) Fresh leaves
b) Shoulder girdle affected I st c) Flowering tops d) Roots
c) Pelvic girdle affected first 739. Fatal period in sulphuric acid poisoning
d) None of the above is- (AIIMS 8l,BIHAR 90)
725. Critical level of alcohol on blood should be- a) 2-4 hours b) 6-10 hours
a) 0.1% b)0.15% (AJIMS78,DELHI93) c) 12-16hours d) 18-14hours
c) 0.2% d) 0.3% 740. Blood cholinesterase level shonld be estimated for
726. People suffering from achlorhydria may not three weeks, in non-fatal cases of poisoning
sufferfrom toxic effe~t of oral ingestion of- (AMU with- (JIPMER 90)
a) Phosphorus b) Copper sulphate 86) a) Parathion b) Eedrine
c) Arsenic trioxide d) Potassium cyanide c) Thallium sulphate d) Arsenic oxide
727. Shaking palsy is associated with poisoning with- 741. Large bones of about 6 inches are required to be
a) Lead b) Mercury (AIIMS87,AI93) preserved in- (AIIMS 81, BIHAR 89)
c) Arsenic d) Strontium a) Arsenic & antimony poisoning
728. Poisoning by irritants may be mistaken for -(AIIMS b) Aconite poisoning
a) Peritonitis b) Cholera 81,UPSC 88)
c) Dhatura poisoning
c) Gastroenterits d) Intestinal obstruction
d) Smack poisoning
729. Antidote for acute iron poisoning- (PGI 81,
742. An addict develops delusions, misleading
a) Desferioxamine b)NaEDTA DELHI92)
hallucination of sight and bearing and the felling
c) BAL d) Penicillamine
of creeping things of "grains of sand " in the skin
730. The following are true about chloral hydrate except
be is addicted to- (AIIMS 87)
a) Peculiar pungent odour (AIIMS 86)
a) Heroin b) Methadone
b) Crystalline
c) Used as hypnotic c) Cocaine d) Amphetamines
d) Sweetish taste 743. About 20 grams of hairs are required to be
731. At the following blood carbon carbon monoxide preserved in- (AIIMS 79,AMU 89)
concentration the clinic picture will simulate a) Minerals poisoning b) Aconite poisoning
alcobolicintoxication- (JIPMER81, DELHI93) c) Iodine poisoning d) Codeine poisoning
a) 100/o b) 70% 744. Which of the following is not used in methyl
c)30% d)50% alcohol poisoning- (UPSC 85,87, DELHI 87)
732. A pinch of which of the following poisons can kill a) Gastric lavage b) Dialysis
as many as five persons- (BIHAR 92) c) Ethyl alcohol d) High does of vitamin B 12
a) Arsenic trioxide b) Copper sulphate 745. In salicylate poisoning the treatment
c) Lead sulphate d) Arsenic disulphide recommendedis- (AIIMS 87, DNB 88)
733. Which of the following is used in forgeries as an. a) Forced diuresis b) Chelating agents
ink remover solution- (PGI 80, DNB 90) c) Atropine d) Conservative
a) Sulphuric acid b) Nitric acid 746. Average fatal dose of croton oil seeds
c) Carbolic acid d) Oxalic acid is - (PGI 80, AIIMS 85)
734. It is true about formalin- (AIIMS 80,DNB 90) a) 500 drops b) Handful of seeds
a) That it can be used as preservative in alcohol c) About 4 to 5 drops d) 20 drops
poisoning 747. Amyl nitrite is an antidote for poisoning
b) Never used as a preservative for chemical analysis with- (UPSC 85, 86, DELHI 87)
c) Used as a preservative in poisoning with digitalis a) H SO b)Ammonia
4
d) None of the above c) HCN d) Carbon monoxide
735. Which of the following is known as "Thorn 748. One of the following substance produce injuries
apple"- (AIIMS 82) which stimulate contussion-(JIPMER 80, AIIMS 83)
a) Dhatura alba b) Dhatura niger a) Semicarpus anacardium b) Ricinus communis
c) Dbatura stramonium d) Dhatura metet c) Abrus percatorius d) Capsicum annum

722)d 723)d 724)a 725)b 726)d 727)b 728)c 729)a 730)d 73l)c 732)a 733)d 734)b 735)a,b
736)b 737)c 738)c 739)d 740)a 74l)a 742)c 743)None 744)d 745)a 746)d 747)c 748)a
FORENSIC [ 445]

749. In burnt bones, the following can be detected- 763. The average fatal period of copper poisoning is-
a) Arsenic (AIIMS 79,81) a) 30-60minutes b) 1-3 days (AIIMS 87)
b) Lead c) 3-7days d)7-14days
c) Oraganoposphorus compound 764. Ptysalism is seen in- (AIIMS 80, UPSC 84)
d) None of the above a) Arsenic poisoning b) Copper poisoning
750. Which of the following is considered as an antidote c) Dhatura poisoning d) Atropine sulphate
for cocaine poisoning- (PGI 78,AIIMS 80) 765. Which ofthe following tests are nsed for arsenic
a) Phenobarbitone b) Morphine poisoning- (JIPMER 80,PGI 83)
c) Amyl nitrate d) Ether a) Marsh's test b) Reinsh's test
751. Which of the following commonly known as c) Macewen's test d) Marquri' test
766. Widmark's formula is used to calculate the quantity
"thorn apple"- (AIIMS 8l,AMC 83)
of the following in the body after equilibrium between
a) Dhatura alba b) Dhatura niger
the blood and tissues is reached- (ROHATAK 86)
c) Dhatura atrox d) Dhatura stramonimn
a) Methyl alcohol b) Nitrous oxide
752. Nux vomica seeds contain 2 alkaoids, strychnine c) Ethyl alcohol d) Lithimn carbonate
and- (AIIMS 84) 767. The fatal period ofAconite is usually -(.UPMER 80,
a) Hyoscine b) Hyoscyamine a) 5-lOminutes b) 15-30minutesAMU89)
c) Brucine d) Atropine c) 1-5 hours d) 12-48 hours
753. In low concentration of carbon monoxide (4%) 768. All of the following are aryl phosphate except-
the commonest symptom is- (AIIMS 80, UPSC 82) a) Folidol b) Parathion (AIIMS 82)
a) Nausea b) Headache c) Diazinon(Tick20) d) Malathion
c) Muscular weakness d) Ringing in ears 769. Arsenophagists are- (AIIMS 80)
754. The following alkaloids are presents in opium a) Criminals using arsenic for homicidal purpose
except- (AP 86) b) Person using arsenic as abortion stick
a) Hyoscyamine b) Morphine c) Persons who can tolerate high doses of arsenic
c) Papaverine d) Narcotine after taking arsenic in low doses at frequent intervals
755. Chromodacryorroea (shedding of pink tears due d) Persons using it as cattle poison
to accumulation of porphyrin) may be seen in 770. The upper limit of safety for carbonmonoxide in
poisoningwith- (PGI 82) air is- (PGI 81)
a) Arsenic b) Barbiturate a) 0.2% b)O.Ol%
c) CuS04 d) Oraganophosphorous c) 0.4% d)0.005%
756. Which of the following is the lateral curve of the 771. Narcotic Drugs and psychotropic substance act
body in strychnine poisoning-(.UPMER 80, UPSC 83) was passed in the year - (AIIMS 79,DNB 90)
a) Opisthotonus b) Emprosthotonus a) 1981 b) 1983
c) Pleurosthotonus d) None of the above c) 1985 d) 1986
757. Which of the following is also known as 772. Velvety appearance of stomach is seen in poisoning
"Kasoomba" or Madak Chaudu- (AIIMS 83) with- (KERALA 95)
a) Dhatura b) Cannabis a) Abrus percatorius b) Barbiturate
c) Opimn d) Calotropis c) Arsenic d) Lead
758. Magnan's symptom is- (PGI 79) 773. All of the following causes constriction of pupil
a) Illusion b) Delusion except- (AI 95)
c) Formication d) Depersonalization a) Dhatura
759. Which of the following is considered as an b) Morphine
antidote for cocaine poisoning- (AIIMS 80, 81) c) Organophosphorus poisoning
a) Oxygen b)Diazepam d) Pontine haemorrahge
c) Ethyl alcohol d) Nitric oxide 774. Followingdrugecausesdelirium- (AI 95)
760. The fatal dose of absoiute alcohol in an adult is- a) Dhatura b) Cannabis
a) 30ml b)60ml (AP88) c) Morphine d) Barbiturates
c) 90ml d) 150ml 775. Macewans's sign is associated with- (Al95)
761. The alcohol content in 'Absolute alcolol'- (PGI 79, a) Alcohol poisoning b) Pontine haemorrhage
a) 900/o b)95% AIIMS 81) c) Cerebellartmnours d) Cocaine intoxication
c) 99.95% d) 100% 776. Tactile hallucinations occurs iu- (AI 95)
762. The active principles of Dhatura are all of the a) Acute barbiturate toxicity
following except- (AIIMS 84) b) Chronic cocaine poisoning
a) Phyricatachol b) Hyoscyamine c) Parkinsonism
c) Atropine d) Hyascine d) Chronic depression

749)a 750)a 751)a,b 752)c 753)None 754)a 755)d 756)c 757)c 758)c 759)b 760)None 761)c
762)a 763)b 764)b 765)a,b 766)c 767)c 768)d 769)c 770)b 771)c 772)c 773)a 774)a,b 775)a 776)b
FORENSIC [ 446]

777. Cyanide poisoning acts by- (DELHI 96) 791. Almond like bitter smell is positive in- (AIIMS 96)
a) Inhibiting DNA synthesis a) Chloral hydrate poisoning
b) Inhibiting enzymes of proteins synthesis b) Hydrocyanic acid
c) Inhibiting cellular respiration c) Hydrocyanic poisoning
d) Inhibiting protein breakdown d) Oleander poisoning
778. Antidote for oxalic acid poisoning- (KAR. 95) 792. Dryness of mouth dialted pupil & delirium are
a) BAL b) Animal charcoal symotoms of- (UP 97)
c) Calcium gluconate d) Magnesium sulphate a) Chronic lead poisoning
779. Which metal poisoning can be detected analysing b) Opium addiction
bone- (KERALA 96) c) Chronic arsenic poisoning
a) Arsenic b) Nickel d) Dhatura poisoning
c) Chromium d) Lead 793. Heavy metal poisoning causes proximal tubular
e) Mercury necrosis - (PGI 97)
780. Fataldoesofmethanol- (KERALA 96) a) Hg b)Cadium
a) 15ml b)30-60ml c) Lead d) Gold
c) 60-250ml d)500ml 794. Proximal convoluted tubule necrosis is seen in
781. What is not seen with bite of pit viper- (PGI 95) .•...poisoning- (AIIMS 97)
a) CVS collapse b) Absence oflocal sign a) Phenol b) Arsenic
c) Convulsion d) DIC c) Aconite d) Lead
782. Hyperpigmentation of palms & soles is seen in 795. Almond bitter smell is seen with- (KERALA 97)
which poisoning- (CUPGEE 96) a) Cyanide poisoning b) Arsenic poisoning
a) Arsenic b) Mercury c) Lead poisoning d) None of the above
c) Lead d) Copper 796. Haemodialysis is done in all except .....poisoning-
783. Carbolic acid poisoning; Postmortem fmding are- a) Barbiturates b) Salicylates (AI 98)
a) Greenish stomach (A.P 96) c) Kerosene d) Alcohol
b) Yellowcharredstomach 797. In acute organophosphorous poisoning which of
c) Leathery stomach the following is seen- (AP 96)
d) Black charred stomach a) Dry lungs b) Edematous lungs
784. Marsh test used in diagnosis of poisoning due to- c) Pneumonia d) Pleural rub
a) Arsenic b) Lead (AIIMS 97) 798. Widmarks formula is used for which poisoning-
c) Iron d) Copper sulphate a) Alcohol b) Mercury (MP 98)
785. Hemoglobinuria is seen in all except- (AIIMS 97) c) Cyniad d) Lead
a) Copper sulphate poisoning 799. Asthma like symptoms is seenin •.••••.••••poisoning-
b) Snake venom a) Arsenic b) Organophosphorous
c) Clostridium welchii sepsis c) Lead d) Gold (M.R 98)
d) Thalassemia 800. Gastric lavage is contraindicated in .•••• poisoning-
786. Opium poisoning produces ••••••• types of a) Kerosene b) Carbolic acid (M.P 98)
respiration- (AIIMS 97) c) Alcohol d) Salicylate
a) Rapid and shallow b) Rapid & deep 801. Average fatal period of copper poisoning-
c) Shallow d) Diaphagmatic a) 4 hrs b) 18-36hrs (Kerala 98)
787. Arsenic poisoning mimics- (AI 97) c) 24-48hrs d) >5 days
a) Cholera b) Food poisoning 802. In barbitutate poisoning which is not
c) Alcohol withdraw! d) Tetanus present- (ROHATAK97)
788. 'Liquid gold' with reference to urine is seen in a) Hypotrension b) Hypothermia
presenceof- (AI97) c) Pupils not reactive d) Pulmonary edema
a) Heavy metals (222-M) b) Barbiturates e) None
c) Organophosphorous d) Lead poisoning 803. Morbid jealousy is diagnostic of- (ROHATAK 97)
789. Post mortem cherry red discoloration is due to- a) Cocaine b) Cannabis
a) Asphyxia (AI 97) c) Alcoholism d) Tabacco Intoxication
b) Carbon monoxide poisoning e) Barbiturate
c) Drowning 804. Erethism is seen in toxicity of- (AIIMS 98)
d) Organophosphorus poisoning a) Arsenic b) Mercury
790. Run Amok Homicide delusion is characteristic c) Lead d) Iron
of- (AIIMS 96) 805. Stability of venom is dependent on- (AMC 99)
a) Cannabis b) Cocaine a) H bond b) Disuphlide bond
c) Dhatura d) Alcohol poisoning c) Vander waal 's d) Ionic interaction

777)c 778)c 779)a 780)c 78l)b 782)a 783)c 784)a 785)d 786)c 787)a 788)b 789)b 790)a
79l)b . 792)d 793)a,b 794)a,b 795)a 796)c 797)b 798)a 799)b 800)a 80l)b 802)c 803)c 804)b
805)b
FORENSIC [ 447]

806. Venom of sea snake is mostly- (UPSC 2K) 820. Amyl Nitrate is used as an antidote for- (AIIMS 87)
a) Neurotoxic b) Haemolytic a) Co poisoning b) Cyanide poisoning
c) Myotoxic d) Hepatotoxic c) C0 2 poisoning d) Dapsone poisoning
807. Widmark formula is used for- (MP 2K) e) Barbiturate poisoning
a) Alcohol b) Barbiturate 821. A patient with headache, nausea, vomiting, fatigue,
c) Cannabis d) Opium jaundice,hepatomegaly and oliguria may have an
808. Perforation of stomach is more common due to occupational disease caused by- (AIIMS 79, PGI 80)
a) ~S b) Carbon tetrachloride
ingestion of- ·(ORISSA 98)
c) Benzol d) Mercury
a) Nitric acid b) Sulphuric acid
e) Lead
c) Hydrochloric acid d) Carbolic acid 822. Amenorrhoea and sterility may occur in women
809. Metal fume fever is common in chronic poisoning as a result of occupational exposure to-(Jimper 80,
with- (Orissa 98) a) Zinc b) Mercury
a) Mercury b) Lead c) Chromium d)CO
c) Zinc d) Thallium e) Lead
810. Elevated levels of cyanide is seen in which of the 823. A comatose child has a garlic like odour to the
following- (AIIMS 2K) breath. The most probable diagnosis is -(UPSC 82)
a) Cold b) Scald a) Flavism b) Atropine poisoning
c) Starvation d) Thermal burns c) Lead poisoning d) Arsenic poisoning
811. Least toxic compound oflead is- (MAHE 2001) 824. Haemodialysis is indicated in- (UPSC 2K)
a) Lead arsenate b) Lead oxide a) Organophosphorous poisoning
c) Lead carbonate d) Lead sulphide b) Salicylate poisoning
812. Linseeds used for (cattle/animal) poisoning c) Mushroom poisoning
contain which of the following principle poisoning d) Phenobarbitone poisoning
substance.- (AI 2002) 825. Sodium Fluoride is used as a preservative of blood
a) Aconite b) Hydrocyanic acid in the estimation of- (PGI 93)
c) Pilocarpine d) Atropine a) Cyanide b) Mercury
813. Narcotic abuse case with jet black, tongue, the c) Arsenin d)Alcohol
cause is- (AIIMS 2001) 826. Macewen's sign is seen in poisoning with-
a) Salicylates b)Alcohol (A/93)
a) Cocaine poisoning b) Morphine poisoning
c) Barbiturates d) Diazepan
c) Cannabis toxicity d) LSD poisoning
82 7. Hemodialysis is useful in poisoning with all except-
814. Patient is brought to casualty with poisoning .
a) Barbiturates b) Methanol
Stomach wash lavage fluid on heating produced
c) Diazepam d) Salicylates
black precipitate. The poison is- (AIIMS 2001)
828. A patient is brought with a history of excessive
a) TIC20 b)AgN0 3
thirst and agitated behaviour. On examination,
c) Malathione d) Celphos
the pupils were dilated and temperature was
815. Alternate contraction & dilatation of pupils is raised with dry skin. The patient was picking on
seen in •••••• poisoning- (TN 2002) clothes. The most likely diagnosis is- ( UPSC 95)
a) Aconite b) Cyanide a) Organophosphorous poisoning
c) Hemlock d) Organophosphate b) Heat storke
816. Non irritant gas with systemic toxicity -(PGI 2002) c) Dhatura poisoning
a) N2 b)CO d) Thyroid crisis
c)~ d)Cl 829. A 3 year old male child has features of gripping
e)HCL abdominal pain, severe vomiting, bloody diarrhoea
817. Putrefaction is retarded by- (AIIMS 02) and fever. The most likely diagnosis is- (UPSC 95)
a) Carbolic acid a) Iron poisoning
b) Oxalic acid b) Lead poisoning
c) Organophosphorous poison c) Phosphorous poisoning
d) Hydrochloric acid d) Magnesium poisoning
818. Acute lead poisoning in child presents as- (AI 99) 830. Features of chronic lead poisoning are all except-
a) Status epilepticus b)Ataxia a) Encepalopathy b) Burtonion line (AI 95)
c) Focal deficit d) Papilloedema c) Cutaneous blisters d) Constipation
819. Amyl nitrate is usually administered 831. Symptoms in first two days of poisoning with
by- (ORISSA 91) methyl isocyanate are all except- (AMU 95)
a) Inhalation b) Sublingual route a) Hemoptysis b) Lacrimation
c) Subcutaneous route d) Ingestion c) Chest pain d) None

806)c 807)a 808)b 809)c 810)d 81l)d 812)b 813)a 814)d 815)a 816)b 817)a 818)a 819)a
820)b 821)b 822)e 823)None 824)b,d 825)d 826)b 827)c 828)c 829)a 830)c 831)a
FORENSIC [ 448]

832. Type of respiration in Morphine poisoning is- 846. Specimens for toxicological studies is preserved
a) Deep Rapid b) Rapid shallow (AJIMS 97) in- (PGI 99)
c) Slow d) Diaphragmatic a) 10% offormaldehyde
833. Haemodialysis is useful in all except- (AJIMS 96) b) Alcohol
a) Barbiturate poisoning b) Methanol poisoning c) Supersaturated solution of common salt
c) Salicylate toxicity d) Digoxin toxicity d) Normal saline
834. Not seen in Barbiturate poisoning is- (AIIMS 97) 84 7. The principle of using nitrate in cyanide poisoning
a) Hypothermia b) Hypotension is to reduce- (PGI 99)
c) Flaccid coma d) Non reacting pupil a) Methetnoglobin production
835. Antidote of choice in methyl alocohol poisoning b) Sufi Hb production
is - (AIIMS 98) c) To correct metabolic abnormalities
a) Atropine b) PAM d) Carboxy Hb production
c) Neostigmine d) Adrenaline 848. Preservative used for toxicological specimen-
e) None a) 20%formalin (PGI 2000)
836. An overdose of narcotics causes all of the following b) Saturated sodium chloride
except- (KARNAT98) c) 20% alcohol
a) Dilated pupils b) Hypotension d) 10% alcohol
c) Depressed reflexes d) Coma 849. Proximal tubule proteinuria and painful bone
837. Which of the following gas causes toxicity lesions are seen in overdose of- (PGI 2000)
(systemic) without causing respiratory irritation- a) Cadmium b) Lead
a) Sulphur b) Carbon monoxide (AI 02) c) Mercury d) Phenol
c) Ammonia d) Hydrochloric acid 850. Pinpoint pupil is seen in which of the following
838. Heavy metal poisoning causing PCT necrosis is- conditions- (PGI 01)
a) Cadmium b)Lead (PG/97) a) Organophosphorus poisoning
c) Mercury d) Gold b) Opium poisoning
839. Which is excreted typically in lead poisoning- c) Celephos poisoning
a) Urobilinogen b) Coproporphyrin (PGI 97) d) Dhatura (Dhatura fastuosa)
c) Bilirubin d) Bile salts e) Hyoscine
840. In organophosphorous poisoning, following are 851. Cherry red discoloration of skin is seen in which
seen except- (PGI 97) poisoing in post mortem lividity- (PGI 01)
a) Pupillary dilatation b) Salivation a) Lead b) Arsenic
c) Bronchospasm d) Sweating c) HCN d)CO
841. Atropine is used for following except- (PGI 97) 852. Opium poisoning is featured by- (PGI 01)
a) Organophosphorus posioning a) Constricted pupil
b) Mushroom poisoning b) Respiratory depression
c) Physostigmine overdose c) Tachycardia
d) Glaucoma d) Tachypnea
842. Postmortem fmding in carbon monoxide poisoning e) Hallucination
is- (PG/97) 853. Blue hypostasis is seen poisoning due to -(PGI 02)
a) Cherry red blood b) Intense cyanosis a) ~S b) Phosphorus
c) Excessive salivation d) Pin point pupil c) CD d) Organophosphorus compounds
843. In aluminium phosphide poisoning, not true is- e) Arsenic
a) Accumulation of acetylcholine at NM (PGI 98) 854. Amylnitrate is used in which of the following
junction poisioning- (PGI 02)
b) Cytochrome oxidase a) Hydrocyanide poisoning b) Red phosphorus
c) Phosphine formation c) Yellow phosporus d) Arsenic poisoning
d) Metabolic acidosis e) Lead poisoning
844. Dhatura poisoning is characterized by- (PGI 99) 855. Which of the following is neurotoxic- (PGI 02)
a) pinpoint pupil a) Sea snake b) Russel viper
b) Dilated salivary gland c) Krait d) Cobra
c) Dilated pupil with facial flush e) Pitviper
d) Decrease temp 856. Wid mark's formula is used in which poisor-
845. Marsh testis for detection of- (PGI 99) a) Alcohol b) Barbiturate (PGI 02)
a) Lead b) Strychnine c) Carbolic acid d) Clephos poisoning
c) Arsenic d)Opium e) Lead

832)c 833)d 834)d 835)e 836)a 837)b 838)c 839)b 840)a 841)d 842)a 843)a 844)c 845)c
846)c 847)a 848)b 849)a 850)a,b 85l)d 852) a,b,e 853) a 854) a 855) c,d 856) a
FORENSIC [ 449]

857. Chronic arsenic poisoning causes- (PGI 02) 869. Thue about Blindness due to methanol poisoning-
a) Pure sensory neuropathy a) Due to direct off of formic acid
b) Pure motor neuropathy b) Formipezole, a specific alcohol dehydrogenase
c) Mixed sensory and motor neuropathy inhibitor is helpful
d) Painful neuropathy c) Ethanol is useful reduced blindness
e) Hyperkeratosis of skin d) Gastric lavage not helpful
858. In a patient with acute arsenic poisoning, which e) Activated charcoal is highly effective
of these would show accumulation of as- (PGI 02) 870. Deep blue colour of hypostasis is seen in death
a) Liver b) Bone marrow due to poisoning by- (Al 04)
c) Skin d) Kidney a) Potassium cyanide b) Phosphorous
c) Aniline dyes d) Carbon monoxide
e) Brain
871. Which of the following is the most reliable method
859. A person feels that grains of sand are lying under
of estimating blood alcohollevel- (AI 04)
the skin or some smaU insects are creeping on the a) Cavett's test
skin giving rise to itching sensation; the condition b) Breath alcohol analyzer
is seen in - (PGI 03) c) Gas liquid chromatography
a) Cocaine poisoning d) Thin layer chromatography
b) Organophosphorus poisoning 872. Which of the following are poisonous -(Jharkand 03)
c) Morphine poisoning a) Krait b) Hydropis
d) Alcohol withdrawal c) Cobra d) All
e) Carbon monoxide poisoning 873. Rain drop pigmientation is seen in- (Jharkand 03)
860. Viper snake venom is- (PGI 03) a) Arsenic poisoning
a) Haematotoxic b) Vasculotoxic b) Phosphorous poisoning
c) Myotoxic d) Hepatotoxic c) Mercury Poisioning
e) Neurotoxic d) All
861. Delirium is seen in which of the following 874. Magnan's Symptom is- (Kerala 04)
poisonings- (PGI 03) a) illusion b) Delusion
a) Dhatura b) Lead c) Auditory hallucination d) Formication
c) Naxvomika d) Opioid 875. Cyanide poisoning kit does not contain(Kerala 04)
e) Cannabis a) Sodium thiosulfate b) Sodium nitrate
862. Activated charcoal is used in- (PGI 03) c) Sodium bicarbonate d) Amyl nitrate
a) Alcohol b) Barbiturate 876. Cage test is used in - (TN 04)
c) Heavy metal poisoning d) Lead poisoning a) Alcohol poisoning b) Opiate poisoning
863. Lead poisoning is characterised by- (PGI 03) c) Datura poisoning d) Cannabis poisoning
a) Diarrhoea b) Encephalopathy 877. The poison causing delirium is- (TN 04)
c) Neuropathy d) Anaemia a) Strychinine b) Dhatura
e) Constipation c) Cannabis d) Caster
864. Opium poisoning is treated with- (PGI 03) 878. The Following are features of cyanide poisoning
a) Nalorphine b) Atropine except• (SGPG/05)
c) Neostigmine d) Physostigmine a) Fixed dilated nonreacting pupil
865. Gatric lavage turns black in presence of silver b) Loss of consciounsness
nitrate, most probable poisoning was- (PGI 04) c) Areflexia
a) Celfos b) Malathion d) Normal myocardial activity
c) Organophosphorus d) Parathion 879. FoUowing is not correct about mercury poisoning-
866. Jet black tongue found in- (PGI 04) a) Highest concentration of (SGPGI 05)
a) Cocaine b) Arsenic consumed mercury is found in kidneys
c) Opium d) Bismuth b) Mercuria lentis is due to deposition of mercury
867. Kritpoisoningis- (PGI 04) c) Tremors are more common in workers of this
a) Vasculotoxic b) Neurotoxic industry
c) Cardiotoxic d)Haemotoxic d) penicillamine is not effective in its treatment
e) Vaslulotoxic 880. A new drug that reduces craving for alcohol is-
868. In a suspected case of death due to poisoning a) Naltrexone b)Buspirone (COMEDK05)
where cadaveric rigidity is lasting longer than c) Trazadone d) Phenindamine
usual, it may be a case of poisoning due to- 881. All are adulterants of heroin-
a) Lead b) Arsenic (A/ 03) a) Chalk powder b) Quinine
c) Mercury d) Copper c) Charcoal d) Fructose

857)c,e 858)a,c,d 859)a 860)a,b 86l)a,b,d,e 862)b 863)All 864)a,d 865)a 866)a 867)b 868) b 869) a,b,c
870)c 871)c 872)d 873)a 874)d 875)c 876)a 877)b 878)d 879)d 880)a 881)c
FORENSIC [ 450 ]

882. Treatment of aluminium phosphide consists of- 894. Most Poisonous Snake is- (PUNJAB 06)
a) Magnesium sulphate IV infusion (J & K 05) a) Cobra b) King Cobra
b) Acidosis correction with IV sodium carbonate c) Viper d) Krait
c) Adequate hydration with IV infusion 895. Universal antidote is made up of all except?
d) All of the above a) Tannic acid (Manipal 06)
883. A person in coma has pin point pupils. The cause b) Potassium permanganate
maybeallexcept- (APPGE05) c) Magnesium oxide
a) Barbiturate poisoning d) Activated charcoal
b) Morphine poisoning 896. Burtonian line is seen with poisoning of- (AI 07)
c) Organophosphate poisoning a) Mercury b) Lead
d) Pontine hemorrhage c) Arsenic d) Zinc
884. Xanthoproteic reaction is seen in- (Orissa 05) 897. Acrodynia is associated with- (AJ 07)
a) HN03 b) HCL acid poisoning a) Mercury b) Phenolic acid
c) H2 S04 poisoning d) Carbolic acid poisoning c) Oxalic acid d) Carbolic acid poisoning
885. All of the following poison's are dialyzable 898. Mercury will affect which part of the renal tubule -
except- (AIIMSNOV05) a) PCf b) DCT (AI 07)
a) Ethylene glycol b) Methanol c) Cf d) Loop ofHenle
c) Barbiturates d) Copper sulphate 899. The drug of choice for mushroom poisoning is-
886. In methyl alcohol poisoning there is CNS a) Atropine b) Physostigmine (AI 07)
depression, cardiac depression and optic nerve c) Adrenaline d) Carbachol
atrophy. These effects are produced due to- 900. Which ofthe following is not as aryl phosphate?
a) Formaldehyde and formic acid (AI 05) a) TIK20 b) Malathion (AI 07)
b) Acetaldehyde c) Parathion d) Follidol
c) Pyridine 901. Which is not an aryl phosphate- (Aiims May 07)
d) Acetic acid a) Parathion b)Tik-20
887. In chronic arsenic poisoning the following samples c) Malathion d) Paraoxon
can be sent for laboratory examination, except- 902. Dialysis is useful in poisoning with all of the
a) Nail clippings b) Hair samples (AI 05) following except- (Aiims May 07)
c) Bone biopsy d) Blood sample a) Methyl alcohol b) Barbiturates
888. At autopsy, the cyanide poisoning case will show c) Ethylene glycol d) Copper sulphate
the following features, except- (AI 05) 903. Hemoperfusion with charcoal is useful in poisoning
a) Characteristic bitter lemon smell with- (Aiims May07)
b) Congested organs a) Barbiturate poisoning b) Methyl alcohol
c) The skin may be pinkish or cherry red in colour c) Lithium d) Ethylene glycol
d) Erosion and haemorrhages in oesophagus and 904. Which of the following poison is detects after
stomach death is- (UP 07, 06)
889. Diffusion of oxygen at the tissue level is affected in a) Lead b) Arsenic
all the following poisonings except -(AIIMS May 05) c) Thallium d) Zinc
a) Carbon monoxide b) Curare 905. Green colored urine seen in- (PGI June 07)
c) Phosgene d) Cyanides a) Kerosene b) OPCs
890. Priapism occurs in- (AIIMS 06) c) Carbolic acid d) Paracetamol
a) Snakebite b) Ratti poisoning 906. Diagnosis of CO poisoning poste mortem-
c) Cantharide poisoning d) Arsenic poisoning a) Blood thin and red coloured (PGI June 07)
891. In OPC poisoning, OPC actions are- b) Congestion of all organs
a) Phosphorylated enzyme (PGI June 06) c) Cyanosis
b) Irreversibly inhibit cholinesterase 907. Formication is associated with abuse-
c) Oximes effective when given beyond 24 hours a) Alcohol b) Cocaine (Corned 07)
d) Atropine can't reverse in early stage c) Benzodiazipine d) Analgesics
892. "Plant Penicillin" is the term used for- 908. Which ofthe following poisons has only local action?
a) Organophosphorous compounds a) Sulphuric acid b) Carbolic acid (Corned 07)
b) Endrin c) Oxalic acid d) Phosphorus
c) Arsenious oxide 909. Copper sulphate poisoning manifests with-
d) Cyanide a) Acute hemolysis (Corned 08)
893. Milky juice obtained by incising the leaves and stalk b) High anion gap acidosis
ofthe plant and fed for female infanticide is- c) Peripheral neuropathy
a) Madar juice b) Avandi juice (COMED 06) d) Rhabdomyolysis
c) Nacpalajuice d) Ratijuice

882)d 883)a 884)a 885)d 886)a 887)d 888)a 889)b 890)c 891) a,b 892) b 893) a 894)b 895)b
896)b 897)a 898)a 899)a 900)b 90l)c 902)d 903)a 904)b 905) c 906) a,b 907) b 908)a 909)a
FORENSIC [ 451 ]

910. The poison that can he detected after death in hair 922. Gastric Lavage is contraindicated in poisoning with-
is - (Corned 08) a) Kerosene (PGI Dec 08)
a) Lead b) Copper b) Organophosphorus
c) Mercury d) Arsenic c) Arsenic
911. The following are complications of cocaine poisoning d) Morphine
except- (UPSC-I 08) e) Dhatura
a) Angina and myocardial infarction 923. A man working as a pest killer comes to OPD with
b) Epileptic seizures pain abdomen, garlic odour in breath and transverse
c) Hypothermia lines on nails. Most likely the person is having-
d) Hypertension a) Arsenic poisoning (AIIMS Nov 08)
912. Which one among the following is generally used b) Lead poisoning
as a fungicide, while others are 'generally used as c) Mercury poisoning
insecticides' ? (UPSC-I 08) d) Cadmium poisoning
a) Aldrin b)Dieldrin 924. A man working as a pest killer comes to OPD with
c) Heptachlor d) Hexachlorobenzene pain abdomen, garlic odour in breath and transverse
913. Cherry red colour postmortom appearance is seen lines on nails. Most likely the person is having:
in- (UP08) a) Arsenic poisoning (AIIMS May 09)
a) Cyanide poisoning b) Phosphorus b) Lead poisoning
c) Arsenic d) Mercury c) Mercury poisoning
914. Over doses of acetaminophen causes hepatic finding d) Cadmium poisoning
in postmortem- (UP 08) 925. Formication and delusion of persecution occurs
a) Centrilobularnecrosis together in- (AIIMS May 09)
b) Scattered patchy necrosis a) LSD b)Amphetamine
c) Liquefactive necrosis c) Canabis d) Cocaine
d) Coagulative necrosis 926. Mee's lines are seen in poisoning with-
915. Which of the following is an example of a a) Cadmium b)Lead (PGIJune09)
polychlorinated hydrocarbons- (AI 08) c) Mercury d) Arsenic
a) Parathion b) Malathion e) Thallium
c) Diazinion d) Endrin 927. plumbism is caused by- (PGI June 09)
916. Yellow fatty liver is characteristic of poisoning with- a) Lead poisoning b) Mercury poisoning
a) Arsenic b) Mercury (AI 08} c) Thallium poisoning d) Copper poisoning
e) Arseni~ poisoning
c) Phosphorus d) Oxalic acid
928. Dark browJt postmortem lividity is seen in -
917. Which of the following heavy metal poisoning may
a) Aniime poisoning (COMED 09)
cause colitis that resembles diptheritic colitis ?
b) Carbon-monoxide poisoning
a) Lead b) Arsenic (AI 08) c) Phosphorus poisoning
c) Mercury d) Copper d) Cyanide poisoning
918. A middle aged man from West Bengal presents with 929. Thechiefcompoundincannabisis- (COMED09)
paraesthesia of hands and feet, hyperkeratosis of a) 9-A-tetrahydrocannabinol
palms, lines on his nails and rain drop pigmentation. b) 6-A-tetrahydrocannabinol
The most likely causative toxin for the above c) S-A-tetrahydrocannabinol
mentioned presentation is- (AI 08) d) 4-A-tetrahydrocannabinol
a) Lead b) Arsenic 930. Tactile hallucination is seen in poisoning with-
c) Thallium d) Mercury a) Carbolic acid b) Cannabis (COMED 09)
919. Alkalinisation of urine may be done in cases of c) Cocaine d) Chloral hydrate
poisoningwith- (AI 08} 931. All the following methods are used in the detection
a) Barbiturates b) Amphetamines ofheavymetals,except- (DELHIPGFeb. 09)
c) Alcohol d) Morphine a) Neutron activation analysis
920. Gastric lavage contraindicated in poisoning by- b) Atomic absorption spectroscopy
a) Kerosene b) Morphin (PGI June 08) c) Scanning electron microscope-energy dispersive
c) Barbiturates d) Dhatura X-ray analysis
e) Cynide d) Paraffin test
921. All are organophosphorus compounds except- 932. Putrefaction is delayed in- (DELHI PG Feb. 09)
a) DDT b) HETP (Manipal 08) a) Peritonitis b) Septicemia
c) 1EPP d) OMPA c) Anasarca d) Zinc chloride poisoning

910)d 9ll)c 912)d 913)a 914)a 915)d 916)c 917)c 918)b 919)a 920)a 921)a 922)a 923)a
924)a 925)d 926)d 927)a 928)c 929)a 930)c 931)d 932)d
FORENSIC [ 452 ]

933. Kennedyphenomenondealswith-(DELHIPGFeb. 09) 945. True about role of nitrites in cyanide poisoning-
a) Evaluation of exit and entry wounds a) Deoxygenation (PGI Nov 09)
b) Evaluation ofburn wounds b) Chelation of cyanides
c) Effects of poisoning on stomach mucosa c) Methemoglobin formation
d) Estimation of fetal age d) Sulfhemoglobin formation
934. The most preferred side for collection of blood in 946. Caustic poison corrodes mucosa because of -
alcohol poisoning is- (DELHI PG Feb. 09) a) It's hygroscopic nature (AIIMS Nov 09)
b) It has glue like action
a) Femoral vein b) Radial artery
c) It has affinity for mucosa
c) Subclavian vein d) Cardiac
d) It is prograrmned to stick
935. Cholera like action is seen in poisoning by- 947. Hatter's shakes are seen in which poisoning-
a) Ricinus communis (DELHI PG Feb. 09) a) Arsenic b) Mercury (AIIMS Nov 09)
b) Calotropis gigantea c) Copper d) Lead
c) Ergot 948. A child brought with suspected ingestion, presented
d) Abrus precatorius with dry mouth, dilated pupil, difficulty in swallowing,
936. Magnesium sulphate lavage is done in poisoning delirium, dry & warm skin, the substance is-
with- (DELHI PG Feb. 09) a) Anti-cholinergic (AIIMS May 10)
a) Aniline b) Copper sulphate b) Sympathetic
c) Ethyl alcohol d) Phenol c) Cholinergic
937. Parotid swelling occurs in poisoning with : d) Alpha-blocker
a) Iodine b) Mercury (DELHI PG Feb. 09) 949. A factory worker presents with excessive salivation,
c) Thallium d) Barium blue lines on gums, tremors, disturbed personality,
938. Cobra snake venom is- (DELHI PG Mar. 09) insomnia and loss· of appetite. The most likely
a) Vasculotoxic b) Urotoxic poisoningis- (AI 10)
a) Mercury b) Lead
c) Musculotoxic d) Nephrotoxic
c) Arsenic d) Phosphorus
939. Green-coloured urine is characteristic of which
950. Aconite poisoning causes all except- (AI 10)
poisoning? (DELHI PG Mar. 09}
a) Hypersalivation b) Tingling and numbness
a) Phenol b) Copper sulphate c) Increased BP d) Chest pain
c) Barbiturates d) Lead 951. Which of the following drugs is used for
940. False snese of perception without any external narcoanalysis- (AI 10)
stimuli is a characteristic of -(DELHI PG Mar. 09) a) Atropine b) Phenobarbitone
a) Illusion b) Delusion c) Scopolamine d) Pethidine
c) Hallucination d) Impulse 952. Vesiculation can be caused by application of-
941. Tactile sensations over the body are a characteristic a) Cantheridin b)Brucine (Maharashtra10)
of which poisoning? (DELHI PG Mar. 09) c) Dhatura d) Capsicum
a) Cocaine b) Opium 953. Brown-coloured urine is seen in the- (DPG 10)
c) Cannabis d) Barbiturate a) Nitric acid poisoning
942. Hatter's shakes are present in poisoning b) Carbolic acid poisoning
by- (DELHIPGMar. 09) c) Hydrochloric acid poisoning
a) Lead b) Arsenic d) Sulphuric acid poisoning
c) Copper sulphate d) Mercury 954. Which ofthe following is a non- poisonous snake?
a) Viper b) Krait (DPG 10)
943. Which one of the following is most characteristic of
c) Sea snake d) Rat snake
Elapidae snake envenomation ? (UPSC-I 09)
955. In methyl alcohol poisoning there is CNS
a) Bleeding manifestation depression, cardiac depression and optic nerve
b) Neuro-paralytic symptoms atrophy. These effects are produced due to-(DPG 10)
c) Rhabdomyolysis a) Formaldehyde and formic acid
d) Cardiotoxicity b) Acetaldehyde
944. A 28-year-old male patient is brought to casualty in c) Pyridine
comatose state, with pin-point pupils, reduced d) Acetic acid
respiratory rate and bradycardia. What is the most 956. Priapism occurs in- (DPG 10)
likely diagnosis ? (UPSC-I 09) a) Snake bite b) Ratti Pojsoning
a) Tricyclic antidepressant poisoning c) Cantharide poisoning d) Arsenic poisoning
b) Opioid poisoning 957. Green coloured urine is seen in poisoning with-
c) Benzodiazepine poisoning a) Phenol b) Ethyl alcohol (Punjab 1 0)
d) Organophosphorus poisoning c) Ethylene glycol d) Kerosene

933)a 934)a 935)a 936)c 937)a 938)None 939)a 940)c 94l)a 942)a 943)b 944)b 945)c 946)a
947)b 948)a 949)a 950)c 95l)c ~~a ~~a ~~d ~~a ~~a 957)a
FORENSIC [ 453]

958. A person feels that grains of sand are lying under 971. Feature (s) of CO poisoning is/are- (PGI Nov. 10)
the skin or some small insects are creeping on the a) Cherry red colour and thinning of blood
skin giving rise to itching sensation; the condition b) Congestion of all organs
is seen in - (Punjab 10) c) Severe cyanosis
a) Organophosphorus poising d) Parkinsonism
b) Alcohol withdraw} e) Blister on skin
c) Cocaine poisoning 972. Gastric Lavage is contraindicated in poisoining
d) Morphine poisoning with- (PGI Nov. 10)
959. Gastric lavage turns black in the presence of silver a) Kerosene b) Organophosphorus
nitrate, most probable poisoning is- (Punjab 10) c) Arsenic d) Morphine
a) Celphos b) Malathion e) Dhatura
c) Organophosphorus d) Parathion 973. Patient is having bronchodilation, increased
960. "Hatter's shakes" are see in poisoning due to- temperature, constipation and tachycardia. The
a) Mercury b) Arsenic (Jipmer 10) diagnosis is- (AIIMS Nov 10)
c) Lead d) Copper a) Organophosphate poisoning
961. Acrodyniaisafeatureof- (Jipmer 10) b) Paracetamol poisoning
a) Iron deficiency anemia b) Mercury poisoning c) Mushroom poisoning
c) Copper poisoning d) Zinc poisoning d) Atropine poisoning
962. Colourofurineinphenolpoisoning- (Jipmer 10) 974. Aluminium phosphide poisoning- all true except-
a) Green b)Red a) Subendocardial infarction (AIIMS Nov 10)
c) Blue d) Yellow b) Produces phosphine gas
963. All are features of cocainism except- c) Oesophageal stricture
a) Hemorrhagicstroke (Jipmer 10) d) Inhibits cytochrome a oxidase
b) Seizures 975. A middle aged man comes to OPD with paraesthesia
c) Co-existing alcohol addiction of hands and feet, hyperkeratosis of palms, rain drop
d) Priapism pigmentation and transverse lines on nails. Most
964. Blindness along with metabolic acidosis occur in- likely the person is having- (AIIMS May 11)
a) Methanol poisoning (Jipmer 10) a) Arsenic poisoning b) Lead poisoning
b) Ethanol poisoning c) Mercury poisoning d) Cadmium poisoning
c) Barbiturate poisoning 976. Which of the following is an illuminous, translucent
d) Salicylate poisoning and waxy poison- (AI 11)
965. Spinal cord is preserved in the poisoning of- a) Iodine b)Arnmonium bromide
a) Alcohol b) Oleander (Jipmer 10) c) Cobra venom d) Yellow phosphorus
c) Opioid d) Strychnine 977. A patient presents to the emergency department with
966. The state of muscles in between convulsions in bluish, discolouration of conjunctiva, mucus
strychnine poisoning is~ (Jipmer 10) membranes and nails one hour after ingestion of a
a) Fasciculations b) Relaxation poison. Examination reveals tachycardia and
c) Rigidity d) Tonic contraction hypotension. Which of the following is the most
967. Sarcoidosis is mimicked by chronic _ _ likely ingested poison- (AI 11)
poisoning- (Comed 10) a) Mercury b) Arsenic
a) Mercury b)Beryllium c) Lead d) Silver
c) Aluminium d) Lead 978. Which ofthe following is commonly used as a Rave
968. All the following statements are TRUE regarding drug- (AI ll)
methyl alcohol poisoning EXCEPT- (Comed 10) a) Cannabis b) Cocaine
a) It is characterized by high anion gap acidosis c) Heroin d) Ecstasy
b) It causes ocular toxicity 979. All are true about aluminium phosphide poisoning
c) It causes high osmolal gap except- (PUNJAB 11)
d) Concurrent ethanol ingestion worsens the clinical a) Accumulation of acetylcholine at neuromuscular
features junction
969. Dimercaprol is/are used in poisoning with- b) Inhibit cytochrome oxidase
a) Ph b)Hg (PGIMay10) c) Production of phosphine
c) As d) Acetaminophen d) Metabolic acidosis
e) Iron 980. Maximum damage to esophagus is by-
970. Red brown post mortem staining is seen in- a) f4S04 (PUNJAB 11)
a) Dhatura b)Aniline (PGINov.10) b) Nitric acid
c) m d) Nitrites c) Sodium hydroxide (NaoH)
e) Cyanide d) Acetic acid

958) c 959) a 960) a 961) b 962) a 963) d 964) a 965)d 966)b 967)b 968)d 969) a,b,c 970) a,d
971)a,b,d,e 972)a 973)d 974)d 975)a 976)d 977)b 978)d 979)a 980)c
FORENSIC [ 454]

981. Colour of urine in phenol poisoning- (Jipmer 11) and pulmonary oedema. The most likely diagnosis is-
a) Red b) Blue a) Acute left ventricualr failure (UPSC 97)
c) Green d)Yellow b) Organophosporus poisoning
982. Dhatura poison contains all except- (Jipmer 11) c) Snake bite
a) Atropine b) Hyoscine d) Aluminium phosphide poisoning
c) Hyoscyamine d) Muscarine 995. Sewer gas is- (PGI 03)
983. Spinal cord is preserved in- (Jipmer 11) a) Phosgene b) ~S
a) Strychnine poisoning b) Arsenic poisoning c) C02 d) Aluminium phosphide
c) Oleander poisoning d) War gas poisoning 996. Not a test of circulation- (Orissa 04)
984. Basophilic stippling ofthe RBCs is a sensitive index a) Magnus test b) Icard test
of- (UPSCII 11) c) Winslow test d) Diaphanous test
997. Weight of a virgin uterus is- (Orissa 04)
a) Lead poisoning b) Arsenic poisoning
a) 50gm b) lOOgm
c) Asbestosis d) Silicosis
c) 200gm d) 150 gm
998. Nasal swabs preserved in- (Manipal 04)
MISCELLANEOUS
a) Drowning b) Anaphylaxis
c) Cocaine poisoning d) None
985. A civil wrong is known as- (AIIMS 80)
999. False negative hydrostatic test in a live born foetus
a) Mutatis mutandis b) Pari passu
may be seen in- (AI 08)
c) Tort d) Ultra wires
a) Atelectasis b) Artificial respiration
986. Hatter's Shake is seen in- (AI 93)
c) Emphysema d) Putrefaction
a) Arsenic b) Mercury 1000. Study of death in all its aspects is known as-
c) Phosphorus d) Lead a) Trichology b) Ornithology (AIIMS Nov 08)
987. Colour changes of putrefaction are first observed c) Odontology d) Thanatology
in the- (AIIMS 86) 1001. Constituents of a typical embalming solution are all
a) iliac fossae b) Popliteal fossae except- (AIIMS Nov 08)
c) Cubital fossae d) Arm pits a) Phenol b) Glycerine
e) Neck c) Formalin d) Ethanol
988. A congnizable offence signifies- (AI 93) 1002. The best method of injection in embalming
a) Imprisonment upto 2 yrs is - (DELHI PG Feb. 09)
b) Imprisonment upto 3 yrs+Rs. 1000 fme a) Continuous injection and drainage
c) Arrest without warrant b) Continous injection with disrupted drainage
d) Only fme upto Rs 1000 c) Alternate injection and drainage
989. Criminal responsbility in Indian constitution is d) Discontinuous injection and drainage
included in IPC section- (PGI 81, Kerala 90) 1003. Ewing's postulates deals with relationship between-
a) 84 b)300 a) Trauma and tumor (DELHI PG Feb. 09)
c) 302 d)304 b) TraumaandMI
990. Erethism is seen in which poisoning- (AIIMS 97) c) Trauma and SAH
a) Mercury b) Arsenic d) Trauma and poisoning
c) Aconite d) Phenol 1004. Which of the following organs/tissues are presently
not being used for organ/tissue transplantation.-
991. Sexual consent cannot be given by a girl
a) Blood vessels b) Lung (AI 11)
below- (TN 99)
c) Liver d) Urinary Bladder
a)15yrs b)17yrs
c) 18 yrs d) 21 yrs Recent Years NEETIDNB Questions
992. Which of the following sections of IPC are 1005. Cobras belong to- (NEETIDNB Pattern)
concerned with dowry death- (Kerala 2K) a) 'Viperidae b) Elapidae
a) 300 b) 302 c) Colubridae d) Crotalide
c) 304 d) 304-A 1006. Voyeurism is also known as- (NEETIDNB Pattern)
e) 304-B a) Scotophilia b) Priapism
993. Fibrosis is seen in a thrombus after •••. weeks- c) Frotteurism d) Onanism
a) One b)Two (AI 89) 1007. Co meal tattooing is done by- (NEETIDNB Pattern)
c) Four d) Five a) Gold chloride
994. A farmer who worked in he field whole day suddenly b) Calcium chloride
took ill and was brought to the hospital with vomiting, c) Copper sulphate
diarrhoea, salivation perspiration, constricted pupils d) Potassium permanganate

98l)c 982)d 983)a 984)a 985)c 986) b 987) a 988) c 989) a 990) a 991) a 992) e 993) b 994) b
995) b,c 996) c 997)a 998)a 999)a lOOO)d 1001)a,d 1002)d 1003)a 1004)d 1005)b 1006)a 1007)a
FORENSIC [ 455 J

1008. Angel dust is- (NEETIDNB Pattern) 1021. Medical etiquette is related to -
a) Phencyclidine b) LSD a) Le~l responsibility of physicians
c) Morphine d) Cannibis b) Ru1es to be followed by physician
1009. 1\vo identical twins will not have same- (NEETIDNB c) Laws of courtesy observed between physicians
a) Fingerprints b) Iris color Pattern) d) Moral principles
c) DNA copies d) Blood group 1022. Colour of urine in carbolic acid poisoning-
1010. La facies symapthique is seen in- a) Dark yellow b) Reddish (NEETIDNB Pattern)
a) Hanging (NEETIDNB Pattern) c) Black d) Dark green
b) Strangulation 1023. Which snake bite causes hematologic abnormalities-
c) Myocardial insufficiency a) Cobra b)Crait (NEETIDNBPattern)
d) Railway accident c) Viper d) Sea snake
1011. According to 2000 act~ juvenile is boy and girl 1024. InjurybyLathi- (NEETIDNB Pattern)
respectively less than- (NEETIDNB Pattern) a) Contusion b) Laceration
a) 16and18 b)18and21 c) Stab wound d) Abrasion
c) 14and 16 d) 18 and 18 1025. Numberofmilkteeth- (NEETIDNB Pattern)
1012. CrPC 26- (NEETIDNB Pattern) a) 16 b) 12
a) Division of offence c) 20 d) 8
b) Division of negligence 1026. In nux vomica poisoning~ posture assumed by the
c) Division of malpractice spine is - (NEETIDNB Pattern)
d) None a) Opisthotonus b) Emprosthotonus
1013. Contributory negligence is negligence due to- c) Pleurosthotonus d) band c
a) Doctor (NEETIDNB Pattern) 1027. Rule of Hasse is used to determine- (NEETIDNB
b) Patient a) Age of fetus b) Height ofadu1t Pattern)
c) Doctor and patient c) Race of person d) Identification
d) Hospital administrator 1028. Widmark's formula helps in the measurement of
1014. Best site for blood collection for toxicology sampHng- blood level of- (NEET/DNB Pattern)
a) Abdominal aorta (NEETIDNB Pattern) a) Cocaine b)Alcohol
b) Femoral vein c) Barbiturates d) Benzodiazepines
c) Carotid artery
1029. Gastric lavage is contraindicated in poisoning by-
d) Heart
a) Kerosene (NEETIDNB Pattern)
1015. Borax is- (NEETIDNB Pattern)
b) Morphine
a) Gastrointestinal irritants
b) Genitourinary irritants c) Barbiturates
c) Ecbolics d) Cyanide
d) Emmenagogues 1030. Marshalls triad is seen in- (NEETIDNB Pattern)
1016. Not a part of firearm- (NEETIDNB Pattern) a) Explosive injury b) Gunshot injury
a) Bolt b) Piston c) Drowning injury d) None
c) Extractor d)Muzzle 1031. What acts as both poison & antidote- (NEETIDNB
1017. Pupil condition in opium poisoning- (NEETIDNB a) CuS04 b)HgC12 Pattern)
a) Miosis b) Mydriasis Pattern) c) Arp3 d)Thallium arsenate
c) Irregular d) No change 1032. Yellow discoloration of skin and mucosa is seen in
1018. Methadone is used in treatment of overdose of- which poisoning? (NEETIDNB Pattern)
a) Cocaine b) Heroin (NEETIDNB Pattern} a) Nitrous oxide b) Nitric acid
c) Amphetamine d) Barbiturate c) Sulphuric acid d) Phosphoric acid
1019. What is first external sign of decomposition of dead 1033. Organs firstto be injured in blast- (NEETIDNB
body- (NEETIDNB Pattern) a) Ear, lung b) Kidney, spleen Pattern)
a) Decomposition ofliver and intestine c) Pancreas, duodenum d) Liver, muscle
b) Greenish discolouration over right iliac fossa 1034. In which ofthe following case police inquest is not
c) Greenish discolouration over dependent parts done-
d) Blood stained froth from mouth a) Dowry death b) Suicide
1020. In strychnos nux vomica poisoning- (NEETIDNB c) Death by accident d) Death by animals
a) Pt becomes unconscious immediately Pattern)
1035. Liking to wear opposite sex clothes- (NEETIDNB
b) Pt remains conscious throughout
a) Fetichism b) Masochism Pattern)
c) Pt becomes unconscious at end stage
c) Transvestism d) Sadism
d) Pt becomes unconscious in 1 hr or so

1008)a 1009)a lOIO)a lOll)d 1012)a 1013)b 1014)b 1015)a 1016)b 1017)a 1018)b 1019)b 1020)b 1021)c
1022)d 1023)c 1024)a 1025)c 1026)d 1027)a 1028)b 1029)a 1030)a 1031)a 1032)b 1033)a 1034)a 1035)c
FORENSIC [ 456 ]

1036. Cause of death for drowing in cold water- 1050. Questionnaire is to check toxicity after-
a) Vagal (NEET!IJNB Pattern) a) Alcohol b) Nicotine (NEETIDNB Pattern)
b) Asphyxia c) Opium d) Heroine
c) Loss of consciousness 1051. Law of Criminal responsibility is in which section -
d) None a) 8I b) 80 (NEETIDNB
1037. Cephalic index is used for- (NEETIDNB Pattern) c) 83 d) 84 Pattern)
a) Race b) Age 1052. Blackening of eye most common because of-
c) Sex d) Caste a) Friction abrasion (NEETIDNB Pattern)
1038. Telefono is- (NEETIDNB Pattern) b) Patterned abrasion
a) Beating on soles b) Pulling of hair c) Imprint abrasion
c) Beating on ears d) Beating on fmgers
d) Contusion
1039. Plantiffis a person who- (NEETIDNB Pattern)
1053. Incised looking laceration is seen in all except-
a) Files case in civil court b) Acts as defender
c) Gives judgement d) None a) Iliac crest (NEETIDNB Pattern)
1040. Incised looking laceration is seen at ?(NEETIDNB b) Zygomatic bone
a) Forehead b) Hand Pattern) c) Shin
c) Thorax d)Abdomen d) Chest
1041. Dirt collar or grease collar is seen in- (Jipmer 2005) 1054. In vitrous what is measured for time since death-
a) Punctured wound by sharp weapon a) Sodium (NEETIDNB Pattern)
b) Lacerated wound b) Potssium
c) Firearmentrywound c) Proteins
d) Stab wound d) Chloride
1042. A person knows the truth & wilfully concealed the 1055. In criminal cases, conduct money is paid by-
truth &gives false evidence in the court oflaw. He a) Court (NEETIDNB Pattern)
will be punished under which IPC- (NEETIDNB b) Magistrate
a) I90 b) I92 Pattern) c) Judge
~Wl ~I~ d) No conduct money is given
1043. Sexual assanltof child, hymen is not ruptured due to- 1056. DNA finger printing cannot be taken from-
a) Deep seated (NEETIDNB Pattern) a) Saliva (NEETIDNB Pattern)
b) Underdeveloped b) Tooth
c) Too tough to rupture c) Buccal mucosa
d) Distensible d) Blood
1044. Boy died in emergency room with white froth in 1057. Sexual asyphyxia is associated with-
mouth. The poisoning is due to- a) Sadism (NEETIDNB Pattern)
a) Orgnophosphorus b) Opium
b) Masochism
c) HCN d) None
c) Fetichism
1045. Burking includes- (NEETIDNB Pattern)
a) Choking b) Ligature d) Vouyerism
c) Overlaying d) Traumatic asphysia 1058. Ectopic bruise is most commonly seen in-
1046. Phossy jaw is caused by- (NEETIDNB Pattern) a) Leg (NEETIDNB Pattern)
a) White phosphorous b)Yellowphosphorous b) Eye
c) Arsenic d) Antimony c) Pinna
1047. Victim was choked with mouth & nose covered with d) Scalp
elbow around neck. It is called as--(NEETIDNB Pattern) 1059. Kennedy phenomenon is seen in-
a) Mugging b) Garrotting a) Road traffic accident (NEETIDNB Pattern)
c) Bansadola d) None of the above b) Gunshot injury
1048. Crural index is - (NEETIDNB Pattern) c) Burns
a) Length of tibia I femur x 100 d) Contusion
b) Lengthofradiuslhumerus x 100 1060. Muttering delirium is seen with- (NEETIDNB
c) Length of fibula/ tibia x I 00 a) Ricinus b) Datura Pattern)
d) Length of radius/ ulna x I 00 c) Cocaine d) Aconite
1049. Falanga is- (NEETIDNB Pattern) 1061. Treatment ofparacetamol poisoning-
a) Beating on soles with blunt object a) N acetyl cysteine (NEETIDNB Pattern)
b) Beating on soles with palms b) Activated charcoal
c) Beating on the abdomen c) Dialysis
d) Suspension by wrists d) Alkaline diuresis

1036)a I037)a 1038)c 1039)a I040)a I04I)c l042)d 1043)a I044)b I045)d 1046)a 1047)a 1048)a I049)a
1050)a 105l)d 1052)d 1053)d 1054)b 1055)d 1056)b 1057)b 1058)b 1059)b 1060)b 1061)a
FORENSIC [ 457]

1062. Police Inquest is all except- (NEETIDNB Pattern) 1075. IPC 94 states that- (NEETIDNBPattern)
a) Senior head constable can investigate a) Criminal responsibility
b) Most common inquest b) Act to which a person is compelled by threats is
c) Panchnama has to be signed by IO not offence
d) IOmaynotsummon c) Act to which a person is compelled by threats is
1063. Emysema aquosum is found in-(NEETIDNB Pattern) anoffence ·
a) Dry drowning b) Wet drowning d) Civil responsibility
c) Immersion syndrome d) Secondary drowning 1076. Pink disease or acrodynia is seen with-
1064. Post mortem caloricity is seen in- a) Arsenic b) Mercury (NEETIDNB Pattern)
a) Tetanus (NEETIDNB Pattern) c) Copper d) Lead
b) Cholera 1077. Who is called as the Father of toxicology-
c) Strychnine poisoning a) Paracelsus b) Galen (NEETIDNB Pattern)
d) All of the above c) Galton d) Guftafson
1065. Ethylene glycol when ingested affects kidney by 1078. Cognisable offence comes under which section of
forming- (NEETIDNB Pattern) Cr. P.C.- (NEETIDNB Pattern)
a) Formaldehyde b)Oxalates a) 2C b)2B
c) Isoproylalcohol d) Phosphates c) lA d)None
1066. All are cardiotoxic except- (NEETIDNB Pattern) 1079. Which ofthe following is not seen in finger prints-
a) Aconite b) Opium a) Loop b) Circles (NEETIDNB Pattern)
c) Oleander d) Nicotine c) Whorl d) Arch
1067. Whichofthefollowingretardsputrefaction? 1080. speedballiscocaineplus- (NEETIDNB Pattern)
a) Mercury b)Lead (MH 2003) a) Heroin b) Cannabis
c) Arsenic d) Copper c) Amphetamine d) LSD
1068. Gunshot residue on hands can be detected by which 1081. Brutonian lines on gums is seen in poisoning with ?
test- (NEETIDNB Pattern)
a) Lead b) Mercury (NEETIDNB Pattern)
c) Arsenic d) Zinc
a) Benzidine test
1082. In how many hours does a dead body float in India in
b) Phenolphthalein test
summer-
c) Dermal nitrate test a) 6 hours b) 12 hours
d) Hydrogen activation analysis c) 24 hours d) 48 hours
1069. Best means to identify a person-(NEETIDNB Pattern) 1083. Method ofidentification using lips? (NEETIDNB
a) Photography b) Dactylography a) Dactylography b) Podogram Pattern)
c) Anthropometry d) Handwriting c) Cheiloscopy d) None
1070. Color of postmortem lividity in hypothermic 1084. Not used for iron poisonin? (NEETIDNB Pattern)
death- (NEETIDNB Pattern) a) Penicillamine b) Desferrioxamine
a) Pwple b) Deep red c) Gastric lavage d) Magnesium hydroxide
c) Cherry red d) Bright pink 1085. IPC for criminal responsibility defining non-offence
1071. Chronic arsenic poisoning does not cause- is ? (NEETIDNB Pattern)
a) Mixed sensory & motor neuropathy a) 82 b) 182
b) Mesothelioma (NEET/DNB Pattern) c) 88 d) 188
c) Hyperkeratosis of skin 1086. Chemical name for ectasy is- (NEETIDNB Pattern)
d) Anemia a) MDMA b)MDHA
1072. In civil cases, conduct money is decided by- c) EDHA d) MDAM
a) Opposite party (NEETIDNB Pattern) 1087. Most common drug abuse in India- (NEETIDNB
b) Judge a) Cannabis b) Amphetamine Pattern)
c) Doctor c) Cocaine d) Heroine
d) Witness 1088. Dathura poisoning is characterised by-
1073. For DNA sampling which preservative is used- a) Pinpoint pupil (NEETIDNB Pattern)
a) EDTA b) Citrate (NEET!DNB Pattern) b) Dilated salivary gland
c) Dilated pupil with facial flush
c) Heparin d) Fluoride
d) Decreased temperature
1074. Frigidity is- (NEETIDNB Pattern)
1089. Stellate wound may be seen in which of the following
a) Inability to initiate sexual arousal in female bullet entry wounds- (NEETIDNB Pattern)
b) Inability to initiate sexual arousal in male a) Contact shot b) Close shot
c) Ejaculation occurring immediately after penetration c) Distance shot d) Two feet distance
d) None

1062)d 1063)b 1064)d 1065)b 1066)b 1067)c l068)c 1069)b 1070)c 107l)b 1072)b l073)a 1074)a 1075)b
1076)b 1077)a 1078)a 1079)b 1080)a 108l)a>b l082)c 1083)c 1084)a 1085)a 1086)a 1087)d 1088)c
l089)a
FORENSIC [ 458]

1090. In a3 month fetus- (NEETIDNB Pattern) 1103. Lastorgantoputrifyinmalesis-(NEET/DNBPattern)


a) Nailcomes a) Bones b) Prostate
b) Limbs well formed c) Brain d) Spleen
c) Anus is seen as dark spot 1104. Instantaneous rigor is seen in -(NEETIDNB Pattern)
d) Meconium is found in duodenum a) Cadaveric rigidity b) Rigor mortis
1091. Paltaurs hemorrhages are seen in- c) Cold stiffening d) Gas stiffening
a) Hanging (NEETIDNB Pattern) 1105. o/oofCOHb thatcancausedeath- (NEETIDNB
b) Strangulation a) 70% b) 82% Pattern)
c) Drowning c) 75% d)80%
d) Thermal injury 1106. In a case of Hanging neck ligature marks are
exampleof- (NEETIDNB Pattern)
1092. First aid should be given in acid contact- (NEETI
a) Contusion b) Pressure abrasion
a) Wash with normal water soap DNB Pattern)
c) Laceration d) Bruise
b) Wash with mild alkali agent
1107. Grievous injury is punishable under which section
c) Wash with strong alkali ofiPC- (NEETIDNB Pattern)
d) Refer to higher centre a) 354 b)320
1093. Hyoidbonefractureiscommonwith- (NEETIDNB c) 321 d)402
a) Hanging b) Strangulation Pattern) 1108. Bright red color on post martum staining found in-
c) Choking d) Throttling a) m b) HCN (NEETIDNB Pattern)
1094. Dhatura seeds resemble- (NEET/DNB Pattern) ~ H2S d)P
a) Risinus communis b) Capsicum 1109. Opiumisderivedfrom- (NEETIDNB Pattern)
c) Opium d) None a) Leaf b) Root
1095. Black papper adultrantis- c) Poppy seed d) Unripe capsule
a) Khesari da1 b) Dried papaya seed 1110. Neurotoxin is which snake- (NEETIDNB Pattern)
c) Fine sand d) None a) Viper b) Krait
1096. Amotivationalsyndromeisseenwith- (NEETIDNB c) Sea snake d) None
a) Cannabis b) Cocaine Pattern) 1111. Mechanism of cyanide poisoning is by inhibiting-
c) LSD d) None a) DNA synthesis (NEETIDNB Pattern)
1097. Hydrogen Peroxide is used in all of the following b) Cytochrome oxidase
chemical tests for blood except-(NEETIDNB Pattern) c) Protein breakdown
a) Benzidine test b) Kastle Mayer test d) Protein synthesis
c) Ortho toludine test d) Teichman test 1112. Most cop~mon hymen rupture in a virgin is-
1098. IPC 82 states that: child cannot be punished under a) Anterior (NEETIDNB Pattern)
what age- (NEETIDNB Pattern) b) Anterolateral
c) Posterolateral
a) 5yrs b) 8yrs
d) Posterior
c) 7yrs d) 1Oyrs
1113. A patient with rain drop pigmentation, which
1099. Oral evidence is more important than written
poisoningcausedit- (NEETIDNB Pattern)
testimony as- (NEETIDNB Pattern) a) Copper b) Arsenic
a) Oral evidence cannot be cross examined c) Lead d) Mercury
b) Oral evidence can be cross examined 1114. Which section belongs to marital rape- (NEETIDNB
c) Documentary evidence requires no proof a) 375A b)376A Pattern)
d) None c) 376B d)375B
1100. Knockout drops are- (NEETIDNB Pattern) 1115. Frotteruism is- (NEETIDNB Pattern)
a) Paraldehyde b) Chloral hydrate a) Sexual pleasure is obtained by witnessing the act
c) Kerosene d) Turpentine of urination
1101. Oochronosis is seen in which poisoning- b) Sexual gratification by rubbing private parts
a) Hydrochloric acid (NEETIDNB Pattern) c) Sexual practise involving three people
b) Carbolic acid d) None
c) Oxalic acid 1116. Foamy liver is seen as- (NEETIDNB Pattern)
d) Formic acid a) Adipocere b) Putrefaction
1102. Dilated pupil with coma is seen in which poisoning- c) Mummification d) Drowing
a) Opium (NEETIDNB Pattern) 1117. What test is used to check if blood is of human
b) Datura origin- (NEETIDNB Pattern)
c) Mushroom a) Takayama test b) Benzidine test
d) Pilocarpus c) Precipitin test d) Teichmann's test

1090)a 109l)c 1092)b 1093)b 1094)b 1095)b 1096)a 1097)d 1098)c 1099)b llOO)b llOl)b 1102)b 1103)a
1104)a 1105)b 1106)b 1107)b 1108)c 1109)d lllO)b llll)b 1112)c 1113)b 1114)b 1115)b 1116)b 1117)c
FORENSIC [ 459]

1118. Vasculotoxic is- (NEETIDNB Pattern) c) Persistent body spasm


a) Sea snake b) Viper d) None
c) Krait d) Cobra 1127. Lendrumstainisdonefor- (NEETIDNB Pattern)
1119. Shotgun does not contain- (NEETIDNB Pattern) a) Airembolism b)Fatembolism
a) Barrel b) Choke bore c) Amniotic fluid embolism d) None
c) Bullets d) Muzzle 1128. Barium carbonate poisoning causes-
1120. Vitriolage is used for- (NEETIDNB Pattern) a) Respiratory distress (NEETIDNB Pattern)
a) Using vitriol for suicide b) Gastrointestinal irritation
b) Using vitriol for homiside c) Muscular weakness
c) Vitriol throwing d) Cyanosis
d) Perforation of stomach caused by sulphuric acid. 1129. TrueaboutStabWounds- (NEETIDNB Pattern)
1121. Doctrine of res ipsa loquitur means- a) Depth is Greater than Breadth
a) Commonknowledge (NEETIDNB Pattern) b) Breadth is greater than depth
b) Medical maloccurance c) Length is greater than breadth
c) Fact speaks for itself d) It has wound of entry and exit
d) Oral evidence 1130. Concealment of an Infant is under which section of
1122. In medicolegal autopsy, cavity to he opened first IPC- (NEETIDNB Pattern)
is - (NEETIDNB Pattern) a) 320 b)304
a) Thoracic b) Abdomen c) 375 d)318
c) Cervical d) Any of the above 1131. Galton's system is used for- (NEETIDNB Pattern)
1123. Antidote for Strychnine poisoning is- (NEETIDNB a) Dactylography b) Poroscopy
a) Barbiturates b)Physotigmine Pattern) c) Ridgeology d) None
c) Fomepizole d) Naloxone 1132. Oximes are contraindicated in which poisoning?
1124. Cyanide odour is of- (NEETIDNB Pattern) a) Malathion b)diazinon (NEETIDNBPattem)
a) Rotten egg b) Fish c) Phorate d) Carbamate
c) Fruity d) Bitter almond 1133. Ethylene Glycol antidote is- (NEETIDNB Pattern)
1125. First treatment for snake bite is- a) Barbiturates b) Acetycysteine
a) Antisnake venom (NEETIDNB Pattern) c) Ferricchloride d)Fomipizole
b) Application of tourniquet 1134. Post mortem caloricity is seen with -
c) Clean the wound with soap & water a) Arsenic b) Lead (NEETIDNB Pattern)
d) Local incision & suction c) Datura d) Strychnine
1126. Algor mortis is - (NEETIDNB Pattern) 1135. Tetany is caused by which poisoning- (NEETIDNB
a) Rise of body temperature after death a) Oxalic acid b) Carbolic acid Pattern)
b) Cooling of body c) Sulphuric acid d) Nitric acid

***

1118)b 1119)c 1120)b 1121)c 1122)d 1123)a 1124)d 1125)b 1126)b 1127)c 1128)b 1129)b 1130)d 113l)a
1132)d 1133)d 1134)d 1135)a
ANAE SIA
ANAESTHESIA

ANAESTHETIC EQUIPMENT& 13. Pin indexsafety system what is true- (Punjab II)
a) Holes in the anaesthesia machine
1. True about Boyle's apparatus- (PGI Dec 06) b) Pin index ofNP is 2,5
a) Continuous flow machine c) Pins on the cylinder- yokes
b) Liquid anesthetic vapours not used d) Pin index of0 2 - 2, 5
c) Resistance very high 14. True about Np cylinder- (PGI 00, PGI June 04)
d) Resistance low a) Pressure is 2200 PSI b) Blue in colour
2. True about anaesthesia machine- (PGI June 08) c) Gas in liquid form d) Pin index 3 .5
a) Cylinder is a part of high pressure system e) It is flammable
b) 0 2 flush delivers< 35lit 15. All statements are true about gas cylinders except-
c) 0 2 flush delivers> 35lit a) Pressure ofNp is 745 psig at 20°C
d) Pipeline is a part of low pressure system b) Higher pressure indicates impurity in Np
3. For high pressure storage of compressed gases c) Npisinliquidform (Jipmer02,MAHE04)
cylinders are made up of- d) Emergency oxygen 'E' cylinder has more gas than
a) Molybdenum steel b) Iron + Mo 'H' cylinder
c) Steel+ Cu d) Cast iron 16. Color ofN20 cylinder is- (Maharashtra OJ)
4. Gas is filled as liquid in cylinder in- (PGI Dec 07) a) Black with white shoulders
a) 0 2 b)C02 b) White with black shoulders
c) Np d) Cyclopropane c) Grey
Halothane d) Blue
17. TrueaboutNp- (PGI June 08)
a) Pinindex3,5 b)Blueincolour
c) Stored as liquid d) MAC 105
6. An anaesthetist orders a new attendant to bring the 18. TrueaboutRotameter- (MP 06)
oxygen cylinder. He will ask the attendant to identify a) Rotameter reading may not get affected by dirt
tbe correct cylinder by following color code -(AI 03) inside the tube
a) Black cylinders with white shoulders b) Rotameter reading may not get affected by static
b) Black cylinders with grey shoulders electricity
c) White cylinders with black shoulders c) The height to which bobbin rises indicates the
d) Grey cylinder with white shoulders flow rate
7. As per ISO, colorofN20cylinderis- (MH 11) d) A rotameter is a variable pressure flowmeter
a) Blue b)Black 19. If an anaesthetist at high altitude uses plenum
c) White d)Red vaporizers. It will deliver the vapor concentration -
8. Colour of oxygen cylinder is- (AI 97) a) Higher than the original concentration at high
a) Grey b)Blue partial pressure (MP 06)
c) Black with white shoulder d) Orange · b) Lower than the concentration at lower partial
9. Pin index system is a safety feature adopted in pressure
anaesthesia machines to prevent- (AIIMS 03) c) Higher than the concentration at same partial
a) Incorrect attachment of anaesthesia machines pressure
b) Incorrect attachment of anaesthesia face masks d) Lower than concentration at same partial pressure
c) Incorrect inhalation agent delivery 20. All of the following are used to maintain proper
d) Incorrect cylinder attachment oxygen now to the patient except- (AIIMS May 08)
a) Placement of Oxygen flowmeter downstream of
the Nitrogen flowmeter
b) A proportionater between N2 and Oz control valve
c) Different pin index for nitrogen and oxygen
d) Calibrated oxygen concentration analyses
11. Truestatementregardingpin index- (PGI May 10) 21. All ofthe following are safety measures to prevent
a) Pin is present on cylinder the delivery of hypoxic gas mixture to the patient
b) Pin is present on machine except- (AIIMSMay08)
c) Not effective if wrong gas is filled in cylinder a) Location of oxygen valve after theN 0 valve
Pin index of air is 2,5 b) Presence of a PIN index system to prevent wrong
attachment of theN 0 and oxygen cylinders
c) Location of a fail-safe valve downstream from the
nitrous oxide supply source

l)a,d 2)a,c 3) a 4) b,c,d 5) a 6)a 7)a 8)c 9)d lO)a ll)b,c 12)d 13) c,d 14) b,c,d
15)d 16)d 17)a,b,c,d 18)c 19)c 20)c 21)b
ANAESTHESIA [ 496]

22. True regarding anesthesia apparatus- (MP 05) 34. RebreathingsystemsareAIE- (PG/95,SGPG/OO,
a) Rotameter reading may not get affected by dirt a) To & fro system b) Circle system AI91)
inside the tube c) Water's system d) Mapleson F
b) Rotameter reading may not get affected by static 35. The most important constituent in soda lime for
electricity reabsorption of C0 2 in a closed circuit is-
c) The Boyle machine is an intermittent flow device a) Sodium hydroxide (AI/MS May 12)
d) An oxygen failure safety device operate even in b) Barium hydroxide
the absences of nitrous oxide c) Calcium hydroxide
23. 0 is given during anaesthesia to prevent-(Jipmer 95) d) Potassium hydroxide
a)2 Pain b)Hypoxia 36. All are constituents of soda lime except- (PGI 96,
c) Hypercapnia d) Hypotension a) Ca(OH)2 b)Ba(OH)2 Jipmer2K)
24. All of the following are suitable anaesthetic circuits c) Silica d) Moisture
for both controlled and assisted ventilation except- 3 7. The constitution of soda lime is - (Delhi 96)
a) Mapleson A b) Mapleson B & C (AIIMS 03) a) 5%Ca(OHh,95%NaOH
· c) Mapleson D d) Mapleson E b) 5% NaOH, 95% Ca (OH)2
25. A 25 year old male is undergoing incision and c) 5%KoH95%NaOH
drainage of abscess under general anaesthesia with d) IOO%Ca(OH)2
spontaneous respiration. The most efficient 38. 100 gms ofsoda lime absorbs...... % of C02 -
anaesthetic circuit is - (SGPGI 05) a) 15 b) 25 (Kerala 98)
a) Maplelson A b) Mapleson B c) 150 d) 1000
U~UjJ1...,.,\JU c d) D 39. Clayton is used in a close breathing system for the
purpose of- (PGI 96, AMU 98)
a) As a hardner b) As an absorbent
c) As a softner d) As an indicator
40. Water is used for hardening in- (PGI 96, AI 92)
a) Sodalime b)Baralirne
c) Both d) None
41. Allk02 absorption in circuit except-(PG/Dec 07)
an
a) Resistance in circuit b) High flow
a) Open mask ventilation
c) Small granule size d) Medium granule size
b) Non - breathing circuit
e) Chanelling
c) Magill's system
42. Sodalime is used to absorb C02 in - (MH 09)
d) Ayre's T. Piece
a) Bain's circuit b) Magill's circuit
28. Anaesthesia breathing circuit recommended for
spontaneous breathing is- (Karn 11)
c) Closed circuit d) Jackson system
a) Mapleson A b) Mapleson B 43. Bag and mask ventilation is absolutely
c) Mapleson C d) Mapleson D contraindicated in- (Rajasthan 97)
29. In magil circuit airflow is- (AI 00, 98, MP 99) a) Tracheo-esophageal fistula with esophageal
a) \6 of minute volume b) equalto M. V. atreasia and esophagus connected to trachea
c) 2xm.v. d)3 xm.v. b) Diaphragmatic hernia
30. Bains' circuit is Mapelson type- circuit- (MH 06, c) Meconium aspiration
a) Tyep A b) Type B MP 99) 44. True about laryngeal mask airway (LMA)-
c) TypeD d) Type E a) Available in 8 sizes (PG/ June 08)
'"CAo~W![EETJJ!JNB b) Intubation can be done
a;;, !li!! <O•~ c) Size 1 for neonate
d) Size 3 for adults
True about Bain circuit- (PGI June, 09, Nov 10) e) Full protection from aspiration
a) Mapleson type B 45. Laryngeal mask airway size 2 is recommended for-
b) Mapleson type D a) <6.5kg b) 6.5-20kg (Karn 11)
c) Can be used for spontaneous respiration c) 20-30kg d)30-50kg
d) Can be used for controlled vantilation 46. Laryngeal mask airway is indicated in- (PGI OJ)
e) Coaxial a) To prevent aspiration of stomach contents
33. Anaesthetic circuit that prevents rebreathing of b) Short surgical procedure
C02- (Maharashtra 08) c) Where endotracheal intubation is contra-indicated
a) Magil's circuit b) MaplesonD circuit d) Difficult airway
c) Ayre's piece d) Circle system e) Facilitate endotracheal intubation

22)d 23)b 24)a 25)a 26)a 27)d 28)a 29) b 30) c 31) c 32) b,c,d,e 33) d 34) d 35) c
36)b 37)b 38)b 39)d 40)b 41)a,d 42)c 43) a,b,c 44) a,b,c,d 45) b 46) b,d,e
ANAESTHESIA [ 497]

47. The laryngeal mask airway used for securing the 57. During laryngoscopy and endo-tracheal intubation
airway of a patient in all of the following conditions which of the maneuver is not performed-(AIIMS 03)
except- a) Flexionoftheneck
a) Ina difficult intubation (AI 05) b) Extension of Head at the atlanto-occipitaljoint
b) In cardiopulmonary resuscitation c) The laryngoscope is lifted upwards levering over
c) In a child undergoing an elective I routine eye the upper incisors
surgery d) In a straight blade laryngoscope, the epiglottis is
d) In a patient with a large twnour in the oral cavity lifted by the tip
48. Laryngeal mask Airway (LMA) is used for- 58. Pressure necrosis which causes venous flow to stop
a) Maintenance of the airway (AIIMS 03) during endotracheal intubation occurs at-(Jipmer 01)
b) Facilitating laryngeal surgery a) 20-30mmHg b) l5-20mmHg
c) Prevention of aspiration c) 30-40mmHg d) l6-15mmHg
d) Removing oral secretions 59. Nasal intubation is contra indicated in- (AIIMS 95)
49. True about Larynegeal mask airway-(PGJ June, 09, a) CSFRhinorrhea b)Fracturecervicalspine
a) More reHable than face mask Nov 10) c) Fracture mandible d) Short neck
b) Prevent aspiration 60. Contraindication for nasotracheal intubation -
c) Alternative to Endotracheal tube (E.T.T) a) Basal skull fractures (Karn 04)
d) Does not require laryngoscope & visualisation b) Neonate
e) Can be used in full stomach c) Difficult intubation
SO. Thue about laryngeal mask airway is- (Punjab 11) d) Prolonged intubation
a) Can be used in full stomach patients 61. Merits ofnasotracheal intubation is-
b) Used as alternative to ET intubation a) Good oral hygiene (AIIMS May 07)
c) Provides full protection from aspiration b) Less infection
d) Difficult airway is an indication for use c) Less muscosal damage and bleeding
51. Wbicb of the following are used to protect airways- d) More movement or displacement of endotracheal
a) LMA b) Endotracheal tube tube
c) Ryles tube d)Combitube (PGINov09) 62. Blind nasal intubation is indicated in -(SGPGI 04)
e) Sengsten blackmore tube a) Temporomandibular joint ankylosis
52. Endotracheal tube level is- (CMC Veilore 97) b) Cervical spondylitis
a) 5 ems above xiphisternum c) Fracture mandible
b) 4 ems above camia d) Traumatic quadriplegia
c) 1 em above carnia 63. Thue about laryngoscopy & intubation-(PGIJune 04)
d) At camia a) Hypertension
53. Both oral and Nasal intubation are C/1- (AIIMS 95) b) Tachycardia
a) Laryngeal edema c) tiCT
b) CSF-Rhinorrhoea d) t Intra ocular pressure
c) Comatose patient e) -k Lower oesophageal sphincter tone
d) Acute Tracheo-Laryngo-bronchitis 64. Thue about endotracheal intubation (during the
54. Thue about endotracheal intubation is- (AI 99) process) is all except- (PGI OJ)
a) It reduces normal anatomical dead space a) Hypertension & tachycardia
b) It produces decrease in resistance to respiration b) Raised lOT
c) Subglottic edema is most common complication c) RaisediCT
d) All of the above d) Arrhythmias
55. Which of the following is not an indication for e) Increased oesophageal peristalsis
endotracheal intubation? (AI 06) 65. True about endotracheal cuff- (PGI June 04)
a) Maintenance of a patent airway a) Low-volume, high pressure
b) To provide positive pressure ventilation b) Low-volume, low pressure
c) Pulmonary toilet c) High-volume, low pressure
d) Pneumothorax d) High-volume, high pressure
56. Position for orotracheal intubation- e) Equal-volume, pressure
a) Extension of neck and atlanto-occipital joint 66. The following is true about endotracheal cuff. It has -
b) Extensionofonlyneck (Maharashtra 09) a) Low volume, high pressure (DELHI PG Feb. 09)
c) Flexion of neck only b) Low volume, low pressure
d) Flexion of neck and extension of atlanto-occipital c) High volume, high pressure
joint d) Equal volume and pressure

47)d 48)a 49)a,c,d 50)b,d 5l)b 52)b 53)a,d 54)a 55)d 56)d 57)c 58)c 59) a 60)a
61)a 62)a 63) a,b,c,d 64) e 65) a,c 66) a
ANAESTHESIA [ 498]

67. In infant (full term) diameter (mm) length (em) of 77. Fixedperformancedeviceis- (Jipmer 11)
ETTusedare- (SGPGI 01) a) Venturymask b) Nasal cannula
a)3.5,l6 b)7,12 c) Simple mask d) Nonrebreathingmask
c) 3.5,12 d)7,10 78. Which one ofthe following device provides tlxed
68. What should be the internal diameter of the performance oxygen therapy-(AHMSMay 05, UP 00)
endotracheal tube in a child weighing 1500 gm ? a) Nasal Cannula b) Venturi mask
a) 2nnn b)2.5nnn (MH 07) c) 02 by T-piece d) Edinburg mask
c) 3nnn d) 3.5 nnn 79. In venturi mask maximum 0 2 concentration attained
69. Armoured enforcement ofET tube is used in- is- (Bihar 05)
a) Neurosurgery b) Cardiac surgery (Calcutta a) 9()0/o b) 100%
c) Liver surgery d) Intestinal Surgery 2K) c)60% d)80%
70. With regard to laryngeal trauma associated with 80. Which of the following is used for oxygen therapy-
the placement of an endotracheal tube (ETT) - a) Pulse oximeter b) Nick name's catheter
a) It is most likely to occur at the thyroid cartilage c) Hudson's mask d) Guedel's airway (MH 01)
b) Trauma to the vocal cords usually involves the 81. At a flow rate of 3L/min, inspiratory (FU0 2)
right (DELHI PG Mar 09) concentration of 30-60% cau be achieved by
c) Vocal cord paralysis is caused by injury to the using a- (Jipmer 98)
anterior branch of the recurrent laryngeal nerve a) Nasopharyngeal catheter b) Simple face mask
d) Postoperative hoarseness greater than one week c) Venturimask d)Headbox
is likely to represent transient nerve injury 82. Side effects of oxygen therapy are aU except-
71. A 27 year old female was brought to emergency a) Absorption atelectasis (AIIMS May 07)
department for acute abdominal pain following which b) Increased pulmonary compliance
she was shifted to the operation theatre for c) Decreased vital capacity
laparotomy. A speedy intubation was performed but d) Endothelial damage
after the intubation, breath sounds were observed to 83. In hyperbaric oxygenation the maximum allowed
be decreased on the left side and a high end tidal pressure of02 is- (Delhi 96)
C02 was recorded. The likely diagnosis is- (AI 10) a) 3 atm b) 1 atm
a) Endotracheal tube blockage c) 5 atm d)9atm
b) Bronchospasm
84. Mac Intosh curved blade laryngoscope is a type
c) Esophageal intubation
of- (Jipmer 11)
Endobronchial intubation
a) Indirect laryngoscope b) Direct laryngoscope
c) Bronchoscope d) Video laryngoscope
85. Laryngeal spasm induced during intubation can be
prevented by the following drugs except -(UPSC 04)
a) Fentanyl b) Local anesthetic spray
c) Rofecoxib d) Diltiazem
86. Which of the following produces the least damage to
73. What is the volume of reservoir bag used in neonatal blood elements- (AIIMS Nov 04)
resuscitation- (Jipmer 98) a) Disc oxygenator b) Membrane oxygenator
a) 240ml b)500ml c) Bubble oxygenator d) Screen oxygenator
c) 750ml d) IOOOml 87. Trueaboutoxygenconcentrator- (PGIJune05)
74. All of the following are example of definite airways, a) Zeolite activation b) Delivers 0 2
except- (AI 11) c) Requires power supply d) Gives at 100%
a) Nasotracheal tube b) Orotracheal tube
c) Laryngeal Mask airway d) Cricothyroidotomy
75. Seen after tracheostomy- (AIIMS May 09)
a) Inversion ofV/P ratio
b) Increased V/P ratio PREOPERATIVE ASSESSMENT &
c) Decreased in dead space
d) Increased resistance of air flow MONITORING IN ANAESTHESIA
76. Indications of tracheostomy are- (PGI 03)
a) Flail chest b) Head injury 89. ASA classification is done for- (Rajasthan 01)
c) Tetanus d) Cardiac tamponade a) Status ofpatient b)Risk
e) Foreign body c) Pain d) Lung disease

67)c 68)c 69)a 70)a,c 71)d 72)b 73)a 74)c 75) c 76) a,b,c,e 77) a 78) b 79) c 80)c
81)a 82)b 83)a 84)b 85)c 86)b 87) a,b,c 88) c 89)a,b
ANAESTHESIA [ 499]

;:~u!~~~~f~!~~~~~K·.
90 99. Drugs commonly used in pre-anaesthetic medication-
· •.. a) Diazepam b) Scopolamine (PGI Dec 04)
~··· ·. :.l?J·Mi~F~~ · ·· itl:imacro · c) Morphine d) Succinylcholine
,. . . . ·J1~~$~~~ . ·..YJ:~!!C 100.
e) Atracurium
Glycopyrolate is better than atrophine because-
91. Malampatti grading is for- a) It crosses blood brain barrier (Jharkhand 04)
a) Mobility of cervical spine b) Not crosses blood brain barrier
b) Mobility of atlanto axial joint c) More sedation
c) Assessment of free rotation of neck before d) Antisialague
intubation 101. Atropine as preanesthesia has all effects except-
d) Inspection of oral cavity before intubation a) Decrease secretion (AI 96)
92. Plan C of anesthetic airway management is- b) Bronchoconstriction
a) Standard laryngoscopy & intubation (Mahe 05) c) Prevent bradycardia
b) Intubation catheter guided intubation d) Prevent hypotension
c) Insertion oflaryngeal mask airway & fiberoptic 102. Most potent antiemetic agent used in preoperative
bronchoscopy period- (AIIMS 96)
d) Cancel the surgery or perform tracheostomy a) Glycopyrolate b) Hyoscine
93. A patient who was on aspirin for long period was c) Atropine d) Metochlorpromide
selected for an elective snrgery. What shonld be 103. Which of the following is used as preanaesthetic
done- (AIIMS OJ) medication causes longest amnesia- (MP 98,
a) Stop aspirin for 7 days a) Diazepam b) Lorezapam DPG 09)
b) Infusion of fresh frozen plasma c) Midazolam d) Flunitrazepam
c) Infusion of platelet concentration 104. Which of the following is not a cardiovascular
d) Go ahead with surgery maintaining adequate monitoring technique- (AIIMS Nov 05)
hemostasis a) Transesophageal echocardio-graphy
94. Calcium channel blockers in anesthesia, True is- b) Central venous pressure monitoring
a) Needs to be decreased as they augment c) Pulmonary artery catheterization
hypotension & muscle relaxation (PGI 2000) . d) Capnography
b) Withheld because they lower LES pressure 105. A non ventilated preterm baby in incubator is under
c) Should be given in normal doses as they prevent observation. Which is the best way to monitor the
MI&angina baby's breathing and detect apnea- (AI 07)
d) All of the above a) Capnography
95. A Patient is on regular medications for co-existing b) Impedence pulmonometry
medical problems. Which of the following drugs c) Chest movement monitoring
may be stopped safely with minimal risk of a«!verse d) Infrared end Tidal C0 2 measurement
effects before an abdominal surgery- (AI 12) 106. Pulse oxymetry detects inaccurately in presence of-
a) Statins a) Hyperbilirubinemia (PGI June 05)
b) Bta Blockers b) Nail polish
c) ACE inhibitors/ACE Receptor Blockers c) Methemoglobinemia
d) Steroids d) Skin pigmentation
96. Preanesthetic medication is used for AlE- (PGI 02) 107. Pulse oximetry- (MP 05)
a) Decrease of anesthetic dose a) Locates the isobestic point of hemoglobin and
b) Decrease BP measure oxygen saturation
c) Prevent aspiration b) Is inaccurate in the presence offetal hemoglobin
d) Produce anmesia for peri-operative events c) Is inaccurate in the presence of caboxyhemoglobin
e) Relieve anxiety d) Measures oxygen content
97. Premedication in anaesthesia is given for- 108. Pulse oximeter is used for monitoring-
a) Decreasing the dose of anaesthetic drugs a) Oxygen content ofblood (Maharashtra 03)
b) Decreasing post-operative complications b) Oxygen saturation
c) Relieving the anxiety (MH 02) c) Partial pressure of oxygen
d) All of the above All of the above
98. Premedication is prescribed to- (SGPGI 04) ·s.~
a) Allay anxiety
b) Make the patient asleep before coming for operation
c) Reduce the dose of induction agents
d) Produce anmesia

90)d 9l)d 92) a,b 93) a 94) c 95) c 96) b 97)d 98)a,c,d 99)a,b,c lOO)b lOl)b 102)b 103)a
104)d 105)b 106) b,c,d 107) a,c 108) b 109) c
ANAESTHESIA [ 500]

110. Capnography is usefulfor- (MH 11) 119. Swan Ganz catheter measure- (PGI Dec 06)
a) Detennining Vaporizer malfunction or contamination a) PCWP
b) Determining circuit hypoxia b) c.o.
c) Determining the appropriate placement of c) Mixed venous 0 2 saturation
endotracheal tube d) Right atrial pressure
d) Detecting concentration of oxygen in the 120. Use oflntraarterial cannula in major surgery-
anesthetic circuit. a) Measurement of direct intra arterial BP (PGI 05)
111. Flat capnogram found inAJE- (AllMS May 09) b) Sample for ABG
a) Disconnection of anesthetic tubing c) Drug injection
b) Accidental extubation d) BT (Blood transfusion)
c) Mechanical ventilation failure
FUNDAMENTAL CONCEPTS
d) Bronchospasm
112. Most common cause for end tidal C02 falls due
121. The main controlling agent for respiratory drive is
to- (Jharkhand 05)
which of the following- (Mahe 07)
a) Cardiac arrest b) Extubation a) C02 b) Oxygen
c) Hypothermia d) Hyperhermia c) NO d)HB03
113. Rise in end tidal C02 during thyroid surgery can be 122. Anatomic dead space is increase by all except-
duetoallexcept- (AI 11) a) Atropine (AI 99)
a) Anaphylaxis b) Halothane
b) Malignant hyperthermia c) Massive pleural effusion
c) Thyroid storm d) Inspiration
d) Neuroleptic malignant syndrome 123. Dead space is increased by all except-
114. Which of the following may result in sudden a) Anticholinergic drugs (AIIMS May 09)
increaseinETC02- (MH 11) b) Standing
a) Malignant hyperthermia b) Hyperthyrodism c) HyperextensionofNeck
c) Shivering d) All of the above d) Endotracheal intubation
115. The most sensitive and practical technique for 124. The physiological dead space is decreased by-
detection of myocardial ischemia in the perioperative a) Upright position (AIIMS May 05)
period is- (AIIMS Nov 05) b) Positive pressure ventilation
a) Magnetic Resonance Spectroscopy c) Neckflexion
b) Radio labeled lactate determination d) Emphysema
c) Direct measurement of end diastolic pressure 125. Regarding critical temperature which of the
d) Regional wall motion abnormality detected with following is true- (PGI Dec 08)
a) Tcof02 is-ll9°C b)TcofN2 is ll9°C
the help of2D transoesophageal echocardiography
c) Tc ofN2 is 36.5°C d) Tc ofN2 0 is 36.5°C
116. All are the Complication of CVP line except-
e) Tcofairis-140.6°C
a) Airway injury b) Haemothroax (PGI June 09)
126. Which is the critical temperature ofN20- (AP 08)
c) Air embolism d) Septicemia a) 1180C b)88°C
e) Pseudoaneurysm c) 2~c d) 36.5°C
117. ImportanceofCVPmeasurementsis- (PGIOI)
a) Need for blood transfusion MUSCLE RELAXANTS
b) Assess amount of fluid to be given
c) Need for inotropic support 127. Features of depolarising neuromuscular blocking
d) Rate of intravenous fluid replacement agents are all/ except- (PGI 2000, Jipmer 99)
e) Assess need for plasma transfusion a) Cause muscle fasciculation
118. While introducing the Swan-ganz catheter, its b) No fade
placement in the pulmonary artery can be identified c) No post tetanic facilitation
by the following pressure tracing- (AIIMS Nov 05) d) Isoflurane potentiates
a) Diastolic pressure is lower in PA than in RV e) Reversed by neostigmine
b) Diastolie pressure is higher in PA than in RV 128. For non depolarization block which of the following
c) PA pressure tracing has dicrotic notch from statementis correct- (PG/99)
closure of pulmonary valve a) Post tetanic potentiation is seen
d) RV pressure tracing for plateau and sharp drop in b) Tetanic fade is absent
early diastole c) Train of four is absent
d) Anticholinergic drugs potentiation of block

llO)c lll)d 112)b 113)a 114)d 115)d 116)a 117)a,b,d,e 118)c>b 119)a,b,c,d 120)a,b,c,d 12l)a
122)c 123)d 124)c 125)a,d,e 126)d . 127)e 128)a
ANAESTHESIA [ 501 ]

129. All of the following statements about neuromuscular 141. Muscle relaxant excreted exclusively by kidney -
blockage produced by succinylcholine are true, a) Gallamine b)Atracurium (AIIMS98)
Except- (AI 10) c) Vercuronium d) Sch
a) No fade on train of four stimulation 142. Muscle relaxant contraindicated in Renal failure
b) Fade on tetanic stimulation is- (AIIMS 95, Jharkhand 04, AI 99)
c) No post tetanic facilitation a) Gallamine b)d-TC
d) Train offour ratio> 0.4 c) Vecuronium d)Atracurium
130. Train offour is characteristically used in concern
with- . (AI 08)
a) Malignant hyperthermia
b) Non-depolarizing neuromuscular blockers
c) Mechanical ventilation
d) To check hemodynamic parameters
131. M.C. nerve used for monitoring during anaesthesia-
a) Ulnar nerve b) Facial nerve
c) Radial nerve d) Median nerve
132. What is the time gap given in train offour- (AP 08) of the following can be given in hepatic as
a) lOsec b)20sec well as in renal failure- (Maharashtra 10)
c) 40 sec d) 60 sec a) Atracurium b) Vecuronium
133. Which of the following is not an amino steroid c) Pancuronium d) Mivacurium
derivative- (AP 08) 146. The ideal muscle relaxant used for a neonate
a) Alcuronium b) Vecuronium undergoing porto-enterostomy for biliary atresia is-
c) Pancuronium d) Pipecuronium a) Atracurium b)Vecuronium (AIIMS 03)
134. Which of the following is/are non-depolarizing Pancuronium d) Rocuronium
muscle relaxant(s)- (PGI Nov 09)
a) Mivacurium b) Halothane
c) Desflurane d) Isoflurane
e) Ether
135. D-tubocurarine acts by- (DPG 09) 01ce in hepati~~~a~~~ifailure­
a) Inhibiting nicotinic receptors at myoneural a) Cisatracurium b) Rocuronium (AIJMS May 07)
junction c) Vecuronium d) Rapacuronium
b) Inhibiting nicotinic receptors at autonomic 149. A child with bladder exstrophy and chronic renal
ganglion failure. The Anesthesia of choice for the child while
c) Producing depolarizing block operating exstrophy is- (AIIMS Nov 09)
d) By inhibiting reuptake of acetylcholine a) Atracurium b) Mivacurium
136. Site of action ofvecuronium is- (AIIMS 95, 96) c) Pancuronium d) Rocuronium
a) Cerebrum b) Reticular formation 150. A patient with bilirubin value of8 mgldl and serum
c) Motor neuron d) Myoneural junction creatinine of 1.9 mgldl is planned for surgery. What
137. Shortest acting non depolarizing muscle relaxant- is the muscle relaxant of choice in this patient-
a) Mevacurium (AI 98, 03, SGPGI01, a) Vecuronium b)Pancuronium (AI 10)
b) Vercuronium MH 06, 07, 08) c) Atracurium d) Rocuronium
c) Atracurium 151. A 21 year old lady with a history of hypersensitivity
d) Succinylcholine to neostigmine is posted for an elective caesarean
138. Shortest acting skeletal muscle relaxant is-(PGI 98) section under general anesthesia. The best muscle
a) Suxamethonium b) Mivacurium relaxant of choice in this patient should be- (AIIMS
c) Pancuronium d) Vecuronium a) Pancuronium b) Atracurium May 04)
139. Which of the following is the neuromuscular blocking c) Rocuronium d) Vecuronium
agent with the shortest onset of action?(AllMSM«J! 06) 152. At the end of a balanced anaesthesia technique with
a) Mivacurium b) Vecuronium non-depolarizing muscle relaxant, a patient
c) Rapacuronium d) Succinylcholine recovered spontaneously from the effect of muscle
140. Which of the following muscle relaxant has the relaxant without any reversal. Which is the most
maximum duration of action- (AI 06, MH 09) probable relaxant the patient had received- (AI 03)
a) Atracurium b) Vecuronium a) Pancuronium b) Gallamine
c) Rocuronium d) Doxacurium c) Atracurium d) Vecuronium

129)b 130)b 131)a 132) None 133) a 134) a 135) a 136) d 137)a 138)a 139)d 140)d 14l)a 142)a
143)a 144)b 145)a 146)a 147)a 148)a 149)a lSO)c 15l)b 152)c
ANAESTHESIA [ 502]

153. An ICU patient on atracurium infusion develops 167. Fasciculation are caused by- (MH 05, 02, AI 97)
seizures after 2 days. The most probable cause is- a) Suxamethonium b) Pancuronium
a) Accumulation of landonosine (AI 06) c) d-TC d) Vecuronium
b) Allergy to drug 168. Phase II block is seen in- (AIIMS 98)
c) Due to prolong infusion a) Sexamethonium(Sch) b)d-TC
154. All of the following drugs are eliminated by kidney c) Ether d)Np
except- (AIIMS Nov 04) 169. A 70 kg old athlete was posted for surgery, Patient
a) Pancuronium bromide b) Atracurium besylate was administered succinylcholine due to
c) Vecuronium bromide d) Pipecuronium unavailability ofvecuronium. It was administered in
155. as atracurium is preferred over atracurium due to intermittent dosing (total640 mg). During recovery
advantage of- (AIIMSMay II) patient was not able to respire spontaneously & move
a) Rapid onset b) Short duration of action limbs. What is the explanation- (AIIMS Nov 10)
c) No histamine release d) Less cardiodepressant a) Pseudocholinesterase deficiency increasing
156. Laudanosine is metabolite of- (AIIMSMay 11) action of syccinylcholine
a) Cisatracurium b)Atracurium b)Phase 2 blockade produced by succinylcholine
c) Pancuronium d) Gallamine c) Undiagnosed muscular dystrophy and muscular
157. Which non depolarizing agent is a ganglion blocker- weakness
a) Atracurium b)Pancuronium(AIIMSMay95) d) Muscular weakness due to fasciculation produced
c) D-TC d) Gallamine by succinylcholine
158. Cardiovascular side effects are minimal with- 170. Dual block is by- (Jharkhand 06)
a) Pancuronium b)Rocuronium(PGIDec06, 02) a) Vecuronium +pancuronium b)Atracurium
c) Doxacurium d) Vecuronium c) Vecuronium +rocuronium d) Succinylcholine
e) Mivacurium 171. The use of succinylcholine is not contraindicated
159. Agents safe in renal failure- (SGPG/2000) in- (AIIMS 02)
a) Gallamine b) Pancuronium a) Tetanus b) Closed head injury
c) Vecuronium d) Mivacurium c) Cerebral stroke d) Hepatic failure
160. All of the following drugs are eliminated by kidney 172. Hyperkalemia due to scoline is seen in all except -
except- (AIIMS Nov 04) a) Crush injury b) Burn (AIIMS 2K)
a) Pancuronium bromide b) Atracurium besylate c) Abdominal sepsis d) Muscular dystrophy
c) Vecuronium bromide d) Pipecuronium 173. Risk of adverse effect of scoline is greater in-(MH 07)
161. Succinylcholine is a muscle relaxant which act by-
a) Spinal cord injury b) Thoracic injury
a) Persistent depolarisation (UP 02)
c) Bone injury d) Head injury
b) Competitive blockade
174. Administration of scoline (Sch) produces dangerous
c) Mechanism of action uncertain
d) a and b hyperkalemia in- (AIIMS 99)
162. Which muscle relaxant increases intracranial a) Acute renal failure (ARF) b) Raised ICT
pressure ? (AIIMS 02) c) Fracture femur d) Paraplegia
a) Mivacurium b)Atracurium ·t~,
c) Suxamethonium d) Vecuronium
163. Muscle pain after anaesthesia is caused by -
a) Vecuronium (UP 96, PG/99)
b) D-tubocurare 176. The administration of succinylcholine to a
c) Suxamethonium paraplegic patient led to the appearance of
d) All dysarrhythmias, conduction abnormalities and
164. Bradycardia is common after injection of- (AllMS finally cardiac arrest. The most likely cause is -
a) Midazolam b) Succinylcholine Nov 05) a) Hypercalcemia b) Hyperkalemia (AIIMS03)
c) Dopamine d) Isoprenaline c) Anaphylaxis d) Hypermagnesemia
165. Depolarizingmusclerelaxant- (MH02) 177. In a young patient who had extensive soft tissue and
a) Scoline b) Ketamine muscle injury, which ofthese muscle relaxants used
c) Halothane d)Atracurium for endotracheal intubation might lead to cardiac
166. Effective adjuvant in attenuatating hypertension and arrest- (AIIMS 03)
tachycardia associated with laryngoscopy and a) Atracurium b) Suxamethonium
intubation- (MH 09) c) Vecuronium d) Pancuronium
a) Fentanyl b) Suxamethonium
c) Atracurium d) Ketamine

153)a 154)b 155)c 156)b>a 157)c 158)c,d 159)c,d 160)b 161)a 162)c 163)c 164)b 165)a l66)b
167)a 168)a 169)b 170)d 171)d 172)None 173)a 174)d 175)d 176)b 177)b
ANAESTHESIA [ 503]

178. All of the following statements are incorrect about 188. Which of the following statement is not correct for
the treatment of prolonged suxamethonium apnoea Vecuronium- (AIIMS 02)
due to plasma cholinesterase deficiency (after a a) It has high incidence of cardiovascular side effects
single dose of suxametbonium) except- b) It has short duration of neuromuscular block
a) Reversal with incremental doses of neostigmine c) In usual doses the dose adjustment is not required
b) Continue anaesthesia and mechanical ventilation in kidney disease
till recovery (AIIMS Nov 04) d) It has high lipophilic property
c) Transfusion of fresh frozen plasma 189. Which of the following muscle relaxants is free of
d) Plasmapheresis cardiovascular effects over the entire clinical dose
179. Abnormal plasma choline esterase produces apnea range- (MP 04)
due to which of the following- (Maharashtra 03) a) Pancuronium b) Vecuronium
a) Halothane b) Lignocaine c) Atracurium d) Pipecuronium
c) Suxamethonium d) Isoflurane 190. Mivacurium when given in high doses, all are true
180. After suxamethonium 50 mg, apnea persists for one except- (AIIMS Nov 07)
hour- (MP 05) a) Bronchospasm
a) Treatment with cholnesterase is indicated b) Hypertension
b) Probably an atypical cholinesterase is present c) Flushing
c) Treatment with stored blood is indicated d) Increase in dose increases the rapidity of onset
d) Apnea may be due to low serum potassium 191. True about d-TC is all except- (PGI 98)
concentration a) Excreted unchanged by kidney
181. Which of the following is not seen in scoline apnea? b) Causes hypotension by ganglion blocking action
a) It is due to succinylcholine (AIIMS May 12) c) Vagolytic action
b) It can be inherited d) Effects lasts for 2-3 hours
c) Patients usually do not die of scoline apnea if 192. Muscle most resistant to non depolarizing block is-
they are properly managed a) Intercostal b)Abdominal (Jipmer 03)
d) It occurs due to deficiency of cholinesterase c) Diaphragm d) Adductors
182. Succinylcholine when given by endotracheal 193. Which of the following occurs first with the use of
intnbation causes AlE- (MP 99) DTC- (TN97)
a) Hypokalemia in paraplegic patient a) Diaphragmatic relaxation
b) Prolonged apnea b) Abdominal wall relaxation
c) Raised intra ocular pressure c) Diplopia
d) Bradycardia d) Ptosis
183. Which one of the following increase the potassium 194. dTc is a- (MP 97, OJ)
level in the body- (UPSC 2K) a) Centrally acting agent
a) &tubocurarine b)Gallarnine b) Ganglionic blocker
c) Suxamethonium d) Pancuronium c) Depolarizing agent
184. Succinylcholine is short acting due to -(Jipmer 95) d) Competitive neuromuscular blocker
a) Rapid excretion b) Poor absorption 195. All of the following can be used for reversal of muscle
c) Rapid hydrolysis d) None relaxant EXCEPT- (SGPGI 05)
185. True about scoline are following except- (PGI 97) a) Neostigmine b) Atropine
a) Fasciculations c) Physostigmine d) Edrophonium
b) ICT increases 196. Only anticholinergic commonly used to reverse the
c) Non depolarising neuromuscular blocker action of neuromuscular blocker-(Maharashtra 10)
d) Short acting muscle relaxant a) Edrophonium b) Physostigmine
186. Action of scoline is potentiated by- (MH-11) c) Pyridostigrnine d) Neostigmine
a) Nitrous oxide b) Sevoflurane 197. Which ofthe following is consistent with return
c) lsoflurane d) Halothane of muscle tone adequate to protect the airway from
187. True about non-depolarizing muscle relaxants- aspiration after reversal from anaesthesia ?
a) Competitive inhibitor of acetylcholine a) TOF ratio of 0.7 (DELHI PG Mar. 09)
b) Metabolised by pseudocholinesterase b) No fade detectable on Double Burst Tetanic
c) Mg2+ predisposes the block (PGI June 05) Stimulation
d) Ca2+ antagonizes the block c) Return of normal resting minute ventilation
e) Hypothermia prolongs the block d) Five seconds sustained head lift

178)b 179)c 180)b 181)d 182)a 183)c 184)c 185)c 186)a 187) a,c,d,e 188) a,b 189) b 190) b
19l)c,d 192)c 193)c,d 194)d 195)b 196)d 197)a,d
ANAESTHESIA [ 504]

198. Neostigmine is used for reversing the adverse effect 210. NotintravenousAnaesthetic agent- (MP 97,A195)
of- (PGI97) a) Ketamine b) Thiopentone
a) dTC + pancuronium b) d TC only c) Etornidate d) Cyclopropane
c) Alcuronium only d) Ketamine complication 211. A 20 yr old patient presented with early pregnancy
199. The reversal ofneutromuscular blockade with d-T- for Medical termination of Pregnancy (MTP) in day
C is done with- (Delhi 96) care facility. What will be the anesthetic induction
a) Scoline b) Neostimine agent of choice? (AIIMS May 06)
c) Atropine d) Dantrolene a) Thiopentone b) Ketamine
200. Drugs metabolized by cholinesterase-(J & K 01, PGI c) Propofol d) Diazepam
a) Succinylcholine b) Mivacurium Dec 04) 212. Which of the following induction agent is used to
c) Esmolol d) Remifentanyl produce a 'street-fit' person following surgery-
e) Ketamine a) Midazolam b) Propofol (Jipmer 2 K)
201. Myasthenics are resistant to following muscle c) Alfentany1 d) Thiopentone
relaxant- (PGI 2000) 213. Allaretrueaboutpropofolexcept- (PGI99,
a) Suxamethonium b) Pancurium a) Pleasant sedation & recovery SGPGI 02)
c) Atracuronium d) Vecuronium b) Safe in porphyria
202. A25 year old overweight female was given fentanyl- c) Antiemetic effect
pancuronium anesthesia for surgery. Mter surgery d) Suppression of airway reflexes
and extubation she was observed to have limited e) Cardiac stimulant
movement of the upper body and chest wall in the 214. Which of the following is NOT TRUE regarding
recovery room. She was consscious and alert but PROPOFOL? (AI 08)
voluntary respiratory effort has limited. Her blood a) It is used in day care anesthesia
pressure and heart rate were normal. The likely b) It is contraindicated in porphyria
diagnosis is- (AI 10) c) Commercial preparation contains egg extract
a) Incomplete reversal ofpancuronium d) It does not cause airway irritation
b) Pulmonary embolism 215. Trueaboutpropofol- (PGI May 10)
c) Fentanyl induced chest wall rigidity a) Indicated in egg allergy
d) Respiratory depression b) Can be used in porphyria
203. In myasthenia gravis, which drugs should not be used- c) It is of barbiturate group
a) Gallamine b) Neostigmine (PGI 02) d) Used in day care surgery
c) Aminoglycosides d) Metronidazole 216. Regarding features of propofol, which of the
e) Ampicilin following statement is correct- (PGI Dec 05)
204. Which of the following muscle relaxants causes a) It suppresses adrenocortical hormone secretion
maximal pain on injection- (AI 12) b) I.M. inj. is painful
a) Succinyl choline b) Vecuronium c) Undergoes hepatic metabolism
c) Rocuronium d) Cistracurium d) Chemically it is derivative ofD-isopropyl phenol
e) Cerebral protective action
GENERAL ANAESTHESIA 217. Thueregardingpropofol- (SGPGI OJ)
a) Emetogenic
205. In normal anesthesia fresh minimum gas flows 0 2 b) Intramuscular injection is painful
delivery- (Bihar 05) c) Undergoes hepatic metabolism
a) 21% b)33% d) Cerebral protection
c) 51% d)75% 218. A severely ill patient was maintained on an infusional
206. The concentration of02 in adequate anesthesia is- anaesthetic agent On the 2nd day he started detiorating.
a) 15% b) 18% (Jipmer 95, Delhi 96) The probable culprit may be- (PGI 99, Jipmer 04)
c) 33% d)50% a) Etomidate b) Propofol
207. Oxygen content in anaesthetic mixture is- c) Opioid d) Barbiturate
a) 25% b) 30% (DELHI PG Feb. 09) 219. Thiopentone is used for induction anesthesia,
c) 33% d)38% because it is- (Jipmer 98)
208. 100% 0 2 given to wash out 94-98% N 2 in- a) Smooth induction b)Rapidlyredistributed
a) 2min b) 5 min (Orrisa 98) c) easy to adminster d) easy to monitor
c) 15min d)20min 220. Sodium Thiopentone is ultra short acting d/t-
209. Commonly used route of administration for a) Rapid absorption (DPG 09, Maharashtra 05,
general anaesthesia is- {Maharashtra 02) b) Rapid metabolism Jipmer 00)
a) Inhalational b) Intravenous c) Rapid redistribution
c) Intra-arterial d) Subcutaneous d) Rapid excretion

198)a 199) b 200)a,b,d 201)a 202)a 203)a,c 204) c 205)b 206)c 207)c 208) None>a 209)b 210)d
211)c 212)b 213)e 214)b 215)b,d 216)c,d,e 217)c,d 218)b 219)a 220)c
ANAESTHESIA [ 505]

221. Which of the following anaesthetic agent lacks 233. UseofThiopentone- (PGI Dec 04)
analgesic effect- (PGI Dec 05) a) Seizure b) Truth spell
a) Np b) Thiopentone c) Reduction ofiCP d) Cerebral protection
c) Methohexitone d) Ketamine e) Maintenance of an anesthesia
e) Fentanyl 234. Induction agent that may cause adrenal cortex
222. %of thiopentone-used in induction- (AIIMS 95) suppression is- (PGI June 06, AI 03, AP 98)
a) 0.5% · b) 1.5% a) Ketamine b) Etomidate
c) 2.5% d)4.5% c) Propofol d) Thiopentone
223. Dose of thiopentone used for induction is- e) Fentanyl
a) 1 mglkg (Jipmer 96)
235. Which of the following anaesthetic agent causes
adrenal suppression- (AIIMS Nov 10)
b) 2 mg I kg c) 5 mg I kg
a) Etomidate b) Thiopentone
d) lOmg/kg e) 15mg/kg
c) Ketamine d) Propofol
224. A patient selected for surgery was induced with
236. Following is an adverse effect ofEtomidate-
thiopentone iv through one of anticuhital vein a) Adrenocortical suppression (Karn 11)
complains of severe pain of whole hand. The next b) Hypertension
lineofmanagement- (AIIMSOJ) c) Hyperkalemia
a) Give IV propofol through same needle d) Emergence phenomena
b) IV ketamine through same needle 237. An intravenous anesthetic agent that is associated
c) IV lignocaine through same needle with hemodynamic stability, maintenence of CVP
d) Leave it alone with post operative nausea, vomiting and myoclonus-
225. If thiopentone is injected accidently into an artery a) Ketamine b) Etomidate
the first symptom is- (AI 97) c) Propofol d) Opioids
a) Analgesia b) Paralysis
Z~) .~~\{g~~~~~·~il~[·
c) Skin ulceration d) Pain

~;,.~~j~;1; ..::t;i·,~~{·;:U?;j
226. Intra arterial injection of thiopentone
causes- (AIIMS 95) ,:iDD·&··;,···.···
a) Vasospasm b) Vasodialation 23 9. Which anesthetic induction agent produces cardiac
c) Necrosis of vessel wall d) Hypotension stability. In other words cardiostable anesthetic is-
227. Pentothiol sodium should preferablay be injected a) Ketamine (MH JO,AIIMS May 09)
into- (MAHE 98) b) Propofol
a) Femoral vein b) Antecubital vein c) Thiopental
c) Neck vein d) Veinsonouteraspectofforeann d) Etomidate
228. Intravenous thiopentone pento:x, produces- (PGI 03) 240. Ketamine belongs to which of the following group of
a) Rash b)Pain drugs ? (MH 08)
c) Spasm d) Hypotension a) Phencyclidine b) Phenols
e) Muscular excitation (locally)
229. One of the following is true about the physical state c) Barbiturate· ......·.·.· ...... ·..· · ..·. d)~enzodiazepine
241~. '\\')Jichofth~follo~;~pm .
of thiopentone- (Delhi 96)
~nest~~~ia -.(UH JO, (J§~.OJ; ~2, • .. . .. ••.. > .fJ!p
a) Colorless b) Pale yellow color a) Kytari;)ine . . • ·.' ·,.,JV.EJJ.T//J.JiB:<P4~te.1:f,)
c) Green color d) Acidic solution \ J)). ..• 1..
i·?·~\''1;~; ;7',~;'J; '''i~.>f;;:;~:~*?''h:jf .;. ;;;
230. All are true about thiopentone except-
a) NaHC03 is a preservative (AIJMS Nov 07) ,i~'kntl~~!6~~(/i . C.'>,: . ':·',: ·...
b) Contraindicated in porphyria
c) Agent of choice in shock
242: ··rtcicause(fby- · , (AiirTt/d,Yii::ri9i/
a) Ketamine b) Thiopentane
d) Has cerebroprotective action c) Halothane d) Propofol
231. Thiopentone is C/1 in- (TN 00, AIIMS 97) 243. Cerebral metabolism and 0 2 consumption are
a) Acute intermittent porphyria increased by- (AIIMS Nov 07)
b) Bronchial asthma a) Propofol b)Ketamine
c) Both c) Atracurium d) Fentanyl
d) None 24-t:·'~cbQftli~ton~~g,(iftlgs!s~f)li~~~tetim~
232. Thiopentone has cerbroprotective effect because of- · . ··•· ~•. p:aii~ltf-\vjtli'tai~~~ intfa~t'.tnfal · · ~.+;, ..··.:· ·
a) Decrease cerebral metabolism (AI 12)
b) Reduction of vasospasm
.·•aJT;lllop:elit6n~:,,rnrl.ii~. .. ·........·......... ·.......•..•,·..,xt~8)
h~Pt'QpofOl~)' ·:'.•.;c:>;· ··~I·JjEF/llft'Ji!ZJ,'f~ftl/pi)
c) Calcium channel blockage ··~•. .:.~.;..···"'.·.·o.•lam·..
•·.c··.) .'."'
lVllu.tl,;l; , .····.·· ,Yi ·
,.,_ --, .'_._- .. ··· ·;§?.>-.-:.'
0 0 ·•• · '; , ·
d) Free redical removal ...·.~.ta.·
·.d·.·)·.·K·' .. ril.fne. :.':/. ~ . .;;.·: ' ·' · ~~.:'''•.
- --~:.-~--~:~ <'

221)b,c 222)c 223)c 224)c 225)d 226)a 227)d 228) a,b,d,e 229) b 230) c 231) a 232) a 233) a,b,c,d
234)b 235)a 236)a 237)b 238)a 239)d 240)a 241)a 242)a 243)b 244)d
ANAESTHESIA [ 506]

245. Maximum analgesic action found in- (AI 97)


a) Catecholamine b) Propofol
Ketamine e:t: /Y',
Ketamine is safe in -
a) RaisediCT b) Open eye injury
c) Ischemic heart disease d) Severe shock
24 7. Which of the following anaesthetic agents have good ·;iifU
analgesic property? (PGI June 06)
a) Ketamine b) Nitrous oxide i;l'f'; .c%·~·~:~.ff!'~.
c) Thiopentone d) Propofol With regard to ketamine, all of the following are
e) Midazolam trueexcept- (AIIMS Nov 05)
248. Which drug of anesthetics causes hallucination- a) It is a direct myocardial depressant
a) Ketamine (MH 06, DPG 09,AP 08, AI 98) b) Emergence phenomenon are more likely if
b) Trilene anticholinergic premedication is used
c) Halothane c) It may induce cardiac dysarythmias in patients
d) Trichloroethylene receiving tricyclic antidepressants
249. Emergence delirium is associated with- (AP 96) d) Has no effect on intracranialrw"."'"''",.,.
a) Pentothal sodium b) Ketamine
c) Droperidol d) Halothane
250. The incidence of unpleasant dreams associated
with emergence from ketamine anaesthesia can be
reduced by administration of- (COMED 09)
a) Atropine b) Scopolamine 264. Increased requirement of curare- (Jipmer 2000)
c) Physostigmine d) Diazepam a) Ketamine b) Althesin
251. Hypertension witb tachycardia is due to- (MP 98) c) Nitrous oxide d) Histotoxic hypoxia
a) Morphine b) Thiopentone 265. Site of action of ketamine is- (Bihar 04)
c) Ketamine d) Halothane a) Spinal cord
252. Increased cardiac oxygen demand is caused b) Local nerve fibers
by- (Rohtak 99) c) Primary action on thalamoneocortical projection
a) Ketamine b) Nitrous oxide d) Posterior column
c) Thiopentone d) Halothane 266. The anesthetic that affects the laryngeal &
253. Which of the following anesthetic agent does not pharyngeal reflexes minimally- (AP 97)
cause hypotension on injection? (Jipmer 11) a) Propanidid b) Methohexitane
a) Ketamine b) Propofol c) Thiopentone d) Ketamine
c) Halothane d) Thiopentone 26!~·;·:!ril.ifciJigY:~g~Jjt)~f .rua:.;.
254. Ketamine can be used in all ofthe situations except-
a) Status asthmaticus (SGPGI 2000) . ·:··:· .:~)J<... • . .·.:1·..... . l29!9t.·........ ·..........
b) For analgesia & sedation 268. Dose of ketamine by 1M route is (in mglkg) -
c) Obstetric hemorrhage a) 4 b)6 (MH OJ)
d) Ischemic heart disease c) 8 d) 10
e) Aortic aneurysm 269. A patient presented with bhmt trauma to the abdomen
255. Ketamine is contraindicated in- to the emergency department. His heart rate is
a) RaisediCT b) Hypertension 150/min and his BP is 80/50 mm Hg. He is
c) lliD d) All scheduled to undergo an emergency laparotomy.
2$~l'<~):t' Which of the following is the anaesthetic agent of
choice? (AIIMS May 12)
,.·•: ';>,'? . a) Thiopentone b) Midazolam
257. Which of the anesthetic c) Ketamine d) Propofol
contraindicated in patients with hypertension? 270. A 32-year old male with no past medical history is
a) Ketamine b) Propofol (AI 11) brought to the emergency department after being
c) Etomidate d) Diazepam injured in a bomb blast. On examination he is
258. All statements regarding ketamine are true except- suspected to have a splenic injury and is supposed
a) May be arrythmogenic (PGI 02, SGPGI 04) to undergo an emergency laparotomy. Which of
b) Raised ICT do not respond to C02 level the following is the ideal anaesthetic agent of
c) Vasodilator and negative ionotropic effect choice? (AIIMS May 12)
d) Psychomimetic emergence a) Remifentanil b) Morphine
e) Indirectly sympathetic action c) Etomidate d) Halothane

245)c 246)b 247)a,b 248)a 249)b 250)d 251)c 252)a 253)a 254)d,e 255)All 256) b 257)a 258)b
259)c 260)d 261)b 262)d 263)a 264)a 265)c 266)d 267)c 268)a 269)c 270)c
ANAESTHESIA [ 507)

271. All of the following cause myocardial depression 284. llighly lipid soluble agent would be associated with-
except- (PG!June 09) a) Potent anaesthetic action (MARE 98)
a) Halothane b) Etomidate b) Potent analgesic action
c) Thiopentone d) Ketamine c) Excellent muscle relaxant action
e) Propofol d) Least respiratory depression
272. The following anaesthetic drug causes pain on 285. Lowest concentration of anesthetic agent in alveoli
intravenous administration- (AIIMS May 06) needed to produce immobility in response to painful
a) Midazolam b) Propofol stimulus in 50% of individual is termed as-
c) Ketamine d) Thiopentone sodium a) Minimum alveolar concentration (AIIMS 98)
273. Pain during injection occurs with all except-(MH 07) b) Minimal analgesic concentration
a) Propofol b) Thiopentone c) Minimal anaesthetic concentration
c) Ketamine d) Etornidate d) Maximum alveolar concentration
274. Sufentanylis a potent- (AIIMS 97)
a) Myocardial depressant b) Analgesic
c) Anticonvulsnt d) Hepatotoxin
275. Which one ofthe following is the shortest acting
intravenous analgesic- (AIIMS 03)
a) Remifantanil b) Fentanyl 287. Minimum alveolar concentration (MAC) of an
c) Alfentanil d) Sufentanil anaesthetic agent means it produces lack of reflex
276. Morepotentopiod- (Jipmer 04) response to skin incision in ----of subjects -
a) Fentanyl b) Morphine a) 25% b)50% (MH06)
c) Sufentanyl d) Methadone c) 75% d) 100%
277. Characteristics ofRemifentanyl- (PGI Dec 07) 288. Which ofthe following has minimum MAC ?
a) Metabolised by plasma esterase a) Methoxyflurane b) Np (Bihar 05)
b) Short halflife c) Halothane d) Desflurane
c) More potent than Alfentanyl
289. MACofdesfluraneis- (DPG 09)
d) Dose reduced in hepatic and renal disease
a) 1.15 b) 2
e) Duration of action more than Alfentanyl
c) 4 d)6
2 78. Significant muscle rigidity is caused by-(Kerala 04)
290. MAC of halothane is- (DPG 08)
a) Domperidone b) Ketamine
a) 0.75% b) 1-2%
c) Halothane d) Fentanyl
c) 6% d)2%
279. Which one of the common side effects is seen with
fentanyl? (AIIMS May 06)
291. Minimum Alveolar Concentration (MAC) of
a) Chest wall rigidly b) Tachycardia sevoflurane in % is - (Karn 11)
c) Pain in abdomen d) Hypertension a) 0.75 b)0.42
280. Neuroleptanalgesia all are true except- (PGI 98) c) 1.15 d) 2
a) Can be used along with 0 2 & Np 292. All of the following factors decrease the minimum
b) Causes focal dystonia alveolar concentration (MAC) of an inhalation
c) Fentanyl-droperidol anaesthetic agent except- (AIIMS 03)
d) Causes hypotension a) Hypothermia b) Hyponatremia
e) None c) Hypocalcemia d)Anemia
281. Partition coefficient of gas- (PGI 98) 293. Exception of Meyer overton rule are AlE-
a) Measure of potency a) Nonanaesthetics (Jipmer 03, J&K 04)
b) Directly proportional to potency b) Nonimmobilizer
c) Measures solubility c) Cut off effect
d) All of the above d) Hydrophobic site
282. Index of potency of general anaesthesia- (AP 97) 294. Which of the following inhalational agents has the
a) Minimum alveolar concentration minimum blood gas solubility coefficient?
b) Diffusion coefficient (WB 06, Jharkhand 05, AIIMS May 06, AIIMS 02)
c) Dead space concentration a) Isoflurane b) Sevoflurane
d) Alveolar blood concentration c) Desflurane d) Nitrous oxide
283. The potency of an inhalational anesthetic depends 295. Least diffusion coefficientis for-(Jipmer 04, AMU 05)
on- (PG/99) a) Isoflurane b) Enflurane
a) Blood gas partition co-efficient c) Halothane d)Np
b) Oil-gas partition co-efficient 296. Least soluble anesthetic agent is- (Jharkhand 05,
c) Gas pressure a) Desflurane b) Sevoflurane TN 05)
d) Blood pressure c) Halothane d) Methoxyflurane

27l)b 272)b>d 273)c 274)b 275)a 276)c 277) a,b,c 278) d 279) a 280) e 281) a,c 282) a 283)b 284)a
285)a 286)a 287)b 288)a 289)d 290)a 291)d 292)c 293)d 294)a 295)d 296)a
ANAESTHESIA [ 508 ]

297. Fastest acting agent- (J & K 06, Jharkhand 04) 311. Definitive sign of phase I of anaesthesia
a) Sevoflurane b) Desflurane is - (JIPMER 95, DPG 09)
c) lsoflurane d) None a) Fixation of eyeball b) Pupillary dilatation
298. Which one of the following inhalational anesthetic c) Blurring of vision d) Intercostal paralysis
agents has rapid onset and rapid recovery actions 312. All are fluorinated anaesthetics except- (Jipmer 98)
and is most desirable for use day•..... case (outpatient) a) Halothane b) Isoflurane
surgical procedures- (Karn 03) c) Enflurane d) Methoxyflurane
a) Ether b) Halothane 313. True about halothane- (PGI 03)
c) Trichloroethylene d) Desflurane a) 1% thymol is used as preservative
b) It senitizes heart to catecholamines at 1 MAC
299. Which one of the following is the fastest acting
c) 20%metabolized
inhalationalagent? (AI 05)
d) It is not usually given in same patient within 3
a) Halothane b) Isoflurane
months
c) Ether d) Sevoflurane
e) It forms compound- A with sodalime
314. TrneaboutHalothane- (PGI Dec 06)
a) Non irritant
b) Antiarrhythmic
Diffusion hypoxia is seen c) It antagonizes bronchospasm
a) Induction of anaesthesia d) Vasodilator
b) Recovering anaesthesia 315. True about halothane- (PGI 02)
c) Preoperatively a) Causes bronchodilation
d) Postoperatively b) Anti-arrhthmic
302. At the end of anaesthesia after discontinuation of c) Can be used in hepatitis
nitrous oxide and removal of endotracheal tube, d) Uterine contraction occurs
100% oxygen is administered to the patient to e) Causes hepatitis
prevent- (AIIMS 03) 316. Which ofthe are the following contraindication for
a) Diffusion Hypoxia b) Second gas effect halothane used- (PGI 01)
c) Hyperoxia d) Bronchospasm a) Malesex
303. Diffusion hypoxia is seen with- (Jipmer 03) b) Middle age
a) Ether b) Cyclopropane c) Recent halothane use
c) Halothane d) Nitrous oxide d) Associated liver pathology
e) Obesity
317. In a 2 months old infant undergoing surgery for
biliary atresia, you would avoid one of the following
anaesthetic- (AIIMS 03)
a) Thiopentone b) Halothane
""''"u"'""'" were established c) Propofol d) Sevoflurane
b)N20 (AIIMS97)
318. Which of the following inhalational anaesthetic
d) Chloroform agents is contraindicated in Uver disease- (MH 06)
a) Methoxyflurane b) Halothane
c) Ether d) Isoflurane
319. The anaesthetic drug that is contraindicated in the
presence of jaundice is- (CMC 98)
a) Stage I b) Stage II a) Halothane b) Ether
c) Stage III d) Stage IV c) Gallamine d) Nitrous oxide
308. In the second stage of anaesthesia the pupil is- 320. Hepatotoxic agent is- (AIIMS 97)
a) Constricted b) Partially dilated (KA 96) a) Ketamine b) Ether
c) Normal in size d) Totally dilated c) Np d) Halothane
309. Lacrimation during anaesthesia indicates- 321. Allaretrueexcept- (AI OJ)
a) Stage! b) Stagell (CUPGEE95) a) Halothane is good analgesic agent
c) Stage III d) Stage IV b) Halothane sensitize h~art to catacholamines
310. In which plane of anaesthesia, movement of eyeball c) Halothane relaxes bronchi
is lost- (AP 96) d) Halothane causes hepatitis and liver cell necrosis
a) Stage 3, Plane 1 b) Stage 1, Plane 3 32~~·.·······.~.·~~e ~;ne a~g~t::)J~t9tll~J!e1e~~ep~@Vt?ET/pJ\TB·
c) Stage 2, Plane 3 d) Stage 4 •· ;; · '~}Iachyqardi~ .· • ·. · · · : <~tl).,~;a~p~~ti§¥jlt«tw-n)•
; ~ / ' ~
c)Bronch.pdilatl!.tion. · · li)itrterme:relaxaiion
' " " ' ' "', ; • • ... ' ' • ' • '. ' - • : ~ " ·' -. • ;' • ';__ - • ._ • >

297)b 298) d 299) d 300) d 301) b 302) a 303) d 304)b 305) a 306) b 307) c 308) b 309) c 310) a
3ll)a 312)None 313)b,c,d 314)a,c,d 315)a,e 316)c 317)b 318)None>b 319)None>a 320)d 32l)a 322)a
ANAESTHESIA [ 509]

323. Which of the following tlnorinated anaesthetics c) Desflurane has lower blood-gas partition
corrodes metal in vaporizers and breathing systems? coefficient than sevoflurane
a) Sevoflurane b) Enflurane (AIIMS May 06) d) Sevoflurane has a higher MAC than isoflurane
c) Isoflurane d) Halothane 336. Least cardiotoxic agent- (AI 98, 96)
324. Regarding the use of halothane in obsteric surgery, a) Isoflurane b) Enflurane
which is true- (CUPGEE 96) c) Sevoflurane d) Halothane
a) There is increased risk ofPPH 337. Least cardiovascular side effects found in- (AI 2000)
b) It has no effect on uterine musculature a) Isoflurane b) Enflurane
c) Best used in caesarean section c) Trilene d) Ketamine
d) None is true 338. Anesthetic of choice for cardiac anesthesia-
325. Best uterine relaxation is seen with -(DELHI PG a) Isoflurane b)Ketamine (SGPGI03)
a) Chloroform b) Nitrous oxide Feb. 09) c) Methoxyflurane d) Halothane
c) Ether d) Halothane 339. Which of the following statements regarding
326. Post operative jaundice is because of use of- destlnrane is correct? (AIIMS Nov 04)
a) Isoflurane b)NzO (PGI99) a) It causes severe myocardial depression
c) Methoxyflurane d) Halothane b) It is a structural analogue of isoflurane
327. Repeated useofhalothane causes- (AI99, PGI97) c) It has very high blood and tissue gas partition
a) Hepatitis b}Pancreatitis coefficient
c) Encephalitis d) Meningitis d) It is metabolically unstable
328. Shivering is most commonly seen with use of- 340. All are trne regarding Destlnrane EXCEPT-
a) Halothane b) Cyclopropane (KA 97) a) Low blood/gas partition coefficient (Karn II)
c) Ether d) Thiopentone b) Minimum alveolar concentration range from
329. True statement regarding halothane is- (Mahe 98) 5-7%
a) Reduces arterial pressure c) Sweet smelling agent
b) Increases heart rate d) Suitable for day care surgery
c) Decreases cardiac output 341. A seventy year old patient is posted for a surgery
d) Increases sympathoadrenal activity which is likely to last 4-6 hours. The best
330. Anaesthetic agent that predisposes to maximum inhalational agent of choice for maintenance of
arrhythmias- (MH 07, PGI 96) anesthesia in such a case is- (AIIMS May 04)
a) Isoflurance b) Enflurane a) Methoxyflurane b) Ether
c) Halothane d)Ether c) Trichlorethylene d) Desflurane
331. A six year old child is posted for elective urology 342. Which ofthe following is contraindicated in CRF-
surgery under general anesthesia. He refuses to a) Halothane b)Enflurane (MH08)
allow the anaesthesiologist an I. V. access. The best c) Desflurane d) Isoflurane
inhalational agent of choice for induction of 343. Which of the following should be considered as the
anesthesia is - (AIIMS May 04) cause of generalized convulsions 20 minntes
a) Sevoflurane b) Methoxyflurane postoperatively? (DPG 09)
c) Desflurane d) Isoflurane a) Halothane b) Enflurane
332. In a child, the anesthetic of choice is- (SGPGI OJ) c) Isoflurane d) Sevoflurane
a) Enflurane b) Isoflurane 344. Which of the following inhalational agent is
c) Sevoflurane d) Halothane contraindicated in a patient with history of epilepsy-
333. Which is C/1 in closed system anesthesiology- a) Isoflurance b) Enflurane (AIIMS 03)
a) Methoxyflurane b) Isoflurane (Bihar 05) c) Halothane d) Sevoflurane
c) Sevoflurane d) Desflurane 345. Which of the following is an epileptogenic anesthetic
334. True about Sevotlnrane- (PGI Dec 04) agent- (AI 11)
a) Isopropyl ether a) Isoflurane b) Sevoflurane
b) MACis2% c) Methoxyflurane d) Halothane
c) Good to use in old age 346. Nephrotoxic agentis- (AI 98, AIIMS 97)
d) Blood gas partition coefficient is more than a) Methoxy flurone b) Isoflurone
halothane c) Halothane d) NzO
e) Formation of compound A with baralyme 347. Fluoride content is least- (PGI 97)
335. Which ofthe following statements abont inhalation a) Methoxyflurane b) Enflurane
anesthetic agents is wrong? (AI 08) c) Isoflurane d) Sevoflurane
a) Sevoflurane is more potent than isoflurane j4ft FJ~9i'i® c!Jnte,pt.ls'leis~m~(W.ltE'T/:VNs i/?4tteinJ
b) Sevoflurane is less cardiodepressant than . . . ,. . · . .c~r~®~¥~ ~ ~,"L :)~;:;;·,·t~~;·.~.i~~);~~6~ti.r~~~ c< "·.·· :.'
1

isoflurane · ·•· · .. .Cf· ~eyo:flUiAAe / · · .· · ·.ti);ID.esfiti~e ·

323)d 324)a 325)d 326)d 327)a 328)a 329) a,c 330)c 33l)a 332)c 333)c 334) a,b,e 335) a,b
336)a 337)a 338)a 339)b 340)c 34l)d 342)b 343)b 344)b 345)b 346)a 347)c 348)d
ANAESTHESIA [51 0 ]

349. Anaesthetic agent of choice in renal failure - c) Cautery cannot be used


a) Methoxyflurane b) Isoflurane (Jharkhand 06) d) Affects blood pressure and is liable to produce
c) Enflurane d) None arrhythmias
350. A patient with alcoholic liver failure requires 363. Ether is not nsed in modern surgical practice,
general anesthesia for surgery. The agent to be used- because it is- (Kerala 97)
a) Ether b)Halothane(MH09,SGPGI03) a) Highly explosive
c) Methoxyflurane d) Isoflurane . b) Poor anaesthetic
351. Bone marrow depression is seen with- (AIIMS 96) c) Expensive
a) Halothene b) N20 d) Complicated apparatus needed
c) Ether d) Isoflurane 364. Anaesthetic agent which is explosive in the presence
352. Which anesthetic gas has high propensity to
of cautery- (Delhi PG Feb. 09)
accumulate in cavities? (Jipmer 11)
a) Nitrous oxide b) Ether
a) Halothane b) Nitrous oxide
c) Ether d) Sevoflurane c) Trilene d) Halothane
365. Maximum emesis causing anaesthetic- (Jipmer 95)
a) N20 b) Diethyl ether
c) Chloroform d) Thiopental
366. Not compatible with soda lime- (AIIMS 97)
a) Ether b) Halothane
Use of nitrous oxide is contraindicated in all ofthe c) Trilene d) Np
following surgeries except- (AIIMS Nov 08) 367. Agent which dissolves rubber- (Punjab 10)
a) Cochlear implant b) Micro laryngeal surgery a) Halothane b) Enflurane
c) Vitrioretinal surgery d) Exentration operation c) Desflurane d) Isoflurane
355. Hpneumocephalus created either by surgery or by 368. Which of the following is not true about Xenon
performance of a pneumoencephalogram, it is anaesthesia- (AIIMS Nov 06, 11)
suggested that nitrous oxide be avoided for how many a) Non explosive
days- (Jipmer 95) b) Minimal cardiovascular side effects
a) 4 days b) 5 days c) Slow induction and slow recovery
c) 6 days d) 7 days d) Low blood gas solubility
356. Anaesthesia contraindicated in volvulus of gut is- 369. Trueaboutxenonanaesthesia- (PGINov.JO)
a) Halothane (J & K OJ, CUPGEE 96) a) Rapid induction and recovery
b) Nitrous oxide b) Low potency
c) Ketamine c) High blood solubility
d) Pancuronium d) Non- explosive
357. Anesthetic agent contraindicated in Emphysema is- e) Heavier than air
a) Np b) Halothane (SGPGI OJ)
370. Anaesthetic that has a smooth induction is-
c) Ether d) Isoflurane a) Diethyl ether b) Thiopental , (DPG 09)
358. Wbich anaesthetic gas is contraindicated in the c) Np d) Halothane
presence of pneumothorax? (COMED 09) 371. Pungent volatile anaesthetic agents are-
a) Halothane b) Isoflurane a) Halothane b) Isoflurane (PGI Nov 09)
c) Nitrous oxide d) Chloroform c) Sevoflurane d) Desflurane
359. Pulmonary vasodilatation associated with - e)Np
a) Halothane b) Lignocaine (Jharkhand 05) 372. In emergency caesarian section rapid induction of
c) Isoflurane d) Ketamine anaesthesia is done to- (Jipmer 2 K)
360. Which of the following anaesthetic agents causes a a) Prevent gastric aspiration
rise in the Intracranial pressure- (AI 05) b) Prevent fetal depression
a) Sevoflurane b) Thiopentone sodium c) To decrease awareness
c) Lignocaine d) Propofol ~7~. D,~~gr~p!li4I.Jli n.~Ja1l~t~esia · ·
361. Heart is not sensitized by-
a) Halothane
c) Diethyl-ether
b) Chloroform
d) None
(MP 0 J)
'fi~~~"*~~~\~~'i> ·.
. c);,$,%~P!~~"wi~tB}!!,R<?~~1l~~~~~,: .
362. All of the following are the disadvantages of ;,~H~~f, . , · Is · · hflfbil~ftiti~::~·~1l~,\1at~~the(or~
anesthetic ether, except- (AI 05)
a) Induction is slow 374. Stress free anae~the~ia ~c~ur~ in- . (Bihar 04)
b) Irritant nature of ether increases salivary and a) Alfentanil b) Sufentanil
bronchial secretions c) Fentanyl d) Morphine

349)b 350)d 351)b 352)b 353)b 354)d 355)d 356)b 357)a 358)c 359)a>c 360)a 361)c 362)d
363)a 364)b 365)b 366)c 367)a 368)c 369)a,b,d,e 370)b 371)b,d 372)a 373)a 374)All
ANAESTHESIA [ 511 ]

375. Which of the following intravenous induction agent 386. Which of the following statements about Total Intra-
is the most suitable for day care surgery-(A! 03, 06, Venous Anesthesia (TIVA) is true- (AI 12)
~) Morphine b) Ketamine AIIMS 98, MH 09) a) Causes More Renal Toxicity
c) Propofol d)Diazepam b) Reduces Cerebral Metabolic Rate
376. Which ofthe following is the best indication for c) Risk of Malignant Hyperthermia is high
propofol as an intravenous induction agent? d) Inhibits Hypoxic Pulmonary Vasconstriction
a) Neurosurgery (Manipal 09, AI 04)
b) Day care surgery
c) Patients with coronary artery disease
d) In neonates
377. The following combination of agents are the most
preferred for short day care surgeries- (AIIMS 03)
Which of the following responses is least attc~ctE~d
a) Propofol, fentanyl, isoflurane
by Anesthesia- (AI 12)
b) Thiopentone sodium, morphine, halothane
c) Ketamine, pethidine, halothane a) Brainstem Auditory Evoked Response (BAER)
d) Propofol, morphine, halothane b) Visual Evoked Response (VER)
378. Anesthetic agent(s) safe to use in iiCP-(PGI Nov 09) c) Somatosensory Evoked potential (SSEP)
a) Halothane b) Thiopentone d) Electroencephalogram (EEG)
c) Ketamine d) Ether 389. Which of the following statements about Anesthetic
379. Bradycardia during anaesthesia seen in- Machine is true - (AI 12)
a) Pancuronium b) Vecuronium (PGI June 04) a) Temperature ofDesflurane Vaporizer Chamber is
c) Atracurium d) Propofol 39"C
b) Flow-meters (Rotameters) are Interchangeable
c) Oxygen sensors are placed on the expiratory
limb of the anesthesia circuit
d) All of the above

REGIONAL ANAESTHESIA
year o em erwent surgery.
Postoperatively she told the anesthetist that she was
aware of preoperative events. Individual
intraoperative awareness is evaluated by- (AI 11)
a) Pulse oximetry b) Colour doppler
c) Bispectral imaging d) End tidal C0 2
382. Individual intraoperative awareness is evaluated
by- (AI 12)
a) Cocaine acts by decreasing norepinephrine
a) Pulse oximetry b) Colour doppler
b) Act by decreasing sodium entry into cell
c) Bispectral imaging d) End tidal C0 2
383. A 40-year female has to undergo insicional hernia c) Lignocaine is an amide
surgery under general anesthesia. She complains d) Dibucaine is drug ofchoice for epidural anaesthesia
of awareness during her past cesarean section. 392. True statements about local anaesthesia- (PGI 03)
Which of the following monitoring techniques can a) It inhibits the generation of action potential
be used to prevent such awareness? b) Unmyelinated thin fibers are most susceptible than
a) Color doppler (AIIMS May 12) myelinated large fibers
b) Bispectral index monitoring c) Toxicity is reduced by addition of vasoconstrictor
c) Transesophageal echocardiography d) Blocks all modalities of sensation at the same time
d) Pulse plethysmography 393. In local anesthesia, which of the following is the
384. Which of the following have analgesic property ? first sequence to go- (Maharashtra 05)
a) Np b) Ketamine (PGI June 08, OJ) a) Temprature b) Parasympathetic
c) Thiopentone d.) Etomidate c) Motor d) Preganglionic sympathetic
385. Which of following is/are false- (PGI Nov. 1 0) 394. The afferent nerve fibres which are most sensitive
a) Enflurane interacts with sodalime to local anaesthetic belong to Group -
b) Sevoflurane casues seizures a) A b)B (AIIMSMay04,Al95)
c) Rapid recovery from propofol c) C d)D
d) Ketamine acts through GAB A-A receptors
e) MAC indicates potency of inhalational agents

375)c 376)b 377)a 378)b 379)d,e 380)d 38l)c 382)c 383) b 384) a,b 385) a,d 386) b 387) b 388) a
389)a 390)a 391) b,c 392) a,c 393) d 394) b
ANAESTHESIA [ 512]

395. True about local anaesthetic agents- (PGI 04) 408. Local anaesthetic acting for more than 2 hrs-(PGI 00
a) Duration depends on protein binding a) Bupivaciane b) Etodocaine SGPGI 00)
b) Potency depends upon lipid solubility c) Lignocaine d) Cholorprocaine
c) LA with low PKa is more active 409. ShortactingL.A- (PGI 2000)
d) Higher dose produces more block a) Procaine b) Lignocaine
e) Signal transduction blockade c) Bupivacaine d) Tetracaine
396. Sodium bicarbonate when given with local 410. Long acting local anestheticis- {Jipmer 11)
anaesthetics has which of the following effect - a) Procaine b) Lignocaine
a) Increases speed and quality of anaesthesia(AI 09) c) Bupivacaine d) Etidocaine
b) Decreases diffusion of the anaesthetic drug 411. Longest acting L.A.- (MH 02)
c) Causes rapid elimination of the local anaesthetic a) Bupivacaine b) Tetracaine
d) Decreases speed and quality of anaesthesia c) Xylocaine d) Procaine
412. Shortest acting local anesthetic agentis- (AI 97)
a) Procaine b) Lidocaine
c) Tetracaine d) Bupivacaine
413. Which iocal anaesthetic has highest protein binding
capacity- (MH-11)
a) Lignocaine b) Tetracaine
c) Prilocaine d) Procaine

399. From which of the following routes absorption of


local anaesthetic is maximum? (AIIMS Nov 08)
a) Intercostal b) Epidural 415. Which local anesthetic causes vasoconstriction-
c) Bronchial d) Caudal a) Cocaine b) Lidocaine (SGPGI 04)
400. Which one of the following is not an amide- (AI07, c) Bupivacaine d) Procaine
a) Lignocaine b) Procaine 98, AIIMS 03)
c) Bupivacaine d) Dibucaine
401. Which ofthe following is not an ester- (MH 08, 07)
a) Amethocaine b) Benzocaine 417. All are vasodilators except- (AI 98)
c) Procaine d) Bupivacaine a) Procaine b) Lidocaine
402. Which of the following is an ester linked local c) Cocaine d) Chlorprocaine
anaesthesia- (Kerala 98) 418. Local Anesthetic first used clinically -(AIIMS Nov 09)
a) Cocaine b) Lidocaine a) Procaine b) Lignocaine
c) Bupivacaine d) Dubicaine c) Bupivacaine d) Cocaine
403. Which of the following are not local anesthetics- ~'"~;':~'~c~er~srfi~st:Ji~~~a~'l~~~~'iifieit
· a) Bupivacaine b) Mepivacaine (PGI Nov 09) ?:v , , 11J.fiolti'f&,w~I .·
c) Mivacurium
e) Buprenorphine
d) Butorphenol
42~.· N'~tl.i;~uyocc~~~·~i~BI1~·:·~,}~:L~. ('tferala 97)
404. Which ofthe following local anaesthetics is most a) Cocaine b) Lidocaine
likely to produce an allergic reaction- (Jipmer 05) c) Bupivacaine d) Tetracaine
a) Prilocaine b) Ropivacaine 421. Pupil fails to dilate by giving cocaine in-
c) Etidocaine d) Benzocaine a) Central sympathetic paralysis (AIIMS June 2000)
405. Allergic reactions to amide linked anesthetic are- b) Central parasympathetic paralysis
a) Less common than that of ester linked local c) Peripheral parasympathetic paralysis
anesthetics d) Peripheral sympathetic paralysis.
b) More common than that of ester linked anesthetics 422. Maximum dose of plain lignocaine (in mg)-
c) Not known (Maharashtra 03) a) 300 b) 500 (Maharashtra 03)
d) None ofthe above c) 700 d) 1000
406. Cholinesterase metabolizes following except- 423. Maximum dose of lig!Jocaine with adrenaline
a) Tetracaine b) Procaine (PGI 97) (in mg/kg)- (AIIMSMay 12)
c) Acetylcholine d) Bupivacaine a) 4 b) 5
407. Which of the following is long acting local anesthetic c) 7 d) lO
> 2hrs- (PGI Dec 05) 424. Concentration ofHgnocaine used- (PGI Dec 07)
a) Bupivacaine b) Prilocaine a) 2% b)4%
c) Etidocaine d) Dibucaine c) 5% d) 10%
e) Tetracaine e) 1%

395)All 396) a 397) b 398) b 399) a 400)b 401)d 402)a 403)c,d,e 404)d 405)a 406)d 407) a,c,d,e
408) a,b 409) a 4lO)c,d 41l)a,b 412)a 413)b 414)d 415)a,c 416)a 417)c 418)d 419)a 420)a 421)d
422)a 423)c 424) a,b,c,e
ANAESTHESIA [ 513]

425. AllaretrueaboutUgnocaineexcept- (JH 05) 435. Lignocaine in high dose produces- (PGI June 04)
a) Most commonly used anesthetic agent a) Convulsion b) Respiratory depression
b) Effective for 45-60 minute c) Hypotension d) Cardiac arrest
c) Safe for person prone to malignant hyperthermia e) Hypothermia
d) None 436. Bupivacainetoxicitytreatedwith- (PGI Dec 97)
426. About lidocaine, all are true except- (PGI 98) a) Esmolol b) Epinephrine
a) LA effect b) Cardiac arrhythmia c) Lignociaine d) 5 percent dextrose
c) Ester d) Acts on mucous membranes e) Benzodiazepines
427. Cardiac or central nervous system toxicity may 43 7. In acute convulsions due to toxicity to local anaesthetic
result when standard lidocaine doses are most important step in immediate management is-
administered to patients witb circulatory failure.
a) Secure airway (Kerala 95)
This may be due to the following reason- (AI 03)
b) Adrenaline
a) Lidocaine concentration are initially higher in
c) IV short acting barbiturate
relatively well perfused tissues such as brain and
heart d) Diazepam IV
b) Histamine receptors in brain and heart gets 438. Least useful treatment of local anesthetic induced
suddenly activated in circulatroy failure convulsions- (Maharashtra 08)
c) There is a sudden out-burst ofrelease of adreneline, a) Phenytoin b) Thiopentone
noradreneline and dopamine in brain and heart c) Midazolam d) Propofol
d) Lidocaine is converted into a toxic metabolite due 439. Treatment ofbupivacaine toxicity includes-
to its longer stay in liver a) Isoprenaline b) Epinephrine (PGI Nov. 10)
428. Which one of the following local anesthetic is highly c) Bretylium d) Metoprolol
cardiotoxic- (AIIMS May 05) e) Lignocaine
a) Lignocaine b) Procaine 440. A 25 year old male with roadside accident underwent
c) Mepivacaine d) Bupivacaine debridement and reduction of fractured both bones
429. A 30 year old lady is to undergo surgery under right forearm under axillary block. On the second
intravenous regional anesthesia for her left 'trigger postoperative day the patient complained of
rmger'. Which one of the following should not be persistent numbness and paresthesia in the right
used for patient? (A/ISM May 04) forearm and the hand. The commonest cause of this
a) Lignocaine b) Bupivacaine neurological dysfunction could be all of the following
c) Prilocaine d) Lignocaine+ ketorolac except- (AI 04)
430. Correct statement regarding bupivacaine includes- a) Crush injury to the hand and lacerated nerves
a) Less carditoxic than prilocaine (PGI Dec 05) b) A tight cast or dressing
b) It is an amide c) Systemic toxicity oflocal anaesthetics
c) The maximum tolerable dose is 8 mglkg body wt
d) Tourniquet pressure
d) Duration more than 2 hrs
441. The topical use offollowing local anesthetic is not
431. Levo-bupivacaineis administered by which ofthe
recommended? (AIIMS 02)
following route- (PGI June 04)
a) Nasogastric b) Epidural a) Ligocaine b) Bupivacaine
c) Intravenous d) Intra-theccal c) Cocaine d) Dibucaine
e) Oral
432. The following statements aboutBupivacaine are true
except- (WB 96)
a) Must never be injected into a vein 443. Which among the following is NOT used for surface
b) More cardiotoxic than lignocaine anaesthesia - (COMED 09)
c) 0.25 percent is effective for sensory block a) Lignocaine b) Procaine
d) Long acting drug c) Tetracaine d) Benzocaine
It produces methaemaglobinaemia

a) Can be used for intubation


434. Earliest sign of systemic absorption of local b) Mixture oflocal anesthesia
anaesthetic is- (COMED 09) c) Faster acting
a) Convulsions b) Circulatory collapse d) Used in children
c) Circumoral numbness d) Respiratory arrest

425)c 426)c 427)a 428)d 429)b 430)b,d 431)b 432)e 433)b 434)c 435) a,b,c,d 436) b,e 437) a
438)a 439) b,c 440) c 441)b 442)c 443)b 444)a 445)b,d
ANAESTHESIA [ 514]

"' ~ ,~ir~~·i,.,Wi~tJit!iY~
4 457. Structure (s) pierce in Lumber spinal puncture
is/are - (PGI June 09) '
•• a) Ligamentum flavum
. c}. J.;id()~aine f B.upivt,~.c'ai,w:l f't'; i·: ' b) Duramater
·... ;~'iia~,#~iY:~~~~~r:f,~t · · '~£i¥a~±~'i·1:·~a?: c) Supraspinous ligament
d) Anterior longitudinal ligament
44 7. Interscalene approach to brachial plexus block does
not provide optimal surgical anaesthesia in the area e) Posterior longitudinal ligament
458. Which of the following nerves are affected after
of distribution of which of the following nerve-
spinal anesthetic block- (Maharashtra J 0)
a) Musculocutaneous b) Ulnar (AI03, OJ) a) Motor and sensory only
c) Radial d) Median b) Sensory and autonomic nerve only
448. For reduction of shoulder one. of the following c) Sensory nerves only
technique is appropriate- (MP 05) d) Sensory, autonomic and motor nerves
a) Spinal anesthesia b) lnterscalene block 459. Spinal anesthesia should be injected into the space
c) Axillazy brachial block d) Bier block between- (AI 97)
449. Which of the following is not a sign of successful a)T 12 -L 1 b)L 1 -L2
stellate ganglion block- (AI 09) c) LrL4 d)L5 -S 1
a) Nasal stuffness b) Guttman sign 460. Spinal anesthesia is given at which of the following
c) Homer's syndrome d) Bradycardia levels-
450. A successful stellate ganglion block can produce- a) Ll-2 b) L2-4 (AI 11)
a) Hypotension (Kerala 97) c) Midline ofthorax d) Below L5 (caudal)
b) Homer's syndrome 461. In high spinal anaesthesia what is seen-
c) Brachial plexus involvement a) Hypotension & Bradycardia (Kerala 00,
d) Hemifacial anaesthesia b) Hypotension & Tachycardia AIIMS OJ)
c) Hypertension & Bradycardia
451. Most common complication of celiac plexus block-
d) Hypertension & Tachycardia
a) Pneumothorax (AIIMS May 09)
462. Following spinal subarachnoid block a patient
b) Postural hypotension develops hypotension. This can be managed by the
c) Retroperitoneal hemorrhage following means except-
d)Intra-arterialinjection a) Lowering the head end (AIIMS 03)
452. The anesthetic drug injected for paravertabral block b) Administration of I 000 ml ofRingers lacate before
is least likely to diffuse to- (AI 09) the block
a) Epidural space c) Vasopressor drug like methoxamine
b) Subarachnoid space d) Use of ionotrope like dopamine
c) Intercostal space 463. Best way to prevent hypotension during spinal
d) Superior and inferior paravertebral spaces anesthesia- (AI 95)
453. A patient in ICU was on invasive monitoring with a) Preloading with crystalloids
intra arterial canulation through right radial artery b) Mephentermine
for last 3 days. Later he developed swelling and c) Dopamine
discolouration of right hand. The next line of d) Trendelenburg's position
managementis- (UP 03, AIIMS OJ) 464. Vasopressor of choice in hypotension produced
a) Stellate ganglion block b) Paravertebral block during sub-arachnoid- (PGI Dec 04, SGPGI 00)
a) Ephedrine b) Mephenteramine
c) Radial Nerve block d) Celiac plexus block
c) Adrenaline d) Dopamine
454. For Bier's block drug of choice is- (MP 00) e) Steroids
a) Prilocaine b) Bupivacaine 465. Post dural puncture headache, true about-
c) Lignocaine d) None a) Common in elderly (PGI June 04, PGI OJ)
455. In spinal anesthesia, the needle is piereced upto - b) Small bore needle prevents it
a) Subdural space (Maharashtra OJ) c) Early ambulation increases incidence
b) Extradural space d) Occurs immediately after spinal anaesthesia
c) Epidural space e) Blood patch is the first line of treatment
d) Subarachnoid space 466. Best treatment for post dural puncture headache-
456. In spinal anesthesia, the drug is deposited between- a) Caffeine b) NSAIDs
a) Dura and arachnoid (DELHIPGFeb. 09) c) Extradural blood patch d) IV fluid
b) Pia and arachnoid 467. Post spinal headache is due to- (AI 95)
c) Dura and vertebra a) Meningitis b) Encephalitis
d) Into the cord substance c) CSF leak d) Increased ICT

446)b 447)b 448)b 449)d 450) b 451) b 452) b 453) a 454) c >a 455) d 456) b 457) a,b,c 458) d
459)c 460)b 46l)a 462)b 463)None>a 464)a 465)b,d 466)c 467)c
ANAESTHESIA [ 515]

480. Epidural narcotic is preferred over epidural LA


because it causes- (AIIMS 96)
a) Less respiratory depression
b) Less dose is required
c) No motor paralysis
. .:"- " ·..~..,.:·.
d) No retention of urine
Cranial nerve not involved in 481. True about epidural anaesthesia in pregnancy -
a) }Stand lOth b)3rd and 6th (Rohtak 99) a) Given through subarachnoid space (PGI 03)
c) 2nd and 4th d) 7th and 8th b) Increases cardiac output
4 70. A patient was admitted epidural anaesthesia with 15 c) Decreases venous return
ml of 1.5%. Lignocaine with adrenaline for hernia d) Venous pooling
surgery. He devoled hypotension and respiratory e) Decreased placental circulation
depression within 3 minutes after administration of 482. Saddle block is used for- (MP 03)
block. The most common cause would be- a) Surgery oflower limbs
a) Allergy to drug administered (AIIMS May 05, 03) b) Upper abdominal surgery
b) Systemic toxicity to drug administered c) Inguinal hernia repair
c) Patient got vasovagal shock d) Perianal surgery
d) Drug has entered the subarachnoid space 483. In all of the following conditions neuraxial blockade
4 71. An anesthesia resident was giving spinal anesthesia is absolutely contraindicated, except- (AI 03)
when the patient had sudden aphonia and loss of a) Patient refusal
consciousness. What could have happended? (AI 11) b) Coagulopathy
a) Total spinal b) Partial spinal c) Severe hypovolemia
c) Vasovagal attack d)Intravascularinjection d) Pre-existing neurological deficits
472. Tuohy needle is used for- (MH 02) 484. Subarachnoid block as anesthesia is contraindicated
a) Spinal anesthesia b) Epidural anesthesia in- (KA 98)
c) CSF tapping d) Biopsy a) Diabetic gangrene b) Burgers disease
4 73. Epidural anesthesia decrease risk of- (UP 2K) c) Atherosclerotic gangrene d) Hemophilia
a) Headache b) Hypotension 485. Contraindications to epidural analgesia include the
c) Meningitis d) Arachnoiditis followingEXCEPT- (UPSC-II09)
474. All of the following are advantage of epidural a) Raised intracranial pressure
anaesthesia over spinal anaesthesia except- b) Coagulopathy
a) Dural puncture is avoided (Orissa 98) c) Infection over the back
b) Chances of meningitis are very low d) Hypertension
c) Post spinal headache is avoided 486. Contraindication of spinal anesthesia is- (DPG 08)
d) Its duration of action is very short a) Hypertension b) Renal disease
475. True about Epidural anesthesia- (PGI May 10) c) Clotting disorders d) Diabetes or,
a) Effects start immediately 487. Centrineuraxial (spinal and epidural) anaesthesia
b) C/1 in coagulopathies is not contraindicated in- (AI 07)
c) Given in subarachnoid space a) Platelets< 80,000
d) Venous return decreases b) Patient on aspirin
476. In epidural anaesthesia all are used except- c) Patient on oral anticoagulants
a) Pentazocine b) Buprenorphine (Bihar 04) d) Patient on LV.UFH
c) Bupivacaine d) Morphine, fentanyl 488. A young boy has sickle cell trait. Which of the
477. Which cant be given by Epidural anaesthesia- following anaesthesia is contraindicated ?
a) Morphine b) Remifentanil (NEETIDNB a) IV regional anaesthesia (AIIMSMay 10)
c) Alfentanil d) Fentanyl Pattern) b) Brachial plexus block by supraclavicular approach
478. Site of action of epidural analgesia- (AIIMS 98) c) Brachial plexus block by infraclavicular approach
a) Sensory nerve ending d) Brachial plexus block by axillary approach
b) Ventral hom 489. Which anaesthetic modality is to be avoided in sickle
c) Substantia gelatinosa cell disease- (AI 11)
d) Cortex a) General anaesthesia
479:-·Ma:·· ·u~.conc~nt,;aJioJifor'epidurai ~loc~··- . · b) Brachial plexus block
aJ:zJ3 ''' a:fiig ··.~''il)LfdodiiJlf':• , 'JJ./li;f!!IDNIJ, c) I. V. Regional Anaesthesia
c) ·· · d) Chlorproc~ine ·• · · '/j(lttern} d) Spinal

468) c 469) a 470) d 471)c 472)b 473) a,c,d 474) d 475) b,d 476) None 477)b 478)c 479)d 480)c
481) c,d 482) d 483) d 484)d 485)d 486) c 487) b 488) a 489) c
ANAESTHESIA [ 516 J

490. Anesthetic agent with vasoconstriction is


contraindicated in? (AI II)
a) Finger block b) Spinal block
c) Epidural block d) Regional anesthesia

COMPLICATIONS OF ANAESTHESIA
502. All the following cause malignant hyperpyrexia
491. Mendelson syndrome is due to- (PGI 98) except- (Karnataka 05)
a) Aspiration pneumonitis a) N20 b) Halothane
b) Chemical pneumonitis c) Methyoxyflurane d) Isoflurane
c) Oesophagitis 503. End-tidal C02 is increased to maximum level in-
d) Oesophageal spasm a) Pul. embolism (PGI 2000)
492. The following are true of Mendelson's b) Malignant hyperthermia
syndrome- (Delhi PG Mar. 09) c) Extubation
a) Critical volume of aspirate is 50 mls d) Blockage of secretion
b) Critical pH of gastric aspirate is 1.5 504. All are seen in malignant hyperthermia except-
c) Onset of symptoms generally occurs within 30 a) Bradycardia (AIIMS May 07)
minutes b) Hyperkalemia
d) Steroids have been shown to improve outcome c) Metabolic acidosis
493. True about aspiration pneumonia- (PGI June 04) d) Hypertension
a) Affected by volume of aspiration 505. Treatment of malignant Hyperthermia includes-
b) Affected by pH of aspiration fluid a) Dantrolene (PGI Dec 08)
c) Occurs during induction b) Cooling
d) Inflammation c) Deepening plane of inhalational anaesthesia
e) Infection d) Discontinue inhalational anaesthesia
494. Sallick's manouvere is used for-(PGI 09, (AIIMS 97) e) GiveO therapywith 100%0.
a) To prevent gastric aspiration 506. About malignant hyperthermialrue- (PGI June 08)
b) To facilitate Respiration a) Succinylcholine & Halothane predisposes
b) Dantrolene usefull in all cases
c) To reduce dead space
c) Ketanserine can be used as an alternative to
d) To prevent alveolar collapse
Dantrolene
495. During anaesthetic procedure, compression of
d) Propofol is safe
cricoid cartilage- (MP 98)
e) Muscle biopsy of diagnostic
a) Prevents spinal headache 507. A 10-year old child is undergoing squint surgery.
b) Helps in preventing regurgitation He suddenly developed increased heart rate,
c) Aid in intubation arrhythmia, high fever, metabolic and respiratory
d) Help to relieve reflex bronchospasm acidosis on arterial blood gases and elevation of
496. Which of the following drugs are believed to be end tidal C02• Which of the following would be the
effective in the treatment of post-operative shivering? first agent of choice in the management of this
a) Ondansetron (Delhi PG Mar. 09, AI 06) condition? (AIIMS May 12)
b) Diclofenac Sodium a) Dantrolene b) Paracetamol
c) Pethidine c) Procainamide d) Sodium bicarbonate
d) Paracetamol 508. Drugs used to control post-operative vomiting
497. Hypothermia is used in all except- (UP 04, PGI 98) a) Diazepam b) Phenobarbitone
a) Neonatal asphyxia b) Cardiac surgery c) Aprepitant d) Droperidol (PGI June 08)
c) Hyperthermia d) Arrythmia e) Ondansetron
498. Hypothermia is used in- (PGI 2000) 509. The following are used fortreatmentofpostoperative
a) Hyperpyrexia nausea and vomiting following squint surgery in
b) Prolonged surgeries children except- (AI 05)
c) Massive blood transfusion a) Ketamine . b) Ondansetron
d) Hypertension c) Propofol d) Dexamethasone
499. Which of the following anesthetic agents does not 510. The most common rhythm disturbance during early
post-operative period is- (AIIMS May 05)
trigger malignant hyperthermia? (AI 06)
a) Bradycardia b) Ventricular fibrillation
a) Halothane b) Isoflurane
c) Tachycardia d) Complete heart block
c) Suxamethonium d) Thiopentone

490)a 49l)a,b 492)c 493)All 494)a 495) b 496) c 497) d 498) a,b 499) d 500) c 50l)a 502)a 503)b
504)a 505) a,b,d,e 506) a,b,d,e 507) a 508) a,c,d,e 509) a 51 0) c
ANAESTHESIA [ 517]

511. Which of the following is not nsed in controlling 521. Which of the following surgical incisions is
heart rate intraoperatively- (AI 07) associated with the highest risk of postoperative
a) Propanolol/ Metoprolol b) Verapamil pulmonary complications ? (Delhi PG Mar. 09)
c) Esmolol d) Procainamide a) Vertical laparotomy b) Horizontal laparotomy
512. ffitra short acting beta blocker most commonly used c) Lateral thoracotomy d) Median sternotomy
in anaesthesia is- (TN 02) 522. 5 yr old child going to sitting craniotomy while
a) Esmolol b)Nadalol positioning in O.T. developed end tidal C02 Omm
c) Propanalol d) Atenolol Hg P02 80 mm Hg implies- (AIIMS 00)
513. With respect to cardiac arrests occurring during a) Endotracheal tube in oesophagus
anaesthesia - (Delhi PG Mar. 09) b) E.T. blocked with secretion
a) The most common preceding arrhythmia is c) Venous air Embolism
bradycardia d) Left lung collapse
b) The most common cause is failure of ventilation 523. 55 year old female a known case ofthryotoxicosis in
c) Most occur during emergence from anaesthesia co.ntrol posted for abdomino perineal resection
d) Most are considered unpreventable and untreatable during surgery there was sudden drop in B.P., and
514. Cause of post operative hypertension -(PGIJune 04) end tidal C02 decreased from 40 to 10 mmHg. On
a) Pre-operative hypertension auscultation there was a mill wheel murmur, what
b) Inadequate analgesia is the diagnosis- (AIIMS Nov 09)
c) Phaeochromocytoma a) Thyroid storm b) Bleeding
d) Hypoxemia c) Hypoxia d)Airembolism
e) Hypercarbia 524. Which of the following is true to the immediate
515. Which of the following signs of congestive cardiac postoperative period in a young patient with
failure constitute a major risk to the surgical patient previously normal lungs having upper abdominal
undergoing anaesthesia ? (DELHI PG Mar. 09) surgery ? (Delhi PG Mar. 09)
a) Jugular venous distention and third heart sound a) Arterial oxygen tension will typically be reduced
b) Cardiomegaly by an average of 10 mmHg when breathing room air
c) Pedal oedema b) Arterial oxygen tension will normalize after 15
d) Basal crepitations on auscultation minutes
516. Trendelenberg position produces decrease in all of c) Diffusion hypoxia is the major determinant of
thefollowingexcept- (AI 91) arterial hypoxaemia
a) Vital capacity b) Functional residual capacity d) The CXR will typically reveal no abnormalities
c) Compliance d) Respiratory rate
517. Upper respiratory tract infection is a common ANAESTHESIA FOR SPECIAL SITUATIONS
problem in children. All the following anaesthetic
complications can occur in children with respiratory 525. A 5 year old child is suffering from cyanotic heart
infections, except- (AI 02) disease. He is planned for corrective surgery. The
a) Bacteremia induction agent ofthe choice would by-(AIIMSNov05)
b) Halothane granuloma a) Thiopentone b) Ketamine
c) Increased mucosal bleeding c) Halothane d) Midazolam
d) Larygospasm 526. A 6 month old child is suffering from patent ductus
518. During intraoperative anaesthesia mismatch BT arteriosus (PDA) with congestive cardiac failure.
develops- (PGI Dec 07) Ligation of ductus arteriosus was d.ecided for
a) Increased bleeding b) Hypotension surgical management. The most appropriate
c) Bronchospasm d) Rash inhalational anaesthetic agent of choice with minimal
e) Movements oflimbs haemodynamic alteration for induction of
519. Mismatched blood transfusion manifests anaesthesia is - (AIIMS Nov 05)
intraoperatively as- (PGI 99) a) Sevoflurane b) Isoflurane
a) Rise in B.P. b) Excessive bleeding c) Enflurane d) Halothane
c) Dyspnoea d) Hematuria 527. A 52 year old male diagnosed as triple vessel
520. During surgery transfusion reaction is manifested coronary artery disease with poor left ventricular
as- (PGI02) function. Coronary artery bypass grafting surgery
a) Hypothermia was decided. During maintenance of anaesthesia
b) Hypotension which one of the following agents should be
c) Bleeding preferred? (AIIMS Nov 04)
d) Increased muscle movement a) IV opioids b) lsoflurane
e) Decreased respiratory rate c) Halothane d) Nitrous oxide

511)None>a 512)a 513)a,b 514)All 515)a 516) d 517)b 518)a,b,c,d 519)b 520)b,c 521)a 522)c
523) d 524) a 525) b 526) a 527) a >b
ANAESTHESIA [ 518]

528. During surgery for aortic arch aneurysm under 539. Current mode of analgesia best for intrapartum pain
deep hypothermic circulatory arrest, which of the relief- (MH 06)
following anesthetic agent administered prior to a) Epidural analgesia b) Spinal anaesthesia
circulatory arrest that also provides cerebral c) Inhalational d) Local analgesia
protection? (AIIMS 02) 540. A lower Segment Caesarean Section (LSCS) can
a) Etomidate b) Thiopental sodium be carried out under all the following techniques
c) Propofol d) Ketamine of anesthesia, except- (Delhi PG Feb. 09, AI 05)
529. A 30 year old woman with coarctation of aorta is a) General anesthesia
admitted to the labour room for elective caesarean b) Spinal anesthesia
section. Which ofthe following is the anaesthesia c) Caudal anesthesia
technique of choice- (AIIMS Nov 05) d) Combined spinal epidural anesthesia
a) Spinal anaesthesia 541. In general, the last muscle to be rendered akinetic
b) Epidural anaesthesia with a retrobulbar anesthetic block is-(AIIMS May 06)
c) General anaesthesia a) Superior rectus b) Superior oblique
d) Local anaesthesia with nerve block c) Inferior oblique d) Levator palpebral superioris
530. Patient with mitral stenosis is having surgery 542. Complication ofperibulbar block- (PGT Dec 04)
tomorrow. There is some liver compromise. Which a) Retrobulbar hemorrhage
of the following inhalational agent is preferred- b) Globe rupture
a) Halothane b) Enflurane (AIIMSNov JO) c) Optic neuritis
c) Xenon d) Sevoflurane d) Local anaesthetic solution can migrate to brain
531. The most common cause of morbidity and mortality e) Vasovagal syncope
in patients undergoing major vascular surgery is- 543. A five year old child is scheduled for strabismus
a) Renal complications (AIIMS May 05) (squint) correction. Induction of anesthesia is
b) Thromboembolic phenomenon uneventful. After conjunctival incision as the
c) Coagulopathies . surgeon grasps the medial rectus, the
d) Cardiac complications
anaesthesiologist looks at the cardiac monitor. Why
532. A patient of alcohlic liver faliure requires general
do you think he did that? (AIIMS 02)
anesthesiaAOC is- (AI OJ)
a) He wanted to check the depth of anaesthesia
a) Ether b) Halothane
b) He wanted to be sure that the blood pressure did
c) lsoflurane d) Methoxyflurane
not fall
533. The volatility of an anesthetic agent is directly
proportional to lowering the flow in the portal vein. c) He wanted to see if there was an oculocardiac
Portal flow is maximally reduced by- reflex
a) Ether b) Halothane (AIIMS May J0) d) He wanted to make sure there were no ventricular
c) lsoflurane d) Enflurane dysarhythmias which normally accompany
534. Anesthesia of choice in renalfailure- (AI OJ) incision
a) Methoxyflurane b) Isoflurane 544. During ocular surgery, when eye surgeon touches
c) Enflurane d) None the medial rectus, he asked to assistant to measure
535. Anaesthesia of choice in renal disease-(PG/June 08) heart rate because of- (Bihar 06)
a) Atracurium b) Cisatracurium a) Occulocardiac reflex b) Occulocephalic reflex
c) Vecuronium d) Rocuronium c) Occulovagal reflex d) None
e) Mivacuronium 545. Following will be the choice of anaesthesia in an
536. Best anaesthesia for status asthmaticus -(AIIMS 97) infected tooth posted for extraction- {MP 06)
a) Thiopentone b) Ketarnine a) Local block with lignocaine
c) Ether d) Np b) Local block with lignocaine and adrenaline
537. Which is safest to be used in asthmatic patients- c) Isoflurane
a) Nitrazepam b) Phenobarbitone (PGI OJ) d) Enflurane
c) Chloral hydrate d) All hypnotics are safe 546. Which of the following anaesthesia will produce
e) Morphine decreased EEG activities- (AIIMS Nov 06, JJ)
538. After Hyperventilating for some time holding the a) Hypothermia b) Early hypoxia
breath is dangerous because- (AIIMS Nov 00) c) Ketarnine d) Np
a) Decrease C0 2 shift the 0 2 dissociation curve to 547. Fire breaks out during laser vocal cord surgery.
the left What is notto be done? (AI 11)
b) Alkalosis can lead to tetany a) Pouring sterile water
c) It can lead to C0 2 Narcosis b) Removing endotracheal tube
d) Due to lack of stimulation by CO 2, anoxia can go c) 100% oxygen after discontinuing anesthetic gases
into dangerous levels d) Treatment with steroid and antibiotics

528)b 529)c 530)c 531)d 532)c 533)b 534)b 535) a,b,e 536) b 537) a 538) d 539) a 540)c 54l)b
542)All 543) c 544)a 545)d 546)a 547)c
ANAESTHESIA [ 519]

548. Anaesthesia used in microlaryngoscopy is- c) Induction with intravenous suxamethonium and
a) Pollarad tube ofl 0 mm diameter with heavy sedation Np; and halothane for maintenance
b) Pollaradtubeof15mmdiameterwithtopicalxylocaine d) Induction with inhalation halothane and N 20;
c) Pollarad tube with infiltration block (AIIMS 96) oxygen for maintenance
d) Heavy sedation on and endotracheal intubation 557. The most appropriate circuit for ventilating a
549. Which of the following anesthetic drugs is spontaneously breathings infant during anaesthesia
contraindicated in chronic renal failure -(AI 12) is- (AI 05)
a) Morphine b) Fentanyl a) Jackson Rees modification of Ayres T piece
c) Pethidine d) Atracurium b) MaplesonA or Magill's circuit
550. In a pregnant female, there is decreased requirement c) Mapleson Cor Waters to and frocanister
of the spinal anaesthetic agent because of all of the d) Bains circuit
following except- (AJIMS May 12) 558. The narrowest part of larynx in infants is at the
a) Exaggerated lumbar lordosis cricoid level. In administering anesthesia this may
b) Decreased volume of subarachnoid space lead to all except- (AIIMS 03)
c) Engorgement of epidural veins a) Choosing a smaller size endotracheal tube
d) Increased sensitivity of the nerves to anaesthetic b) Trauma to the subglottic region
agent c) Post operative stridor
551. A six year old child is posted for elective urology d) Laryngeal oedema
surgery under general anesthesia. He refuses to 559. A 5 year old patient is scheduled for tonsillectomy.
allow the anaesthesiologist an I.V. access. The best On the day of surgery he had running nose,
inhalational agent of choice for induction of temperature 37.5°C and dry cough. Which ofthe
anesthesia is - (AIIMS May 04) following should be the most appropriate decision
a) Sevoflurane b) Methoxyflurane for surgery-
c) Desflurane d) Isoflurane a) Surgery should be cancelled
b) Can proceed for surgery if chest is clear and there
552. Which of the following inhalational agents is the
is no history of asthma
induction agent of choice in children -(AI 06, 04,PGI
c) Should get X-ray chest before proceeding for
a) Methoxyflurane b) Sevoflurane 08, 05, MH 08)
surgery
c) Desflurane d) Isoflurane
d) Cancel surgery for 3 weeks and patient to be on
553. All agents can be given for induction of anesthesia
antibiotic
in children except? (Jharkhand 06)
560. A six year old boy is scheduled for examination of
a) Halothane b) Sevoflurane
the eye under anaesthesia. The father informed that
c) Morphine d) N20 for the past six months the child is developing
554. In a child with intestinal obstruction with deranged progressive weakness of both legs. His elder sibling
liver function test, the anesthetic of choice is- had died atthe age of14 years. Which drug would
a) Enflurane b) lsoflurane (PGI June 06) you definitely avoid during the anaesthetic
c) Halothane d) Sevoflurane management? (AIIMS 02)
e) Ether a) Succinylcholine b) Thiopentone
555. A 5 year old boy suffering from Duchenne Muscular c) Nitrous oxide d) Vecuronium
dystrophy and polymyositis has been fasting for 8 561. A two month old infant has undergone a major
hrs and has to undergo tendon lengthening. Which surgical procedure. Regarding postoperative pain
anaesthesia should be used- (AIIMS 2000) relief which one of the following is recommended-
a) Induction by IV thiopentone & maintenance by a) No medication is needed as infant does not feel
Np & halothene pain after surgery due to immaturity of nervous
b) Induction by IV propofol and maintenance by Np system (AI 06)
&02 b) Only paracetamol suppository is adequate
c) Induction by IV scoline and maintenance by Np c) Spinal narcotics via intrathecal route
&Halothane d) Intravenous narcotic infusion in lower dosage
d) Inhalation Np, 0 2 & halothene 562. Regarding neonatal circumcision, which one ofthe
556. A 5 year old boy suffering from Duchenne muscular following is true- (AIIMS 03)
dystrophy has to undergo tendon lengthening a) It should be done without anaesthesia, as it is
procedure. The most appropriate anaesthetic would hazardous to give anaesthesia
be- (AI 03) b) It should be done without anesthesia, as neonates
a) Induction with intravenous thiopentone and Np; do not perceive pain as adults
and halothane for maintenance c) It should be done under local anaesthesia only
b) Induction with intravenous suxamethonium and d) General anaesthesia should be given to neonate
N 20; and oxygen for maintenance for circumcision as they also feel pain as adults

548)c 549)c 550)a 55l)a 552)b 553)c 554)c 555)d 556)a 557)a 558)a 559)d 560)a 56I)d
562)c
ANAESTHESIA [ 520 ]

CARDIOPULMONARY RESUSCITATION AND 572. The outcome following resuscitation of cardiac


MECHANICAL VENTILATION arrest is worsened if during resuscitation patient is
given- (AIIMS Nov 05)
563. First step in CPR (cardio pulmonary resuciatation) a) Ringer's lactate b) Colloids
should be- (PGI 2000) c) 5% Dextrose d) Whole blood transfusion
a) IV adrenaline b) lntracardic atropine 573. During cardiac resuscitation, the following can
c) Airway maintainance d) Hystrectomy occur except- (PGI OJ)
564. In a patient with multiple injuries, first thing to be a) Rupture oflungs b) Rupture of liver
done is- (DPG 09, PGI 99} c) Rupture of stomach d) Rupture of spleen
a) Patency of airway e) Disseminated intravascular coagulation occurs
b) Maintenance ofB.P. 574. During cardiopulmonary resuscitation, cardiac
c) Immobilize cervical spine massage is given- (MH 06)
d) Lateral position with mouth gas a) Upper third of sternum b) Mid third of sternum
565. A 40 year old man who met with a motor vehicle c) Lower third of sternum d) Precordium
catastrophe came to the casualty hospital in an hour 575. In controlled ventilation- (PGI Nov 09)
with severe maxillofacial trauma. His Pulse rate was a) Patient is active b) Patient is passive
120/min, BP was 100/70 mm Hg, Sp02- 80% with c) Ventilator is active d) IPPV
oxygen. Whatwould he the immediate management- e) Ventilater is passive
a) Nasotracheal intubation (AIIMS Nov 10) 576. In volume-cycled ventilation the inspiratory tlow rate
b) Orotracheal intubation is set at- (AIIMS 02)
c) Intravenous fluid a) 140-160Limin b) 110-130Limin
d) Tracheostomy c) 60-100Limin d)30-50Limin
566. In a patient with cardiorespiratory arrest, basic life 577. The following modes of ventilation may be used for
support is given to support which of the following weaning off patients from mechanical ventilation
system- (PGI OJ) except- (AI 05)
a) Respiratory system b) Cardiovascular system a) Controlled mechanical ventilation (CMV)
c) Renal system d) Gastrointestinal system b) Synchronized intermittent mandatory ventilation
e) CNS (SIMV)
567. According to 2005 AHA guidelines true about c) Pressure support ventilation (PSV)
number of chest compression in CPR- d) Assist-control ventilation (ACV)
a) 80/min including neonate (PGI Dec 07) 578. All are true about PEEP except- (PGI 99)
b) 90/min including neontate a) Useful in situations where P0 2 is low
c) 100/min excluding neonate b) teo
d) 120/min including neonate c) Impaired renal function
e) 120/min excluding neonate d) J.ICT
568. IN CPR drug of choice is- (AP 96) 579. About CPAP all are true except- (A.IIMS May 09)
a) NaHC03 b) Isoprenaline a) Given prophylactically in all preterm with
c) Propranolol d)Adrenaline respiratory distress
569. True about adrenaline in CPR- (PGI OJ) b) Started withFi02 50-60%
a) Can be given intratracheally c) Given in infants less than 28 weeks and less than
b) LV. route better than intracardiac 1 kg weight
c) Intracardiac route better than IV d) Improves oxygenation and improves lung
d) Converts coarse fibrillation into fine ones compliance
e) The dose used is 2ml containing 1 in 1000 580. Complication of positive pressure ventilation-(Kerala
concentration a) Pneumothorax b) Bradycardia 98)
570. Concentration of adrenaline in CPR is- (MP 03) c) Decreased ventilation d) Arrhythmias
a) 1: lOlakhs b) 1: 1lakhs 581. Placement of a double lumen tube for lung surgery
c) 1 : 10,000 d) 1 : 1000 is best confirmed by- (PGI Dec 08, AIIMS Nov 05)
571. Which of the following drugs in NOT recommended a) EtC02
forintracheal administration during cardiopulmonary b) Airway pressure measurement
resuscitation- (Karn 96) c) Clinically by auscultation
a) Atropine b) Sodium bicarbonate d) Bronchoscopy
c) Adrenaline d) Lignocaine

563)c 564)a 565)b 566) a,b,e 567) c 568) d 569) a,b 570) d 571) b 572) c 573) e 574)c 575)b,c,d
576)c 577)a 578)d 579)c 580)a 581)d
ANAESTHESIA [ 521 ]

582. The most sensitive and practical technique for 592. Best antagonist of morphine is- (AIIMS 97)
detection of myocardial ischemia in the perioperative a) Nalorphine b) Naloxone
periodis- (AIIMSNov05) c) Buprenorphine d) Pentazocine
a) Magnetic Resonance Spectroscopy 593. Which one of the following is the shortest acting
b) Radio labeled lactate determination intravenous analgesic- (AIIMS 03)
c) Direct measurement of end diastolic pressure a) Remifantanil b) Fentanyl
d) Regional wall motion abnormality detected with c) Alfentanil d) Sufentanil
the help of2D transoesophageal echocardiography 594. Established routes of administration of morphine
include- (PGI June 02)
MISCELLANEOUS a) Inhalation b) Rectal
c)SC d)N
583. Which one of the following is the description used e)IM
for the term anodynia during pain management? 595. Which of the following drug can be given
a) Absence of pain perception (AIIMS May 06) intravenously, epid urally or by transdermal route-
b) Complete lack of pain sensation a) Fentanyl b)Thiopental (DPG 08)
c) Unpleasant sensation with or without a stimulus c) Succinylcholine d) Vecuronium
d) Perception of an ordinarily non-noxious stimulus 596. Opioidantagonistthatcan begivenorally-(MH 10)
as severe pain a) Naltrexone b)Naloxone
584. Perception of ordinarily non noxious stimuli as pain, c) Nalbuphene d) Pentazocine
is better known as- (PGI 2000, SGPGI 03) 597. True about nitric oxide- (PGI June 07)
a) Allodynia b) Hyperalgesia a) Formed from L-Arginine by NO synthase
c) Hyperesthesia d) Radiculopathy b) Causes vasodilation in all vessels
585. Allodyniais- (AIIMS 98) c) Used in portal hypertension
a) Feeling pain without stimulus d) Interacts with Hb
b) Feeling pain to a normal non painful stimulus e) Used in erectile dysfunction
c) Both 598. Which of the following inhaled gas is used to
d) None decrease pulmonary artery pressure in adults and
586. Visual analogue scale {VAS) most widely used to infants ? (AIIMS 02)
measure- (AIIMS 05, 03) a) Nitrous oxide b) Nitrogen dioxide
a) Sleep b) Sedation c) Nitric oxide · d) Nitrogen
c) Pain intensity d) Depth of anaesthesia 599. True about inhaled nitric oxide- (PG!June 07)
587. Pain rating index is provided by-(PGI 02,Jipmer 00) a) Causes generalized vasodilation
a) Faces scale b) Visual analogue scale b) Dilates pulmonary arteries
c) Me Gill questionniare d) CHEOP scale c) Causes hypotension
588. Best scale to measure pain in children of 5 years d) Least systemic effects
age would be- (SGPGI 02) e) Better ventilation perfusion match
a) VAS b) Me Gill scale 600. True about action ofinhaled NO- (PGJ Nov. 10)
c) Faces scale d) CHEOPS a) Causes pulmonary vasodilation
589. A children hospital Eastern Ontaria Pain Scale b) Increase pulmonary artery pressure
(CHEOPS) for rating potoperative pain in children c) Inproves ventilation- perfusion matching
includes all- (AIIMS Nov 08) d) Systemic vasodilation
a) Cry b) Touch e) Least systemic effect
c) Torso d) Oxygen saturation 601. Which of the following statements is true regarding
590. True regarding morphine- (PGI June 06) Nitric oxide- (PGJ 01)
a) Tolerance develops for all except miosis and a) Used in pulmonary hypertension
constipation b) Decreases the dose of anaesthetics
b) Tolerance to all effects develops with chronic usage c) Sympathomimetic action
c) Tolerance develops for all except euphoria and sedation d) Causes systemic hypotension
d) Tolerance can develop to all effects in high doses e) Used as a vasoconstrictor
591. All are opioid agonist - antagonist compounds 602. Which of the following agents is not used to provide
except- (PGI02,Jipmer01) induced hypotension during surgery ? (AI 06)
a) Buprenorphine b) Nalbuphiue a) Sodium nitroprusside b) Hydralazine
c) Pentazocine d) Nalmefene c) Mephenteramine d) Esmolol
e) Papaverine

582)d 583)d 584)a 585)b 586) c 587) c 588) d 589) d 590) a 59l)d,e 592)b 593)a 594) b,c,d,e
595)a 596)a 597)All 598)c 599) b,d,e 600) a,c,e 601) a,d 602) c
ANAESTHESIA [ 522]

603. Hypotensive anesthesia in nasopharyngeal 614. Which of the following does not represent a
angiofibroma is/are given by- (PG/June 09) significant anaesthetic problem in the morbidly
a) Propofol b) Ketamine obese patient? (AIIMS Nov 04)
c) Phentolamine d) Halothane a) Difficulties in endotracheal intubation
e) Na Nitroprusside b) Suboptimal arterial oxygen tension
604. Sodium nitroprusside infusion may result c) Increased metabolism of volatile agents
in- (AIIMSMay05) d) Decreased cardiac output relative to total body mass
a) Hypertension b) Pulmonary oedema 615. Characteristic of an ideal gas is- (PGI 98)
c) Cyanide toxicity d) Heart block a) Volume is directly proportional to change in pressure
605. The drug which is not suitable for patients with acute b) Volume is inversely proportional to change in
porphyriaforintravenousinduction is-(AIIMSMay05) temperature
a) Thiopentone sodium b) Propofol c) At absolute temp. volume of gas is 1
c) Midazolam d) Etomidate d) Obeys Charles, Byles andAvagadro laws
606. Drugs contraindicated in AlP- (PGI Dec 07, 04) 616. The gas used to create pneumoperitoneum is-
a) Thiopentone b) Etomidate a) C02 b)N2 (PGIOJ)
c) Ketamine d) Propofol
c) 0 2 d) Room air
e) Midazolam
607. In porphyria all drugs are safe except- (MH 09)
e) Np
617. lncubatorheatisdeliveredbyexcept- (PGI98)
a) Propofol b)Midazolam
a) Conduction b) Convection
c) Afentanyl d) Pentazocin
608. Anesthetic agent contraindicated in porphyria - c) Radiation d) Evaporation
a) Thiopentone b) Propofol (PGI Nov 09) 618. True about scavenging action in OT- (MP 06)
c) Ketamine d) Etomidale a) Outlet connection is 22 mm male fitting
e) Methadone b) Interface limit pressure is 0 to +2 em H20
609. A patient after undergoing thoracotomy complains c) The charcoal as an adsorbent should not be used
of severe pain. He can be managed by- d) Scavenging is defmed as the collection of excess
a) Intercostal cryoanalgesia (AIIMS Nov 10) gases from equipment used in administrating
b) I.V fentanyl anesthesia or exhaled by the patient and removal
c) Oral morphine of these gases to an appropriate places discharge
d) Oral brufen outside the working environment
610. Management of chronic pain includes-(PGINov. 10) 619. The gas which produces systemic toxicity without
a) Intra-thecal hyperbaric phenol causing local irritation is- (AI 02)
b) Anterolateral cordotomy a) Ammonia b) Carbon monoxide
c) Epidural fentanyl c) Hydrocyanic acid d) Sulfur dioxide
d) Patient controlled analgesia (PCA) 620. In a patient with fixed respiratory obstructionHeHum
e) Anticonvulsant drugs is used along with Oxygen instead of plain oxygen
611. TrueaboutHeliox- (PGI Nov. 10) because- (AI 02)
a) Helium is an inert gas a) It decreases oxygenation
b) Less viscous than air b) It decreases turbulence
c) Higher density than air
c) It decreases the dead space
d) Reduces work of breathing
d) It provides analgesia
e) Mixture ofHe & 0 2
621. Drugs which interfere with anesthesia are-
612. Criteria for brain death AlE- (PGI Dec 07)
a) Em b)EEG
a) Calcium cbarmel blocker nifedipine (PGI OJ)
c) Brain stem reflex d) J, body temperature b) Beta blockers
e) Pupillary dilatation c) Aminoglycosides
613. Clinical criteria of brain death is all except- d) Steroid administration
a) Coma (Jipmer 05) e) D-tubocurane
b) Absent brain stem reflex 622. In an injured patient with hypovolemia intravenous
c) Absent spinal cord reflex fluid administerved is guided by- (PGI 03)
d) Absent motor activity a) Central venous pressure b) Blood pressure
c) Urine output d) Pulse rate

***

603)c,d,e 604)c 605)a 606)a,b 607)d 608)a,d 609)b 610)All 61l)a,d,e 612)a,b,d,e 613)c 614)d 615)d
616)a,c,d,e 617)a 618)d 619)b 620)b 621)a,b,c 622)a,b,c,d
RADI y
RADIOLOGY

FUNDAMENTALS IN RADIOLOGY 13. Grid is a device used for- {Punjab 11)


a) Reducing scattered radiation
1. X-rays were discovered by- (MP 04) b) Reducing patient's exposure time
a) Godfrey Houns:field b) Roentgen c) Reducing the contrast of the X-ray
c) Coulomb d) Sievert d) All of the above
2. The active ingredient ofX-ray film is- (DPG 08) 14. CATwasinventedby- (Jharkhand 03)
a) Silver chloride b) Silver bromide a) Hounsfield b) Roentgen
c) Silver nitrate d) Gold chloride c) Cormack d)Tesla
3. X-rays are modified- (DPG Feb 09) 15. Hounsfield units depends on- (AIIMS May 09)
a) Protons b) Electrons a) Electron density
b) Mass density
c) Neutrons d) Positrons
c) Effective atomic number
4. i energy linear acceleration used in-(PGIJune 04) d) Attenuation coefficient
a) X-ray b) Cathode rays
16. In computed tomography (CT), the attenuation value
c) Photon rays d) a- rays are measured in hounsefield units (HU). An
e) y-rays attenuation value of '0' (zero) HV corresponds to-
5. X-raysareproducedwhen- (AIIMS 02) a) Water b) Very dense bone structure
a) Electron beam strike the nucleus of the atom AIR Fat
b) Electron beam strikes the anode
c) Electron beam reacts with the electromagnetic field
d) Electron beam strikes the cathode
6. Which is provided by Unear accelerator-(AIIMS 95) Slice of tissue X-rays is -
a) Electron b) Neutron a) Tomography b) Mammography
c) Proton d) Infrared rays c) Contrast studies d) All of the above
7. Ion which scatters X-ray most is- (AI 97) 19. ffigh- resolution CT of the lung is a specialized CT
a) ca++ b)Hg technique for greater detail oflnng parenchyma and
c) Pb d)H+ it utilizes- (AIIMS 02)
8. Maximum scattering in X-ray plate occurs in- a) Special lung :filters
a) Carbon b) Mercury (AI 96) b) Thick collimation
c) W d)Ca++ c) Bone algorithm for image reconstruction
9. What is needed for proper radiographic image in a d) Large fieldofview
heavybonybuiltperson? (AI 02) 20. ffigh resolution computed tomography of the chest
a) iedma b) ikvp is the ideal modality for evaluating- (AI 03)
c) i ed exposure time d) i ed developing time a) Pleural effusion b) Interstitial lung disease
c) Lung mass d) Mediastinal adenopathy
10. ContrastinX-raysisdependent- {Assam 95)
21. Which one ofthe foDowingimaging techniques gives
a) KV
maximum radiation exposure to the patient?
b)Ma
a) Chest X-ray b)MRI (AI06,AIIMS,May 11)
c) Durationofexposure c) CT scan d) Bone scan
d) Distance between source and object 22. Radiation hazard is absent in- (PGI June 06)
11. Photoelectric effect can be best described as an- a) MRI
a) Interaction between high energy incident photon b) Doppler USG
and the inner shell electron (AI 08) c) Digital substraction angiography
b) Interaction between low energy incident photon d) Tc 99 scan
and the outer shell electron 23. Calcification is best detected by- (PGI 03)
c) Interaction of the high energy incident photon a) X-ray b) USG
and the outer shell electron c) CT Scan d) MRl
d) Interaction between a low energy incident photon e) PETScen
and the inner shell electron 24. Regarding CT scan all are true except-
12. Photon transferring some of its energy to electron a) 50% reduction in kVp reduces 50% radiation dose
is- (Jipmer 06) b) Decrease in mA (milliamperes) decreases radiation
a) Photoelectric effect dose exposure significantly in pediatric chest
b) Bremsstrahlung effect c) Quality of the radiation generated depends upon
c) Compton effect voltage (AIIMS Nov 08)
d) Ionization d) Radiation dose exposure is directly related to the
time of exposure

I) b 2)b 3)b 4)a 5)b 6)a 7)c 8)a 9)a,b lO)a ·u)d 12)c 13)a 14)a
15)d 16)a l7)b 18)a 19)c 20)b 2l)c 22)a,b 23)c 24)a
RADIOLOGY [ 544]

25. Walls of the CT scanner room are coated with- 39. lnMRithefieldusedis- (AI 97)
a) Lead b)Glass (AI 10) a) 0.05 tesla b) 100 tesla
Tungsten Iron c) 1.1 tesla d) 11 tesla
40. T 1 and T 2 images are a feature of which diagnostic
test- {MH 06, Kar 04)
a) USG b) CT Scan
c) Duplex scan d) MRI
27. Ultrasound frequency nsed for diagnostic purposes 41. On a MRI scan on T weighted images all appear
in obstetrics - (PGI 01) hypointensve except! (DPG Mar. 09)
a) l-20MHz b)20-40MHz a) Chronichemorrhage b)Air
c) 40-60 MHz d) 60-80 MHz c) Cortical bone d) Ligaments
e) 80-lOOMHz
28. Frequency of sound waves used in USG- (Bihar 04)
a)< 1000Hz b)> 5000Hz
c) >lOOOOHz d)>20000Hz 43. AdvantageofMRI- (PGI Dec 08)
29. USGwavetravelinhuman body attherateof- a) Calcification best seen
a) 1500m/s b)2500m/s(Jhark03)
b) CSF hypointense on T 1 & hyperintense on T 2
c) 3500m/s d) None
c) Best to visualise vascular structures
30. Ultrasonography transducer contain- (Bihar 05)
d) Best to differentiate benign/malignant pulmonary
a) Sodium fluoride b) Lead Zirconate
nodule
c) Caesium fluoride d) None
e) Can be used in multiple plain
31. The ultrasonic sound waves employed in ultrasound
44. Which of the following is not true about MRI?
machine for medical sonography have the frequency
of- (MH06)
a) MRI is useful in locating small lesions (AI 12)
a) 2- 20 MHz b) Less than l MHz b) CT is better than MRI in bone lesions
c) 20-20,000Hz d)GreaterthanlOOMHz c) MRI is better for calcified lesions
32. Which is notechogenic while doing ultrasonography- d) None
a) Bile b) Gas (Jipmer 03) 45. An absolute contraindication ofMRI is-
c) Bone d) Gall stones a) Pacemaker (Aiims May 08)
33. Acousticshadowisproducedbythefollowingexcept- b) Prosthetic cardiac valves
a) Bone b) Calculus (MH03) c) Insulin pump
c) Fat d)Gas d) Choclear implants
34. The intensity ofcolour in Doppler is determined by- 46. Absolute C/1 ofMRI is- (PGI June 05)
a) Direction flow (PGI 98) a) Pacemaker
b) Velocityofflow b) Pregnancy at 1st trimester
c) Strength of returning echo c) Aneurysmal clip
d) None of the above d) Phobia
35. Contrasts used in USG- (PGI Dec 07) 47. Patient with a metallicforeign body in eye, which
a) Urograffin b) Ultragraffin investigation is not done- (PGI Dec 06)
c) Sonavist d) Conray a) MRI b)USG
e) Barium c) X-ray d)CT
36. Time sector scanning of neonates is preferred
because of the following reason. Most practical
reason- {PGI95)
a) Open fontanelles
b) Inexpensive
c) Children more co-operative us~ DOJl-lOitnzm2 ra<Uatlon
d) Better resolution a) Ultrasonography
37. In which of the following form of imaging, Harmonic b) Thermography
Imaging is related - (DPG Mar. 09) c) MRI
a) Sonography b) Digital radiography d) Radiography
c) MRCP d)Nuclearirnaging 50. Which one is non-ionising radiation- (PGI 97)
38. NMR based in the principle of- (AJIMS 2000, AI 99} a) MRI b) CT Scan
a) Electron beam b) Proton beam c) X-ray d) Position emission scintigraphy
c) Magnetic field d) Neutron beam

25)a 26)a 27)a 28)d 29)a 30)b 3l)a 32)a 33)c 34)b 35)c 36)a 37)a 38)b>c
39)c 40)d 41)a 42)b 43)c,e 44)c 45)a 46)a,c 47)a 48)a 49)d 50) a
RADIOLOGY [ 545]

51. ''Time ofF1ight'' technique is employed in-(DPG Mar. 64. Non-Iodine containing contrast is- (AJIMS May 12)
a) SpiralCT b) MRimaging 09) a) GdDTPA b) Visipaque
c) CT angiography d) Digital radiography c) Iohexanol d) Diatrozoate
52. Most radio dense substance is - (DPG Feb. 09) 65. Allergic reactions to radiological contrast agents are-
a) Fluid b) Soft tissue a) Anaphylactic reactions (AI 08)
c) Brain d) Bone b) IgE mediated reactions
53. MRI rooms are shielded completely by a continuous c) Urticaria
sheet or wire mesh of copper or aluminum to shield d) Edema
the imager from external electromagnetic 66. All of the following are true about iodinated
radiations, etc. It is called - (AIIMS 03) intravascular contrast media except-
a) Maxwell cage b) Faraday cage a) They are used in digital subtraction angiography
c) Edison's cage d) Ohms cage b) They are radio opaque (Kerala 00)
54. The EEG cabins should be completely shielded by a c) They can cause anaphylactic reactions
continuous sheet of wire mesh of copper to avoid the d) They are used in magnetic resonance imaging
picking of noise from external electromagnetic e) They are excreted mainly by the kidneys
distrubances. Such a shielding is called as- (AllMS Nov 67. IN contrast is not used in- (Punjab ll)
a) Maxwell cage b) Edison's cage 04) a) HRCT b)MRI
c) Faraday ca_ge d) Ohm's cage c) lVP d) Myelography
55. PACS in medical imaging stands for-(Aiims May 08)
a) Plaru;ted archiving common system RESPIRATORY SYSTEM
b) Planned archiving computerized system
c) Picture archiving and communication system 68. For chestX-raybestview is- (Bihar 06)
d) Picture archiving or computerized system a) P.A. view b)A.P. view
56. Which of the following statements about contrast in c) Lateral view d) None
radiographyistrue- (AI 11) 69. Decubitus view is useful in diagnosing- (PGI OJ)
a) Ionic monomers have three iodine atoms per two a) Pleural effusion
particles in solution b) Pleural effusion with dependent hemithorax
b) Osmolar contrast agents may be ionic or non ionic c) Pericardial effusion
c) Gadolinummaycross the blood brain barrier d) Middle lobe consolidation
d) Iohexol is a high osmolar contrast media 70. Minimal ple·ural effusion is best detected by
57. ContrastusedinCT- (PGI 2000) X-ray...view- (UP 98)
a) Gadolinium b) Technitium a) A.P. b) P.A
c) Iodine d) Chromium c) Lateral d) Oblique
58. ContrastusedforMRI- (PGI 2000) 71. What minimal amount of pleural fluid is seen on
a) Iodine b) Gadolinium chest radiograph- (MH 03)
c) Metvazamide d) Pmnipaque a) SOml b) 150ml
59. Radiocontrast is contraindicated in all except- c) 500ml d)800ml
a) Renal failure b) Patient on metformin 72. Right anterior oblique view of chest X-ray true is/
c) Dehydration d)Obesity (AJ07) are- (PGI June 08)
60. Which of the following contrast agents is preferred a) Cassette near right shoulder
in a patient with decreased renal function to avoid b) Cassette near left shoulder
contrastnephropathy- (AI ll) c) Arch of Aorta best seen
a) Acetyl cystine b) Fenoldapam d) Left atrial enlargement can be diagnosed
c) Mannitol d)Lowosmolarcontrast Mitral & valves better seen
61. Contrast media ofchoice for myelogram is-(ADMS 95)
a) Urografin 75% b)Conray470
c) Iohexol d) Biligrafin
62. Contrast material used in the diagnosis of 74. Apex oflung is best assessed by- (Kerala 03)
esophageal atresia is - (PG/97) a) APview b)PAview
a) Gastrograffin b) Conray 420 c) Lordotic view d) Oblique view
c) Dianosil d) Myodii 75. Thelordoticviewisvalubleinconfirmingthepresence
63. Nonionic dye is- (NEETIDNB Pattern) oflesion in the lung apex and also in the -(Jipmer 96)
a) Ioxoglate b) Iohexol a) Posterior mediastinum b) Lingular segment
c) Iothalamate d) None c) Posterior basal segment d) Hilum

Sl)b 52)d 53)b 54)c 55)c 56)a 57)c 58)b 59)d 60)d 61)c 62)c 63)b 64)a
65)a 66)d 67}d 68)a 69)a,b 70)c 71)None>a 72)a,d 73)b 74) c 75)b
RADIOLOGY [ 546]

76. Right lung is seen to best advantage on the following 89. True about chest X-ray- (PGI Nov 09)
view- (UP 99) a) 40% lung tissue seen obscured by bony structure
a) Right posterior oblique b)Rightanterioroblique & mediastinum.
c) Left anterior oblique d) Lateral b) 60% lung tissue seen obscured by bony structure
77. Bestviewtodiagnosepneumothorax- (PGI98) & mediastinum
a) Lateral oblique c) Right dome higher than left dome
b) PA view in full expiration d) Right dome pushed up by liver
c) PA view in full inspiration e) Should be taken in expiration
d) AP view in full expiration 90. All are true about thymus swelling except-
78. X-ray film should be taken in closed pneumothorax a) Widening of mediastinum on X-ray (AIIMS 99)
during- (MP 2000) b) Sharp border with sail like appearance
a) Deep expiration b) Deep inspiration c) Steroid administration reduces size of swelling
c) In supine position d) In lateral position d) Shift of trachea on X-ray
79. Normal hilar shadow in X-ray chest is produced by 91. Tracheal bifurcation on X-ray corsponds to-
all except- (MP 00, AIIMS 97) a) T 5T6 b) T4T5 (PGI OJ)
a) Pulmonary artery b) Bronchus c) Sternal angle d) Thoracic inlet
c) Lower lobe vi ens d) Upper lobe veins
80. True about chest X-ray is all except- (PGI 97)
a) Left hilum is higher
b) Left dome is higher
c) All fissures are clearly seen on lateral film 93. All are true about Kerley B line except- (AIIMS 97)
d) None a) Horizontal
81. Structure forming right border of heart- b) Runs from hilar area to peripheral area
a) SVC b)IVC (PGI June 06) c) Due to thickening of septa
c) Rt. atrium d) Lt. atrium appendage d) Due to pulmonary venous hypertension
e) Pulmonary vessels 94. Kerley's 'B' lines are found in- (PGI 03)
a) Interstitial edema
b) Pulmonary venous congestion
c) Pericardia! effusion
83. Right border of the heart in CXR is formed by- d) Mitral stenosis
a) Pulmonary artery (PGI June 07) 95. Kerley B lines are seen in- (PGINov 09)
b) Superior vena cava a) Pleural effusion b) Mitral stenosis
c) Rtatrium c) Pericardia! effusion d) Interstitial edema
d) Rt ventricle e) Lymphangitis carcinomatosis
84. Lt border of the heart in CXR is formed by -(PGI 04) 96. Air bronchogram may be seen in the following
a) Pul. artery b) Pul. vein except- (MH03)
c) Abdominal aorta d) Arch of aorta a) Consolidation of lung b) Lung collapse
e) Rt ventricles c) Bronchoalveolar Ca d) Normal variant
85. Left border of heart on chest X-ray is formed 97. Silhouetting ofleft border of heart (silhoutte sign
by- (PGINov. 10) positive) on chest radiograph indicates what
a) Aortic arch b) Left Pulmonary artery pathology- (Maharashtra 05)
c) Right atrium d) Left ventricle a) Upper lobe b) Hilar
e) Right ventricle c) Lingular d) Lowerlobe
86. Base of heart is formed by- (PGI Dec 06) 98. Consolidation of which portion ofthe lung is likely
a) Rt. ventricle b)LV to obliterate the Aortic knuckle on X- ray chest-
c) LV+RV d)RA+ RV a) Left Lingula (AI 11)
e) RA+LA b) Right upper lobe
87. In X-ray right border of mediastinum is formed by c) Apexoflowerlobe
all except- (AI 98) d) Left upper lobe (posterior part)
a) SVC b) Right atrium 99. A triangular opacity with clear borders, base towards
c) Right ventricle d) Right brachiocephalic veins midline and obliterating right heart border on a chest
88. Right side of mediastinum shadow is not formed by- radiograph suggest that the pathology is likely to be in-
a) Superior venacava (AI 95) a) Apical segment of right lower lobe (MH 11)
b) Right innominates b) Medial segment of right lower lobe
c) Rightatrium c) Right middle lobe
d) Right ventricle d) Any of the above

76)a,c 77)b 78)a 79)c 80) b 81) a,b,c 82) d 83) b,c 84) a,d 85) a,d 86) e 87)c 88)d 89)a,c,d
90)d 9l)b,c 92)b 93)b 94)a,b,d 95)b,d,e 96)b 97)c 98)d 99)c
RADIOLOGY [ 547]

100. Bulging fissures in lungs in seen in- (PGI 99) 111. Hyperinflation oflung in CXR is seen in-
a) Klebsiella pneumonia b) Staph pneumonia a) CCF (PGI Dec 04)
c) Pulmonary oedema d) Pneumoconiosis b) Congenital lobar emphysema
101. Floating water-my sign is feature of -(AIIMS Nov 07) c) Diaphragmatic hernia
a) Lung hydatid b) Bronchial adenoma d) Foreign body
c) Lung abscess d)Aspergilloma e) Mucoviscidosis
102. Honeycombimg oflung in C.X.R. is seen in-
a) R.A. b)T.B. (PG/03)
c) Scleroderma d) Carcinoma
e) Interstitial lung disease
103. Groqnd glass appearance is not seen in-(MH06, AIOJ)
a) Hyaline membrane disease 113. Which of the following is a cause of unilateral
b) Left to right shunts hyperlucent lung on chest radiography?(COMED 09)
c) Pneumonia a) Poland syndrome b) Asthma
d) Obstructive TAPVC c) Acute bronchiolitis d) Pleural effusion
104. Bilateral pleural plaques and pulmonary fibrosis 114. The cause of homogenous opacity on X-ray is all
involving bases is hallmark of- (MH I 0) except- (Karnataka 2000)
a) Asbestosis b) Silicosis a) Pleural effusion
c) Scleroderma d) Byssinosis b) Diaphragmatic Hernia
105. A 48 years old man, resident of Baroda outskirts c) Massive consolidation
near a textile mill presents to his family physician d) Emphysema
with respiratory symptoms. D~:;!.:tor advices X-ray 115. In lung X-ray, heterogenous shadow is due to-
chest which showed - fine reticular & nodular a) Haemangioma (PGI 97)
pattern in lower zone with loss of clarity of b) Pulmonary infarction
diaphragm & cardiac shadows. He also doubts about c) Metastatic lesion
the presence of small pleural effusion. The probable d)1B
diagnosis is? (AI 99)
116. Causes of a cavitating lesion in the chest radiograph
a) Stannosis b) Asbestosis
are all except- (WB 2000)
c) Silicosis d) Coal worker's pneumoconiosis
a) Hamartoma
106. Extensive pleural thickening and calcification
b) Pulmonary infarction
especially involving the diaphragmatic pleura are
c) Squamous carcinoma of the bronchus
classical features of- (AI 03)
d) Caplan's syndrome
a) Coal worker's pneumoconiosis
b) Asbestosis e) Haematoma
c) Silicosis 117. Miliarymottlingseenin- (PGI Dec 06)
d) Siderosis a) 1B b) Sarcoidosis
107. Solitary nodule lung cannot be- (DPG Feb. 09) c) Silicosis d) P. carinii pneumo9-ia
a) Tuberculoma b)Neurofibroma 118. "Miliary shadow" on chest X-ray is seen in- (PGI 03)
c) Bronchogenic carcinoma d) Lymphoma a) Tuberculosis b) Rheumatoid arthritis
108. Which of the following signs suggest benign nature c) Pneumoconiosis d)COPD
in the evaluation of solitary pulmonary nodule on e) Metastasis
CTscan? (DPG Mar. 09) 119. Miliarymottlingfoundin- (PGI June 06)
a) Air bronchogram in nodule a) Rheumatoid arthritis
b) Presence of amorphous calcification b)1B
c) Fat density within the lesion c) Pneumocystis carinii pneumonia
d) Spiculated outline d) Congestive heart failure
109. A patient presents with a solitary pulmonary nodule l:'ulmonaty edema
(SPN) on X-ray. The best investigation to come to a
diagnosis would be- (AIIMS 02)
a) MRI b) CT Scan
c) US(} d) Image guided biopsy
110. Hypertranslucency oflung unilaterally is seen on
allexcept- (PGI 2000)
a) Pneumothorax a) Histoplasmosis
b) Resection of mammary gland b) Sarcoidosis
c) Pulm art. obstruction c) Secondaries from Ca. colon
d) Pneumonectomy d) Gonococal pneumonia

lOO)a IOl)a 102)a,b,c,e 103)b 104)a 105)b 106)b 107)d 108)c 109)d llO)d lll)b 112)c l13)a
114)d 115)c,d 116)a 117)a,b,c 118)a,b,c,e 119)a,b,e 120)c 12l)a,b,c
RADIOLOGY [ 548]

122. Miliarymottling on chest radiograph can be seen in- 135. FeaturesofpneumothoraxareaUexcept- (MP05)
a) Pneumoconiosis b) TB (MH 05) a) Devoid oflung markings
c) Mitral stenosis d)All b) Hypertranslucency
123. Egg shell calcification found in -(PG/June 06, PGI 03) c) Shift of mediastinum to same side
a) Silicosis b) Sarcoidosis d) Collapse of ipsilateral lung
c) Metastatic node d)Lymphoma 136. All of the following are indirect radiologic signs of
e) Tuberculosis collapse oflung EXCEPT- (Karn 11)
124. Egg shell calcification is seen in- (Punjab 11) a) Mediastinal displacement
a) Lymphoma b) Hilar displacement
b) Sarcoidosis c) Compensatory hyperinflation
c) Carcinoma metastatic to lymph node d) Loss of aeration
d) T.B. 137. A 55 year old man who bas been on bed rest for the
125. Mediastinal lymph node calcification is seen in past 10 days, complain of breathlessness and chest
which one of the following? (COMED 09) pain. The chest X-ray is normaL The next step in
a) Metastatic neoplasm b) Lymphoma investigation should be- (AI 04, 03)
c) Sarcoidosis d) Bronchiectasis a) Lung ventilation - perfusion scan
126. Egg shell calcification is seen in all except- b) Pulmonary arteriography
a) Sarcoidosis (AIIMS 98) c) Pulmonary venous wedge angiography
b) Silicosis d) Echocardiography
c) Post irradiation lymphoma 138. Best method to diagnose pulmonary embolism-
d) Bronchogenic CA a) Pulmonary angiography (PGI97, AP 95)
127. Egg shell calcification is seen in- (PGI June 09) b) Scintillation perfusion scan
a) Sarcoidosis b) Sillicosis c) CTScan
c) Lung Ca d) pneumoconiosis d) X-ray chest
e) Lymphoma following radiation treatment
139. In patient with high clinical suspicion of pulmonary
128. "Egg shell" calcification is seen in- (PGI OJ)
thromboembolism, best investigation would be?
a) Bronchiolitis b) Silicosis
a) D-dirner (AIIMS Nov 07)
c) Ca. bronchus d) Sarcoidosis
b) CT angiography
e) Histoplasmosis
c) Catheter angiography
129. Egg shell calcification in hilar lymph nodes is seen
d) Color Doppler
in- (PGIOJ)
a) Sarcoidosis b) Histoplasmosis
140. Good standard for diagnosing pulmonary embolism-
a) X-ray chest (PGI 97)
c) Tuberculosis d) Carcinoma lung
e) Silicosis b) Ventilation perfusion scan
130. Egg shell calcification is seen in- (PGI June 09) c) Blood gas analysis
a) Sarcoidosis b) Sillicosis d) Doppler
c) Lung Ca d) pneumoconiosis 141. Investigation of choice for pulmonary embolism-
e) Lymphoma following radiation treatment a) CT scan (AI 98)
131. Popcorn calcification is characteristically seen in- b) Contrast CT
a) 1B (AI 97) c) Ventilation - Perfusion scan
b) Metastasis d)MRI
c) Pulmonaryhamartoma 142. Pulmonary embolism is best diagnosed by-
d) Fungal invagination a) USG (MH 02, AI 96)
132. Well defined lesion in tbe lung with popcorn b) X -ray chest
calcification on X-ray suggestive of- (Delhi 2000) c) Ventilation- Perfusion scan
a) Ca lung b) Adenoma d) CT scan
c) Hamartoma d) Hydatid cyst 143. In pulmonary embolism, findings in perfusion scan
133. Chest X-ray showing opacity in the lung with is - (PG/2000)
irregular calcification is suggestive of- (MH 10) a) Perfusion segmental defect
a) Lung carcinoma b) Hamartoma b) Perfusion defect with normallWlg scan & radiography
c) Hydatid cyst - d)Adenoma c) Tenting of diaphragm
134. Characteristics of BENIGN tumour ofluug in X-ray d) Normal chest scan
is- (Assam 99) 144. Hamptonhumpisfeatureof? (AIIMSNov07)
a) Size> 5 ems diameter a) Pulmonary tuberculosis
b) Cavitation b) Pulmonary embolism
c) Peripheral location c) Pulmonary hemorrhage
d) Concentric dense calcification d) Bronchogenic carcinoma

l22)d 123) a,b,d,e 124) a,b,d 125) b,c 126) d 127) a,b,d,e 128) b,d,e 129) a,b,c,e 130) a,b,d,e 13l)c 132)c
133)b 134)d 135)c 136)d 137)a 138)c 139)b 140)None>b 14l)b 142)d 143)a,b,d l44)b
RADIOLOGY [ 549]

145. Uremiclungmostoftenresultsdueto- (PGI98) 154. Apgar scores were 3, and 6 at 1 and 5 minutes. At 10
a) Pulmonary edema b) Fibrosis minutes child shows features ofbreathlessness,on
c) Alveolarinjury d)CVCliver, CXR-mediastinal shift was there, possible causes -
146. A young man with pulmonary tuberculosis presents a) Bilateral choana! atresia (PGI 2000)
with massive recurrent hemoptysis. For b) Pneumothorax
angiographic treatment, which vascular structure c) Congenital diaphragmatic hernia
should be evaluated first- (AI 04, AIIMS 03, 02) d) Hyaline membrane disease
a) Pulmonary artery b) Bronchial artery 155. A child with acute respiratory distress showing
c) Pulmonary vein d) Superior vena cava hyperinflation of unilateral lung in X-ray is due to-
147. A 40 years old man presents with a recurrent a) Staphylococcal bronchopneumonia (AI 02)
hemoptysis and purulent cough. X-ray was found to b) Aspiration pneumonia
benonnaL The next investigation done to aid in c) Congenital lobar emphysema
diagnosis is- (AIIMS02, OJ) d) Foreign body aspiration
a) MRI b) Bronchoscopy 156. Pappu 2 yrs old boy presents in the causality with
c) HRCT d) CT guided biopsy B/0 sudden onset of respiratory difficulty & stridor
148. The following is not the differential diagnosis of an on auscultation, ..l.ed breath sound & wheeze on the
anterior mediastinal mass- (AIIMS 02, MP 99) RT side. The X-ray shows RT opaque hemithorax.
a) Teratoma b) Neurogenic tumor What will be diagnosis- (AI 02)
c) Thymoma d) Lymphoma a) Pneumothorax
149. Which among the following is a cause of posterior b) Acute epiglottitis
mediastinal opacity on Posterior--Anterior (PA) and c) Massive pleural effusion
lateralviewofchestradiograph? (Kam 11) d) Foreign body aspiration
a) Tortuous innominate artery 157. In X-ray, loops ofbowel on left sideofhemithorax and
b) Bochdalek's Hernia shift of heart shadow- (PGI 98)
a) Eventration of diaphragm
c) Enlarged pulmonary artery
b) Foramen ofbochdalek hernia
d) Thymoma
c) Morgagni 's hernia
150. Commonest mass in the middle mediastinum is-
d) Any of the above
a) Lipoma (AIIMS Sep 96)
158. A 25 year old man presented with fever, cough,
b) Aneurysm expectoration and breathlessness of 2 months
c) Lymph node mass duration. Contrast enhanced computed tomography
d) Congenital cysts of the chest showed bilateral upper lobe fibrotic
151. The most likely diagnosis in a newborn who had a lesions and mediastinum had enlarged necrotic
radiopaque shadow with an air fluid level in the chest nodes with peripheral rim enhancement. Which one
along with hemivertebrae of the ()th thoracic vertebra of the following is the most probable diagnosis?
onplainX-rayis- (AIIMS02) a) Sarcoidosis (AIIMS 04, AI 03)
a) Congenital diaphragmatic hernia b) Tuberculosis
b) Oesophageal duplication cyst c) Lymphoma
c) Bronchogenic cyst d) Silicosis
d) Staphylococcal pneumonia 159. A patient suffering from AIDS presents with history
152. A child has respiratory distress, chest X.;.ray shows of dyspnea and non-productive cough x-ray shows
multiple air filled space, the differential diagnosis bilateral perihilar opacities without pleural effusion
is all except- (AIIMS 98) and lymphadenopathy. Most probable etiological
a) Congenital lung cyst agentis- (AIIMS02)
b) Congenital diaphragmatic hernia a) Tuberculosis b)CMV
c) Congenital lobar aplasia oflung c) Kaposis sarcoma d) Pneumocystis carinii
d) Congenital adenomatous malformation 160. Centrally located cavitating lung lesion is most likely
153. A neonate presents with respiratory distress, to be- (MP 99)
a) Sarcoma b) Small cell carcinoma
contralateral mediastinal shift and multiple cystic
c) Adenocarcinoma d) Squamous cell carcinoma
air filled lesions in the chest. Most likely diagnosis
161. Which ofthe following organs should always be
is- (AI OJ)
imaged in a suspected case of broncogenic
a) Pneumonia
carcinoma- (AIIMS Nov 04)
b) Congenital lung cyst a) Adrenals b) Spleen
c) Congenital diaphragmatic hernia c) Kidney d) Pancreas
d) Congenital lobar emphysema

145)a 146)b 147)b 148)b 149)b 150)c 15l)b 152)c 153)c>d 154)b,c 155)d 156)d 157)d 158)b
159)d 160)d 161)a
RADIOLOGY [ 550 ]

162. Soft tissue mass in chest with rib erosion in X-ray is CARDIOVASCULAR SYSTEM
seen in all except- (AIIMS 96)
a) Leukemia b) Ewing's sarcoma 174. In all ofthe following increased cardiac silhouette
c) Multiple myeloma d) Osteosarcoma sign is seen except- (MP 2000)
163. Unilateral elevation of diaphragm is commonly due a) Tetralogy ofF allot's b) Pericardia! effusion
to- (CMC96) c) Aortic regurgitation d) Ebstein anomaly
a) Obesity b) Large liver 175. Ifthe right cardiac silhouette is obliterated, it means
c) Scoliosis d) Congenital causes the pathology involves- (AIIMS May 08)
164. Sequestrated lung is most commonly supplied a) Right lower lobe b) Right atrium
by- (MH 08, 06; Jipmer 00) c) Right middle lobe d) Right ventricle
a) Bronchial artery b) Subclavian artery 176. Left cardiac border bulge can be seen in all, except-
c) Puhnonary artery d) Ascending aorta a) Enlarged azygous vein (AI 2000)
165. In rt. sided hemithorax on chest X-ray PA view what b) Left appendicular overgrowth
can be excluded- (PGI 97) c) Coronary artery aneurysm
a) a:F b)TB d) Pericardia! defect
c) Puhnonaryinfarct d) None of the above 177. Left sided cardiac bulge seen on chest X-ray is/are
166. The nodular pattern often seen in the lungs following ditto- (PGI May 10)
renal transplantation is thought to be due to- a) Enlargement ofleft atrial appendage
a) Fungus b) Inafarcts (CMC 96) b) Azygous vein enlargement
c) Cytomegalovirus d) Autoimmune reaction c) Coronary artery aneurysm
167. Chest X-ray shows opacity after blunt trauma, most d) Puhnonary edema
likely diagnosis is- (MP 99) e) Right atria 1hypertrophy
a) Haemothorax b) Pneumothorax 178. Wbicbamongtbefollowingisnotachestradiographic
c) Pneumonia d) Puhnonary embolism feature ofleft atrial enlargement? (Karn 11)
~68•.\·~tU:Dagijlg modalifY·iitbi(fJ!¢1iit~~:~w~!l~-4 a) Double left heart border

;· •.i~;;i.•~6pchoscop; ·:i=:~S~f~rv~&0!tf:~}!
.•
b) Elevated left main bronchus
c) Splaying of carina
169. Investigation of choice for detection & d) Enlargement ofleft atrial appendage
characterization of interstitial lung disease is- 179. Earliest sign of left atrial enlargement is - (AI 08)
a) MRI (DPG 08, AI 08,Maharashtra 07 a) Posterior displacement of esophagus
b) Chest X-ray AIIMS 05) b) Widening of carinal angle
c) High resolution CT scan c) Elevation ofleft bronchus
d) Ventilation perfusion scan d) Double shadow of right border
170. High resolution computed tomography of the chest 180. A patient is having mitral stenosis. His X-ray will
is the ideal modality for evaluating- (AI 03) show all ofthe following finding except- (AIIMS 02)
a) Pleural effusion b) Interstitial lung disease a) Lifting up ofleft bronchus
c) Lung mass d) Mediastinal adenopathy b) Double atrial shadow
171. Best method to diagnose Bronchiectasis is- c) Obliteration of retrostemal shadow on lateral X-rays
a) X-ray b) Bronchography (Karn OJ) d) Posterior displacement of esophagus on barium
c) M.R.I. d) HRCT swallow
172. A3 year old female child developed fever, cough and 181. All are radiological features ofMitral stenosis except-
respiratory distress. On chest X-ray consolidation a) Straight left border of heart (AIIMS 2000)
is seen in right lower lobe. She improved with b) Oligemia ofupperlung fields
antibiotics but on follow up at 8 weeks was again c) Puhnonary hemosiderosis
found to have increasing consolidation in right lower d) Lifting ofleft bronchus
lobe. Your next investigation would be- 182. Radiological feature of mitral stenosis is/are -
a) Bronchoscopy (AIIMS Nov 07) a) Double contour of right heart border
b) Bacterial culture ofthe nasopharynx b) Straightening ofleft heart border
c) CT scan of the chest c) Splaying of carinal angle (PGI June 09)
d) Allergen sensitivity test d) Prominent aortic knuckle
173. Chest X-ray of an industrial worker exposed to e) Kerely lines
asbestos over 20 years shows an ill-defined rounded 183. Earliest radiological sign of pulmonary venous
opacity in the lower lobe with a 'comet tail' appearance hypertension in chest X-ray is- (AI 08)
on PA view. Which ofthe following is tbe most likely a) Cephalization of pulmonary vascularity
diagnosis- (AI 12) b) Pleural effusion
a) Mesothelioma b) Bronchogenic carcinoma c) Kerley B lines
c) Round Atelectasis d) Puhnonary infarct d) Alveolar pulmonary edema

l62)a 163)b,c l64)d l65)a 166)c 167)a 168) a 169) c 170) b 17l)d 172)c 173)c 174)a 175)c
176)a 177)a 178)a 179)a 180)c l8l)b 182)a,b,c,e 183)a
RADIOLOGY [ 551 ]

184. Earliest feature of pulmonary venous hypertension 194. Plethoric lung fields are seen in all ofthe following
is - (DPGEE 08) conditions, except- (AIIMS May 06)
a) Upper lobar vessel dilatation b) Kerley B lines a) Atrial septal defect (ASD)
c) Left atrial enlargement d) Pleural effusion b) TAPVC(Total anomalouspulmonary venous
185. Features ofpulmonary venous hypertension are AlE- connection)
a) Perihilarhaze (PGI 06, Jipmer 05) c) Ebstein's anomaly
b) Peribroncheal cuffing Ventricular defect
c) Upper lobar diversion
d) Uniformly branching lines parallel to pleura
e) Pulmonary ossicles & fine nodular pattern
196. Spring water cyst is another name for- (PGI 97)
186. Chest X-ray picture in CCF- (PGI Dec 04)
a) Hydatid cyst oflung b) Lung amoebic cyst
a) Cardiomegaly
c) Pleura pericardia! cyst d) Enterogenous cyst
b) Thick interlobar septum
197. Egg-on-side' appearance on X-ray chest is seen in-
c) Superior mediastinal widening a) Tetralogy ofFallot (AIIMS May 08, Nov 07
d) Multinodular parenchymal lesion b) Uncorrected TGA AI 10, PGI Nov 09)
187. All are seen in congestive cardiac failure except- c) Tricuspid atresia
a) Kerley B lines (AI 2000) d) Ebstein's anomaly
b) Prominent lower lobe vessel 198. Egg in cup appearance is seen in- (Kerala 04)
c) Pleural effusions a) TOF
d) Cardiomegaly b)MR
188. Left heart failure findings are all except- c) Constrictive pericarditis
a) Kerley B lines (AIIMS May 09) d) Transposition of great vessels
b) Redistribution of blood vessels to apex
c) Oligemic lung field
d) Cardiomegaly
189. Prunning of pulmonary arteries is seen in- (AI 09)
a) Pulmonary hypertension
b) Chronic bronchitis Figure of 8 in chest
c) Pulmonary infections a) Ebsteinanoma1y (AI 2K, PGI May, 10)
d) Pulmonary transplant b) Total anomalous pulmonary venous connection
, tlib'D.;cnest~ (TAPVC)
c) Tetrology of fallot
d) Transposition of great vessels
201. In which ofthe following a 'Coeur en Sabot' shape of
the heart is seen- (AI 2000, 04)
a) Tricuspid atresia
b) Ventricular septal defect
c) Transposition of great arteries
a) Small Lt. ventricle b) Small Rt. ventricle d) Tetralogy offallot
c) Dilated Lt. atrium d) Dilated pulmonary veins 202. X-ray chest showing globular cardiomegaly with
e) Dilated pulmonary arteries oligemic lung fields in a child is suggestive of-
192 . X-ray features ofASD are all except- (MH 07) a) TOF b)TAPVC (MH 10)
a) Right atrial enlargement c) Ebstein'sanomaly d)PDA
b) Left atrial enlargement 203. Which of the following is the most common feature
c) Pulmonaryplethora ofAortitis on chest X-ray- (AI 08)
d) Small aortic knockle a) Calcification of ascending aorta
193. Which is the objective sign ofidentifying pulmonary b) Calcification of descending aorta
plethora in a chest radiograph? (AI 11) c) Calcification of pulmonary artery
a) Diameter of the main pulmonary artery> 16 mm d) Focal oligemia
b) Diameter of the left pulmonary artery> 16 mm ~g~,~f;,f;!l~~ij?lg!~~~y~tfi~~Iar,~~P~Irtis,s~~n i!l'" ... : , ,
c) Diameter of the descending right pulmonary artery .. ;;:''~\WP~. .o·.:·· .]t),f!g~M,•:t~::. ;f!#J;~'f!D"{B
>16mm · ~) 'fGA .............·. . . . d)(}HF .: . : . ., lJ.attern)
d) Diameter of the descending left pulmonary artery to
205. Drng used perlorin stress E(5.Ho: ···· · ·· ·
>16mm a) Thallium b) Dobutarnine (AIIMS May 08)
c) Adrenaline d) Adenosine

184)a 185)d 186)a,b 187)b 188)c 189)a 190)c 191)c,e 192)b 193)c 194)c 195)b 196)c 197)c
198)c 199) b 200)b 201)d 202)c 203)a 204) b 205)b
RADIOLOGY [ 552)

206. Investigation of choice for Aortic dissection is- 216. Isotope used in myocardial perfusion scan is-
a) Aortography (AIIMS 97, AI 96) a) Technetium (ROHTAK 95)
b) CTscan b) Thallium
c) MRI c) Stannous pyrophosphate
d) X-ray chest d) Gallium
207. Most important investigation for pericardial 217. Acute myocardial infarct scintigraphy is done with-
effusion- (MP 98, AI 97) a) Thallium (AP 98)
a) Cardiac catheterization b) USG b) Gallium
c) Echocardiography d) X-ray chest c) Neodynium
208. Cardiotoxicity caused by radiotherapy & d) Tc stannous pyrophosphate
218. In diagnosis of acute myocardial infarction hot spot
chemotherapy is best detected by- (AI 99)
is seen with - (AP 97)
a) ECHO
a) Thallium201 scan b) Tc 99 scan
b) KG
c) Strontium 90 scan d) L 127 scan
c) Radionucletide scan
d) Endomyocardial biopsy
209. Test of choice for Reversible Myocardial Ischemia-
a) Thallium scan b) MUGA scan (AI 09)
c) Resting ECHO d) Coronary angiography
210. Which of the following is not true regarding
myocardial ischemia- (AIIMS Nov. 11)
a) Late enhancement on Gd MRI suggestive of scar is done for- (MP 02)
b) Akinetic area does not benfit by revascularization a) Testing drug toxicity
c) Low dose dobutamine may be used in hibernating b) Ventricular function & Anthracyclin induced
myocardium toxicity
d) Rest reinfection thallium is used in hibernating c) Myocardial perfusion
mycardium d) None
211. In a Down's syndrome patient posted for surgery, 221. Which one of the following investigations is
the necessary preoperative investigation to be done considered to be "Gold standard" technique for
is- (AI 09) diagnosis of arterial occlusive disease -(UPSC-II 09)
a) CTBmin b) Echocardiography a) Doppler ultrasound blood flow detection
c) Ultmsound Abdomen d) X-ray cervical spine b) Duplex imaging
212. The most accurate investigation for assessing c) Treadmill
d) Digital substraction angiography (DSA)
ventricular function is- (AI 06)
222. Rib notching is found in - (PGI 01)
a) Multislice CT b) Echocardiography
a) Neurofibromatosis
c) Nuclear scan d) MRI
b) Lymphangiomyomatosis
213. Most sensitive investigation for air embolism is-
c) Aortic aneurysm
a) Decreased tidal volume of C02 (AI 99) d) Taussig-Bing operation
b) Decreased tidal volume ofN02 e) Aortic obstruction
c) Doppler ultrasound 223. Inferior rib notching is seen in- (AI 08)
d) Central venous presure a) Coarctation of aorta b) Rickets
214. About diagnosing air embolism with c) ASD d)Multiplemyeloma
transesophageal echocardiography, which of the 224. Which of the following causes rib-notching on the
following is false - chest radiograph? (AI 06)
a) It can quantify the volume of air embolised a) Bidirectional Glem shunt
b) It is a very sensitive investigation (AIIMS 02) b) Modified Blalock-Taussing shunt
c) Continuous monitoring is needed to detect venous c) IVC occlusion
embolism d) Coarctation of aorta
d) Interferes with doppler when used together 225. True regarding radiological picture of coarctation
215. The most recent advance in noninvasive cardiac ofaortaAIE- (PGI04, WB05)
output monitoring is use of- (AIIMS 02) a) Involvement of upper two ribs
a) PA catheter b) Bilateral
b) Thermodilution technique c) Inferior rib notching
c) Echocardiography d) Usually before 5 years of age
d) Electrical impedance cardiography technology e) '3' sign

206)c 207)c 208) d 209)a 210)b 21l)b 212)b 213)c 214)d 215)d 216) a,b 217)d 218)b 219)c
220)b 22l)d 222)a,d,e 223)a 224)d>b 225)a,d
RADIOLOGY [ 553]

226. Bilateral rib notching is seen in- (MH 05) 236. Investigation of choice for Zenker's diverticulum is-
a) Coarctationofaorta b)PDA a) Barium swallow (AIIMS Nov. 11)
c) TAPVC d) All ofthe above b) Endoscopy
227. Rib notching is seen in all except- (AIIMS Nov 09) c) Esophageal manometry
a) Waterston Cooley shunt d)CT
b) Aortic obstruction 23 7. Diffuse esophageal dilatation on barium swallow is
c) Blalock Taussig shunt seen in - (PGI June 05)
d) Pulmonary atresia with large VSD a) Achlasia b) Trypanosomiasis
228. Superior rib notching is/are caused by-(PG/June 09) c) Etidronate therapy d) Scleroderma
a) Hyperparathyroidism b) Poliomyelitis
c) Blalock tausing shunt d) Marfan syndrome
e) Coarctation of aorta
229. Radiological features of pericardial effusion AlE -
a) Flask shaped heart (MH 05)
b) Pulmonary oligemia
c) Cardiomegaly 239. Cork-screw opperance of esophagus is seen-
d) Pulmonary plethora a) Achalasia cardia (MH-11)
230. A child presents with respiratory distress. A b) Carcinoma esophagus
vascular ring is suspected. Investigation of choice c) Diffuse esophageal spasm
is - (AIIMS Nov 09) Caustic
~PET b)CT
c) MRI d) Angiography
231. An asymptomatic old patient presents with bruit in
the carotid artery. Which of the following is the
investigation of choice? (Jipmer 11)
a) Doppler ultrasonography
b) Internal carotid angiography
c) Aortic arch angiography
d) Spiral CT angiography
232. On barium swallow which of the following will cause
242. Double bubble sign in children is seen in AlE -
posterior impression- (AJIMS Nov 09)
a) Ladd's band (PGI Dec 04)
a) Leftatrium
b) Annular pancreas
b) Aortic knuckle c) Pancreatic pseudocyst
c) Pulmonary sling d) Duodenal atresia
d) Aberrant right subclavian artery e) Diaphragmatic hernia
233. Which artery is dissected most commonly following 243. Double bubble sign is seen in AlE- (Punjab 11)
arteriographybyfemoralroute- (AIIMS May 11) a) Duodenal atresia b) Diaphragmatic hernia
a) Celiac trunk c) Ladd band d) Annular pancreas
b) Superior mesenteric artery 244. Double bubble sign on X-ray is seen in-(AIIMS 99)
c) Inferior mesenteric artery a) Duodenal atresia b) Oesophageal atresia
d) Gastroduodenal artery c) Colonic atresia d) Pyloric stenosis
234. Duringcardiacimagingthephaseofminimummotion 245. "Double bubble" sign with air shadows absent in
ofheartis- (AI 2010) distal bowel coils on X-ray abdomen in characteristic
a) Late systole b) Mid systole of- (MH 10, 06)
c) Late diastole d) Mid diastole a) Duodenal webs b) Duodenal atresia
c) CHPH d) All of the above
GIT AND HEPATOBILIRAY SYSTEM

235. Investigation of choice in diffuse esophageal spasm


Carman's meniscus sign is uu1~~:,uu1u.a\;
is -
a) Manometry (AI 08) a) Peptic ulcer
b) Esophagoscopy b) Cholecystitis
c) Carcinoma of stomach
c) Barium examination showing tertiary contractions
d) Meconium ileus
d) CTthorax

226)a 227)a 228)a,b,d 229)d 230)c 23l)a 232)d 233)c 234)d 235)a 236) a 237) a,b 238) b 239) c
240)c 24l)a 242)c,e 243)b 244)a 245)b>a 246)b 247)c
RADIOLOGY [ 554]

248. Following are common features of malignant gastric 259. String sign is seen in - (Kerala 98)
ulcer on Barium meal, Except- (AIIMS May 04) a) Crohns disease
a) Location on the greater curvature b) TB of the ileocaecal region
b) Cannan's meniscus sign c) Idiopathic hyperophic pyloric stenosis
c) Radiating folds which do not reach the edge of d) All of the above
the ulcer 260. String of Kantor is seen in - (AIIMS 96, UP 03,
d) Lesser curvature ulcer with a nodular rim a) Chron's disease b) Ulcerative colitis Kerala 2K)
249. Trifoliate appearance is in - (Jharkhand 06) c) TB d) Carcinoma
a) Peptic ulcer 261. "String of beads" appearance on abdominal X-ray
b) Pyloric stenosis is seen in - (MH 07, MP 06)
c) Carcinoma head of pancreas a) Small bowel obstruction
d) None b) Large bowel obstruction
250. X-ray feature of pyloric stenosis is- (AIIMS 97) c) Carcinoma of stomach
a) Single bubble appearance d) Gastric outlet obstruction
b) Double bubble appearance 262. Triad of vomiting, abdominal distension and "String
c) Triple bubble appearance of beads" sign on abdominal X-ray is typically
d) Multiple air fluid levels suggestive of- (MH09)
a) Duodenal atresia
b) Small bowel obstruction
c) Large bowel obstruction
d) Gastric volvulus
252. All correlates with USG finidngs of pyloric stenosis 263. The following are radiological features of sigmoid
except- (AIIMS Nov. 11) volvolus except - (Kern 04)
a) Accuracy 95% a) Inverted U shaped bowel loop
b) High gastric residue b) Liver overlap sign
c) Segment length> 16 mm c) Bird of prey
d) Thickness of pylorus> 4 mm d) Cupola sign
253. A newborn presenting with intestinal obstruction 264. "Coffee bean" sign is seen in - (MP 04)
showed on abdominal X-ray, multiple air fluid levels. a) Intussusception
The diagnosis is not likely to be - (AI 02) b) Bowel ischaemia
a) Pyloric obstruction b) Duodenal atresia c) Sigmoid volvulus
c) Illeal atresia d) Ladd s bands d) Congenital hypertrophic pyloric stenosis
254. Which of the following is earliest sign of ulcerative 265. X-ray appearance ofCBD stone on cholangiography
colitis on double contrast barium enema study - is - (AIIMS 98)
a) Mucosal granularity (MH 05) a) Meniscus appearance b) Sudden cut off
b) Loss of haustrations c) Smooth tapering d) Eccentric occlusion
c) 'Collar- button' ulcer 266. All of the following are diagnostic barium follow
d) Lead pipe colon through features of ileocecal tuberculosis Except-
255. Radiological signs of crohn's disease-(PG/ June 08) a) Apple - core appearance (AI 08)
a) String sign of Kantor b) Pulled up contracted cecum
b) Pipestem appearance c) Widening of ileocecal angle
c) Pseudopolyp d) Strictures involving terminal ileum
d) Backwash ileitis 267. Ileocecal tuberculosis presents with all except-
256. Lead pipe appearance is seen in - (PGI 94, UP 02) a) Rapid emptying of narrowed terminal ileum
a) Chron's disease b) Ulcerative colitis b) Inverted umbrella sign (PGI 06, Jipmer 07)
c) Schistosomiasis d) Carcinoma colon c) Stellate ulcer with elevated margins
257. Scalloping ofthe edge of sigmoid colon on barium d) Longitudinal ulcers are more common
enema seen in - (CMC 2K) e) Napkin ring stenosis
a) Diverticulosis b) Ulcerative colitis 268. Radiological finding in ileal atresia - (PGI 00)
c) Carcinoma colon d) Pneumatosis intestinalis a) Microcolon on Ba enema
258. String sign is a radiological feature of- (MH 06) b) Double bubble sign
a) Crohn's disease b) Ulcerative colitis c) Coil spring appearance in Ba-Enema
c) Ileocecal tuberculosis d) Ischemic colitis d) Obstruction in Ba meal

248)d 249)a 250)a 25l)b 252)b 253)a 254)a 255)a 256)b 257)b 258)c 259)d 260)a 261)a
262)b 263)d 264)c 265)a 266)a 267)d 268)a,d
RADIOLOGY [ 555 ]

269. Following are features of ischaemic colitis except- 279. In case of suspected perforation which view is best-
a) Thumb printing (MP 2000) a) Erect b) Supine (Jharkhand 03)
b) Serrated mucosa c) Lateral decubitus d) None
c) Increased mucosal fold thickness 280. The best view to visuali:re minimum pneumoperitoneum
d) Dilution of barium is- (Delhi 95, AI 12)
270. Radiological feature of ischemic colitis is - a) AP view of abdomen
a) Saw toothing (Karn 01, UP 99, PGI 96)
b) Erect :film of abdomen
c) Rt lateral decubitus with horizontal beam
b) Craggy popcorn appearance
d) Lt lateral decubitus with horizontal beam
c) Thumb printing
281. Cupola sign- (Jharkhand 06)
d) Cobble stone appearance a) Radiological finding in supine posture for
271. Soap bubble appearance in X-ray is seen in - pneumoperitoneum
a) Multiple cystic kidney (AIIMS 99) b) Radiological finding in supine posture for
b) Neuroblastoma pneumothorax
c) Cystic lymphangiectasis c) Air in Morrison pouch
d) Meconium ileus d) Both wall of bowel is seen
272. Feathery appearance in jejunum is due to-(PGI 99) 282. Typical "Saw-tooth" colon on barium enema is seen
a) Valvulae conniventes b) Haustrations with- ~HO~
c) Luminal gas d) Vascular network a) Colonic diverticulosis b) Colonic volvulus
273. Which among the following is false regarding small c) Colonic carcinoma d) Ulcerative colitis
bowel appearance on abdominal radiograph? 283. "Hat sign" on double contrast barium enema. is seen
a) Valvulae conniventes are present (Karn 11) in- ~H09)
b) Peripheral distribution a) Ulcer b) Polyp
c) Radius of curvature is small c) Carcinoma d) Diverticulum
d) Solid faeces are absent 284. Couinaud's segments are used to divide which organ-
274. Small intestine in intestinal obstruction is a) Liver b) Lung (MH 02)
distinguished radiologically from large intestine by- c) Spleen d) Kidney
a) Haustration (MP 2005) 285. According couinaud's classification of functional
b) Valvulae conniventes segments of liver, which of the following is segment
c) Can not be distinguished IV of liver? (AIIMS Nov 07)
None a) Left lobe b) Right lobe
c) Caudate lobe d) Quadrate lobe
286. A 22 year old man presents with a solitary 2 em
space occupying lesion of mixed echogenicity in
276. Investigation for small intestine includes all except- the right lobe of liver on ultrasound examination.
a) Enteroclysis (PGI Dec 05) The rest of the liver is normal. Which of the
b) Radionucleide enteroclysis following tests should be done next-(AIIMS Nov 02)
c) MRI enteroclysis a) Ultrasound guided biopsy of the lesion
d) CT enteroclysis b) Hepatic scintigraphy
e) USG enteroclysis c) Hepatic angiography
277. Which among the following is false regarding need of d) Contrast enhanced CT scan of the liver
chest radiograph in a patient ofacuteabdomen-(Kam 11) 287. Solitary hypoechoice lesion of the liver without
a) Supine chest radiograph is the best radiograph septate or debris is most likely to be-(AIIMS Nov 05)
for showing presence ofa small pneumoperitoneum a) Hydatid cyst b) Caroli's disease
b) A number of chest conditions can present as acute c) Liver abscess d) Simple cyst
288. Which one of the following hepatic lesions can be
abdominal condition
diagnosed with high accuracy by using nuclear
c) Acute abdominal condition may be complicated
imaging? (AIIMS Nov 04)
by the chest pathology
a) Hepatocellular carcinoma
d) Even when the chest radiograph is normal it acts
b) Hepatic adenoma
as a most valuable baseline
c) Focal nodular hyperplacia
278. Free gas in abdomen (under the diaphragm) can be
d) Cholangiocarcinoma
best diagnosed by which X-ray - (MH 02)
289. CT finding of acute pancreatitis are all except-
a) Standing a) Dilation of pancreatic duct (UP 2K)
b) Right lateral recumbent view b) Fuzzy outline of pancreas
c) Left lateral recumbent view c) Peripancreatic fluid collection
d) Sitting position d) Edematous pancreas

269)d 270)c 271)d 272)a 273)b 274)b 275)a 276)b,c,d,e 277)a 278)a 279)a 280)None&d 28l)a
282)a 283)b 284)a 285)d 286)b 287)d 288)c 289)a
RADIOLOGY [ 556]

290. CT findings of acute pancreatitis are - (MP 00) 303. True about OCG is- (PGI 2000)
a) Fuzzy outline of pancreas a) First done by Graham Cole in 1942
b) Dilatation of pancreatic duct system b) Dye ingested at rate of 1 ml!kg
c) Peripancreatic fluid collection c) USG has replaced it
d) All d) Dye used is telepaque
291. Radiological signs of acute pancreatitis include the 304. Which is not required for visualisation of gall bladder
followingexcept- (Maharashtra 08) in oral cholecystography- (AIIMS 9 5)
a) Functioning liver
a) Colon cut-off sign b) Cullen's sign
b) Motor mechanisms of gall bladder
c) Renal halo sign d) Sentinel loop sign
c) Patency of cystic duct
292. Colon cut off sign seen in - (MP 06) d) Ability to absorb water
a) Acute pancreatitis b) Diverculities 305. The investigation of choice for acute cholecystitis is-
c) Appendicitis d) Carcinoma colon a) USG b)HIDA-Scan(PGI02)
293. Chain oflakes appearance in ERCP is seen is-(AJ 96) c) CT-Scan d)X-ray
a) Acute pancreatitis b) Chronic pancreatitis 306. Investigation of choice in obstructive jaundice is-
c) Carcinoma pancreas d) Ductal adenoma a) ERCP b)USG (AI 97)
294. Widening of the C loop in X-ray is diagnostic of- c) Cholecystography d) Laproscopy
a) Chronic pancreatitis (DPG Feb. 09) 307. Investigation of choice for gall stone - (PGI 0 J)
b) Carcinoma head of pancreas a) X-ray b)USG
c) Periampullary carcinoma c) Cholecystography d) CAT Scan
d) Calculi in the ampullar ofVater 308. Most common investigation done for obstructive
295: "Spongy appearance" with central sunburst jaundice- (AIIMS 99)
calcification is seen in - (AIIMS May 07) a) CT scan b) USG
a) Pancreatic adenocarcinoma c) X-ray d)ERCP
309. Minimal ascites can be best detected by- (AIIMS 98)
b) Mucinous cyst adenocarcinomas
a) USG b)PlainX-rayabdomen
c) Somatostatinoma
c) MRI d) CT scan
d) Serous cystadenoma
310. Focal and diffuse thickening of gallbladder wall with
296. ERCP in pancratitis is done to know about-(PGI OJ) high amplitnde reflections and 'comet tail' artifacts
a) Gall stone b) Associated cholangitis on USG suggest the diagnosis of-
c) Ascites d) Pancreatic divisum a) Xanthogranulomatous cholecysitis
e) Annular pancreas b) Carcinoma of gall bladder
297. Most sensitive investigation of pancreatic carcinoma c) Adenomyomatosis (AI 09,AIIMS Nov 08)
is- (WB 95) d) Cholesterolosis
a) Angiography b) ERCP 311. Which ofthe following is not a diagnostic feature of
c) Ultrasound d) CT scan gall stone ileus on plain abdominal radiograph?
298. A patient complains of epigastric pain, radiating to a) Ectopic calcified gallstone (MH 03)
back otT and on. The investigation of choice is- b) Stone< 2.5 em size in the intestine
a) MRI b) CT scan (AIIMS 99) c) Signs of small intestinal obstruction
c) USG d) Radio nucleotide scan d) Gas in the biliary tree
299. Investigation of choice for small intestine tumor- 312. Air in biliary tract is seen in all except- (AIIMS95)
a) Gall stone ileus
a) Ba follow through (Jipmer 98)
b) Sclerosing cholangitis
b) Echo
c) Carcinoma gall bladder
c) X-ray abdomen
d) Endoscopic papillotomy
CT scan with contrast 313. Thickened gall bladder wall in USG seen in-
a) Acute cholecystitis (PGI Dec 04)
b) Mucosal thickening
c) Cholesterosis
301. Contrast used in barium enema is- (Jipmer 03) d) Ascites
a) Barium oxide b) Barium sulphide e) AIDS cholangitis
c) Barium sulphate d) Lead sulphate 314. Computed Tomography (CT scan) is least accurate
302. True about features of cholecystitis on USG- fordiagnostsof- (AI 11)
a) Thick fibrosed gallbladder wall (PGI OJ) a) Aneurysm in the Hepatic Artery
b) Stone impacted at neck of gall bladder b) Lymph node in the para- aortic region
c) Perigallbladder halo c) Mass in the tail of pancreas
d) Increased vascularity d) Gall stones

290)a,c 291)b 292)a 293)b 294) b >a 295) d 296) a,b,d,e 297) c 298) b 299) a 300) c 301) c 302) a,b,c
303) c,d 304) b 305)a 306)b 307)b 308)b 309)a 310)c 311)b 312)b 313)a,b,d,e 314)d
RADIOLOGY [ 557 ]

315. Central dot sign is seen in ~(AIIMS Nov 08, May 11) c) Nuclear scintigraphy
a) Caroli's disease d) Magnetic resonance imaging
b) Primary sclerosing cholangitis 327. Spleenic injury is diagnosed on X-ray by- (PG/97)
c) Polycystic liver disease a) Half stomach shadow
d) Liver hamartoma b) Obliteration of spleenic shadow
316. TrueaboutareMRCP- (PGI Dec 07} c) Rib fracture
a) MRI is used to obtain the image d) Gas under diaphragm
b) CT is used for the images 328. Best imaging modality for neuroendocrinal tumor~
c) It shows the biliary tree a) PECT (AIIMS OJ)
d) Dye has to be injected endoscopically b) CECf
e) It is an invasive procedure c) Radio nucleotide scan
317. Technitium-99 scan is used for diagnosis of- d) MRI with gadolinium scan
a) Meckel's diverticulum (Manipal 08) 329. Protein losing enteropathy diagnosis, all used except?
b) Appendix a) Tc albumin (AIIMS May 11)
c) Volvulus b) Tc dextran
d) Obstruction c) In transferrin
318. Investigation of choice for recurrent GIST- d) Tc seclosumab
a) MIBG
c) MRI
b) PET
d) CECT
(AIIMS Nov 08)
~aXt~~··.N~~~~.·.t~~~~n~r~~~~~~~s!~;~~~tJ,~t:;!j
319. Gasless abdomen is a feature of- (MP 0 J) a) Gala9tosenli~F. · ·.. ·.... Jj)iliti~~~~~~in[\:1· ·...
a) High obstruction · · ~c) ¥{~litJitiS::~·xi~Ir)~t~lli:~~~).·~~i~~r6t&~rtl~·~~:. ~15tc~·l
b) Acute pancreatitis
c) Congenital diaphragmatic hernia UROGENITAL SYSTEM
d) All
320. Frostberg's reverse 3 sign is seen in- (MH OJ) 331. Functional analysis of kidney is best done by -
a) Periampullary carcinoma a) Radionuclide scanning (DPG Feb. 09, PG/97)
b) Annular pancreatic carcinoma b)NP
c) Carcinoma head of pancreas c) Ultrasound
d) All d)MRI
321. DD for right upper quadrant calcification- 332. Which ofthe following agents is used to measure
a) Gallstone (PGI June 06) glomerular fdtration rate (GFR)- (AI 08, AI 95)
b) Renal stone a) Iodohippurate b) Tc99m- DTPA
c) Calcification in vessels c) Tc99m-MAG3 d)Tc99m-DMSA
d) Hepatic hemangioma 333. A patient presents with acute renal failure and
322. Calcific hepatic metastases are seen in-(Comed 09) anuria. The USG is normal. Which of the following
a) Adenocarcinoma ofthe colon investigation will give best information regarding
b) Carcinoid tumours renal function- (AI JO)
c) Renal cell carcinoma a) Intravenous pyelogram
d) Lymphoma b) Retrograde pyelography
323. USG is sensitive in- (PGI June 04) c) Antegrade pyelography
a) Ureteric colic b) Gall stone d) DTPA scan
c) Blunt abdominal trauma d) Appendicitis 334. The investigation of choice for Renal Scarring defect
e) Pancreatic pathology in Kidney is- (AI J2)
324. In aCT scan at the level of celiac trunk, following a) DMSA scan b) DTPA scan
structures will be seen- (PGI 02) c) DEXAscan d)MCU
a) Pancreas b) Gall bladder 335. A patient presented with ARFwith complete anuria
c) Inferior vena cava d) Duodenum but a normal ultrasound. Next investigation is-
e) Portal vein a) NP (A/99)
325. Investigation of choice for diagnosis of spleenic b) Ante grade pyelography
rupture~ (AIIMS95) c) Retrograde pyelography
a) Peritoneal lavage b) Ultrasound d) Radiorenogram
c) CT scan d) MRI 336. Adenserenogramisobtained by- (AIJO)
326. For the evaluation ofblunt abdominal trauma, which a) Dehydrating the patient
of the following imaging modalities is ideal? b) Increasing the dose of contrast media
a) Ultrasonography (AIIMS 04) c) Rapid (Bolus) injection of dye
b) Computed tomography d) Using non- ionic media

315) a 316) a,c 317) a 318)b>d 319)d 320)b 32l)a,b,c,d 322)a 323) b,c,d 324) a,c,e 325) c 326) b
327) b,c 328) c 329) d 330) b 331) a 332) b 333) d 334) a 335)d 336)c
RADIOLOGY [ 558]

337. A dense persistent nephrogram may be seen in all of 348. Teardrop bladder are seen in- (PGI May 1 0)
the following except· (AJIMS 02) a) Pelvic hematoma
a) Acute ureteral obstruction b) Pelvic lipomatosis
b) Systemic hypertension c) T.B.
c) Severe hydronephrosis d) Neurogenic bladder
d) Dehydration e) Intraperitoneal bladder rupture
338. Non-visualisation of kidney in excretory urogram 349. Which offollowing is not a feature of renovascular
is seen in - (PGI Dec 05, 04) hypertension on intravenous urography- (MH 05)
a) Duplication a) Delayed 'wash - out' of contrast
b) Renal vein thrombosis b) Filling defect in calyces
c) Hydronephrosis c) Ureteral kinking
d) Hypoplasia d) Normal or small sized kidney
e) Amyloidosis 350. Most sensitive and specific investigation for
339. 'Stipple sign' in transitional cell carcinoma of the screening of Renovascular hypertension-
renal collecting system is best demonstrated by - a) MRl (AIIMS OJ)
a) Intravenous urography (COMED 09) b) Captopril enhanced radionuclide scan
b) Retrograde pyeloureterography c) Spiral CT Scan
c) Radionuclide scan d)
d) Ultrasound scan
340. Cobra head deformity is characterstic of-
a) Posterior urethral valve (Corned 09, AIIMS 97)
b) Ureterocele
c) Bladdertumor
d) Cytitis 352. Not a radiological finding of papillary necrosis on
341. "Adder - head" appearance on voiding excretory urogram is- (AIIMS May 09)
cystourethrogram in bladder is feature of- (MH 07) a) Tracks and horns from calyces
a) Horse shoe kidney b)VUR b) Ring shadow
c) Ureterocele d) Carcinomaofbladder c) Increased dense nephrogram
342. IVP is contraindicated in- (WB 96, 2K) d) Egg in cup appearance
a) Multiple myeloma b) Kidney stones 353. For renal stone, diagnosis is not done by-
c) Renal cyst d) Transplanted kidney a) lVP b)MRI (PGIMaylO)
c) PET-Scan d) USG
e) CTscan

344. 'Rim' and 'ball' nephrograms in intravenous


urography are seen in - (COMED 09)
a) Normal kidneys
b) Acute obstructive nephropathy 355. In Renal cell carcinoma investigation of choice to
c) Chronic obstructive nephropathy evaluate inferior venacava & renal vein for
d) Chronic renal failure thrombus- (AJIMS 99)
345. Spider leg appearance is found in- (PGI 99) a) lVP b) Coloured doppler
a) Polycystic kidney b) Pyelonephritis c) USG d) CT scan
c) Hydronephrosis d) Renal artery stenosis
346. IVP of polycystic kidney disease shows-
a) Cobra head ( PGI Nov 09)
b) Dropping lily
c) Flower base appearance
d) Spider leg deformity urethral valve is -
e) Fish hook appearance a) Urethroscopy
347. Tear- drop bladder is seen in- (MH 02) b) lVP
a) Pelvic abscess b) Pelvic lipomatosis c) Retrograde cystogram
c) Bladder rupture d) All d) Micturating cystogram (MCU)

337)b 338)None 339)a 340)b 34l)c 342)a 343)b 344)c 345)a 346) d 347) d 348) a,b,c,e 349) b
350)c 351)b 352)None 353)b,c 354)c 355)b 356)b 357)d
RADIOLOGY [ 559]

358. Causes of bladder calcification are- (PGI Dec 06) SKELETAL SYSTEM
a) Schistosmosis b) Urethral cell
c) 1B d) Carcinoma 369. Which one ofthe following is a recognized X-ray
359. The investigation of choice for imaging of urinary feature ofrheumatoid arthritis? (AI 03)
tract tuberculosis is- (AIIMS 04) a) Juxta-articular osteosclerosis
a) Plain X-ray b) Intravenous urography b) Sacroilitis
c) Ultrasound d) Computed tomography c) Bone erosions
360. Which of the following imaging modality is most d) Peri-articular calcification
sensitive to detect early renal tuberculosis- 370. Epiphyseal enlargement is seen in - (AIIMS 97)
a) Intravenous urography (AIIMS 03, 02, AI 05) a) Rickets
b) Ultrasound b) Scurvy
c) Computed tomography c) Spondylo-epiphyseal dysgenesis
d) Magnetic resonance imaging d) Juvenile rheumatoid arthritis
361. Renal tuberculosis can be diagnosed earliest by- 371. Fraying and cupping of metaphyses oflong bones
a) CT scan b)IVP (AIIMS 98) in a child does not occur in- (AI 03)
c) Angiography d)USG a) Rickets b) Lead poisoning
362. The most important sign of significance of renal c) Metaphyseal dysplasia d) Hypophosphatasia
artery stenosis on an angiogram is- (AI 06) 372. Splaying and cupping of the metaphysis is seen in-
a) A percentage diameter stenosis> 70% a) Rickets b) Scurvy (PGI 99)
b) Presence of collaterals c) Paget's disease d) Lead poisoning
c) A systolic pressure gradient> 20 mm Hg across 373. Dense metaphyseal band is seen in- (PGI June 08)
the lesion a) Hypervitaminosis A b) Hypervitaminosis B
d) Post-stenotic dilatation of the renal artery c) Scurvy d) Hypervitaminosis D
363. The posterior urethra is best visualized by- 374. Vertebra plana is seen in - (AIIMS 98)
a) Static cystogram (AIIMS Nov 05) a) Malignancy b) Fracture
b) Retrograde urethrogram c) Tuberculosis d) Eosinophilic granuloma
c) Voidingcystogram 375. An eight year old boy presents with back pain and
d) CT cystogram mild fever. His plain X-ray of the dorsolumbar
364. Which of the following is not an appropriate spine reveals a solitary collapsed dorsal vertebra
investigation for anterior urethral stricture ? with preserved disc spaces. There was no associated
a) Magnetic resonance imaging (AIIMS 03) soft tissue shadow. The most likely diagnosis is -
b) Retrograde urethrogram a) Ewing's sarcoma b) Tuberculosis (AI 03)
c) Micturating cystourethrogram c) Histiocytosis d) Metastasis
d) High frequency ultrasound 376. Vertebra plana is seen in- (PGI June 05)
365. Abdominal ultra-sonography in a3 year old boy show a) Eosinophilic granuloma b) Trauma
a solid circumscribed hypoechnoic renal mass. Most c) Paget's disease d) Malignancy
likely diagnosis is- (AI 02) 377. X-ray of which bone (s) would be diagnostic in
a) Wilm's tumor b) Renal cell carcinoma hyperparathyroidism - (PGI 2000)
c) Mesoblastic nephroma d) Oncocytoma a) Skull b) Phalanges
366. Transrectal ultrasonography in carcinoma prostate c) Long bones d) Scapula
is most useful for- (AI 08) e) Spine
a) Guided prostatic biopsies 378. Pathognomic feature of hyperparathyroidism-
b) Seminal vesicle involvement a) Osteopenia (AIIMS 98)
c) Measurement of prostatic volume b) Loss of lamina dura
d) To detect hypoechoic area c) Brown's tumor
36 7. Hypoechoic lesion within prostate in USG seen in - d) Sub periosteal resorption of phalanges
a) Adenocarcinoma (PGI Dec 04) 379. X-ray features of hypoparathyroidism are the
b) Normal prostate tissue following except- (MH 10, 03)
c) Infertility a) Osteosclerosis
d) Urethral obstruction b) Calvarial thickening
e) BPH c) Subperiosteol resorption
368. Radiation exposure is the least in the following Subcutaneous calcification
procedure- (AIIMS Nov. 10)
a) Micturating cystourethrogram
b) lVP
c) Bilateral nephrostomogram
d) Spiral CT for stones

358)a,c,d 359)b 360)a 361)b 362)b 363)c 364)a 365)a 366)a 367)a,e 368)a 369)c 370)a,c,d
371)b 372)a 373)d 374)d>a,c 375)c 376)a,b,d 377)a,b 378)d 379)c 380)a
RADIOLOGY [ 560 ]

381. Champagne glass pelvis is seen in- (Jipmer 02) 393. Wind swept deformity is seen in - (AIIMS 98)
a) ffiH b) Down's syndrome a) Ankylosing spondylitis b) Scurvy
c) Cretinism d) Achondroplasia c) Rheumatoid arthritis d) Rickets
382. Multiple lytic lesions of skull with beveled edges
are seen in - (MH 06)
a) Eosinophilic granuloma b) Metastases
c) Multiple myeloma d) Neuroblastoma
383. Geographic lytic lesions in vault of skull with
beveled edges are seen with- (AIIMS 97)
a) Multiple myeloma a) Spondylosis (AIIMS 92, Jipmer 99, 90)
b) Eosinophilic granuloma b) Spondylolisthesis
c) Hyperparathyroidism c) Lumbar canal stenosis
d) Reticular cell Ca d) Slipped disc
384. 76 year old man presents with lytic lesion in the 396. In spondolysthesis following radiological features
vertebrae. X-ray skull showed multiple punched seen- (PGI Dec 06)
out lesions. The diagnosis is- (AIIMS 2000) a) Scotty dog
a) Metastasis b) Multiple myeloma b) Scotty dog wearing a collar
c) Osteomalacia d) Hyperparathyroidism c) Beheaded scotty dog terrier sign
385. Multiple punched out lesions on skull X-ray is d) Nepolean sign
found in- (AIIMS 94) 397. Calcification around the joint is seen in - (PGI 99)
a) Down's b) Hyperparathyroidism a) Pseudogout b) Hyperparathyroidism
c) Multiple myeloma d) All c) Rh. arthritis d) Gout
:J9s~· ... •.·cwcmc~ttrlin'fi:rmrervil'$
386. In scurvy all the following radiological signs are
seen except-
a) Pelican spur
b) Soap bubble appearance
(AI 04)
':';·~· ~::·:~:~~;~i~~t2ri~~[?. -•:.
399. A 40 year old male female patient on long term
~~&6)
c) Zone of demarcation near epiphysis steroid therapy presents with recent onset of severe
d) Frenkel's line pain in the right hip. Imaging modality of choice
387. All are radiological sign ofVit C deficiency except- for this problem is - (AIIMS May 05)
a) White line of frenkel (AIIMS 95) a) CT scan b) Bone scan
b) Wimberger line c) MRl d) Plain X-ray
c) Osteoporosis of bone 400. Looser's zones is seen in - (PGI 02)
d) Widening of epiphysis a) Osteoporosis b) Hyperparathyrodism
388. Radiological findings of scurvy are AlE -(PGI 99) c) Osteomalacia d) Renal osteodystrophy
a) Epiphyseal widening b) Metaphyseal porosis e) Paget's disease
c) White line d) Pelkan spur
401. Looser's zone is seen in - (MH JO, 03)
389. Pelkan spur is seen in - (MH OJ)
a) Osteogenesis imperfecta
a) Rickets b) Scurvy
b) Osteopetrosis
c) Hemophilia d) All
c) Osteomalacia
390. Radiological features of rickets include -
a) Narrowing of epiphysis (PGI Dec 05) d) Hypoparathyroidism
b) Cupping of metaphysis ;to2•• '~~ll.«to~~~ttiii~~l ;~-~~p;~~j~~ttiwa;~ETl
c) Ricketic rosary · ·....... a);Hste:oporosis" i eoJJefrosi'l?(. •. DNB. 'f.l:cmemj
d) Pelkan's spur :.'r::l\ F•;~q);:.~~t~~~~\~4i~·;~~,i~)f~~~~--~;~;t~;p·;;~-~:,.:,:;~.·-·········r•.?ti
391. Not a radiologicalfeature of rickets- (MH09) 403. Which endocrine disorder is associated with
a) Splaying of metaphysis epiphyseal dysgenesis? (AI 03)
b) Cupping of metaphysis a) Hypothyroidism b) Cushings syndrome
c) Subluxation of epiphysis c) Addison's disease d) Hypoparathyroidism
d) Widening of metaphysis 404. Flaring of anterior ends of the ribs is
392. Earliest evidence of healing in rickets is provided characteristically seen in - (AI 08)
by- ++ a) Neurofibromatosis b) Scurvy
a) S. Ca (AI 95) c) Rickets d) Hypothyroidism
3
b) S. P04 - 405. Bamboo spine is seen in- (Maharashtra 05, OJ)
c) Radiological examination of growing bone ends a) Ankylosing spondylitis b) RA
d) S. Alkaline phosphate level c) Paget's disease d) All

38l)d 382) a 383) b 384) b 385) c 386)b 387)d 388)a 389)b 390)b 39l)c 392)c 393) c >d 394) c
395)b 396) c,d 397) a,d 398) a,c 399) c 400)b,c,d,e 40l)c 402)c 403)a 404)c 405)a
RADIOLOGY [ 561 ]

406. X-ray finding of osteomyelitis within 8 day is - 416. Best radiographic view for fracture of C1, C2
a) Cystic swelling (DPG 09, PGI 99) vertebrae is - (AIIMS May 09, AI 08)
b) Soft tissue swelling a) AP view b) Odontoid view
c) New bone formation c) Lateral view d) Oblique view
d) Sequestrum formation 417. The gold standard for the diagnosis of osteoporosis
407. "Hair-on end" appearance is seen in - is - (AI 05, AIIMS 04)
a) Thalassemia (AIIMS May 08) a) Dual energy X-ray absorptiometry
b) Sickle cell anemia b) Single energy X-ray absorptiometry
c) Hemochromatosis c) Ultrasound
d) Megaloblastic anemia d) Quantitative computed tomography
408. Hair on end appearance is characteristically seen 418. Bone Density is best studied by- (AI J2)
in- (Maharashtra OJ) a) CT scan b) DEXA scan
a) Thalassemia major c) MRI scan d) Bone scan
b) Sickle cell anemia 419. Which of the following is not a cause of generalized
c) G6PD deficiency increase in bone density in adults?(DPG Mar. 09)
d) Hereditary spherocytosis a) Myelosclerosis b) Renal osteodystrophy
409. Wormian bones are seen in all except - c) Fluorosis d) Caffey's disease
a) Hypothyroidism (Maharashtra OJ) 420. Increased bone density is seen in all except -
b) Down syndrome a) Primary hyperparathyroidism (Maharashtra 03)
c) Tnrner syndrome b) Osteopetrosis
d) Osteogenesis imperfecta c) Fluorosis
410. Which one of the following is the earliest d) Hypoparathyroidism
radiographic manifestation of childhood leukemia? 421. Malten- wax appearance is seen in- (MH 07)
a) Radiolucent transverse metaphyseal band a) Osteoporosis b) Osteopoikilosis
b) Diffuse demineralization of bones c) Melorheostosis d) Osteogenesis imperfecta
422. "Flowing wax" appearance on anterior and posterior
c) Osteoblastic lesions in skull (AIIMS Nov 04)
borders ofvertebrae with normal intervertebral disc
d) Parenchymal pulmonary lesions on chest films
space occuring due to ligament calcification is seen
411. A 2 yr old boy suffering from leukaemia, following
in- ~HO~
are the X-ray finding -
a) Ankylosing spondylitis
a) Osteolytic lesion in flat bones (PGJ 03)
b) Diffuse idiopathic skeletal hypertrophy
b) Metaphysial osteoporosis c) Psoriatic spondyloarthropathy
c) Periosteal new bone formation d) RA
d) Osteosclerosis of long bone 423. Codman's triangle is a feature of- (MH-11)
e) Transverse line of dark band below the growth a) Osteosarcoma b) Osteochondroma
plate Osteoid osteoma Chondrosarcoma
412. All are radiological features of sickle cell anemia
except- (PGI June 08)
a) Vertebra plana b) Floating teeth
c) Bone infarct d) Marrow hyperplasia ray appearance seen
e) Secondary osteomyelitis a) Osteosarcoma b) Ewing sarcoma
413. On MRI the differential diagnosis of spinal cord c) Osteoclastoma d) Multiple myeloma
edema is - (AIIMS May 06)
a) Myelodysplasia b) Myelomalacia
c) Myeloschisis d) Cord tumors
414. Which of the following is not true regarding ossified
posterior longitudinal ligament (OPLL)? (AI 09)
a) Most commonly involves thoracic spine Ra,dio.logiical inv. shows sun ray appearance; dx -
b) Gradient echo MR sequence may overestimate a) Osteosarcoma b)GCT (AllMS 95)
the canal stenosis c) Osteomyelitis d) Ewing's sarcoma
c) MRl is best for diagnosis 428. Radiological feature of osteosarcoma is -(AIIMS 99)
d) Low signal intensity on all MR sequences a) New bone formation b) Sunray appearance
415. Sonographic fmding of spina bifida -(PGI June 04) c) Cotton wool app. d) Osteoid formation
a) Ventriculomegaly 429. Radiographic appearance ofPindborg's tumor is-
b) Obliteration of cisterna magna a) Onion- peel appearance (AI 02)
c) SmallBPD b) Sun burst appearance
d) Abnormal curvature of cerebellum c) Cherry- blosom appearance
e) Club foot d) Driven - snow appearance

406)b 407)a 408)a 409)c 410)a 411)a,c,d,e 412)a,b 413)b 414)a 415)a,b,d 416)b 417)a 418)b
419)d 420)a 42l)c 422)b 423)a 424) b 425)a,b 426)c>b,d 427)a>d 428)b 429)d
RADIOLOGY [ 562]

430. Expansile lytic lesion with fluid •fluid levels within NERVOUS SYSTEM
the metaphysis of fibula seen on CT and MRI in an
early adolescent female is typical of- (MH10) 444. Most common cause of suprasellar enlargement with
a) Giant cell tumor b) Aneurysmal bone cyst calcification in children is- (AIIMS Dec 94,
c) Hemangioma d) Fibrous dysplasia a) Craniopharyngioma AI 97)
431. A lady dimple has a lytic lesion in X-ray of upper end b) Astrocytoma
of humerus. The diagnosis is - (AIIMS99) c) Meningioma
a) Osteosarcoma b) Osteochondroma d) Suprasellar tuberculoma
c) Unicameral bone cyst d) Osteoclastoma 445. A 6 year old boy has been complaining of headache,
432. A classical expansive lytic lesion in the transverse ignoring to see the objects on the sides for four
process of a vertebra is seen in - (AI 03) months. On examination he is not mentally retarded,
a) Osteosarcoma b) Aneurysmal bone cyst his grades at school are good, and visual acuity is
c) Osteoblastoma d) Metastasis diminished in both the eyes. Visual charting showed
433. X-ray shows soap bubble appearance at lower end of significant field defect. CT scan ofthe bead showed
redius, treatment of choice is - (AIIMS98) suprasellar mass with calcification. Which ofthe
a) Local excision b) Excision & Bone grafting following is the most probable diagnosis ?
c) Amputation d)RT a) Astrocytoma (AIIMS Nov 04)
434. Lytic lesion in skull are seen in following except- b) Craniopharyngioma
a) Multiple myeloma b) Metastasis bronchus c) Pituitary adenoma
c) Thalassemia d) Ca prostate (PG/97) d) Meningioma
446. Intracranial calcification with cystic lesion in plain
X-ray skull is seen in - (AIIMS 96)
a) Meningioma b) Glioma
436. Dense calcification is seen in - (AIIMS 96) c) Craniopharyngioma d) Medulloblastoma
a) Chondroblatoma b) Chondrosarcoma 447. Suprasellar calcification with growth retardation
c) Osteosarcoma d) Fibrosarcoma is seen in - (AIIMS 95)
a) Pineal body tumor b) Pituitary tumor
c) Thalamic tumor d) Craniopharyngioma
448. Which of the following is the most common cause of
a mixed cystic and solid suprasellar mass seen on
cranial MR scan of a 10 years old child -
a) Pituitary adenoma (AIIMS May 05)
a) MR1 b)CT b) Craniopharyngioma
c) Bone scan d)X-ray c) Optic chiasma glioma
439. Bestinvestigation for bone metastasis is- (AI 11) d) Genninoma
a) MRl b)CT 449. Physiological calcification of Skull in X-ray is seen
c) Bone scan d) X- ray in- ~m~
a) Pineal gland b) Choroid plexus
c) Red nucleus d) Basal ganglion
450. Basal ganglia calcification is seen in - (PGI 03)
a) Hyperparathyrodism b) Hyperthyroidism
c) Hypoparathyroidism d) Hypothyroidism
e) Acromegaly
441. Investigation of choice for a lesion oftemporal bone- 451. Basal ganglia calcification is seen in all except-
a) CT b) MRI (AIIMS May 10) a) Hypoparathyroidism (AI 07)
c) USG d)PlainX-ray b) Wilson's disease
442. Bare orbit is/are seen in - (PGI Nov. 10) c) Perinatal hypoxia
a) Metastasis b) Neuroblastoma d) Fahr's syndrome
c) Optic nerve glioma d) Osteomyelitis 452. Calcification of basal ganglia is seen in AlE-
e) Pseudotumor cerebri a) Berry's aneurysm (AIIMS 97)
b) Cysticercosis
c) Idiopathic hyperparathyroidism
d) Wilson's disease

430)b 431)c 432)b 433)b 434)d 435)a 436)b>c 437)a 438)c 439)c 440)c 441)a 442)a 443)a
444)a 445)b 446)c 447)d 448)b 449) a,b,d 450) a,c,d 451) b 452) d
RADIOLOGY [ 563]

453. Which of these is not a sign of increased ICT- 462. Characteristic featurs of meningioma are aU except-
a) Erosion of dorsum sella (AIIMS May 08) a) Dural tail sign (Maharashtra 02)
b) Sutural diastasis b) Hyperostosis
c) Ballooning of sella c) Prominentvascularity
d) Copper-beaten appearance d) Intra-axial tumor
454. In children raised ICT is manifested by- (MP 98) 463. A 45 year old female presented with lower limb
a) Lytic lesions in skull wall weakness, spasticity, urinary hesitancy and mid
b) Rarefaction of dorsum of sella dorsal intradural enhancing mass in MRI. What is
c) Sutural diastasis the diagnosis- (A/07, AIIMS May 10, Nov 06)
d) None a) Lipoma b) Meningioma
455. The MR imaging in multiple sclerosis will show c) Dermoid cyst d) Neuroepithelial cyst
lesions in - (AI 06) 464. A 20 yearfemale patient with 6th cranial nerve palsy
a) White matter b) Grey matter on T2 weighted MRI shows a hyperintense lesion in
c) Thalamus d) Basal ganglia cavernous sinus which shows homogenous contrast
456. Extensive involvement of deep white matter with enhancement. Most probable diagnosis is?
bilateral hyperdense thalami on non-contrast CT a) Schwannoma (AIIMS Nov. 1 0)
scan of the brain is virtually diagnostic of- b) Meningioma
a) Alexander's disease (AIIMS Nov 07) c) Cavernous sinus hemangioma
b) Krabbe's disease d) Astrocytoma
c) Canavan's disease 465. A male was brought unconscious to the hospital with
d) Metachromatic leukodystrophy external injuries. CT brain showed no midline shift,
457. Which ofthefoUowing is not a MRI feature of Mesial but basal cisterns were compressed with multiple
temporal sclerosis? (AI 09) small hemorrhages. What is the diagnosis-
a) Atrophy of mammillary body a) Cortical contusion (AI 07, AIIMS May 10,
b) Atrophyoffomix b) Cerebral laceration Nov 06)
c) Blurring of Grey white matter junction of ipsilateral c) Multiple infarcts
temporal lobe d) Diffuse axonal injuries
d) Atrophy of hippocampus 466. Which one of the following tumors shows
458. A 15 year old boy had 10-12 partial complex seizures calcification on CT scan- (AI 05)
per day inspite of adequate 4 drug antiepileptic a) Ependymoma b) Medulloblastoma
regime. He had history of repeated high grade fever c) Meningioma d) CNS lymphoma
in childhood. MRI for epilepsy protocol revealed 467. All of the foUowing calcify except- (Jipmer 02)
normal brain scan. What should be the best non- a) Medulloblastoma
invasive strategy to make a definite diagnosis so b) Sturge weber syndrome
that he can be prepared to undergo epilepsy surgery- c) Meningioma
a) InterictalscalpEEG (AIIMS 02) d) Ependymoma
b) VideoEEG 468. Ring enhancing lesion in brain is/are seen in -
c) Interictal 18F-FDg PET. a) Brain abscess (PGI Dec 08)
d) Video EEGwithlctal 99m Tc- HMPAO Brain SPECT b) Resolving haematoma
459. Finding in meningioma are aU except- (PGI 97) c) PrimaryCNSlymphoma
a) Vascular markings around falx d) Encephalopathy
b) Calcification e) Infarction
c) Erosion 469. Local cerebral lesion with ring on CT scan is caused
d) Osteosclerosis by- (PGI02)
460. Meningioma on plain radiography reveals - a) Toxoplasmosis b) Intracranial haemorrhage
a) Calcification b) Erosion (PGI June 09) c) Cysts d) Hamartoma
c) Sutural diastasis d) Osteosclerosis 470. Periventricular calcification is often due to-(PGI 97)
e) Vascularmarknig a) Toxoplasmosis b) CMV infection
461. A 40 years old female patient presented with c) Congenital syphilis d) All of the above
recurrent headaches. MRI showed an extra-axial, 471. "Bracket Calcification" on Skull X-Ray is seen in-
dural based and enhancing lesion. The most likely a) TuberousSclerosis (AIIMS May 12)
diagnosis is- (DPG 09, AIIMS May 06, AI 12) b) SturgeWebeSyndrome
a) Meningioma b)Glioma c) Lipoma of corpus callosum
c) Schwannoma d) Pituitary adenoma d) Meningioma

453) c 454) c 455) a 456) b 457) c 458) d 459) c 460) a,d,e 461) a 462) d 463) b 464) b 465) d
466)c>a 467)a 468)a,b,c,e 469)a 470)b 47l)c
RADIOLOGY [ 564]

472. Tram track appearance on CT scan of head is seen 483. The procedure of choice for the evaluation of an
in- (AIIMS 01) aneurysm is- (AI 06)
a) Sturge weber syndrome a) Ultrasonography
b) Von Hipple Iindau syndrome b) Computed tomography
c) Tuberous sclerosis c) Magnetic resonance imaging
d) Neurofibroma d) Arteriography
473. Subdural haematoma most commonly results from-
a) Rupture of intracranial aneurysm (AIIMS 04)
b) Rupture of cerebral AVM
c) Injury to cortical bridging veins
d) Hemophilia
47 4. Which of the following is classic CT appearance of
an acute subdural hematoma- (MH 04, AI04) 485. The best investigation to diagnose a case of acoustic
a) Lentiform-shaped hyperdense lesion neuroma is- (AIIMS May 08)
b) Cresent-shaped hypodense lesion a) Gadolinium enhanced MRI b) CT scan
c) Cresent-shaped hyperdense lesion c) Audiometric analysis d) PET scan
d) Lentiform-shaped hypodense lesion 486. Which of the following is the best choice to evaluate
475. Characteristic of subdural hematoma is- radiologically a posterior fossa tumor? (MH 05,
a) Convex hyperdensity (AIIMS 98) a) CTscan b)MRI SGPGI04,AI03)
b) Concavo convex hyperdense c) Angiography d) Myelography
c) Biconvex hyperdense 487. Investigation of choice for meningeal carcinomatosis
d) Concavo convex hypodense is- (AIIMS Nov. 11)
476. CTscauofapatientwithhistoryofheadinjuryshows a) Contrast enhanced MRI b) PET-CT
a biconvex hyperdense lesion displacing the grey- g SPET ~NCCT
white matter interface. The most likely diagnosis 488. Parameningeal Rhabdomyosarcoma is best
is- diagnosedby- (AIIMSOl)
a) Subdural hematoma (AI 03) a) MRI b)CTScan
b) Diffuse axonal injury c) SPECT d) PET
c) Extradural hematoma 489. Cerebral blood tlow in an asphyxiated child is best
d) Hemorrhagic contusion measuredby- (AIIMSMay09)
477. Which tumor is suggested by "Trouser leg Appearance" a) NIRS b)PET
on an ascending myelogram? (DPG Mar. 09) c) Radionuclide imaging d) MRI angiography
a) Extradural b) Extramedullary
c) Intramedullary d) Vertebral
478. The first investigation of choice in a patient with
suspected subarachnoid haemorrhage should be- 491. Dye for myelography is injected in which space-
a) Non-contrast computed tomography (AI 04) a) Subdural b) Epidural (MH 02)
b) CSF examination c) Subarachnoid d) Extradural
c) Magnetic resonance imaging (MRI) 492. Investigation of choice for diagnostic evaluation of
d) Contrast-enhanced computed tomography hydrocephalus in one month old child- (MH 05)
479. Investigation of choice for acute subarachnoid a) X-rayskull b)USG
haemorrhage- (DPG Feb. 09, Calcutta 2K, c) CT d)MRI
a) EnhancedMRI b)MRI Assam2K,AI98) 493. MRI finding in covemous angioma -(AIJMS Nov. 11)
c) CT-scan d) Angiography a) Reticular popcorn like pattern
480. Best test to determine etiology of SAH- (AIIMS 95) b) Arterial feeders
a) EnhancedCT c) Well defined nidus
b) Unenhanced CT Phebectasia
c) Intra arterial digital substraction angiography
MRI
481.

i;.;\:\) . . ... :. : ·:'.rr·:•:::·:'"<tl OBSTETRICS AND GYNAECOLOGY


482. Investigation ofchoitre f(tr tuvenlile n:aso:angjofibr(Jima
a) X-ray b)Angiography (AIIMS97) 495. USG can detect gestation sac earliest at- (PGI 97)
c) USG d) CT Scan- contrast enhanced a) 5-6 weeks of gestation b) 7-8 weeks ofgestation
c) 10 weeks of gestation d) 12 weeks of gestation

472)a 473)c 474)c 475)b 476) c 477)c 478)a 479)c 480)c 481)b 482)d 483)d 484)b 485)a
486)b 487)a 488)a 489)a 490)d 49l)c 492)b 493)a 494)a 495)a
RADIOLOGY [ 565]

496. Earliest sign offetal life is best detected by- (AI 04) 508. All are signs I features of ectopic pregnancy on USG
a) X-ray b) Fetoscopy except- {PGI 03)
c) Real time USG d) Doppler a) Pseudo sac
497. Molar pregnancy can be best diagnosed by -(MH 01) b) Hyperechoic rim
a) Clinical history & examination c) Adenexal mass
b) Ultrasound study d) Echogenic mass with multicystic spaces within
c) Laproscopy endometrial cavity
d) CTScan e) Doughnut sign
498. The investigation of choice for an ectopic pregnancy 509. Ectopic pregnancy, characteristic finding in USG
is - (AIIMS May 08, PGI97) is- (AIJMS 98)
a) CT scan b) Transvaginal USG a) Absence of gestational sac in uterus
c) Serum HCG levels d) MRI b) Complex adnexal mass
499. Most accurate assessment of gestational age by USG c) Resistance in coloured doppler
is done by- (Jipmer 04) d) Free fluid in peritoneal cavity
a) Femur length b) Gestational sac size 510. USGcandiagnoseaUexcept-
c) Menstrual history d) Crown rump length a) Anencephaly b) Neural tube defect
500. Parameters used to estimate gestational age in last c) Placenta previa d) Down's syndrome
trimester- (PGI97) 511. On USG finding of cystic hygroma in fetus is
a) CR length b) Abdominal circumference suggestive of- (AIIMS 97)
c) BPD d) Femurlength a) Down's syndrome b) Marphan's syndrome
501. In second trimester of pregnancy, the diognosis of c) Turner's syndrome d) Klinfelter's syndrome
IUGR can be best made by assessing which of the 512. Popcorn calcification is seen in- (NEETIDNB
foUowing parameter- (MH-11) a) Fibroadenoma b) Fibroadenosis Pattern)
a) HC b)AC c) Carcinoma breast d) None
~ CRL d)BPD 513. Mammographic abnormality seen inCA breast is-
502. Which one of the following congenital malformation a) Change in density (PGI June 05)
ofthe fetus can be diagnosed in first trimester by b) Clusters ofMicrocalcification
ultrasound? (AIIMS Nov 08, AI 06) c) Change in architecture
a) Anencephaly b) Inencephaly d) All
c) Microcephaly d) Holoprosencephaly 514. Hallmark of breast malignancy on mammography-
503. Anencephaly can be diagnosed by USG at- a) Low density lesion (Maharashtra 10)
a) 10-12 weeks of gestation (AIIMS 96) b) Smooth margin
b) 14-18 weeks of gestation c) Clusters ofmicrocalcification
c) 20-24 weeks of gestation d) Popcorn calcification
d) 24-28 weeks of gestation 515. Which ofthe following features on mammogram
504. Earliest detectable congenital malformation by USG would suggest malignancy- (AIIMS May 06)
fs? (AllMS Nov 08) a) Well defmed lesion
a) Spina bifida b) Cystic hygroma b) A mass of decreased density
c) Anencephaly d) Encephalocele c) Areas of speculated microcalcifications
505. Ultrasonography of umbilical artery is done to know d) Smooth borders
about- (PGI97) 516. On mammogram all of the following are the features
a) Heart beat b) Gastational age of a malignant tumor except- (AIIMS 03)
c) Fetal weight d) Fetal maturity a) Spiculation b) Microcalcification
506. True about antenatal doppler analysis is aU except- c) Macrocalcification d) Irregular mass
a) Reduction in end diastolic flow is associated with 517. The sensitivity of mammography is low in young
poor out come (Jipmer 06, PGI 06) females because- (AI 09)
b) Reduction in EDF is associated with IUGR a) Less glandular tissue and more fat
c) In normal gestation placental resistance is high b) Young females are less cooperative
d) SID ratio is high in IUGR c) Young breast have dense tissue
e) Investigation of choice in pregnancy d) Because of less fat content
507. USG done at18-20 weeks mainly to- (Kerala 01) 518. Which of the following does not contain fat on
a) Detect fetal abnormality b) Determine sex mammography? (AI 09)
c) Estimate liquor d) Determine maturity a) Post-traumatic cyst b) Hamartoma
c) Seborrbic keratosis d) Galactocele

496)d 497)b 498)b 499)d 500)b,c,d 501)d 502)a 503)a 504)c 505)a 506) c,e 507) a 508)d 509)b
510)None>d 5ll)c 512)a 513)d 514)c 515)c 516)c 517)c 518)c
RADIOLOGY [ 566 ]

519. The most sensitive investigation for DCIS (Du(:tal 530. GFR is measured with which ofthe following?
carcinoma in situ) of breast? (AI 09, AIIMS, Nov. 10) a) Iodohippurate b)Tc99m-DTPA (AI08)
a) Mammography b) Ultrasound c) Tc99m- MAG3 d) Tc99m- DMSA
c) MRI d) PET scan 531. 1mpared renalfunction is assessed by- (AJ99)
520. Investigation to diagnose stage-I carcinoma breast- a) MAG3 b) Iodohippurate
a) BtL mammogram b) X-ray chest (PGI 2K) c) DMSA scan d) DTPA
c) Bone scan d) Liver scan 532. Distribution offunctional renal tissue is seen by-
521. Mammography can be used in- (MH OJ) a) DMSA (Jipmer 05, PGI 05)
a) Early breast carcinoma b) Mastitis b)DTPA
c) Fibroadenoma d) Phyllodes tumor c) MAG3- Tc99
522. Triple assessment for carcinoma breast includes- d) 1123 iodocholesterol
a) History, clinical examination, biopsy/cytology 533. Vesicoureteric reflux is demonstrated by using-
b) Clinical examination, mammography, biopsy/ a) DMSA (Jipmer 06, TN 04)
cytology (AI 09) b) DTPA
c) History, clinical examination, ultrasonography c) MAG3- Tc99
d) Observation, ultrasonography, biopsy/cytology d) 1123 iodocholesterol
534. In Radionuclide imaging the most useful radio
NUCLEAR MEDICINE pharmaceutical for skeletal imaging is-
(AI 05, 95, 94, AIIMS 95,PGI 03, 02, 00)
523. Gamma camera in Nuclear Medicine is used for- a) Gallium 67 (67Ga)
a) Organ imaging (AI 05) b) Technetium-sulphur-colloid (99mTc-Sc)
b) Measuring the radioactivity c) Technetium-99m (99mTc)
c) Monitoring the surface contamination d) Technetium-99m linked to Methylene
d) RIA disphosphonate (99mTc-MDP)
524. On 3 phase 99m Tc- MDP bone scan, which ofthe 535. Sestamibi scan is used in- (PGI 03)
following bone lesions will show least osteoblastic a) Ectopic thyroid
activity- (AIIMS 03) b) Ectopic parathyroid
a) Paget' disease b) Osteoid osteoma c) Parathyroid adenoma
c) Fibrous dysplasia d) Fibrous cortical defect d) Extra adrenal pheochromocytoma
525. Bone scan in multiple myeloma shows- e) Adrenal pheochromocytoma
a) Hot spot (AIIMS Nov 06) 536. Isotope for thyroid scaning- (PGI 2000)
a) p29 b) p3t
b) Cold spot
c) Technetium99 d) Selenium
c) Diffusely increased uptake
537. All isotopes are used for thyroid except- (PGI 02)
d) Diffusely decreased uptake
a) 1-131 b)l-123
526. Hot spots in bone scan are seen in the following
c) 1-122 d)I-125
conditions conditions except- (Maharashtra 07) e) 1-129
a) Osteomyelitis b) Multiple myeloma 538. lsotopeusedinRAIU- (AI 07)
c) Hyperparathyroidism d) Metastases a) 1131 b) 1123
527. HotspotinMiis seen in- (AI98,AIIMS96) c) 1125 d) 1127
a) Th- 20 b) Gallium 539. Tc labelled RBC's are used for- (AI 95)
c) Pyrophosphate Tc99 d)Albumin a) Biliary tree b) Renal disease
528. Which test is performed to detect reversible c) Pulmonary embolism d) Spleenic disease
myocardialischemia? (AIIMS 03) 540. Which ofthe following techniques is the best for
a) Coronary angiography differentiating recurrence of brain tumour from
b)MUGAscan radiation therapy induced necrosis?(AJIMSMay 05)
c) Thallium scan a) MRI b) Contrast enhanced MRI
d) Resting echocardiography c) PET scan d) CT scan
529. RenalGFRisbestmeasuredby- (PGIDec05) 541. Radio isotopes are used in the following technique
a) Tc99DMSA except- (AI 04)
b) Tc99 Pyrophosphate scan a) Mass spectroscopy
c) Tc99DTPA b) RIA
d) Creatinine clearance c) EUSA
e) Tc99 albumin scan d) Sequencing of nucleic acid

519) c 520)a 521)a 522) b 523) b 524) d 525) b 526) b 527) c 528) c 529)c 530)b 531)a>b,d
532)a 533)a 534)d 535)b,c 536)b,c 537)c,d,e 538)b 539)d 540)c 541)c
RADIOLOGY [ 567]

554. Most commonly radiothearpy uses- (AIIMS 99)


a) X-rays b) Gamma rays
c) Alpha rays d) Electron
555. Radioactive emissions used in radiotherapy are-
RADIOTHERAPY a) a.-Particles b) ~-Particles (PG103, OJ)
c) y-rays d) X-ray
543. Whatisatomicnumber- (PGI OJ) e) Infrared rays
a) Proton b) Electrons + protons 556. Beams can be used for cancer treatment are-
c) Protons + neutrons d) Protons + protons a) y- rays b) a. -rays (PGJ 02)
544. Substance with same atomic number but different c) Neutrons d) Protons
massnumber- (MH 10, 09, 06, 03) e) X-rays
a) Isotope b) Isobar 557. In radiation theray rays used are- (PG199)
c) Atom d) Mineral a) a.,~ b)a.,y
545. Principle used in radiotherapy is- (AI 97) c) ~. y d) y, a.,~
a) Cytoplasmic coagulation b) Ionization ofmolecule 558. About X-rays which is/are not true- (PGI Dec 08)
c) DNA damage d) Necrosis of tissue a) Low frequency b) Long wavelength
546. Ionization radiation acts on tissue leading to - c) Dual character d) Penetrating
a) Linear acceleration injury (AI 98) e) Electrically neutral
b) Excitation of electron from orbit 559. The major difference between X-Rays and Light is-
c) Formation ofpyramidine dimer a) Energy b) Mass (AI 10)
d) Thermal injury c) Speed d) Type of wave
547. Functional basic ofionising radiation depends on- 560. Which one of the following has the maximum
a) Pyramidine base pairing (AIIMS 96) ionization potential? (AI 06)
b) Removal of orbital electron a) Electron b) Proton
c) Linear energy transfer c) Helium ion d) Gamma- Photon
d) Adding orbital electron 561. Which of the following has highest ionizing
548. Most sensitive structure in cell for radiotherapy potential- (Maharashtra 1 0)
is- (PGI02)
a) X-ray b)Gammaray
a) Cell membrane b) Mitochondrial membrane
c) a.- rays d)~- rays
c) DNA d) Enzymes
562. Which of the following is the most penetration beam-
e)ER
a) Electron beam b) 5 MV photons (AI 04)
549. Which of the following statements best describes
'Background Radiation'- (All 0) c) 18MV photons d) Proton beam
a) Radiation in the background of nuclear reactors 563. Which of the following has mot penetrating power?
b) Radiation in the backgroung dwjng radiological a) a. particle b)~- particle (MH 07, AI 02)
investigations c) y- radiation d) Electron beam
c) Radiation present constantly from natural sources 564. Most harmful to individual cell- (PGI 98)
d) Radiation from nuclear fall out a) X -rays b) a.- particles
550. Radioactive substance emits the following except- c) ~-particles d)X-rays(gammarays)
a) Gamma b) Beta (MH08, 02) 565. Which of the following is the most ionizing
c) Alpha d) X-rays radiation- (AI 10)
551. Principles of Linear accelerators is used in- a) Alpha b)Beta
a) X-rays b)Gamma-rays (PGIJune06) c) X-rays d)Gamma
c) Alpha rays d) Infrared rays 566. For the treatment ofdeep seated tumors, the following
e) Alpha particles raysareused- (AIIMS03)
552. Brachytherapyis- (MH 05) a) X rays and Gamma -rays
a) Radiotherapy with the source of radiation outside b) Alpha rays and Beta- rays
the body, well at a distance c) Electrons and positrons
b) External beam radiation therapy d) High power laser beams
c) Radiation source used in body cavities or implated 567. SI unit of dose of radiation absorption-
directly into tissues a) Rad b)REM (MH 10, 06)
d) Per oral radiation therapy c) Gray d) Curie
553. Most common used rays for radiotheraphy-
568. Curie is unit of- (PGJ 97)
a) X rays b )'y rays (PGI June 06, PGI 02)
a) Radiation exposure b) Radiation absorption
c) a. rays d) ~ rays
c) Radioactivity d) All of the above

542)a 543)a 544)a 545)b,c 546)b 547)b 548)c 549)c 550)d 55l)a 552)c 553) a,b 554) a >b
555)a,b,c 556)All 557)d 558)a,b 559)a 560)c 561)c 562)c 563)c 564)b 565)a 566) a 567) c 568) c
RADIOLOGY [ 568 J

~~~\i~;~f,'·<·····:.:n.'''"'
584. All of the following radioisotopes are used as
systemic radionucleide, except- (AI 06)
a) Phosphorus-32 b) Strontium- 89
570. ICurieequalsto- (APPG08) c) Iridium-192 d)Samarium-153
a) 3.7 Mbecqueral b) 37M becqueral 585. Radioactive isotopes that are used in treatment of
c) 3.7Gb d)37Gb cancer are- (PGI 03)
571. I becquerel is equal (Disinegration/sec) to- a) Cesium b) Cobalt
a) 3.7x10 10 b)2.7x10 10 (PGIMaylO) c) Carbon d) Technetium
c) 1.7x 10 10 d)3.7x 10-2 e) Nitrogen
e) 1
586. Isotopes used for radiotherapy- (PGI 2000)
572. Unitofradiationexposure- (MH06, 05)
a) Radon b) Cobalt60
~Rad b)R~
c) Iridium d) Cesium
c) Gray d) Roentgen
587. Usedinradiotherapy- (PGI June 08)
573. Which of the following best estimated the amount of
radiation delivered to an organ in the radiation field- a) Iodine-131 b) Co-60
a) Absorbed dose b) Equivalent dose (AI 1 0) c) Ir-192 d)Lu-177
c) Effective dose d) Exposure dose e) Iodine-125
574. Conventional daily dose regimen in external beam 588. Phosphorus32 emits- (AI 06)
radiotberapyis- (MH03) a) Beta particles b) Alpha particles
a) 100-150 cGy b) 150- 170 cGy c) Neutrons d) X-rays
c) 180-200 cGy d) 225-250 cGy 589. All of the following are pure beta emitters Except-
575. Isotope (s) used in high dose rate brachytherapy- a) Yttrium-90 (AIIMS May 11)
a) Ir192 b) Co-60 (PGI June 09) b) Phosphorus-32
c) Cs-133 d)Ra226 c) Strontium
e) Pd103 d) Samarium-153
576. Which of the following is used for permanent 590. Radium emits which of the following radiations-
interstitial implant brachytherapy- (MH 03) a) Alpha rays b) Beta rays (PGI 00, OJ
a) Boron b)Cesium-131 c) Gammarays d) X- rays PGI, May 10)
c) Phosphorus d) Iridium e) Neutrons
577. Which one of the following radioisotope is not used 591. Radioactive isotope/substance emits the following
as permanent implant: (AI 05, DPG 09) radiations except- (MH 08)
a) Iodine-125 b)Palladium-103 a) Alpha b) Beta
c) Gold-198 d) Caesium-137 c) Gamma d) X-ray
578. All may be used in interstitial brachytherapy except- 592. Longest half life is seen in- (AIIMS 97)
a) Cs 137 b)Au 198 (AI 99) a) Radon b) Radium
c) Ir192 d) Co 60 c) Uranium d) Cobalt
579. Which of the following radioactive isotopes is not 593. ThehalflifeofCobalt-60is- (AIIMS97)
usedforbrachytherapy- (AIIMS 05) a) 3.4 years b) 5.2 years
a) Iodine-125 b) Iodine-131 c) 1.2 years d) 2.3 years
c) Cobalt-60 d)Iridium-192 594. Which is/are false about T 112 of radioisotopes-
580. Which ofthe following radioisotopes is commonly a) Ra-226: 1626years (PGI May 10)
used as a source for external beam radiotherapy in b) I-131:60years
the treatment of cancer patients- (AI 03) c) Co-60: 5.26 years
a) Strontium-89 b)Radium-226 d) Ir-192: 74years
c) Cobalt-59 d) Cobalt-60 e) Cs-137: 30years
581. For teletheraphy, isotopes commonly used are- 595. Half-life oflodine 131 is- (DPG 08)
a) I-123 b) Cs-137 (PGI02, Nov. 10) a) 8 hours b) 8 days
c) Co-60 d) Tc- 99 c) 8 weeks d) 8 months
e) Ir-191 596. Most suitable radioisotope of Iodine for treating
582. For teletheraphy setup all are used except -(AliMS 98) hyperthyroidism is- (AI 03)
a) Iridium 127 b) Cobalt 60 a) pn b) Jl25
c) Simulator d) Computer c) JI31 d) p32
583. Which is used in teletheraphy & Brachytherapy 597. Which of the following is used in the treatment of
both- (AIIMS98) differentiated thyroid cancer? (AI 06)
a) Iridium 127 b) Cobalt 60 a) I3IJ b)99mTc
c) Pallidium d) Iodine 131 c) 32p d)311-miBG

569)a 570)c 571)e 572)d 573)a 574)c 575)a,b 576)b 577)d 578)None 579)b 580)d 58l)b,c 582)a
583)b 584)c 585)a,b 586)a,b,c,d 587)All 588)a 589)d 590)a,b,c,e 591)d 592)b 593)b 594)b,d 595)b
596)c 59?) a
RADIOLOGY [ 569]

598. Radioactive phosphorus is used in the treatment of- 610. Which of the following malignant tumors is radio
a) Polycythemia (DPG Feb 09) resistant? (DPG 09, AI 06)
b) Thyroid metastasis a) Ewing Tumour b) Retinoblastoma
c) Multiple myeloma c) Osteosarcoma d) Neuroblastoma
d) Embrynal cell carcinoma 611. Which of these tumors is least radiosensitive-
599. Tore about Cobalt- 60 is- (PGI Dec 04) a) Ewing's sarcoma (AIIMS May 07)
a) Natural radioactive agent b) Osteosarcoma
b) At.wt. 59 c) Wilm'stumor
c) Emits j3 andy-rays d) Neuroblastoma
d) Halflife is 5.3 years 612. Most radiosensitive brain tumor is- (AIIMS 97)
e) Used in both brachy & teletherapy a) Astrocytoma b) Ependymoma
600. Cobalt- 60 is- (MH 03 07, 03) c) Medulloblastoma d) Craniopharyngeoma
a) Natural radioactive source 613. The most radiosensitive tumour among the following
b) Natural radioactive material is- (AI 06)
c) Artificial radioactive source a) Bronchogenic carcinoma b) Carcinoma parotid
d) Artificial stable substance c) Dysgerminoma d) Osteogenic sarcoma
601. Artifical radioisotops- (PGI 02) 614. Most Radiosenstitive ovarian tumor is- (AIIMS 97)
a) Radium b) Uranium a) Serus cystadenoma b) Dysgerminoma
c) Plutonium d) Iridium · c) Demoid cyst d) Teratoma
e) Cobalt 615. Most Radiosensitive tumor- (AIIMS 95)
602. Which of the following. elements is obsolete in a) Brenner's tumor b) Dysgerminoma
radiotherapy- (AIIMS Nov 09) c) Mucinous cystadenoma d) Teratoma
a) Radium 226 b) Cobalt 60 616. Tumor responding best to radiation include
c) Iridium 192 d) Cesium 137 following- (PGI 97)
603. Intercavitatory radiotherapy is treatment modality a) Melanoma b) Dysgerminoma
for- (AIIMS 99) c) Teratoma d) Choriocarcinoma
a) Ca Cervix b) Ca Oesophagus 617. Most Radiosensitive tumor of the following is-
c) Ca Stomach d) Renal cell CA a) CaKidney b)CaColon (AIOJ)
604. Advantage ofbrachytherapy- (PGI Dec 04) c) Ca Pancreas d) Ca Cervix
a) Non-invasive 618. Which one of the following therapeutic mode is
b) Less radiation hazard to normal tissue commonly employed in intra-operative radiotherapy?
c) Max.radiation to diseased tissue a) Electron (DPG 09, AIIMS 03)
d) Can be given in all malignancies b) Photon
e) Doesn't require trained personnel c) X-ray
605. Features of interstitial radiotherapy are all except- d) Gamma rays
a) Only used in head & neck (PGI June 06) 619. IntraoperativeRTisgivenin- (AIIMS98)
b) .t Damage to normal tissue a) Ca Cervix b) Ca Breast
c) Temporary or permanent c) Ca Pancreas d) Ca Thyroid
d) Only iridium used 620. Prophylactic intracranial irradiations are given in-
e) Used for easily accessible organ a) Small cell Ca oflung (PGI 98)
606. Radiation emitts by Ir-192- (PGI Dec 07) b) Testicular Ca
a) 0.5 Mev b) 0.6 Mev c) Ca breast
c) 0.66 Mev d) 0.6 66Mev d) Ca stomach
e) 0.47Mev 621. Prophylactic cranial irradiation not indicated in
607. Radiosensitive tumors are- (PGI June 06) treatment of- (AI 02)
a) Seminoma b)Lymphoma a) SmallcellCaoflung b)ALL
c) Sarcoma d) Ewing's sarcoma c) Hodgkin's lymphoma d)NHL
e) Leukemia 622. Craniospinal irradiation is used in the Treatment of
608. Thmor(s) most responding to radiotherapy -(PGJ 03) a) Oligodendroglioma (AI 02)
a) Sarcoma b) Seminoma b) Pilocytic astrocytoma
c) Lymphoma d) Leukaemia c) Mixed oligoastrocytoma
609. Which ofthe following is the most radiosensitive d) Medulloblastoma
tumour? (AIIMS May 05) 623. Stereotactic radiosurgery uses all except-
a) Ewing tumour a) Proton (AIIMS May 09)
b) Hodgkin's disease b) Electron
·c) Carcinoma Cervix c) Linear accelerator
d) Malignant fibrous histocytoma d) Gammaknife

598)a 599)All 600)c 601)d,e 602)a 603)a 604)b,c 605)a,d,e 606)e 607)a,b,d 608)b,c 609)a 610)c
6ll)b 612)c 613}c 614)b 615)b 616)b 617)d 618)a 619)c 620)a 621)c 622)d 623)b
RADIOLOGY [ 570]

624. Stereotactic Radio-surgery is a form of -(AIIMS 03) 636. Radioprotective drug is- (PGI Dec. 08, AI 01, 12)
a) Radiotherapy b) Radioiodine therapy a) Paclitaxem b) Vmcristine
c) Robotic Surgery d) Cryo Surgery c) Amifostine d) Etoposide
625. Stereotactic Radiotherapy is used in- 637. Amifostine, protects all of the following except-
a) Miliary Lung Metastasis (AIIMS May 12) a) CNS b) Salivary glands (AI 09)
b) Inoperable Stage 1 Lung Tumor c) Kidneys d) GIT
c) Lymphangitis Carcinomatosa 638. Which ofthe following statements about 'Stochastic
d) Tumor at the base of tongue with new Lymph Node
effects' of radiation is true- (AI 10)
enlargement
a) Severity of effect is a function of dose
626. Gamma knife- (PGI June 06)
a) Steel knife b) Probability of effect is a function of dose
b) Used for cutting tumours in difficult location c) It has a threshold
c) Cobalt is used d) Erythema and cataract are common examples
d) Recovery delayed 639. Most sensitive tissue to Radiation is- (AIIMS 95)
627. Isotopes used in relief of metastatic hone pain a) Liver b) Gonads
includes- (PGI May 10) c) Spleen d) Skin
a) Strontium89 b)l-131 640. Organs sensitive to radiation are- (PGI 03)
c) Gold-198 d)P-32 a) Gonad b)Bonemarrow
e) Rhenium- 186 c) Liver d) Fat
628. The technique employed in radiotherapy to e) Nervous tissue
counteract the effect of tumour motion due to 641. What is radioresistant- (AIIMS 97)
hreathingis known as- (AI 05) a) Cartilage b) Seminoma
a) Arc technique b) Modulation c) Ewings sarcoma d) GI epithelium
c) Gating d) Shunting 642. Most radiosensitive tissue of body among the
629. For which malignancy, intensity Modulated following is- (MH 02)
Radiotherapy (IMRT) is the most suitable- a) Bone marrow b) Spleen
a) Lung b)Prostate(AIIMSNov05) c) Kidney d) Brain
c) Leukemias d) Stomach 643. Most common skin manifestation seen after 2 days
630. Hyperfractionation radiotherapy is used in the of radiation therapy is- (DPG 09, MH 05, AI 98)
management of- (DPG Feb. 09) a) Erythema b) Atopy
a) Lung cancer b) Breast cancer c) Hyperpigmentation d) Dermatitis
c) Seminoma d) Ovarian cancer 644. Most common hormone dificiency seen after
631. Radiosensitizer drug(s) is/are- (PGI Dec 08) intracranial radiation therapy- (AIIMSMay 07)
a) Misonidazole b)ActinomycinD
a) Prolactin b) Gonadotropins
c) Oxygen d)Hyperthermia
c) ACTII d) Growth hormone
e) Amifostine
645. Most common presentation of radiation carditis is-
632. Radiation therapy to hypoxic tissues may be
a) Pyogenic Pericarditis (Al97)
potentiatedbythetreatmentwith- (AIIMS 03)
b) Pericardia! Effusion
a) Mycostatin b) Metronidazole
c) Myocardial Fibrosis
c) Methotrexate d) Melphalan
d) Atheromatous Plaque
633. All are radiosensitizer except- (AIIMS 97)
a) 5-Fu b)BUDR 646. Late effects of radiation thearpy- (PGI 02)
c) Cyclophosphamide d) Hydroxyrea a) Mucositis b) Enteritis
634. A patient with cancer received extreme degree of c) Nausea and vomiting d) Pneumonitis
radiation toxicity. Further history revealed that the e) Somatic mutations
dose adjustment of a particular drug was missed 647. LowdoseRadiationcause- (PGINov.10)
during the course of radiotherapy. Which of the a) Lung cancer b)AML
following drugs required a dose adjustment in that c) Cervical cancer d) Glioma
patient during radiotherapy in order to prevent e) Meningioma
radiation toxicity- (AIIMS May 04) 648. MC cancer due to Radiation- (PGI Dec 07)
a) Vincristine b) Dactinomycin a) Leukaemia b) Bronchogenic Ca
c) Cyclophosphamide d) 6- Mercaptopurine c) Thyroid Ca d) Breast cancer
635. Which of the following drugs requires a dose e) Bonetumour
adjustment in patient during radiotherapy in order 649. Radiation exposure during infancy has been linked
to prevent radiation toxicity? (DPG Mcu: 09) to which one of the following carcinoma- (AI 03)
a) Vincristine b) Dactinomycin a) Breast b) Melanoma
c) Cyclophosphamide d) 6-Mercaptopurine c) Thyroid d) Lung

624)a 625)b 626)b,c 627)a,d,e 628)c 629)b 630)a 631)a,b,c,d 632)b 633)c 634)b 635)b 636)c 637)a
638)b 639)b 640)a,b 641)a 642)a 643)a 644)d 645)b 646)a,b,d,e 647)a,b,d,e 648)a 649)c
RADIOLOGY [ 571 ]

650. Post-radiation transverse myelitis usually presents 664. Emergency radiotherapy is given in- (PGI June 05)
at what time after radiation therapy- (MH05) a) Superior vena cava syndrome
a) 1-2 weeks b) 1-2 months b) Pericardia! temponade
c) 4-6 months d) 1 year c) Increased ICP
651. Most Radio sensitive stage -(DPG 09, AI 08, 07,98,96, d) Spinal cord compression
a) S phase PGI Dec 05, 97, AIIMS 02) 665. What dose of radiation therapy is recommended for
b) G 1phase pain reliefin bone metastases- (AIIMS May 04)
a) 8 Gy in one fraction b) 20 Gy in 5 fractions
c) G2 phase
c) 30 Gy in 10 fractions d) Above 70 Gy
d) G2Mphase
666. The ideal timing of radiotherapy for Wilms tumour
652. Most radio resistant phase in cell cycle -(Jipmer 11) after surgery is- (AI 06)
a) q b)EarlyS a) Within I 0 days b) Within 2 weeks
c) LateS d)G2 c) Within 3 weeks d) Any time after surgery
653. Ionoising radiation most sensitive in -(PGI June 05) 667. In which malignancy postoperative radiotherapy is
a) Hypoxia b) S phage minimallyused- (AI 04)
c) G2Mphage d) Activating cell a) Head and neck b) Stomach
654. The phase of cell cycle, most sensitive to radiation c) Colon d) Soft tissue sarcomas
is I are- (PGINov09) 668. Radiation induced necrosis can be diagnosed by-
a) M Phase b)G2 Phase a) PEf b) CT (AIIMSNov 09)
c) S Phase d) Early G 1 Phase c) MRl d) Biopsy
e) G0 Phase 669. Most stable radio-isotope among the following-
655. The maximum permissible radiation exposure per a) 0-18 b)C-14 (MH-11)
year recommended by NCRP for radiation worker c) P-32 d)l-125
is- (MH06, DPG08)
a) 3rad b)8md MISCELLANEOUS
c) lOrad d)50mSv
656. Maximum permissible radiation dose in pregnancy 670. Invertogram to be done in a new born- (PGI 98)
is- (MH03, 02,AIIMS03) a) Immediately b) After 2 hours
a) 0.5rad b) l.Orad c) After 4 hours d) After 6 hours
c) 1.5rad d)rad 671. First sign of hydrocephalus in children is -(PGI 98)
657. Cell most sensitive to radiation - (DPG Feb. 09) a) Post clinoid erosion b) Large head
a) Lymphocytes b) Neutrophils c) Sutural diastasis d) Thinned out vault
c) Basophils d) Platelets 672. William's syndrome is associated with- (AIIMS 98)
a) Congenital supravalvular aortic stenosis
658. The radiation tolerance of whole liver as - (AI04)
a) 15Gy b)30Gy b) Congenital subvalvular aortic stenosis
c) 40Gy d)45Gy c) VSD
d)ASD
659. Point A and point B Manchester location are
673. Which one of the following is the most preferred
important for treatment of which cancer-
route to perform cerebral angiography? (AI 05)
a) Cervix b) Vagina (MH 10, 03)
a) Transfemoral route b) Transaxillary route
c) Ovary d) Uterus
c) Direct carotid puncture d) Transbrachial route
660. Point B in treatment of Ca Cervix by radiotherapy
~th;'';flhi~i'UU··'Duidti•bl;~s~¢n
corresponds to - (DPG Feb. 09)
a) Obturator nodes b) Machenrodt's ligament . .•.·;;•········..·: .·. . ~J41'1l))lllPatternJ
··m~~ ~:·6}ltB 9fendQpiewum:
c) Ischial tuberosity d) Round ligament ~;,:: _;:;';~
<!}~~~e.
;;£;;,, "\ .. .
,--:-.;.:;;:
661. Quantum of Radiation advocated at point B in 675. Investigation of choice in parathyroid pathology is-
carcinoma cervix- (MH 11) a) CT Scan (MH 02, AI 95)
a) lOOOcGy b)2000cGy b) Galliwn scan
c) 5000cGy d)8000cGy c) Thallium scan
662. Maximum recommended external beam radiation d) Tc - Thallium substraction scan
therapy dose for a case of carcinoma cervix is - 676. Which ofthe following is not a CTfeature ofAdrenal
a) 80Gy b)70Gy (MH-11) adenoma- (AI 09)
c) 50Gy d)35Gy a) Low attenuation
663. Photodynamic therapy with hematoporphyrins and b) Homogenous density and well defined borders
light is increasingly used in treatment of? c) Enhances rapidly, contrast stays in it for a relatively
a) OvaryCa b) SkinCa (MHJO) longer time and washes out late
c) ColonCa d) All of the above d) Calcification is rare

650)d 65l)d 652)c 653)c,d 654)a,b 655)d 656)a 657)a 658)c 659)a 660)a 66l)c 662)c 663)d
664)a,b,c,d 665)a 666)a 667)b 668)a>d 669)a 670)d 67l)c 672)a 673)a 674)a 675)d 676)c
RADIOLOGY [ 572 ]

677. All of the following statements about CT scan 685. X-ray view for supra orbital fissure- (AIIMS 97)
features of adrenal adenoma are true, Except- (All 0) a) Towne's b)AP
a) Calcification is Rare c) Caldwell d) Basal
b) Low Attenuation 686. Caldwell lac view (occipito-frontal) can visualise-
c) Early enhancement with slow wash out of contrast a) Spenoid sinus b) Nasal bone (PGI 02)
d) Regular margins c) Maxillary bone d) Ethmoid
678. Which one of the following imaging modalities is e) Frontal sinus
most sensitive for evaluation of extra-adrenal 687. Basal skull view (submentovertical view) X-ray is
Pheochrmocytoma- (AIIMS 03) best to visualize - (PGI 2K)
a) Ethmoid sinus b) Frontal sinus
a) Ultrasound b)CT
c) Sphenoid sinus d) Maxillary sinus
c) MRl d) MIBG scan
e) Nasopharynx
679. The diagnostic procedure not done in case of
688. Neural tube defectis best detected by- (AI 95)
pheochromocytoma- (AIIMSMay07)
a) USG b) Chromosomal analysis
a) CT scan b)MRI c) Amniocentesis d) Placentography
c) FNAC d)MffiGscan 689. Trueaboutvirtnalcolonoscopy- (PGI Nov. 10)
680. Which of the following feature of thyroid nodule on a) Provide endoluminal view
ultrasonogram is not suggestive of malignancy? b) Biopsy can be taken
a) Hyperechogenisity (AI 09, AIIMS May 10) c) CT and MRI used
b) Hypoechogenisity d) Used even when conventional colonoscopy fails
c) Nonhomogenous e) Used of screening of ca colon
d) Microcalcification 690. Which ofthe following is used to differentiate between
681. All of the following modalities can be used for in- arachnoid cyst and epidermoid cyst -(AJIMS Nov. 11)
situ ablation of liver secondaries, except- (AI 06) a) FLAlR b)Tl weightedMRI
a) Ultrasonic waves b) Cryotherapy c) Smooth margin d) Contrast enhancement
c) Alcohol d) Radio frequency 691. In cerebral angiography the dye is injected through-
682. All are X-ray fmdings of retinoblastoma except- a) Femoral artery b) Brachial artery (AI 99)
a) Widening of optic canal (AIIMS99) c) Axillary artery d) Radial artery
b) Intra cerebral calcification 692. Which of the following liver metastasis appear
c) Intraocular calcification hypoechoicon Ultrasonography- (AI 12)
d) Secondaries in cranial bones a) Breast cancer b) Colon cancer
683. 'Thumb print' sign seen in- (PGI June 08) c) Renal cancer d) Mucinous adenocarcinoma
a) Candida b) Aspergillus 693. A child was taken for CECT Chest and contrast was
injected; child had swelling which gradually increased.
c) Thrrmomyces d) Epiglotitis
There is numbness. There is pain on passive extension
· 684. On DSA, typical 'string of beads' appearance of
of fingers. He is not allowing you to touch the arm.
arteris is seen in- (MH 10, AIIMS May 04)
Pulse was present What will you do?
a) Takayasu's disease
a) High Dose Prednisolone (AIIMS May 12)
b) Atherosclerotic stenesis b) Arterial Thrombectomy
c) Fibromuscular dysplasia c) Immediate Fasciotomy
d) Middle aortic syndrome d) Antihistaminics I Anticoagulants

***

677)c 678)d 679)c 680)a 681)a 682)b 683)d 684)c 685)c 686) b,c,d,e 687) c 688) c 689) a,c,d,e
690)a 691)a 692)a 693)c
ANATOMY OF SKIN

2. True about stratum lucidum- (PGI June 09)


a) Sandwitched between S. spinosum and Regarding Fordyce spots- ·
S.granulosum a) Represent internal malignancy
b) Sandwitched between S. corneum and
b) Ectopic sebaceous glands
S.granulosum
c) Present in axillae
c) Contain hair follicle
d) Found in healthy people
d) Also known as prickle cell layer
e) Contain degenerated cells e) Are erythematous
3. Which layer of epidermis is underdeveloped in the 16. Fordyce's disease mainly involves-(ftllMS2K, PGI 97)
VLBW infants in the initial7 days- (AJIMS Nov 02) a) Lips b) Buccal mucosa
a) Stratum germinativum b) Stratum granulosum c) Neck d) Trunk
c) Stratumlucidum d) Stratum corneum 17. InFoxFordycedisease,trueis/are- (PGI 2002)
4. Melanocytes are present in- (PGI 99) a) Commoninadultwoman
a) Stratum corneum b) Stratum basale b) Bullous lesions are common
c) Stratum granulosum d) Dermis c) Common in areola & axilla
5. All statements are true regarding skin except- d) Associated with other malignancies
a) Skin is stratified squamous epithelium (PGI 96) 18. Acnevulgarisinvloves- (AIIMS2001)
b) Melanocyte & merkel cells are immigrant cells a) Pilosebaceous gland b) Sebaceous gland
c) Keratin filaments are a hallmark of epidermal cells c) Eccrine gland d) Apocrine gland
d) Keratinization process cause hydration of cells 19. Acne vulgaris involves- (PGI 96)
e) Spines of spinous cells are formed from house a) Sweat glands b) Sebaceous gland
keeping organelle c) Hair follicles d) Ceruminous glands
6. Basal cell degeneration seen in- (PGI 02) 20. Acne vulgaris is due to involvement of-(PGIJune 06)
a) Lichen planus b) Psoriasis a) Sebaceous glands b) Eccrine glands
c) Pemphigus c) Pilosebaceous glands d) Apocrine glands
7. Acantholysis involves- (PGI Dec 08) e) Sweat glands
a) Epidermis 21. Comedones are seen in- (AIIMS 98, Kerala 94)
b) Dermis a) Acne vulgaris b) Lichen planus
c) Dermo-epidermal junction c) Adenoma sebaceum d) Pityriasis
d Subcutaneous tissue 22. Features of acne vulgaris- (PGI June 06)
a) t secretion
b) Not seen after 20 years
c) t Androgen production causes
9. Dyskeratosis is characteristic feature of -(PGI 2K) d) Rx is penicillin for 1 week
a) Darrier's ds b) Pemphigus vulgaris 23. Causative factor for acne are following except-
c) Psoriasis d) Bowen's disease a) Androgen (PGI 2000, SGPGI 00)
e) Hailey-Hailey ds b) Only food '
10. In which ofthe following conditions Parakeratosis c) Bacterial contamination
most frequently occurs- (AI 06) d) Hypercomification of duct
a) Actinic keratoses ;~~:~d~JJ;~JU:gJil~oltof.sebumleads
b) Seborrheic keratoses · ·.. '· .,. · ~~.·.,£.•:·• ;"(NPE/lltDNB E~tti:wn}
c) Molluscum contagiosum . id:; Y>.~:: t :'.. : ')j)Ail1{. .. . .
d) Basal cell carcinoma ·; · · .. P , ~t~ ..Y$ .·. ·. ·,. ! }:ll,,M~!J,iri!\ · •
11. Fox Fordyce's disease is a disease of- (DPG 08) 25. Treatment of choice for Acne vulgaris -(PGI Dec 05)
a) Eccrine glands b) Apocrine glands a) Minocycline for inflammatory acne
c) Sebaceous glands d) Pilosebaceous glands b) Retinoids for comedonal acne
12. Fordyce's spot involves- (SGPGI 05) c) Etretinate
a) Sebaceous glands b) Pilosebaceous glands d) Rifumpicin
c) Hair d) Nails e) Dapsone

l)a 2)b,e 3)d 4)b S)d,e 6)a 7)a 8) a 9) a,d,e 10) a ll)b 12)a 13)a 14)c
15) b,d 16) a>b 17) a,c 18)a 19)b 20)a,c 21)a 22) a,c 23) b 24) b 25)a,b
SKIN [596]

26. Drug treatment of acne vulgaris- (PGI June 05) 37. A male presents with focal alopecia and boggy
a) Minocycline b) Penicillin swelling on scalp and easily plnckable hair.
c) Co-triamoxazole d) Gentamycin Diagnosis is - (AIIMS 95)
e) Tetracycline a) Alopecia areata b) Patterned baldness
27. Treatment of acne- (PG!June 05) c) Telogen effluvium d) Tinea capitis
a) 13-cis retinol b) Minocycline 38. Anagen phase of the hair indicates-(AIIMSMay 06}
c) Erythromycin d) Dapsone a) The phase of activity and growth
e) Rifampicin b) The phase of transition
28. A patient presented with multiple nodulocystic c) The phase of resting
d) The phase of degeneration
lesions on the face. The drug of choice is-
39. Growth phase of hair is- (PGI 99, Al98)
a) Retinoids b) Antibiotics (AIIMS May 02)
a) Anagen b) Metagen
c) Steroids d) UV light
c) Telogen d) None
29. A 24 year old unmarried woman has multiple
40. This phase in hair cycle determines the length to
nodular, cystic, pustular and comadonic lesions on which hair in different body areas can grow-
face, upper back and shoulders for 2 years. The drug a) Anagen b) Catagen (Karn 11)
of choice for her treatment would be- (AI 06) c) Telogen d) Kenogen
a) Acitretin b) Isotretinoin 41. Cicatricial alopecia is seen in- (PGI Dec 07)
c) Doxycycline d) Azitbromycin a) DIE b) Psoriasis
30. Treatment ofnodulocystic acne is-(AIIMS 95, AI 94) c) Alopecia areata d) Lichen planus
a) Erythromycin b) Tertacycline e) SI.E
c) Isoretinonine d) Steroids 42. This is a cause for scarring alopecia- (Karn 11)
31. A teenager girl with moderate acne is also a) Alopecia areata b) Aplasia cutis
complaining of irregular menses. Drug of choice c) Trichotillomania d) Sarcoidosis
will be- (AIIMS Nov J0) 43. All of the following are causes of cicatrizing alopecia
a) Oral isotretinon b) Oral acitretin except- (AIIMSNov07)
c) Oral minocycline d) Cyproterone acetate a) Lichen planus b) Discoid lupus erythematosus
32. A female patient presents with diffuse alopecia to c) Alopecia areata d) Lupus vulgaris
you. She had suffered from typhoid fever 4 months 44. Cicatricial alopecia is seen in- (PGI Dec 04)
back. Most probable diagnosis is- (AIJMS Nov 07) a) DIE b)SLE
a) Androgenetic alopecia b) Telogen effluvium c) Secondary syphilis d) Psoriasis
c) Anagen effluvium d) Alopecia areata e) Lichen planus
33. A 30 year old female developed diffuse hair loss 3 45. Scarring alopecia is seen in- (AI 09)
a) T. Capitis b)Androgenic alopecia
months after delivery of her first child. The probable
c) Alopecia areata d) Lichen planus
diagnosis is- (PGI OJ, SGPGI03)
46. Which ofthe following conditions causes alopecia
a) Androgenic alopecia b) Endocrinal alopecia
without scarring- (J & K 05)
c) Telogeneffluvium d)SLE
a) DIE b) Herpes Zoster
34. Diagnosis of a man with diffuse hair loss involving c) Alopecia areata d) Lichen planus pilaris
crown & frontal scalp with maintenance offrontal 47. Cicatricial alopecia is seen in- (AI 99)
hair line- (Jipmer 98) a) Tenia capitis b) Psoriasis
a) Alopecia areata DIE Alo ecia aereta
b) Anagen effluvium liatt:ifnJ
c) Male pattern baldness :J~tst\~r
d) Female pattern baldness :t'' ~{
35. A male patient presents with patchy loss of hair on Exclamation mark alopecia is a feature of-
scalp, eyebrows and beard. He also gives history of a) Telogen effluvium (AIIMS Nov 05, Bihar 06,
rapid graying of hair in a few areas. The likely b) Androgenic alopecia Kerala OJ, Karn 05)
diagnosis is- (AI 08) c) Alopecia aerata
a) Alopecia areata b) Anagen effluvium d) Alopecia mucinosa
c) Telogen effluvium d) Androgenic alopecia 50. In alopecia areata, seen is- (PGI 97)
36. A 8 year old child has localized non cicatrial alopecia a) Exclamationmarkhair b) Scarring
over scalp with itching and scales. The diagnosis c) Fungal infection d) Traumatic
is- (AIIMS93) 51. Anagen effluvium is induced by all except-
a) Tinea Barbae b) Alopecia areata a) Radiation b) PEM (Karn 08)
c) Tinea Capitis d) Lichen planus c) Anticancer drug d) Prolonged labor

26) a,c,e 27) a,b,c,d 28) a 29) b 30) c 31) d 32) b 33)c 34)d 35)a 36)c 37)d 38)a 39)a
40) a 41) a,d,e 42) b,d 43) c 44) a,b,e 45) d 46) c 47)c 48)b 49)c 50) a 51)d
SKIN [597]

52. Contraindicated in Androgenic Alopecia- (AI 2000) 64. A24 year old man had multiple, small hypopigmented
a) Testosterone b) Minoxidil macules on the upper chest and back for the last
c) Cyproterone d) Finasteride three months. The macules were circular, arranged
53. Nail is involved in- (PGI Dec 07) around follicles and many bad coalesced to form
a) Psoriasis b) Lichen planus large sheets. The surface of the macules showed
c) Fungal infection d) Alopecia fine scaling. He had similar lesions one year age
e) Viral infection which subsided with treatment The most appropriate
54. Nails are involved in all except- (SGPGI 05) investigation to confirm the diagnosis is -
a) Psoriasis b) Lichen planus a) Potassium hydroxide preparation of scales
c) Taenia d)DLE b) Slit skin smear from discrete macules
55. Nail involvement is not a feature of- (AIIMS 94) c) Tzanck test (AIIMS Nov 03)
a) Psoriasis b) Lichen planus d) Skin biopsy of coalesced macules
c) Dermatophytosis d)DLE 65. An adult presents with oval scaly hypopigmented
56. Pitting nail dystrophy seen in-(PGIJune 08, DPG 09) macules over chest and back. The diagnosis
a) Dermatophytic infection is- (AIIMS OJ)
b) Psoriasis a) Leprosy b) Lupus Vulgaris
c) Lichen planus c) Pityriasis versicolour d) Lichen Planus
d) Seborrheic dermatitis 66. Babloo a 5 year boy presents with small
57. Pitting of nails is seen in- (DELHIPGFeb. 09) hypopigmented scaly macule on check. Some of his
a) Lichen planus b) Psoriasis classmates also have similar lesions. The most
c) Pemphigus d) Arsenic poisoning probable diagnosis is- (AIIMS 2000)
58. Moth eaten alopecia is seen in- (Corned 08) a) Pityriasis rosea b) Pityriasis versicolour
a) Black dot tinea b) Telogen effluvium c) Indeterminate leprosy d) Pityriasis alba
c) Alopecia areata d) Secondary syphilis 67. 7 year old boy chandu presents with reccurent,
scaly, hypopigmented patch on face, diagnosis is -
a) Vitiligo (AIIMS 95, 96, 98, 99, OJ)
b) Indeterminate leprosy
c) pityriasis alba
d) Pityriasis rosea
periungal fibroma is seen in- 68. A 5 year boy has multiple asymptomatic oval and
a) Tuberous sclerosis (PGI 96, Jipmer 02) circular faintly hypopigmented macules with fine
b) Neurofibromatosis scaling on his face. The most probable cilinical
c) Psoriasis diagnosis is- (AI 03)
d) Alopecia aerata a) Ptyriasis versicolor b) Indeterminate leprosy
61. Wrong statement is- (AI 2000) c) Ptyriasis alba d)Acrofacial vitiligo
a) Mees line in arsenic poisoning 69. An 8year old boy from Bihar presents with a 6 month
b) Pterygium of nails in Lichen planus H/o an ill defined hypopigmented slightly atrophic
c) Onycholysis in Psoriasis macule on the face. The most likely diagnosis is-
d) Koilonychia in Megaloblastic Anemia (B 12 det) a) Ptyriasis alba (AI 05,DPG 09)
b) Indeterminate leprosy
SKIN LESIONS AND DISORDERS OF c) Morphacea
PIGMENTATION d) Calcium deficiency
70. A 6 year old child from Tamil N adu presents with a
62. A boy comes from Bihar with non-anesthetic solitary nonanaesthetic patch on face, the possible
hypopigmented atrophic patch over face, diagnosis diagnosis is- (AIIMS98)
is- (AIIMS 99) a) Pityriasis alba b) Indeterminate leprosy
a) P. alba b) P. versicolour c) Pityriasis versicolor d) Tinea infection
c) Indeterminate leprosy d) Borderline leprosy 71. Piebaldism refers to- (KARNATAKA PGMEE 06)
63. A child presents with solitary white well defmed a) Androgenetic alopecia
patch on his right thigh. Diagnosis is- (AJIMS 2000) b) Erythema nodosum leprosum
a) Piebaldism b)Albinism c) Associated with white forelock
c) Nevus achromicus d) Acral vitiligo d) None of the above

52) a 53)All 54)d 55)d 56) b,c 57) a,b 58) d 59) a 60)a 6l)d 62)c 63)c 64)a 65)c
66)d 67)c 68)c 69)b 70)b 7l)c
SKIN [598]

85. Vitiligo vulvaris, treatment is- (Jharkhand 06)


a) PUVA b) Steroids
c) Coaltar d)All
86. True about vitiligo are all except-(Jipmer 04, WB 05)
a) Genetic predisposition is known
b) Leucotrichia is associated with good prognosis
73. Which of the following is/are not the cause of c) PUVA is used for treatment
hypopigmentation· (PGI May 10) d) Topical steroids give good results
a) Leprosy b) Pinta 87. With regard to vitiligo, which one of the foUowing is
c) Syphilis d) Pytriasis alba NOT correct- (UPSC-I 09)
e) Pytriasis versicolor a) Generalized vitiligo is often symmetrical
7 4. Hypopigmented lesions are seen iu- b) Trauma and sunburn may precipitate vitiligo
a) Melasma (DELHI PG Mar. 09) c) It is a congenital condition
b) Pityriasis alba d) Focal areas of melanocyte loss are observed
c) Lichen planus pigmentosus 88. A female has hypopigmented lesion on centre of
d) Urticaria pigmentosa forehead. Drug responsible is ? (AIIMS Nov 08)
75. Hypopigmented patches can be seen in-(PGIDec 05) a) Hydroquinone
a) Becker naevus b) Freckles b) Ether metabolite ofhydroquinone
c) Nevus Ito d) Nevus Ota c) Para tertiary butyl catechol
e) Nevus anemicus d) Para tertiary butylphenol
76. Hypo.depigmented lesion seen in- (PGI 04) 89. A 23 year old lady develops brown macular lesions
a) Naevas Ito b) Naevus depigmentosa over bridge of Nose and cheeks foUoWing exposure
c) Naevas Ota d) Naevas anaemicus to light. The probable diagnosis is-(AIIMS 99, AI 01)
e) Freckles a) Chloasma (AIIMS May 12)
77. Hypopigmentedpatchespresentin- (SGPGI 02) b) Acne rosacea
a) Nevus Ota b) Nevus Ito c) Systemic lupus erythematosis
c) Nevus anemicus d) Becker nevus d) Photodermatitis
78. True about pitryiasis alba- (PGI 0 1) 90. True about acanthosis nigricans- (PGI 03)
a) No active treatment required a) Most commonly seen in obesity
b) Common in elderly b) Seen in axilla
c) Variantofvitiligo c) It signifies internal malignancy
d) Common over the face d) It is associated with insulin resistance
e) Presents as scaly, whitish macules e) Seen in old age
79. All of the following is given for the treatment for 91. Acanthosis Nigricans in old pt. UsuaUy indicate-
Pityriasis versicoler Except- (AI 02) a) Skin disorder (Jharkhand 04)
a) Ketoconazole b) Griseofulvin b) Malignancy
c) Clotrimazole Selenium sulphate c) Senile brain
d) Usually found in negro
92. Acanthosis nigricans histological show - (MH 10)
a) Papillomatosis b) Marked acanthosis
c) Hypermelanosis d) Ail of the above
93. Diffuse form ofhyperpigmentation is seen in-(MH06)
81. Commonesttypeofvitiligo- (MP 08) a) Vitamin B 12 deficiency
a) Vulgaris b) Segmental b) Ectopic ACTH secretion
c) Acrofacial d) Facial c) Whipple's disease
82. An increased incidence ofvitiligo is found is-(AIIMS 94) d) All of the above
a) Psoriasis b) Nutritional deficiency 94. A 20 year old young female had history of recurrent
c) Old age d) Diabetes mellitus acne on the face treated with many antibiotics like
83. Most common endocrine disturbance associated with clindamycin, tetracycline, erythromycin and others.
vitiligo is- (Jipmer 11) Now she complained of multiple hyperpigmented
a) Addison's disease b) Thyroid disease spots at the site of old acne lesions. Which of the
c) Diabetes insipidus d) Diabetes mellitus following drug might have been responsible for
84. Vitiligo vulvaris, best treatment is - (Bihar 03) this - (AI 10)
a) PUVA b) Steroids a) Erythromycin b) Tetracycline
c) Coaltar d) All c) Minocycline d)Demeclocycline

72)d 73)c 74)b 75)e 76)b,d 77)c 78)a,d,e 79)b 80)c 8l)a 82)d 83)b 84)a 85)d
86)b 87)c 88)d 89)a 90) a,b,c,d 91) b 92) a 93) d 94) c
SKIN [599]

95. A 17 years old girl with acne has been taking a drug 105. A 25 year old man presents with recurrent episodes
for the last two years. She now presents with blue of flexural exzema contact urticaria, recurrent
black pigmentation of nails. The likely medication skin infections and severe abdominal cramps and
causing the above pigmention is- (AI I 0) diarrhoea upon taking sea foods. He is suffering
a) Tetracycline b) Minocycline from- (AI 04, Manipal 09)
c) Doxycycline d)Azithromycin a) Seborrheic dermatitis
96. Rain drop pigmentation is seen in- (AP 96) b) Atopic dermatitis
a) Chronic lead poisoning
c) Airborne contact dermatitis
b) Chronic Arsenic poisoning
d) Nummular dermatitis
c) Chronic Mercury poisoning
d) All of the above 106. Rakesh, a 7-year old boy had itchy, excoriated papules
97. Topical azelaic acid is used in- (COMED 09) on the forehead and the exposed parts of the arms
a) Hyperpigmentary disorders and legs for 3 years. The disease was most severe
b) Dermatophyte infections in the. rainy season and improved completely in
c) Vitiligo winter. Most likely diagnosis is -(AIIMS May 12)
Mixed infections a) Insect bite hypersensitivity
b) Scabies
c) Urticaria
d) Atopic dermatitis
107. Seborrhoeic dermatitis is frequently seen in -
a) Lipid storage disorders (COMED 09)
b) Psoriasis vulgaris
c) Hypertension
100. Neither raised nor depressed is- (PGI97) d) Parkinson's disease
a) Macule b) Plaque
108. Spongiosis is seen in - (PGI97)
c) Nodule d) Papule
a) Acute eczema b) Lichen planus
DERMATITIS c) Psoriasis d) Pemphigus
109. Characteristic feature of atopic dermatitis is-
101. An infant presented with erythematous lesions on a) Pruritus (MP 05)
cheek, extensor aspect of limbs, mother has history b) Dennie's Lines
of bronchial asthma, the probable diagnosis is - c) Scaling skin (Lichenification)
a) Air borne contact dermatitis (AI 07) d) Rash
b) Atopic dermatitis 110. 'Itch is disease' is true for - (UPSC 2K)
c) Seborraehic dermatitis a) Atopic dermatitis b) Insect bites
d) Infectious eczematoid dermatitis c) Seborrheic dermatitis d) Tinea cruris
102. A 3 months old infant is having thick scales on 111. Dennie-Morgan fold is seen in -(Maharashtra 07)
scalp with dandruff. Erythmatous scaly lesions on a) Dermotomyositis b) SLE
nasolabial folds are also present. Diagnosis is - c) Psoriasis vulgaris d) Atopic dermatitis
a) Atopic dermatitis (SGPGI 03) 112. M.C.site of Atopic dermatitis- (AIIMS 95, Jipmer
b) Seborrheic dermatitis a) Scalp b) Elbow 93, DPG 09)
c) Xerotic eczema
c) Antecubital fossa d) Trunk
d) Napkin dermatitis
113. Atopic dermatitis is diagnosed by- (AI99)
103. A 24 years male has eczematous dermatitis involving
scalp and face with greasy yellow scales. He is also a) Patch test b) Wood lamp
suffering from severe dandruff. He is having - c) Clinical examination d) IgE
a) Atopic dermatitis (DPG 03) 114. Minor clinical feature in diagnosis of atopic
b) Contact dermatitis dermatitis AlE - (PGI Dec 04)
c) Seborrheic dermatitis a) Dry skin b) Pruritus
Discoid dermatitis c) Morgagnian fold d) Pitriasis alba
e) Dermographism

95)b 96)b 97)a 98)d 99)a lOO)a lOl)a 102)b 103)c 104)a 105)b l06)a 107)d 108)a
109)a llO)a lll)d 112)c 113)c 114)b 115)c
SKIN [600]

116. In India, the plant which causes dermatitis most 129. All may lead to hives and wheals except- (PGI 97)
commonly isM (AIIMS May 08) a) Cold b) Hepatitis C
a) Parthenium grass b) Cotton fibers c) Serum sickness d) Typhoid fever
c) Poison Ivy d) Ragweed 130. A patient gets reccurent urticaria while doing
117. M.C. cause of Airborn Dermatitis in India is M exercise and on exposure to sunligh. Which of the
a) Parthenium b) Calotropis (AI99) following is most like cause - (AIIMS 2000)
c) Crysophillus d) Dust a) Chronic Idiopathic Utricaria
b) Universal dermographism
118. Most common cause of allergic contact dermatitis
c) Cholinergic utricaria
in Indian female is M (AIIMS 2000)
d) photodermatitis
a) Vegetables b) Nail polish 131. A 22 year old woman developed small itchy wheals
c) Detergents d) Dyes after physical exertion, walking in the sun, eating
119. M.C. precipitant of contact dermatitis is M(AI 93) hot spicy food and when she was angry. The most
a) Gold b) Nickle likely diagnosis is M (AIIMS Nov 03)
c) Silver d) Iron a) Chronic idiopathic utricaria
120. Patch test is read after- (AI 99) b) Heat urticaria
a) 2 hours b) 2 days c) Solar urticaria
c) 2 weeks d) 2 months d) Cholinergic urticaria
121. Patch testing is done for - (PGI Dec 08) 132. Following is a type of physical urticaria -
a) Atopic dermatitis a) Dermographism (DELHI PG Mar. 09)
b) Irritant contact dermatitis b) Urticaria pigmenthosa
c) Allergic contact dermatitis c) Urticaria vasculitis
d) Discoid dermatitis d) Auto-immune urticaria
e) Seborrheic dermatitis 133. Urticarial lesions are bestdescribedas-(CO.MED 09)
a) Nonpruritic b) Bullous
122. Air-borne contact dermatitis can be diagnosed by-
c) Evanescent d) Macular
a) Skin biopsy (AIIMS May 06, AI 92, DPG 09)
134. A 5 year old male child has multiple
b) Patch test hyperprigmented macules over the trunk. On rubbing
c) Prick test the lesion with the rounded end of a pen, he developed
d) Estimation of serum IgE levels urticarial wheal, confined to the border of the lesion.
123. Patch test is done to document - (AIIMS May 09) The most likely diagnosis is -(AI 04, Manipal 09)
a) Type I hypersensitivity a) Fixed drug eruption b) Lichen planus
b) Delayed type hypersensitivity c) Urticaria pigmentosa d) Urticarial vasculitis
c) Autoimmune disease 135. A 9 year old has multiple itchy erythematous wheals
d) Irnmunocomplex deposition .all over the body for 2 days. There is no respiratory
124. A 55 year old male, with uncontrolled diabetes difficulty. Which is the best treatment?(AJJMS 04)
mellitus and hypertension, developed severe air- a) Antihelminthics
borne contact dermatitis. The most appropriate b) Systemic corticosteroids
drug for his treatment would be - (AIIMS 04) c) Antihistamines
a) Systemic corticosteroids b) Thalidomide ·d) Adrenaline
c) Azathioprine d) Cyclosporine 136. All are true regarding hereditary angioedema,
except ? (PGI 98)
125. Coin shaped eczema is - (Jharkhand 05)
a) Dysfunction of enzyme is most common cause
a) Nummular eczema b) Atopic acema
b) Enzyme involved is Cl INH
c) Infantile eczema d) Endogenous eczema c) C 1 inhibitor targests Hageman factor
126. Exfolative dermatitis can be due to all of the following d) Complement C4 & C2 decrease
disease except- (AI 02) e) Bradykinin level decrease during attack
a) Drug hypersensitivity b) Pityriasis rubm pilaris 137 A man takes peanut and develops, tongue swelling,
c) Pityriasis rosea d) Psoriasis neck swelling, stridor, hoarseness of voice. What
127. Causes of erythroderma- (PGI June 05) is the probable diagnosis M (AIIMS Nov 06)
a) Pityriasis alba b) Pityriasis versicolor a) Angioneurotic edema b) FB bronchus
c) Psoriasis d) Lichen planus c) Parapharyngeal abscess d) FB in larynx
e) Eczema 138. A patient presents with history of episodic painful
128. Gold poisoning leading to exfoliative dermatitis is edema of face and larynx. Which of the following
treated by- (AIIMS 95) is likely to be deficient- (AI 09)
a) Chloroquin b) Steroid a) Complement C3 b) Complement C5
c) Antibiotics d) Antihistaminics c) Cl Esterase Inhibitor d) Properidin

116)a 117)a 118)None 119)b 120)b 121)c 122)b 123)b 124)c 125)a 126)c 127) c,d,e 128) b 129) d
130)c 131)d 132)a 133)c 134)c 135)c 136)a,e 137)a 138)c
SKIN [601]

139. A person present with recurrent swelling on face 149. A child is presenting with erythmatous follicular
and lips due to emotional stress, cause is - papules on trunk. There are areas of normal skin
a) Cl esterase inhibitor deficiency (AJIMS May 09) within the lesions. There is thickening of Palms
b) Allergy & soles. This child is suffering from- (MP 07)
c) Anaphylaxis a) Pityriasis rosea b) Pityriasis rubra pilaris
d) None of the above c) Psoriasis d) Seborrheic dermatitis
140. Immediately after eating, a man develops swelling 150. A patient presented with scarring alopecia, thinned
·of face and lips, respiratory distress, intense nails, hypopigmented macular lesions over the trunk
pruiritis, hypotension and feeling of impending and oral mucosa. The diagnosis is- (AIIMS OJ)
doom. The most likely diagnosis is- (AI 09}
a) Psoriasis b) Leprosy
c) Lichen planus d) Pemphigus
a) Angioneurotic edema b) Anaphylaxis
151. A young female presented with lucy linear lesions on
c) Myocardial infarction d) Food stuck in throat
tongue since a month with elongation of nail fold
141. Not true about angioneurotic edema? (AI 09}
beyond the nail bed. What is the diagnosis -(AI J0)
a) Pitting edema offace, lips and mucous membrane a) Psoriasis b) Geographic tongue
b) C 1 Esterase inhibitor deficiency can cause it c) Lichen planus d) Candidiasis
c) Extreme temperature exposure can provoke it 152. Multiple erythematous annular lesions with
d) Known with ACE inhibitors peripheral collorette of scales arranged
predominantly on trunk are seen in -(AI J2, AI 02}
PAPULOSQUAMOUS DISORDERS a) Pityriasis vesicular
b) Pytiriasis rubra pilaris
142. All are causes ofpapulo squamous lesions except- c) Pityriasis rosea
a) Psoriasis (PGI 2000, SGPGI 02) d) Pityriasis lichennoides chronica
b) Parapsoriasis 153. A 16 year old boy presented with asymptomatic,
c) Squamous cell carcinoma multiple erythamatous annular lesions with a
d) Mycosis fungoides collarette of scales at periphery of the lesions
e) Congenital syphilis present on the trunk. The most likely diagnosis
143. All of the following may lead to plaque formation is- (AI 05)
except- (PGI 99, SGPGI 03) a) Pityriasis versicolor b) Pityriasis alba
a) Psoriasis b) Lichen planus c) Pityriasis rosacea d) Pityriasis rubra pilaris
c) Pityriasis rosea d) Pemphigus 154. Which of the following seen in Psoriasis -
144. The important feature of psoriasis is -(AMC 99) a) Pautrier's micro abscess (PGI Dec 07, Dec 06)
a) Crusting b) Scaling b) Micro-munro abscess
c) Oozing d) Erythema c) Grenz zone
d) Hyperkeratosis
145. A 30 years old male presented with silvery scales
e) Suprapapillary thinning
on elbow and knee, that bleed on removal. The
155. All are true about psoriasis except - (PGI 2000)
probable diagnosis is - (AI 98)
a) Very pruritic
a) Pityriasis b) Seborrhoeic dermatitis b) Pitting of nails
c) Psoriasis d). Secondary syphilis c) Joint involvement in 5-10%
146. A male is presenting with a violaceous papule d) Parakeratosis & acanthosis
which is pruritic. There is pterygium of nail with e) Munro abscess
cicatracial alopecia. Diagnosis is - (PGI 98) 156. Typical silvery scales of psoriasis are absent in -
a) Psoriasis b) Pityriasis rosea a) Scalp b) Kee (Kerala 06)
c) Lichen planus d) Seborrheic dermatitis c) Groin d) Elbow
147. 10 year old child has violaceous papule and 157. Least common site involvement in psoriasis is-
pterygium of nails. The diagnosis is - (AIIMS 99) a) Scalp (AI 98, SGPGI OJ)
a) Psoriasis b) Pemphigus b) Nail involvement
c) Pemphigoid d) Lichen planus c) CNS involvement
148. An adult is presenting with severe dandruff and d) Arthritis
greasy yellow scales on scalp. Diagnosis is - 158. All are true regarding Psoriasis except -(AI 2000)
a) Seborrheic dermatitis (PGI 94) a) Arthritis in 5%
b) Pityriasis rubra pilaris b) Abscess is seen
c) Pityriasis rosea c) Head, neck and face are not involved
d) Lichen planus d) No scaly, red lesions are seen in inframammary
and natal area

139)a 140)b 14l)a 142)e 143)d 144)b 145)c 146)c 147)d 148)a 149)b 150)None 15l)c
152)c 153)c 154)b,d,e 155)a 156)c 157)c 158)c
SKIN [602]

159. A patient presents with erythematous scaly lesions 169. Bleeding spots seen on removal of scales in psoriasis
on extensor aspect of elbows and knee. The clinical is called as - ·(PGI June 08)
diagnosis is got by - (AIIMS May 02) a) Auspitz sign b) Punctate hemorrhage
a) Auspitz sign b) KOH smear c) Nikolysk's sign d) Darrier sign
c) Tzanck smear d) Skin biopsy
160. Most common type of psoriasis- (DNB 07)
a) Plaque psoriasis b) Psoriasis vulgaris
c) Flexural psoriasis d) Penile psoriasis 171. All of the following are seen in psoriasis except-
161. This type of psoriasis is commonly seen in children a)Auspitz sign present (SGPGI 03)
and may follow a streptococcal sore tbroat-(Karn 11) b)10% associated with arthritis
a) Guttate b) Stable plaque c)It is premalignant disease
c) Pustular d) Artbropathic d)Worsening of disease during winter
162. Psoriasis associated with p- hemolytic streptococcal 172. Psoriasis is characterized by all, except- (DEUD PG
infection- (Maharashtra 09) a) Defmite pink plaque with clear margin Feb. 09)
a) Psoriasis vulgaris b) In children disappear in 2 weeks to reappear again
b) Erytbrodermic psoriasis c) Always associated with nail infection
c) Pustular psoriasis d) Involves knee and elbow
d) Guttate psoriasis 173. Treatment of psoriasis - (PGI June 05)
163. Normal healthy child developing psoriatic lesions a) PUVA b) Methotrexate
after an episode of streptococcal infection. Which c) Systemic steroids d) Femicycline
type of lesion is this? (Maharashtra 08) e) Terbinafine
a) Plaque psoriasis b) Inverse psoriasis 174. Vitamin D analogue calcitriol is useful in the
c) Pustular psoriasis d) Guttate psoriasis treatment of-
164. "Auspitz" sign is characteristically seen in - a) Lichen planus b) Psoriasis
a) Plaque Psoriasis (AIIMS May 08) c) Phemphigus d) Leprosy
b) Pustular Psoriasis 175. Treatment of pustular psoriasis is-(AIIMS May 02)
c) Lichen planus a) Thalidomide b) Retinoids
d) Inverse Psoriasis c) Hydroxyurea d) Metholtrexate
165. A patient with psoriasis was started on systemic 176. Treatment of choice in Pustular psoriasis- (AI 94)
steroids. After stopping the treatment, patient a) Psorialin + UV therapy b) Systemic steroid
developed universally red scaly skin with plaques c) Methotrexate d) Estrogen
losing their margins all over his body. The most 177. DOC for a pregnant woman in 2nd trimester with
likely cause is - (AIIMS OJ) pustular psoriasis is- (AI 08)
a) Drug induced reaction a) Prednisolone b) Dapsone
b) Pustular psoriasis c) Acitretin d) Methotrexate
c) Bacterial infection 178. The treatment of choice for erythrodermic psoriasis
d) Erytbrodermic Psoriasis is - (SGPGI 04)
166. A patient with psoriasis was started on systemic a) Corticosteroids b) Methotrexate
steroids. After stopping treatment, the patient c) Coaltar topically d) Topical corticosteroids
developed generalized pustules all over the body. 179. A primigravida female presented with erythematous
The cause is most likely to be - (AI 02) skin changes and lesions with multiple pus lakes.
a) Drug induced reactionb) Pustular psoriasis Which of the following should be the most
c) Bacterial infections d) Septicemia appropriate- (AI 10)
167. Munro micro abscess is seen in- (PGI 99) a) Corticosteroids b) Atretinin
a) Dermal tissue b) Stratum basale c) Psorolen + PUVA d) PUVA
c) Stratum corneum d) Stratum malpighi 180. Drug of choice for impetigo herpetiformis is -
168. About micro-munro abscess which ofthe following a) Prednisolone b) Dapsone (DPG 08)
statements are true - (PGI June 09) c) Acitretin d) Methotrexate
a) Seen in stratum corneum 181. The only indication of giving corticosteroids in
b) Seen in psoriasis pustular psoriasis is- (AI 05)
c) Contain neutrophils and lymphocyte a) Psoriatic erythroderma with pregnancy
d) Contain neutrophil only b) Psoriasis in a pt. with alchoholic cirrhosis
e) Associated pustules are normally seen c) Moderate arthritis
d) Extensive lesions

159)a 160)a,b 16l)a 162)d 163)d 164)a 165)d 166)b 167)c 168)a,b,d 169)a 170)b 171)c 172)c
173) a,b 174) b 175) b 176)b 177)a 178)b 179)a 180)a 181)a
SKIN[603]

182. The only defmitive indication of systemic steroids 195. Lacy white lesion in mouth with pterygium is seen
in psoriasis- (AIIMS Nov 11) in - (SGPGI 01)
a) Pustular psoriasis a) Psoriasis b) Ptirysis alba
b) Impetigo herpetiformis c) Lichen planus d) Leprosy
c) Psoriatic arthropathy 196. A young lady present with white lacy lesions in
d) Erythroderma oral cavity and her proximal nail fold has extended
183. Psoriasis is exacerbated by- (AI 98) onto the nail bed. What is the likely diagnosis -
a) Lithium b) p - blockers a) Psoriasis (AI 12, 10)
c) Antimalarials d) All of the above b) Geogl'aphic tongue
184. All of the following drugs can worsen psoriatic c) Lichen planus
lesion, except - (J & K 05) d) Candidiasis
a) Chloroquine b) Beta-blockers 197. Regarding Lichen planus all are true except-
c) Acitretin d) Lithium a) Hypopigmentation in residual disease (AI 2000)
185. Treatment of psoriasis include all except-(MH03) b) Lymphatic infiltration in supradermallayer
a) Retinoids b) Methotrexate c) Itchy, polygonal, purple, papule
c) Cyclosporine d) Oral corticosteroides d) Skin, Hair & oral mucosa commonly involved
198. True about Lichen planus - (PGI Dec 04)
a) Basal cell degeneration
b) Colloid bodies seen
c) Epidermal hyperplasia in chronic cases
d) Wickham's striae seen
e) Autoimmune disease
199. Cicatrising alopecia with perifollicular blue- gray
patches is most commonly associated with -(AI 11)
a) Pitting of nails
b) Whitish lesions in the buccal mucosa
c) Arthritis
d) Discoid plaques in the face
200. Civatte bodies are found in - (Kerala 98)
a) Lichen planus b) Psoriasis
189. The most characteristic finding in lichen planus is- c) Dermatophytosis d) Vitiligo
a) Civatte bodies (PGI99) 201. All of the following regarding Lichen planus are
b) Basal cell degeneration true except - (MP 05)
c) Thinning of nail plate a) Does not involve mucous membrane
d) Violaceous lesions b) Associated with Hepatitis 'C'
190. Basal cell degeneration seen in-(PGI 02,Jipmer 98, c) Topical steroid are the mainstay of therapy ·
a) Lichen planus b) Psoriasis 02) d) Spontaneous remissions 6mo to 2 years
c) Pemphigus d) DLE 202. Pityriasis rosea true - {AIIMS Nov 06, AI 07)
191. Max. Joseph's space is a histopatho-logical feature a) Selflirniting
of - (AIIMS May 06, MH 11) b) Chronic relapsing
a) Psoriasis vulgaris b) Lichen planus c) Life threatening infection
c) Pityriasis rosea d) Parapsoriasis d) Caused by dermatophytes
192. Wickehm's stria seen in-(AI 02, SGPGI 02, Bihar 03) 203. Annular herald patch is seen in - (AIIMS 97,
a) Lichen niditus b) Lichenoid eruption a) Psoriasis b) P. alba Jharkhand 05)
c) P. rosea d) Nocardiasis
c) Lichen striates d) Lichen planus
204. A patient ofhypertension ofACE inhibitors developed
193. Most characteristic feature of lichen planus is -
rosea skin erruptions. True statement regarding
a) Thinning of nail plate is most common
this situation is - (SGPGI 03)
b)Non scarring alopecia (PGI98) a) Drug may be the cause and discontinuation may
c) Violaceous lesions on skin and mucous membrane improve the skin condition
d) Wickham striae b) High dose steroids are needed initially
c) ACE inhibitors are safe and cannot lead to skin
eruptions
d) Drug may be the cause but discontinuation is
not required

182)b 183)d 184)c 185)d 186)c 187)d 188)c 189)b 190)a 19l)b 192)d 193)d 194)b 195)c
196)c 197)a 198)All 199)b 200)a 20l)a 202)a 203)c 204)a
SKIN [604]

205. Which viral association is found in pityriasis VESICULOBULLOUS (BLISTERING)


rosea- (Jharkhand 04) DISORDERS
a) HHV 7 b) CMV
c) Vericella Zoster d) EBV
206. Which of the following statements is true regarding 217. A patient is presenting with Non pruritic, Painless
Pityriasis rubra pilaris- (PGI OJ) erythema surrounded by vesicles with erythematous
a) Isolated patches of normal skin are found halo on face & hand. There is history of drug
b) Cephalocaudal distribution intake for fever 2 weeks back. The patient is
c) LV. cyclosporine is effective suffering from - (PGI 98)
d) More common in females a) Bullous pemphigoid
e) Oral cyclosporine is effective b) Erythema multiforme
207. Keratoderma is a feature of- (DELHI PG Mar. 09) c) Herpes zoster
a) Pityriasis rubra pilaris b) Pemphigus d) Dermatitis herpatiformis
c) Pityriasis rosea d) Psoriasis 218. A 28 year old patient has multiple grouped
208. Koebner's phenomenon is characteristic of- papulovesicular lesions on both elbows, knees
a) Psoriasis (PGI 99) buttocks and upper back associated with severe
b) Pemphigus vulgaris itching. The most likely diagnosis is-(AIIMS Nov 02)
c) Pityriasis rosea a) Pemphigus vulgaris b) Bullous pemphigoid
d) Lupus vulgaris c) Dermatitis herpetiformis d) Herpes Zoster
209. Koebner's phenomenon is seen in followin 219. A 30 year old male had severely ithcy papula-
except- (MH II) vesicular lesions on extremities, knees, elbows
a) Erythema b) Scleromyxedema and butocks for one year. Direct immunofluorescence
c) Molluscum contagiosum d) Darrier's disease staining of the lesions showed IgA deposition at
dermoepidermal junction. The most probable
diagnosis is - (AI 12, 04)
a) Pemphigus vulgaris b) Bullous pemphigoid
c) Dermatitis herpetiforms d) Nummular eczema
220. Extremely pruritic excoriation & papules on
buttocks with autoantibodies against epidermal
phenomenon is seen in - (PGI Dec 04) transglutaminase and IgA deposition in dermis on
a) Lichen planus b) Warts immunohistological examination of normal
c) Bechet syndrome d) Psoriasis perilesional skin. Diagnosis is - (SGPGI OJ)
e) Vitiligo a) Pemphigus vulgaris
212. Kohner phenomenon is present in all except- b) Pemphigoid
a) Psoriasis b) Lichen planus (Bihar 06) c) Linear IgA disease
c) Warts d) None d) Dermatitis herpetiformis
221. A 30-year-old male had severely itchy
papulovesicular lesions on extremities, knees, elbows
and buttocks for one year. Direct immunofluorescence
staining of the lesions showed lgA deposition at
214. "Isomorphic response" can be a feature of the dermo epidermal junction. The most probable
following except- (MH 03, 02)
diagnosis is - · (Manipal 09)
a) Warts b) Tinea a) Pemphigus vulgaris
c) Psoriasis d) Molluscum contagiosum b) Bullous pemphigoid
215. Pseudoisomorphic phenomenon is seen in - c) Dermatitis herpetiformis
d) Nummular eczema
a) Psoriasis b) Lichen planus (AI 11)
222. A 40 year male presenting with painful Oaccid
c) Vitiligo d) Plane warts
216. Pseudo koebener's phenomeneon is/are seen in- vesicles on face and oral mucosa with assymetric
a) Warts (PGINov.IO)
distribution. Immunoftuorescence shows
intercellular IgG deposition in epidermis. Diagnosis
b) Molluscum contagiosum
is- (AI 04)
c) Lichen planus
d) Psoriasis a) Erythema multiformis b) Bullous pemphigoid
c) Pemphigus vulgaris d) Herpes zoster

205)a 206) a,b 207) a 208) a 209)b 210)d 21l)a,b,d,e 212)d 213)a 214)b 215)d 216)a,b 217)b
218)c 219)c 220)d 221)c 222)c
SKIN [605]

223. A 23 years old lady presents with vesicular lesion 230. A 56 year old male lallu presents with painful
in the buccal mucosa & crusted lesion on the bullous lesion in lower extrimity, the most likely
skin, possible diagnosis is - (AI97) diagnosis is- (AIIMS 97)
a) Pemphigus Vulgaris a) Pemphigus vulgaris b) Bullous pemphigoid
b) Pemphigus foliaceous c) Necrotic pemphigus d) Contact eczema
c) Pemphigoid 231. A 40 year old male had multiple blisters over the
d) Dermatitis Herpitifonnis trunk & Extremities. Direct immuno fluoresce
224. A middle aged female has flaccid bullae in the studies showed linear IgG deposits along the
skin and oral erosion. Histopathology shows basement membrane, which of the following is the
intraepidermal acanthololytic blisters. The most most likely diagnosis- (AIIMS Nov 04)
likely diagnosis is - (PGI Dec 05) a) Pemphigus vulgaris
a) Pemphigus vulgaris b) Bullous pemphigoid
b) Paraneoplastic pemphigus c) Pemphigus foliaceous
c) Bullous pemphigoid d) Dermatitis herpetiformis
d) Dermatitis hepatifonnis 232. A patient with Bullous eruption on lower limb and
e) Epidermolysis bullosa acquista trunk, biopsy show epidermal bullae. The correct
225. A female presents with persistent painful oral diagnosis is - (AI 2000)
lesions, with acantholytic cells. Most likely a) Pemphigoid b) Pemphigus vulgaris
diagnosis is - (AI 08) c) Impetigo d) Internal malignancy
a) Pemphigus vulgaris 233. A 45 year old male has multiple grouped vesicular
b) Dermatitis herpetiformis lesions present on the T10 segment dermatome
c) Epidermolysis bullosa associated with pain. The most likely diagnosis
d) Bullous pemphigoid is - (AIIMS Nov 02)
226. A 50 year old male known case of myasthenia with a) Herpes zoster
erythemated shallow erosions with few blisters b) Dermatitis herpetiformis
and scales. Oral mucosa is not involved c) Herpes simplex
immunopathology demonstrates lgG deposition on d) Scabies
keratinocytes and auto antibodies against Dsg- 1. 234. Most common pemphigus is - (Jharkhand 06)
The diagnosis is - (PGI98) a) Pemphigus vulgaris
a) Pemphigus vulgaris · b) Pemphigus vegetans
b) Bullous pemphigoid c) Pemphigus foliaceus
c) Pemphigus foliaceus d) Pemphigus erythematosus
d) Dermatitis herpetifonnis 235. In pemphigus vulgaris, antibodies are present
227. A 24 years old female has flaccid bullae in the against - (PGI 2000, SGPGI 2000)
skin and oral erosions. Histopathology shows a) Basement membrane b) Intercellular substance
intraepidermal acantbolytic blister. The most likely c) Cell nucleus d) Keratin
diagnosis is - (AIIMS May 03) 236. Acantholysis is characteristic of- (AI 03)
a) Pemphigoid a) Pemphigus vulgaris
b) Erythema multiforme b) Pemphigoid
c) Pemphigus vulgaris c) Erythema multoforme
d) Dermatitis herpetiformis d) Dermatitis herpetifonnis
228. 25 years old female bas palatal ulcer and skin 237. A 50 years old man has a 2 year history offacial
blister most likely diagnosis is- (A/97) bullae & oral ulcers. Microscopic smear from
a) Pemphigus vulgaris skin lesions is most likely to disclose - (AI 96)
b) Pemphigus foliaceous a) Tzanck cells b) Acantholytic cells
c) Dermatitis herpetifonnis c) Necrosis d) Koilocytosis
d) Pemphigoid
229. A 85 yr old woman with Nikolsky sign-ve, blisters
on thigh & trunk, lesions come on & off. What is
the cause - (PGI 2000)
a) Pemphigus vulgaris 239. Acantholytic cells are - (SGPGI OJ)
b) Pemphigoid a) Epidermal cells b) Plasma cells
c) Lichen planus c) Keratinocytes d) Giant cells
d) Dermatitis herpetifonnis

223)a 224)a 225)a 226)c 227)c 228)a 229)b 230)b 23l)b 232)c 233)a 234)a 235)b 236)a
237) a,b 238) b 239)a,c
SKIN [606]

240-'/ ,Jztili).{ eeu;;:rs".:~ \;;;;':; t '·~

·· ,.~/:r,~::r;e< .. ;?~~·
250. Gluten free diet is beneficial in -
a) Psoriasis (Kerala 96, MH 07, 06, 05)
b) Exfoliative dermatitis
24t: A.cantllolytic 'cells iii pemphigus is - (PGI 96) c) Dermatitis herpetiformis
a) Cell with hyperchromatic nuclei and perinuclear d) Pemphigoid
halo 251. All are true about linear IgA disease except -
b) Cell with hypochromatic nuclei and perinuclear a) Subepidermal involvement (PGI OJ)
halo b) Severe itching
c) Multinucleated cells c) Granular deposition of lgA
d) Are candidates for gluten free diet
d) None of the above
e) A variant of dermatitis herpetiformis
242. Flaccid bullae lesions with oral mucosal lesions.
252. All causes erythema multiforme except- (AI 99)
The finding in immunotlourescence is -
a) HSV b) Drugs
a) Fish net lgG in epidermis (AIIMS Nov 09)
c) Idiopathic d) HPV
b) Linear IgG in DEJ 253. Erythema multiforme is most often related to -
c) Granular lgA in dermal papillae a) Herpes zoster infection (COMED 06)
d) Linear IgA in reticular dermis b) Herpes simplex infection
243. All are associated with pemphigus except - c) Molluscum contagiosum infection
a) Thymoma (SGPGI 02, PGI 98) d) Human papilloma virus infection
b)QL 254. Most common cause of erythema multiforme -
c) Myasthenia gravis a) Herpes simplex b) DM (MH 10)
d) Non-Hodgkins lymphoma c) Pityriasis rosae d) Erysipelas
e) Atrophic gastritis 255. Target or Iris lesion seen in -(AI 98, AIIMS 97)
244. True about pemphigus vulgaris- (PGI June 09) a) Urticaria b) Erythema multiforme
a) Subepidermal c) Scabies d) Lichen planus
b) Autoimmune disease 256. Regarding erythema multiforme all are true
c) Tzanck smear shows acantholytic cells except- (AI 2000)
d) Antibody are formed against desmogleins a) No vesicles
e) Blister on skin & mucosa b) Target lesions are seen
245. Spontaneous remission is most frequent with - c) Involves face and neck regions
a) Herpes labialis (SGPGI 04) d) Sign of internal malignancy
b) Herpes genitalis 257. A 60-year old patient presented with several bullous
c) Herpetic chancroid lesions for the last 3 days; each bulla was surrounded
by an erythematous halo. There were multiple target
d) Herpes gestationis
lesions. Patient also had oral erosions. The most
246. Drug induced pemphigus is most commonly caused
likely diagnosis is - (SGPGI 04)
by- (COMED 06)
a) Chicken pox b) Herpes simplex
a) Penicillamine b) Phenytoin sodium
c) Herpes zoster d) Steven- Johnzon syndrome
c) Carbamazepine d) Penicillin 258. All are true about erythema multiformis except -
247. Subepidermal bullae on trunk and flexural a) Due to herpes simplex (Jharkhand 05)
surface with linear C/IgG basement membrane b) Due to sulphonamide
deposite are feature of- (MH 08) c) Lesion are symmetrical
a) Bullous pemphigoid d) Mucous membrane is usually involved
b) Pemphigus vulgaris 259. Features of epidermolysis bullosa- (PGI June 06)
c) Linear Ig A disease a) t Adhesion fibrils
d) Dermatitis herpatiformis b) Present as miliary & scaring
248. Mucosa is never involved in wbich ofthe following c) IgG deposits in epidermo-dermal junction
disorders - (J & K 05) 260. In an 8 day old child with no history of consanguinity
a) Dermatitis herpetiformis in the parents, the mother reports blisters and
b) Pemphigus vulgaris bleeding off of the skin at the site of handling and
c) Epidermolysis bullosa acquisita pressure. There was a similar history in the previous
d) Toxic epidermal necrolysis child which proved to be fataL The diagnosis is -
249. One of the following is the t/fof choice for dermatitis a) Bullous pemphigoid (AIIMS May 01, AI 12)
herpetiformis- (AIIMS May 05) b) Congenital syphilis
a) Corticosteroids b) Dapsone c) Congenital epidermolysis bullosa
c) Mtx d) Retinoids d) Letterer - Swiwe disease

240)a 24l)a 242)a 243)e 244)b,c,d,e 245)d 246)a 247)a 248)a 249)b 250)c 251)c,d,e 252)d 253)b
254)a 255)b 256)a 257)d 258)d 259)a 260)c
SKIN [607]

261. Genetic predisposition is seen in which disease - 272. The following diseases are characterized by bullous
a) Lichen planus (PGI 97) eruptions EXCEPT- (UPSC-I 09)
b) Bullous pemphigoid a) Porphyria cutanea tarda
c) Pemphigus vulgaris b) Variegate porphyria
d) Epidermolysis Bullosa c) Dermatitis herpetiformis
262. Etiology of Epidermolysis bullosa is -(Rohtak 97) d) Lichen planus
a) Genetic b) Infections 273. Bullous skin lesions are seen in all except -
c) Senile d) Malignant a) Impetigo b) DM (MH 05)
e) Metabolic c) Porphyria d) Lupus vulgaris
263. In congenital dystrophic variety of epidermolysis 274. Sub-epidermal splitting is not found in- (PGI 99)
bullosa, mutation is seen in the gene conding a) Bullous pemphigoid
for- (AIIMS Nov 08, May 11) b) Pemphigus foliaceous
a) Laminin 4 b) Collagen type 7 c) Dermatitis herpetiformis
c) Alpha 6 integerin d) Keratin 14 d) Burns
264. Which of the following shows granular deposition 275. Subepidermal lesion are all except- (PGI June 09)
of lgA in dermal papilla - (AIIMS May 09, Nov. 11) a) Bullous pemphigoid b) Pemphigus vulgaris
a) Dermatitis herpetiformis c) Hailey- Hailey disease d) Darrier's disease
b) IgA papillomatosis of childhood e) Acanthosis nigricans
c) Bullous pemphigoid 276. Intraepidermal acantholytic blister is seen in -
d) Gestational herpes a) Pemphigus vulgaris (PGI June 06)
265. Skin disease not showing DIF (Direct b) Bullous pemphigoid
immunoftuorescence- (PGI Nov. 10) c) Paraneoplastic pemphigus
a) Darrier's disease d) Dermatitis herpetiformis
b) Hailey-Hailey disease e) Epidermolysis bullosa
c) Cicatricial pemphigoid 277. Intra-epidermal lesions seen in- (PGI June 08)
d) Dermatitis herpatiformis a) Pemphigus vulgaris
e) Pemphigus b) Epidermolysis bullosa acquisita
266. Direct immunoftuorescence is positive in-(PGI 02) c) Dermatitis herpetiformis
a) Atopic dermatitis b) SLE d) Bullous pemphigoid
c) Pemphigus d) Secondary syphilis e) Paraneoplastic pemphigus
267. Intraepidermal intercellular lgG deposition is 278. Intra epithelial bulla is seen in- (AI 91, AIIMS 95)
seen in - (AIIMS Nov. 11, May 09) a) Pemphigus b) Pemphigoid
a) Pemphigus b) Bullous pemphigoid c) Dermatitis herpetiformis d) Light eruption
c) Herpes genitalis d) Not recalled 279. Bullous skin lesions with 'intraepidermal split' is
268. A patient presented with multiple vesicles over the feature of- (MH 11)
body with fish net like IgG deposition in the a) Pemphigus vulgaris b) Pemphigoid
epidermis. Most likely diagnosis is -(PUNJAB 11) c) Epidermolysis bullosa d) All
a) Pemphigus vulgaris
b) Pemphigus folliaceous
c) Dermatitis herpetiformis
d) Bullous pemphigoid
269. Fishy net pattern is seen in - (Jipmer 11)
a) Pemphigus vulgaris Intra-epidermal are seen in -
b) Darier's disease a) Pemphigus vulgaris (PGI June 04)
c) Bullous pemphigoid b) Carcinomatous pemphigus
d) Dermatitis herpetiformis c) Dermatitis herpetiformis
270. All are vesiculo bullous lesions except-(Kerala 96) d) Congenital epidermolysis bullosa
a) Dermatitis herpetiformis b) Scabies e) Trauma
c) Pemphigus d) Pemphigoid 282. Intraepidermal blisters are seen in- (PGI Nov 09)
271. All of the following are bullous lesions except - a) Bullous pemphigoid
a) Pemphigus vulgaris (SGPGI 03) b) Pemphigus folliaceous
b) Dermatitis herpetiformis c) Dermatitis herpatiformis
c) Atopic dermatitis d) Bullous SLE
d) Pemphigoid e) Bullous impetigo

26l)d 262) a 263) b 264) a 265) a,b 266) c 267)a 268)a 269)a 270)b 27l)None>c 272)d 273)d
274)b 275) b,c,d,e 276) a,c,e 277) a,e 278) a 279)a 280)c 281) a,b 282) b,e
SKIN [608]

~-~;%?\
296. Acantholysis is seen in all of the following except-
a) Pemphigus (MP 06)
b) Bullous pemphigoid
c) Steven-Johnson syndrome
d) Toxic epidermal necrolysis
297. Nikolsky sign is positive in all except -
284. Intraepidermal bulla is seen in - (COMED a) Pemphigus (AIIMS 00, MP 06)
a) Pemphigoid b) Pemphigus b) Bullous pemphigoid
c) Dermatitis herpetiformis d) Light reaction c) Toxic epidermal Necrolysis
285. Which layer of skin causes vesicular change in d) Staphylococcal skin scaled syndrome
298. Nikolsky sign not present in - (Bihar 06)
case of burn - (AIIMS 97)
a) Pemphigus b) Pemphigoid
a) Basal layer b) Papillary layer
c) Vitiligo d) Staphylococcal scalded syndrome
c) Epidermis d) Dermis
299. Nikolsky's sign is positive in - (PGI 98)
286. Blister formation in bum case is in - (Bihar 06) a) Pemphigus b) Dermatitis herpatiformis
a) Intraepidermal b) Subepidermal c) Pemphigoid d) Rubella
c) Subdermal d) Subfascial
, .., 300. Hailey - hailey disease is - (Jharkhand 03)
a) Benign familial chronic pemphigus
b) Pemphigus acutus
c) Pemphigus
d) Lyell's syndrome

PHOTOSENSITIVE DISORDERS

301. 23 year old lady sony develops brown macular


lesions over bridge of nose and cheek following
in- (PGI97) exposure to light. The probable diagnosis is -
a) Chicken pox b) Psoriasis a) SLE b) Acne Rosacea (AIIMS 99)
c) Molluscum contagiosum d) Pemphigus vulgaris c) Chloasma d) Photodermatitis
290. A patient has Bullous Lesion; on Tzank smear - 302. A girl of 19 years with arthritis and photosensitive
a) Langerhans cells are seen (AI 96) rash on cheeks, likely diagnosis is -
b) Acontholysis a) SLE (AI 01, Bihar 03)
c) Leucocytosis b) Chlosma
d) Absence of melanin pigment c) Steven's Johnson syndrome
d) Lyme's disease
Z91i ·Ji~inj»r~i$1p9~ftive;iJl;*(NE1~J'!~~'ltat.tf!Wt'J,
303. A 40 year old woman presents with a 2 year old
' · ~· ,. "~'~,~~~w~;\S;~~~e1°~·~ ~;,~,: . ·~;if~}Jr:tli!~f~~~tJ~:·E~i1 H/o erythematous papulo pustular lesions on
convexities of the face. There in a background of
292. Acantholysis is seen in - (PGI June 09) erytherma & telengiectasia. The most likely
a) Bullous pemphigoid diagnosis is - (AI 05)
b) Dermatitis herpetiformis a) Acne vulgaris b) Acne rosacea
c) Hailey - Hailey disease c) SlE d) Polymorphic light eruption
d) Darrier's disease 304. 19 year old girl presents with ligh brown pigmentation
e) Pemphigus vulgaris over Malar eminence, likely diagnosis is- (AI OJ)
293. Acantholysis is seen in - (PGIMaylO) a) Chloasma b) SLE
a) SSSS b) Impetigo c) Melasma d) Melanoma
c) Hailey- Haiky disease d) Darrier's disease 305. A 45 year old fanner has itchy erythematous papular
e) Pemphigus vulgaris lesions on face, neck, 'V' area of chest, dorsum of
294. Acantholysis is seen in - (All India 95) hands and forearms for 3 years. The lesions are
a) Epidermis b) Dermis more severe in summers and improve by 75% in
c) Dermo-epidermjunction d) Subcutaneous tissue winters. The most appropriate test to diagnose the
295. Acantholysis is due to destruction of- (AIIMS 97) condition would be- (AI 06)
a) Epidermis a) Skin biopsy
b) Subepidermis b) Estimation of lgE levels in blood
c) Basement membrane c) Patch test
d) Intercellular substance d) Intradermal prick test

283)d 284)b 285)d 286)b 287)a 288)a 289)a 290)b 29l)a 292) c,d,e 293) a,c,d,e 294) a 295) d
296) b,d 297) b 298) b,c 299) a 300)a 30l)c 302)a 303)b 304)a 305)c
SKIN [609]

306. Exposure to sunlight can precipitate- (UPSC 2K) BACTERIAL INFECTIONS


a) Chloasma
b) Discoid lupus erytyhematosus 316. Leprosydoesnotinvolve- (PGI 98)
c) Dermatitis herpatiformis a) CNS b) Testis
c) Skin d) Cornea
317. Skin smear is negative in which leprosy- (AI 97)
a) Indeterminate b) Neuritic
c) Lepromatous d) Borderline
of the following are pbc>tm•en!!iti,re 318. Most common type ofleprosy in India- (PGI 97)
a) SIE b) Liver spots a) BT b)TT
c) Calcinosis cutis d) Morphea c) IL d)BL
e) Prophyria cutanea tarda 319. Virchow's cells are seen in- (UP 95)
309. Photosensitivity is a feature of- a) Henoch scholein pw:pura
a) Porphyria cutanea tarda (DELHI PG Mar. 09) b) Toxic Epidermal necrolysis
b) Psoriasis c) Congenital syphilis
c) Pemphigoid d) Leprosy
d) Pompholyx 320. Lepra cell is a- (Maharashtra 0 J)
a) Plasma cell b) Neutrophil
c) Lymphocyte d) Histiocyte
321. Satellite lesion are seen in- (AI 99)
311. Rhinopyma is (Potate nose)- (AP 96) a) Tuberculoid leprosy
a) Septal deviation of nose b) Lepromatous leprosy
b) Sweat gland hypertrophy c) Borderline tuberculoid leprosy
c) Mucous gland hypertrophy d) Histoid leprosy
d) Sebaceous gland hypertrophy 322. Inverted saucer shaped lesion is found in-
312. A 40 year old farmer with a history of recurrent a) Lepromatous leprosy (AIIMSMay J2, 95)
attacks of porphyria complains of itching when b) Tuberculoid leprosy
exposes to the sun and maculopapular rash on sun c) Borderline leprosy
exposed areas, his symptoms are exaggerated in d) Indeterminate leprosy
the summer. The diagnosis is - (AIIMS May OJ) 323. In leprosy nerves commonly involved- (PGI 97)
a) Seborrheic dermatitis a) High ulnar, low median
b) Contact dermatitis b) High median, low ulnar
c) Psoriasis c) Triple nerve palsy
d) Porphyria cutanea tarda d) High radial, low median
313. Chandu 32 years male presents with abdominal 324. Skin biopsy in leprosy is characterized by- (AI 97)
pain and vomitting. He also complain of some a) Periappendgeal bacilli
psychiatric symptoms & visual hallucination. Most b) Periappendgeal lymphocytosis
likely diagnosis is - (AI OJ) c) Perivascular lymphocytosis
a) Hypothyroidism b) Hyperthyroidism d) All of the above
c) Hysteria d) Intermittent porphyria 325. All are true lepromatous leprosy except- (TN 96)
314. A girl on sulphonamides developed abdominal pain a) Presence ofglobi
and presented to emergency with seizure. What b) Subepidermal free zone
is the probable cause ? (AIIMS Nov 08) c) Decreased cell mediated immunity
a) Acute intermittent porphyria d) Presence of granulomas subdermally
b) Congenital erythropoietic porphyria 3 26. A 45 year old male had multiple hypoaesthetic mildly
c) Infectious mononucleosis erythematous large plaques with elevated margins
d) Kawasaki's disease on trunk and extremities. His ulnar and lateral
315. Porphyrins are synthesized mainly in -(AIIMS 95) popliteal nerves on both sides were enlarged. The
a) Spleen most probable diagnosis is- (AIIMS Nov 03)
b) Liver and spleen a) Lepromatous leprosy
c) Bone marrow and spleen b) Borderline leprosy
d) Liver and bone marrow c) Borderline tuberculoid leprosy
d) Borderline lepromatous leprosy

306) a,b 307) d 308)a,b,e 309) a 310) b 311) d 312)d 313)d 314)a 315)d 316)a 317)b 318)b 319)d
320)d 32l)c 322) c 323) a 324) d 325) d 326)d
SKIN [ 610]

327. Multiple hypoaesthetic, hypopigmented macules on 339. The following test is not used for diagnosis of
right lateral forearm with numerous acid fast ba- leprosy- (AIIMS 06)
cilli is indicative of- (AIIMSMay 12) a) Lepromin test
a) Tuberculoid leprosy b) Slit skin smear
b) Lepromatous leprosy c) Fine needle aspiration cytology
c) Indeterminate leprosy d) Skin biopsy
d) Borderline leprosy 340. A 16 year old student reported for the evaluation of
, 328. In leprosy which ofthe following is not seen- multiple hypopigmented macules on the trunk and
a) Abnormal EMG (PGI 2000) limbs. AU of the following tests are useful in making
b) Voluntary muscle wasting
a diagnosis ofleprosy, except- (AJIMSNov 03)
c) Decreased proprioception.
a) Sensation testing b) Lepromin test
d) Decreased response to tactile sensation
c) Slit smears d) Skin biopsy
e) Increased response to tactile sensation
341. ENL is seen in which form ofleprosy- (PGI Dec 05)
329. Earlist sensation to he lost in Hansens disease is-
a) Pain b) Touch (UP 98) a) Indeterminate b)BT
c) Vibration d) Temperature c) IL d)BL
330. A single hypopigmented anesthetic patch with e)TI
satelite lesion on forearm, likely diagnosis 342. The main cytokine, involved in erythema nodosum
is· (AIIMS 94) leprosum (ENL) reaction is- (AIIMS May 06)
a) Indeterminate leprosy b) Tuberculoid leprosy a) Interleukin- 2
c) Neuritic leprosy d) Lupus vulgaris b) Interferon- gamma
331. All lesions are seen in leprosy except- (AI 97) c) Tumor necrosis factor- alpha
a) Erythematous Macule d) Macrophage colony stimulating factor
b) Hypopigmented patch 343. Manifestation of ENL include all of the following
c) Vesicles except- (PGI Dec 05)
d) Flat & raised patches a) Pancreatitis b) Fever
332. All are features of lepromatous leprosy except- c) Hepatitis d)Arthritis
a) Gynaecomastia (TN 96, AIIMS 94) e) Cutaneous nodules
b) Madarosis 344. Type-I lepra reaction is commonly seen in :
c) Saddle nose a) Histoid leprosy (DELHI PG Mar. 09)
d) Perforating Ulcer b) Lepromatous leprosy
333. Symmetrical multiple lesions are seen in which c) Borderline tuberculoid leprosy
type ofleprosy- (Maharashtra OJ) d) Polyneuritic leprosy
a) Borderline b) Neuritic
345. The following drug is used for the treatment of type
c) Lepromatous c) Tubercular
ll lepra reaction, except- (AIIMS 06)
334. Which of the following is an indication of active
a) Chloroquine b) Thaliodomide
leprosy- (CUPGEE 96)
c) Cyclosporin d) Corticosteroid
a) New skin lesions b) Nerve tenderness
c) Erythema d) All 346. DOC for lepra ll reaction is- (AI 08)
335. In leprosy neural involvement occurs in what a) Steroids b) Thalidomide
percentage of patients- (Orissa 99) c) Clofazimine d) Dapsone
a) 30% b)60% 347. Drug of choice for lepra ll reaction is- (DPGEE 08)
c) 90% d) 100% a) Steroids b) Thalidomide
336. Iris pearl is seen in- (Calcutta 00) c) Clofazimine d) Dapsone
a) Leprosy b) Sarcoidosis 348. Thalidomide is drug of choice for- (AI 99)
c) Tuberculosis d) Chloasma a) Lepra I reaction b) Lepra II reaction
337. About tuberculoid leprosy, false is- (Jharkhand 06) c) Both d) Nerve abscess
a) CMI is depressed 349. DOC in type I lepra reaction with severe neuritis-
b) CMI is increased a) Thalidomide b)Clafazamide (AJ95)
c) A single anesthetic patch c) Dapsone d) Systemic corticosteroid
d) None 350. Antileprotic drug also used in lepra reaction is-
338. Lepromin test is positive in which leprosy- a) Rifampicin b) Dapsone (AIIMS 97)
a) Lepromatous (AIIMS 2000, MH 10) c) Ciprofloxacin d) Clofazimine
b) Indeterminate
c) Histoid
d) Tuberculoid r0~:::7i~1~·r~~,~~~
327)d 328) c,e 329) d 330) None >b 331) c 332) None 333) c 334)All 335) d 336)a 337)b 338)d 339)a
340)b 34l)c,d 342)c 343)a 344)c 345)c 346)a 347)a 348)b 349)d 350)d
SKIN [ 611]

352. Duration of treatment in multibacillary leprosy 362. According to WHO latest treatment ofleprosy is-
according to WHO is- (PGI June 02) a) 2 yrs. Or negative slit smear negative which ever
a) 6 months b) 1 year is longer for multi bacillary (PGI 96)
c) 2 years d) 5 years b) Life long treatment
e) Lifelong c) Six months treatment for paucibacillary
353. Under leprosy eradication programme the d) Not specified
managment of single lesion is- (AJIMS 02; PGI June 363. All of the following are true of tuberculoid leprosy
a) Single dose ofRifampicin and Dapsone 00) except- (Karnat 96)
b) Rifampicin and Dapsone for 6 months a) It is infective b) Positive lepromin test
c) Rifampicin, ofloxacin and minocycline single dose c) Treatment for 5 years d) Body immunity is high
d) Rifampicin and rninocycline for 6 months 364. One ofthe following is a side effect of clofazimine
354. In multibacillary leprosy the follow up examination used in leprosy therapy- (AI 96)
after adequate treatment should be done yearly for- a) Hyperpigmentation
a) 3 years b) 3 years (Delhi 96) b) Erythema
c) 5 years d) 10 years c) Discoloration ofbody secretions
355. A patient with leprosy had slightly erythematous, d) Macular rash
anesthetic plaques on the trunk and upper limbs. He 365. Ifa claw hand develops in a patient with Leprosy, the
was treated with paucibacillary multidrug therapy deformityis- (All2)
(PB-:MD1) for 6 months. At the end of 6 months, he a) GradeO b) Gradel
had persistent erythema and induration in the c) Grade Il d) Grade ill
plaque. The next step of action recommended by 366. M.C. type of cutaneous T.B. is- (PGI Dec 06)
the World Health Organization (WHO) in such a a) Lupus vulgaris b) Scrofuloderma
patient is- (AIIMS Nov 04) c) T.B. verruca cutis d) Erythemainduratum
a) Stop antileprosy treatment 367. Most common form of cutaneous tuberculosis in
India- (Maharashtra 09)
b) Continue PB-MDT till erythema subsides
a) Scrofuloderma
c) Biopsy the lesion to document activity
b) Lupus vulgaris
d) Continue dapsone
c) Primary cutaneous tuberculosis
356. Treatment of severe ulnar neuritis in borderline d) Tuberculosis verucous cutis
tuberculoid leprosy is- (AI 07) 368. Tuberculosis of skin is called as- (AJIMS 98)
a) MDT only b) MDT + steroid a) Lupus vulgaris b) Lupus pernio
c) Wait and watch d) MDT+ thalidomide c) Lupus profundus d) Scrofuloderma
357. Best method of treatment ofulner never abscess in 369. Skin manifestation ofT.B.- (PGI 04)
case ofleprosy is- (AJIMS 96, MAHE 05) a) Lupus vulgaris b) Lupus pernio
a) High does of steroid c) Scrofuloderma d) Butcher warts
b) Incision and drainage 370. Forms ofskin tuberculosis include all except-
c) Thalidomide a) Lupus vulgaris (Maharashtra 06)
d) High does of clofazamine b) Scrofuloderma
358. Treatment of acute neuritis in lepra I reaction is c) Erythema nodosum
AlE- (AI 95) d) Erythema annulare
a) Dapsone b) Steroid 371. True about lupus vulgaris- (PGI Dec 04)
c) Thalidomide d) Incision and drainage a) Apple jelly nodule at root of nose
359. Reversal lepra reaction shown no response to- b) TB of skin & mucosa
a) Cloafazirnine b) Dapsone (Jipmer 95) c) Also known as scrofuloderma
c) Glucocorticoids d) Thalidomide . d) ATT is helpful
360. The most effective drug against M.leprae is- 372. Lupus vulgaris is- (SGPGI 04)
a) Dapsone b)Rifampicin (AI 03) a) Tuberculosis of skin b) Basal cell CA
c) Clofazamine d) Prothionamide c) Squamous cell CA d) Fox bite
373. Apple-jelly nodules is/are seen in- (PGI Nov. 10)
361. WHO regime for paucibacillary leprosy- (AJ96)
a) Lupus vulgaris b)DLE
a) 100 mg Dapsone daily + Rifampcin monthly
c) Lichen planus d) Psoriasis
(600mg)
374. Skin TB which involves skin after involving lymph
b) Dapsone daily+ Rifampcin daily nodes- (AIIMS 96)
c) Dapsone+ Rifarnpcin + Clofazimine daily a) Scrofuloderma b) Lupus vulgaris
d) Rifampcin + Clofazimine daily c) Lupus erythematous d) Lupus pernio

35l)b 352)b 353)c 354) c 355) a 356) b 357)b 358)c 359)d 360)b 36l)a 362)c 363)c 364)a,c
365)c 366)a 367)b 368) a,d 369) a,c 370) d 37l)a,b,d 372)a 373)a 374)a
SKIN [ 612]

375. Cutaneous tuberculosis secondary to underlying 385. Mycobacterium causeing skin ulcer- (PGI 02)
tissue is called as - (AI 99) a) M. smegmatis b) M. scrofulaceum
a) Lupus vulgaris b) Scrofuloderma c) M. ulcerans d) M. fortuitum
c) Spina ventosa d) Tuberculous verrucosa cutis e) M. marinum
376. Scrofulodermais- (Maharashtra 05) 3 86. Epitheliod granuloma is characterstic of-(AIIMS 95)
a) Chronic infection by staphylococcus epidermidis a) Sarcoidosis b) Eosinophilia
b) Tubercular sinus with a discharge c) T.B. d) Mycosis fungoides
c) Rupture of a lymph node with involvenent of skin 387. Thberculosis verrucosa cutis is a form of-
d) Lesion in lupus vulgaris a) Tuberculid (SGPGI 05)
377. Thberculides are seen in- (AHMSMay 07, Nov 06) b) Primary tuberculosis
a) Lupus vulgaris b) Scrofuloderma c) Postprimary tuberculosis with good resistance
c) Lichen scrofulosorum d) Erythema nodosum d) Post primary tuberculosis with poor resistance
378. Which ofthe following is/are tuberculides- 388. A Farmer has a single warty lesion on leg. Which
of the following could be most likely lesion -
a) Lichen scrofulosorum (PGI 2000, June 07)
a) Verruca vulgaris (AIIMS Nov J 0)
b) Lichen nichidus
b) Tuberculosis verrucosa cutis
c) Lichen aureus
c) Mycetoma
d) Erythema nodosum d) Lichen planus hypertrophicus
3 79. Which of the following tuberculids is characterized 389. Which ofthe following are bacterial infections of
by involvement of sweat glands, and hair follicles skin - (PGI June 05)
with noncaseating epitheloid granuloma- (AI 09) a) Pyoderma gangrenosum b) Piedra
a) Lichen scrofulosorum b) Miliary tuberculosis c) Impetigo contagiosa d)Jmpetigoherpetiformis
c) Papulonecrotic type d) Lupus vulgaris e) Ecthyma
380. A 12 year old hoy had a gradually progressive plaque 390. Falsestatementaboutimpetigo- (PGI OJ)
on a buttock for the last 3 years. The plaque was 15 a) Mostly caused by staphylococcus or
em in diameter, annular in shape, with crusting and streptococcus or both
induration at the periphery and scarring at the b) It predisposes to glomerulonephritis
center. The most likely diagnosis -(AIIMS Nov 03) c) Produces scar on healing
a) Tinea corporis b) Granuloma annulare d) Erythromycin is drug of choice
c) Lupus vulgaris d) Borderline leprosy e) It is infectious lesion
381. A 36 years old factory worker developed itchy 391. True about Impetigo is- (PGI June 08)
annular scaly plaques in both groins. Application a) Contagious b) Bacterial infection
of a carticosteroid ointment led to temporary relief c) Non contagious d) Honey coloured crust
but the plaques continued to extend at the periphery. e) VIral infection
The most likely diagnosis is- (AI 05) 392. Staphylococcus causes AlE- (PGI 99)
a) Erythema annulare centrifugam a) Scarlet fever b) TSS
b) Granuloma annulare c) Carbuncle d) Sycosis barbae
c) Annular lichen planus 393. Staphylococcus can cause- (PGI June OJ)
d) Tenia cruris a) Ecthyma b) Erythrasma
382. 20 year old male from Jaipur with erythermatous c) Furuncle d) Impetigo contagiosa
lesion on cheek with central crusting likely e) Sycosis barbae
diagnosis is- (AI OJ)
394. Staphylococcal infection causes all disease except-
a) Impetigo b) Erysipelas (AI 97)
a) SI.E b) Lupus vulgaris
c) Ecthyma d) Scaldy skin syndrome
c) Chillblain d) Cutaneous leishmaniasis
395. Impetigo contageosa most commonly due to-(JH 04)
383. Ayoungboypresentedwithlesionoverhisrightbuttock a) Group B streptococcous
which had peripheral scaling and central scarring. b) Staphalococous
The investigation of choice would be-(AIIMS Nov OJ) c) Moniliasis
a) Tzank Smear b) KOH preparation d) Streptococcus viridans
c) Biopsy d) Saboraud's agar 396. A 3 months old male infant developed otitis media
384. An 8 year old boy present with well defined annular for which he was given a course of Co-trimoxazole.
lesion over the buttock with central scarring that is A few days later, he developed extensive peeling of
gradually progressive over the last 8'months. The the skin; there were no mucosal lesions and the
diagnosis is- (AIIMS OJ) baby was not toxic. The most likely diagnosis is-
a) Annular psoriasis a) Toxic epidermal necrolysis (AIIMS 04)
b) Lupus vulgaris b) Staphylococcal scalded skin syndrome
c) Tinea corporis c) Steven Johnsom syndrome
d) Chronic granulomatous disease d) Infantile pemphigus

375)b 376)c 377)c 378)a,d 379)a 380)c 38l)d 382)d 383)c 384)b 385)c,d,e 386)a,c 387)c
388)b 389) c,e 390) c 391) a,b,d 392) a 393) a,c,d,e 394) b 395) b 396) b
SKIN [ 613]

397. Following is a bacterial infection- (DPG March 09) 411. Erythrasma is caused by- (MH 01)
a) Pyoderma gangrenosum a) Corynebacterium b) Staph
b) Impetigo herpitiformis c) Streptococci d) Viruses
c) Pitted keratolysis 412. Corynebacterium minutissimum causes- (MH 02)
d) Mycosis fungoides a) Erysipelas b) Erythrasma
398. Skinhazardsofswimmingare- (PGIOJ) c) Ecthyma d) Impetigo
a) Verrucae
b) Pyoderma gangrenosum FUNGAL INFECTION, SCABIES,
c) M. marinum gangrenosum PEDICULOSIS
d) M. ulcerans infection 413. Which of the following are fungal infection of
399. Which of these statements is false for erythema skin - (PGI June 05)
marginatum- (MP 04) a) Sporotrichosis b) Molluscum contagiousm
a) Lesions are serpiginous c) Madura foot d) Tinea
b) Characteristically it is an evanescent e) Erysipelas
c) Rash worsens on application of heart 414. Stain used for staining fungal elements- (AIIMS Nov
d) Rash is itchy a) Acid fast stain b)Mucicarmic 09)
400. Erysipeloid is transmitted by- (PGI 99) c) Methenamine d) Gram stain
a) Droplet b) Feco-oral 415. Which dye is most suitable for fungus demonstration
c) Mosquito bite d) Contact with animal in biopsy- (AIIMS Nov 06)
401. Herpes zoster is commonly seen in-(DEUIIPG Feb. a) Alizarin red b) Veirhoff dye
a) Cervical region b) Thoracic region 09) c) Mason's trichome d) PAS
c) Lumber region d) Geniculate ganglion 416. Which of the following stains is used to study
402. Shingles is the other name for- (COMED 06) fungal morphology in tissue sections- (All 0)
a) Herpex simplex b) Varicella a) PAS b) Von- kossa
c) Herpes Zoster d) Herpes labialis c) Alzarin- red d) Masson's Trichrome
403. Jarish herxheimer reaction is seen in early cases 417. Dermatophyte infection is- (Bihar 06)
of- (PG/98) a) Superficial b) Subdermal
c) Subfascial d) Muscular
a) Syphilis b) Gonorrhoea
418. Ring worn infection affects- (UP 97)
c) I.GV d) Granuloma inguinale
a) Dermis b) Papillary layer
404. Jarisch Herxheimer reaction is commonly seen in-
c) Stratum corneum d) Prickle cell layer
a) Early syphilis (SGPGI 05)
419. T. capitis (endothrix) is caused by- (PGI Dec 2000)
b) Late congenital syphilis
a) Epidermophyton b) T.tonsurans
c) Latent syphilis
c) T.violaceum d) Microsporum
d) Syphilis of cardiovascular system
e) T.rubrum
405. Eczema herpeticum is caused by- (PGI June 07, 420. Which does not cause Tinea capitis-
a) Herpes simplex virus b) Varicella PGI 2K) a) Epidermophyton (Jipmer 97, AIIMS 94, 92)
c) CMV d)HPV b) Microsporum
e)HSV-6 c) Trichophyton rubrum
406. Kaposivaricelliformlesionseenin-(PG/04,PG/ Dec05) d) Trichophyton violaceum
a) Atopic dermatitis b) Darriers disease 421. Kerion is seen in- (AIIMS 98)
c) Lichen planus d) Varicella zoster a) Candida infection b) Trichomoniasis
407. DOC for herpes zoster is- (AIIMS 98) c) Pityriasis d) Dermatophytosis
a) Vidarabine b) Acyclovir 422. Most common organism causing T. capitis is-(AJ 01)
c) Idoxuridine d)Aclidine a) Trichophyton tonsurans b) Microsporum
408. Mter 3 days of fever patient developed maculo c) Epidermophyton d) Candida albicans
erythematous rash lasting for 48 hrs diagnosis is- 423. An 8 yr old boy presents with boggy swelling and
a) Fifth disease b) Rubella (A/02) easily pluckable hair, Diagnosis is- (AIIMS 98)
c) Measles d) Roseola infantum a) Tinea capitis b) Alopecia areata
409. Recurrent balanoposthitis seen in- (PGI 02) c) Tuberculorid leprosy d) Pityriasis alba
a) DM b) Herpes simplex 424. A 7 year old boy with boggy swelling of the scalp
c) Smoking d)Alcohol with multiple discharging sinuses with cervical
e) Bad hygiene lymphadenopathy with easily pluckable hair. What
410. Discharging sinus is seen in- (AI 96) would be done for diagnosis- (AIIMS Nov 09, 96, 94;
a) Syphilis b) Herpes a) Pus for culture b)KOHmount A/01)
c) Actinomycosis d) Molluscum contagiosum c) Biopsy d) Patch test

397)c 398)a,c 399)d 400)d 401)b 402)c 403)a 404)a 405) a 406) a,b 407) b 408) d 409)d 410)c
41l)a 412)b 413)a,c,d 414)c 415)d 416)a 417)a 418)c 419)b,c 420)a 421)d 422)b 423)a 424)b
SKIN [614]

425. The test likely to help in diagnosis of a patient 436. DOCforTineaUnguim- (UP 95)
who presents with an itchy annular plaque on the a) Ampthotericin B b) Miconazole
face is- (AI 03) c) Grisoefulvin d) Nystatin
a) Gram's stain b) Potassium hydroxide mount 437. Griseofulvin is used in- (Bihar 04)
c) Tissue smear d) Wood's lamp examination a) Tinea captis b) P. versicolor
426•. :$e8f1Uig8Wstlt testt'orfungat~IGJ[.tl{f~~timi·· c) Candidiasis d)ALL
a) ... 1 .i, .:r,i/iiJ!1.!f'IDNli 438. Grisefulvin given for the treatment of fungal
,y ';ci,'lwi~01:~!h,, J·~· ..... ··.i infection in the fmger nail dermatophytosis for how
C:J .!:i.t" .· : ~· ;:. ;";['11f.terrl/
427. A 60-year old male presented with discolouration, muchduration- (AI02)
thickening and tunneling of2 fingernails and one a) 4 weeks b) 6 weeks
c) 2 months d) 3 months
toe nail. Which ofthe following will clinch the diag-
439. Dermatophytes are-(PGI 03, SGPGI 04, PGI June 06)
nosis at the earliest? (AIIMS May 12)
a) Sporothrix b) Tinea versicularis
a) Wood's lamp examination b) KOH mount
c) Microsporidium d) Trichophyton rubrum
c) Slit smear (split skin smear) d) Gram stain
e) All of the above
428. Tinea unguium effects- (AI 95) 440. A 24 year old man had multiple, smallhypopigmented
a) Nail fold b) Nail plate macules on the upper chest and back for the last
c) Joints d) Inter digital space three months. The macules were circular, arranged
429. Tinea incognito is seen with- (Jipmer 02, PGI 99) around follicles and many had coalesced to form
a) Steroid treatment b) I% BHi3 large sheets. The surface of the macules showed
c) 5% permethrin d) Antibiotics fine scaling. He had similar lesions one year ago
430. A 22 years old male patient presents with a which subsided with treatment The most appropriate
complaints of severe itching and white scaly lesions investigation to confirm the diagnosis is-
in the groin for past month. Which of the following a) Potassium hydroxide preparation of scales
ismostlikelytobethecausativeagent-(AJIMSNov03) b) Slit skin smear from discrete macules
a) Trichophyton rubrum b) Candida albicans c) Tzanck test (AIIMS May 12, Nov 03)
c) Candida glabrata d) Malassezia furfur d) Skin biopsy of coalesced macules
431. A 36- year- old factory worker developed itchy, 441. The following drug is effective in treatment of
annular scaly plaques in both groins. Application of ptyriasis versicolor- (AIIMS May 03)
a corticosteroid ointment led to temporary relief but a) Ketoconazole b) Metronidazole
the plaques continued to extend at the periphery. c) Griseofulvin d) Chloroquine
The most likely diagnosis is - (AI 05) 442. All the following drugs are effective in t/t ofptyriasis
a) Erythema annulare centrifugum versicolor except- (Bihar 05, AI 05)
b) Granuloma annulare a) Selenium sulphide b) Ketoconazole
c) Grieseofulvin d) Clotrimazole
c) Annular lichen planus
443. Commonest fungal infection of the female genitalia
d) Tinea cruris
in diabets- (Kamat 98)
432. A 30 yr old female presents with history ofitching
a) Cryptococcal b) Madura mycosis
under right breast. On examination annular c) Candidial d) Aspergellosis
ring lesion was present under the breast. The 444. A washer man presents with thickness erosion &
diagnosis is- (AIIMS May 02) discoloration ofweb spaces oftoes diagnosis is-(PGI96)
a) Trichophyton rubrum b) Candida albicans a) Psoriasis b) Tinea Unguium
c) Epidermophyton d) Microsporum c) Both d) Candidiasis
433. Treatment of tinea unguium- (PG!June 05) 445. An otherwise healthy male presents with a creamy
a) Fluticasone b) Itraconazole curd like white patch on the tongue. The probable
c) Oleamine oil d) Turbinafm diagnosis is- (AI 10)
e) Neomycin a) Candidiasis b) Histoplasmosis
434. Ciclipirox oleamine is used in- (AP 96) c) Lichen planus d) Aspergillosis
a) Dermatophytosis b) Acne 446. Linear lesion is seen in- (PGI97)
c) Psoriasis d) Lichen planus a) Sporotrichosis b) Lichen planus
435. An eleven year old boy is having tinea capitis on his c) Psoriasis d) Pemphigus
scalp. The most appropriate line of treatment is - 44 7. Potassium iodide is useful in the treatment of:
a) Oral griseofulvin therapy (AI 03) a) Sporotrichosis (DELHI PG Mar. 09)
b) Topical griseofulvin therapy b) Impetigo
c) Shaving of the scalp c) Vrral warts
d) Selenium sulphide shampoo d) Dermatitis herpetiformis

425)b 426)a 427)b 428)b 429)a 430)a 431)d 432)a 433) b,d 434) a 435)a 436)c 437)a
438)None>d 439)d 440)a 44l)a 442)c 443)c 444)d 445)a 446)a 447)a
SKIN [ 615]

448. All the following antimicrobial agents are used 460. An infant presenting with itchy lesions over groin
topically except- (AI 95) and prepuce, all is indicated except- (AI 01)
a) Clotrimazole b) Griseofulvin a) Bath & apply scabicidal solution
c) Nystatin d) Miconazole b) Treatment of all family members
449. The Burrows in scabies is in the- (Karn. 98) c) Dispose all clothes by burning
a) Stratum basale b) Stratum granulosum d) IV antibiotics
c) Stratum corneum d) Dennis 461. lvermectin iu indicated in the treatment of-
4~~,.,~ i.·xl»l.i.n
CA./
a. •.·~.•·.t."'.··.~·.·s···li~Je~J9.~?9l's~~j~~;~:,Ji#~l!J:W{~~
.•.• .·•.·.f...:.....
,t,.p~Y-L-J;,W_Y-t:;:~_: -
e
;_:;:;:,~-, <---~-i--tL·<-->:f~-"~-
a) Syphilis
c) Tuberculosis
b) Scabies (AIIMS May 06)
d) Dermatophytosis
i:\--f:i:N:~--'BRl:~~J~;?)~:-:0{~--f~:--~#.J\s::tf~~J;~u~~~:-:_;f~:-i~-Y 462. Oral medication used in the treatment of scabies-
451. Incubation period of scabies is- (Calcutta 2K) a) Albendazole b) Metronidazole (Karn 11)
a) 7 days b) 2 weeks c) Ketoconazole d) lvermectin
c) 4 weeks d) 2-3 days 463. A 6 month old infant had itchy erythematous papules
452. Adult scabies is characterized by- (PGI Dec 04) and exudative lesions on the scalp, face, groins and
a) IiJ.Volve palm & soles b) Involve face axillae for one month. She also had vesicular lesions
c) Involve anterior abdomen d) Involve web space on the palms. The most likely diagnosis is- (AI 06)
e) Involve genitalia a) Congenital syphilis b) Sebonhoeicderrnatitis
453. Scabies in children differs from that in adults in c) Scabies d) Psoriasis
that it affects- (Jipmer 2K) 464. A 6 month old infant presented with multiple papules
a) Webspace b)Face & exudative lesions on the face, scalp, trunk and few
c) Genitalia d) Axilla vesicles on palms and soles for 2 weeks. His mother
454. Scabies in adults differs from that in children by- has H/0 itchy lesions. The most likely diagnosis
a) Not involving face (MAHE 07, Manipal 08) is- (AIIMSMay 12, 05)
b) Non involving genitals a) Scabies
c) Not involves areola b) Infantile exzema
d) Involves whole body c) Infantile sebornheic dermatitis
455. Circle of he bra is associated with- (PGI June 08) d) Impetigo contagiosa
a) Syphilis b) Scabies 465. Papulovesicular lesion on face, trunk, palm and sole
c) Leprosy d) Lichen planus in a 9 month old child is seen iu-(AIIMS May 12, 95)
a) Scabies

~J'&~;a~r
b) Drug reaction
c) Atopic dermatitis
d) Seborrheic dermatitis
is c)~FanulN;1ifrstbry fo.ul1d ' ~·::,~;i·c:;t(., . 466. A child has itchy vesicles on face, palm and sole the
'·.i~Jc0z·,· .d)f~v~J,ri~·~ ~~11i!l'fll~ttf~~~g;~~~·l':i£t~ ;?:x~,, . . . 4~Wf' diagnosis is - (AIIMS 96)
457:· NodUiarscabie!JisfoundiD~ ~.. · ·· (AilMS97}
a) Scabies b) Seborrhic dermatitis
a) Web space offmger b)Axilla c) Chicken pox d) Atopic dermatitis
c) Abdomen d) Scrotum 467. A child has multiple itchy papular lesions on the
458. An 8 month old child presented with itchy, exudative genitalia and fingers. Similar lesions are also seen
lesions on the face, palms and soles. The siblings in the younger brother. Which of the following is
also have similar complaints. The treatment of most possible diagnosis? (AIIMS Nov 02)
choice is such a patient is - (AI 03) a) Papularurticaria b) Scabies
a) Systemic ampicillin c) Atopic dermatitis d)Ailergiccontactdennatitis
b) Topical betamethasone 468. Scabies, an infection of the skin caused by Sarcoptes
c) Systemic prednisolone scabies, is an example of- (AIIMS Nov 02)
d) Topical permethrin a) Water borne disease b) Water washed disease
459. An infant presented with itchy eczematous crusted c) Water based disease d) Water related disease
lesions with exudation on palm, sole, glans penis 469. Vagabond's disease is- (Calcutta 2K, Bihar 04)
and face. All are true except- (PGI 02) a) Pediculosis corporis b) Scabies
a) Family should be examined c) Eczema d)Ringworm
b) Only patient needs drug treatment 470. Macula cerulea is seen in- (Orissa 99, Bihar 06)
c) All clothing & linen should be discarded or burnt a) Pediculosis humanis corporis
d) Distribution oflesion help in diagnosis making b) Pediculosis capitis
c) Pthiris pubis
e) Drug should be applied to affected areas for
d) Lupus erythematosis
whole day

448)b 449)c 450)a 451)c 452)c,d,e 453)b 454)a 455)b 456)d 457)d 458)d 459)b,c,e 460)c 46l)b
462)d 463)c 464)a 465)a 466) a 467) b 468) b 469) a 470) a,c
SKIN [ 616]

471. TOC for pediculosis corporis is- (AIIMS 98) 480. A 24 year old male presents to a STD clinic with
a) 3 application ofBHC a single painless ulcer on external genitalia. The
b) 4 application ofBHC choice of laboratory test to look for the etiological
c) Disinfection of all clothes and beddings agent would be- (AIIMS May 03)
d) DDT application a) Scrappings from ulcer for culture on chocolate
472. Permethrin is used in treatment of- (AI99) agar with antibiotic supplement
a) Scabies b) Leprosy b) Serology from detection of specific IgM antibodies
c) Body Louse d) Leishmaniaris
c) Scrappings from ulcer for dark field microscopy
d) Scrappings from ulcer for tissue culture
SEXUALLY TRANSMITTED DISEASES
481. Painful ulcers over genitalia are caused by-(MH 03)
473. A 23 year old male had unprotected sexual a) Primary syphilis b) Granuloma inguinale
intercourse with a commercial sex worker. Two c) Chancroid d) LGV
weeks later, he developed a painless, indurated 482. Tenderness is uncommon in - (Maharashtra I 0)
ulcer on the glans which exuded clear serum on a) Herpes genitalis b) Chancroid
pressure. Inguinal lymph nodes in both groins c) Donovanosis d) All of the above
were enlarged and not tender. The most appropriate 483. Ulcer on penis which bleeds on touch, is feature
diagnostic test is - (AIIMS 04) that best in which of the following - (MH 11)
a) Gram's stain of ulcer discharge a) Donovanosis b) Syphilis
b) Darkfield microscopy of ulcer discharge c) Herpes genitalis d) LGV
c) Giemsa stain of lymph node aspirate 484. Beefy red, foul smelling genital granuloma with a
d) ELISA for IDV infection bleeding ulcer on touch is seen in- (Jipmer 11)
474. 20 year old male lalu develops multiple ulcers over a) ll.N b) Chancroid
prepuce and glans which are painful along with c) Primary syphilis d) Donovanosis
suppurative lymphadenopathy, 5 days after having
485. Genital ulcer is seen in AlE - (PGI 97)
sexual intercourse with a sex worker, most probable
a) Granuloma inguinale b) Syphillis
diseases is - (AIIMS 96)
a) Herpes simplex b) Molluscum contagiousm c) ll.N d) Donovanosis
c) Syphilis d) Chancroid 486. Genital ulcer is/are caused by - (PGI Nov 09)
475. A man having multiple, painful, indurated, a) HPV b)HSV
undermined, sloughed edged glans which occurred c) lllV d) Treponema pallidum
5 days after exposures; most likely diagnosis is - 487. The most frequent cause of recurrent genital
a) Chancroid b) Primary chancre (AI 08) ulceration in a sexually active male is- (AI 03)
c) Herpes genitalis d)LGV a) Herpes genitalis b) Aphthous ulcer
476. 19 years male develops painless penlle ulcers 9 c) Syphilis d) Chancroid
days after sexual intercourse with a professional 488. All of the following are sexually transmited, Except-
sex worker likely diagnosis is - (AI OJ) a) Candida albicans (AI 02)
a) Chancroid b) Herpes b) Echinococcus
c) Chancre d) Traumatic ulcer c) Molluscum contagiosum
477. A 20 years old male developed multiple tender non- d) Group B streptococcus
indurated bleeding ulcers over prepuce 4 days 489. NotA sexually transmitted disease- (JH 04)
after sexual exposure. The most probable diagnosis
a) Pinta b) Candidiasis
in this patient is - (DELHI PG Feb. 09)
c) HBV d) Trichomonisis
a) Primary syphilis b) Lymphogranuloma venerum
c) Chancroid d) Donovanosis 490. Which is a noveneral form of trephonemal infection-
478. Multiple painful ulcers over glans without induration a) Yaws b) Pinta (TN95)
is suggestive of- (AIIMS 93) c) Syphillis d) GV
a) ll.N b) Granuloma inguinale 491. Which of the following is an example of a nonvenereal
c) Chancroid d) 2° syphilis disease - (CUPGEE 96)
479. A 30 year old male patient has a large, spreading and a) Endemic syphilis b) Gonorrhea
exuberant ulcer with bright red granulation tissue c) Chancroid d) WV
over the glans penis. There was no lymphadenopathy. e) Primary herpes
The most likely causative organism is - 492. Commonest trophozoite infection by sexual
a) Treponema pallidum (AIIMS Nov 03) intercourse- (Orissa 99)
b) Herpes simplex virus type 1 a) Entamoeba histolytica b) Trichomonas vagina1is
c) Herpes simplex virus type 2 c) Trepanama palidium d) Giardia intestinales
d) Calymmatobacterium granulomatis

47l)c 472)a,c 473)b 474)d 475)a 476) c 477)c 478)c 479)d 480)c 481)c 482)c 483)a 484)d
485)c 486) b,d 487) a 488) b 489)a 490)a,b 491)a 492)b
SKIN [ 617]

493. Secondary syphilis is manifested by- (PGI 03) 505. 'Chancre redox' is a clinical feature of-
a) Painless lymphadenopathy a) Early relapsing syphilis (AIIMS May 06)
b) Pruritic rash b) Late syphilis
c) Mucosal erosion c) Chancroid
d) Asymptomatic rash d) Recurrent herpes simplex infection
e) Mostly asymptomatic 506. Ollendorff sign is seen in- (Maharashtra 10)
494. In secondary syphilis all are seen except - a) Primary syphilis b) Secondary syphilis
a) Condyloma lata (MP 05, PGI 98) c) Latent syphilis d) Neurosyphilis
b) Interstitial keratitis 507. Characteristic feature of early congenital syphilis
c) Arthritis is- (TN 95)
a) Microcephaly
d) Proteinuria
b) Saddle nose
495. The skin lesions of secondary syphilis include all
c) Interstitial keratitis with saber shin
of the following except - (AIIMS 97, 98)
d) Vesicular rash with bulla over palms and soles
a) Macules b) Vesicles & Bullae 508. After giving treatment for Syphilis, the response
c) Nodule d) Papulosquamous to treatment can be best assessed by -(AIJMS Nov
496. In secondary syphilis, true about rash is -(PGI 98) a) FTA-ABS b) VDRL 99, AI 02)
a) Pruritic b) Vesicular c) TPHA d) Immobilization test
·c) Asymptomatic d) Tender 509. To indicate effective therapy in syphilis VDRL test
497. Condylomata latae are seen in - (AIIMS 99, 98) should a fall in titre of- (Orissa 99)
a) Congenital syphilis b) Primary syphilis a) One fold b) Two fold
c) Secondary syphilis d) Tertiary syphilis c) Three fold d) Four fold
498. Treponema pallidum causes - (Maharashtra 02) 510. True about syphilis is AlE- (AI 98)
a) Condyloma acuminata b) Condyloma lata a) VDRL is sensitive but not specific
c) Both d) None b) Infection leads to life long immunity
499. Candyloma lata is caused by - (Maharashtra 05) c) IgM & IgA
a) H. ducreyi d) T. pallidium when inoculated in rabbit produce
b) Calymmatobactor granulomatis progressive disease
c) Treponema pallidum 511. DOC in primary syphillis is - (AI 92)
d)HSV a) Corticosteroid b) Oral penicilline
500. The lesion characteristic of secondary syphilis - c) Benzathine penicilline d) Crystalline penicilline
a) Ulcer (Bihar 05) 512. Gonococcus does not involve - (AI 96)
b) Condyloma acumicata a) Testis b) Fallopian tube
c) Condyloma lata c) Epididymis d) Cervix
513. TOC for penicillin resistant gonorrhoea-(AIIMS 98)
d) Gumma
a) Ciprofloxacin b) Ceftriaxone
501. Which stage of syphilis is most contagions ?
c) Streptomycin d) Erythromycin
a) Primary (DELHI PG Mar. 09)
514. Donovanosis is caused by -
b) Secondary a) Calymmatobacter granulomatis
c) Early latent b) T. pertenue
d) Later latent c) Chlamydia trachomatis
502. A young man presents to the emergency department d) H. ducreyi
with a maculopapular rash 2 weeks after healing 515. Granuloma venereum is caused by- (Bihar 06)
of a painless genital ulcer. The most likely a) Donovania granulomatis b) Chlamydia
etiological agent is- (AI 11) c) Hemophillus ducrey d) None
a) Treponema Pallidum 516. Chancroid is caused by- (JH 06, MH 06)
b) Treponema Pertunae a) H. ducrey b) Y. pestis
Trachomatis · c) UN d) Tinea pedis
517. Chancroid may be caused by- (AI 99)
a) T. pallidium b) G. donovani
c) Chlamydia trachomatis d) Herpes hominis virus
518. LGV is caused by - (Bihar 05, AIIMS 99)
a) Chalamydia trachomatis
b) Haemophylus ducrei
a) Chancroid c) HTLV type II
c) Tularemia d) Donovanosis granulomatis

493)a,c,d,e 494)b 495)b 496)c 497)a,c 498)b 499)c 500)c 501)b 502)a 503)a 504)b 505)a 506)b
507)d 508)b 509)d 510)b,c 511)c 512)a 513)b 514)a 515)a 516)a 517)d 518)a
SKIN [ 618]

519. Frie test is done in - (AI 96) 534. Regarding podophyllin resin which of following
a) Donovanosis b) LGV statement is true - (PGI Dec 08)
c) Syphillis d) Leprosy a) Dervied from plant source
520. Groove sign is seen in - (MH 09, 06) b) Safe in pregnancy
a) Donovanosis b) LGV c) Teratogenic
c) Chancroid d) Genital herpes d) High recurrence rate of wart after podophyllin
521. Genital elephantiasis is caused by- (AI 02) resin treatment
a) Donovanosis b) Congenital syphilis e) Individual variation in response rate
c) Herpes genitalis d) Lymphogranuloma venerum 535. Topical immunomodulator used for the treatment
522. Painful lymphadenopathy is seen in - (PGI 02) of genital warts is - (AI 08)
a) Imiquimod b) Podophylline
a) Donovanosis b) Syphilis
c) Interferon d) Acyclovir
c) Chancroid d) Herpes simplex
536. Imiquimod used in treatment of anogenital warts
e) Gonorrhea
acts manly as - (Maharashtra 1 0)
a) Antifungal b) Antiviral
c) Immunostimulator d) Keratolytic
537. 3 year old female child develops umbHcated nodule
524. Lymphadenopathy is seen in AlE- (AI 95) over face, following a trivial infection. Probable
a) Syphilis 1st stage b) Donovanosis diagnosis - (AIIMS 99)
c) lGV d) Chancroid a) Molluscum contagiosum b) Scabies
525. DOC in chancroid is - (AIIMS 98) c) Chicken pox d) L. planus
a) Tetracycline b) Doxycycline 538. An eight year old boy presents with multiple
c) Erythromycin d) Streptomycin umbilicated papules on trunk. Diagnosis is -
526. Treatment of both partners is recommended in a) Molluscum contagiosum (AIIMS 98)
AlE- (UP 03, PGI 99) b) Chicken pox
a) Candida infection b) Gardenella c) Herpes zoster
c) Herpes d) Trichomonas vaginalis d) Dermatophytosis
527. Genital warts are most commonly caused by which 539. UmbHcated pearly white asymptomatic skin lesion
of the following serotypes of HPV? (AIIMS May 08) diagnosis is- (AIIMS 96)
a) HPV 6 b) HPV 16 a) HSV b) Molluscum contagiosum
c) HPV 18 d) HPV 33 c) :EBV d) None
528. Myrmecia warts are - (Jipmer 98) 540. The syndromic management of urethral discharge
a) Planer wart b) Plantar wart includes treatment of- (AI 03)
c) Verrucous wart d) Palmer wart a) Neisseria gonorrhoeae and herpes genitalis
b) Chlamydia trachomatis and herpes genitalis
529. All are true regarding viral warts except-(PGJ 97)
c) Neisseria gonorrhoeae and chlamydia trachomatis
a) Basophilic stippling
d) Syphilis and chancroid
b) Koilocytes are characteristic
541. Most common site of affection of herpes simplex-
c) Spontaneous regression common in children a) Thorax b) Abdomen (CUP GEE 96)
d) Perinucear vacuolization c) Face d) Extremities
e) Verruca vulgaris is associated with HPV 7 542. Herpes resistant to acyclovir is treated by-
530. Condyloma acuminata is caused by - a) Foscamet b) Lamivudine (Jipmer 02)
(Calcutta 03, Jharkhand 04, Maharashtra 11) c) Ganciclovir d) Valacyclovir
a) HPV b)HSV 543. The syndromic management of genital ulcer
c) HIV d) AU syndrome in India includes which of the following
531. Verrucosa vulgaris is caused by- (Bihar 04) disease- (AIIMS Nov 11)
a) HPV b)EBV a) Chanchroid and primarychancre
c) CMV d)HIV b) Chanchroid and herpes simplex
532. Type of human papilloma virus associated with c) Chanchroid, primarychancre and herpes simplex
cacinoma cervix- (JIPMER 98) d) Herpes simplex and primary chancre
a) Types 6,12,18 b) Types 16,18,31
c) Types 6,8,11 d) Types 3,10,19 AUTOIMMUNE SKIN DISORDERS
533. Podophyllum resin is indicated in the treatment
of- (AIIMS May 03) 544. An autoimmune disease is- (AI 2000)
a) Psoriasis b) Pemphigus a) Pemphigus vulgaris b) Psoriasis
c) Condyloma acuminata d) Condylomata lata c) Lichen planus d) Acne vulgaris

519)b 520)b 521)d 522)b,c,d,e 523)a 524)b 525)c 526) a,b 527)a 528)b 529)e 530)a 531)e 532)b
533) c 534) a,c,d,e 535) a 536) c 537) a 538) a 539) b 540) c 541) c 542) a 543) c 544) a,b,c
SKIN [ 619]

545. All are manifestations of SLE except- (PGI 98) 556. Palpable pupura is seen in all conditions except-
a) Lesions resemblig Chr. DLE a) Cryoglobulinuria b) H.S. pupura (AI 2K)
b) Butterfly rash c) Gaint cell arteritis d) Drug induced vasculitis
.c) Photosensitivity 557. Palpable purpura is seen in- (MP 06)
d) Constitutional symptom a) PAN b)ITP
e) Sex ratio in nearly equal c) TIP d)DIC
546. All of the following are causes of "lupus" except- 558. "Pinch" purpura is diagnostic of- (AIIMS May 05)
a) Systemic primary amyloidosis
a) Hydralazine b) Clofibrate (NIMHANS 06)
b) Secondary systemic amyloidosis
c) Penicillamne d) Chlorpromazine
c) Idiopathic thrombocytopenic purpura
547. True about drug induced SLE is- (PGI 2000) d) Drug induced purpura
a) CNS manifestation are common 559. Which of the following is/are not the features of
b) Renal involvement is common henoch-Schonlein purpura (HSP)- (PGI Dec 08)
c) Antihistone antibodies are found in many a) Abdominal pain b) Splinter hemorrhage
d) All with antibodies progress to lupus c) Thrombocytopenia d) Epistaxis
e) Sex ratio is nearly equal e) Arthritis
548. A 40 year old woman presented with a 8 month 560. A child develops non-blanching macules and papules
history of erythema and swelling of the periorbital on lower extremities, mild abdominal pain and skin
region & papules & plaques an the dorsolateral biopsy showed IgA deposition. Most appropriate
aspect of forearms & knuckles with ragged cuticles. diagnosis is- (AIIMS May 09)
There was no muscle weakness. The most likely a) Drug induced vasculitis b)HSP
diagnosis is - (AJIMS Nov 04) c) Wegener's granulomatosis d) Kawasaki disease
a) SIE 561. A 42 year old female bas palpable purpura with rash
b) Dermatomyositis over buttocks, pain in abdomen, and arthropathy
diagnosis is- (AI 08)
c) Systemic sclerosis
a) Sweet syndrome b) HSP
d) Mixed connective tissue disorder
c) Purpura fulminans d) Meningococcemia
549. Gottron's papules are - (COMED 09) 562. lgA deposits on skin biopsy- (AIJMS May 09)
a) Erythematous papules on cheeks a) Henoch schouleiln puspura
b) Erythema seen over elbows and knees b) Giant cell arteritis
c) Periorbitallesions c) Microscopic polyangitis
d) Violaceous papules over knuckles d) Wegener's granulomatosis
550. 'Gottron papules' seen in dermatomyositis are- 563. All regarding HSP is true except- (PGI 08)
a) Vesicular b) Psoriatic (MH 09, 07) a) Hematuria resolve without treatment
c) Lichenoid d) Pustular b) Steroids best treat skin lesions
551. All are dermatological manifestations of c) Selflirniting arthralgia
dermatomyositis except- (AIIMS Nov 09) d) Excellent prognosis
a) Gottron's patches b) Mechanic's hands e) Purpura fulrninans
c) Periungual telangiectasia d) Salmon Rash 564. A 27 year old sexually active male develops a
552. All are seen in dermatomyositis except -(Jipmer 11) vesiculobullous lesion on the glans soon after taking
a) Calcinosis cutis b) Heliotropic rash tablet paracetamol for fever. The lesion healed with
hyperpigmentation. The most likely diagnosis is-
c) Shawl sign d) Gottron's rash
a) Behcet's syndrome (AI 05)
553. All are cutaneous manifestations of systemic
b) Herpes genitalis
sclerosis, except- (AI 1 0)
c) Fixed drug eruption
a) Sclerodactyly b) Gottron's papule d) Pemphigus vulgaris
c) Telengiectasia d) Esophageal dysmotility 565. Sexually active male comes with complaints of
554. Female presents with history of color change from recurrent ulcers over the glans which heals with
pallor to cyanosis on exposure to cold in fingers. hyperpigmentation, probable diagnosis is -
This condition is mostly associated with ? a) Aphathous ulcer (AIIMS Nov 09)
a) Scleroderma (AIIMS Nov 08) b) Fixed drug eruption
b) Leukemia c) Herpes genitalis
c) Lung infections d) Chlamydia! infection
d) Hepatosplenomegaly 566. Fixed drug eruptions can be seen more frequently
555. Palpable purpura is seen in all except- (PGI 99) with- (DELHI PG Mar. 09)
a) Wegeners GN b) ITP a) Penicillin b) Sulfonamide
c) HSP d) Serum sickness c) Cetrizine d) Roxitbromycin

545)e 546)b 547) c,e 548) b 549)d 550)c 551)d 552)a 553)b>d 554)a 555)b 556)c 55?) a 558)a
559)c 560)b 561)b 562)a 563) b,e 564) c 565)b 566)b
SKIN [620]

567. Which ofthese statements is false for lesions of 576. All are seen in Tuberous sclerosis except- (AI 2000)
Erythemanodosum~ (MP 04) a) Iris nodule
a) They are considered as hypersensitivity reaction b) Renal cortical cyst
b) The skin overlying the lesions is red, smooth c) Rhabdomyoma of heart and lung
and shiny d) Adenoma sebaceum
c) They are usually non tender 577. Adenoma sebaceum is a feature of- (AJIMS Nov 05)
d) They can be associated with tuberculosis a) Neurofibromatosis
568. Erythema nodosum is due to~ (Jharkhand 04) b) Tuberous sclerosis
a) Contraceptive pills b) Barbiturates c) Xanthomatosis
c) Penicillin d) Sulphonamides d) lncontinenetia pigmenti
578. Babloo a 4 year male presents with history of
569. Erythema nodosum is seen in all of the following
seizures. On examination there is hypopigmedted
except~ (AI 11)
patches on face & mental retardation. Most probable
a) Pregnancy b) Tuberculosis
diagnosis is~ (AIIMS 2000)
c) SlE d) Chronic pancreatitis a) Neurofibromatosis b) Tuberous sclerosis
d) Calymmatobacter granulomatis c) Sturge weber syndrome d) Incontinenta pigmenti
570. 25 years old male having fever & malaise since 2 579. Ash leaf maculae is found in- (JH 05, DPG 08)
weeks, arthritis of ankle joint and tender a) Tuberous sclerosis b) Neurofibromatosis
erythematous nodules over the shin. Diagnosis is- c) Lymphangioma d) None
a) Erythemanodosum (AIIMSMay 10} 580. Koenen's periungual fibromas are seen in> 50%
b) Hensen's disease ofcaseswith- (Jipmer 02)
c) Weber-christian disease a) Tuberous sclerosis
d) Urticarial vasculitis b) Sturge weber syndrome
571. Genodermal disease that can cause skin malignancy c) Alaxia telangiectasia
are- (PG/03) d) Neurofibroatosis
a) Xeroderma pogmentosa 581. Granular layer is absent in- (Orissa 98)
b) Neurofibromatosis a) Ichthysis vugaris
c) Actinic keratosis b) X linked ischyosis
d) Porphyria cutanea tarda c) Epidermolytic hyperkeratosis
572. Lisch nodule is seen in- (PG/99) d) Lamellar ichthyosis
a) Von reclinghausens disease 582. lchthysis is caused by- (Nimhans 01)
b) Lupus vulgaris a) Hemosiderosis b) Refsum disease
c) Leprosy c) Niacin deficiency d) Steven johnson syndrome
583. Steroid suphatase deficiency is seen in-
IGV
a) Ichthyosis vulgaris (COMED 06)
b) Lamellar Ichthyosis
c) 'X' linked recessive icthyosis
_~;:.,::
d) None of the above
pat1en a seven 1rregu ar yperp•gmented 584. Skin pigmentation & icthyosis like side effects are
macules on the trunk and multiple small seen in - (AI 96)
hyperpigmented macules in the axillae and groins a) Rifampcin b) Clofazimine
since early childhood. There were no other skin c) Dapsone d) Steroid
lesions. Which is the most likely investigation to
support the diagnosis ?
a) Slit lamp examination of eye
b) Measurement of intraocular tension
(AI 06)
~tr~~!~~'illF
586. The mode of inheritance ofincontinentia pigment
i:i'J
c) Examination of fundus is- (Kerala 04)
d) Retinal artery angiography a) Autosomal dominant b) Autosomal recessive
575. A patient has multiple meningiomas, acoustic c) X-linkeddominant d) X-linkedrecessive
neuroma and Hyperpigmented skin lesions; most 587. True about incontinenta pigmeuti include the
likely diagnosis is- (MAHE 07) following except- (AIIMS May 11, AI 09)
a) Neurofibromatosis a) X-linked dominant
b) Tuberous sclerosis b) Primary skin abnormality
c) Von Hippellindau disease c) Avascularity of peripheral retina
d) Sturge weber syndrome d) Ocular involvement is seen in almost l 00% cases
and is typically unilateral

567)c 568) a,d 569) d 570)a 57l)a 572)a 573)a 574)a 575)a 576)a 577)b 578)b 579)a 580)a
58l)a 582)b 583)c 584)b 585)d 586)c 587)d
SKIN [ 621]

588. Cells cultured from patients with this disorder 599. Mycosis fungoides which is not true-
exhibit low activity for the nucleotide excision repair a)It is the most common form of cutaneous
process. This autosomal recessive genetic disease lymphoma (AI 07, AIIMS Nov 06)
includes marked sensitivity to sunlight (Ultra voilet b) Pautriers microabscess
light) with subsequent formation of multiple skin c) Indolent course and good prognosis, easily
cancers and premature death, the disorder is - amenable to treatment
a) Acute intermittent porphyria (Karnatak 98) d) Erythroderma seen and spreads to peripheral
b) Alkaptonuria
circulation
c) Xeroderma pigmentosa
d) Ataxia - Telangiectasa

SKIN TUMORS

treatment of-
a) Mycosis Fungoides b) Psoriasis
590. Maximum malignant potential is in- (Al98) c) Sezary Syndrome d) Brain Metastasis
a) Superficial naevus b) Epidermal naevus 602. Following are signs of internal malignancy except-
c) Junctional naevus d) Intradermis naevus a) Tuberous sclerosis (PGI 97)
591. Which naevi commonly predispose to malignant b) Acanthosis nigricans
melanoma? (DELHI PG Feb. 09) c) Clubbing
a) Dermal b) Junctional d) Dermatomyositis
c) Congenital d) Lentigo 603. Underlying internal malignancy is not shown by-
592. Predisposing factors for skin cancer are-
a) Acanthosis nigricans & annular erytheman
a) Smoking b) U-V-light (PGI Dec 2K)
b) Bullous pyoderma & migratory necrotizing
c) Chronic ulcer d) Infrared light
593. Predisposing factors for skin ca are- (PGI 02) erythema (AI 96)
a) Lichen planus b) Bowen's disease c) Granuloma annulare
c) Psoriasis d) Bechet's disease d) Erythema gyratum repens
e) UVrays
594. Changes of squamous cell carcinoma are seen in -
a) Seborrhoeic keratosis (PGI 97)
b) Bowen's disease
c) Lichen planus
d) DIE
595. Actinic keratosis is seen in- (AIIMS May 02) MISCELLANEOUS
a) Basal cell carcinoma
b) Squamous cell carcinoma 605. The most potent topical corticosteroids is-(DPG 09,
c) Malignantmelanoma a) Hydrocortisone butyrate cream 0.1% Orrisa 98)
d) Epithelial cell carcinoma b) Betamethasone valerate cream 0.1%
596. Which of the following is associated with sun
c) Clobetasol propionate cream 0.5%
exposure? (AIIMSMay 12)
d) Clobetasone butyrate cream 0.5%
a) Actinic keratosis
b) Seborrheic keratosis 606. Uses ofPUVA- (PGI Dec 04)
c) Sebaceous cell carcinoma a) Pigmented purpuric lesion
d) Syringoma b) Herpes zoster
597. Mycosis fungoides affects- (CMC 98) c) Mycosis fungoides
a) T Cells b) B Cells d) Lupus panniculitis
c) NKCells d)KCells e) Lichenoid dermatitis ofGougerot & Blum
e) None of the above 607. Photochemotherapyis useful in- (SGPGI 03)
598. Pautrier's micro-abcess is a histological feature a) Psoriasis b) Lichen planus
of- (AIIMS Nov 05, TN 99) c) Tinea d) Icthyosis
a) Sarcoidosis 608. PUVA therapy is used in- (PGI 02)
b)TB a) Psoriasis b) Lichen planus
c) Mycosis fungoides c) Freckles d) Melasma
d) Ptyriasis lichenoides chronica e) Vitiligo

588)c 589)d 590)c 59l)c 592) b,c 593) a,b,e 594) b,c,d 595) b 596) a 597) a 598) c 599) c 600) a 601) a
602)a 603)c 604)d 605)c 606) a,c,e 607) a >b 608) a,b,e
SKIN [622]

609. Psoralen-A is used in the treatment of- 624. Mucous lesions are seen in- (PGI 02)
a) Pemphigus b) Vitiligo (DELHIPGFeb. 09) a) Sec. Syphilis b) Dermatitis herpetiformis
c) Pityriasis alba d) Icthyosis c) Psoriasis d) Pemphigus
610. Best range of UV light used for treatment of skin e) Porphyria
diseases- (Maharashtra OJ) 625. A 27-year-old male bad burning micturition and
a) 100-200nm b)200-400nm urethral discharge. After 4 weeks be developed joint
c) 400-700nm d) >700nm pains involving both the knees and ankles, redness
611. Woods lamp used in diagnosis of- (PGI Dec 06) of the eye and skin lesion. The most probable clinical
a) P. versicolor b) Vitiligo diagnosis is- (AIIMS May 05)
c) Porphyria d) Psoriasis
a) Psoriasis vulgaris b) Reiter's syndrome
e) Lichen planus
c) Behcet's syndrome d) Sarcoidosis
612. Wood's lamp light is used in the diagnosis of-
626. Which of the following is not included in the triad of
a) Tinea capitis (SGPGI 02, AIIMS May 02)
Reiter's syndrome- (CUPGEE 96)
b) Candida albicans
c) Histoplasma a) Conjuctivitis b) Urethritis
d) Cryptococcus c) Arthritis d) Keratoderma blenorrhagica
613. Wood lamp is used to diagnose- (MH05) 62 7. All are seen in reiter' s syndrome except ?
a) Psoriasis b) Tinea versicolor a) Subcutaneous nodules (AIIMS Nov 08)
c) Pityriasis rosae d) Erysipelas b) Oral ulcers
614. Woodslampbasafrequencyof- (TN04) c) Keratoderma blenorrhagicum
a) 250nm b)300nm d) Circinate balanitis
c) 320nm d)360nm 628. A person suffers from B27 associated reactive
615. Coral red colour on wood lamp seen is-(MH J 0, 08, arthritis, urethritis and conjunctivitis. Which is
a) Erythrasma b) Pseudomonas Bihar 04) most likely organism involved in this case ?
c) Tinea d)ALL a) Borrelia burgdorferi (AIIMSNov08)
616. Yellow fluorescence on wood's lamp is seen in- b) Ureaplasma urealyticum
a) Tuberous sclerosis (MH 07) c) Beta hemolytic streptococci
b) Pseudomonas infection d) Streptococcus bovis
c) Tinea capitis 629. Noninfective cause of genital ulcer is- (J & K 04)
d) Erytbrasm a) Behcets disease b) Hansen's disease
617. Diascopyisveryhelpfulindiagnosisof- (MH 09) c) Ramsons's disease d) Syphilis
a) Cutaneous vasculitis b) Nevus anaemicus
c) Lupus vulgaris d) All of the above
618. Morbilliform eruptions is seen in- (PGI OJ)
a) Scarlet fever b) Rubella
c) Toxic shock syndrome d) Measles
631. A patient gives a history ofrecnrrent oral ulcers.
e) Mumps
The ulcers are small with a yellow floor surrounded
619. Exantbemsarecausedbyallexcept- (MHOJ)
a) Malaria b) Typhoid by an erythematous halo on the lips. He also has
c) Measles d) Rubella multiple, tender nodules on the skin. He most likely
620. Which of the following are pruritic lesions- has- (AIIMSNov 11)
a) Lichen planus b) Sun bums (PGI 2K) a) Behcet's syndrome b) Herpes labialis
c) Pemphigoid d) Psoriasis c) Fixed drug eruption d) Pemphigus vulgaris
e) SI.E 632. Reccurent oral ulcers with pain and erythematous
621. Pruritis is seen in all except- (Kerala 95, DPG 09) halo around them, diagnosis is- (AIIMS 96)
a) Pemphigus b) Lichen simplex cbrouicus a) Apthus ulcer b) Herpes
c) Psoriasis d) Contact dermitits c) Chickenpox d) Measels
622. Mucosa is involved in- (PGI Dec 07) 633. A child with fever presents with multiple tender
a) Psoriasis b) Lichen planus erythematous skin lesions. On microscopic
c) Alopecia d) Scabies examination the skin lesions are seen to have
e) Porphyria neutrophilic infiltration in the dermis. What is the
623. Oral examination is done in case of- (PGI 97) diagnosis- (AI 11, 09)
a) Peutzjeghersyndrome a) Sweet syndrome b) Behcet's syndrome
b) Psoriasis c) Pyoderma gangrenosum d) Juvenile dermatosis
c) Beri-beri
d) Plummer vinson syndrome

609)b 610)b 61I)a,b,c 612)a 613)b 614)d 615)a 616)c 617)d 618)b,d 619)a 620)a,b,c 62l)a,c
622)b 623)a 624) a,d 625) b 626) d 627) a 628)b 629)a 630)a 631)a 632)a 633)a
SKIN [ 623]

634. A young female presents with a history of fever and 644. Hirsutism can be caused by the following except-
nodular lesion over the skin. Histopathology reveals a) Flutamide b) Norethisterone (AI 10)
foamy histiocytes with neutrophilic infiltration. c) Danazole d) Phenytoin
There is no evidence of vasculitis. Most probable 645. Child presents with linear verrucous plaques on
diagnosis is- (AI 11) the trunk with vacuolization of keratinocytes in
a) Sweet's syndrome S.Spinosum and S.Granulosum. Diagnosis is ?
b) Erythemanodosum a) Incontinenta pigmenti (AIIMSMay 11, Nov. 08)
c) Erythema nodusum leprosum b) Delayed hypersensitivity reaction
d) Behcet's syndrome c) Verrucous epidermal nevus
635. Pyoderma gangrenosum is seen in- (TN 97)
d) Linear darriers disease
a) Crohns disease b) Divertuculosis
c) Ulcerative colitis d) Ca. colon

seen in-
a) Dowling - Degos disease
b) Rothmund thompson syndrome
c) Cockyane syndrome
d) Bloom's syndrome
638. Flakypaintappearence ofskiu is seen in- (A/95) 648. Child with erythematous non blanching bosselated
a) Dermatitis b) Pellagra lesion on right side of face, Rx. is - (AIIMS may 11)
c) Marasmus d) Kwashiorkar a) Erbium laser
639. Casal's paint necklace is caused by- (PGI97) b) Nd-YAG laser
a) Lichen planus b) Pellagra c) Flash light pumped dye laser
c) Pernicious anemia d) SLE d) Q ruby laser
640. Which is false about acrodermatitis enteropathica- 649. Neonatal fat necrosis resembles which of the
a) Triad of diarrhoea, dementia and dermatitis following- (AIIMS may 11)
b) Low serum zinc levels (AIIMS may II) a) Post steroidal panniculitis
c) Symptoms improve with zinc supplementation b) Erythema induratum
Autosomal recessive c) Lipodermatosclerosis
d) Lupus panniculitis
650. Chemical peeling is done by all except-(AllMS Nov 10)
a) Trichloroacetic acid b) Phosphoric acid
c) Carbolic acid d) Kojic acid
642. "Calcinosis universalis" occurs in- (J & K 05)
651. Lines ofBlaschko represent- (AI 11)
a) Ehlers Danlos syndrome
a) Lines along lymphatics
b) Christian Weber syndrome
c) Dermatomyositis b) Lines along blood vessels
d) Scleroderma c) Lines along nerves
d) Lines of development

***

634)b>a 635)a,c 636)b 637)a 638)d 639)b 640)a 64I)b 642)c 643)a 644)a 645)c 646)a 647)a
648) c 649) a,c,d 650) b 651) d 652) d
PSYC RY
PSYCHIATRY

SYMPTOMS & SIGNS IN PSYCHIATRY AND 13. Hallucinations are- {Maharashtra 02, AI 99)
CLASSIFICATION a) Feeling of familiarity with unfaniiliar thing
b) Alteration of perception of ones reality
1. Mutism and akinesis in a person who appears awake c) Misinterpretation of stimuli
and even alert, is best described as- {AI 06) d) Perception occurring without external stimulation
a) Twilight state b) Oneroid state 14. A patient sees a rope and fears like a snake. It is
c) Steupor d) Delirium called- {PGI 02)
2. Emotion is controlled by- {PGI 97) a) Illusion b) Hallucination
a) Limbic system b) Frontal lobe c) Delusion d) Depersonalization
e) Derealization
c) Temporal lobe d) Occipital lobe
15. A 8 year old child after tonsillectomy sees a bear in
3. A 25 year old woman complains of intense depressed
her room. She screams in fright. A nurse who rushes
mood for 6 months with inability to enjoy previously
in switches on the light, finds a rug wrapped on a
pleasurable activities. This symptom is known as -
armchair. She pacifies the child what the child
a) Anhedonia b)Avolition {AIIMSMay05) experienced was a- {Kerala 97)
c) Apathy d)Amotivation a) Delusion b) Illusion
4. Alexithymia is- {Kerala 2K) c) Hallucination d) None of the above
a) A feeling of intense rapture 16. Which is not true about hallucination -
b) Pathological sadness a) It is as vivid as normal stimulus {AIIMS May 09)
c) Affective flattening b) It occurs in inner subjective space
d) Inability to recognise and describe feelings c) It is independent of will of observer
e) Inappropriate mood d) It occurs in absence of perceptual stimuli
5. A person who laughs one minute and cries the next 17. All of the following feature of hallucination except-
without any clear stimulus is said to have- a) Depends on will of observer and is under voluntary
a) Incongruent effect {AIIMSNov05) control {SGPGI 03, AI 02)
b) Euphoria b) Occur in inner subjective space
c) Labile affect c) It is vivid as sensory perception
d) "Split personality" d) It occurs in absence of perceptual stimulus
6. Ambivalence is most commonly seen in- 18. All ofthe following are features of hallucinations,
a) Schizophrenia b) Hysteria except- {AI 03)
c) Mania d)OCN {AIIMS 96) a) It is independent of the will of the observer
7. Ambivalence is most commonly associated with- b) Sensory organs are not involved
a) Depression {MP 02) c) It is as vivid as that in a true sense perception
d) It occurs in the absence of a perceptual stimulus
b) Generalized anxiety disorder
19. Visual hallucinations are seen in- {PGI June 09)
c) Schizophrenia
a) Hebephrenic schizophrenia
OCD
b) Residual schizophrenia
c) Simple schizophrenia
d) Delirium
e) Temporal lobe epilepsy
9. Hallucination and illusion are disturbances of- 20. Formed visual hallucinations are seen in lesion of-
a) Thought b) Perception {MH09) a) Frontal {PGI June 06, 2K)
c) Sensation d) Mood b) Occipital
c) Temporal
d) Arcuate fasciculus
e) Dentate nucleus
11. Altered perception of real objects is -{AllMS Nov 07) 21. Whatishypnopompicphenomenon? {Bihar 06)
a) lllusion b) Delusion a) If experienced while falling asleep
c) Hallucination d) Delirium b) If experienced while awakening
12. False sense of perception without any external object c) After head trauma
or stimulus is known as- d) After convulsion
a) lllusion {AIIMS May 03, Jharkhand 05, 22. Hallucinations which occur at the 'start' of sleep is-
b) Impulse AIIMS 96, AI 98, Maharashtra 07) a) Hypnagogic hallucinations {Jipmer 02)
c) Hallucination b) Hypnopompic hallucinations
d) Phobia c) Jactatio nocturna capitis
d) Non- specific hallucination

l)c 2)b 3)a 4)d S)c 6)a 7)c 8)b 9)b IO)c ll)a 12)c 13)d 14)a
15)b 16)b 17)a 18)b 19)All 20)c 2l)b 22)a
PSYCHIATRY [ 646]

23. Reflex hallucination is a morbid variety of- 34. A man hits his neighbour. Next day he feels that
a) Kinesthesia (AIIMS May 11, 09) police is behind him and his brain is being controlled
b) Parasthesia by radio wa"\l'es by his neighbour. The probable
c) Hyperasthesia diagnosis is- (AIIMS 99)
d) Synaesthesia a) Thought insertion
24. A patient gets tingling when his bed light is flashed b) Passivity feeling
suddenly. Which type of hallucination is this- c) Delusion of persecution
a) Hypnagogic hallucinations (Maharashtra 10) d) Obscessive compulsive disorder
b) Hypnopompic hallucinations 35. A 25 year old university student had a fight with the
c) .Reflex hallucinations neighbouring boy. On the next day while out, he
d) Functional hallucinations started feeling that two men in police uniform were
25. Hallucinations are seen in all except- (MP 99) observing this movements. When he reached home
a) Schizophrenia in the evening he was frightened. He expressed that
b) Seizure due to intracerebral space occupying lesions police was after him and would arrest him. Ills
c) LSD symptoms represent- (AIIMS Nov 03)
d) Anxiety a) Delusion of persecution b) Ideas of reference
26. Cognition is- (Jharkhand 03, AI 97) c) Passivity d) Thought insertion
a) Perception b) Thought 36. Delusion is not present in- (AI 02)
c) Action d) Feeling a) Delirium b) Mania
27. Cognitive disorders- (PGI June 06, 07, Dec 03) c) Depression d) Compulsive disorder
a) Delirium b) Depersonalization 37. Delusions are seen in all the following except-
c) Dementia d) Secondary gain a) Obsessive compulsive disorder (SGPGI 02)
28. All are format thought disorders except - (Bihar 07) b) Depression
a) Schizophrenia c) Mania
b) Delusion d) Schizophrenia
c) Loosening of association 38. Delusion is not seen in- (Jipmer 95)
d) Obsessive compulsive neurosis a) Depression b)Anxiety
29. Loosening of association is an example of- (AI 06) c) Schizophrenia d) Mania
a) Formal thought disorder 39. Following are correctly matched except- (PGI 98)
b) Schneider's frrst symptoms a) Schizophrenia-Auditory hallucination
c) Perseveration b) OCD-infidelity
d) Concrete thinking c) Alcoholism- auditory hallucination
30. Which of the following is not a disorder of form d) Depression - guilt feeling
ofthought? (AIIMS May 12) 40. Delusion of persecution occur in- (PGI June 09)
a) Derailment a) Schizophrenia
b) Tangentiality b) Paranoid psychosis
c) Thought block c) Manic episode
d) Loosening of association 41. Delusion of grandiosity seen in- (PGI Nov. 1 0)
31. Delusion is a disorder of- (AIIMS Nov 06, AIIMS 97, a) Hypomania
a) Thought AI 07, 99, Jharkhand 04, b) Schizoaffective mania
b) Perception Maharashtra 03, UP 08) c) Paranoid schizophrenia
c) Insight d) Kleptomania
d) Cognition e) Pyromania

~~~F~
32. Persistent belief in something which is not a
fact is- (UPSC 95, WB OJ)
a) Illusion b) Hallucination
c) Delusion d)Delirium Perseveration is-
33. Nihilistic ideas are seen in- (PGI Dec 08) a) Persistent and inappropriate repetition of the same
a) Simple schizophrenia thoughts
b) Paranoid schizophrenia b) When a patient feels very distressed about it
c) Cotard's syndrome c) Characteristic of schizophrenia
d) Depression d) Characteristic of obsessive compulsive disorder
e) Body dysmorphic disorder (OCD)

23)d 24)c 25)d 26) b 27) a,c 28) b,d 29) a 30) c 31)a 32)c 33)c,d 34)c 35)a 36)d
37)a 38)b 39)b 40)a,b,c 41)b,c 42)a 43)a
PSYCHIATRY [ 647]

44. Thoughts iu mind with rhyming rather than with 54. Consultation - liation (C-L) psychiatry involves
meaning ofthoughts- (Maharashtra 10) diagnosing- (MAHE 06, SGPGI 04)
a) Flight of ideas a) Psychiatric illness in medically ill
b) Perseveration b) Medical illness in psychiatric patients
c) Circumstantiality c) Sucidal tendency in psychiatric patients
d) Clang association d) Sucidal tendency in medically ill
45. Loss of capability of doing goal directed things, to 55. Rorschach inkblottestis- (Bihar 03)
express ideas, making irrelevant comments, but a) Projective b) Subjective
ultimately succeeding in going back to original point c) Both d) None
is known as- (Maharashtra 07, 06) 56. Rorschach test measures- (PGI 99)
a) Commentalism b) Tangentiality a) Intelligence b) Creativity
c) Dysprosodia d) Circumstantiality c) Personality d) Neuraticism
46. Repetition of movements, act, words and phrases; 57. Signs of organic brain damage are evident on-
beyondpointofrelavance- (Maharashtra 08) a) Bender Gestalt test (AI 04)
a) Fusion b) Mannerism b) Rorschach test
c) Perseveration d) Stereotypy c) Sentence completion test
47. Confabulationisdefectof- (MP 00, AIIMS 98) d) Thematic appreception test
a) Memory b) Intelligence 58. l'be halstead reitan battery involves all except-
c) Attention d) Concentration a) Finger oscillation (PGI99)
48. True about confabulation- (Jharkhand 04) b) Constructional praxis
a) A memory disorder c) Rhythm
b) Synonymous with false memory syndrome d) Tactual performance
c) It is congenital 59. Which of the following is not a cognitive error/
d) It is age related disease dysfunction- (AI 10)
49. Confabulation is typically seen in- (Kerala 98) a) Catasrophic thinking b) Arbitrary inference
a) Mania b) Depression c) Overgeneralization d) Thought block
c) Alcoholism d)Delirium 60. Healthy thinking includes all except-(AJIMS May 11)
50. Confabulationis- (SGPGI03) a) Continuity b) Constancy
a) A state of confusion where patient is not able to c) Organization d) Clarity
describe the details
b) Purposefully fabricating stories to project a certain ORGANIC MENTAL DISORDERS
image
c) Filling up to gaps by fabrication to cover lapses in 61. All are seen in organic psychosis except-
a) Disorientation in place, time & person
memory
b) Clouding of consciousness (Kerala 96)
d) Seen in delirium
c) Hallucination
51. Unfamiliarity of familiar things is seen in-
d) Normal common knowledge
a) Deja vu (Kerala 99, Jipmer 02, Karn 94)
e) Intellectual impairment
b) Jamaisvu 62. Disturbed orientation is seen in- (MP 99)
c) Deja entendu a) Schizophrenia
d) Deja pence b) Neurosis
52. Catatonic features are seen in schizophrenia they c) Paranoid personality
are also seen in - (PGI June 08) d) Organic mental disorder
a) Severe depression b) Conversion disorder 63. Which of the following behavioral problems would
c) Personality disorder d) Somatisation disorder suggest an organic brain lesion- (SGPGI 05)
53. The DSMJV classification of psychiatric disorder a) Formal thought disorder
as proposed by American Psychiatry Association b) Auditory hallucinations
classifies and helps in diagnosing patient on multiple c) Visual hallucinations
axes. Of these, axis V represents the degree of- d) Depression
a) Present state of symptoms (Maharashtra 09) 64. The following is suggestive of an organic cause of
b) Comorbid medical condition the behavioral symptoms- (AI 02)
c) Global assessment of function a) Formal thought disorder
d) Comorbid psychological problem b) Auditory hallucinations
c) Delusionoffruit
d) Prominent visual hallucination

44)d 45)d 46)c 47)a 48)a 49)c 50)c 5l)b 52)a,b 53)c 54) a 55) a 56)c 57) a
58)b 59)d 60)b 61)d 62)d 63)c 64)d
PSYCHIATRY [ 648]

65. Features of delirium- (PGI Nov. J0, June 08) 76. Notafeatureofdementiais- (AIIMS 99)
a) Deficit of attention a) Loss of sensorium b) Wearing of dirty clothes
b) Autonomic instability c) Forget fullness d) Loss of neurons in brain
c) Altered sleep wake pattern 77. Catastrophic reaction is feature of- (MH-11)
d) Hallucination a) Dementia b) Schizophrenia
e) Delirium can not diagnosed clinically c) Delirium d) Anxiety
78. Characteristic feature of subcortical dementia is-
a) Memory loss b) Aphasia (AIIMS 99)
c) Dyslexia d) Tactile agnosia

67. Allarefeaturesofdeliriumexcept- (AIIMS 96)


a) Confusion b) Disorientation
c) Hyperactivity d) Loss of memory
68. A patient semiconscious with altered sensorium 80. All are causes of subcortical dementia except-
with visual hallucination and fragmented delusions a) Alzheimer's disease (AIIMS May 09)
is suffering from, which of the following disorder- b) Parkinson's disease
a) Delirium b) Delusion (SGPGI 02) c) Supranuclear palsy
c) Schizophrenia d) Mania d) HIV associated dementia
69. A patient with pneumonia for 5 days is admitted to 81. Which one of the following is a subcortical
the hospital in altered sensorium. He suddenly dementia? (UPSC-1 08)
ceases to rl:lcognize the doctor and staff. He thinks a) Alzheimer's disease
that he is in jail and complains of scorpion attacking b) Huntington's chorea
him. His probable diagnosis is- (AI OJ) c) Normal pressure Hydrocephalus
a) Acute dementia b) Acute delirium d) Vit B 12 deficiency
c) Acute schizophrenia d) Acute paranoia 82. All are examples of cortical dementia except-
70. Global cerebral dysfunction characterized by a) Alzheimer's disease (Jipmer 03, SGPGI OJ)
alteration in cognitive function and consciousness b) Multiple sclerosis
is known as - (Maharashtra 09) c) Creutzfeldt Jacob disease
a) Acute anxiety b) Depression d) Pick's disease
c) Delirium. d) Dementia 83. One of the following is a reversible cause of
71. An alcoholic is brought to the Emergency OPD dementia- (MP 03)
with the complaint of irrelevant talking. He bad a) Toxic dementia b)Alzheimer's ds.
stopped using alcohol three days back. On c) Multi-infarct dementia d) Pick ds.
examination, be is found to be disoriented to time, 84. Reversiblecauseofdementia- (AP 96, AI95)
place and person. He also bas visual illusions and a) Post encephalitis b) Multi infarct
hallucinations. There is no history of bead injury. c) Hydrocephalus d) Senile dementia
Most likely diagnosis is- (AI 03) 85. Reversible cause of dementia is- (AI 95)
a) Demetia praecox b) Delirium tremens a) Multi infarct b) Senile dementia
c) Schizophrenia d) Korsakoff's psychosis c) Post encephalitis d) Huntington's chorea
72. Delirium tremens is characterized by confusion 86. Reversiblecanseofdementia- (PGI June 04)
associated with- (SGPGI 04, AI 03) a) Hypothyroidism b)Alzheimer's disease
a) Autonomic hyperactivity and tremors c) Vitamin B 12 deficiency d) VitaminAdeficiency
b) Features of intoxication due to alcohol 87. Treatable causes of dementia are- (PGI 0 J)
c) Sixth nerve palsy a) Alzheimer's ds. b) Hypothyroidism
d) Korsakoff psychosis c) Multi-infarct dementia d) SDH (subdural h'ge)
73. All are true about Delirium tremens except- e) Hydrocephalus
a) Severe depression b) Hallucination (ADMS 98) 88. Dementia is seen in all except- (MP 97)
c) Extreme anxiety d) Delusion a) Schizophrenia b) Head injury
74. Slowwaves inEEGactivityseenin- (PGI 98) c) Huntington's chorea d) Frontallobe tumour
a) Depression b) Delirium 89. Dementia is seen in all except- (MP 98, WB 2K)
c) Schizophrenia d) Mania a) Schizophrenia b)Alzheimer's disease
75. Which of the following is not used in delirium- c) Huntington's chorea d) Pick's ds.
a) Haloperidol b) Lithium (PGI Dec 05) 90. Most common cause dementia in adult-(Jharkhand 06)
c) Diazepam d) Olanzapine a) Alzheimer's b) Multiinfarct
e) Resperidone c) Pick disease d) Metabolic cause

65) a,b,c,d 66) a 67) d 68) a 69) b 70)c 71)b 72)a 73)a 74)b 75)b 76)a 77)a
78)None>a 79)b 80)a 81)b 82)b 83)a 84)a,c 85)c 86) a,c 87) b,d,e 88) a 89)a 90)a
PSYCHIATRY [ 649)

91. Pseudodementiaisseenin-(PGI97,Delhi03,Kam.JJ) 102. AlE are true regarding Alzheimer's disease-


a) Alcoholism b) Depression a) Gradually progressive (PGI Feb 08)
c) Schizophrenia d) Mania b) Abrupt onset acute exacerbation
92. A 65 year old male is brought to tbe outpatient clinic c) Episodic memory
with one year illness characterized by marked d) Fronto temporal disorder
forgetfulness, visual hallucination, suspiciousness, e) Ubiquitin lewy bodies
personality decline, poor self care and progresssive 103. Progressive cognitive impairment in Alzheimer's
deterioration in his condition. His Mini Mental disease is characterized by- (Maharashtra 06)
Status Examinations (MMSE) Score is 10. His most a) Nominal aphasia
likely diagnosis is - (AIIMS Nov 02)
b) Recent memory loss
c) No behavioral abnormalities
a) Dementia b) Schizophrenia
d) All
c) Mania d) Depression
104. Frontotemporal dementias include AlE- (UP 07)
93. Vascular dementia is characterized by- (PGI 03) a) Pick's disease b) Nonfluent aphasia
a) Disorientation b) Memory deficit c) Semantic dementia d) Alzheimer's disease
c) Emotional liability d) Visual hallucination 105. Rivastigmine and Donepezil are drugs used
e) Personality deterioration predominantly in the management of- (AI 06)
94. A 70 yr old man presents with hlo prosopagnosia, a) Depression b) Dissociation
loss of memory, 3rd person hallucination since 1 c) Delusion d) Dementia
month. On examination deep Tendon reflexes are 106. Donepezilis used in treatment of- (MP 05)
increased, minimental examination score is 20/30. a) Alzheimer's dementia b) Schizophrenia
What is most likely diagnosis- (AIIMS OJ) c) Anxiety disorder d) Depression
a) Dissociated dementia b) Schizophrenia 107. Korsakoff's psychosis is seen in- (PGI Dec 05)
c) Alzheimers disease d) Psychotic disorder a) CRF b) Alcohol withdrawal
95. Protein involved in Alzheimer's disease-.(Nimhans 01) c) Marasmus d) Cirrhosis
a) Apo E gene b) Presenilin II 108. The site oflesion in Korsakoff's psychosis is-
c) Amyloid portion d) All a) Frontal lobe (AIIMS 96)
96. Biochemical etiology ofAlzheimer's disease relates b) Corpus striatum
to- (AIIMS 96) c) Mamrnilary body
a) Serotonin b) Dopamine d) Cingulate gyrus
c) Acetyl choline d) GABA 109. Wernicke's encephalopaty involves which part
97. Following are predispositions to Alzheimer's of CNS- (PGI 2000)
disease except- (AI 99, WB 99) a) Mannnillary body b) Thalamus
a) Down's syndrome b) Head trauma Frontal lobe Arcuate fasciculus
c) Smoking d) Low education group
98. All the following are featuring Alzheimer's disease
except- (WB 02)
a) Cerebellar atrophy
b) Common in 5th and 6th decade
c) Atrophied gyri and widened sulci
d) Progressive dementia year
99. Neurofibrillary tangles with senile plaques are presents with confusion, nystagmus and ataxia.
seen in - (PGI 2000) Examination reveals 6th cranial nerve weakness.
a) Parkinson's disease b)Alzheimer's disease He is most likely to be suffering from - (AI 05)
c) Schizophrenia d) Tuberous sclerosis a) Korsakoff's psychosis
100. Dementia ofAlzheimer's type is not associated with b) Wernicke encephalopathy
oneofthefollowing- (AIIMSNov05) c) DeClerambault syndrome
a) Depressive symptoms b) Delusions d) Delirium tremens
c) Apraxia and aphasia d) Cerebral infarcts 112. A alcoholic person comes to your office, be can't tell
101. Regarding Alzheimer's disease which is/are his name, there is gross incoordination in walking,
not true- (PGI June 09, PGI Dec 08) and bis eye is deviated to one side, what is your
a) Initial loss oflong term memory diagnosis- (Bihar 06)
b) Delayed loss of short term memory a) Wernicke's encephalopathy
c) Step ladder pattern b) Korsakoff's psychosis
d) Cognitive impairment c) Alcoholic hallucination
e) Judgment impairment d) Delirium tremens

91)b 92)a 93) a,b,c,d 94) c 95) d 96) c 97) c 98)a 99)b IOO)d 101) a,b,c 102) b,d,e 103) b
104)d 105)d 106)a 107)a,c 108)c 109)a,b llO)a lll)b ll2)a
PSYCHIATRY [ 650]

f!~4~g;~y~~~~
123. Not seen in Korsakoff's syndrome- (Jipmer 2000)
a) Clear consciousness
b) Inability to learn new things
c) Halluciations
114. All are the features of Korsakoff's syndrome d) Confabulation
except- (Jipmer 98, UP 02) 124. Wernicke's encephalopathy can be prevented by-
a) Confabulation b) Retrograde amnesia a) IV saline (PGI 01, AIIMS 98)
c) Antegrade amnesia d) Defective motor skill b) Large amounts of glucose
115. In korsakoff psychosis all are seen except- c) Thiamine supplement
a) Loss of remote memory (Jipmer 98) d) Imipramine
b) Loss of intellectual function but preservation of 125. 35 year old male comes with h/o 10 year of
memory alcoholism and past history of ataxia with B/L
c) Lack of insight, unable to understand the disability lateral rectus palsy. He was admitted & treated.
d) Reversible state What changes expected to see- (PGI June 08)
116. True statement about Korsakoff's psychosis a) Progression to Korsakoff's psychosis
is - (Rohtak 2000, Jipmer 99, UP 99, PG197) b) Residual ataxia in 50% of patients
a) Severe antegrade +Mild retrograde memory defect c) Extraocular palsy disappear in hour
b) Mild antegrade +severe retrograde memory defect d) Immediate relief from symptoms
c) Only antegrade memory defect 126. Anterograde amnesia is seen in -
d) Only retrograde memory defect a) Head injury (AIIMS Nov 10)
117. Korsakoff's psychosis is diagnosed by- (AJ2000) b) Stroke
a) Peripheral neuropathy c) Spinal cord injury (Traumatic paraplegia)
b) Visual hallucination d) Alzheimer's disease
c) Impairment oflong term memory 127. Amnesiaisfoundin- (AI 2000)
d) Seizures a) Head injury b) Mania
118. All are relatively normal in Korsakoff's psychosis c) Schizophrenia d) Psychiatric state
except- (MAHA 03, KA 03, J & K 00) 128. Biological amnesia is- (AIIMS 98)
a) Implicitmemory b) Intelligence a) Lack of interest b) Presenile dementia
c) Language d) Learning c) Opioid addiction d) Hypothyroidism
119. Korsakoff's syndrome true is- (PGI 00, UP 03, 129. When information memorized afterwards is
a) Cerebral cortex is mostly affected Jipmer 01) interfered by the information learnt earlier, it is
b) Learning is relatively normal called- (AIIMS May 04)
a) Retroactive inhibition b) Proactive inhibition
c) Retrograde memory is not affected
c) Simple inhibition d) Inhibition
d) Cannot be caused by malnutrition
130. All are true except- (PGI Feb 08)
e) Confabulation may be present
a) Procedural learning is from past experiences
120. An alcoholic presents with complaints of loss of b) Implicit learning is procedural skill acquirement
memory of recent events, impaired insight, c) Amnestic syndromes lose semantic memory
confabulation, and dementia offewmonths duration. d) Implicit memory is declarative
The patientis suffering from- (MAHE 01) e) Anterograde amnesia affects long term memory
a) Wernicke's encephalopathy more
b) Korsakoff's psychosis 131. Minimentalstatusis- (Jipmer 02)
c) Delirium tremens a) Method to investigate common psychiatric
d) Binswanger's disease problem
121. In chronic alcoholic, loss of memory & b) 30 point programme to evaluate cognitive function
confabulation is due to- (Bihar 05) c) To evaluate schizophrenia

.;~~::;s¥::~'
a) Wernick's encephalopathy d) Instrument to measure delirium
b) Korsakoff's psychosis
c) Alcoholic hallucination
d) Delirium tremens
122. Korsakoff's psychosis is diagnosed by-
a) Peripheral neuropathy (Jharkhand 06) 133. The following are the psychiatric sequelae after
b) Visual hallucination stroke in elderly- (PGI 03)
c) Impairment oflong-term memory a) Depression b) Post traumatic stress disorder
d) Seizure c) Dementia

113)c 114)d 115)a,b,d 116)a 117)c 118)d 119)e 120)b 12l)b 122)c 123)c 124)c l25)a,b,c
126)a,b,d 127)a 128)b 129)b l30)a,c,d,e 131)b 132)b 133)a,c
PSYCHIATRY [ 651 ]

134. In case of above 60 years of age psychiatric disorder 146. A boy is having diarrhoea, rhinorrhoea, sweating
in India is mostly due to- (Calcutta 2 K) & lacrimation, What is the most probable diagnosis-
a) Depression b) Dementia a) Cocaine withdrawal (AIIMS Nov 10)
c) Hysteria d) Schizophrenia b) Heroin withdrawal
135. A60year manbadundergonecardiacbypasssurgery c) Alcohol withdrawal
2 days back. Now be started forgetting things and d) LSD withdrawal
was not able to recall names and phone numbers of 147. The following symptoms may be seen in opium
his relatives. What is the probable diagnosis- withdrawal- (DELHIPGFeb. 09, Calcutta01)
a) Depression (AI 10) a) Tremors b) Lacrimation
b) Post traumatic psychosis
c) Dry nose and mouth d) Constipation
c) Cognitive dysfunction
148. Yawningisacommonfeatureof- (AJIMSMay03)
d) Alzheimer's disease
a) Alcohol withdrawal b) Cocaine withdrawal
c) Cannabis withdrawal d) Opioid withdrawal
SUBSTANCE ABUSE
149. Not a feature of heroin (smack) withdrawal- (KA 96)
136. Substance dependence is due to all except? a) Yawning b) Muscle cramps
a) Personality (AIIMS Nov 09) c) Hypersomnia d) Hypertension
b) Peer Pressure 150. Which of the following drugs is not used in opioid
c) Family history of substance abuse dependence? (AIIMS May 09)
d) Intelligence a) Disulfiram b) Clonidine
137. Which of the following is not a feature of drug c) Lorazepam d)Naltrexone
dependence- (AI 12) 151. Naltrexone is used in opioid addiction because-
a) Tolerance a) To treat withdrawal symptoms (AI 07, AIIMS
b) Persistant desire to quit drug b) To treat overdose of opioids Nov 06, May 07)
c) Consumption of Large amount of drug over c) Prevent relapse
prolonged period d) Has addiction potential
d) Use of illegal substances 152. False regarding opiate withdrawal syndrome-
138. Treatment is not required in withdrawal of- (AI 96) a) Characterized by body pain (UP 99, SGPGI 99)
a) Cannabis b) Alcohol b) Characterized by dilatation of pupils
c) Amphetamine d) LSD c) Treated with naltrexone
139. The withdrawal from all ofthe following may produce d) Treated with methadone
suicidal tendencies except -(Jipmer 96, Nimhans 97) 153. The drug which is used for long term maintenance
a) Cocaine b) Amphetamine in opioid addiction- (AIIMS Nov 06)
c) Alcohol d) Cannabis a) Naloxone b) Nalorphine
140. Symptomatic treatment is not required in withdrawal
c) Butarphanol d) Methadione
of- (AI 98)
154. Which ofthe following is an alternative to methadone
a) Cannabis b) Morphine
for maintenance treatment of opiate dependence?
c) Alcohol d) Cocaine
141. Physical dependence is not seen with- (AIIMS 95) a) Diazepam (AIIMS May 05)
a) Alcohol b) Raw opium b) Chlordiazepoxide
c) Cannabis d) Benzodiazepines c) Buprenorphine
142. Physical dependence is not seen with- (MP 97) d) Dextropropoxyphene
a) Cannabis b)LSD 155. Opiate withdrawal is treated with- (AIIMS 95)
c) Alcohol d) Benzodiazepines a) CPZ b) Nalorphine
143: Mostcofumohlyabrisedopioid;-(/11t9/PltfN/JPaitem) c) Pethidine d) Methadone
a)~:M;et:phine ' ·· ·,b,:])i~cetylljl()iphine 156. Naltrexone is used in opioid dependene to-
<,c}Oxyco~ < ' ' d)BUprell19tPliiUe · .· a) Prevent respiratory depression (AIIMS May 10)
144. All are seen in morphine poisoning except- (AI 97) b) Treat withdrawal symptoms
a) Cyanosis c) Prevent relapse
b) Pinpoint pupil d) Treatment of opioid overdose
c) Hypertension 157. Drugs used in Heroin withdrawal areA!E-(AllM'S 98)
d) Respiratory depression a) Buprenorphine b) Clonidine
145. Acute opioid withdrawl is characterized by- c) Dextropropoxyphene d) Haloperidol
a) Rhinorrhoea b) Piloerection 158. Metbadoneisindicatedin- (UP 99)
c) Miosis d) Insomnia a) Opioid overdose b) Opioid dependence
e) Constipation (PGI Dec 04) c) Opioid withdrawal d) Alcohol withdrawal

134)b 135)c 136)d 137)d 138) a,d 139) d 140)a 141)c 142)b>a 143)b 144)c 145)a,b,d 146)b
147)b 148)d 149)c I 50) a 151)c 152)c 153)d 154)c 155)d 156)c 157)d 158)b,c
PSYCHIATRY [ 652]

159. "Smack" is the "common" name for -(CUPGEE 96) 169. 46 year old chronic alcoholic, one day after abstinence
a) Heroin b) Cocaine from alcohol comes to casualty presenting with
c) Opium d) None restlessness, seizures, coarse tremors. He is not
160. All are adulterants of Heroin, except -(AIIMSMay 05) able to recognize the doctor, he think himself in a
a) Chalk powder b) Quinine jail. He also complains uncontrolable fear of being
c) Charcoal d) Fructose murdered, insomnia and insects crawling over body,
161. In an alcoholic man, Blackouts are seen in-(MH-11) probable diagnosis is- (MP 03)
a) Alcohol abstinence a) Alcoho lie seizures (rum fits)
b) Hepatic encephalopathy b) Korsakoff's psychosis
c) Alcohol intoxication c) Delirium tremens
d) Alcohol withdrawal d) Alcoholic hallucinosis
162. Most common symptom of alcohol withdrawal is- 170. In alcohol withdrawal which is not seen- (Bihar 05)
a) Bodyache b) Tremor (AI 07) a) Seizure b) Blackout
c) Diarrhoea d) Rhinorrheo c) Coarse tremor d) Hallucination
163. Features of alcohol withdrawal are all except-(A/98) 171. Withdrawal, delrium, hallucinations and amnestic
a) Epileptic seizure b) Restlessness disorder is seen with- (Calcutta 2K)
c) Hypersomnolence d) Hallucination a) Cannabis b) Opiates
c) Alcohol d) Amphetamines
172. Alcohol withdrawal is characterised by- (Mahe 99)
'g''' i#g a) Phobia b) Delusion
a ou e ium tre~ens- 05) c) Illusion d) Hallucinations
a) Clouding of consciousness b) Coarse tremors 173. Morbid jealousy is most often seen in patients taking-
c) Chronic delirious behaviour d) Hallucination a) Alcohol b)Amphetamine
e) Autonomic dysfunction c) LSD d) Cannabis (UP 97)
166. A 30 year old male with history of alcohol abuse for 174. Alcoholic Paranoia is associated with- (AI 10)
15 years is brought to the hospital emergency with a) Fixed delusions b) Hallucinations
complaints offearfulness, mis-recognition, talking c) Drowsiness d) Impulse agitation
to self, aggressive behavior, tremulousness and seeing 175. Effects of chronic alcoholism are- (Jipmer 98)
snakes and reptiles that are not visible to others a) Confabulation b) Amnesia
around bim. There is history of drinking alcohol two c) Kosarkoff psychosis d) Malnutrition
days prior to the onset of the present complaints. He e) All of the above
is most likely suffering from- (AIIMS Nov 03) 176. Wernicke-korsakoff syndrome is due to the
a) Delirium tremens b) Alcoholic hallucinosis deficiency of which ofthe following- (Jipmer 00)
c) Schizophrenia d) Seizure disorder a) Pyridoxine b) Thiamine
167. An alcoholic addicted presents in emergency with c) Vitamine B 12 d) Riboflavin
irrelevant talking & disoriented to time, place and 177. True about alcoholic dependence syndrome-
person. There is HJO not taking alcohol from last 3 a) Tolerance (PGI June 09)
days & no head injury. He also has visual b) Withdrawal
hallucination. The diagnosis- (AI 02) c) CAGE questionnaire
a) Dementia praecox b) Delirium tremens d) Physical dependence
c) Schizophrenia d) Korsakoffpsychosis 178. By which alcohol dependence is best indicated-
168. A 40 year old male, with history of daily alcohol a) Blackouts (Assam 96, AIIMS 95)
consumption for the last 7 years, is brought to the b) Early morning drinking
hospital emergency room with acute onsent of seeing c) Withdrawal symptoms
snakes all around him in the room, not recognizing d) Physical dependence
family members, violent behavior and tremulousness 179. All are anticraving agent for alcohol except-
for few hours. There is history of his having missed a) Lorazepam b) Naltrexone (AIIMS May 09)
the alcohol drink since 2 days. Examination reveals c) Topiramate d) Acamprosate
increased blood pressure, tremors, increased 180. Which of the following is used to maintain abstinence
psychomotor activity, fearful affect, hallucinatory in alcohol dependence? (DPG 09)
behaviour, disorientation, impaired judgement and a) Naltrexone b) Clonidine
insight. He is most likely to be suffering from-(AI 05) c) Disulfiram d) Naloxone
a) Alcoholic hallucinosis 181. All of the following agents are used in the treatment
b) Delirium tremens ofalcohol dependence, except- (AI 11)
c) Wernicke encephalopathy a) Flumazenil b) Acomprosate
d) Korsakoff's psychosis c) Naltrexone d)Disulfiram

159)a 160)c 16l)c 162)b 163)c 164)a 165)a,b,d,e 166)a 167)b 168)b 169)c 170)b l71)c 172)d
173)a 174)a 175)e 176)b 177)a,b,c,d 178)c 179)a 180)a 18l)a
PSYCHIATRY [ 653]

182. A 40 year old man presents to casualty with history 196.


of regular and heavy use of alcohol for ten years and
morning drinking for one year. The last alcohol
intake was three days back. There is no history of
head injury or seizures. On examination, there is
no icterus, sign of hepatic encephalopathy or focal
neurological sign. The patient had coarse tremors, 197.
visual haHucinations and had disorientation to time.
Which of the foHowing is the best medicine to be
prescribed for such a patient ? (AI 04)
a) Diazepam b) Haloperidol
c) Imipramine d)Naltrexone
183. Drug of choice in delirium tremens is- (SGPGI 04,
a) Diazepalm b) Phenytoin AI 95)
c) Chlordiazepoxide d) Morphine
184. In an uncomplicated case of alcohol withdrawal, the
drug which can be given safely is- (AIIMS May 08) 200.
a) Benzodiazepines b) Clonidine
c) Morphine d) Disulfiram
185. T/tofalcoholwithdrawal- (PGI June 07)
a) B-blocker b) BZD
c) Amitryptiline d) SSRis
186. Inalcoholwithdrawal,drugofchoiceis-(DEUf!PGFeb.
a) 1EP b) Chlormethazole 09) 201.
c) Chlordiazepoxide d) Buspirone
187. Alcohol intoxication is a form of- (Maharashtra 03)
a) Psychosis b) Anxiety neurosis
202.
c) Dependence d) Withdrawal syndrome
188. Psychiatric complications of alcohol dependence -
a) Anxiety b)Suicide (PGI01)
c) Depression d) Schizophrenia
e) Mania 203.
\l~~:'.f50Jt «lWPi.gl1ttstiliijJ:zcoMultW~loij'~~a.Ji:be
· · ·.··'
::~-:_- ·;-:~; -tl< .
··.·.··· ,,;: '"'·''' • .:,..ini!J;Tii:?ir.fff..irt/.JiYt.;. "rj<'
~- ,, ~~-~-~-~~-/~"t-·~::-t-:,::p . _t.~~~~P!'£1:~1,:~~-- :J{··f;Z:.;-~"'nJ ;

~.,;k~l~'t .. . . ..
190. Formication and delusion of persecution, both are
:·.,:;:<·.:A';;t 204.

together seen in- (AIIMS May 11, 09)


a) LSD psychosis b) Amphetamine psychosis
c) Cocaine psychosis d) Cannabis psychosis 205.
191. Paranoid Psychosis with cocaine is seen due to-(A/12)
a) Tolerance b) With drawl
c) Reverse Tolerance d) Intoxication
192. Jet black pigmentation of tongue with tactile and 206.
visual hallucination is feature of which poisoning-
a) Cocaine b)Arsenic (RJ98)
c) Cannabis d) Heroin 207.
193. 'Flash-back' phenomenon is seen with the use of-
a) Cannbis b) Psilocybin (KA 99)
c) LSD d) All of the above
194. Which is associated with flashback phenomenon-
a) Cannabis b)Cocaine(Burwan2K)
c) Amphetamine d) Heroin
195. Amotivational syndrome is seen in- (MH 10, TN 99)
a) Cannabis b) Cocaine
c) Amphetamine d) LSD

182)a 183)c 184)a 185)b 186)c 187)a 188)a,b,c 189)b 190)c 191)d 192)a 193)d 194)a 195)a
196)b 197)c 198)a 199) b,d 200) b 201)d 202) c 203) b 204) b 205) a,b,c,e 206) b,c,d 207)a 208) b
PSYCHIATRY [ 654]

SCHIZOPHRENIA AND OTHER PSYCHOTIC 221. Most specific of psychosis- (AIIMS 95)
DISORDERS a) Neologism b) Incoherence
c) Pressure of speech d) Persveration

~2:~---!~~:;~
209. Lack ofinsight is not a feature of-(AI 05,SGPGI 03)
a) Panic disorder b) Schizophrenia
c) Mania d) Reactive psychosis
210. Which ofthe following features is not included in 223. The term catatonia is coined by- (CIP 01)
psychosis- (PGI June 04) a) Kahlbaurn b) Freud
a) Panic attack b) Delusion c) Maxwelljones d)Adler
c) Hallucination d) Hypochondriasis 22~.. .. Term schi,qpli,~t~lii~"Va§,coinea~:i: . '(Nlt~TIDNB
211. Impaired insight is found in- (PGI 97)
a) Traumatic psychosis
b) Schizophrenia
-~~--~r~;t~;,·,:, ),:·;·~.J~t~=~~~~4·. ·Putt~.:~
225. One of the following drug abuse produces a psychosis
c) Anxiety neurosis
closely resembling paranoid schizophrenia-(MP 98,
d) Obsessive compulsive neurosis
212. Impaired insight is evident in- (AIIMS 92) a) Barbiturates b)AmphetaminesJipmer 11)
a) Psychosomatic disorder c) Opioids d) Benzodiazepines
b) Anxiety neurosis 226. Paranoid schizophrenia is caused by- (AIIMS 97)
c) Post traumatic stress disorder a) L-Dopa b) Fluoxetine
d) Schizophrenia c) Amphetamine d) LSD
213. Differentiating features b/w neurosis and psychosis, 227. Which does not produce psychotic effects-
In psychosis - (MP 01) a) Amphetamine b) Cocaine (Punjab 11)
a) Insight is preserved c) Ketamine d) Marijuana
b) Lack of insight 228. Incidence of schizophrenia in India- (TN 99)
c) Personality and behavior preserved a) 1-5 per 1000 b) 0.15-0.5 per 1000
d) None c) 5-10per1000 d)10-15per1000
214. Difference between neurosis and psychosis is- 229. Percentage of monozygotic twins with schizophrenia
a) Severity b) Insight (AI 95) is- (MP04, WB03, UPOJ)
c) Clinical features d) Duration a) OJ% b) 1%
215. Lack ofinsight is not a feature of- (AIIMS NOV 02) c) 10% d)50%
a) Panic disorder b) Schizophrenia 230. Biochemical abnormalities in schizophemia-
c) Mania d) Reactive Psychosis a) Increased dopamine (PGI 02)
216. Patient presents with altered behaviour, delusions b) Increased dopamine and increased serotonin
& hallucination suggest- (AI 97) c) Decreased dopamine
a) Psyshotic disorder b) Confirms schizophrenia d) Increased dopamine and decreased serotonin
c) Korsakoff's psychosis d) Depression 231. The most commonly implicated neurotransmitter
217. A patient presents with one month history of in the etiology of schizophrenia is - (SGPGI 04)
abnormal hallucination and delusion. The patient a) Norepinephrine b) Serotonin
lacks insight to his changed behaviour. Likely c) Dopamine d) GABA
diagnosis is - (AI 97) 232. Neurotransmitter related to schizophrenia
a) Paronia b) Depression pathology is- (PGI 97)
c) Psychosis d) Schizophrenia a) Ach b) Dopamine
218. Delusions are seen in all of the following, c) Serotonin d) NA
Except- (AI 12) 233. All ofthe following neurotransmitters are suspected
a) Shizophrenia b) Depression of being involved in the pathophysiology of
c) Dementia d) Conversion Disorder schizophrenia except- (MAHE 95, UP OJ)
219. Which of the following feature is included in a) Prostaglandins E b) Norepinephrine
psychosis - (PGI June 06) c) Ascorbic acid d) 5-HT
a) Panic attack b) Delusion 234. Schizophrenia mostly occurs in- (DPG 08, AIIMS 99)
c) Hallucination d) Hypochondriasis a) Adolescents b) Children
e) Emotional withdrawal c) Middle age d) Old age
220. A psychotic disorder most commonly pertains to 235. Which is more appropriate in a case of schizophrenia-
which of the following symptoms? a) Low socioeconomic group (PGI 2000)
a) Weeping or laughing (DELHI PG Mar. 09) b) Seen in adolescents
b) Agitation or retardation c) Common in primitive societies
c) Obsessions or compulsions d) Mfluent society influences the incidence
d) Delusions or hallucinations

209)a 210)a 211)a,b 212)d 213)b 214)b>c 215)a 216)a 217)c 218)d 219)b,c 220)d 221)a 222)c
223)a 224)c 225)b 226)c 227) None 228) b 229) d 230) b 231) c 232) b,c,d 233) a,c 234) a 235) a,b
PSYCHIATRY [ 655]

236. Expressed emotionality is related to- (MH 10) 248. Somatic passivity is a feature of- (AIJMS Nov 11)
a) Depression b) Schizophrenia a) Depressive illness b) Paranoid Schizophrenia
c) Mania d) Somatiformdisorder . c) Hypochondriasis d) Panic disorder
237. Bleur's symptoms of schizophrenia include AlE- 249. True about schizophrenia- (PGJ 03)
a) Autism (PGI Dec 05) a) Thought broadcasting
b) Automatism b) Third person hallucination
c) Affect disturbance c) Makes violence
d) Elated mood
d) Loosening of association
e) Takes good self care
e) Ambivalence
250. Delusions in clear consciousness are seen in -
238. Eugene Bleuler's four' A' of schizophrenia are all a) Dementia b)Delirium (AMU 98)
except- (Maharashtra 10, 07) c) Schizophrenia d) Neurosis
a) Autism b) Affect 251. Auditory hallucinations, features of affect, delusion
c) Anhedonia d) Association of persecution occur in- (TN 95)
s '-'-'"·-·~'-':""'
a) Depression b) Hysteria
c) Schizophrenia d) Anxiety neurosis
252. Schizophrenia is a disorder of- (TN 03)
a) Thought b) Belief
c) Memory d) Perception
253. In schizophrenia, characteristic feature is-
a) Formal thought disorder b) Delusion (PG/97)
c) Hallucination d) Apathy
254. Schizophrenia is characterized by- (Bihar 03)
a) Formed through disorder b) Hallucination
c) Delusion d)All
255. Schizophrenia is characterized by AlE-
a) Delusion (AIIMS 00, 98)
b) Auditory hallucination
c) Elation
242. All of the following are schneider first symptoms d) Catatonia
except- (MP 06) 256. Which of the following hallucinations is
a) Hallucination b) Delusion pathognomonicofschizophrenia- (DPG 03)
a) Auditory hallucinations commanding the patient
c) Passivity phenomenon d) Elation
b) Auditory hallucinations giving running commentry
243. The following is a Schneider's first rank symptom-
c) Auditory hallucinations criticizing the patient
a) Persecutory delusion
d) Auditory hallucinations talking to patient
b) Voices commenting on actions
257. 18 years old male hears voices discussing him in
c) Delusion of guilt
3rd person has- (AJIMS 95)
d) Incoherence
a) Obsession b) Depression
244. Which is first rank symptom described by- (TN 95)
c) Mania d) Schizophrenia
a) Echolalia b) Thought insertion 258. Feature (s) suggestive of schizophrenia rather than
c) Autism d) Suicidal tendency organic psychosis is/are- (PGI June 09)
245. Which offollowing is not a schneider's f'rrst rank a) Third person hallucination b) Split personality
symptom- (Delhi 97) c) Visual hallucination d) Altered sensorium
a) Passivity phenominon e) Systemized delusion
b) Auditory hallucinations 259. Characteristic clinical manifestation of
c) Delusions of self reference schizophrenia is- (PG/98)
d) Delusional perception a) Confusion b) Anxiety
246. Schneider's 1st rank symptoms seen iu-(PG/June 04) c) Auditory hallucinations d) Visual hallucinations
a) Delusion b) Hallucination 260. Most common hallucination iu schizophrenia is-
c) Schizoid personality d) Schizophrenia a) Auditory b) Visual (Jipmer 95, CMC 99)
247. Which of the following is NOT a first rank symptom c) Olfactory d) Tactile
ofschizophrenia? (Karn. 11) 261. Schizophrenia false is- (AIIMS Nov 07)
a) Echo de la Pensee a) 3'd person auditory hallucination
b) Passivity experience b) Inappropriate emotions
c) Delusions of persecution c) Sustained mood changes
d) Broadcasting of thoughts d) Formal thought disorder

236)b 237)b 238)c 239)a,b,e 240)a 241)d 242)d 243)b 244)b 245)c 246)d 247)None 248)b
249) a,b,c 250) c 251) c 252) a >d 253) b,c,d 254) d 255) c 256) b 257) d 258) a 259) c 260) a 261) c
PSYCHIATRY [ 656]

262. Hallucinations in schizophrenia are characterized c) Hebephrenic Schizophrenia


byallexcept- (Maharashtra 02) d) Simple Schizophrenia
a) Hallucinations commanding and controlling 272. Following are features of catatonic schizophrenia,
action of the person except- · (MP 2000)
b) Hallucinations as singing songs a) Mutism
c) Hallucinations almost always continuous b) Echolalia
d) Hallucinations commenting on actions of the c) Waxyflexibility
person d) Deep tendon reflexes are increased
263. In a young male, history of firm unshakable belief 273. Features like increased psychomotor activity, waxy
of hands passing through the body & gripping his flexibility at time are seen classically in - (TN 97)
lungs indicates- (Maharashtra 03) a) Simple schizophrenia
a) Schizophrenia b) Depression b) Hebephrenic schizophrenia
c) Somatization d) Asthma c) Catatonic schizophrenia
264. A 28 year old female presents with complains of d) None of the above
reduced sleep with disorganized behaviour since 9 274. In which type of schizophrenia is grimacing a
months. She feels there is a camera fixed behind her feature of- (Karnataka 98)
head which always follows her. Which of the following a) Juvenile b) Hebephrenic
is likely diagnosis - (MH-11) c) Paranoid d) Catatonic
a) Psychotic depression 275. All are features of catatonia except- (Bihar 03)
b) Paranoid schizophrenia a) Automatic obedience b) Cataplexy
c) Delusional disorder c) Catalepsy d) Negativism
d) Insomnia 276. Catatonia is associated with - (Nimhans 01)
265. Most common type of schizophrenia- (AJIMS 95, a) Hebephemic b) Catatonic ·
a) Simple Jharkhand 04, Nimharzs 01, c) Paranoid d)Undifferentiated
b) Hebephrenic PGI Dec 04) 277. Waxy flexibility is characteristic of-
c) Catatonic a) Excitatory catatonia (Orissa 04, Jharkhand 06)
d) Paranoid b) Stuporous cataonia
266. In schizophrenia early onset with poor prognosis is c) OCD
seen in - (AI 96) d) All
a) Simple b) Hebephrenic 278. All of the following are true about paranoid
c) Catatonic d) Paranoid schizophrenia except- (MP 97)
267. Which subtype of schizophrenia carries the best a) Most common type of schizophrenia
prognosiS - (MP 04) b) Onset in 3'd /4th decade
a) Undifferentiated b) Catatonic c) Delusions of grandeur
c) Pranoid d) Hebephrenic d) Rapid deterioration of personality
268. Which of the following is associated with bad 279. Delusions of control, persecution and self reference
prognosis- (DPG 09) are seen in - (PGI 97)
a) Catatonic schizophrenia a) Paranoia b) Paranoid schizophrenia
b) Hebephrenic schizophrenis c) Mania d) OCD
d) Paranoid schizophrenia 280. Grossly disorganised, severe personality
c) Undifferentiated deterioration and worst prognosis is seen is-
269. Defect of conation is typically seen in - a) Hebephrenic b) Catatonic (Orissa 07)
a) Simple shcizophrenia (PGI 97, AJIMS 96, UP 06) c) Simple d) Paranoid
b) Hebephrenia schizophrenia 281. Schizophrenia and depression both have AlE -
c) Catatonic schizophrenia a) Formal thought disorder (PGI 02)
d) Paranoid schizophrenia b) Social withdrawal
270. In catatonic schizophrenia which is not found- c) Poor personal care
a) Waxy flexibility (PGI Dec 08) d) Concrete thinking
b) Automatic obedience 282. Van Gogh syndrome is seen in- (PGI 03)
c) Somatic passivity a) Mania b) Depression
d) Gegenhalten c) Schizophrenia d) OCD
e) Hallucination
271. Patient presents with waxy flexibility, negativism &
rigidity diagnosis is- (Bihar 04, AI OJ)
a) Paranoid schizophrenia
b) Catatonic schizophrenia

262) c 263) a 264) b 265) d 266) b 267) b 268) b 269) c 270) c 271) b 272) d 273) c 274) d 275) b
276)a,b,c,d 277)b 278)d 279)b 280)a 281)a 282)c 283)d
PSYCHIATRY [ 657]

284. Trueaboutlateonsetschizophrenia-(AJ/MSNov 10) to himself and sometimes muttering to bimselfloudly.


a) Onset after 45 yrs The likely diagnosis is- (AIIMS May 02)
b) Onset between 25- 30 yrs a) Schizophrenia b) Conversion disorder
c) Prognosis is poor c) Major depression d) Delusion
d) Olfactory halucinations are common 295. Kalin a 24 year old occasional alcoholic had got a
285. All of the following are considered to be negative change in behaviour. He has become suspicious that
symptoms ofSchizophrenia EXCEPT -(COMED 09) people are trying to conspire against him, though
a) Apathy b)Ambivalence his father states that there is no reason for his fears.
c) Auditory hallucinations d) Affective flattening He is getting hallucinations of voices commenting
286. 1yPe two schizophrenia is characterized by all except-
on his actions what is most probable diagnosis-
a) Negative symptoms (AIIMS Nov 08)
a) Delirium tremens (CIP 01, AIIMS 00)
b) Poor response to TT
b) Alcohol induced psychosis
c) Disorganised behaviour
c) Schizophrenia
d) CT scan abnormal
287. All of the following are associated with better d) Delusional disorder
prognosis in schizophrenia except- (AI 06) 296. A 60 year old male suffering from auditory
a) Late onset b)Married hallucination says that people staying upwards are
c) Negative symptoms d) Acute onset talking about him and conspiring against him. He
288. Bad prognostic indicator of Schizophrenia is- dropped a police compliant against them but
a) Late onset (AIIMS 97) the allegations were proved to be wrong. The
b) Family history of Schizophrenia diagnosis is- (AIIMS OJ)
c) Positive precipitating factors a) Depression b)Dementia
d) Prominent affective symptoms c) Delusional disorder d) Schizophrenia
289. Bad prognostic factor for schizophrenia is - 297. Lallu 40 years has recently started writing books. But
a) Late onset of disease (AIIMS 97) the matter in this book could not be understood by
b) Catatonia anybody; since it contained words which were never
c) Presence of depression there in any dictionary and the theme was very
Absence of disjoint. Nowadays be has become very shy and self
absorbed. When he addresses people, bespeaks about
meta philosophical ideas. What is likely diagnosis-
a) Mania b) Schizophrenia (AIIMS 00)
c) A genius writer d) Delusional disorder
298. A patient complains that people living upstairs are
ass1ocilatE~d ••••••• personalities -
always talking about him and conspires against him.
b) Psthesis (AIIMS 97)
His son complains about his weired behaviour like
c) Asthenic d) All
keeping shoes in Fridge and wearing vest over shirt.
292. Which of the follwoing is the most common cause of
The likely diagnosis is - (AIIMS 99)
prematuredeathinschizophrenia- (AI 11)
a) Homicide a) Depression b) Delusional disorder
b) Suicide c) Dementia d) Schizophrenia
c) Toxicity of antipsychotic drug 299. A 60 year old man is brought to a psychiatrist with a
d) Hospital acquired infection 10 year history, that be suspects his neighbours and
293. A 23 year old engineering student is brought by his he feels that whenever he passes by they sneeze and
family to be hospital with history of gradual onset of plan against behind his back. He feels that his wife
suspiciousness, muttering and smiling without clear bas been replaced by a double and calls police for
reason, decreased socialization, violent outbursts, help. He is quite well groomed alert, occasionaly
and lack of interest in studies for 8 months. Mental consumes alcohol, likely diagnosis is-(AIIMSMay 02)
status examination revealed a blunt affect, thought a) Paranoid personality b) Paranoid schizophrenia
broadcast, a relatively preserved cognition, impaired c) Alcohol withdrawal d) Conversion disorder
judgment and insight. He is most likely to be 300. A 70 year old male, Babulal was brought to the
suffering from- (AIIMS May 04) hospital with the hi!!tory of auditory hallucinations
a) Delusional disorder b) Depression with third person hallucinations. Be has no history
c) Schizophrenia d) Anxiety disorder ofsimilar problems previously. The diagnosis is most
294. A patient is brought with 6 month history of odd likely to be- (AIIMS 0 I)
behaviour. There is history ofa family member having a) Dementia b) Delusional disorders
disappeared some years back. He seems to be talking c) Schizophrenia d) Acute psychosis

284)a 285)c 286)c 287)c 288)b 289)None 290)b 29l)c 292)b293)c 294)a 295)c 296)d 297)b
298)d 299)b 300)c
PSYCHIATRY [ 658]

301. A 16yearold boy does not attend school because of behavior. The most likely diagnosis is- (AI 10)
the fear of being harmed by school mates. He thinks a) Paranoid personality disorder
that his classmates laugh at and talk about him. He b) Persistent delusion disorder
is even scared of going out to the market. He is most c) Schizophrenia
likely suffering from- (AI 04) d) Acute and transient psychosis
a) Anxiety neurosis 312. A 41 year old woman working as an executive director
b) Manic depressive psychosis is convinced that the management has denied her
c) Adjustment reaction promotion by preparing false reports about her
d) Schizophrenia competence and have forged her signature on
302. A patient of schizophrenia on neuroleptics, his
sensitive documents so as to convict her. She files a
psychotic symptoms get relieved but developed
complaint in the police station and requests for
sadness, talks less to others, remain on bed, all of
security. Despite all this she attends to her work and
following are likely causes except- (AIIMS 2000)
manages the household. She is suffering from-
a) Parkinsonism
b) Major depression a) Paranoid schizophrenia (AI 04)
c) Negative symptoms are still persisting b) Late onset psychosis
d) He is reacting to external stimuli c) Persistent delusional disorder
303. Drug treatment of schizophrenia are- (PGI 03) d) Obsessive compulsive disorder
a) Chlordiazepoxide b) Trifluperazine 313. A man hits his neighbour next day be feels that police
c) Clozapine d) Rivastigmine is behind him and his brain is being controlled
e) Haloperidol by radio waves by his neighbour. The probable
304. A patient of schizophrenia on CPZ (chlorpromazine) diagnosis is- (AI 2000)
develops auditory hallucination again. The next drut a) Personality disorder
to be given is- (AI 2000) b) Passivity feeling
a) Haloperidol b) Clozapine c) Delusion of persecution
c) Sulpiride d) Tianeptin d) Organic brain syndrome
305. A patient of depression is getting chlorpromazine, 314. A 30 year old unmarried woman from a low
but his auditory hallucination are not controlled. socioeconomic status family believes that a rich boy
Tbenextdruggivenis- (AIIMS 1999) staying in her neighbourhood is in deep love with
a) Haloperidol b) Clozapine her. The boy clearly denies his love towards this lady.
c) Sulpiride d) Tianeptin Still the lady insists that his denial is a secret
306. Drugs used in schizophrenia include- (PGI 02) affirmation of his love towards her. She makes
a) Chlorpromazine b) Haloperidol desperate attempts to meet the boy despite resistance
c) Olanzapine d) Imipramine from her family. She also develops sadness at times
e) Risperidone when her effort to meet the boy does not materialize.
307. Schizophrenia can be treated with all of the following She is able to maintain her daily routine. She
except- (MP 04) however, remains preoccupied with the thoughts of
a) Pemoline b) Olanzapine this boy. She is likely to be suffering from- (AI 04)
c) Sulpiride d) Chlorpromazine a) Delusional disorder b) Depression
308. Atypical antipsychotic are all except?(AIIMSMay 08) c) Mania d) Schizophrenia
a) Olanzepine b) Clozapine 315. A 30 year old unmarried woman of average socio-
c) Risperidone d) Thioridazone economic background believes that her boss is in
309. Antipsychotic drugs are- (Manipal 08) secretly love with her. She rings him up at odd hours
a) Clomipramine b) Carbamazepine and writes love letter to him despite his serious
c) Sodium valproate d) Chlorpromazine warning not to do so. She holds this belief despite
310. Resperidone is most commonly used to treat which contradiction from family members & his denial.
of the following disorders? (DELHI PG Mar. 09} However she is able to manage her daily activities as
a) Dementia b) Depression before. She is most likely to be suffering from-
c) Schizophrenia d) Obsessive-compulsive disorder a) Depression (AIIMS Nov 04)
311. A 30 year old man since 2 months suspects that his b) Shizophrenia
wife is having an affair with his boss. He thinks his c) Delusional disorder
friend is also involved from abroad and is providing d) No psychatric ailment
technology support. He thinks people talk ill about 316. Delusion that someone from high socio economic
him. His friends tried to convince him but be is not status is loving you is in- (PGI 98)
convinced at all. Otherwise be is normal, be doesn't a) Othello syndrome b) Capgras syndrome
have any thought disorder or any other inappropriate c) De clerambault syndrome d) Franklin syndrome

30l)d 302)d 303)b,c,e 304)b 305)b 306)a,b,c,e 307)a 308)d 309)d 310)c 311)b 312)c 313)c 314)a
315)c 316)c
PSYCHIATRY [ 659]

317. A person of 35 years is having firm belief about 324. A 20 year old boy c/o hearing of voices, aggressive
infidelity involving the spouse. And he never allow behavior since 2 days. He has fever since 2 days.
her to go out of home alone. He often locks his house, When asked to his family, they says that be has been
while going office. Inspite ofall this he is persistantly muttering to self and gestic~lating. There is no hlo
suspicious about her character. Tbe probable of psychiatric illness. Likely diagnosis is- (AIIMS
diagnosis is - (AIIMS 99) a) Dementia b) Acute psychosis Nov 1 0)
a) Schizophrenia c) Delirium d) Delusional disorder
b) Delusional parasitosis 325. Folie a deux is seen in- (PGI 99)
c) Clerambault's syndrome a) Hysteria b) Paranoid
d) Othello syndrome c) CXD d) Neurasthenia
318. lnfidility & jealousy involving spouse is the thought 326. Maximumheritabilityisseenin-(MP04, WB03, UPOJ)
content of which disorder- (PGI 99, AI 98) a) Depression b) Mania
a) Capgras syndrome c) Schizophrenia d) Panic disorder
b) Othello syndrome 327. Strokeiscommonin- (Jipmer 11)
c) Hypochondria! paranomia a) Mania b) Depression
d) Declerambault's syndrome c) Bipolar disorder d) Schizophrenia
319. A 25 year old housewife came to the Psychiatry out 328. Sudden onset of hallucinations in a 35 year old man
patient department(OPD) complaining that her nose with no previous history & relevant family history
was longer than usual. She feltthat her husband did maybedueto- (Jipmer 11)
not like her because ofthe deformity and had a) Substance abuse b) Schizophrenia
developed relation-ship with the neighbouring girl. c) Frotterism d) Depression
Further she complained that people made fun of her.
It was not possible to convince her that there was no MOOD DISORDERS
deform-ity. Her symptoms include- (AI 04)
a) Delusion b) Depersonalization 329. Most common age for depression is-
c) Depression d) Hallucination a) Middle age men (Jharkhand 05, AI 96, 98)
320. A patient came with complaints ofhaving a deformed b) Middle age female
nose and also complained that nobody takes him c) Younggirl
seriously because of the deformity of his nose. He d) Children
has visited several cosmetic surgeons but they have 330. True about major depression is/are-
sent him back saying that there is nothing wrong a) Common in males (Maharashtra 06)
with his nose. He is probably sufferingfrom-(AliMS 0J) b) Markedly diminished interest in all activities most
a) Hypochondriasis b) Somatization of the day but no fatigue or loss of energy or
c) Delusional disorder d)OCD psychomotor retardation.
3 21. Basanti 27 years old female thinks her nose is ugly. c) Feeling of worthlessness or excessive guilt
Her idea is fixed and not shared by anyone else. d) All
Whenever she goes out of home she hides her face. 331. True about "Major depression" include the following
She visits a surgeon for plastic surgery. Next step except- (Maharashtra 08)
would be- (AI OJ) a) Two times more common in men than women
a) Investigate & then operate b) Incidence increases with increasing age
b) Reassure the patient c) In females affected age is adulthood
c) Immediate operation d) Is genetic origin
d) Refer to psychiatrist 332. Daily depressed mood for minimum --- weeks is
322. Delusionofdoublesisfoundin- (AIIMS 99) must to classify someone as suffering from major
a) Schizo affective disorder depression disease- (Maharashtra 09)
b) Capgras syndrome a) 1 b)2
c) Reactive psychosis c) 3 d)4
d) Paranoid schizophrenia 333. Nihilistic ideas are seen in- (SGPGI OJ, AIIMS 98)
323. Ramo, a 22 year old single unmarried man is a) Depression b) Schizophrenia
suffering from sudden onset of 3rd person c) Mania d) OCN
hallucination for the past 2 weeks. He is suspicious 334. A lady always thinking that her stomach is rotten
of his family members and had decreased sleep and decreased appetite, having hallucination, thinking
appetite. The diagnosis is - (AIIMS 01) about world is going to be end. Diagnosis is -
a) Schizophrenia b) Acute Psychosis a) Psychosis b) Mania (Bihar 06)
c) Acute mania d) Acute delirium c) Depression d) None

317)d 318)b 319)a 320)c 32l)d 322)b 323)b 324)b 325)b 326)c 327)d 328)b 329)b 330)c
331)a 332)b 333)a 334)c
PSYCHIATRY [ 660]

335. True about psychotic feature in depression- 343. Intense nihilism, somatization and agitation in old
a) Found in severe depression (PGI Dec 04) age are the hallmark symptoms of- (AI 06)
b) Found in moderate depression a) Involutional melancholia b) Atypical depression
c) Mood incongruent psychotic feature c) Somatized depression d)·Depressive stupor
d) Cyclothymia 344. A 50 year old male presents with a 3 year history
e) Dysthymia of irritability, low mood, lack of interest in
336. "Nihilistic delusions" are seen in- (PGI 00) surroundings and general dissatisfaction with
a) Endogenous depression everything. There is no significant disruption in
b) Double depression his sleep or appetite. He is likely to be suffering
c) Depression in invoiutional stage from- (AIIMS May 05)
d) Cyclothymia
a) Major depression b) No psychiatric disorder
e) Dysthymia
c) Dysthymia d) Chronic fatique syndrome
337. Major psychosis is- (Kerala 95)
345. Dysthymiais- (Jipmer 02)
a) Neurosis
b) Dementia a) Chronic mild depression
c) Reactive depression b) Chronic severe depression
d) Endogenous depression c) Bipolar disorder
338. A 68 years old lady thinks that she has done a sin, d) Personality disorder
she is not worth to live this world, she is also having 346. Neurotic depression may be characterised by all
anorexia and insomnia, she is suffering from- except- (UPSC 2K)
a) Exogenous depression (SGPGI OJ) a) Ravenous appetite b) Hypersomnia
b) Endogenous depression c) Increased libido d) Weight gain
c) Neurotic depression 34 7. Delusion of Nihilism and Early morning insomnia
d) Dissociate disorder are characteristic features of- (AI 97)
a) Mania b) Major depression
c) Personality disorder d) Schizophrenia
348. All of the following are feature of depression except-
year a) Nihilistic delusions (MP 97)
history of feeling low, lack of interest in activities, b) Diurnal variation in mood
lethargy, multiple body-aches, ideas of worthlessness, c) Suicidal tendency
decreased appetite and disturbed sleep with d) Elation
early morning awakening. She is likely to benefit 349. Which of the following has to be present for
form- (AJIMS May 04) diagnosis of major depressive disorder-
a) Anti-psychotics b) Anti-depressants a) Loss of interest or pleasure (MH 10, 0 7)
c) Anxiolytics d) Hypno-sedatives
b) Recurrent suicidal tendency
341. A middle aged man presented with pain in back, lack
c) Insomnia
of interest in recreational activities, low mood,
d) Indecisiveness
lethargy, decreased sleep and appetite for two
months. There was no historysnggestive of delusions 350. Early morning awakeness is a feature of- (MH 03)
or hallucinations. He did not suffer from any chronic a) Psychosis b) Anxiety neurosis
medical illness. There was no family history of c) Depression d) Mania
psychiatric illness. Routine investigations including 351. Disruption or disorganization of biological rhythm
haemogram, renal function tests, liver function tests is observed in- (MH 09)
electrocadiogram did not reveal any abnormality. a) Schizophrenia b) Anxiety
This patient should be treated with- (AI 03, AIIMS c) Depression d) Mania
a) Haloperidol b) Sertraline May 03) 352. Early morning awakening and reduced latency
c) Alprazolam d) Olanzapine of REM sleep is suggestive of- (MH-11)
342. A 41 year old woman presented with a history of a) Delirium b) Depression
aches and pains all over the body and generalized c) Schizophrenia d) Anxiety
weakness for four years. She cannot sleep because 353. Otto veraguth sign is found in- (PGI 03)
of the illness and has lost her appetite as well. She a) Mania b) Anxiety
has lack of interest in work and doesn't like to meet c) <XD d) Depression
friends and relatives. She denies feelings of sadness. 354. Suicidal tendencies seen in- (PGI Dec 06)
Her most likely diagnosis is- (AIIMS Nov 02) a) Depression b) Posttraumatic stress disorder
a) Somatoform pain disorder b) Major depression c) Schizophrenia d) Substance abuse
c) Somatization disorder d) Dissociative e) Anxiety

335)a,c 336)a,b,c 337)d 338)b 339)b 340)b 34l)b 342)b 343)a 344)c 345)a 346)a,c 347)b 348)d
349)a 350)c 351)c 352)b 353)d 354)a,c,d
PSYCHIATRY [ 661 ]

355. Most common mental disorder as a cause of 368. Blood of 45 yrs old male shows -HVA. This patient is
suicide- (Jipmer 02, SGPGI 03) most likely to show- (AIIMS Nov 08)
a) Mania b) Depression a) .P<'r b) Schizophrenia
Alcohol dependence d) Schizophrenia c) Depression d) Parkinson's ds
369. Depression is seen in - (SRMC 02)
a) Hyperthyroidism b) Hypoglycemia
c) Adrenal disorder d) Pheochromocytoma
357. Increase in suicidal behavior is due to- (MP 06) 370. Depression is a feature of which of the following
a) Increase in serotonin condition- (UP OJ)
b) Decrease in serotonin a) Hypopituitarism b) Hyperthyroidism
c) Increase in nor-adrenaline c) Hypothyroidism · d) Hypoglycemia
d) Reactive depression 371. Depression is not caused by- (UP 99)
358. Increased suicidal tendency is associated with- a) Clonazepam b) Levodopa
a) Noraderenaline b) Serotonin (AIIMS May 08) c) Metronidazole d) Corticosteroid
c) Dopamine d)GABA 372. Depression is not a side effect of- (MP 07)
359. Suicidal tendencies are most common in- a) Propranolol b) Oral contraceptives
a) Involutional depression (SGPGI 00, PGI 99) c) L-dopa d) Flupenthixol
b) Reactive depression 373. Most common type of post puerperal psychosis is-
c) Psychotic depression a) Depression b)Anxiety (PGI 99)
d) Childhood depression c) Mania d) Suicide
360. Suicidal risk is common with which type ofdepression- 374. Commonest psychological feature of AIDS-
a) Reactive depression (PGI 99) a) Depression b)Mania(CUPGEE95)
b) Endogenous depression c) Suicidal tendency d) None (AIIMS 96)
c) Childhood depression 375. Most common type of postpartum psychosis is-
a) Manic b) Depressive (DPGEE 08)
d) Depression in involution
c) Organic d) Hallucination
361. Most common cause of suicide is- (Kamataka 2000)
376. Not true about major depressive disorder- (PGI 03)
a) Childhood depression
a) Commonly seen in female
b) Depression in evolutional stage
b) Recovery is complete after treatment
c) Psychiatric depression
c) Associated with hypothyroidism
d) Reactive depression
d) Family H/0 major depression
362. Suicide is most commonly associated with which of
377. A 40 yr old female patient present with history of
thefollowingdisorders? (DELHI PG Mar. 09)
depressed mood, loss of appetite insomnia and no
a) Anxiety
interest in surroundings for the past 1 yr. These
b) Depression
symptoms followed soon after a business loss 1 yr
c) Obsessive-compulsive disorder back.Which of the following statements is true
d) Post-traumatic Stress disorder regarding the management ofthis patient- (AI 11)
363. Suicidal tendency seen in- (PGI 2002) a) No treatment is necessary as it is due to business
a) Schizophrenia b) PTSD loss
c) Depression d) OCD b) SSRI is the most efficacious of the available drugs
e) Acetylcholine c) Antidepressant treatment based on the side effect
364. Incidence ofsuicide is- (PGI June 05) profile of the durgs
a) 8-10/100 population d) Combination therapy of 2 antidepressant drugs
b) 8-10/10000 population 378. Drug of choice in depression in old person is-
c) 8-1 0/llac population a) Fluoxetine b) Buspirone (AI 96)
d) 8-10/1 billion population c) Amitryptyline d) Imipramine
365. Neurotransmittors involved in depression are- 379. Rathi, 26 years old female diagnosed to be suffering
a) GABAanddopamine (AI 95) from depression. Now for the past 2 days had suicidal
b) Serotonin and norepineprine tendency, thought & ideas. The best treatment is-
c) Serotonin and dopamine a) Amitryptiline b) Selegiline (AIIMS 01)
d) Norepinephrine and GABA c) Haloperidol+CPZ d)ECT
380. A patient comes in stupor condition patient's parents
give history of being continually sad and suicidal
attempts, not eating and sleeping most of the time.
The treatmentis- (AIIMS 2000)
a) Serotonin b)Dopamine a) ECT b) Antidepressant
c) Epinephrine d) Histamine c) Antipsychotic d) Sedatives

355) b 356)d 357)b 358)b 359)a,c 360)b,d 361)b,c 362)b 363)a,c 364)c 365)b 366)a 367)a 368)c
369)a,c 370)a,b,c 37l)c 372)d 373)a 374)a 375)b 376)b 377)c 378)a 379)d 380)a
PSYCHIATRY [ 662]

381. Treatment of choice for endogenous depression with 393. A clinical sign of mania is- (MH-11)
suicidal tendency- (MAHE 01) a) Neologism b) Perseveration
a) Lithium b) Chlorpromazine c) Echolalia d) Flights ofideas
c) ECf d) Psychoanalysis 394. AllarefeaturesofManiaexcept- (Rohtak 98)
382. Treatment of choice in severe depression is- a) Insomnia b) Pressure of speech
a) Fluoxetine b)Alprazolam(MP 04) c) Disorientation d) Granduer delusion
c) ECf d) None 395. All of the following are true about mania except-
383. A 60 year male is brought by his wife. He thinks a) Elevated mood b) Flight of ideas
that he had committed sins all through his life. He c) Delusions of grandeur d) Word salad (J & K OJ)
is very much depressed and bas considered 396. All of the following are feature of mania except-
committing suicide but bas not through bot do go
a) Decreased need for sleep (MP 05)
about it. He had also attached sessions with a
b) Elated mood
spiritual guru. He is not conviced by his wife that he
c) Delusion of grandeur
bas lead a pious life. He does not want to bear
anything on the contrary. How will you treat him- d) Decreased motor activity
a) Antipsychotic+ Antidepressant (All 0) 397. A 20 year old man has presented with increased
b) Antidepressant with cognitive behavioural therapy alcohol consumption and sexual indulgence,
c) Guidance & recounselling with guru + irritability, lack of sleep, and not feeling fatigued
Antidepressant even on prolonged periods of activity. All these
d) Anti depressant alone changes have been present for 3 weeks. The most
384. All of the following agents are antidepressants likely diagnosis is- (AI 03)
excepts- (Jharkhand 03) a) Alcohol dependence
a) Trazodone b)Amitryptiline b) Schizophrenia
c) Fluoxetine d) Pimozide c) Mania
385. Fluoxetineisa- (SGPGI03) d) Impulsive control disorder
a) Noradrenaline uptake inhibitor 398. A 67 year old lady is brought in by her 6 children
b) Serotonin uptake enhancer saying that she has gone senile. Six months after
c) Serotonin uptake inhibitor her husband's death she bas become more religious,
d) MAO spiritual and gives lot of money in donation. She is
386. Drugs used for recurrant depressive episode- occupied in too many activities and sleep less. She
a) Imipramine b) Carbamazepine (PGI June 09) now believes that she has a goal to change the society.
c) Fluoxetine d) Na valproate She does not like being brought to the hospital and
387. Repeated Transcranial Magnetic Stimulation is argumentative on being questioned on her doings.
(rTMS) of brain is used in the treatment of- The diagnosis is- (AllMS May 02)
a) Depression (AI 12)
a) Acute manic excitement b) Delusion
b) Resistant Schizophrenia
c) Schizophrenia d) Depression
c) Obsessive-Compulsive Disorder
399. Lallu a 22 yrs. old male suffers from decreased sleep,
d) Acute Psychosis
388. Stimulation of which ofthe following nerve causes increased sexual activity, excitement and spending
elevationinmood- (AIIMS Nov 09) excessive money for last 8 days. The diagnosis is-
a) Olfactory N b) Optic N a) Confusion b)Mania (AIIMSMayOJ)
c) Trigeminal N d) Vagus N c) Hyperactivity d) Loss of memory
389. The clinical features of mania include- (PGI June 06) 400. Mania is a- (TN 99)
a) Anhedoia b) Elated mood a) Mood disorder b) Psychological disorders
c) Avolition d) Delusion of grandiosity c) Obsessive disorder d) Neurotic disorder
e) Distractibility 401. BPAD includes- (PGI 02)
390. Flight ofidea is seen in- (PGI 03) a) Recurrent depressive episodes
a) Mania b) Schizophrenia b) Recurrent manic episodes
c) Depression d) Delirium c) Depressive episodes and hypomanic episodes
e) Neurosis d) Manic episodes and dysthymia
391. Mania is characterized by- (PGI 99) e) Manic episodes and depressive episodes
a) Paranoid delusion b) Loss of orientation 402. Which of the following is/are included in bipolar
c) High self esteem d) Loss of insight disease- (PGI Nov. 10)
392. Talkativeness and flight ofideas are suggestive of- a) Hypomania b) Cyclothymia
a) Schizophrenia b) Hysteria (KA 96) c) Paranoid disorder d) Hyperthymia
c) Mania d) Tricyclic antidepressants e) Kleptomania

38l)c 382)a 383)a 384)d 385)c 386) a,c 387) a 388)d 389)b,d,e 390)a 39l)a,c,d 392)c 393)d
394)c 395)d 396)d 397)c 398)a 399)b 400)a 401) b,c,e 402) a,b
PSYCHIATRY [ 663]

403. All of the following are inclnded in diagnosis of 414. Lithium is usedinprophylaxisof- (AIIMS 97)
Bipolar disorder except- (AI 07) a) Recurrent phobia
a) Mania alone b) Depression alone b) Alcohol dependence
c) Mania and depression d) Mania and anxiety c) Paranoid schizophrenia
404. Type n bipolar mood disorderinclndes a combination d) Recurrent Manic depressive attacks
ofwhichofthefollowing- (AIIMS Nov 11) 415. Lithium is used in the prophylactic treatment of-
a) Mania and subsyndromal depression a) Schizophrenia (DELHI PG Feb. 09)
b) Hypomania and depression b)MDP
c) Hypomania and subsyndromal depression c) Acute depression
d) Hypomania and dysthymia d) Conversion reaction
405. Bipolar ll disorder includes -(PGI 05, PGI 02,Jipmer ,_. .. , .. c.c.•;"""'L'C'-.c f6i:p,Olitim(lo(l~r~~r- ... ...... .•
a) Cyclothymic disorder 03, AIIMS Nov. 11) ... . ·· .(tf!J!:E'f/DlN'Bfaft.~rn)
b) Dysthymia
c) Single manic episode
d) Major depression & hypomania ...
406. The period ofnormalsyis seen between two psychosis. 417. Best use of lithium is in- (SGPGI 99)
The diagnosis is - (AI 99) a) Treatment of schizophrenia
a) Schizophrenia b) Treatment of recurrent depression
b) Manic depressive psyhosis (MDP) c) Treatment of first depressive episode
c) Alcoholism d) Prophylaxis ofbipolar mood disorder
d) Depression 418. DOCforrapidcyclingMDP- (AI99,AIIMS95)
407. Chromosome associated with bipolar disease- a) li b) Valproate
a) Chromosome 16 (PGI Dec 05)
c) Calcium channel blocker d) Carbamazepine
b) Chromosome 13
419. Drug(s)usedinbipolardisorderis/are- (PGINov
c) Chromosome 18
a) LiC03 b) Na valproate 09)
d) Chromosome 11
c) Phenytoin d) Lamotrigine
e) Chromosome23
408. All drugs are used in Treatment ofMania except- e) Carbamazepine
a) Haloperidol b) Trithex phenidyl (Jipmer 98) 420. DOC for "rapid cyclers" type of bipolar disorder is-
c) Lithium d) Carbamazepine a) Carbamazepine b) Valproate (DPGEE 08)
409. A 42 year old male with a past history of a manic c) Phenytoin d) Lithium
episode presents with an illness of 1 month duration 421. Differential diagnosis of premenstrual tension
characterized by depressed mood, anhedonia and includes all of the following except -(AIIMSNov 02)
profound psychomotor retardation. The most a) Psychiatric depressive disorder
appropriate management strategy is prescribing a b) Panic disorder
combination of- (AIIMS May 04) c) General anxiety disorder
a) Antipsychotics and antidepressants d) Chronic fatigue syndrome
b) Antidepressants and mood stabilizers 422. Cognitive model of depression is given by-
c) Antipsychotics and mood stabilizers a) Ellis (AIIMS May ll, Nov 08)
d) Antidepressants and benzodiazepines b) Beck
410. NotgiveninprophylaxisofBPD- (PGI Nov. 10) c) Godfrey
a) Lithium b)Olanzapine d) Meicheinbanon
c) Valproate d) Lamotrigine 423. A patient presents to the emergency department with
e) Carbamazepine self harm and indicates suicidal intent. Which of
411. Drugs used for prophylaxis in BPD- (PGI 02) the following conditions does not warrant an
a) Cblotpromazine b) Lithium immediate specialist assessment- (AI I 0)
c) Carbamazepine d) Zolpidem
a) Formal thought disorder
e) Sodium valproate
b) Acute alcohol intoxication
412. DrugnotusedinprophylaxisofMDP- (AIIMS 95)
c) Chronic severe physical illness
a) Haloperidol b)Lithium
d) Social isolation
c) Carbamazepine d) Valproate
413. In prophylaxic ofManic depressive psychosis, drug ;p•iim(im':»A:l:J¥'lo$s•is ~r6t 'the·fo1lowing
used is- (AI 96)
, }... ·.'Y~)
,.· · · · · (!.fSl!;!/!)NB Pattern}
a) Lithium carbonate b) Carbamazepine Unipolar depression
c) Valproate d) Haloperidol ,Cd 3:)2~ia

403)b 404)b 405)d 406)b 407)c 408)b 409)b 410)None 411)b,c,e 412)a 413)a,b,c 414)d 415)b
416)b 417)d 418)b 419)All 420)b 421)d 422)b 423)b 424)b
PSYCHIATRY [ 664]

ANXIETY DISORDERS, STRESS DISORDERS 436. A 30 year old lady c/o sudden onset breathlessness,
AND GRIEF anxiety, palpitation & feeling of impending doom.
Physical examination is normal. What is the
425. Which of the following conditions is classified diagnosis- (AIIMS Nov JO)
under anxiety disorders according to DSMITI- a) Panic attack b) Anxiety disorder
a) Obsessive compulsive disorder (Karn. 03) c) Conversion disorder d) Acute psychosis
b) Depression 437. Pappu, a 49-year male complained ofsudden onset
c) Schizophrenia of palpitation and apprehension. He is sweating for
d) Bipolar disorder last 10 minute and fears of impending death.
426. Phobia is- (AJ98, Jharkhand 05, Assam 97) Diagnosis is - (Bihar 04)
a) Psychosis b)Anxiety a) Hysteria b) Cystic fibrosis
c) Fearofanimal d) Neurosis c) Panic attack d) Generalized anxiety disorder
427. Phobiaistypeof- (Assam 95) 438. Differential diagnosis of panic disorder-
a) Psychosis b) Neurosis a) Pheochromocytoma (PGI June 04)
c) Mania d) Depression b) Myocardial infarction
c) Mitral valve prolapse
4~$:~~ 'Alitie~is .: .~·.·.:.::.i;·.·.;s.~,:;'.~··· 'fWJlJ$/i:tfl~:jlJatl~i(if}, d) Depression
a) ~e1Jr<}sis! . . .... ~
_ c)- Pt:tsonal!ty·. g@s()ider}i·}\' e) Carcinoid syndrome
429. All are features of anxiety neurosis except- 439. Male with symptoms mimicking heart disease with
a) Dryness of mouth (Bihar 05) normal ECG & X-ray, the diagnosis is- (AI 08)
b) Sweating a) Angina pectoris b) Panic attack
c) Palpitation c) ANS instability d) Vasovagal attack
d) Abnormal gait 440. Drug of choice for panic disorders is- (Jipmer 95)
430. All are seen in anxiety except- (Kerala 96) a) Nitrazepam b) Imipramine
a) Decreased sweating b) Hyperventilation c) Diazepam d) Clonidine
c) Cold extremities d) Palpitations 441. Drugs used in Generalized anxiety disorder-
e) Pupillary dilatation a) Alprazolam b) Paroxetine
431. Anxiety neurosis is manifested by- (Rajasthan 96) c) Venlafaxine d) Buspirone
a) Difficulty in breathing e) Carbamazepine (PGI Dec 07)
b) Complete consciousness 442. All ofthe following can be considered as treatment
c) Negative thinking for anxiety except- (COMED 09)
d) All a) Buspirone
432. Panic attack is associated with a disturbance in b) Benzodiazepines
all of the following neurotransmitters except- c) Serotonin reuptake inhibitors
a) Serotonine b) Glutamate d) Doparninergic blockers
c) GABA d) Dopamine ~43:··pJt()liti;is:··- ·- edas.--.·..
433. Acute feeling of restlessness and sense of
impending doom is seen in- (Kerala OJ)
:··: :~iP~t . "ir"f ·
a) Acute paranoia b) Schizophrenia . . . >¢) stlilt1WBltiotl';;:,••••·•/•
c) Acute anxiety d) Mania ·. . }'.c(
434. Baby 20 year old female complains of sudden 444. Agoraphobia is- (SGPGI 03)
onset palpitation and apprehension. She is sweating a) Getting caught in places from where escape would
for last 10 minutes and fears ofimpending death. be difficult
Diagnosis is- (AI OJ) b) Fear of heights
a) Hysteria b) Generalized anxiety disorder c) Fearofanimals
c) Cystic fibrosis d) Panic attack d) Fear of closed spaces
435. A patient presented in casualty with a history of 445. Agoraphobiais- (PGJ97)
sudden palpitation, sensation of impending doom a) Fear of open spaces b) Fear of closed speces
and constriction in his chest. This lasted for about c) Fear of heights d) Fear of crowded places
10-15 minutes after which he became all right. 446. WhatisAgoraphobia? (DPG Feb. 09)
The diagnosis is likely to be- (AJIMS OJ) a) Fear of heights b) Fear of closed spaces
a) Phobia c) Fear of open space d) Fear of dogs
b) Personality disorder 447. Agoraphobia is- (Maharashtra 03)
c) Generalized anxiety disorder a) Fear of closed space b) Fear of crowds
d) Panic attack c) Fear of death d) Fear of fire

425)a 426)b,d 427)b 428)a 429)d 430)a 431)d 432)b 433)c 434)d 435)d 436)a 437)c
438)a,b,c,e 439)b 440)b 441)a,b,c,d 442)d 443)b 444)a 445)a,d 446)c 447)b
PSYCHIATRY [ 665]

448. Claustrophobiameans- (Jipmer 95) 458. True about obsession - compulsive disorder are
a) Fearofheights b)Fearoflizards AlE- (MP 01, AI 95)
c) Fear of closed spaces d) Fear of open spaces a) Ego-alien b) Patient tries to resist against
449. Thanatophobiaisfearof- (UP 98) c) Ego syntonic d) Insight is present
a) Closed spaces b) Flights 459. Truestatementsaboutobsession- (PGI 03)
c) High places d) Death a) It is the repetitive thoughts or images
450. A middle aged person reported to Psychiatric OPD b) The patient thought that the images or thoughts
with the complaints of fear of leaving home, fear are imposed by other's
of travelling alone and fear ofbeing in a crowd. He c) Content about sex or God
develops marked anxiety with palpitations and d) The patient gets disturbed when unable to remove
sweating if he is in these situations. He often the ideas or thoughts
avoids public transport to go his place of work. 460. Features of obsessive-compulsive neurosis are-
The most likely diagnosis is- (AJIMS May 06) a) Repetitiveness b) Irresistibility (PGI 02)
a) Generalised anxiety disorder b) Schizophrenia c) Unpleasantness d) Poor personal care
c) Personality disorder d) Agoraphobia
451. Trueaboutsocialphobiais- (UP OJ)
a) Fear of closed spaces
b) Irrational fear of situation True about obsessive compulsive disorder is/are-
c) Irrational fear of activities a) Irresistable desire to do a thing repeatedly
d) Irrational fear of specified objects b) Is a dissociative disorder (PGI OJ)
452. A medical student could not deliver seminar, fearing c) Denial is the defence mechanism against O.C.D.
seniors. He is aware of irrationality of his fear. d) Patient is conscious about the disorder
Likely diagnosis is- (Maharashtra 08, 07) e) Can cause severe distress
a) Agoraphobia b) Social phobia 463. Which of the following statements differentiates the
c) Claustrophobia d) Specific phobia obsessional idea from delusions- (AIIMS Nov 05)
453. A person has poor performance in front of his a) The idea is not a conventional belief
seniors in public place, has tachycardia when he b) The idea is held inspite of contrary evidence
has to deliver a lecture, avoids going to parties. The c) The idea is regarded as senseless by patient
diagnosis is- (AJIMS J999) d) The idea is held on inadequate ground
a) Panic disorder b) Schizophrenia 464. A 35 year old man with an obsessive compulsive
c) Socialphobia d)Avoidantpersonality personality disorder is likely to exhibit all of the
454. Agoraphobia treated with- (PGI Dec 07) following features, except- (AJIMS Nov 04)
a) Systemic desensitization a) Perfectionism interfering with performance
b) Psychodynamic therapy b) Compulsive checking behaviour
c) Exposure therapy c) Preoccupation with rule
d) Relaxation therapy d) Indecisiveness
e) Behaviour therapy 465. Obsessive compulsive neurosis is characterised by
455. A fifty year old male feels uncomfortable in using allexcept- (DNB 95)
lift, being in crowded places and traveling. The a) Elaborate checking b) Repeated hand washing
most appropriate line of treatment is - c) Sexual thoughts d) Excessive slowness
a) Counselling (AJIMS Nov 05) 466. Fear to contamination, counting behaviours, and
b) Relaxation therapy having to check and recheck are features
c) Exposure and response prevention characteristic of- (UPSC-I 09)
d) Covert sensitization a) Panic attacks
456. Treatment of choice for phobic disorder is - b) Agoraphobia
a) Psychotherapy (PGI June 05, Jharkhand 05, c) Obsessive-compulsive disorder
b) Behaviour therapy Assam 99, 97, AI 97) d) Generalized anxiety disorder
c) SSRI 467. A 15 year old boy feels that the dirt has hung onto
d) Benzodiazepam him whenever he passes through the dirty street.
457. Irresistable urge to do a thing repeatedly is seen in- This repetitive thought causes much distress and
a) Obsessive- compulsive disorder (MH 03) anxiety. He knows that there is actually no such
b) Schizophrenia thing after he has cleaned once but he is not
c) Schizoaffective disorder satisfied and is compelled to think so. This has led
d) Depression to social withdrawal. He spends much of his time

448)c 449)d 450)d 45l)b,c 452)b 453)c 454)All 455)c 456)b 457)a 458)c 459) a,d 460) a,b,c
46l)b 462) a,d,e 463) c 464) d 465) None 466) c 467) a
PSYCHIATRY [ 666]

thinking about the dirt and contamination. This 476. The drug of choice for obsessive compulsive
has affected his studies also. The most likely disorder- (PGI Dec 05)
diagnosis is - a) Imipramine b) Fluoxetine
a) Obsessive compulsive disorder (AI 03) c) Chlorpromazine d) Benzodiazepine
b) Conduct disorder 477. Drug of choice for obsessive compulsive neurosis
c) Agoraphobia is- (AI 98, 96, 95, 94, Jharkhand 05, Jipmer 95)
d) Adjustment disorder ~Imipramine b)CPZ
468. A person going to temple gets irresistable sense of c) Carbamazapine d) Clomipramine
idea to abuse God which can't be resisted. Likely 478. Drug of choice in Obsessive compulsive disorder
diagnosis is - (AIIMS 99, UP 98) is- (AIIMSMay 10)
a) Mania a) Imipramine b) Fluoxetine
b) Obscessive-Compulsive disorder c) Alprazolam d) Chlorpromazine
c) Schizophrenia 479. In obsessive compulsive disorder, which is not
d) Delusion given- (Jharkhand 06)
469. A patient with obsessive compulsive neurosis is more a) Clomipramine b) Haloperidol
likely to develop- (MP OJ) c) Sertaline d) Cabamazepine
a) Depression b) Schizophrenia 480. All drugs are used for treatment of OCD except-
c) Delusional disorders d) Conversion symptom a) Carbamazapine (PGI 99, Jipmer 02, MAHE 03)
470. Transmitters mainly involved in OCN is- b) Lithium
a) GABA b)NE (AIJMS 95)
c) Fluoxetine
c) Dopamine d) Serotonin
d) Diazepam
471. InOCDtreatmentisbasedon- (Bihar 05)
481. All are treatment modalities ofOCD except- (AI 96)
a) Serotonergic receptor
b) Dopaminergic receptor a) Exposure & response prevention
c) NA receptor b) Psychoanalytic psychotherapy
d) Glutamate receptor c) Clomipramine
472. TreatmentofchoiceforOCD- (MP 06, AP 99) d) MAO inhibitors
a) Behavior therapy 482. The following modes of therapy may be useful for
b) Drug therapy treatment ofobsessive compulsive disorder EXCEPT-
c) Psychosurgery a) Fluoxetine (UPSC-I 09)
d) Combination of behavior and drug therapy b) Clomipramine
473. Treatment of obscessive-compulsive disorder c) Behaviour therapy
includes- (PGI Dec 08) d) Electroconvulsive therapy
a) Exposure & response prevention 483. Post traumatic stress syndrome is due to-
b) Flooding a) Head injury (Jharkhand 05, AI 98)
c) Psychoanalytic therapy b)CVD
d) Supportive psychotherapy involving family c) Minor stress
members d) Major life threatening events
e) Systemic densensitization 484. Which of the following is NOT a clinical feature of
47 4. A 25 year old female presents with 2 year history of post-traumatic stress disorder (PTSD)?(DPG08,Al 08)
repetitive, irresistible thoughts of contamination a) Flashbacks b) Hyperarousal
with dirt associated with repetitive hand washing. c) Hallucinations d) Emotional numbing
She reports these thoughts to be her own and 485.· Posttraumatic stress-disorder is associated with all
distressing; but is not able to overcome them along
except- (PGI 2000)
with medication. She is most likely to benefit from
a) Flash back
which of the following therapies- (AI 05)
b) Severe traumatic injury
a) Exposure and response prevention
b) Systematic desensitization c) Re-experiencing stressful events
c) Assertiveness training d) Anhedonia
d) Sensate focusing e) It does't develop after 6 months of stress
475. A lady washes her hands at least 40 times per day 486. All are true for PTSD except- (PGI 02)
no apparent reason. Cognitive Behaviour Therapy a) Hippocampus & amygdala
forhertreatmentshouldinclude- (All2) b) Anhedonia
a) Response Prevention b) Exposure c) Depression and guilt
c) Thought stopping d) Desensitization d) Insomnia & poor concentration
e) Anxiolytic is treatment of choice

468)b 469)a 470)d 47l)a 472)a>c,d 473)All 474)a 475)a>b 476) b 477)d 478)b 479)b 480)d
481)None 482)None>d 483)d 484)c 485)e 486)e
PSYCHIATRY [ 667]

487. TrueforPTSDareallexcept- (PGIOJ) 495. A man coming from mountain whose wife died 6
a) Past h/o psychiatric illness months prior says that his wife appeared to him
b) Women are more predisposed and asked him to join her. The diagnosis is -
c) Occur in intellectuals a) Nonnalgrief (AIIMS 2000)
d) Numbness b) Grief psychosis
e) Detachment c) Berevementreaction
488. A lady while driving a car meets with an accident. d) Supernatural phenomenon
She was admitted in an ICU for 6 months. After 496. An elderly house wife lost her husband who died
being discharged she often gets up in night and suddenly of Myocardial infarction couple of years
feels terrified. She is afraid to sit in a car again. ago. They had been staying alone for almost a
The diagnosis is- (AI OJ, 00, AIIMS 99) decade with infrequent visits from her son and
a) Anxiety disorder grandchildren. About a week after the death she
b) Phobia heard his voice clearly talking to her as he would
c) Conversion disorder in a routine manner from the next room. She went
d) Post traumatic stress disorder to check but saw nothing. Subsequently she often
489. Three years back a woman suffered during an heard his voice conversing with her and she would
earthquake and she was successfully saved. After also discuss her daily matters with him. This
recovery she has nightmares about the episode and however, provoked anxiety and sadness of mood when
she also gets up in the night and feels terrified. The she was preoccupied with his thought. She should
most probable diagnosis is- (AIIMS May 02) betreatedwith- (AIIMSMay03)
a) Major depression b) Post-traumatic stress disorder a) Clomipramine b) Alprazolam
c) Mania d) Schizophrenia c) Electroconvulsive therapy d) Haloperidol
490. A victim ofTsunami has difficulty in overcoming the 497. An elderly house wife lost her husband who died
experience. She still recollects the happening in suddenly of Myocardial infarction couple of years
ago. They had been staying alone for almost a decade
dreams and thoughts. The most likely diagnosis is-
with infrequent visits from her son and
a) Post traumatic stress disorder (Maharashtra 07)
grandchildren. About a week after the death she
b) Conversion disorder
heard his voice clearly talking to her as he would in
c) Panic disorder .
routine manner from the next room. She went to
d) Phobia
check but saw nothing. Subsequently she often heard
491. Flashback is characteristic of- (TN 02)
his voice conversing with her and she would also
a) Hypomania
discuss her daily matters with him This however,
b) Post traumatic stess disorder
provoked anxiety and sadness of mood when she was
c) Postnatal depression
preoccupied with his thought. She should be treated
d) Griefreaction with- (AIIMS May 2003)
492. The diagnostic feature that differentiates PTSD a) Clomipramine b) Alprazolam
from other disorders that occur following a stressful c) Electroconvulsive therapy d) Haloperidol
incident is- (AIIMS May 12) 498. MS. B, a 27 year old nurse had extracurricular
a) Episodic occurrence of symptoms interest in trekking and painting. She broke up
b) Severe anxiety and autonomic arousal relationship with her boy friend. Two months later
c) Re-experiencing and avoidance of trauma she lost interest in her hobbies and was convinced
d) Nightmares about the event that she would not be able to work again. She
493. True about posttraumatic stress disorder- (PGI 2000) thought life was not worth living and has consumed
a) Recall of traumatic events 60 tablets of phenobarbitone to end her life. She is
b) Associated with major trauma like pelvic fracture most likely suffering from-
c) Treatment is ECT a) Adjustment disorder (AI 04)
d) Disturbed sleep b) Acute stress disorder
494. Which of the following is NOT TRUE about post c) Depressive disorder
traumatic stress disorder? (Karn 11) d) Post-traumatic stress disorder (PTSD)
a) The onset of illness may occur many months after 499. Two months after knowing that his son was suffering
the traumatic event from leukemia, a 45 year old father presents with
b) Individual debriefing and early counselling are sleep deprivation, lethargy, headache and low mood.
most effective method of treatment He interacts reasonably well with others, but he
c) Eye movement desensitization and reprocessing absented himself from work. The most probable
is effective diagnosis is - (AI 08)
d) There is high risk of alcohol abuse in patients a) Depression b) Psychogenic headache
suffering from this disorder c) Adjustment disorder d) Somatization disorder

487) c 488) d 489) b 490) a 491) b 492) c 493) a,d 494) b 495) a,c 496) a 497) d 498) a 499) c
PSYCHIATRY [ 668]

500. A 25 year old lady presented with sadness, background of normal inves-tigation. The most
palpitation, loss of appetite and insomnia. There is probable diagnosis is- (AI 02)
no complaint of hopelessness, suicidal thoughts and a) Hypochondriasis
there is no past history of any precipitating event. b) Somatization disorder
She is remarkably well in other areas of life. She is c) Somatoform pain disorder
doing her office job normally and her social life is d) Conversion disorder
also normal. What is the probable diagnosis in this 507. A 35 years old male, with pre-morbid anxious traits
case- (AI 10) & heavy smoker, believes that be bas been suffering
a) GAD b) Mixed anxiety depression from 'lung carcinoma' for a year, No significant
c) Adjustment disorder d) Mild depressive episode clinical finding is detected on examination and
relevant investigation. He continues to stick to his
SOMATOFORM DISORDERS beHef despite evidence to the contrary. In the process,
he has spent a huge amount of money, time and
501. Which one of the following is not a specific energy in getting himself unduly investigated. He is
somatoform disorder- (AIIMSNov 11) most likely suffering from- (AIIMS Nov 04)
a) Somatization disorder a) Carcinoma lung
b) Chronic fatigue syndrome b) Hypochondriacal disorder
c) Irritable bowel syndrone c) Delusional disorder
d) Fibromyalgia d) Malingering
502. All are true regarding somatization disorder except- 508. A 41 year old married female presented with
a) Maintain sick role (PGI 00, Jipmer 04, TN 00) headache for the last 6 months. She bad several
b) 4-Pain symptoms consultations. All her investigations were found to
c) 1-Sexual symptom be within normal limits. She still insists that there
d) 1-Pseudo neurological symptom is something wrong in her bead and seeks another
consultation. The most likely diagnosis is -(AI 03)
a) Phobia b) Psychogenic headache
c) Hypochondriasis d) Depression
509. A 40 year old male is admitted with complaints of
abdominal pain and headache. General physical
examination revealed six scars on the abdomen from
previous surgeries. He seems to maintain a sick
A 45 year old ......1"' ....._,.,nt•
role and seeks attention from the nurses. He demands
and vague body pains, off & on diarrhoea & multiple diagnostic tests including a liver biopsy.
constipation, impotence and tingling and parastbesia The treating team failed to diagnose any major
is glove stocking pattern. The probable diagnosis physical illness in the patient. His mental status
is- (AI 00, Jipmer 02) examination did not reveal any major psychopathology.
a) Hypochondriasis b) Somatization disorder One of the treating staff recognized him to have
c) Conversion disorder d) Factitious disorder appeared in several other hospital with abdominal
505. A 36 year old woman presents with headache, pain and some other vague complaints. He is most
backache, epigastric fullness, and decreased sexual likely suffering from- (AIIMS Nov 03)
desire. She bas lost -2.5 kg over the past 3 years, a) Schizophrenia b) Malingering
and her appetite is otherwise normal. Her husband c) Somatisation disorder d) Factitious disorder
bad a major vehicular accident 8 years back. Which 510. A 28 year old male admitted in the hospital with a
of the following diagnoses best describe the mental history of vague pains. His examination revealed
status of this patient? (Maharashtra 06) many scars of previous surgeries. He was very
a) Depersonalization disorder curious about knowing his diagnosis and was
b) Adjustment disorder and depression persistently asking for various diagnostic
c) Somatizationdisorder procedures and biopsy. He gave the past history of
d) Posttraumatic stress disorder (PTSD) gall stone & appendicitis pain diagnosed by previous
506. Lain prasad 45 yrs old male presents to OPD with doctors, but the history regarding this was
complaining the continuous non-progresive inappropriate and seemed to be manipulative. There
headache from last 7 days. He believes that be bas were no previous records suggesting this diagnosis.
brain tumor. for that there is be consulting many Tbeprobablediagnosisis-(JipmerOO,J&K02,MH03)
neurologists in past even all of investigation have a) Hypochondriasis
been with in normal limits. Patient requested for b) Somatization disorder
other investigation to reveal that be bas brain tumor. c) Conversion disorder
Psychiatric evaluation shows the disease on the d) Factitious disorder

500)a 501)b,c,d 502)a 503)a 504)b 505)c 506)a 507)b 508)c 509)d 510)d
PSYCHIATRY [ 669]

511. A patient has been seeking repeated admissions in 519. By the term 'hypochondriasis' is meant-
various hospitals with a variety ofsymptoms. He had a) Pathologic pre-occupation with the normality of
undergone appendicectomy, cholecystectomy, and the physical state & perception of normal bodily
other exploratory laparotomies on previous phenomena as disease (CUPGEE 96)
admissions. The most likely diagnosis is - b) A complete disregard for physical ill being
a) Conversion reaction (UPSC 95) c) Complete disregard for the feelings of other people
b) Hypochondriasis d) None
520. Maintaining sick role by any means is a characteristic
c) Munchausen syndrome
feature of- (Jipmer 02, Mahe 04, Rohtak 02)
d) Ganser's syndrome
a) Hypochondriasis
512. A 30 years old lady presents to physician with b) Somatization disorder
complaints of hematuria. On evaluation RBCs were c) Conversion disorder
found in the urine but no cause is found on further d) Factitious disorder
enquiry it is found that she bas gone to many doctors 521. Munchausen's syndrome by proxy involves-(MH-11)
with the same complaint and demanding in patient a) Drug abuse
care. She pricks her finger and mixes blood in the b) Toxin-mediated neuropsychiatric
urine sample. Her diagnosis is - (Karn 11) c) Illness caused by care giver
a) Malingering b) Factitious illness d) All of the above
c) Dissociative disorder d) Hypochondriasis 522. Dissociative conversion disorder was previously
513. All are features of somatization disorder except- described as- (TN 02)
a) Frequently changing pain sites a) Hysteria b) Dementia precox
b) Sexual symptoms (MH 05, Jipmer 03) c) Melancholia d) Hypochondriasis
c) Paresthesia 523. In conversion disorder all are found except-
d) Professional patients a) Jealousy b) Paralysis (Bihar 05)
c) Anaesthesia d) Abnormal gait
514. 20 years old female with complaints of nausea,
524. All are true for conversion reaction except-
vomiting and pain in the legs. Her physical
a) Secondary gain (AIIMS 98)
examination and lab investigations are normal. What b) Onset in late age
would be the most probable diagnosis? c) Patient does not consciously produce symptoms
a) Generalized anxiety disorder (AIIMS Nov 09) d) Relation with stress
b) Conversion disorder 525. Differentiation of hysterical fits from epileptic fit-
c) Somato form pain disorder a) Occurs in sleep (DPG 09, Calcutta 02)
d) Somatisation disorder b) Injuries to person
515. 45 years old Chandu is always preoccupied with c) Incontinence
feeling of illness. The diagnosis is - (AI 00) d) Occurs when people are watching
a) Somatoform disorder 526. Epileptic fits are distinguished from hysterical fits by-
b) Hypochondriasis a) Amnesia b) Sleep convulsion (UP 96)
c) Munchausen syndrome c) Incontinence d) Injury
d) Dissociative disorder 52 7. A female presented with history ofepisodes of altered
516. Person preoccupied by worries about ill health is- sensorium, involuntary movements, memory deficits
a) Hypochondriac (Punjab-ll, PGI 98) and headache since the last 6 months. 4 episodes
occurred during tbe day and 2 during sleep. Sbe has
b) Maniac
been married since 1 year. Most likely diagnosis is-
c) Depressed
a) Epilepsy (AIIMSMay08)
d) Delirium
b) Somatization disorder
517. Persistent preoccupation with serious illness and c) Somatoform disorder
normal body function is called- (UP 08) d) Hypochondriasis
a) Obsession b) Somatization 528. In Dissociative disorder all are seen except- (AI 96)
c) Hypochondriasis d) Conversion disorder a) Fugue b) Amnesia
518. One of the following usually differentiates hysterical c) Multiple personality d) Hearing loss
symptoms from hypochondriacal symptoms- (AI 05) 529. Following are included in dissociative disorder-
a) Symptoms do not normally reflect understandable a) Multiple personality disorder
physiological or pathological mechanism b) Fugue (PGI 03, June 07, Dec 07)
b) Physical symptoms are prominent which are not c) Hypochondriasis
explained by organic factors d) Somatization disorder
c) Personality traits are significant e) Obsession
d) Symptoms run a chronic course f) Borderline personality

51l)c 512)b 513)d 514)Noneord 515)b>a 516) a 517)c 518)a 519)a 520)d 521)c 522)a 523)a
524)b 525)d 526) All 527) a 528) d 529) a,b
PSYCHIATRY [ 670)

530. Which of the following is dissociative phenomenon- 541. A person has different multiple personality is
a) Fugue b) Amnesia (Nimhans OJ) suffering from- (PGI 97)
c) Deafness d) Lack of insight a) Mania b) Personality disorder
531. Commonestsymptom of hysteria is- (Kerala 96) c) Paranoid schizophrenia d) Dissociative disorder
a) Motor or sensory phenomena 542. All are seen in hysterical somatisation except -
b) Hypochondriasis a) Positive physical signs (Kerala 96)
c) Irrelevant talk b) Hemiplegia
c) Blindness
d) Fugue
d) Fugue
e) Insomina . e) Normal intelligence
532. The most common form of dissociation hysteria is- 543. All are true about Ganser's syndrome except-
a) Fugue b) Amnesia (MH 2K) a) Approximate answer (PGI 99, Jipmer 00, UP 00)
c) Multiple perosonality d) Somnambulism b) Apparent clouding of consciousness
533. Psychogenic amnesia is characterized by- c) Only found in prisoners
a) Antegrade amnesia (AIIMS 97) d) Hallucinations
b) Retrograde amnesia 544. Regarding Ganser's syndrome true is-
c) Both with confabulation a) Repeated lying (TN 95, AIIMS 98)
d) Patchy impairment of personal memories b) Approximate answers
534. A yonng female presented with halo's, abdominal c) Unconscious episodes
pain and amnesia she is likely to be suffering from- d) Malingering
a) Conversion disorder (AI 97) 545. La belle indifference is seen in - (AIIMS 98)
b) Dissociative disorder a) Conversion reaction b) Schizophrenia
c) Depresonalization disorder c) Mania d) Depression
d) Mania 546. La bella indifference is seen in- (Kerala 98)
535. Dissociative fugue occurs in- (Jipmer 03) a) Mania b) Depression
a) Schizophrenia b) Mania c) Delirium d) Dissociative disorders
c) Hysteria d) Neurosis 54 7. La belle indifference, which is true- (CUPGEE 96)
536. Fugue state may be seen in- (SGPGI 99) a) Excessive occupation with minor ailments
a) Schizophrenia b) Hysteria b) Characterize by coprolalia
c) Epilepsy d) All of the above c) Disregard for symptoms despite apparent severity
537. A person is missing away from home and wandering d) None
with loss of recent memory with depersonalization.
The diagnosis is- (Maharashtra 08) PERSONALITY DISORDERS
a) Dissociative fugue
b) Dissociative amnesia 548. Which is not included in personality trait?
a) Sensation seeking (AIIMS Nov 09)
c) Dissociative personality
b) Problem solving
d) Any of the above
c) Openness to experience
538. A person missing from home is found wandering
d) Neuroticism
purposefully. He is well groomed and denies of having
549. All are cluster B disorders of personality except-
any amnesia. Most likely diagnosis is- (AI 01)
a) Antisocial b) Narcissistic (PGI 99)
a) Dementia b) Dissociative amnesia
c) Avoidant d) Borderline
c) Dissociative fugue d) Schizophrenia ·
5)9.. difl'~~a,fXon &deperS:olbillsatr6 .. m 5$}).: 'JYP~~e~r•op;ality fJisfi~tJer"'YiJ~B[l~Nf1P:(l1Je&z)
~).A:v~i~i.per~~wllitY ~is(.}rd~r~ ... :)•;,
1
...... ·.b).... ,·
· ofdi~order-. ..... ·.··· . ·• . < <. ··> < } . . T ··d' •. d' ...... , ..... ·'' ·····
a)Dis~ociative· disor~~J;h).~.ef8qn~li~~d~Qrd~rs·
~)~.Ma¢~ " , ~~~-:4li-N~~~-'':(_;~,~J,~ '--~~;~-ii~;_:; ):~ ,L\f";~~N~t~i§~i~FL a r , tsor er 1:. ;:c·r·~c~i'
540. An 18 year old boy came to the Psychiatry OPD with 551. Which personality disorder of DSM-IV is not
a complaint of feeling changed from inside. He classified as PD & is placed with schizophrenia in
described himself as feeling strange as if he is ICD10- (WB 98)
different from his normal self. He was very tense a) Schizoid b) Paranoid
and anxious yet could not point out the precise c) Narcissistic d) Schizotypal
change in him. This phenomena is best called as- 552. True about personality disorder- (PGI June 07)
a) Delusional mood (AI 05) a) Onset in early childhood & adolescent
b) Depersonalization b) Matures around adulthood
c) Autochthonous delusion c) Not associated to social norms
d) Over valued idea d) Direct result of disease or damage

530)a,b 53l)a 532)b 533) b,d 534) b 535) b 536)d 537)a 538)c 539)a 540)b 54l)d 542)a 543)c
544)b 545)a 546)d 547)c 548)b 549)c 550)a 551)d 552)a,b,c
PSYCHIATRY [ 671 ]

553. Trueaboutpersonalitydisorder(PD)- (PGI 03) 562. A 16 year old girl was brought to the psychiatric
a) Typically onset of early childhood & adolescent emergency after she slashed her wrist in an attempt
b) Matured around age 30-40 yrs to commit suicide. On enquiry her father revealed
c) Egodystonic that she had made several such attempts of wrist
d) Dramatic, emotional and erratic behaviour in slashing in the past, mostly in response to trivial
paranoidPD fights in her house. Further she had marked
e) Pervasive and maladaptive behaviour fluctuations in her mood with a pervasive pattern of
unable interpersonal relationship. The most
554. Trueaboutpersonalitydisorder- (PGI June 08)
probable diagnosis is- (AIIMS Nov 02)
a) It arises during childhood
a) Borderline personality disorder
b) It matures during adulthood b) Major depression
c) Suspiciousness is seen in paranoid personality c) Histrionic personality disorder
disorder d) Adjustment disorder
d) Excessive preoccupation with fantasy seen in 563. Pervasive pattern of instability of interpersonal
schizoid personality disorder relationship, selfimage, and affects, with marked
555. Markedly inappropriate sensitivity, self importance impulsing that begins at early adulthood and present
and suspiciousness are clinical features of- in varities of context is characteristic of-
a) Antisocial (AMU 02, TN 99) a) Bipolar disorder (Maharashtra I 0, 07)
b) Histrionic b) Schizoaffective disorder
c) Schizoid c) Borderline personality disorder
d) Paranoid d) Schizotypal personality
556. Characteristic feature of schizoid personality is- 564. A lady has very unstable relation with her husband,
a) Conversion reaction (AIIMS 99) always threatening to suicide infact she one time
b) Not concerned with disease eat some poison, then diagnosis is- (Bihar 06)
a) Borderline personality disorder
c) Checks details of all things
b) Depression
d) Emotional coldness
c) Avoident personality disorder
d) None
565. A lady has changed multiple boy friend in last 6
months and she breaks the relationship, she
attempted suicide many time. Most likely diagnosis
.}~) .tros}ieE is? (MP 06)
558. Antisocial peJrso!nality a) Borderline personality
a) Drug abuse b) Post traumatic stress
c) OCN d) None c) Acute depression
559. Oddities of speech, mannerism and clothing with d) Acute panic attack
magical thinking is seen in which type of personality 566. A person having the habit of repeated self inflicted
disorder- (Jipmer 2k) injuries, what is the type of personality-
a) Schizoid b) Paranoid a) Borderline personality disorder (PGI June 04)
c) Schizotype d) Borderline b) Schizoid personality disorder
560. A young lady was admitted after taking overdose of c) Histrionic personality disorder
d) Narcissistic personality disorder
diazepam after broken affair. She has history of
e) Depressive disorder
slitting her wrist previously. Most likely diagnosis
5.6.7. N1~stelfectjte.1freatm.
-, , ~ e.n.f.b.t,bor. ""'' , rfrt_ep*~c.n.
, -, Arderlin. alii)'·····.··-
is- (AIIMS 00) ~:}::,; -~ -, "., / / ""
'·" •r> '''''''':~;'::<:''f' t~tf{J.,'f!il!l/ltvNl/PtitternJ
a) Narcisstic personality disorder :c:C;,. ·. • . / '· ·~ra.pr·"·· ">·:;•!:,::; ,,~, ) ~:.
b) Dependent PD
· 'b)l3~11avrow~t)l~apy ·• .·. . /<· < .·. . ·. >;.. ".'' ·
c) Borderline PD . ·. c) Gl}m\jiilati&n:;· rit . ··both . phartn:licQI:lte'r~py and
d) Histrionic PD ·,;; ::.·~"~~'· • ~lttlierap~·~[l, ' ;) ; " · · · · <::: '''·
561. A female presents with the history of slashed wrists : ;:\\ \" ";-, ·g_)-_: ~t~:~$-. :::;~~_f< t'~~- i~,))_f~1i ~>- ' I~-~ ·F-' ,--- f ">i~i ., -; :·-~\~::,;:' ~.;.: :'

and attempted suicide, now presents with similar 568. False regardiJJg tY}:te A penollality -(AiiMS Nov 07)
history. The diagnosis is- (AIIMS OJ) a) Hostile b) Time pressure
a) Borderline personality c) Competitiveness d) Mood fluctuations
b)OCD 569. TypeD personality has been recently found to be a
c) Conversion reaction risk factor for- (A/IMS Nov 11)
d) Histrionic personality a) Coronary artery disease b) Depression
c) Personality disorder d) Schizophrenia

553) a,b,e 554) a,b,c,d 555)d 556)d 557)c 558)a 559)c 560)c 56l)a 562)a 563)c 564)a 565)a
566) a 567) b 568) d 569)a
PSYCHIATRY [ 672]

570. True about treatment of personality disorder- 582. Not a feature of paradoxical sleep is- (PGI 99)
a) Antipsychotic drugs are used (PGI May 10) a) Decreased muscle tone
b) SSRI used in treatment b) Rapid eye movements
c) Behaviour therapy c) Brain shows increased metabolism
d) No need of treatment d) EEG shows decreased activity
571. All of the following are impulse control disorders 583. Which one of the following phenomenon is closely
except- (AI 03) associated with slow wave sleep- (AJIMS Nov 04)
a) Pyromania b) Trichotillomania a) Dreaming b) Sleep walking
c) Kleptomania d) Capgras syndrome c) Atonia d) Irregular heart rate
572. One of the following is not a compulsive and habit
formingdisorder- (KA 95)
a) Kleptomania b) Pyromania
c) Nymphomania d) Pathological gambling
573. Kleptomaniais- (Kerala 98)
a) Delusion b) Obsession
c) Impulse d) Compulsion The Non-REM (NREM) sleep is commonly ass.[)Ciatted
e) Hallucination with- (AI 04)
574. Kleptomaniameans- (KA 98, AI 99) a) Frequent dreaming
a) Irresistable desire to set fire b) Frequent penile erections
b) Irresistable desire to steal things c) Increased blood pressure
c) Compulsive hair pulling d) Night terrors
d) Pathological gambling 587. Sleepwalking occur in which stage of sleep -
575. Oniomania is a disorder of compulsive- a) REM (PGI 97)
a) Buying (MARE 03, Jipmer 00) b) NREM stage
b) Cellular phone use c) NREM stage 3 or 4
c) Internet use d) Can occur in any stage ofNREM sleep
d) Selfmultilation 588. Pover nocturnus is- (APPG 97)
a) Sleep terror b) Sleep apnea
SLEEP DISORDERS AND EATING c) Sleep bruxism d) Somnambulism
DISORDERS 589. Not true about nocturnal penile tumerscence is-
a) Totals about I 00 min I night (AIIMS 95)
576. Max duration of time spent is in NREM stage- b) Normal phenomenon
a) I b) II(NEETIDNB Pattern) c) Occurs in NREM sleep
c)ill d)IV d) Can be used to distinguish between psychological
577. a-rhythm is seen in- (PGI 97) or organic impotence
a) Sleep with eyes closed with mind wandering 590. True about treatment of nocturnal enuresis-
b) Mental activity a) Imipramine (DPG 09, PGI June 05, AIIMS 96)
c) Awake with eyes open b)CPZ
d) REM sleep c) Alprazolam
578. Sleep waves in hippo-campal area is- (MP 00) d) Haloperidol
a) Delta b) Theta 591. DOC of night terrors- (PGI 98)
c) Beta d)Alpha a) Meprobamate b) Tricyclic antidepressant
579. What are the EEG waves recored forparietooccipital c) Clonazepam d) Diazepam
region with subject awake & eye closed-(Kerala 97) 592. Not true about narcolepsy- (PGI Dec 06)
a) Alpha waves b) Beta waves a) Sleep sudden b)Longduration(>3hrs)ofsleep
c) Delta waves d) Theta waves c) Cataplexy d) Presents in lind decade
580. The EEG recorded shown below is normally 593. Narcolepsy is due to abnormality in- (NEET/DNB
recordable during which stage of sleep ? (AI 03) a) Hypothalamus b) Neocortex Pattern)
a) Stage I b) Stage II c) Cerebellum d) Medulla oblongata
c) Stage III d) Stage IV 594. Hallucinations which arises when a patient is falling
581. False regarding REM sleep is- (UP 99) asleep- (Jipmer 02)
a) Dreaming sleep a) Hypnagogic hallucination
b) Slow eye movements present b) Hypnopompic hallucination
c) Paradoxical sleep c) Non specific hallucinations
d) Alpha waves are seen d) Jactatio nocturna capitis

570)a,b,c 57l)d 572)c 573)c 574)b 575)a 576)b 577) b,d 578) b 579)a 580)b 58l)b 582)d 583)b
584) a 585) a 586) d 587) c 588)a 589)c 590)a 591)d 592)b 593)a 594)a
PSYCHIATRY [ 673]

595. Cataplexy is seen in- (Jipmer 02) 606. One ofthe following conditions the only means of
a) Catatonic schizophrenia sexual excitement are in inanimate objects-
b) Narcolepsy a) Exhibitionism b) Fetishism (Kerala 2 K)
c) Chronic depression c) Sadism d) Masochism
d) Neuro -malignant syndrome 607. Exhibitionism is- (CUPGEE 02)
a) Shameless exposure of genitals
b) Exhibiting one's possessions
c) A type of mania
d) Not a sexual persuasion
608. Which of the following is not a common feature of
Anorexia nervosa? (AI 06)
a) Binge eating b) Amenorrhoea
c) Self perception of being fat d) Under weight
609. FalseregardingAnorexianervosa- (Kerala 97)
a) Psychiatric symptom b)Exercise
598. A homosexual person feels that he is imposed by a c) Weight loss d) Decreased appetite
female body & persistent discomfort with his sex, 610. Anorexia nervosa can be differentiated from bulimia
diagnosis is- (PG1 03) by- (AI1MS Nov 08)
a) Gender identity disorder b) Transvestism a) Intense fear ofwt gain
c) Voyeurism d) Paraphillias b) Disturbance ofbody image
599. 20 years old girl Nelu enjoys wearing male clothes. c) Adolescent age
Wearing male clothes gives her feeling of more d) Peculiar patterns of food handling
confidence and after these episodes she is an 611. With regard to anorexia nervosa the following is
absolutely normal girl. The likely diagnosis is-
true except- (Kerala 2K)
a) Trans sexualism (AI1MS 97)
a) Phobic avoidance of normal weight
b) Fetishism
b) Overperception ofbody image
c) Dual role transvestism
d) Fetishistic transvestism c) Self induced vomiting
600. Excessive sexual desire in males is known as- d) Menorrhagia
a) Nymphomania b) Satyriasis (A11MS May 08) e) Excessive exercise
c) Triabadism d) Sadism 612. A young lady is present with HJO repeated episode of
601. Squeeze technique is used for- (AI 00) over eating followed by purging using laxatives, she
a) Impotence b) Premature ejaculation probably suffering from- (AI1MS May 10, May 09,
c) Infertility d) Priaprism a) Bulimianervosa 07, AI 02)
~~~~x~ b) Schizophrenia UP 04, J & K 01)
c) Anorexia nervosa
d) Benign eating disorders
613. Which of the following is not true about bulimia
nervosa? (UPSC-1 08)
a) Invariable weight loss with endocrine disorders
603. A 30 yr old male presents to the OPD with Erectile b) Occurrence ofboth binge eating and inappropriate
dysfunction. Basic screening evaluation is compensatory behaviour at least twice weekly, on
unremarkable. The next step in the evaluation I an average, for 3 months
management should be? (AI 08) c) Recurrent episodes of eating binges
a) Oral sildenafil citrate trial b) Cavemosometry d) Recurrent self- induced vomiting
c) Doppler study d)Neurological testing 614. WhichoneofthefollowingconditionsisNOTcorrect
604. Most accurate treatment of erectile dysfunction -
in case of bulimia nervosa? (UPSC-1 09)
a) Sildenafil (PG1 02)
a) Recurrent bouts of binge eating
b) Master and Johnson technique
b) Lack of self-control over eating during binges
c) Beta blockers
c) Self-induced vomiting or dieting after binge
d) Papaverine
605. The erectile disorder in males is more specifically d) Weight gain
treated with which of the following agent? 615. InBulimiaallareseenexcept- (Jipmer 98)
a) Sildenafil b) Diazepam (DPGMar. 09) a) Parotitis b) Menorrhagia
c) Fluxetine d) Zolpidem c) Binge eating d) Carries teeth

595)b 596)b 597)c 598)a 599)c 600)a 60l)b 602)a 603)a 604)a 605)a 606)b 607)a 608)a
609)d 610)d 61l)d 612)a 613)a 614)d 615)b
PSYCHIATRY [ 674]

616. Which ofthefollowing conditions characterised by 624. A 10 year old child is always restless, inattentive to
episodes of excessive and uncontrolled eating- study and always wants to play outsite. Parents are
a) Anorexia nervosa (UP 06, Jharkhand 07, TN 05) extremely distressed. What would you advise-
b) Bulimia nervosa a) It is a normal behaviour (AIIMS Nov 08)
c) Body dysmorphobia b) Behaviour therapy
d) All of the above c) It is a serious illness requires medical treatment
d) Needs change in environment
None of the above
625. Treatment of hyperkinetic syndrome include-
a) Imipramine (PGI June 06, PGI OJ)
b) Methylphenidate
c) Haloperidol
d) Clozapine
CHILD PSYCHIATRY e) Amphetamine
626. The treatment of choice in Attention deficit
618. Hyperkinetic disorder is characterised by all, except- hyperactivity disorder is- (NEETIDNB Pattern,
a) Impulsivity b)Distractibility (FN02) a) Haloperidol b) ImipramineAJ/MSMay 03)
c) Hyperactivity d) Hallucinations c) Methylphenidate d)Alprazolam
619. An eleven month old boy is all the time so restless 627. Stimulant drug is given to child for -
that the rest ofthe class is unable to concentrate. He a) Conduct disorder (NEETIDNB Pattern)
is hardly ever in his seat but roams around the hall. b) Speech developmental disorder
He has difficulty playing quietly. The most likely c) Pervasive disorder
d) ADHD
diagnosis is- (AI 02)
628. Drug(s) used in treatment of attention-deficit
a) Attention-deficit hyperactivity disorder hyperactivitydisorder- (PG/ Dec 08)
b) Conduct disorder a) Atomoxetine b) Methylphenidate
c) Depressive disorder c) Dexmethylphenidate d) Quetiapine
d) Schizophrenia e) Dextroamphetamine
620. A 9 yr old child is restless. He is hyperactive and his 629. Which ofthe following is not true aboutthe autistic
teacher complains that he does not listen to the specificdisorder- (AIJO)
teachings. Disturbs other students, he also shows a) Impaired communication
less interest in playing. The likely diagnosis is - b) Impaired imagination
a) Cerebral palsy (AIIMS May 02) c) Language developmental delay
b) Attention deficit hyperkinetic child d) Vision problems
c) Delirium 630. Autism is- (PG/ OJ)
d) Mania a) Biological causation
621. A 9 year old child disturbs other people, is destructive, b) Pervasive social and language communication problem
interferes when two people are ta1king, does not follow c) Metabolic disease
instructions and cannot wait for his turn while d) Mainly due to hypothalamus damage
playing a game. He is likely to be suffering from- e) Onset after 23 yrs. usually
a) Emotionaldisorders (AIIMSNov05) 631. Infantile autism is characterized by- (PGI Dec 04)
b) Behavioural problems a) Impaired vision
c) No disorder b) Impaired Neurobehavioural development
d) Attention deficit hyperactivity disorder c) Impaired folate level
622. A 7 years old child has conservative behavior, not d) A socio-economic hazard
playing quietly with peers, and difficulty in e) Parenting
organization. He also interrupts other in school and 632. Autism is- (PGI 2000)
earily gets distracted. Which of the following is likely a) Neurodevelopmental disorder
diagnosis- (MH-11) b) Social and language communication problem
a) Learning disorder c) Metabolic disease
b) Autistic disorder d) Mainly due to hypothalamus damage
c) Attention deficit hyperactive disorder 633. A 3 year old child developmental milestones normal
d) Conduct disorder with delayed speech, and difficulty in communication,
623. ADHD in childhood can lead to what in future- concentration, not making friends- (AIIMS Nov 06)
a) Schizophrenia b) Alcoholism (PGI 00) a) Autism b)ADHD
c) Intellectual changes d) Antisocial behaviour c) Specific learning disability d) Mental retardation

616)b 617)d 618)d 619)a 620)b 621)d 622)c 623) b,c,d 624) c 625) b,e 626) c 627) d 628) a,b,c,e
629)d 630)a,b 631)b,d 632)a,b 633)a
PSYCHIATRY [ 675]

634. A 3 year old girl has normal developmental milestones 642. IQ is- (DPG 09, Kerala 95)
except delay in speech. She has difficnly in a) Mental age I chronological age x 100
concentration communication and relating to others b) Chronological age I mental age x l 00
and does not make friends but plays with herself. c) Mental age+ chronological age x I 00
Likely diagnosis is- (AIIMS May 10, AI-J2) d) Mental age- chronological age x 100
a) ADHD 643. A 16 year old male is found to have a mental age of9
b) Autism years on I.Q. testing. He has- (AIIMS May 05)
c) Specific learning disorder a) Mild mental retardation
d) Mental retardation b) Moderate mental retardation
635. A 6 year old child has history ofbirth asphyxia does c) Severe mental retardation
not communicate well has slow mental and physical d) Profound mental retardation
growth, does not mix with people, bas limited interests 644. Normal IQ is- (PG/02)
gets wildly agitated if distrubed. Diagnosis is - a) 45 b)85
a) Hyperkineticchild (AIIMS OJ) c) 65 d) 100
b) Autistic disorder e) 110
c) Attention defecit disorder 645. Which of the following I.Q. scores is indicative of a
d) Mixed receptive - Expressive language disorder 'normal intelligence? (DPG Mar. 09)
636. A girl with normal milestones spend her time seeing a) 85 b)65
her own band, do not interact with others, what is the c) 45 d)25
diagnosis? (Aiims May 08) 646. IQ of70 -75 denotes- (Kerala 03)
a) ADHD b) Autism a) Borderline MR b) Moderate MR
c) Asperger's syndrome d) Rett's disorder c) SevereMR . . . . ..... · · .·.·. d) Nonnalintelligence
637. All are features of autistic disorder except- (MP 06) 64'i.f~~e~'hfen'tijf~f¥tc.bl~onis .... ; 7(NEErr!DN8'f:>titternJ
a) Mental retardation
Absent social smile
b)
::>., ~ . ~·"······-·······J~~,J~Si//,.~,~;~~·i~m!r2i>1fYf·L/~
648. Patientwitb1Qof60bas-
0.:~ · •·?:~.;
(AIIMS 95)
....

c) Onset before 2Y:z years


a) Mildmentalretardation b)ModerateM .. R
d) Third person hallucination
c) Severe M.R. d) Profound M.R.
638. A two years old girl child is brought to tbe out patient
649. In a child ofiQ 50, which of the following is true-
witb features ofband wringing stereotype movements,
a) Can look after himself independently (PGI OJ)
impaired language and communication development,
b) Can study upto 8th standard
breath holding spells, poor social skills and
c) Can follow simple verbal commands
deceleration of bead growth after six months of age.
d) Can handle money
The most likely diagnosis is- (AIIMS Nov 03)
e) Recognize family members
a) Asperger syndrome b) Rett's syndrome
650. All of the following statements about 'Imbecile' are
c) Fragile X syndrome d) Cottard syndrome true- (AI 11)
639. Trne about Autism- (PGI May J 0) a) IQis50-60
a) Persistent delusion b) Intellectual capacity equivalent to a child of 3 -7
b) Persistent hallucination years of age
c) Incoordinate social interaction c) Impaired self care
d) Defective reciprocal interaction d) Condition usually congenital or acquired at an early
e) Onset after 5 yr age
640. A 14 year old boy bas difficulty in expressing himself 651. Second most common cause of chromosomal
in writing and makes frequent spelling mistakes, abnormality causing mental retardation- (CIP OJ)
does not follow instruction and cannot wait for his a) Down's syndrome b) Fragile X syndrome
turn while playing a game. He is likely to be sufering c) Edward syndrome d) Patau syndrome
from- (AJIMSNov05) 652. X-Iillked disease leading to mental retardation is-
a) Mental retardation a) Myotonic dystrophy (PGI 2000)
b) Lack of interest in studies b) Fragile-X syndrome
c) Specific learning disability c) Tuberous sclerosis
d)Examinationanxiety d) Phenylketonuria
641. Conduct disorder in a child manifests with- 653. X-Iillked recessive disorder with mental retardation
a) Disregard for right of others (PGI 0 J) and selfmultilation is seen in- (Maharashtra 08)
b) Doesn't care for authority a) Lesch- Nyhan syndrome
c) Backward in studies b) Fragile X- syndrome
d) Decreased head circumference c) Down syndrome
e) Steals things d) All of the above

634)b 635)b 636)b 637)d 638)b 639)c,d 640)c 64l)a,b,c,e 642)a 643)a 644)d 645)None>a 646)a
647)c 648)a 649) a,c,d,e 650) a 651) b 652) b 653) a
PSYCHIATRY [ 676]

654. An 8 year old student complains oflack of interest in 663. ECTisindicatedin- (AIIMS 98)
studies for last 6 months. He has frequent quarrels a) Neurotic depression b) Auditory hallucination
with his parents and has frequent headaches. The c) Schizophrenia d) Delusional depression
most appropriate clinical approach would be -(AI 05) 664. ECT is most useful in- (PGI97, AIIMS 97, UP 08)
a) Leave his as normal adolescent problem a) Mania b) Depression
b) Rule out depression c) OCD d) Schizophrenia
c) Rule out migraine 665. The indication for ECT is- (DPG 08)
d) Rule out an oppositional defiant disorder
a) Delirium tremens b) Alcohol withdrawal
655. Best therapy suited to teach daily life skill to a
c) Severe depression d) Thyrotoxic crisis
mentally challenged child- (AIMS May OJ, AIIMS
a) CBT(CognitiveBehaviortherapy) May 11, 09) 666. Electro convulsive therapy is currently advocated
b) Contingency as a line of treatment in the following conditions
c) Cognitive reconstruction except- (UPSC-I 08)
d) Selfinstruction a) Catatonic schizophrenia
656. Lack of development of speech in a child can be b) Severe depression with psychosis
due to- (PGI OJ) c) Manic-depressive disorder
a) Tongue tie b) Deafness d) Obsessive-compulsive disorder
c) Mental subnormality d) Laryngomalacia 667. Electro convulsive therapy is indicated in - (MP 03)
e) Psychosocial a) Psychotic symptoms
657. Patients suffering from which of the following disease b) Neurotic symptoms
as per ICD/DSM criteria are eligible for disability c) Dissociative symptoms
benefit as per National Trust Act? (AI 09) d) Affective symptoms
a) Schizophrenia b) Bipolar disorder 668. ECT is usefulin- (AIIMS 97)
c) Dementia d) Mental retardation a) Mania (Acute) b) Chronic schizophrenia
658. According to disabilities ACT 1995. Seventh
c) Acute depression d) Panic disorder
disability is usually referred to as ? (AIIMS Nov 08)
669. ECT is advocated in following except-
a) Neurological abnormality
b) Mental illness a) Chronic schizophrenia (Maharashtra 06)
c) Substance abuse b) Catatonic schizophrenia
d) Diability due to road traffic accident c) Severe depression
659. Acholastic performance is impaired in all of the d) Severe psychoses
following, Except- (AI 12) 670. The absolute contraindication for Electro-Convulsive
a) Attention Defecit Hyperactivity Disorder (ADHD) Therapy(EC1)is- (DPG Mar. 09)
b) Specific Learning Disability (SLD) a) Raised Intra-Cranial Tension
c) Anxiety b) Vascular Dementia
d) PICA c) Diabetic Retinopathy
d) Peripheral Neuropathy
TREATMENT IN PSYCHIATRY 671. Absolute contraindication to ECT is-
a) Glaucoma (DPG 09, AIIMS 95)
660. lndicationsforECTare- (PGI 2000) b) Brain tumor
a) Paranoid schizophrenia c) Aortic aneurysm
b) Depression with suicidal tendency
d)Ml
c) Catatonic schizophrenia
672. Contraindication ofECTis- (Maharashtra 03)
d) Neurotic depression
e) Depression in involutional stage a) Pregnancy b) Catatonic schizophrenia
661. Indications ofECT is/are- (PGI May I 0) c) Depression d) Acute schizophrenia
a) Psychotic depression 673. ECT causes- (AIIMS 98)
b) Catatonic schizophrenia a) Antegrade amnesia b) Retrograde amnesia
c) Cyclothymia c) Both d) None
d) Dysthymia 674. MostcommoncomplicationofECTis- (AIIMS 96)
e) Post traumatic stress disorder a) Ante grade amnesia b) Retrograde amnesia
c) Psychosis d) Depression
675. Memory disturbance ofECT recovers in -(AIIMS 96)
a) Few days to few weeks
b) Few weeks to few months
c) Few months to few year
d) Permanent

654)b 655)b 656) b,c 657) d 658) b 659) d 660) b,c 661) a,b 662) b 663)d 664)b 665)c 666)d 667)a
668)c 669)a 670)a 67l)b 672)None 673)c 674)b 675)b
PSYCHIATRY [ 677]

676. Which of the following increases on 688. Operant condition where paradigm pain stimulus are
Electroconvulsive Therapy? (AIIMS Nov 08) given to a child for decreasing a certain undesired
a) 5-hydroxy-indole-acetic acid behaviour can be classified as- (AI 97)
b) Dopamine a) Positive reinforcement
c) Serotonin b) Negativereinforcement
d) Brain derived neurotrophic factor c) Punishment
677. ECT in depressive phase ofMDP is useful because d) Negotiation
it- (PGI 99) 689. Many of our bad habits of day to day life can be
a) Produces recurrence removed by- (AIIMS Nov 04)
b) Reduces recurrence a) Positive conditioning
b) Negative conditioning
c) Shortens duration
c) Biofeedback
d) Increases drug effects
d) Generalization
678. Behaviourtherapyisusefulin- (PGI June 08)
690. Reciprocal inhibition is done by- (SGPGI 2000)
a) Psychosis b)OCD a) Systemic desensitization
c) Personality disorder d) Panic attack b) Behaviour therapy
e) Anxiety c) Flooding
679. In which behaviourtherapyishelpful- (AIIMS 95) 691. Along a pleasant stimulus, noxious stimuli are given
a) Schizophrenia b) Agoraphobia in treatment of alcohol dependece or sexual disorder
c) Personality disorder d) Neurotic depression is which of the following type of behaviour therapy-
680. Systemic de-sensitization therapy is used for- a) Negative reinforcement (Maharashtra 08)
a) Phobia (AIIMS 97, SGPGI 03, b) Aversive therapy
b) Depression Bihar 04,AI 2000) c) Punishment
c) Schizophrenia d) Flooding
d) Organic brain syndrome 692. Cognitive behaviour deals with- (PGI Nov 09)
a) Unconscious motive
b) Maladaptive assumptions
c) Pseudodynamic conflict
Therapeutic exposure is a form - (MH-11) d) Emotional consciousness of automatic thoughts
a) Behavior therapy b) Psychoanalysis 693. A fundamental technique used in psychoanalysis is-
c) Cognitive therapy d) Supportive therapy a) Concentration (Kafn 96)
683. Methods oflearning in psychiatry are all except- b) Hypnosis
a) Modelling (AIIMS Nov 07) c) Empathy
b) Catharsis d) Free association
694. Wrong statement about psychoanalysis is-
c) Exposure
a) Parapraxes has meaning (J & K 08, TN 06)
d) Response prevention
b) Transference is patient's feeling for therapist
684. 'Reinforcement' is used in- (AIIMS 94, 99)
c) Counter transference is clinican's feeling for patient
a) Psychoanalysis b) Hypnoanalysic d) Unguided communication has no meaning
c) Abreaction d) Conditioned learning 695. Counter transference is- (AIIMS Nov 11)
685. Behaviour therapy to change maladaptive behaviour a) Patient's feeling towards therapist
using response as reinforcer uses the principles of- b) Doctor's feeling towards the patient
a) Classical conditioning (AI 03) c) Psychic connection between patient and disease
b) Modeling T e of defence mechanism
c) Sociallearing
d) Operant conditioning
686. All of the following are done in behaviour therapy to
increase a behaviour except- (AI I 0)
a) Punishment
b) Operant conditioning A patient can be taught to con~;tr;~ollf"l.'i; i~;~.;i:;~;t~~~
c) Negative reinforcement physiological responses by which of the following
d) Reward therapies- (Maharashtra 09)
687. Pavlov's experiment is an example of- (AI 06) a) Breathing therapy
a) Operant conditioning b) Classical conditioning b) Stress modification
c) Learned helplessness d) Modeling c) Biofeedback
d) Rational emotive therapy

676)d 677)c 678)b,c,d,e 679)b>c 680)a 68l)c 682)a 683)b 684)d 685)d 686)a 687)b 688)c 689)b
690)a 69l)b 692) b,d 693) d 694) d 695) b 696) a 697) c
PSYCHIATRY [ 678]

698. Bright light treatment has been found to be most 708. TrueregardingmindtheoryisA/E- (PGI Feb 08)
effective in treatment of- (AI 06) a) Id is reservoir of instincts & drives
a) Anorexia nervosa b) Ego is excutive organ of id
b) Seasonal affective disorder c) Superego predominantly consciously offers
c) Schizophrenia approval & disapproval
d) Obsessive compulsive disorder d) Conscious is common component of id, ego &
699. A chronic smoker taking 20 cigaretters per day super ego
developed chronic cough. His family suggested e) ld works on pleasure principle
quitting cigarettes. He is ready to qit but thinks that 709. According to sigmund Freud, primary process
quitting will make him irritable. Which of the thinking is- (Jipmer 11)
following option best describes the satge ofbehavour a) Illogical & Bizarre
change- (AI 11, AIIMSMay 09) b) Rational
a) Precontemplation and preparation c) Absent during sleep
b) Contemplation and Cost analysis d) Logical & unconscious
c) Persuasion 710. Allaremechanismofdefenceexcept- (AIIMS 98)
d) Belief a) Repression b) Transference
700. All of the following are parts of cognitive behaviour c) Projection d) Anticipation
changetechniqueexcept- (Al10) 711. Which of the following is a mature defence
a) Pre-contemplatation b) Consolidation mechanism- (UP 07)
c) Action d) Contemplation a) Projection b) Reaction formation
701. EEG is used in the diagnosis of all except- c) Anticipation d) Denial
a) Intermittent expulsive disorder (Jipmer-11) 712. One of the important defence mechanism is-
b) Panic disorder a) Alienation (Karn OJ)
c) Anxiety disorder b) Confabulation
d) Bipolar disorder c) Repression
d) Suppression
COGNITIVE DEVELOPMENT AND DEFENSE 713. A reluctant child forced to bring suger from a shop
MECHANISM spills half ofit on the way. This is an example of-
a) Hysteria (Jipmer 97, Delhi 02, Raj 04)
702. Which of the following is a stage of intuitive thought b) Passive aggression
appearanceinjean-Piagetscheme- (PGI 99) c) Disobedience
a) Sensorimotor b) Concrete d) Active aggression
c) Preoperational stage d) Formal operations stage 714. A chronic alcoholic blames the family environment
703. Ability to form a concept & generalize is known as- as a cause ofhis alcoholism. This is phenomenon of-
a) Concrete thinking (Jipmer 11) a) Projection b) Denial (AIIMS 2000)
b) Abstract thinking c) Rationalization d) Sublimation
c) Intellectual thinking 715. Which ofthe following is the psychological defense
d) Delusional thinking mechanism by the virtue of which an individual
704. Psychodynamic theory of mental illness is based on- blames others for his mistake? (Maharashtra
a) Unconscious internal conflict (AIIMS Nov 07) 03)
b) Maladjusted reinforcement a) Rationalization b) Compensation
c) Organic neurological problem c) Projection d) Regression
d) Focus on teaching patient to restrain absurd 716. Which ofthe following excludes painful stimuli from
thoughts awareness- (AIIMS 98)
705. Interpretation of dreams by Freud- (UP 01, KA 02) a) Repression b) Reaction formation
a) 1990 b)1900 c) Projection d) Rationalization
1956 d)1919 717. Avoiding awareness of pain of reality by negative
sensory data is in which of the following defence
mechanism'- (MH-11)
a) Distortion b) Denial
707. 'That part of mind which is working on reality c) Displacement d) Dissociation
principle is- (Karn. 01) 718. Postponing paying attention of conscious impulse or
a) ID b)Ego conflict is mature defence mechanism known as-
c) Super ego d) Ego-ideal a) Sublimation b) Suppression
c) Humor d) Anticipation

698)b 699)c 700)b 701)a 702)c 703)b 704)a 705)b 706) a 707)b 708) c,d 709) a 710)b 7ll)c
712)c 713)b 714)c 715)c 716)a 717)b 718)b
PSYCHIATRY [ 679]

719. Most important cause of neurotic reaction is the 733. Lifespanofsigmundfreudwas- (Jipmer 2 K)
excessive use of- (PGI 98, Nihmans OJ, MH 04) a) 1856-39 and he passed most ofhis life in Austria
a) Projection b) Regression b) 1859-36 and he passed most of his life in Austria
c) Suppression d) Sublimation c) 1859-36 and he passed most ofhis life in Germany
720. Displacement reaction is characteristically seen in- d) 1859-36 and he passed most of his life in France
a) Mania b)Phobia (MP98) 734. Therterm 'free association' is coined by- (Jipmer OJ)
c) Conversion disorder d) Depression a) Freud b)Adler
721. Ego's defence mechanism "Undoing" is typically c) Erikson d) Juna
seen in- (PGJ OJ, AJIMS 93, 95) 735. Who introduced cocaine in psychiatry- (Kerala 98)
a) Depression a) Freud b) Jung
b) Schizophrenia c) Milar d) Stanley
c) Obsessive compulsive neurosis 736. Sigmund Freud is associated with which of the following?
d) Hysteria a) Group Psychotherapy (DPG Mar. 09)
722. Defence mechanism in OCD- (PGI Dec 07) b) Psychoanalysis
a) Repression b) Undoing c) Sociogram
c) Rationalization d) Sublimate d) Cognitive Behavior Therapy
e) Displacement 73 7. Who is the founder of school of psychoanalysis and
723. Defence mechanism in OCD- (Maharashtra J0, 07) psychotherapy is after the name of -(Maharashtra 02)
a) Progression b) Regression a) Bleuler b) Freud
c) Magical thinking d) Reaction formation c) Kraeplin d) Melanie klien
724. Mature defence mechanism is seen in- 738. Moral treatment of mentally ill patient was first
a) Altruism (Jipmer 95, WB 05, Assam 06) stressed by- (AJIMS 95, CMC 98, KA OJ, TN 04)
b) Repression a) Pinel b) Morel
c) Regression c) Kraeplin d) Sigmund freud
d) None 739. Which of the following scientist propagated
725. Which of the following is a mature defence therapeutic community concept:-(Karn 2K, Bihar 03)
a) JB Watson b)Maxwelljones
mechanism- (Maharashtra 09)
c) Freud d)Adler
a) Rationalization b) Humor
740. Ross classified five stages of- (MP 06)
c) Denial d) Displacement
a) Schizophrenia b) Delusion
726. Which is not an ego defence mechanism- (PGI 99)
c) Death d) None
a) Rationalization b) Repression
741. The eight stage classification of human life is proposed
c) Identification d) Obsession by- (Burdwan 2K, WB 04, UP 05)
727. Not a defence mechanism- (PGI 98)
a) Sigmund freud b) Pavel
a) Derailment b) Represession c) Strauss d)Erikson
c) Distortion d) Undoing 742. Termcatatoniawasusedby- (CUPGME 03)
728. Oedepius complex bas been described by- (UP 06, a) Karlkahlbaum b)Adolfmeyer
a) Plato b) Socartes Bihar 05, AJ98) c) Leo kanner d) Karen homey
c) Freud d) Huxley 743. The term "Ambivalence" was coined by- (PGI 99,
729. Oedipus complex (given by sigmund freud) is seen a) Hippocrates b) Eugen bleuler AIIMS 95)
in- (PGJ 98, Delhi 98, Assam 04) c) Kraeplin d) Sigmund freud
a) Boys of 1 - 3 years of age 744. Following name is associated with psychodynamic
b) Girls of 1 - 3 years of age theory- (PGI Nov 09)
c) Boys of3- 5 years of age a) CarlJung b) Sigmund Freud
d) Girls of3- 5 years of age c) Emilkraplein d) Engen bleuler
730. In psychoanalytic terms, obsessive compulsive e) Kurl schneider's
disorder is fixed at- (Delhi 98, TN 02, Mah 03) 745. General Adaptation Syndrome (GAS) is seen in-
a) Oedipal stage b) Genital stage a) Panic Attacks b) Depression (AI J2)
c) Oral stage d) Anal stage c) Anxiety d) Stressful Situations
731. Fixation of Hysteria is- (WB 02, J & K 04, PGI 05)
a) Genital b)Anal PHARMACOTHERAPY IN PSYCHIATRY
c) Oral d) Phallic
732. Psychoanalysis wasfounded by which of the following 746. Features of serotonin syndrome associated with SSRI
scientist- (Karnataka 2000) &MAOisare- (PGJ May 10)
a) Eysenck b) Freud a) Tremors b) Agitation
c) Jung d)Adler c) Cardiovascular collapse d) Hypothermia
e) Suicidal tendency

719)b 720)b 72l)c 722) b,e 723) d 724)a 725)b 726) c,d 727) a 728)c 729)c 730)d 73l)d 732)b
733)a 734)a 735)a 736)b 737)b 738)a 739)b 740)c 74l)d 742)a 743)b 744)b 745) d 746) a,b,c
PSYCHIATRY [ 680]

747. The foUowing is not an atypical antipsychotic- the following is the most likely cause for her
a) Thioridazine b) Clozapine (A1lMS Nov 06) symptoms- (AIIMS 2K)
c) Olanzapine d) Risperidone a) Acute drug dystonia b) Conversion reaction
748. Atypical (Second generation or newer) antipsychotics c) Acute psychosis d) Cerebrovascular accident
are- (PGI May 10) 760. A patient with acute psychosis, who is on haloperidol
a) Aripiprazole b) Risperidone 20mg/day for last 2 days, has an episode
characterized by tongue protrusion, oculogyric
c) Pimozide d) Penfluridol
crisis, stiffness and abnormal posture of limbs and
e) Olanzapine
trunk without loss of consciousness for last 20
749 Antipsychotic drug with prolonged action- (Al93)
minutes before presenting to casualty. This improved
a) Trifluperazine b) Thioridazine within a few minutes after administration of
c) Penfluridol d) Fluphenazine diphenhydramine HCl. The most likely diagnosis
750. Depotpreparationareavailablefor- (PGI Nov. 10) is- (AIIMS May 06, AI-11)
a) Haloperidol b) Risperidone a) Acute dystonia
c) Olanzapine d)Imipramine b) Akathisia
e) Fluphenazine c) Tardive dyskinesia
751. AU the foUowing statement about clozapine are true d) Neuroleptic malignant syndrome
except- (Al97) 761. Irresistable urge to move about with inner
a) It is used in schizophrenia restlessness is called- (AI 99)
b) May precipitate seizure a) Akinesia b) Hyperkinesia
c) May cause agranulocytosis c) Dyskinesia d) Akathisia
d) Extrapyramidal side effects are seen 762. A 30 year old manic patient was prescribed
752. Adverseeffectofclozapine- (PG198) haloperidol one week back. For the last two days he
a) Hypertension has become restless and kept pacing in the room for
a day. On examination he was found to have tremors
b) Sialorrhea
of hand. He is most likely suffering from- (AJIMS
c) Extrapyramidal S/E
a) Anhedonia b) Dystonia Nov 03)
d) Neuroleptic malignant syndrome c) Restless leg syndrome d)Akathisia
753. Extrapyramidal side effects of antipsychotics- 763. A 19 year old boy suffering from chronic
a) Dystonia (PGI June 02) schizophrenia is put on haloperidol in the dose of20
b) Akathisia c) Seizures mg/day.A week after the initiation of medication the
d) Diarrhea e) Parkinsonism patient shows restlessness, fidgety, irritability and
754. Least extrapyrimidal 8/E are seen in- (AIIMS 98) cannot sit still at one place. The most appropriate
a) Haloperidol b) Thioridazine treatment strategy is- (AJIMS May 04)
c) Clozapine d)CPZ a) Increase in the dose of haloperidol
755. Antipsychotic drug with least extrapyramidal effects b) Addition of anticholinergic drug
is- (PG/02) c) Addtion ofbeta blocker
a) Clozapine b) Risperidone d) Adding another antipsychotic drug
c) Thioridazine d) Chlorpromazine 764. A 25 year old man with a psychotic illness, was
e) Haloperidol treated with haloperidol30mg/day. On the third day
756. Tardive dyskinesia is produced by foUowing except- he developed pacing, and inability to sit still at one
place. The medication likely to be helpful is-
a) Fluphenazine b) Haloperidol (PGI 98)
a) Phenytoin (AIIMS Nov 02)
c) Chlorpromazine d) Clozapine
b) Propranolol
757. Tardive dyskinesia is least common with- (MH 08) c) Methylphenidate
a) Flupenthiol b) Penfluridol d) Trihexyphenedyl
c) Olanzapine d) None 765. Akathesia is treated by aU except- (DPG 05)
758. Best drug therapy for paranoid schizophrenia in a a) Trihexyphenidyl b) Diazepam
thin built21-years old young male would be-(MH 06) c) Haloperidol d) Promethazine
a) Chlorpromazine b) Risperidone 766. An elderly woman suffering from schizophrenia is
c) Olanzapne d) Quetiapne on antipsychotic medication. She developed
759. Vasanthi 45 years, was brought to casualty with purposeless involuntary facial and limb movements,
abnormal movements which included persistent constant chewing and puffing of cheeks. Which of
deviation of neck to right side one day before she the foUowing drugs is least likely to be involved in
was prescribed Haloperidol 5 mgs three times this side effect- (AIIMS Nov 03)
daily from the psychiatry OPD. She also had an a) Haloperidol b) Clozapine
altercation with her husband recently. Which of c) Fluphenazine d) Loxapine

747)a 748)a,b,e 749)d 750)All 751)d 752)b 753)a,b,e 754)c 755)a 756)d 757)c 758)c 759)a 760)a
76l)d 762) d 763) c 764) b 765) c 766) b
PSYCHIATRY[681]

767. A patient of schizophrenia treated for 5 years 774. Sedation as an adverse effect is MOST with which of
developed perioral movements. Likely diagnosis is - the following atypical antipsychotics ? (Karn 11)
a) Tardive dyskinesia (AI 97) a) Risperidone b) Olanzapine
b) Muscular dystonia c) Quetiapine d)Aripiprazole
c) Akathasia 775. Antipsychotic drug with maximum hypotensive
d) Malignant neurolept syndrome effect- (Jipmer-11)
768. A 30 year old man who was recently started on a) Fluphenazine b) Trifluperazine
haloperidol 30mg/day developed hyperpyrexia, c) Thioridazine d) Haloperidol
muscle rigidity, akinesia, mutism, sweating, 776. AllareAntidepressantsexcept- (AI 97)
tachycardia and increased blood pressure. The a) Trazodone b)Amitryptyline
investigations showed increased WBC count, c) Fluoxetine d)Pimozide
increased creatinine phosphokinase. There is no 777. Mode of action offluoxetine- (AI 99, AIIMS 97, 96)
history of any other drug intake or any signs of a) GABA agonist b) GABA antagonist
infection. The most likely diagnosis - c) Inhibituptakeof5-HT d) lncreaseuptakeof5-HT
a) Drug overdose (AIIMS May 06) 778. Tianeptineactsby- (AIIMS 98)
b) Neuroleptic malignant syndrome a) MAO inhibitor
c) Drug induce Parkinsonism
b) Serotonin uptake inhibitor
d) Tardive dyskinesia
c) Serotonin uptake enhancer
769. A31 year old male, with mood disorder, on 30 mg of
d) 5-HT agonist
haloperidol and 100 mg oflithium, is brought to the
779. Tricyclic antidepressants have all of the following
hospital emergency room with history of acute onset
offever, excessive sweating, confusion, rigidity of actions except- (PGI 97)
limbs and decreased communication for a day. a) Anticholinergic action
Examination reveals tachycardia and labile blood b) MAO inhibition
pressure and investigations reveal increased CPK c) Block 5-HT orNE reuptake
enzyme levels and leucocytosis. He is likely to have d) Causes sedation
developed- (AIIMS May 04) 780. A patient was treated with amitriptyline for
a) Lithium toxicity depressi9n, developed urinary retention, constipation
b) Tardive dyskinesia & blurring of vision. Most likely cause is-
c) Neuroleptic malignant syndrome a) Symptoms of depression persisting (AIIMS 00)
d) Hypertensive encephalopathy b) Anticholinergic side effects
770. A patient on antipsychoticfor past 4 weeks is brought c) Depression medicamentosa
to the emergency with complaint of acute onset of d) Any of above
fever, excessive sweating, confusion, rigidity of 781. A 60 year old male comes to casuality with acute
limbs and decreased communication. Examination retention of urine since 12 hours. On examination
reveals temperature of 104•F; P/R -120/min, BP- there was distended bladder. ffis son gives a history
150/100 with disorientation. What is the most oftaking some drug by the patient. Since 2 days as
probable diagnosis- (PGI Dec 05) be is suffering from depression. The most likely
a) Lithium toxicity b) Aggrevation of psychosis drug is- (AIIMS OJ)
c) Dystonia d)Neuroleptmalignantsyndrome a) CPZ b)Amitriptyline
e) Akathesia c) Haloperidol d) Pimozide
771. True about neurolept malignant syndrome- 782. A patient on t/t for psychiatric disorder takes
a) General antipsychotics are more associated overdose of a drug, develops bradycardia,
b) Symptoms include rigidity, fever and palpitation hypotension, decreased sweating and salivation. The
c) Dantrolene improves symptoms (PGI June 08) likely drug is- (AIIMS 99)
d) Bromocriptine can be used a) Amitryptilline b)Lithium
772. Malignant neurolept hyperthermia (NMS) seen in c) Selegiline d)Amphetamine
all except- (AIIMS Nov 08) 783. Tricyclic anti depressant are contraindicated in-
a) Haloperidol b) Metoclopramide a) Glucoma b) Brain tumor (AI 91)
c) Domperidone d)Amantadine c) Bronchial asthma d) Hypertension
773. A patient was on treatment with trifluperazine for 784. SSRI is first line treatment for- (PGI May 1OJ
some time. He presents with complaint of a) OCD b) Panic disorder
hyperthermia, lethargy and sweating. Needed c) Social phobia d) Post traumatic stress disorder
investigations are- (AIIMS 2000)
e) Adjustment disorder
a) CT Scan brain & hemogram
785. Side effect ofSSRI- (PGI Dec 07)
b) Hemogram, Electrolyte level and creatinine
a) Nausea b) Diarrhea
c) ECG, Chest X-ray and hemogram
c) Weight gain d) Rash
d) Hemogram, CPK and renal function test
e) Sexual dysfunction

767)a 768)b 769)c 770)d 771)a,b,c,d 772)d 773)d 774)c 775)c 776)d 777)c 778)c 779)b 780)b
781)b 782)a 783)a 784) a,b,c,d 785) All
PSYCHIATRY [ 682]

786. The common side effect with tluoxetine therapy is- 802. A patient is brought to tbe casualty in tbe state of
a) Seizure b)Anxiety (AJ06) altered sensorium. He was on lithium treatment for
c) Hypotension d) Loose stools affective fits. On examination he bas tremors,
787. Side effects of tluoxetine are AlE - (AI 94) increased DTR's and incontinence of urine. He has
a) Weight gain b) Sweating · also undergone an episode of severe gastroenteritis
c) Urinary retention d) Diarrhoea 2 days ago. The serum lithium was found to be 1.95
788. · Non sedating antidepressant is- (AIIMS 93) meq/Lit. The probable cause for bis present state is-
a) Fluoxetine b) Mianserine a) Lithium toxicity b) Dehydration (AIIMS 01)
c) Amoxapine d) Imipramine c) Manic episode d) Depressive stupor
789. The newest antidepressantis- (AIIMS 93)
803. Congenital Anomaly produced by lithium therapy
a) Buspirone b) Fluoxetine
is- (AI 93)
c) Penfluridol d) Clozapine
a) Limb shortening b) Anencephaly
790. Following drugs have abuse liability except- (AI 99)
a) Buprenorphine b)Alprazolam c) Heart block d) Renal agenesis
c) Fluoxetine d) Dextropropoxyphene 804. Lithium is not used in treatment of which of tbe
791. A patient on antidepressant therapy developed following? (AI 09)
sudden hypertension on consuming cheese. The a) Major depression b) Vascular headache
antidepressant is possibly- (PGI 99) c) Neutropen:ia d) Generalizedanxietydisorder
a) Amitriptyline b) Tranylcypramine 805. Select the true statements- (PGI 02)
c) Fluoxetine d) Sertraline a) Imipramine is used in treatment of endogenous
792. Antidepressant drug used in nocturnal eneuresis depression
is- (AI 11) b) Fluoxetine causes weight gain
a) Imipramine b) Fluoxetine c) Thioridazine cause less anticholinergic effects
c) Trazdone d) Sertaline d) BZD's have same abuse potential as barbiturates
793. Which drug acts through alpha subunit ofGABA- e) Codeine is an effective antitussive
a) Benzodiazepine b) Barbiturate 806. Not an indication for use of anticonvulsants- (PGI
c) Haloperidol d) TCA (AIIMS Nov 08) a) Chronic pain b) Bipolar disorder Nov. 10)
794. All are an:x:iolytic except- (AIIMS 93) c) Anxiety disorder d) Migraine prophylaxis
a) Buspirone b) Fluoxetine e) Diabetic neuropathy
c) Diazepam d) Nitrazepam 807. Carbamazepine may be used in all except- (AI 97)
795. pblockers are indicated in- (AIIMS 95) a) Man:ia b)Alcohol withdrawal
a) Phobia b) Schizophrenia c) Schizophrenia d) Trigeminal neuralgia
c) An::dety d) Mania
808. As a side effect the metabolic syndrome is most
796. Liis best used in- (AIIMS 92)
commonly associated with which of tbe following
a) MDP- Bipolar b) MDP- Un:ipolar
group of medications? (DPG Mar. 09)
c) MDP- rapid cycles d) Depression
797. Lithium is treatment of choice for- (AIIMS 91) a) Anti-an:xiety drugs b) Anti-depressant drugs
a) Unipolar MDP prophylaxis c) Anti-psychotic drugs d) Anti-cholinergic drugs
b) Bipolar MDP prophylaxis 809. Which of the following anti-hypertensives was once
c) Schizophrenia used as an antipsychotic? (DPG Mar. 09)
d) Acute man:ia a) Atenolol b) Propranolol
798. Prophylactic maintenance serum level of lithium is- c) Clon:idine d) Reserpine
a) 0.2-0.8meq!L b)0.7 -1.2meq/L (AI94) 810. Which ofthefollowing drugs is used to bring back a
c) 1.2-2.0meq!L d)2.0-2.5meq!L large number of memories for events just before a
799. Lithium levels~ normal therapeutic -(PGIDec 06) traumaticevent(e.g.beadinjury)- (DPG Feb. 09)
a) 0.5to0.7mEq/lit b)0.7to 1.1 mEq/lit a) Ketamine b) Thiopentone
c) 0.1 to 0.3 mEqllit d) 1.5 to 2mEq/lit c) Piracetam d) Vitamin B 1
e) 2.5 to 3 mEq/lit 811. Modafinil is approved by FDA for treatment of all
800. Prophylactic blood level ofLi is- (PGI May 10) except- (AI 09)
a) 0.3mEq!L b)0.6mEq/L a) Obstructive sleep apnea syndrome (OSAS)
c) 0.9mEq/L d) 1.2mEq!L b) Shift work syndrome (SWS)
e) 1.5mEq!L c) Narcolepsy
801. True about lithium treatment in mania- d) Lethargy in depression
a) Commonest neurological side effect is tremor 812. Which of the following is not an opioid
b) Toxic level is <1.5 mg/dl serum level (PGI Nov 09) peptide- (AIIMS May 05)
c) Amiloride is DOC for Li induced diabetes insipidus a) Beta-Endorphin b) Epinepbrin
d) Lithium is 90% protein bound
c) Leu-enkephalins d) Met-enkephalins
e) Tremor is treated with propranolol
***
786) d 787) c 788) a 789) b 790) c 791) b 792) a 793) a 794) None 795) a,c > b,d 796) a 797) b,d 798) a 799) b
800)b 80l)a,c,e 802)a 803)c 804)d 805)a,b,e 806)None 807)None>c 808)c 809)d 810)b>a 811)d 812)b

You might also like